Você está na página 1de 570

SUMÁRIO

MATEMÁTICA
FUNDAMENTOS ALGÉBRICOS 5
RAZÃO E PROPORÇÃO 13
LÓGICA 25
CONJUNTOS 33
TEORIA DOS NÚMEROS 43
FUNÇÃO - CONCEITOS 51
TIPOS DE FUNÇÃO 61
FUNÇÃO COMPOSTA E INVERSA 69
FUNÇÃO AFIM 79
FUNÇÃO QUADRÁTICA 87
FUNÇÃO MODULAR 99
FUNÇÃO EXPONENCIAL 109
FUNÇÃO LOGARÍTMICA 121
RESOLUÇÃO DE INEQUAÇÕES 133
ANÁLISE COMBINATÓRIA: ELEMENTOS DISTINTOS 143
ANÁLISE COMBINATÓRIA: ELEMENTOS COM REPETIÇÃO 149
BINÔMIO DE NEWTON E POLINÔMIO DE LEIBNIZ 155
PROBABILIDADE 161
ESTATÍSTICA 169
NÚMEROS COMPLEXOS 181
COMPLEXOS NA FORMA TRIGONOMÉTRICA 187
POLINÔMIOS: CONCEITOS E ALGORITMO DA DIVISÃO 193
POLINÔMIOS: TEOREMAS E DIPOSITIVOS LIGADOS A DIVISÃO 199
EQUAÇÕES POLINOMIAIS 205
PROGRESSÕES 211
MATRIZES 225
DETERMINANTES 235
SISTEMA LINEAR 243
GEOMETRIA PLANA: CONCEITOS FUNDAMENTAIS 253

MAT_PÁGINAS INICIAIS.indd 2 26/11/2022 22:28:15


GEOMETRIA PLANA: TRIÂNGULOS, CONGRUÊNCIA E PONTOS NOTÁVEIS 263
GEOMETRIA PLANA: QUADRILÁTEROS E POLÍGONOS 277
GEOMETRIA PLANA: CIRCUNFERÊNCIA E CÍRCULO 289
GEOMETRIA PLANA: SEGMENTOS PROPORCIONAIS 307
GEOMETRIA PLANA: TRIGONOMETRIA NO TRIÂNGULO RETÂNGULO
E LEIS DOS SENOS E COSSENOS 321
GEOMETRIA PLANA: RELAÇÕES MÉTRICAS NO TRIÂNGULO QUALQUER
E NOS QUADRILÁTEROS 331
GEOMETRIA PLANA: RELAÇÕES MÉTRICAS NOS POLÍGONOS REGULARES
E POTÊNCIA DE PONTO 343
GEOMETRIA PLANA: ÁREAS 357
CÍRCULO TRIGONOMÉTRICO 377
FÓRMULAS DE ADIÇÃO 393
FUNÇÕES TRIGONOMÉTRICAS E FUNÇÕES TRIGONOMÉTRICAS INVERSAS 403
EQUAÇÕES E INEQUAÇÕES TRIGONOMÉTRICAS 421
FUNDAMENTOS DE GEOMETRIA ESPACIAL 429
GEOMETRIA ESPACIAL: DIEDROS, TRIEDROS E POLIEDROS 443
GEOMETRIA ESPACIAL: PRISMAS E PIRÂMIDES 451
GEOMETRIA ESPACIAL: CONE E CILINDRO 459
ESFERAS E TRONCOS 475
GEOMETRIA PLANA: TRIÂNGULOS, CONGRUÊNCIA E PONTOS NOTÁVEIS 491
GEOMETRIA ANALÍTICA: COORDENADAS NO PLANO E DISTÂNCIA 507
GEOMETRIA ANALÍTICA: RETAS 513
GEOMETRIA ANALÍTICA: CIRCUNFERÊNCIA 525
GEOMETRIA ANALÍTICA: ELIPSE 535
GEOMETRIA ANALÍTICA: HIPÉRBOLE 543
GEOMETRIA ANALÍTICA: PARÁBOLA 551
EXERCÍCIOS DE REVISÃO I 559
EXERCÍCIOS DE REVISÃO II 563
EXERCÍCIOS DE REVISÃO III 569

MAT_PÁGINAS INICIAIS.indd 3 26/11/2022 22:28:15


FUNDAMENTOS ALGÉBRICOS

PRODUTOS NOTÁVEIS CUBO DA SOMA E DA DIFERENÇA


Os produtos notáveis são expressões algébricas que aparecem a DE DOIS TERMOS
todo instante na Matemática. Por isso, chamam-se notáveis e, assim a  b   a  b   a  b    a2  2ab  b2   a  b  
3 2

sendo, é fundamental que você memorize as fórmulas que colocarei


a seguir, que serão deduzidas utilizando a propriedade distributiva (o a3  a2b  2a2b  2ab2  b2a  b3  a3  3a2b  3ab2  b3
famoso “chuveirinho”) da multiplicação. Vamos a elas? Não passe
a  b
3
 a3  3a2b  3ab2  b3
para o assunto seguinte (FATORAÇÃO) sem ter decorado todas as
a  b   a   b    a3  3a2   b   3a   b    b  
3
fórmulas abaixo! 3 2 3

a3  3a2b  3ab2  b3
QUADRADO DA SOMA E
a  b
3
 a3  3a2b  3ab2  b3
DA DIFERENÇA DE DOIS TERMOS
 a  b 2   a  b   a  b   a2  ab  ba  b2  a2  2ab  b2 Exemplos:
 x  2  x 3  3x 2  2  3x  22  23  x 3  6x 2  12x  8
3

 a  b 2  a2  2ab  b2
 a  4b   a3  3a2   4b   3a   4b    4b  
3 2 3

 a  b 2   a   b  
2
 a2  2a   b    b   a2  2ab  b2
2
a3  12a2b  48ab2  64b3

 a  b 2  a2  2ab  b2 ProBizu
Exemplos: Pode ser útil escrever estes produtos notáveis das seguintes formas:

 a  4   a  2  a  4  4  a  8a  16
2 2 2 2
 a  b 3  a3  b3  3ab  a  b 
 a  72  a2  2  a  7  72  a2  14a  49  a  b 3  a3  b3  3ab  a  b 

QUADRADO DA SOMA DE TRÊS TERMOS


TRIÂNGULO DE PASCAL
a  b  c     a  b   c    a  b   2  a  b  c  c2 
2 2 2

Podemos generalizar os produtos notáveis 2.1 e 2.4 para calcular


a2  b2  c2  2ab  2ac  2bc expressões como (a + b)4, (a + b)5, (a + b)6,... Para isso, usaremos o
triângulo de Pascal (associado ao binômio de Newton que será
a  b  c 
2
 a2  b2  c2  2ab  2ac  2bc
estudado mais adiante):
Exemplos: 1
1 1
 a  b  1
2
 a2  b2  12  2ab  2a  1  2b  1 
1 2 1
a2  b2  2ab  2a  2b  1 a  2b  3c  
2
1 3 3 1
a2   2b    3c   2a   2b   2a   3c   2   2b    3c  
2 2
1 4 6 4 1
a2  4b2  9c2  4ab  6ac  12bc 1 5 10 10 5 1
1 6 15 20 15 6 1
ProBizu 1 7 21 35 35 21 7 1
Se quisermos calcular o quadrado da soma de mais termos, o 1 8 28 56 70 56 28 8 1
procedimento é completamente análogo. Temos, (a + b + ... +          
 a  b   kk + )²2 = a2  b2    k2  2  2ab  2ac    2a    2k , A linha de número k (a contagem inicia da linha 0) do triângulo
ou seja, o quadrado da soma é igual à soma dos quadrados mais
de Pascal corresponde aos coeficientes da expansão de (a + b)k. Assim,
duas vezes a soma dos produtos dos termos tomados dois a dois.
temos por exemplo:
 a  b 4  a4  4a3b  6a2b2  4ab3  b4
PRODUTO DA SOMA PELA DIFERENÇA  a  b 5  a5  5a4b  10 a3b2  10 a2b3  5aab4  b5
 a  b   a  b   a2  ab  ba  b2  a2  b2
 a  b 6  a6  6a5b  15 a4b2  20 a3b3  15a2b4  6ab5  b6
a  b a  b  a 2
b 2

Exemplo:
Exemplo:  a  2  a5  5a4  2  10a3  22  10a2  23  5a  24  25 
5

 x  4 y   x  4 y   x 2   4 y 2  x 2  16y 2
a5  10a4  40a3  80a2  80a  32

PM_BOOK16 - MAT.indb 5 25/11/2022 19:05:05


FUNDAMENTOS ALGÉBRICOS

Exercício Resolvido FATORAÇÃO


Fatorar uma expressão algébrica significa escrever tal expressão
01. Considere as afirmativas:
como produto de fatores mais simples. Tal procedimento é útil para,
 
2
1. Efetuando 2 3  3 2 , obtemos 30 − 12 6 . por exemplo, resolver equações algébricas. Vejamos agora algumas
2
técnicas de fatoração e fatorações conhecidas que você deve saber!
 7 24  1681 TÉCNICA 1 (COLOCAR EM EVIDÊNCIA): Quando um termo
2. O valor de  1  1  é igual a .
 9 25  225 aparece em todas as parcelas de uma expressão algébrica, é possível
 
2 colocar este em evidência. Matematicamente, temos o seguinte:
 3 3
3. Simplificando  6 27  6  obtemos .

 4 
 4 ab  ac  a b  c 

 
2
4. O valor de 12  3  48 é 6. Exemplo:

Conclua que: 
x 4  x 2y  xy 2  x x 3  xy  y 2 
a) Todas são verdadeiras.
TÉCNICA 2 (AGRUPAMENTO): Esta técnica é muito utilizada e
b) Três são verdadeiras e uma é falsa. consiste na seguinte ideia:
c) Duas são verdadeiras e duas são falsas. ab  ac  bd  cd  a b  c   d b  c   b  c   a  d
d) Somente (3) é falsa.
Veja que o agrupamento nada mais é que uma aplicação sucessiva
e) Todas são falsas. da técnica de colocar em evidência.
Resolução: B Exemplo 1:
1. (VERDADEIRA)  
x 6  x 4  x 2  1  x 4 x 2  1  1 x 2  1  x 2  1 x 4  1     
2   2 3   
2 2 2
33 2  22 3 3 2  3 2 
Exemplo 2:
12  12 6  18  30  12 6 Como consequência do agrupamento, podemos deduzir o
2. VERDADEIRA chamado Produto de Stevin:

 7
2 2 2 2 x 2   a  b  x  ab  x 2  ax  bx  ab 
24   16 49   4 7  20  21 1681
 1  1              x  x  a  b  x  a   x  a  x  b 
 9 25   9 25   3 5  15  225

3. VERDADEIRA • FATORAÇÃO 1 (DIFERENÇA DE QUADRADOS): Esta


2 2 2 fatoração é a mesma coisa que o produto notável (PRODUTO DA
6 3  6 3 27   27 
 27  6    3     3    SOMA PELA DIFERENÇA). Por sua grande importância, estamos
 4   4   4  ressaltando novamente:
2
27  27  a2  b2   a  b   a  b 
 3 9 27 27 3
2
 2 3     3  2   6 
4  4  2 4 4 4
Exemplo:
4. FALSA x 4  81   x 2   92   x 2  9   x 2  9  
2

   2    3
2 2 2
12  3  48 3 34 3 3 x 2
 32   x 2  9    x  3  x  3  x 2  9 

• FATORAÇÃO 2 (SOMA DE CUBOS): O objetivo aqui é fatorar a


Exercício Resolvido expressão a3 + b3. Para isso, vamos nos lembrar da expressão do
cubo da soma como escrita no PROBIZU. Lembre que vimos que
02. O produto ( 3− 2 )( 3 + 2 é igual a ) (a + b)3 – a3 + b3 + 3ab(a + b). Como queremos fatorar a3 + b3,
é natural isolar esta expressão de um dos lados da igualdade que
a) 6 vimos no ProBizu. Assim, temos que a3 + b3 – (a + b)3 – 3ab(a + b).
b) 1
Colocando (a + b) em evidência, temos que
 
c) 0
d) – 1
a3  b3   a  b   a  b   3ab   a  b  a2  ab  b2 .
2
 
e) – 6 Assim, chegamos à seguinte fatoração:
a3  b3   a  b  a2  ab  b2  
Resolução: B
Exemplo:
Para resolver esse produto, podemos aplicar o produto notável
produto da soma pela diferença, ou seja:
x 3  27  x 3  33   x  3 x 2  3x  9  
(a + b) · (a – b) = a2 – b2
• FATORAÇÃO 3 (DIFERENÇA DE CUBOS): Agora, queremos
( )( ) ( 3) − ( 2)
2 2
Assim: 3− 2 3+ 2 = = 3−2 =1 fatorar a expressão a3 − b3. Para isso, vamos aproveitar a
fatoração anterior:
Portanto, o produto é igual a 1.

a3  b3  a3   b    a   b   a2  a   b    b 
3 2

a  b a 2
 ab  b 2

Assim, temos
a3  b3   a  b  a2  ab  b2  

PM_BOOK16 - MAT.indb 6 25/11/2022 19:05:20


FUNDAMENTOS ALGÉBRICOS

Exemplo: 2. Feito isso, temos a seguinte fatoração:



x 3  64  x 3  43   x  4  x 2  4 x  16  ax 2  bx  c  a  x  x1   x  x 2 
Vejamos um exemplo para fixar a ideia.
• FATORAÇÃO 4 (SOMA E DIFERENÇA DE POTÊNCIAS):
Generalizando as duas últimas fatorações, temos os seguintes Exemplo:
resultados um pouco mais avançados. Fatore 6x2 + x – 2.

Para n inteiro positivo qualquer, vale: Vamos seguir o passo a passo:


1 12  4  6   2
x  a   x  a x
n n
 n 1
x n2
a    xa n2
a n 1
 1. As raízes são x1 
26

1 7 1
12

2
e

Observe que as potências de x vão caindo de 1 em 1 e as de a vão 1 12  4  6   2 1 7 2


aumentando de 1 em 1. x2    .
26 12 3
Exemplo:
2. Desta forma, temos que:
x5  32  x5  25   x  2  x 4  x 3  2  x 2  22  x  23  24  
 1  2  1  2
6x 2  x  2  6  x    x       2  x   3  x     2x  1  3x  2
 x  2  x 4  2x 3  4x 2  8x  16   2   3  2  3

Para n inteiro positivo ÍMPAR, vale:


TÉCNICA 5 (ENCONTRANDO RAÍZES DE UMA EXPRESSÃO

xn  an   x  a xn1  xn2a    xan2  an1  ALGÉBRICA): O seguinte resultado é extremamente útil: se uma
expressão algébrica em x se anula para x = a, então x − a deve ser um
Observe que as potências de x vão caindo de 1 em 1 e as de a fator da expressão. Vejamos um exemplo.
vão aumentando de 1 em 1. Além disso, repare também, que os sinais
vão alternando. Exemplo:
Exemplo: Fatore x3 – 6x2 + 11x – 6.

x5  32  x5  25   x  2  x 4  x 3  2  x 2  22  x  23  24   Veja que substituindo x por 1, temos que a expressão se anula.


Desta forma, x − 1 deve ser um fator. Sabendo desta informação,
 x  2  x 4  2x 3  4x 2  8x  16  vamos buscá-lo com alguns artifícios:
x 3  6x 2  11x  6  x 3  x 2  5x 2  5x  6x  6 

TÉCNICA 3 (COMPLETANDO QUADRADOS): Esta técnica é x 2  x  1  5x  x  1  6  x  1   x  1  x 2  5x  6 


muito útil quando temos uma expressão que é quase um quadrado Ainda podemos fatorar x 2  5x  6   x  2  x  3, obtendo assim
perfeito, mas não é. Vejamos dois exemplos para fixar as ideias.
que x 3  6x 2  11x  6   x  1  x  2  x  3.
Exemplos:
(Identidade de Sophie Germain) A expressão a ser fatorada é
a4 + 4b4. Veja que a4 + 4b4 – (a2)2 + (2b2)2. Da forma que escrevemos, Exercício Resolvido
a expressão lembra muito o quadrado de uma soma (de fato já
03. Se a e b são números reais tais que a – b = 7 e a2b – ab2 = 210,
temos o quadrado do primeiro termo e o do segundo termo). Pense
determine o valor de ab.
agora: o que está faltando para isto ser o quadrado de uma soma?
Está faltando o termo do meio!! Para consertar isso, vamos somar e Resolução:
subtrair o termo do meio, que é 2 ⋅ a2 ⋅ (2b2) – 4a2b2. Assim, temos que: Podemos colocar ab em evidência na expressão a2b – ab2, obtendo
4 4 4 2 2
a  4b  a  4a b  4b  4a b  a  2b4 2 2
 2 2 2
   2ab  .
2 que ab(a – b) = 210. Como a – b – 7, temos que 7ab = 210 ⇒
ab = 30.
Agora, temos uma diferença de quadrados e podemos fatorar
nossa expressão, obtendo o seguinte:


a4  4b4  a2  2ab  2b2 a2  2ab  2b2   Exercício Resolvido

04. A razão entre as formas fatoradas dos polinômios ax + 2a + 5x


Vejamos mais um exemplo: + 10 e a2 + 10a + 25 é:
(Identidade de Argand) A expressão a ser fatorada é
x4 + x2 + 1. Você, ao olhar para esta expressão, deve ter pensado: a)
( a + 5)( x − 2)
“poxa, se em vez de x2, fosse 2x2, eu saberia fatorar como quadrado ( a + 5)( a + 5)
de uma soma!”. Você pensou muito bem e com este pensamento, b) a + 5
podemos obter a fatoração. Se você quer que ali seja 2x2, basta somar
c) a–5
e subtrair x2, obtendo assim: x 4  x 2  1  x 4  2x 2  1 x 2   x 2  1  x 2 .
2

Mais uma vez, chegamos a uma diferença de quadrados e obtemos: x−2


d)
a+5
 
x 4  x2  1  x2  x  1 x2  x  1  x+2
e)
a+5
TÉCNICA 4 (EXPRESSÕES DO SEGUNDO GRAU): É bem
possível que você se depare com expressões da forma ax 2 + bx + c Resolução: E
pela frente. Fique tranquilo: é bem fácil fatorar expressões deste tipo.
ax + 2a + 5x + 10 a ( x + 2) + 5 ( x + 2) (=
a + 5)( x + 2) ( x + 2)
Vejamos como: = =
a2 + 10a + 25 ( a + 5)( a + 5) ( a + 5)( a + 5) ( a + 5)
1. Encontre as raízes x1 e x 2 da equação ax 2 + bx + c = 0 usando a
fórmula de Bhaskara, por exemplo.
 b  b2  4ac 
 x1,2  
 2a 
 

PM_BOOK16 - MAT.indb 7 25/11/2022 19:05:32


FUNDAMENTOS ALGÉBRICOS

Exercício Resolvido PROPRIEDADES


05. Fatore a expressão a3 + b3 + c3 – 3abc. TRANSITIVIDADE
Resolução: Se a > b e b > c, então a > c. (Isto é bem intuitivo pensando-se
na reta real)
Vamos escrever a3 + b3 + c3 – 3abc = (a + b)³ + c³ – 3ab(a + b)
– 3abc, onde utilizamos a expressão do ProBizu para o cubo da
soma. Agora, nos dois primeiros termos, usaremos soma de cubos ADIÇÃO
e nos dois últimos colocaremos –3ab em evidência. Desta forma, Se a > b e c > d, podemos somar as duas desigualdades:
temos: a + c > b + d.
a3  b3  c3  3abc 
Cuidado!
  a  b  c    a  b2   a  b c  c2   3ab  a  b  c  Não podemos subtrair duas inequações!
Colocando a + b + c em evidência, segue que Veja o seguinte problema que isso nos causaria: 5 > 3 e 0 > –3.

 
a3  b3  c3  3abc   a  b  c   a  b    a  b  c  c2  3ab 
2 Se pudéssemos subtrair as inequações, chegaríamos a 5 –0 >
3 –(–3), ou seja, 5 > 6, o que é um absurdo! Então grava essa:
 a  b  c   a2  b2  c2  ab  ac  bc  NUNCA SUBTRAIR INEQUAÇÕES!

MULTIPLICAÇÃO POR UMA CONSTANTE


ProBizu
Se c é um número positivo e a > b, então ac > bc.
Se a + b + c = 0, então a3 + b3 + c3 = 3abc. Agora, preste bastante atenção: se c é um número negativo e a > b,
(basta usar a fatoração do exercício resolvido 05). então ac < bc (VEJA QUE O SINAL INVERTE!).
Por exemplo, temos que 3 > 2. Ao multiplicar ambos os lados por
–1, devemos inverter o sinal, obtendo –3 < –2.
INEQUAÇÕES Cuidado!
Se chegarmos a uma situação como ac > bc, temos que analisar
O CONJUNTO DOS REAIS POSITIVOS
dois casos antes de cortar o c: se c for positivo, teremos a > b e se
Podemos fazer uma associação entre os pontos de uma reta c for negativo, teremos a < b (MAIS UMA VEZ O SINAL INVERTE!).
orientada e o conjunto dos números reais. Essa reta é chamada de
Fique bastante atento a isto!
reta real.

INVERTENDO DESIGUALDADES
1 1
Se a > b > 0, então < .
a b

DESIGUALDADE BÁSICA
Se x é um número real, então x2 ≥ 0.
Ainda podemos dividir esta reta real em duas partes: os
números menores que 0 (NEGATIVOS) e os números maiores que Estes conceitos de inequações serão muito úteis para módulos
0 (POSITIVOS). O conjunto dos reais positivos é denotado por *+ e que virão pela frente! Guarde-os bem para não ser surpreendido lá
na frente. Vamos enfrentar uma maratona de exercícios agora? Lute
o conjunto dos reais negativos é denotado por *−.Vejamos abaixo a
bastante com eles e não deixe de ver as soluções no final!
propriedade fundamental acerca do conjunto dos reais positivos:
PROPRIEDADE: Se x, y  *, então xy  * e x  y  *. Em
outras palavras, a soma de dois números positivos é positiva e o EXERCÍCIOS DE

FIXAÇÃO
produto de dois números positivos é positivo.

MAIOR E MENOR
Vamos introduzir agora uma simbologia muito útil:
Dizemos que a > b se a diferença a – b pertence ao conjunto dos 01. Sabendo que x² + y² = 153 e que xy = 36, calcule o valor de
reais positivos, isto é, se a – b > 0. Uma outra maneira de escrever isto (x + y)².
é que b < a.
Dizemos que a ≥ b se a = b ou a > b. 02. Qual o valor numérico da expressão (a – 2b)², sabendo-se que
a² + 4b² = 30 e ab = 5.
Vejamos alguns exemplos:
7>4 03. Se x – y = 7 e xy = 60, então o valor da expressão x² + y² é:
5≥3 a) 53 b) 109 c) 169 d) 420
3≥3
04. A expressão (x – y)² – (x + y)² é equivalente a:
Observação
a) 0 b) 2y² c) –2y³ d) –4xy
Pensando na reta real, a > b significa que a está à direita de b.

PM_BOOK16 - MAT.indb 8 25/11/2022 19:05:36


FUNDAMENTOS ALGÉBRICOS

05. Desenvolva: 04. (ESPM) Sabendo-se que x + y–1 = 7 e que x = 4y, o valor da
a) (x + y)³ = d) (a – 1)³ = expressão x² + y–2 é igual a:
b) (x – y)³ = e) (5 – x)³ = a) 49 c) 45 e) 41
c) (m + 3)³ = b) 47 d) 43

06. A expressão (a + b + c)² é igual a: 3 2


05. Se x + x −1 = então x3 + x–3 é igual a:
a) a² + 2ab + b² + c² 2
9 2 c) 27 2 e) 3 2
b) a² + b² + c² + 2ab + 2ac + 2bc a)
2 8 8
c) a² + b² + c² + 2abc
9 2 27 2
a² + b² + c² + 4abc b) d)
d) 4
4
e) a² + 2ab + b² + 2bc + c²
06. (UPE) Quando resolvemos a expressão (7.777)² – (2.223)²,
07. Seja N o resultado da operação 375² – 374². A soma dos encontramos o seguinte resultado:
algarismos de N é: a) 5,554 · 100 c) 5,554 · 104 e) 5,554 · 108
a) 18 c) 20 e) 22 b) 5,554 · 102 d) 5,554 · 107
b) 19 d) 21
07. (UFRGS) Se x – y = 2 e x² + y² = 8, então x³ – y³ é igual a:
08. Efetuando-se (579865)² – (579863)², obtém-se: a) 12 c) 16 e) 20
a) 4 c) 2 319 448 e) 1 159 728 b) 14 d) 18
b) 2 319 456 d) 2 086 246
08. (UFRGS) Se x + y = 13 e x · y = 1, então x² + y² é:
09. O produto (x + 1)(x² – x + 1) é igual a: a) 166 c) 168 e) 170
a) x³ – 1 b) 167 d) 169
b) x³ + 3x² – 3x + 1
c) x³ + 1 09. (INSPER) Se x² + y² + z² = xy + xz + yz = 6, então um possível valor
d) x³ – 3x² + 3x – 1 para a soma x + y + z é:

e) x² + 2 a) 6. c) 2 3. e) 3 3.
b) 2 2. d) 3 2.
10. Qual é o valor numérico do polinômio 2m + 2n, sabendo que
m + n = 10? 1
10. (CEFET MG) Se x + = 3 e 8x 6 + 4x 3y 2 ≠ 0, então o valor
x
EXERCÍCIOS DE 4x 9 + 2x 6 y 2 + 4x 3 + 2y 2
numérico da expressão ,é igual a:

TREINAMENTO
8x 6 + 4x 3y 2
a) 4 c) 9 e) 18
b) 7 d) 12

01. (UNIOESTE) Considere as seguintes afirmações: 11. (INSPER) Considere dois números positivos x e y, com x > y, tais
x +1 x +1
2  x + y + x − y =8
I. = , para todo x ∈ . que: 
x+2 2  x − y =
2 2
15
II. 2x + 5 = 2(x + 5), para todo x ∈ .
Nessas condições, 2x é igual a:
III. (x – 2)² = x² - 4x + 4, para todo x ∈ .
a) 31 c) 33 e) 35
Assim, é CORRETO afirmar que:
b) 32 d) 34
a) somente a afirmação I está correta.
b) somente a afirmação II está correta. 12. (UPF) Quando a e b assumem quaisquer valores positivos, das
c) somente as afirmações I e II estão corretas. expressões a seguir, a única que não muda de sinal é:
d) somente a afirmação III está correta. a) a² – ab c) b− b e) a² – 2ab + b²
e) as três afirmações estão corretas. b) a² – b² d) a² – 3a

02. (ESPM) O inverso multiplicativo do número 7 + x é o número 1 1


13. (UEPB) Dado x − = 13, o valor de x 2 + 2 é igual a:
7 − x. O valor de x + 1 é igual a: x x
a) 7 c) 12 e) 5 a) 171 c) 167 168
e)
b) 169 d) 130 13
b) 3 d) 8

03. Sabendo que x² + y³ = 1 e x4 + y6 = 2, o valor de (x² – y³)² – x4 – 14. (ESPM) O par ordenado (x,y) ∈  ×  é solução da equação
2x³y³ – y6 é x³ + x²y – 8x – 8y = 7. O valor de x – y é:

a) 0 c) –1 e) -2 a) 1 c) –1 e) –2

b) 1 d) 2 b) 2 d) 0

PM_BOOK16 - MAT.indb 9 25/11/2022 19:05:38


FUNDAMENTOS ALGÉBRICOS

 x −2 − y −2   x 2y + xy 2  22. (CN) Considere o sistema abaixo nas variáveis reais x e y, sendo


15. (EPCAR) O valor da expressão  −1 ⋅
−1   2 
, em que x a e b reais.
x +y   x −y 
2
e y ∈ * e x ≠ y e x ≠ –y, é
375y x − 125y − 375yx + 125x =
2 3 2 3
125b
a) -1 c) 1 
 y + x + 2yx =
2 2
a2

b) -2 d) 2
Nessas condições, qual será o valor de (x2 – y2)6?
16. (EPCAR) Sabendo que y = (2010)² · 2000 – 2000 · (1990)², o valor a) a3b6 d) a3b6
y
de é igual a: b) a b8 6
e) a4b6
107
c) ab
6 2

a) 8 c) 20
b) 16 d) 32
23. (CN) O conjunto solução da equação x + =
1 x 2 + 4x 2 + 4x + 1
em , conjunto dos números reais, é
3
17. (CN) Seja x um número real tal que x + =9. Um possível valor a) . c)  - [-1,∞[.
x e)  1 
3 − ,∞ .
de x − é α . Sendo assim, a soma dos algarismos “α” será: b) [-1,∞[. d) [0,∞[.  2 
x
a) 11 d) 14 24. (ITA) A área do polígono, situado no primeiro quadrante, que é
b) 12 e) 15 delimitado pelos eixos coordenados e pelo conjunto:
c) 13 {(x, y) ∈ 2: 3x2 + 2y2 + 5xy – 9x – 8y + 6 = 0}, é igual a:
a) 6 c) 2 2 10
  y 2  2 e)
1 −    ⋅ x 5 d) 3 3
  x   b)
18. (EPCAR) Simplificando as expressões A =  e 2
( x − y )2 + 2 xy
x 2 − xy
B= , nas quais y > x > 0, é correto afirmar que: 25. (ITA) Se x é um número real que satisfaz x³ = x + 2, então x10 é
2x
igual a:
A c) A·B>0
a) = 2−1 a) 5x 2 + 7x + 9. c) 13x 2 + 16x + 12. e) 9x 2 + 3x + 10.
B d) A + B > 0
b) 3x 2 + 6x + 8 d) 7x 2 + 5x + 9.
B
b) ∈
A

19. (EPCAR) Considere o conjunto de todos os valores de m e EXERCÍCIOS DE


n para os quais a expressão algébrica A abaixo, está definida.
m2 n2

(m − n)−2
COMBATE
=A n2 m2 ⋅ 2 . Nesse conjunto, uma expressão
1 2 1 (m − n2 )−1
+ +
m2 m ⋅ n n2
x  y 3  z3    x 3  y 3  z3 
3 2 2

algébrica equivalente a A é: 01. (CN 1998) A expressão , x ⋅ y ⋅ z ≠ 0,


é equivalente a: y 3  z3
m +n2 2
m +n
2 2
a) m² + n² b) m² – n² c) d)
m2 − n2 m−n a) 4x3 c) 4yx3 e) 4xz3
b) 4yzx 3
d) 4xyz
20. (CN) Analise as afirmativas abaixo:
x+y+z x+y+z+t 02. Um aluno encontrou zero para o valor numérico da expressão
I. =
Se 7=
e 5, então t = 2.
3 4 x 2  y 2  2x  5  4 y. Pode-se concluir que os valores pelos quais
16 + 20 + x + x + x +  + x x + x + x ++ x substituiu as variáveis x e y são tais que sua soma é:
II. Se =
1 2 3 10
=
8, então 1 2 3 10
6.
12 10
a) –2 b) –1 c) 0 d) 1 e) 2
x+y+z x 2 + y 2 + z2 xy + xz + yz 3a2 − b
III.=
Se a=
e =
b, então .
3 3 3 2 03. Qual é o produto notável representado geometricamente, na
Assinale a opção correta. figura abaixo, na qual ABCD é um quadrado?
a) Apenas a afirmativa I é verdadeira. a) a³ + b³
b) Apenas a afirmativa III é verdadeira. b) (a + b)³
c) Apenas as afirmativas II e III são verdadeiras. c) (a + b)²
d) Apenas as afirmativas I e III são verdadeiras. d) (a² + b²)²
e) Apenas as afirmativas I, II e III são verdadeiras. e) (a + b)4

21. (FGV) Fatorando completamente o polinômio x9 – x em polinômios


e monômios com coeficientes inteiros, o número de fatores será
a) 7. d) 3.
b) 5. e) 2.
c) 4.

10

PM_BOOK16 - MAT.indb 10 25/11/2022 19:05:40


FUNDAMENTOS ALGÉBRICOS

04. (EPCAR 1985) A expressão x2 – 2x – 2y – 2z + yx + zx é equivalente


a:
a) (1 – y –z)(x2 – 2)
b) (x – y + z)(2 + x)
d) (x + y + z)(x – 2)
e) (x – y – z)(x + 2)
DESAFIO PRO
c) (x – y – z)(2 – x)

05. (CMRJ 2000) Simplificando a fração algébrica encontramos


2x 3  3x 2  27
1 (UK

x =+
NATIONAL

1 19922 +
19922 1992
+
MATHEMATICS CONTEST

, então qual das afirmações


1992) Se

19932 1993
2x 4  3x 3  9x 2  27x  81
é verdadeira?
a) x + 3 1 a) 1992 < x < 1993
d)
b)
1 x −3 b) x = 1993
x+3 x+3 c) 1993 < x < 1994
e)
c) x–3 x −3 d) x = 1994
e) x > 1994
06. Sejam ‘a‘, ‘b‘ e ‘c‘ números reais não nulos tais que
1

1
ab bc ac

1 a b c a b c
 p ,       q e ab + ac + bc – r. O valor
b a a c c b
2 Sabendo que x, y e z são reais satisfazendo xyz = 1, calcule
o valor da expressão:
de q² + 6q é sempre igual a 1 1 1
A= + + .
2 2 2 2 1 + x + xy 1 + y + yz 1 + z + xz
p r +9 p r − 10
a) d)
4 4r a) 0
2 2
p r − 9p e) p²r² – 12p b) 1
b) c) -1
12
c) p²r² – 9 d) 2
e) -2
07. (EPCAR 1984) Sendo E'   a  b    a  b    a  b   a  b  e
2 2

a−b b−c c−a


3
  3
  3
E''  a  b  a  b , identifique E’ + E”: =
Se x = ,y = ez , então o valor de
a+b b+c c+a
a) (2a – 1)a2 + (6a + 1)b2 + 4ab (1 + x ) (1 + y ) (1 + z )
b) (2a + 1)a2 + (6a – 1)b2 – 4ab é
(1 − x ) (1 − y ) (1 − z )
c) (2a – 1)a2 – (6a + 1)b2 – 4ab a) 0
d) (2a + 1)a2 – (6a + 1)b2 + 4ab b) -1
e) (2a – 1)a2 – (6a – 1)b2 + 4ab c) 1
d) a + b + c
08. (CN 1984) Se 2  2  2  x  y  z  8 e x + y + z = 16,
( a + b )(b + c )( c + a)
o produto x · y · z é:x y z yz xz xy 3 e)
( a − b )(b − c )( c − a)
a) 192
b) 48
c) 32
d) 108
4 Sejam a, b, c números reais não nulos tais que a + b + c = 0
e a³ + b³ + c³ = a5 + b5 + c5. O valor de a2 + b2 + c2 é
a) 1
e) 96 3
b)
4
09. (EPCAR) Considere as expressões P e Q, com os números a, b e c
5
reais positivos e distintos entre si. c)
4
(a6 + b6 + c6 )2 − (a6 − b6 − c6 )2
P= 5
b6 + c6 d)
4
(b − a ) − (b−1 + a−1)−1
−1 −1 −1
Q = −1 6
(a + b−1)−1 − (a−1 − b−1)−1 e)
5
A expressão Q P é representada por

a) b 2a b) a 2b c) a
b
2
d)
1 b
a 2
5 (OMERJ) Um fator entre 1000 e 5000 do número
233 – 219 – 217 – 1 é igual a:
a) 1999
10. Para se explicitar x na equação ax² + bx + c = 0, a ≠ 0, usa-se o b) 1998
recurso da complementação de quadrados. Usando-se o recurso da c) 1993
complementação de cubos um aluno determinou uma raiz real r da
d) 1988
equação x³ – 6x² + 12x – 29 = 9. Pode-se afirmar que:
e) 1983
a) 0 < r < 1 d) 3 < r < 4
b) 1 < r < 2 e) 4 < r < 5
c) 2<r<3

11

PM_BOOK16 - MAT.indb 11 25/11/2022 19:05:45


FUNDAMENTOS ALGÉBRICOS

GABARITO
EXERCÍCIOS DE FIXAÇÃO
01. 225 d) a³ − 3a² + 3a − 1
02. 10 e) 125 − 75x + 15x² − x³
03. C 06. B
04. D 07. C
05. a) x³ + 3x²y + 3xy² + y³ 08. B
b) x³ − 3x²y + 3xy² − y³ 09. C
c) m³ + 9m² + 27m + 27 10. 2(m + n) = 2 · 10 = 20
EXERCÍCIOS DE TREINAMENTO
01. D 08. B 15. A 22. C
02. A 09. D 16. B 23. E
03. D 10. C 17. E 24. B
04. E 11. D 18. C 25. C
05. B 12. E 19. A
06. D 13. A 20. C
07. E 14. C 21. B
EXERCÍCIOS DE COMBATE
01. A 04. D 07. B 10. E
02. B 05. B 08. E
03. C 06. C 09. B
DESAFIO PRO
01. B 03. C 05. E
02. B 04. E

ANOTAÇÕES

12

PM_BOOK16 - MAT.indb 12 25/11/2022 19:05:45


RAZÃO E PROPORÇÃO

RAZÃO Exemplo:
A razão entre dois números a e b é definida como sendo a fração Dividir 100 em partes diretamente proporcionais a 2, 3 e 5.
a Solução:
ou a : b. Em uma razão, a e b são ditos os termos da razão, onde
b
o primeiro é chamado de antecedente e o segundo de consequente. x y z x  y  z 100
     10
Exemplos: 2 3 5 2  3  5 10
• A razão de 3 para 4 é 0,75. x = 2 ⋅ 10 = 20, y = 3 ⋅ 10 = 30 e z = 5 ⋅ 10 = 50
• A velocidade média de um móvel é a razão entre a distância
percorrida e o tempo gasto para percorrê-la. INVERSAMENTE PROPORCIONAIS
• A densidade de um corpo é a razão entre a massa que um Dividir um número em partes inversamente proporcionais a
corpo possui e o volume ocupado por ele. uma lista de números é dividi-lo de forma que cada uma das partes
• A escala é a razão entre a medida de uma distância no resultantes seja proporcional aos inversos dos números dados.
desenho e a medida real. Exemplo:
Dividir 235 em partes inversamente proporcionais a 3, 4 e 5.
Observação
Solução:
Se multiplicarmos ou dividirmos os termos de uma razão por um x y z xyz 235
mesmo número diferente de zero, a razão não se altera.      300
1 1 1 1 1 1 47
 
3 4 5 3 4 5 60
PROPORÇÃO x
1 1 1
 300  100, y   300  75 e z   300  60
É a igualdade entre duas ou mais razões. 3 4 5

Podemos escrever uma proporção das duas formas a seguir:


a c
= ou a : b : c : d. REGRA DE SOCIEDADE
b d A divisão dos lucros ou prejuízos em uma sociedade é diretamente
Dizemos que a e d são os extremos e b e c são os meios da proporcional ao capital aplicado e ao tempo que o mesmo permaneceu
proporção. aplicado.
Exemplo:
PROPRIEDADES
Uma sociedade foi fundada pelos sócios A e B com capitais de
I. O produto dos meios é igual ao produto dos extremos. R$ 300.000,00 e R$ 150.000,00. Após 6 meses do início foi
a c admitido um novo sócio C com capital de R$ 200.000,00. Sabendo
Se = , então ad = bc.
b d que a sociedade obteve um lucro R$ 110.000,00, após 1 ano de
II. a  c  a  b  c  d e a  b  c  d . funcionamento, que parcela do lucro coube a cada um dos sócios?
b d a c b d Solução:
a c ac ac
III.    , se . Sócio A: R$ 300.000,00 por 12 meses
b d bd bd
Sócio B: R$ 150.000,00 por 12 meses
Observação Sócio C: R$ 200.000,00 por 6 meses
a c Parcela de A: a = 300.000 · 12 · k
Quando temos uma proporção do tipo = , podemos utilizar
b d Parcela de B: b = 150.000 · 12 · k
a c
o seguinte artifício:   k  a  b  k e c  d  k , onde k é
b d Parcela de C: c = 200.000 · 6 · k
chamado constante de proporcionalidade. Simplificando os valores, tem-se:
Parcela de A: a = 3 · 2 · k’ = 6k’
Parcela de B: b = 1,5 · 2 · k’ = 3k’
DIVISÃO EM PARTES PROPORCIONAIS
Parcela de C: c = 2 · 1 · k’ = 2k’
E REGRA DE SOCIEDADE Lucro: 6k’ + 3k’ + 2k’ = 110.000 ⇒ k’ = 10.000
Logo, a parcela de A é R$ 60.000,00; a parcela de B é
DIRETAMENTE PROPORCIONAIS R$ 30.000,00 e a de C é R$ 20.000,00.
Dividir um número em partes proporcionais a uma lista de
números é dividi-lo de forma que cada uma das partes resultantes seja
proporcional aos números dados. Vejamos agora um exercício resolvido:

13

PM_BOOK16 - MAT.indb 13 25/11/2022 19:05:51


RAZÃO E PROPORÇÃO

Exercício Resolvido Exemplo:


Um piloto de kart faz um treino para a competição de “1000 metros
01. Sejam a, b, c e k números reais diferentes de zero satisfazendo contra o relógio”, mantendo em cada volta uma velocidade constante e
a b c obtendo, assim, um tempo correspondente, conforme a tabela abaixo:
as relações k    . Qual é o número de possíveis
bc c a ab VELOCIDADE (m/s) TEMPO (s)
valores que k pode assumir?
5 200
a) 0
b) 1 10 100

c) 2 20 x
d) 3 Qual é o valor de x?
e) 4 Note que as grandezas são inversamente proporcionais, ou
seja, quando aumentamos a velocidade, o tempo gasto diminui.
Resolução: C Assim, teremos:
Utilizando a propriedade iii de proporções, temos que: 10 x 1000
 x  x  50
abc abc 20 100 20
k  , se a + b + c ≠ 0. Portanto, o piloto demorará 50 segundos para completar o
b  c    c  a   a  b  2 a  b  c 
percurso a 20 m/s.
1
Assim, se a + b + c ≠ 0, temos que k = .
2 Para resolver problemas de regras de três, vamos nos basear no
a a seguinte:
Por outro lado, se a + b + c = 0, temos que k    1,
b  c a
onde usamos que a + b + c = 0 ⇒ b + c = –a. Observação
Com isso, obtemos dois possíveis valores para k. Uma grandeza pode ser diretamente proporcional a várias
grandezas e inversamente proporcional a outras. Se uma grandeza
x é diretamente proporcional a a, b, c e inversamente proporcional
abc
REGRA DE TRÊS a m, n, p escrevemos x  k 
mnp
, onde k é chamada de constante
Este é um tópico com o qual as pessoas possuem muita de proporcionalidade. Por exemplo, a lei de Coulomb afirma que
familiaridade, pois aparece frequentemente na resolução de dadas duas partículas com cargas Q e q, a uma distância d uma da
problemas de Química, por exemplo. Há métodos esquemáticos para outra, há uma força, chamada força elétrica, que uma faz na outra,
resolver regras de três, mas quando muitas variáveis são envolvidas, kQq
estes métodos podem acabar se tornando complicados. Por isso, cujo módulo é F = 2 . Assim, a força é diretamente proporcional
d
utilizaremos aqui o conceito de funções para resolver problemas às cargas e inversamente proporcional ao quadrado da distância. A
relacionados a regra de três. Antes de começarmos a ver exemplos, constante k é chamada de constante eletrostática do meio.
precisamos de dois conceitos:

Vejamos dois exercícios resolvidos:


GRANDEZAS DIRETAMENTE PROPORCIONAIS
Duas grandezas a e b são ditas diretamente proporcionais se Exercício Resolvido
a
existe uma constante k tal que = k. 02. O dono de uma carpintaria sabe que precisa de 50 operários para
b
Exemplo: fazer 10 armários em 5 dias. Sabendo que ele tem 20 armários para
Um atleta percorre 30 km em 3h. Mantendo o mesmo ritmo, em serem feitos em apenas dois dias, de quantos operários vai precisar?
quanto tempo ele percorrerá 70 km? Resolução:
Montemos uma tabela: Representaremos as variáveis operários, armários e dias por O, A e
D, respectivamente.
PERCURSO (km) TEMPO (h) Como estamos interessados na quantidade de operários
necessários para fazer os armários, analisaremos a relação entre O
30 3
e A e entre O e D.
70 x I. O e A são diretamente proporcionais, pois quanto mais
operários tivermos, mais armários serão produzidos.
Notem que as grandezas são diretamente proporcionais, ou II. O e D são inversamente proporcionais, pois quanto mais
seja, se aumentarmos o percurso, o tempo gasto pelo atleta também operários tivermos, menos dias serão necessários.
aumenta. Logo, devemos conservar a proporção:
A
30 3 Assim, podemos escrever O  k  .
  30x  210  x  7 . D
70 x Agora, analisaremos a situação inicial e a situação final:
Portanto, o atleta percorrerá 70 km em 7 h. • Situação inicial:
10
Substituindo os valores para as variáveis, temos 50  k   k  25
GRANDEZAS INVERSAMENTE 5
• Situação final:
PROPORCIONAIS
A 20
Agora, temos que A = 20 e O = 2, ou seja, O  k   25   250.
Duas grandezas a e b são ditas inversamente proporcionais se D 2
k • Desta forma, são necessários 250 operários para realizar o
existe uma constante k tal que: a = . trabalho.
b

14

PM_BOOK16 - MAT.indb 14 25/11/2022 19:05:59


RAZÃO E PROPORÇÃO

Exercício Resolvido
V
O ralo retira do tanque de água.
03. Se K abelhas, trabalhando K meses do ano, durante K dias do 15
V V V
mês e durante K horas por dia, produzem K litros de mel; então o Assim, em uma hora, entra no tanque   de água.
número de litros de mel produzidos por W abelhas, trabalhando W 12 15 60
horas por dia, em W dias e em W meses do ano será: Sendo t o tempo necessário em horas para encher o tanque
V
completamente, devemos ter  t  V  t  60 horas. Desta
K3 K4 W4 60
a) c) e)
W2 W3 K3 forma, o tanque ficará cheio em 60 horas.
W5 W3
b) d)
K3 K3
Exercício Resolvido
Resolução: E
Neste problema, temos 5 variáveis, representadas pelas letras entre 05. Três máquinas P, Q e R, trabalhando juntas, podem fazer um
parêntesis. trabalho T em x horas. Quando trabalhando sozinha, P necessita
de um adicional de 6 horas para realizar o mesmo trabalho, Q um
Número de abelhas (A)
adicional de 1 hora e R um adicional de x horas. Determine o valor
Número de meses (B) de x.
Número de dias (C) a) 6 2 1
Número de horas por dia (D) b) 3 d) e)
3 6
Quantidade de mel (E) c) 1
Como queremos saber a quantidade de mel produzida,
analisaremos a relação entre a variável E e as outras variáveis: Resolução: D
E e A: são diretamente proporcionais, pois quanto mais abelhas, De acordo com o enunciado, temos:
mais mel será produzido. A máquina P necessita de x + 6 horas para fazer o trabalho.
E e B: são diretamente proporcionais, pois quanto mais meses de A máquina Q necessita de x + 1 horas para fazer o trabalho.
trabalho, mais mel será produzido. A máquina R necessita de x + x = 2x horas para fazer o trabalho.
E e C: são diretamente proporcionais, pois quanto mais dias de Para resolver o problema, devemos analisar a fração do trabalho
trabalho, mais mel será produzido. que cada máquina realiza em uma hora:
E e D: são diretamente proporcionais, pois quanto mais horas por 1
dia, mais mel será produzido. Em uma hora, P realiza do trabalho.
x +6
São diretamente proporcionais, pois quanto mais horas por dia, 1
Em uma hora, Q realiza do trabalho.
mais mel será produzido. x +1
Como a letra K já aparece no problema, usaremos M para constante 1
Em uma hora, R realiza do trabalho.
proporcionalidade e assim podemos escrever E = M ⋅ ABCD. 2x

Na situação inicial, temos A = B = C = D = E = K, o que nos dá Com isso, as máquinas realizam juntas, em uma hora,
1 1 1 1 2x  x  1 2x  x  6   x  1  x  6  5x 2  21x  6
K  M  K4  M  3 .      
K x  6 x  1 2x 2x  x  6   x  1 2x  x  6   x  1 2x  x  6   x  1 2x  x  6   x  1
ABCD
Logo E = . 1 1 1 2x  x  1 2x  x  6   x  1  x  6  5x 2  21x  6
K3       do trabalho.
x  6 x  1 2x 2x  x  6   x  1 2x  x  6   x  1 2x  x  6   x  1 2x  x  6   x  1
Na situação final, temos A = B = C = D = W e assim teremos, o que
nos dá a opção E. O enunciado ainda diz que as três máquinas realizam juntas o
5x 2  21x  6
trabalho em x horas. Desta forma, segue que x  1
2x  x  6   x  1
(igualamos a 1, pois 1 corresponde ao trabalho todo).
PROBLEMAS DO TIPO TORNEIRA
5x 2  21x  6
Os problemas do tipo torneira são problemas envolvendo Com isso, temos que x   1  5x 2  21x  6  2  x  6   x  1  3x 2 
tanques que possuam torneiras para seu enchimento e ralos para seu 2 x  x  6   x  1
5x 2 análogas
 21x  6 a
esvaziamento ou problemas em que ocorram situações
x  1  5x 2  21x  6  2  x  6   x  1  3x 2  7x  6  0
esta. Para resolver estes problemas, devemos analisar  x fração
2 xque 6   x do
1
Resolvendo a equação do segundo grau, temos que
trabalho é realizada em 1 hora. Vejamos dois exercícios resolvidos:
7  72  4  3   6  7  11 2
Exercício Resolvido x  , ou seja, x = –3 ou x = .
23 6 3
Como x é positivo (pois corresponde a uma quantidade de horas),
04. Uma torneira enche um tanque em 12 horas; um ralo esvazia
2
este mesmo tanque em 15 horas. Estando o tanque vazio e temos que x = .
abrindo-se a torneira e o ralo simultaneamente, em quanto tempo 3
o tanque ficará cheio?

Resolução: PORCENTAGEM
Seja V o volume do tanque. No período de uma hora, temos o
seguinte: DEFINIÇÃO
V É uma razão cujo denominador é igual a 100. Representamos a
A torneira coloca no tanque de água.
12 porcentagem pelo símbolo % (por cento).

15

PM_BOOK16 - MAT.indb 15 25/11/2022 19:06:12


RAZÃO E PROPORÇÃO

Exemplos: II. VENDAS COM PREJUÍZO: o preço de venda é obtido pelo preço
13 1 25 de custo menos o prejuízo.
= 13=%, = 25%
100 4 100 V=C−P
Exemplos:
VARIAÇÃO PERCENTUAL 1. Uma blusa social custou R$ 120,00. Por quanto deve ser
vendida para que haja um lucro de 15% sobre o preço de custo?
Esta é a parte mais importante referente à porcentagem e que
causa mais complicações nos alunos, apesar de ser simples. Preste Solução:
bastante atenção para nunca mais errar isto em provas! Temos que V = C + L. Para que haja um lucro de 15%
sobre o preço de custo, devemos ter L = 0,15C, teremos que
AUMENTO PERCENTUAL V = 1,15C = 1,15 ⋅ 120 = R$ 138,00.
Quando uma determinada quantidade x aumenta de i%, seu
i  i  2. Uma mercadoria foi vendida por R$ 180,00, com um prejuízo de
novo valor será x  x   x 1  . Desta forma, para obter o 10% sobre o preço de venda. Qual o preço de custo dessa mercadoria?
100  100 
i Solução:
novo valor, basta multiplicar a quantidade antiga por 1+ .
100 Temos que V = C – P. Para haver um prejuízo de 10% sobre o preço
Exemplo:
Um carro, que custava R$ 12.000,00, sofreu uma valorização de venda, devemos ter P = 0,1V, teremos que C = 1,1V = R$ 198,00.
(acréscimo) de 10% sobre o seu preço. Quanto ele passou a custar?
Vejamos agora um exercício resolvido:
Solução:
Para um aumento de 10%, devemos multiplicar a quantidade Exercício Resolvido
10
inicial por 1  11
, . Desta forma, o carro passou a custar 06. Uma determinada conta a pagar de valor X vence no dia 30 de
100
12000 ⋅ 1,1 = 13200 reais. novembro, mas, se for paga até o dia 30 de setembro, tem 20%
de desconto sobre X e, se for paga até o dia 31 de outubro, tem
10% de desconto sobre X. Alguém reservou o valor exato Y para
DIMINUIÇÃO PERCENTUAL pagar essa conta no dia 30 de setembro, no entanto esqueceu-se
Quando uma determinada quantidade x diminui de i%, seu novo de fazê-lo e só efetuou esse pagamento no dia 31 de outubro.
i  i  Qual a porcentagem a mais sobre Y que terá de pagar?
valor será x  x   x 1  . Desta forma, para obter o novo
100  100  a) 10%
i
valor, basta multiplicar a quantidade antiga por 1− . b) 12,5%
100
Exemplo: c) 17,5%
Uma mercadoria no valor de R$10000,00 sofreu dois aumentos d) 20%
sucessivos de 10% e um desconto de 7%. Qual o novo valor da e) 25%
mercadoria?
Solução: Resolução: A
Dois aumentos de 10% equivalem a multiplicar por Para pagar a conta no dia 30 de setembro, como há 20% de
 10   10  
desconto, a pessoa necessita de 1
20 
1   1   11
,  11
,  1, 21 e um desconto de 7% equivale a  X  0, 8X para efetuar
 100   100   100 
 7  o pagamento, ou seja, Y = 0,8X. Com o pagamento sendo
multiplicar por 1   0, 93 . Assim, o novo valor da mercadoria feito no dia 31 de outubro, o desconto era de apenas 10%
 100 
sobre X e assim a pessoa deve pagar 1
será 10000 ⋅ 1,21 ⋅ 0,93 = 11253 reais. 10 
 X  0, 9X . Sendo
 100 
Observação i% a porcentagem a mais sobre Y que será paga, temos que
Uma grandeza pode ser diretamente proporcional a várias  i   i  i 9
1  Y  0, 9X  1   0, 8X  0, 9X  1 
grandezas e inversamente proporcional a outras. Se uma grandeza  100   100  100 8
x é diretamente proporcional a a, b, c e inversamente proporcional i 9 1 100
Logo  1  i   12, 5 .
abc 100 8 8 8
a m, n, p escrevemos x  k  , onde k é chamada de constante
mnp
de proporcionalidade. Por exemplo, a lei de Coulomb afirma que
dadas duas partículas com cargas Q e q, a uma distância d uma da
outra, há uma força, chamada força elétrica, que uma faz na outra, JUROS
kQq
cujo módulo é F = 2 . Assim, a força é diretamente proporcional DEFINIÇÕES
d
às cargas e inversamente proporcional ao quadrado da distância. A I. Juros: é o rendimento que se recebe pela aplicação de um capital
constante k é chamada de constante eletrostática do meio. a uma determinada taxa durante certo tempo.

Observação

OPERAÇÕES SOBRE MERCADORIAS 1. Quando o problema falar em taxa de juros i, devemos ter bom
senso para saber se devemos considerar realmente i ou i%.
São operações que envolvem a compra e venda de mercadorias e
o lucro ou prejuízo nessas operações. 2. O tempo t deve ser expresso na mesma unidade a que estiver
I. VENDAS COM LUCRO: o preço de venda é obtido pelo preço referenciada a taxa i. Se a taxa de juros estiver ao ano, o
de custo mais o lucro. tempo deve ser expresso em anos, assim como se a taxa de
juros estiver em meses, o tempo deve ser expresso em meses.
V=C+L

16

PM_BOOK16 - MAT.indb 16 25/11/2022 19:06:20


RAZÃO E PROPORÇÃO

II. Juros simples: o regime de juros será simples quando o Exercício Resolvido
percentual de juros incidir apenas sobre o valor principal (valor
inicial que é emprestado ou aplicado, também chamado de 08. César aplicou R$ 10.000,00 num fundo de investimentos que
capital). Sobre os juros gerados a cada período não incidirão rende juros compostos a uma certa taxa de juro anual positiva
novos juros. Podemos calcular os juros através da fórmula i. Após um ano, ele saca desse fundo R$ 7.000,00 e deixa o
J = C ⋅ i% ⋅ t, onde C é o capital, i% é o percentual de juros restante aplicado por mais um ano, quando verifica que o saldo é
incidentes e t é o tempo de aplicação. R$ 6.000,00. O valor de (4i – 1)² é:
III. Montante: é o acúmulo do capital com os juros. a) 0,01
b) 0,02
IV. Juros compostos: chamamos de juros compostos a remuneração
que o capital C recebe após n períodos de aplicação, a percentual c) 0,03
de juros de i% ao período, quando a cada período, a partir do d) 0,04
segundo, os juros são calculados sobre o montante do capital no e) 0,05
período anterior.
Resolução: D
Neste caso, pelas opções, vemos que o valor i citado no enunciado
CÁLCULO DO MONTANTE
já está como percentual. Desta forma, temos a seguinte situação:
JUROS SIMPLES Após um ano, o montante de César era de 10000 (1 + i). Com a
Neste caso, o montante é dado por retirada de 7.000 reais, temos que César ficou com 10000(1 + i)
– 7000 = 3000 + 10000i reais no fundo. Aplicando este valor por
M = C + J = C + C ⋅ i% ⋅ t = C(1 + i% ⋅ t). mais um ano, temos que o montante final será (3000 + 10000i)
M = C(1 + i% ⋅ t) (1 + i). Como o saldo verificado é de 6000 reais, temos:
3 000  10 000i 1  i  6 000  3  10i 1  i  6 
JUROS COMPOSTOS 10i2  13i  3  0
Denotaremos por Mk o montante após k períodos de tempo. 3
Temos então a seguinte equação: Resolvendo esta equação do segundo grau, obtemos i   ou
2
Mk = Mk–1 + (1 + i%)Mk–1 ⇔ Mk = (1 + i%)Mk–1 1 1
i= . Como a taxa i é positiva, temos que i = . Logo
5 5
Desta forma, a sequência dos montantes forma uma progressão 2 2
 4i  12    1       0, 04 .
4 1 1
geométrica de razão 1 + i%. Veremos no próximo módulo que o termo
5   5 25
geral de uma sequência como esta é dado por Mn = M0(1 + i%)n.
Como M0 é igual ao capital inicial, temos que:

Mn = C(1 + i%)n
EXERCÍCIOS DE

Vejamos agora dois exercícios resolvidos:

Exercício Resolvido
FIXAÇÃO
07. Um capital C foi aplicado a uma taxa mensal numericamente 01. (CMRJ) No dia 22 de março, é comemorado o Dia Mundial da
igual ao capital. Quantos meses são necessários para que os juros Água, data criada para nos conscientizar sobre a importância desse
simples sejam iguais ao quadrado do capital? recurso fundamental para a vida no planeta. Em tempos de escassez
a) 20 de água, toda medida de economia é muito bem-vinda. Assim, ao
pesquisar sobre consumo de água em residências, Maria descobre
b) 50 que, nos seus banhos diários de 15 minutos, são gastos 135 litros
c) 100 de água. Assustada com o desperdício, ela resolve reduzir seu banho
d) 200 para 9 minutos, obtendo uma economia considerável de água a
cada banho. Se Maria tomar apenas um banho por dia, o volume
e) 400
economizado de água, em 30 dias será de
Resolução: C a) 1,62 m³ c) 162 dm³ e) 243.000 cm³
Sendo i a taxa de juros (neste caso, como a taxa é numericamente b) 2,43 m³ d) 4,05 m³
igual ao capital, o bom senso nos diz que a taxa de juros é de i
Cit x y z
pontos percentuais, ou seja, i%), temos que J  C  i%  t  . 02. Os números reais x, y e z são tais que = = . Sabendo que
2 100 xyz = 480, o valor de 2x² + y – z é 2 5 6
Ct
Como i = C, obtemos que J = . Para que J = C², devemos ter
100 a) 42 c) 30 e) 22
C2t 2
 C  t  100. b) 36 d) 26
100
03. Os números reais m e n são tais que a razão entre m + n e
1
3m – 2n, nessa ordem, vale . A razão entre os números m + 2n e
2m + n, nessa ordem, vale 4
3 2 6
a) . c) . e) .
7 3 5
8 4
b) . d) .
13 11

17

PM_BOOK16 - MAT.indb 17 25/11/2022 19:06:25


RAZÃO E PROPORÇÃO

04. (FUVEST) A tabela informa a extensão territorial e a população de corredor parte de B, chega a A e volta para B. Os corredores cruzam-se
cada uma das regiões do Brasil, segundo o IBGE. duas vezes, a primeira vez a 800 metros de A e a segunda vez a 500
metros de B. O comprimento da pista, em metros, é
EXTENSÃO POPULAÇÃO
REGIÃO a) 1.000 c) 1.600 e) 2.100
TERRITORIAL (km2) (HABITANTES)
b) 1.300 d) 1.900
Centro-Oeste 1.606.371 14.058.094
Nordeste 1.554.257 53.081.950 09. (EN) Considere uma fração cuja soma de seus termos é 7.
Somando-se três unidades ao seu numerador e retirando-se três
Norte 3.853.327 15.864.454
unidades de seu denominador, obtém-se a fração inversa da primeira.
Sudeste 924.511 80.364.410 Qual é o denominador da nova fração?
Sul 576.409 27.386.891 a) 1 c) 3 e) 5
b) 2 d) 4
IBGE: Sinopse do Censo Demográfico 2010 e Brasil em números, 2011.

Sabendo que a extensão territorial do Brasil é de, aproximadamente, 10. (EN) Em um certo país, o imposto de renda anual é taxado da
8,5 milhões de km2, é correto afirmar que a maneira a seguir:
a) densidade demográfica da região sudeste é de, aproximadamente, 1º) se a renda bruta anual é menor que R$ 10.000,00 não é taxado;
87 habitantes por km2. 2º) se a renda bruta anual é maior ou igual a R$ 10.000,00 e menor
b) região norte corresponde a cerca de 30% do território nacional. que R$ 20.000,00 á taxado em 10%;
c) região sul é a que tem a maior densidade demográfica. 3º) se a renda bruta anual é maior ou igual a R$ 20.000,00 é taxado
em 20%.
d) região centro-oeste corresponde a cerca de 40% do território
nacional. A pessoa que ganhou no ano R$ 17.370,00 após ser descontado o
imposto,tem duas possibilidades para o rendimento bruto. A diferença
e) densidade demográfica da região nordeste é de, aproximadamente,
entre esses rendimentos é
20 habitantes por km2.
a) R$ 17.370,40 c) R$ 3.840,50 e) R$ 1.206,60
05. (FUVEST) Um automóvel, modelo flex, consome 34 litros de b) R$ 15.410,40 d) R$ 2.412,50
gasolina para percorrer 374 Km. Quando se opta pelo uso do álcool,
o automóvel consome 37 litros deste combustível para percorrer
259 Km. Suponha que um litro de gasolina custe R$ 2,20. Qual deve EXERCÍCIOS DE
ser o preço do litro do álcool para que o custo do quilômetro rodado
por esse automóvel, usando somente gasolina ou somente álcool
como combustível, seja o mesmo?
TREINAMENTO
a) R$ 1,00 c) R$ 1,20 e) R$ 1,40
b) R$ 1,10 d) R$ 1,30 01. Dois caminhões-tanque carregam o mesmo volume de misturas
de álcool e gasolina. A mistura de um contém 3% de álcool e a do
06. (FUVEST) Maria quer comprar uma TV que está sendo vendida por outro, 5% de álcool. Os dois caminhões descarregam sua carga em
R$ 1.500,00 à vista ou em 3 parcelas mensais sem juros de R$ 500,00. um reservatório que estava vazio. A razão do volume de álcool para
O dinheiro que Maria reservou para essa compra não é suficiente o de gasolina na mistura formada no reservatório, após os caminhões
para pagar à vista, mas descobriu que o banco oferece uma aplicação terem descarregado, é
financeira que rende 1% ao mês. Após fazer os cálculos, Maria 1 1 1
a) c) e)
concluiu que, se pagar a primeira parcela e, no mesmo dia, aplicar a 25 16 8
quantia restante, conseguirá pagar as duas parcelas que faltam sem 1
1 d)
ter que colocar nem tirar um centavo sequer. b)
24 12
Quanto Maria reservou para essa compra, em reais?
a) 1.450,20 c) 1.485,20 e) 1.490,20 02. “36 está para 4 + x, assim como 5 + x está para 2”. Determine
b) 1.480,20 d) 1.495,20 o valor positivo de x, que torna verdadeira a sentença entre aspas, e
calcule x.
07. (AFA) Três carros, a, b e c, com diferentes taxas de consumo de a) 1 c) 27 e) 3125
combustível, percorrerão, cada um, 600 km por um mesmo caminho. b) 4 d) 256
No ponto de partida, os três estão com tanque cheio.
1 03. Sejam a, b, c e k números reais diferentes de zero satisfazendo
Após terem percorrido, cada um, do total previsto, os carros b e c
5 a b c
foram abastecidos completando novamente seus tanques e gastaram, =
as relações k = = . Qual é o número de possíveis
b+c c +a a+b
juntos, R$ 66,00. valores que k pode assumir?
Ao final dos 600 km, os três carros foram abastecidos, completando a) 0 c) 2 e) 4
seus tanques, e, nesse abastecimento, juntos, gastaram R$ 384,00.
b) 1 d) 3
Considerando o preço do litro do combustível usado pelos três carros
a R$ 3,00, a distância que o carro a percorre, em média, com um litro
04. Dois sócios x e y que montaram uma firma e que têm retirada
de combustível é
mensal de acordo com o capital inicial de cada um, combinaram que
a) 12 km b) 15 km c) 16 km d) 18 km a soma das retiradas totalizaria R$ 5.000,00. Após 6 meses, y passou
a receber por mês mais 15% por ter adquirido algumas cotas de x
08. (FUVEST) Dois atletas correm com velocidades constantes em uma 1
que, consequentemente, passou a receber a menos. Sabendo-se
pista retilínea, partindo simultaneamente de extremos opostos, A e 10
B. Um dos corredores parte de A, chega a B e volta para A. O outro que, mesmo após a mudança, o total da retirada mensal permaneceu

18

PM_BOOK16 - MAT.indb 18 25/11/2022 19:06:26


RAZÃO E PROPORÇÃO

1 310 dias. O tempo necessário para cada vaca comer a parte que lhe
e que x sempre economizou do que recebia, enquanto y sempre corresponde é
12
economizou 12,5%, é INCORRETO afirmar que a) 45 e 60 dias c) 48 e 56 dias e) 42 e 60 dias
a) a economia mensal de ambos era a mesma nos primeiros 6 meses. b) 42 e 56 dias d) 45 e 50 dias
b) x passou a receber menos de R$ 2.800,00 após 6 meses.
11. Uma empresa foi contratada para executar serviço de pintura no
c) a diferença entre as duas retiradas caiu para 40% com a mudança.
alojamento dos alunos do 1° ano CPCAR. O prazo estabelecido no
d) a economia mensal de x diminuiu R$ 30,00 com a alteração das contrato para a conclusão do serviço foi de 10 dias. O serviço começou
retiradas. a ser executado por uma equipe de 6 funcionários da empresa, cada
um trabalhando 6 horas por dia. Ao final do 8°dia de serviço somente
05. Uma mãe dividiu a quantia de R$ 2 100,00 entre seus três filhos de 5/3 do serviço de pintura havia sido executado. Para terminar o serviço
3, 5 e 6 anos. A divisão foi feita em partes inversamente proporcionais dentro do prazo, a equipe de serviço recebeu mais 2 funcionários e
às idades de cada um. Dessa forma, é verdade que todos passaram a trabalhar 9 horas por dia. Com isso a produtividade
a) o filho mais novo recebeu 100 reais a mais que a soma dos valores da equipe duplicou. A nova equipe, para concluir o trabalho, gastou
recebidos pelos outros dois filhos. mais de 1 dia, porém menos de 2 dias. Se h representa o número de
horas que cada funcionário da nova equipe trabalhou no 10° dia de
b) o filho mais velho recebeu 20% a menos que o filho do meio.
trabalho, então h é um número compreendido entre
c) a quantia que o filho do meio recebeu é 40% do que recebeu o
a) 0 e 2 b) 2 e 4 c) 4e6 d) 6 e 8
mais novo.
d) se a divisão fosse feita em partes iguais, o filho mais velho teria 12. A quantidade de suco existente na cantina de uma escola é
sua parte acrescida de 40% em relação ao que realmente recebeu. suficiente para atender o consumo de 30 crianças durante 30 dias.
Sabe-se que cada criança consome, por dia, a mesma quantidade de
06. (IFAL 2017) Um pai deseja dividir R$ 800,00 com seus dois filhos suco que qualquer outra criança desta escola. Passados 18 dias, 6
de 10 anos e de 15 anos, em quantias diretamente proporcionais às crianças tiveram que se ausentar desta escola por motivo de saúde.
suas idades. Quanto recebem, respectivamente, o filho mais novo e o É correto afirmar que, se não houver mais ausências nem retornos, a
filho mais velho? quantidade de suco restante atenderá o grupo remanescente por um
a) R$ 100,00 e R$ 700,00. d) R$ 430,00 e R$ 370,00. período de tempo que somado aos 18 dias já passados, ultrapassa os
b) R$ 210,00 e R$ 590,00. e) R$ 540,00 e R$ 260,00. 30 dias inicialmente previstos em

c) R$ 320,00 e R$ 480,00. a) 10% c) 5%


b) 20% d) 15%
07. (CN 2016) Uma placa será confeccionada de modo que o emblema
da empresa seja feito de um metal que custa R$ 5,00 o centímetro 13. Dois aviões, respeitando as normas de segurança, voam em
quadrado. O emblema consiste em três figuras planas semelhantes linha reta no mesmo sentido, com o objetivo de chegar à cidade D.
que lembram três árvores. Para as bases “árvores”, constroem-se O primeiro, com uma velocidade média de 150000 m/h, passa pela
segmentos de reta proporcionais a 3, 4 e 5. Se o custo da maior árvore cidade A, às 10 horas da manhã de certo dia. O segundo, com uma
do emblema ficou em R$ 800,00, qual o valor, em reais de todo o velocidade média de 2 km/min, passa pela cidade B, no mesmo
emblema? instante em que o primeiro avião passa por A. A cidade B está situada
a) 1600 c) 1200 e) 1020 entre A e D e entre as cidades B e D existe uma torre C, alinhada
com as três cidades. Sabe-se que as cidades A, B e D, bem como a
b) 1500 d) 1120 região onde está localizada a torre C, possuem mesmo fuso horário e
que as velocidades médias dos dois aviões se mantiveram constantes
08. (CN 2016) Adão, Beto e Caio uniram-se num mesmo investimento durante todo o percurso. Sabe-se, também, que a distância entre C
e combinaram que, em janeiro de cada ano, repartiriam o lucro obtido e B é 12000 dam e entre A e C é 3240 hm. Se os aviões chegam à
em partes diretamente proporcionais ao tempo de investimento e ao cidade D, ao mesmo tempo, é correto afirmar que isso ocorreu entre
valor investido. Adão investiu R$ 10.000,00 há nove meses; Beto R$
a) 16h e 20 min e 16 h e 30 min
15.000,00 há oito meses e Caio R$ 12.000,00 há cinco meses. Se o
lucro a ser repartido é de R$ 54.000,00, o maior recebimento será de b) 16 h e 30 min e 16 h e 40 min
a) R$ 10.000,00 d) R$ 18.000,00 c) 16 h e 40 min e 16 h e 50 min
b) R$ 12.000,00 e) R$ 24.000,00 d) 16 h e 50 min e 17 h
c) R$ 15.000,00
14. Um tear eletrônico, trabalhando 5 horas por dia, produz 1200
peças em 3 dias. O número de horas que deverá trabalhar no 8º dia
09. Para 4 números a, b, c e d inteiros e positivos onde 1 < a < b < c
para produzir 1840 peças, se o regime de trabalho fosse 3 horas
a c a c 40
< d e = e + = . O valor mínimo de (d – a) é diárias, seria um número do intervalo
b d b d bc
a) [2, 3[ c) [4, 6[
a) 5 c) 7 e) 12
b) [3, 4[ d) [1, 2[
b) 6 d) 8
15. Uma fábrica recebeu uma encomenda de 50 aviões. A fábrica
10. Um fazendeiro tem 2 vacas e 2 bois em um pasto que comem, montou os aviões em 5 dias, utilizando 6 robôs de mesmo rendimento,
cada um, a mesma quantidade de pasto em um mesmo tempo. A que trabalharam 8 horas por dia. Uma nova encomenda foi feita,
primeira vaca está amarrada em um poste por uma corda de 3m de desta vez 60 aviões. Nessa ocasião, um dos robôs não participou da
comprimento, a segunda vaca está amarrada a um poste por uma montagem. Para atender o cliente, a fábrica trabalhou 12 horas por
corda de m de comprimento. Da mesma forma o primeiro boi está dia. O número de dias necessários para que a fábrica entregasse as
amarrado a outro poste por uma corda de 4m de comprimento duas encomendas foi
e o segundo boi amarrado a outro poste por uma corda de 5m de
comprimento. Se a soma dos tempos que são aplicados a estes 4 a) exatamente 10 c) entre 9 e 10
animais para comerem o pasto que está ao alcance de cada um é de b) mais de 10 d) menos de 9

19

PM_BOOK16 - MAT.indb 19 25/11/2022 19:06:26


RAZÃO E PROPORÇÃO

16. Uma indústria tem um reservatório de água com capacidade para 22. Em 01/03/95, um artigo que custava R$ 250,00 teve seu
900 m³. Quando há necessidade de limpeza do reservatório, toda preço diminuído em p% do seu valor. Em 01/04/95, o novo preço
a água precisa ser escoada. O escoamento da água é feito por seis foi novamente diminuído em p% do seu valor, passando a custar
ralos, e dura 6 horas quando o reservatório está cheio. Esta indústria R$ 211,60. O preço desse artigo em 31/03/95 era:
construirá um novo reservatório, com capacidade de 500 m³, cujo a) R$ 225,80 d) R$ 230,00
escoamento da água deverá ser realizado em 4 horas, quando o
reservatório estiver cheio. Os ralos utilizados no novo reservatório b) R$ 228,00 e) R$ 230,80
deverão ser idênticos aos do já existente. A quantidade de ralos do c) R$ 228,60
novo reservatório deverá ser igual a
a) 2 c) 5 e) 9 23. (CN) Uma determinada conta a pagar de valor X vence no dia 30
de novembro, mas, se for paga até o dia 30 de setembro, tem 20%
b) 4 d) 8
de desconto sobre X e, se for paga até o dia 31 de outubro, tem 10%
de desconto sobre X. Alguém reservou o valor exato Y para pagar essa
17. (EPCAR) Duas máquinas A e B de modelos diferentes, mantendo conta no dia 30 de setembro, no entanto esqueceu-se de fazê-lo e só
cada qual sua velocidade de produção constante, produzem juntas efetuou esse pagamento no dia 31 de outubro. Qual a porcentagem a
n peças iguais, gastando simultaneamente 2 horas e 40 minutos. A mais sobre Y que terá de pagar?
máquina A funcionando sozinha, mantendo sua velocidade constante,
n a) 10% c) 17,5% e) 25%
produziria, em 2 horas de funcionamento, dessas peças. É correto b) 12,5% d) 20%
2
afirmar que a máquina B, mantendo sua velocidade de produção
n 24. O litro da gasolina comum sofreu, há alguns dias, um aumento
constante, produziria também dessas peças em
2 de 7,7% e passou a custar 2,799 reais. Já o litro do álcool sofreu um
a) 40 minutos. c) 160 minutos. aumento de 15,8%, passando a custar 2,199 reais. Sabendo que o
b) 120 minutos. d) 240 minutos. preço do combustível é sempre cotado em milésimo de real, pode-
se afirmar, aproximadamente, que a diferença de se abastecer um
18. (EPCAR) Uma confecção de roupas foi contratada para carro com 10 litros de gasolina e 5 litros de álcool, antes e depois do
confeccionar os agasalhos de todos os alunos do 1º ano CPCAR para aumento, é de:
o ano de 2014. O prazo que a confecção teve para a execução do a) R$ 2,00 c) R$ 3,00 e) R$ 4,00
trabalho foi de 4 dias. Para isso, o gerente da confecção utilizou 6 b) R$ 2,50 d) R$ 3,50
máquinas tipo α, cada uma trabalhando 6 horas por dia e todas com
a mesma produtividade. Ao final do terceiro dia, o gerente da fábrica 25. Um fabricante de camisetas que pretendia vender seu estoque
verificou que somente 0,333... de 9/4 dos agasalhos estavam prontos. no prazo de 4 meses, mantendo o preço de cada camiseta, obteve o
Sendo assim, substituiu, no início do quarto dia, as máquinas do tipo α seguinte resultado:
por 3 outras do tipo β, cada uma trabalhando 8 horas por dia, e cada
uma delas com o triplo da produtividade de uma máquina tipo α. Se • no primeiro mês, vendeu 10% de seu estoque;
as 3 máquinas tipo β tivessem sido utilizadas desde o início, o serviço • no segundo, 20% do restante das mercadorias; e
teria sido realizado em • no terceiro, 50% do que sobrou.
a) 20 horas. c) 12 horas. Ao ver que sobraram 3.600 camisetas, no quarto mês, o fabricante
b) 16 horas. d) 10 horas. 1
reduziu o preço de cada uma em 33 % , conseguindo assim liquidar
3
19. (EPCAR) Uma prestadora de serviços combina um prazo de 9 todo seu estoque e recebendo R$ 21.600,00 pelas vendas deste mês.
dias, utilizando 12 máquinas, para executar certo trabalho. Ao final É correto afirmar que o fabricante
do quarto dia, 4 máquinas estragam, não sendo substituídas e não a) arrecadaria a mesma importância total, durante os 4 meses, se
havendo interrupção do trabalho. As máquinas levam 3 dias para cada camiseta fosse vendida por x reais, x ∈ [7, 8].
serem consertadas, retornando ao trabalho no dia seguinte. Para b) tinha um estoque que superava 834 dúzias de camisetas.
que seja cumprido o prazo combinado no início, a prestadora coloca,
além das 12 máquinas, mais x máquinas iguais às primeiras. É correto c) no terceiro mês, vendeu uma quantidade de camisetas 200% a
afirmar que x é igual a mais que no segundo mês.
a) 3 c) 5 d) no primeiro mês, recebeu mais de R$ 9.000,00.
b) 4 d) 6
26. Um reservatório deve ser cheio completamente com uma mistura
de 76% de gasolina e de 24% de álcool. A torneira que fornece
20. (CFTMG) Para executar uma reforma em uma loja, foram gasolina enche este tanque, sozinha, em 4 horas, e a torneira que
contratados n operários. O mestre de obras argumentou: “para fornece álcool enche este tanque, sozinha em 6 horas. Abrindo-se
entregar a obra 2 dias antes do prazo previsto, seria necessário contratar essas torneiras no mesmo instante, quanto tempo a mais uma delas
mais 3 operários; se, entretanto, 2 operários fossem dispensados a deve ser deixada aberta, depois de a outra ser fechada, para que as
obra atrasaria em 2 dias”. Considerando que os operários trabalhem condições estabelecidas sejam satisfeitas?
da mesma forma, o número n de operários contratados foi
a) 1 h 30 min. d) 1 h 48 min.
a) 6 c) 18
b) 1 h 36 min. e) 1 h 54 min.
b) 12 d) 24
c) 1 h 42 min.
21. Tem-se 500 mL de soro glicosado a 5%. Quando se acrescentam
dez ampolas de 10 mL cada de glicose a 23%, a concentração do 27. Uma mercadoria foi comprada por R$ 20,00. Para que haja um
volume final do soro glicosado será: lucro de 60% sobre o preço de venda, esta mercadoria deve ser
vendida por:
a) 6% d) 7,3%
a) R$ 32,00 d) R$ 45,00
b) 6,3% e) 8%
b) R$ 50,00 e) R$ 58,00
c) 7%
c) R$ 48,00

20

PM_BOOK16 - MAT.indb 20 25/11/2022 19:06:26


RAZÃO E PROPORÇÃO

28. Uma pequena fábrica de tubos de plástico calcula a sua receita 36. Aplicando 1 real a juros compostos durante 12 anos, obtém-se
em milhares de reais, através da função R(x) = 3,8x, onde x representa um montante de 64 reais. Usando a tabela abaixo, pode-se dizer que
o número de tubos vendidos. Sabendo que o custo para a produção a taxa anual de juros é:
do mesmo número de tubos é 40% da receita mais R$ 570,00. Nessas
condições, para evitar prejuízo, o número mínimo de tubos de plástico x 1 2 3 4 5 6
que devem ser produzidos e vendidos pertence ao intervalo:
x 1 1,4142 1,7321 2 2,2361 2,4495
a) [240; 248]. c) [252; 258].
b) [248; 260]. d) [255; 260]. a) 41,42% c) 100% e) 144,95%
b) 73,21% d) 123,61%
29. Uma chácara foi vendida por R$ 2.550.000,00, com prejuízo
de 15% em relação ao seu preço de compra. Portanto, o preço de
37. Para adquirir uma certa mercadoria, são oferecidos ao consumidor
compra da chácara, em reais, foi:
três planos de pagamento possíveis:
a) 2.167.500,00 c) 3.000.000,00 e) 4.717.500,00
I. Pagamento no ato da compra, com 15% de desconto à vista.
b) 2.932.500,00 d) 3.825.000,00
II. Três parcelas mensais fixas iguais, com pagamento da primeira no
ato da compra.
30. Uma fábrica vende por mês 30 camisas ao preço de 25 reais cada.
O custo total de cada camisa para a fábrica é de R$10,00. O gerente III. Seis parcelas mensais fixas iguais, com juros simples de 2% ao
da fábrica observou que, a cada redução de R$0,50 no preço unitário mês, com pagamento da primeira para 30 dias após a compra.
de cada camisa, são vendidas 5 camisas a mais. Considerando essas Se cada uma das parcelas do plano II é de x reais, é CORRETO afirmar
observações, se a fábrica vender 150 camisas, o lucro obtido na venda que
de cada camisa é de y%. O número de divisores positivos de y é a) no plano III, cada prestação é de 0,5x reais.
a) 6 b) 8 c) 10 d) 12 b) no plano I, o valor pago pela mercadoria é de 2,75x reais.
c) a diferença entre o valor pago pela mercadoria nos planos I e III é
31. Dois capitais são empregados a uma mesma taxa de 3% ao ano. de 0,81x reais.
A soma dos capitais é igual a R$ 50.000,00. Cada capital produz R$
600,00 de juros. O primeiro permaneceu empregado 4 meses mais d) a diferença entre o valor pago pela mercadoria nos planos II e III
que o segundo. O segundo capital foi empregado durante: foi de 0,3x reais.
a) 6 meses c) 10 meses e) 3 anos
38. Ao desfazer uma sociedade, dois sócios A e B fizeram a retirada
b) 8 meses d) 2 anos de suas partes que eram diretamente proporcionais a 1 e 3. O sócio
A aplicou, então, o valor de sua retirada à taxa de 50% ao ano. Já o
32. Uma aplicação do mercado financeiro que rende 0,3% ao dia, 2
exige um mínimo de R$ 50.000 ,00 para ser efetuada. Uma pessoa sócio B aplicou a sua parte à taxa de 25% ao ano e do montante
3
que dispõe de R$45.000,00 toma R$ 5.000,00 a taxa de 1% dia para que recebeu após 12 meses foi igual a 150.000 reais. Pode-se afirmar
fazer tal aplicação. Durante quantos dias, no mínimo, deverá aplicar que
para pagar o empréstimo e continuar aplicando? a) a diferença entre os rendimentos dos sócios A e B, após 12 meses,
a) 40 c) 45 e) 50 é, em milhares de reais, um número do intervalo [8,15].
b) 43 d) 47 b) a soma dos capitais retirados por A e B é igual ao montante que o
sócio B conseguiu após 12 meses.
33. A chegada da televisão no Brasil facilitou o acesso à informação. c) o rendimento obtido pelo sócio A é igual a 30% do rendimento
Com o avanço da tecnologia, os aparelhos estão cada dia mais do sócio B.
modernos e consequentemente mais caros.
d) o capital retirado pelo sócio A e o rendimento conseguido pelo
Um consumidor deseja adquirir uma televisão com tecnologia de sócio B são valores iguais.
última geração. Enquanto aguarda o preço da televisão baixar, ele
aplica o capital disponível de R$ 3.000,00 a juros simples de 0,8% 39. (UFU) Um financiamento de R$ 10.000 foi contratado a uma
ao mês em uma instituição financeira, por um período de 18 meses. taxa de juros (compostos) de 3% ao mês. Ele será liquidado em duas
O montante, ao final desse período, é igual a parcelas iguais, a primeira vencendo em 60 dias e a segunda em 90
a) R$ 7.320,00 d) R$ 3.432,00 dias após a efetivação do contrato. O valor de cada parcela desse
financiamento é, aproximadamente, igual a
b) R$ 5.400,00 e) R$ 3.240,00
Dados:
c) R$ 4.320,00
(1 + 0,03)¹ = 1,03 (1 + 0,03)² = 1,0609 (1 + 0,03)³ = 1,0927
34. Um capital é empregado à taxa de 8% a.a. No fim de quanto
1 1 1
tempo os juros simples produzidos ficam iguais a 60% do capital? = 0,9709 = 0,9426 = 0,9151
(1 + 0,03)1 (1 + 0,03)2 (1 + 0,03)3
a) 5 anos e 4 meses d) 6 anos e 4 meses
b) 7 anos e 6 meses e) 7 anos e 3 meses a) R$ 5226,00. c) R$ 5387,00.
c) 8 anos e 2 meses b) R$ 5383,00. d) R$ 5282,00.

35. Um capital foi empregado da seguinte maneira: seus dois quintos 40. (FGV) César aplicou R$ 10.000,00 num fundo de investimentos
rendendo 40% ao ano e a parte restante rendendo 30% ao ano. que rende juros compostos a uma certa taxa de juro anual positiva i.
No fim de um ano, a diferença entre os juros das duas partes foi de Após um ano, ele saca desse fundo R$ 7.000,00 e deixa o restante
R$ 2.700,00. Qual era o capital inicial? aplicado por mais um ano, quando verifica que o saldo é R$ 6.000,00.
a) R$ 94.500,00 d) R$ 120.000,00 O valor de (4i – i)² é:
b) R$ 27.000,00 e) R$ 135.000,00 a) 0,01 c) 0,03 e) 0,05
c) R$ 140.000,00 b) 0,02 d) 0,04

21

PM_BOOK16 - MAT.indb 21 25/11/2022 19:06:27


RAZÃO E PROPORÇÃO

EXERCÍCIOS DE estava atrasado. Acertei meu relógio pelo relógio de ponto e, para

COMBATE compensar meu atraso, pretendo cumprir os 3/4 de minha jornada e


sair para almoçar reduzindo o tempo de meu intervalo de almoço em
1/5. Imediatamente retornarei para o trabalho e sairei no meu horário
habitual.” Considerando que o relógio de ponto estivesse certo e em
perfeito funcionamento, é correto afirmar que, nesse dia, Gabriel,
01. (CN 2009) O combustível A é composto de uma mistura de
com sua pretensão:
20% de álcool e 80% de gasolina. O combustível B é constituído
exclusivamente de álcool. Um motorista quer encher completamente a) sairá para o almoço antes de 12 horas e 23 minutos.
o tanque do seu carro com 50% de álcool e 50% de gasolina. Para b) retornará após o intervalo de almoço, exatamente, às 13 horas e
alcançar o seu objetivo, colocou x litros de A e y litros de B. A razão 50 minutos.
x/y é dada por:
c) cumprirá sua jornada diária na íntegra e ainda sobrarão dois
a) 5/3 c) 2/5 e) 3/2 minutos.
b) 3/5 d) 5/2 d) ficará devendo 1/160 de sua jornada diária.

02. (AFA 2013) Um tanque com capacidade de 300 litros de água 07. (AFA 2007) Um fabricante de camisetas que pretendia vender seu
possui duas torneira: I e II. A torneira I despeja água no tanque a uma estoque no prazo de 4 meses, mantendo o preço de cada camiseta,
vazão de 2 L por minuto. Já a torneira II retira água do tanque a uma obteve o seguinte resultado:
vazão de 1/2 L por minuto. Às 8 h de certo dia, com o tanque vazio, a
• no primeiro mês, vendeu 10% de seu estoque;
torneira I foi aberta e, após 15 minutos, foi fechada. Às 9 h e 30 min
as duas torneiras foram abertas, e assim permaneceram até 11 h e 30 • no segundo, 20% do restante das mercadorias;
min. Neste horário a torneira II é fechada, mas a torneira I permanece • no terceiro, 50% do que sobrou.
aberta até o momento em que a água atinge a capacidade do tanque. Ao ver que sobraram 3.600 camisetas, no quarto mês, o fabricante
Este momento ocorre às: 1
reduziu o preço de cada uma em 33 % , conseguindo assim liquidar
a) 12 h e 10 min c) 12 h e 20 min 3
b) 12 h e 15 min d) 12 h e 25 min todo seu estoque e recebendo R$ 21.600,00 pelas vendas deste mês.
É correto afirmar que o fabricante:
03. (AFA 2009) Sr. Osvaldo possui certa quantia com a qual deseja a) arrecadaria a mesma importância total, durante os 4 meses, se
adquirir um eletrodoméstico. Caso a loja ofereça um desconto de cada camiseta fosse vendida por x reais, x ∈ [7,8]
40%, ainda lhe faltarão 1000 reais. Se o Sr. Osvaldo aplicar sua quantia b) tinha um estoque que superava 834 dúzias de camisetas.
a juros (simples) de 50% ao mês, ajunta, em três meses, o montante
correspondente ao valor do eletrodoméstico sem o desconto. Assim, c) no terceiro mês, vendeu uma quantidade de camisetas 200% a
o valor do eletrodoméstico e da quantia que o Sr. Osvaldo possui mais que no segundo mês.
somam, em reais: d) no primeiro mês, recebeu mais de R$ 9.000,00
a) 4000 c) 7000
08. (AFA 2007) Considere a tabela para cálculo do imposto de renda
b) 5000 d) 8000
a ser pago à Receita Federal no ano de 2007 – ano base 2006 (valores
arredondados para facilitar os cálculos).
04. (EFOMM 2008) Uma empresa utiliza mão de obra terceirizada
para carregar os contêineres. A equipe A carrega completamente um RENDIMENTO PARA BASE ALÍQUOTA PARCELA A
contêiner em 20 horas; a B, em 23 horas; e a C, estando carregado, DE CÁLCULO (R$) (%) DEDUZIR (R$)
o esvazia em 26 horas. Se trabalhassem as três equipes juntas, o
tempo aproximado que as três firmas juntas levariam para esvaziar um até 14.999,99 Isento –
contêiner completamente cheio é:
de 15.000,00 a 30.000,00 15 2.250,00
a) 6 horas e 25 min. d) 8 horas e 40 min.
b) 6 horas e 30 min. e) 9 horas e 10 min. acima de 30.000,00 27,5 6.000,00
c) 7 horas e 35 min.
Para se conhecer o rendimento para base de cálculo, deve-se
subtrair do rendimento bruto todas as deduções a que se tem
05. (EFOMM 2013) O litro da gasolina comum sofreu, há alguns dias,
direito. Esse rendimento para base de cálculo é multiplicado pela
um aumento de 7,7% e passou a custar 2,799 reais. Já o litro do
alíquota correspondente. Em seguida, subtrai-se a parcela a deduzir
álcool sofreu um aumento de 15,8%, passando a custar 2,199 reais.
correspondente, de acordo com a tabela acima, obtendo-se assim o
Sabendo que o preço do combustível é sempre cotado em milésimo
valor do imposto de renda a ser pago. Um trabalhador, cujo rendimento
de real, pode-se afirmar, aproximadamente, que a diferença de se
bruto foi de R$ 50.000,00 teve direito às seguintes deduções:
abastecer um carro com 10 litros de gasolina e 5 litros de álcool, antes
R$ 4.400,00 com o total de gastos em educação, R$ 5.000,00 com o
e depois do aumento, é de:
total pago à Previdência e R$ 1.500,00 por dependente.
a) R$2,00 c) R$3,00 e) R$4,00
Nessas condições, sabendo-se que o valor do imposto pago por
b) R$2,50 d) R$3,50 esse trabalhador, no ano de 2007, foi de R$ 3.515,00, o número de
dependentes considerado foi:
06. (AFA 2009) Perguntaram a Gabriel qual era seu horário de a) 2 b) 3 c) 4 d) 6
trabalho e ele respondeu: “Habitualmente começo às 6 horas da
manhã minha jornada de trabalho que é de 8 horas diárias, dividida
09. (AFA 2007) Apliquei meu capital da seguinte maneira: 30% em
em dois expedientes. Cumpro no primeiro expediente 3/4 dessa
caderneta de poupança, 40% em letras de câmbio e o restante em
jornada, tenho um intervalo de almoço de 1 hora e 45 minutos
ações. Na 1ª aplicação, lucrei 20%; na 2ª, lucrei 30% e na 3ª perdi
e retorno para cumprir o tempo que falta, ou seja, o segundo
25%. Se o resultado final corresponde a um lucro de x% sobre o
expediente. Hoje, excepcionalmente, quando cheguei, o relógio de
capital aplicado, então x é igual a:
ponto registrou um horário tal que o tempo transcorrido do dia era
igual aos 4/11 do tempo restante do dia e eu fui, então, alertado que a) 7,5 b) 10,5 c) 15 d) 17

22

PM_BOOK16 - MAT.indb 22 25/11/2022 19:06:28


RAZÃO E PROPORÇÃO

10. (EFOMM 2012) De todos os empregados de uma empresa de


navegação, 31% optaram por um plano de assistência odontológica.
A firma tem a matriz na capital e somente duas filiais, uma em Macaé
3 (EPCAR) Uma empresa de artigos de perfumaria oferece
a seguinte modalidade na negociação de seus produtos:
“Qualquer pessoa que se cadastre como vendedor tem
e a outra em Piraí. Sabe-se que 50% dos empregados trabalham na autonomia para estabelecer o preço de venda e recebe uma
matriz, 20% dos empregados trabalham na filial Macaé, 30% dos comissão sobre o lucro que conseguir.”
empregados da capital optaram pelo plano de assistência odontológica
e que 35% dos empregados da filial de Macaé também fizeram tal No mês de fevereiro, um vendedor recebeu uma caixa com vários
opção. Qual é, então, a porcentagem dos empregados da filial de Piraí frascos iguais de um perfume que era lançamento para o Dia
que optaram pelo plano? das Mães, e teve duas semanas de prazo para efetuar as vendas
e esgotar o estoque que estava sob sua responsabilidade.
a) 40%
1
b) 35% Ao final da 1ª semana, verificou que restava apenas do
4
c) 30% estoque que recebera, sendo que, assim, ele já havia apurado
d) 25% 39
do valor que a empresa investira na fabricação destes
40
e) 15% perfumes.
Na semana seguinte ele vendeu o restante dos frascos
conservando o mesmo preço de venda.
Sabe-se que o vendedor recebe uma comissão de 45% sobre o

DESAFIO PRO lucro que obtiver.


Neste caso, cada R$ 100,00 que esse vendedor receber com suas
vendas lhe dará direito a uma comissão cujo valor, em reais, está
entre

1 (EPCAR) Uma pessoa aplicou 60.000 reais durante o ano de


2018. Parte desse dinheiro aplicou no investimento P e a
outra parte, no investimento Q.
a) 8 e 10.
b) 10 e 12.
No final de 2018, retirou o dinheiro das duas aplicações e c) 12 e 14.
verificou que, somando os dois valores, não obteve lucro nem d) 14 e 16.
prejuízo.
O investimento P rendeu 10%, mas, sobre o rendimento, foi
cobrada uma taxa de 10%; já o investimento Q deu prejuízo
de 12,6%.
4 (EPCAR) Até a primeira quinzena do mês de março de 2017,
o combustível comercializado nos postos de nosso país era
uma mistura de 1 parte de etanol para 3 partes de gasolina.
Com base nessas informações, pode-se afirmar que Considere esse combustível e um outro que apresenta a mistura
de 4 partes de etanol para 9 partes de gasolina.
a) a razão entre o valor aplicado em Q e o valor aplicado em
1 Juntando-se volumes iguais dos dois combustíveis, a nova
Pé . relação de etanol para gasolina, nesta ordem, será
4
b) com o que essa pessoa recebeu do investimento Q, no final 5
a)
de 2018, seria possível comprar um carro de 23.000 reais. 9
c) a diferença entre o maior e o menor valor aplicados, em 5
b)
reais, é maior que 11.000 reais. 12
d) essa pessoa aplicou mais de 32.000 reais no investimento P. 29
c)
75

2 (EPCAR) As turmas FOX e GOLF do CPCAR 2018, que


possuem 30 e 20 alunos, respectivamente, combinaram
viajar para uma casa de praia num feriado que aconteceu no
d)
2
5
mês de junho de 2018.
Antes de viajar, decidiram dividir todas as despesas entre as 5 (EPCAR) O dono de uma loja de produtos seminovos
adquiriu, parceladamente, dois eletrodomésticos.
turmas de forma diretamente proporcional ao número de 2
Após pagar do valor dessa compra, quando ainda devia
alunos de cada turma. 5
Pagaram todas as despesas, mas não pagaram de forma R$ 600,00, resolveu revendê-los.
proporcional. A turma FOX pagou 12.000 reais e a turma GOLF Com a venda de um dos eletrodomésticos, ele conseguiu um lucro
pagou 10.500 reais. de 20% sobre o custo, mas a venda do outro eletrodoméstico
a) Tendo como base o que as turmas haviam combinado em representou um prejuízo de 10% sobre o custo. Com o valor
relação às despesas da viagem, é correto afirmar que total apurado na revenda, ele pôde liquidar seu débito existente
e ainda lhe sobrou a quantia de R$ 525,00.
b) a despesa correta da turma GOLF seria mais de 10.000 reais.
A razão entre o preço de custo do eletrodoméstico mais caro e
c) a turma FOX pagou a menos 10% do que deveria ter pago.
o preço de custo do eletrodoméstico mais barato, nessa ordem,
d) o que a turma GOLF pagou a mais é um valor maior que é equivalente a
1.800 reais.
a) 5
e) a turma FOX deveria ter pago mais de 10.000 reais.
b) 4
c) 3
d) 2

23

PM_BOOK16 - MAT.indb 23 25/11/2022 19:06:28


RAZÃO E PROPORÇÃO

GABARITO

EXERCÍCIOS DE FIXAÇÃO
01. A 04. A 07. B 10. D
02. C 05. E 08. D
03. D 06. C 09. B
EXERCÍCIOS DE TREINAMENTO
01. B 11. B 21. E 31. B
02. B 12. A 22. D 32. E
03. C 13. C 23. B 33. D
04. D 14. A 24. D 34. B
05. D 15. C 25. A 35. E
06. C 16. C 26. B 36. A
07. A 17. D 27. B 37. C
08. E 18. B 28. B 38. A
09. E 19. D 29. C 39. B
10. A 20. B 30. B 40. D
EXERCÍCIOS DE COMBATE
01. A 04. C 07. A 10. C
02. B 05. D 08. C
03. C 06. D 09. B
DESAFIO PRO
01. D 03. C 05. C
02. D 04. C

ANOTAÇÕES

24

PM_BOOK16 - MAT.indb 24 25/11/2022 19:06:28


LÓGICA

PROPOSIÇÃO CONECTIVOS
Proposição ou Sentença é toda oração declarativa que pode ser A conjunção (e), denotada por p ∧ q ou p · q, é verdadeira se
classificada em verdadeira ou falsa. Toda proposição apresenta um, e p e q são ambas verdadeiras; se ao menos uma delas for falsa, então
somente um, dos valores lógicos: verdadeira (V) ou falsa (F). p ∧ q é falsa.
Exemplo: A disjunção (ou), denotada por p ∨ q ou p + q, é verdadeira se ao
São proposições verdadeiras: 9 ≠ 5 e 2 ∈ . São proposições menos uma das proposições p ou q é verdadeira; se p e q são ambas
falsas: –1 ∈  e 2 > 5. falsas, então p ∨ q é falsa.

TABELA-VERDADE
Tautologia (proposição logicamente verdadeira) é a
proposição que possui valor V (verdadeira) independente dos valores p q p∧q p∨q
lógicos das proposições das quais depende.
Exemplo: V V V V
A frase “O recém-nascido é menino ou menina” é sempre V F F V
verdadeira, pois sendo menino teremos “V ou F”, sendo menina
teremos “F ou V” e em ambos os casos o resultado é verdadeiro. F V F V

F F F F
Contradição (proposição logicamente falsa) é a proposição
que possui valor F (falsa) independente dos valores lógicos das Exemplos:
proposições das quais depende.
(9 > 5) ∧ (0 > 1) é falsa, pois V ∧ F é falsa.
Exemplo:
(9 > 5) ∨ (0 > 1) é verdadeira, pois V ∨ F é verdadeira.
A frase “O recém-nascido é menino e menina” é sempre falsa, pois
sendo menino teremos “V e F”, sendo menina teremos “F e V” e em
ambos os casos o resultado é falso. CONDICIONAIS
O condicional, denotado por p → q é falso somente quando p é
Sentenças abertas são proposições cujo valor lógico depende do verdadeira e q é falsa; caso contrário, p → q é verdadeiro.
valor de uma ou mais variáveis. O bicondicional, denotado por p ↔ q é verdadeiro somente
O conjunto de todos os valores que as variáveis podem assumir quando p e q são ambas verdadeiras ou ambas falsas; se isso não
denomina-se Universo. acontecer p ↔ q é falso.
O subconjunto do universo para o qual a sentença aberta é
verdadeira denomina-se Conjunto-Verdade ou Conjunto-Solução. TABELA-VERDADE
Exemplo: p q p→q p↔q
No universo U =  , o conjunto verdade da sentença x² < 5 é
V = {0,1,2}. V V V V

V F F F
NEGAÇÃO F V V F
A negação de uma proposição p é indicada por p (ou –p) e tem
sempre valor oposto ao de p. F F V V

TABELA-VERDADE A recíproca de p → q é a proposição q → p.


A contrária de p → q é a proposição p → q.
p p
A contrapositiva de p → q é a proposição q → p.
V F
ProBizu
F V
Uma proposição e sua contrapositiva são equivalentes:
Exemplo: p → q ≡ q → p.
A recíproca e a contrária de uma proposição são equivalentes: q
A negação de p: = 9 – 5 (F) é p: 9 ≠ 5 (V). → p ≡ p → q.

25

PM_BOOK16 - MAT.indb 25 25/11/2022 19:06:29


LÓGICA

RELAÇÃO DE IMPLICAÇÃO Exemplo:


Diz-se que p implica q (p ⇒ q) quando na tabela de p e q não Resolver a equação x 2  5x  1  1  2x
ocorre V e F em nenhuma linha, ou seja, quando o condicional
x 2  5x  1  1  2x  x 2  5x  1  2x  1 
p → q é verdadeiro.
Nesse caso pode-se dizer que “p é condição suficiente para q” ou x  5x  1  (2x  1)2  3x 2  9x  0  ( x  0  x  3)
2

que “q é condição necessária para p”. Testando as raízes obtidas verifica-se que x = 0 não é uma raiz
válida. Essa raiz apareceu exatamente quando se elevou ao quadrado
ProBizu ambos os membros da equação, pois, nesse caso, não valia a relação
p⇒q de equivalência, mas somente a implicação. Como se pode notar,
≡ o novo conjunto solução S = {0,3} continha o conjunto solução da
“p é condição suficiente para q” equação inicial S = {3}.
≡ Outra maneira de resolver a equação utilizando somente
“q é condição necessária para p” equivalências seria:
x 2  5x  1  1  2x  x 2  5x  1  2x  1 
Uma condição é dita necessária quando ela precisa acontecer para
o resultado ser verdadeiro, mas não garante que ele seja verdadeiro. ( x† 5x  1  (2x  1)2  (2x  1  0)) 
Exemplo:  2 1   1
 3x  9x  0  x     ( x  0  x  3)   x     x  3
x ≥ 0 é uma condição necessária de x > 1.  2   2 
Um teorema é uma implicação da forma (hipótese ⇒ tese).
Assim, demonstrar um teorema significa mostrar que, sempre que a ÁLGEBRA DAS PROPOSIÇÕES
hipótese for verdadeira, a tese também será verdadeira.
Propriedade Idempotente:
Exemplo:
p∧p⇔p
x = 2 ⇒ x² = 4 (note que a volta não é necessariamente verdadeira)
p∨p⇔p
A proposição (p ⇒ q) será falsa se existir um objeto matemático Propriedade Comutativa:
que satisfaça a hipótese p e não satisfaça a conclusão q (VF). Esse
objeto é chamado contraexemplo para a proposição (p ⇒ q). p∧q⇔q∧p
Por outro lado, um objeto matemático que satisfaça a hipótese p p∨q⇔q∨p
e a tese q se diz um exemplo para a proposição(p ⇒ q). Propriedade Associativa:
(p ∧ q) ∧ r ⇔ p ∧ (q ∧ r)
ProBizu
(p ∨ q) ∨ r ⇔ p ∨ (q ∨ r)
Um único contra exemplo é suficiente para provar que uma Distributividade:
proposição é falsa, entretanto enumerar exemplos não garante
que a proposição seja verdadeira. p ∧ (q ∨ r) ≡ (p ∧ q) ∨ (p ∧ r)
p ∨ (q ∧ r) ≡ (p ∨ q) ∧ (p ∨ r)
Absorção:
RELAÇÃO DE EQUIVALÊNCIA p ∧ (p ∨ q) ≡ p
Diz-se que p é equivalente a q (p ⇔ q ou p ≡ q) quando p e p ∨ (p ∧ q) ≡ p
q têm tabelas-verdades iguais, isto é, quando p e q têm sempre o
mesmo valor lógico, ou seja, p ↔ q é verdadeiro. Dupla negação:
Nesse caso diz-se que “p é condição necessária e suficiente para ~ (~p) ≡ p
q” ou que “p se, e somente se, q”. Contrapositiva:
(p ⇒ q) ≡ (q ⇒ p)
ProBizu
Transformação de implicação em disjunção:
p⇔q (p ⇒ q) ≡ (p ∨ q)

“p é condição necessária e suficiente para q”
≡ QUANTIFICADORES
“p se, e somente se, q” O quantificador universal (∀) indica “qualquer que seja”, “para
todo”.
Uma proposição do tipo “p se, e somente se, q” é verdadeira
Exemplo:
quando p ⇒ q (p é condição necessária para q) e q ⇒ p (p é condição
suficiente para q) são ambas verdadeiras. (∀x ∈ ) (x² ≥ 0)
Exemplo: O quantificador existencial (∃) indica “existe”, “existe pelo menos
3x + 1 = 4 ⇔ 3x = 4 – 1 um”, “existe um”. indica “existe um único”, “existe um e um só”.
Exemplos:
Na resolução de equações e inequações deve-se atentar
para o significado das relações de implicação e equivalência. (∃x ∈ )(x + 1 > 2) e (∃ | x ∈ )(x + 1 < 2).
Passagens relacionadas por equivalência mantêm exatamente o
mesmo conjunto verdade, pois ambas são verdadeiras ou falsas
simultaneamente. Já passagens relacionadas por implicação não
NEGAÇÃO DE PROPOSIÇÕES
garantem o mesmo conjunto verdade. Nesse caso, o novo conjunto (LEIS DE DE MORGAN)
verdade contém o anterior, devendo-se ter cuidado com a introdução A negação de proposições com conectivos ou condicionais é feita
de raízes que não são válidas. Isso ocorre com frequência na resolução com base nas relações seguintes:
de equações irracionais.

26

PM_BOOK16 - MAT.indb 26 25/11/2022 19:06:32


LÓGICA

LEIS DE MORGAN x y x2  y2
 2  x 2  y 2  2xy   x  y   0 
2
 2
y x xy
x y x y
2 2

(p ∧ q) ⇔ p ∨ q y  x  2  xy  2  x  y  2xy   x  y   0  CONTRADIÇÃO.
2 2 2

Logo, a proposição inicial é válida.


(p ∨ q) ⇔ p ∧ q
CONTRAEXEMPLO
(p → q) ⇔ p ∧ q
Para mostrar que uma proposição da forma (∀x ∈ A)(p(x)) é falsa
(F), basta mostrar que a sua negação (∃x ∈ A)(–p(x)) é verdadeira (V),
As Leis de De Morgan mostram que: isto é, que existe pelo menos um elemento x0 ∈ A tal que p(x0) é uma
I. Negar que duas proposições são verdadeiras ao mesmo proposição falsa (F). O elemento x0 diz-se um contraexemplo para a
tempo equivale a afirmar que pelo menos uma delas é falsa. proposição (∀x ∈ A)(p(x)).
II. Negar que pelo menos uma de duas proposições é verdadeira Exemplo:
equivale a afirmar que ambas são falsas. Prove que a proposição (∀x ∈ ) (2n > n²) é falsa.
Exemplos: Basta verificar que para n = 2 tem-se (2² > 2²) é falsa. Logo 2 é um
1. A negação de “Juca é bom e honesto” é “Juca não é bom ou contraexemplo para a proposição apresentada que, em consequência,
não é honesto”. é falsa.
2. A negação de “Juca é bom ou honesto” é “Juca não é bom e
não é honesto”. PRINCÍPIO DA INDUÇÃO FINITA (P.I.F.)
3. A negação de “Se Juca é bom, então é honesto” é “Juca é
bom e não é honesto”. AXIOMAS DE PEANO
A negação de uma proposição do tipo: “Para todo objeto, com O conjunto  dos números naturais é caracterizado pelas
uma certa propriedade, algo acontece” é: “Existe um objeto com a seguintes propriedades:
certa propriedade, tal que aquele algo não acontece.” 1. (1) Existe uma função injetiva s:  →  . A imagem s(n) de
A negação de uma proposição do tipo: “Existe um objeto, com cada número natural n ∈  chama-se o sucessor de n.
uma certa propriedade, para o qual algo acontece” é: “Para todo Todo número natural tem um sucessor, que ainda é um número
objeto com a certa propriedade, aquele algo não acontece.” natural; números diferentes têm sucessores diferentes.
2. Existe um único número natural 1 ∈  tal que 1 ≠ s(n) para
ProBizu todo n ∈  .
Em geral, usa-se o quantificador existencial para negar proposições Existe um único número natural 1 que não é sucessor de nenhum
com quantificador universal, e o quantificador universal para outro.
negar proposições com quantificador existencial.
3. Se um conjunto X ⊂  é tal que 1 ∈ X e s(X) ⊂ X (isto é,
n ∈ X ⇒ s(n) ∈ X) então X =  .
Atente para os casos a seguir:
Se um conjunto de números naturais contém o número 1 e
PROPOSIÇÃO NEGAÇÃO contém também o sucessor de cada um dos seus elementos,
então esse conjunto contém todos os números naturais. (Princípio
Existe pelo menos um da Indução).
Todos os alunos usam óculos.
aluno que não usa óculos.

Algum aluno usa óculos. Nenhum aluno usa óculos. MÉTODO DA INDUÇÃO FINITA
Algum homem não (RECORRÊNCIA)
Todo homem é honesto. “Se uma propriedade P é válida para o número 1 e se, supondo
é honesto.
P válida para o número n daí resultar que P é válida também para seu
Algum homem é honesto. Todo homem não é honesto. sucessor s(n), então P é válida para todos os números naturais.”

9>5 9≤5
APLICAÇÃO DO PIF
Passo 1: demonstrar que a afirmação é verdadeira para um caso
TÉCNICAS DE DEMONSTRAÇÃO particular, por exemplo, n = 1 (ou o menor elemento do conjunto);
Passo 2: supor que a afirmação é válida para n = k (hipótese de
DEMONSTRAÇÃO INDIRETA OU indução);
REDUÇÃO AO ABSURDO Passo 3: demonstrar, a partir disto, que a afirmação é válida para
n = k + 1.
Consiste em admitir a negação da conclusão q e daí deduzir
logicamente uma contradição qualquer c (uma proposição logicamente Exemplo:
falsa como p. ex. p ∧ p). n  n  1
Demonstrar que 1 2  3    n  .
Isso pode ser verificado observando que (~q → c) ⇔ (~~q ∨ c) ⇔ 2
(q ∨ c) ⇔ q .
Exemplo: Vamos aplicar o Princípio da Indução Finita (PIF):
x y Passo 1:
Sendo x, y  * , prove que   2.
y x 1 1 1  
x y n  1: 1  V
Supondo por absurdo a negação da proposição inicial   2, 2
y x
teremos:

27

PM_BOOK16 - MAT.indb 27 25/11/2022 19:06:36


LÓGICA

Passo 2: c) Ana estuda e não será aprovada.


Supondo que a propriedade é válida para n = k, então d) Ana estuda ou não será aprovada.
k  k  1 e) Ana não estuda e não será aprovada.
1 2  3    k  .
2
Passo 3: 05. (ESPM) Ana, Bia e Carla são amigas. Uma delas é loira, outra
morena e outra ruiva, não necessariamente nessa ordem. Apenas uma
Para n = k + 1, temos:
das afirmações abaixo é verdadeira:
k  k  1  • Ana é loira.
1  2  3    k  k  1   k  1 
2
• Bia não é loira.
k  k  1  k  2
 k  1    1  • Carla não é morena.
2  2
Podemos afirmar, com certeza, que:
Como a propriedade é válida também para n = k + 1, ela é válida
para todo natural. C.Q.D. a) Ana é loira e Bia é ruiva. d) Ana é morena e Carla é ruiva.
b) Carla é morena e Bia é loira. e) Carla é loira e Ana é morena.
c) Bia é ruiva e Carla é morena.
EXERCÍCIOS DE

FIXAÇÃO 06. (ESPM) Sobre um número x são feitas 4 afirmações:


I. x=1 II. x≠2 III. x ≠ 3 IV. x ≠ 4
Sabendo-se que apenas uma das afirmações é falsa, podemos afirmar que:
01. (FATEC) São proposições equivalentes para a negação da a) I é verdadeira. d) III é verdadeira.
proposição “todo homem ama lógica”: b) III é falsa. e) IV é falsa.
• “pelo menos um homem não ama lógica”; c) II é verdadeira.
• “algum homem não ama lógica”;
• “existe um homem que não ama lógica”. 07. (ESPM) Lauro, Pedro e Carlos foram a uma festa, cada um
acompanhado de uma garota. Perguntados sobre suas respectivas
Assim, a forma correta de negar a proposição “todas as vagas deste companhias, deram as seguintes respostas:
vestibular são de ensino superior” é
Joana: “Eu estava com o Lauro”.
a) “algumas vagas deste vestibular são de ensino superior”.
Pedro: “Se eu não estivesse com Bruna, Joana estaria com Lauro”.
b) “existem vagas deste vestibular que não são de ensino superior”.
Carlos: “Ou Pedro estava com Joana ou Lauro estava com Estela”.
c) “nenhuma das vagas deste vestibular é de ensino superior”.
d) “pelo menos uma vaga deste vestibular é de ensino superior”. Soube-se depois que apenas Joana não falou a verdade.

e) “todas as vagas deste vestibular não são de ensino superior”. Podemos, então, concluir que:
a) Pedro estava com Joana e Lauro com Estela.
02. (ESPM) Para organizar uma fila, a professora foi fazendo trocas de b) Pedro estava com Joana e Carlos com Bruna.
lugar de dois em dois alunos entre si, de modo que o mais alto sempre c) Lauro estava com Bruna e Carlos com Joana.
ficasse atrás do mais baixo.
d) Lauro estava com Estela e Carlos com Joana.
e) Pedro estava com Estela e Carlos com Bruna.

08. (ESPM) Sabe-se que:


• b≠5
• Se d = 3, então b = 5
• Se a ≠ 2, então d = 3
• C = 4 ou a ≠ 2 ou b = 5
Para passar da configuração A para a configuração B, foram Podemos, então, afirmar que:
necessárias, no mínimo:
a) a + d = 5 c) a≠2 e) d = 3
a) 5 trocas. c) 6 trocas e) 3 trocas.
b) c + d = 7 d) a + c = 6
b) 4 trocas. d) 7 trocas.
09. (PUCPR) O professor de medicina Helton faz as seguintes
03. (ESPM) Não é verdade que Paulo foi à escola e João não foi. Então, afirmações sobre as notas de:
podemos afirmar que:
Marcelo > Carlos e Zélia < Yara;
a) Se João foi à escola, Paulo não foi.
Marcelo > Yara e Carlos > Yara se e somente se Yara > Zélia;
b) Se João não foi à escola, Paulo também não foi.
Roberto ≠ Carlos, se e somente se Yara = Marcelo.
c) Ambos foram à escola.
d) Nenhum deles foi à escola. Sabendo se que todas as afirmações do professor são verdadeiras,
conclui-se que a nota de
e) Apenas um deles foi à escola.
a) Marcelo > Roberto > Zélia > Carlos.
04. (ESPM) Considere a proposição P = “Não é verdade que, se Ana b) Marcelo > Roberto > Yara > Zélia.
estuda, ela será aprovada”. Uma proposição equivalente a essa é: c) Marcelo > Zélia > Carlos > Roberto.
a) Ana não estuda e será aprovada. d) Marcelo > Carlos > Roberto > Zélia.
b) Se Ana não estuda, ela não será aprovada. e) Marcelo > Zélia > Roberto > Carlos.

28

PM_BOOK16 - MAT.indb 28 25/11/2022 19:06:37


LÓGICA

10. (PUCRJ) Os sobrenomes de Roy, Edu e Luan são Todeka, Sharifa e I. Regina vai de ônibus para o Colégio São Pedro.
Arrabeca, não necessariamente nessa ordem. O de sobrenome Sharifa, II. Ana vai de moto.
que não é o Roy, é mais velho que Luan. O de sobrenome Arrabeca é
o mais velho dos três. Concluímos, então, que os sobrenomes de Roy, III. Helô estuda no Colégio Santo Antônio.
Edu e Luan são, respectivamente: Com relação a estas afirmativas, conclui-se:
a) Todeka, Sharifa e Arrabeca. d) Arrabeca, Todeka e Sharifa. a) Apenas a I é verdadeira. d) Apenas a III é verdadeira.
b) Todeka, Arrabeca e Sharifa. e) Sharifa, Todeka e Arrabeca. b) Apenas a I e a II são verdadeiras. e) Todas são verdadeiras.
c) Arrabeca, Sharifa e Todeka. c) Apenas a II é verdadeira.

07. Eduardo mente nas quartas, quintas e sextas e diz a verdade no


EXERCÍCIOS DE resto da semana. André mente aos domingos, segundas e terças e diz

TREINAMENTO a verdade no resto dos dias. Se ambos dizem: “amanhã é um dia no


qual eu minto”. Que dia da semana será amanhã?
a) Sábado c) Quarta-feira
b) Terça-feira d) Sexta-feira
01. (CN) Um torneio de judô é disputado por 10 atletas e deve ter
apenas um campeão. Em cada luta não pode haver empate e aquele
que perder três vezes deve ser eliminado da competição. Qual o 08. (UFF) Na cidade litorânea de Ioretin é rigorosamente obedecida a
número máximo de lutas necessário para se conhecer o campeão? seguinte ordem do prefeito: “Se não chover, então todos os bares à
beira-mar deverão ser abertos.” Pode-se afirmar que:
a) 27 b) 28 c) 29 d) 30 e) 31
a) Se todos os bares à beira-mar estão abertos, então choveu.
02. (CN) Numa prova de vinte questões, valendo meio ponto cada b) Se todos os bares à beira-mar estão abertos, então não choveu.
uma, três questões erradas anulam uma certa. Qual é a nota de um c) Se choveu, então todos os bares à beira-mar não estão abertos.
aluno que errou nove questões em toda essa prova? d) Se choveu, então todos os bares à beira-mar estão abertos.
a) Quatro c) Quatro e meio e) Seis e meio e) Se um bar à beira-mar não está aberto, então choveu.
b) Cinco d) Cinco e meio
09. (CEFET) Qual o número mínimo de vezes que uma pessoa deverá
03. (CN) Um bebedouro que usa garrafão de água tem 2,5 metros de lançar um dado (não viciado) para, com certeza, obter pelo menos três
serpentina por onde a água passa para gelar. Sabe-se que tal serpentina resultados repetidos?
gasta 12 segundos para ficar totalmente gelada. Colocando-se um a) 6 b) 12 c) 13 d) 18 e) 20
garrafão de 10 litros e ligando-se o bebedouro, leva-se 5 minutos para
que toda a água saia gelada. Se nas mesmas condições fosse colocado
10. (UFRJ) Uma amostra de 100 caixas de pílulas anticoncepcionais
um garrafão de 20 litros no lugar do de 10 litros, o tempo gasto para
fabricadas pela Nascebem S.A. foi enviada para a fiscalização sanitária.
que toda a água saísse gelada seria de:
No teste de qualidade, 60 foram aprovadas e 40 reprovadas, por
a) 9 min 36 seg c) 10 min e) 11 min
conterem pílulas de farinha. No teste de quantidade 74 foram
b) 9 min 48 seg d) 10 min 12 seg aprovadas e 26 reprovadas, por conterem um número de pílulas
menor do que o especificado.
04. (CN) Em um navio existem 6 barcos e 15 guarnições. Cada barco O resultado dos dois testes mostrou que 14 caixas foram reprovadas
tem uma guarnição de serviço por dia. Quantos dias, no mínimo, serão em ambos os testes.
necessários para que todas as guarnições tenham ficado de serviço o
mesmo número de vezes? Quantas caixas foram aprovadas em ambos os testes?

a) 5 b) 6 c) 7 d) 8 e) 15 a) 24 b) 30 c) 36 d) 48 e) 50

05. (CN) Uma fábrica de fósforo usa as seguintes definições: 11. Considere os seguintes enunciados:

Caixa: conjunto de 45 fósforos • 16 é múltiplo de 2

Maço: conjunto com 10 caixas • 15 é múltiplo de 7

Pacote: conjunto com 12maços • 8 é número primo

Dividindo-se 13 pacotes, 5 maços, 8 caixas e 22 fósforos por 8, obtém- A proposição que apresenta valor lógico verdadeiro é:
se um número p de pacotes, m de maços, c de caixas e f de fósforos, a) Se 15 é múltiplo de 7 ou 16 é múltiplo de 2 então 8 é número primo
tais que p + m + c + f é igual a: b) Se 16 é múltiplo de 2 ou 8 é número primo então 15 é múltiplo de 7
a) 25 a) 26 a) 27 a) 28 a) 29 c) Se 16 é múltiplo de 2 então 15 é múltiplo de 7 e 8 é número primo
d) Se 15 é múltiplo de 7 e 8 é número primo então 16 é múltiplo de 2
06. (UFF) As três filhas de Seu Anselmo – Ana, Regina e Helô – vão
para o colégio usando, cada uma, seu meio de transporte preferido: e) Se 16 é múltiplo de 2 então 15 é múltiplo de 7 ou 8 é número primo
bicicleta, ônibus ou moto. Uma delas estuda no Colégio Santo
Antônio, outra no São João e outra no São Pedro. Seu Anselmo está 12. Em uma roda de amigos, Jorge, Edson e Geraldo contaram fatos
confuso em relação ao meio de transporte usado e ao colégio em que sobre suas namoradas. Sabe-se que Jorge e Edson mentiram e que
cada filha estuda. Lembra-se, entretanto, de alguns detalhes: Geraldo falou a verdade. Assinale qual das proposições abaixo é
verdadeira:
• Helô é a filha que anda de bicicleta;
a) “Se Geraldo mentiu, então Jorge falou a verdade”
• a filha que anda de ônibus não estuda no Colégio Santo Antônio;
b) “Edson falou a verdade e Geraldo mentiu”
• Ana não estuda no Colégio São João e Regina estuda no Colégio
São Pedro. c) “Se Edson mentiu, então Jorge falou a verdade”
Pretendendo ajudar Seu Anselmo, sua mulher junta essas informações d) “Jorge falou a verdade ou Geraldo mentiu”
e afirma: e) “Edson mentiu e Jorge falou a verdade”

29

PM_BOOK16 - MAT.indb 29 25/11/2022 19:06:37


LÓGICA

13. João não estudou para a prova de matemática; por conta disso, 19. Numa certa sala, os advogados sempre mentem e os médicos
não entendeu o enunciado da primeira questão. A questão era de sempre falam a verdade. Larissa, uma empresária, entrou na sala e
múltipla escolha e tinha as seguintes opções: perguntou ao primeiro: o senhor é advogado? E este, de brincadeira,
a) O problema tem duas soluções, ambas positivas. respondeu à pergunta em chinês. Um segundo disse: vou traduzir. O
primeiro respondeu que não é advogado. Um terceiro disse o primeiro
b) O problema tem duas soluções, uma positiva e outra negativa. realmente é um advogado. Do exposto podemos concluir que:
c) O problema tem mais de uma solução. a) O segundo é advogado.
d) O problema tem pelo menos uma solução. b) O primeiro é médico, mas o terceiro é advogado.
e) O problema tem exatamente uma solução positiva. c) Somente é possível afirmar que o segundo é um médico.
João sabia que só havia uma opção correta. Ele pensou um pouco e d) O segundo e o terceiro são advogados.
cravou a resposta certa. e) O terceiro certamente é médico.
Determine a escolha feita por João.
20. (ESPM) Quanto ao estado civil das funcionárias de um escritório,
14. Uma pessoa que gosta de todas e apenas das pessoas que não é verdade que:
gostam de si mesmas: • Ou Laura não é casada ou Maria é casada.
a) gosta de si mesma d) gosta de alguém • Se Maria é casada, então Paula é divorciada.
b) não gosta de si mesma e) não gosta de ninguém. • Se Paula não é divorciada, então Laura é casada.
c) não existe Com base no exposto, pode-se afirmar que:
a) Laura é casada. d) Laura é solteira.
15. Numa cidade, os seguintes fatos são verdadeiros:
b) Maria é solteira. e) Paula é divorciada.
I. Alguns canhotos não fumam cigarros.
c) Paula é casada.
II. Todos os homens fumam cigarros.
Uma conclusão que se pode tirar é: 21. Maria repartiu, entre seus cinco sobrinhos, o seguinte valor
a) Alguns canhotos são homens monetário: uma moeda de 25 centavos, uma moeda de 50 centavos,
b) Alguns canhotos não são homens uma moeda de 1 real, uma nota de 2 reais e uma nota de 5 reais.
Depois de feita a repartição, todos receberam algum valor monetário.
c) Nenhum canhoto é homem
A respeito da repartição, Maria e seus sobrinhos fizeram os seguintes
d) Alguns homens não são canhotos comentários:
e) Nenhum homem é canhoto Aldo: “Recebi a moeda de 1 real”.
Bruno: “Não recebi a nota de 2 reais”.
16. Cada um dos cartões abaixo tem de um lado um número e do
outro uma letra. Cláudio: “Bruno recebeu mais dinheiro do que eu”.
Daniel: “Aldo recebeu a moeda de 50 centavos”.
A B 2 3 Eric: “Cláudio não recebeu a nota de 2 reais”.
Maria: “Daniel recebeu menos dinheiro do que Aldo”.
Alguém afirmou que todos os cartões que têm uma vogal numa
face têm um número par na outra. Para verificar se tal afirmação é Se apenas uma das seis pessoas disse a verdade em seu comentário, é
verdadeira: correto concluir que Aldo recebeu:
a) é necessário virar todos os cartões a) 25 centavos. d) 2 reais.
b) é suficiente virar os dois primeiros cartões b) 50 centavos. e) 5 reais.
c) é suficiente virar os dois últimos cartões c) 1 real.
d) é suficiente virar os dois cartões do meio
22. Dentro de um grupo de tradutores de livros, todos os que falam
e) é suficiente virar o primeiro e o último cartão alemão também falam inglês, mas nenhum que fala inglês fala
japonês. Além disso, os dois únicos que falam russo também falam
17. Na cidade litorânea de Ioretin é rigorosamente obedecida a coreano. Sabendo que todo integrante desse grupo que fala coreano
seguinte ordem do prefeito: também fala japonês, pode-se concluir que, necessariamente,
“Se não chover, então todos os bares à beira mar deverão ser aberto.” a) todos os tradutores que falam japonês também falam russo.
Pode-se afirmar que: b) todos os tradutores que falam alemão também falam coreano.
a) Se todos os bares à beira-mar estão abertos, então choveu c) pelo menos um tradutor que fala inglês também fala coreano.
b) Se todos os bares à beira-mar estão abertos, então não choveu d) nenhum dos tradutores fala japonês e também russo.
c) Se choveu, então todos os bares à beira-mar não estão abertos e) nenhum dos tradutores fala russo e também alemão.
d) Se choveu, então todos os bares à beira-mar estão abertos
e) Se um bar à beira-mar não está aberto, então choveu 23. As três afirmações abaixo, todas verdadeiras, foram feitas por Luís
para descrever o que pretendia fazer em relação às suas economias e
18. Raul e Cida formam um estranho casal. Raul mente às 4as, 5as planos de viagem.
e 6as feiras, dizendo a verdade no resto da semana. Cida mente aos • Se o preço do dólar cair no final do ano, então eu vou investir
domingos, 2as e 3as feiras dizendo a verdade nos outros dias. Certo dia, em poupança e viajar para o exterior.
ambos declaram: “amanhã é dia de mentir”. O dia em que foi feita • Se eu viajar para o exterior, então vou comprar um equipamento
essa declaração é: de esqui.
a) 3a feira c) 6a feira e) domingo • Se eu alugar ou comprar um equipamento de esqui, então vou
b) 4a feira d) sábado esquiar em Bariloche.

30

PM_BOOK16 - MAT.indb 30 25/11/2022 19:06:38


LÓGICA

A partir das três afirmações e da informação de que Luís não Agora, considere as seguintes afirmativas:
esquiou em Bariloche, pode-se tirar algumas conclusões que são, • Alguns matemáticos são filósofos.
necessariamente, verdadeiras. Dentre as conclusões abaixo, a única
que não é, necessariamente, verdadeira é • Nem todo filósofo é cientista.

a) o preço do dólar não caiu no final do ano. • Alguns filósofos são professores.

b) Luís não investiu em poupança. • Se um filósofo não é matemático, ele é professor.

c) Luís não viajou para o exterior. • Alguns filósofos são matemáticos.

d) Luís não comprou um equipamento de esqui. Partindo do princípio de que as 4 (quatro) primeiras afirmativas são
e) Luís não alugou um equipamento de esqui. verdadeiras, quantas afirmativas do 2º grupo são NECESSARIAMENTE
verdadeiras:
24. No mapa a seguir estão indicados os depósitos de uma rede de a) 0 b) 1 c) 2 d) 3 e) 4
supermercados e as rotas possíveis entre eles.
04. (CPII 2007) Dada a proposição: “Se um quadrilátero é um retângulo
então suas diagonais cortam-se ao meio”, podemos afirmar que:
a) Se um quadrilátero tem as diagonais cortando-se ao meio então
ele é retângulo.
b) Se um quadrilátero não tem as diagonais cortando-se ao meio
então ele não é um retângulo.
c) Se um quadrilátero não é um retângulo então suas diagonais não
se cortam ao meio.
d) Se em um quadrilátero as diagonais se cortam ao meio então ele
não é um retângulo.
Um caminhão saindo do depósito A pode chegar ao depósito H de
várias maneiras. Por exemplo, os trajetos A  D  H e A  B  C  05. (PUCRJ 1993) Três caixas etiquetadas estão sobre uma mesa.
E  G  F  H são duas possibilidades. A quantidade total de trajetos Uma delas contém apenas canetas; outra, apenas lápis; e há uma
que um caminhão da empresa pode fazer, partindo do depósito A que contém lápis e canetas. As etiquetas são “lápis”, “canetas” e
com destino ao depósito H, sem passar mais de uma vez pelo mesmo “lápis e canetas”, porém nenhuma caixa está com a etiqueta correta.
depósito, é igual a É permitida a operação escolher uma caixa e dela retirar um único
objeto. O número mínimo de operações necessárias para identificar
a) 8 b) 12 c) 16 d) 30 e) 64
corretamente as etiquetas é:
a) 0 b) 1 c) 2 d) 3 e) 4

EXERCÍCIOS DE
 
06. (FGV 81) Simplificando a expressão  X  Y   X  Y , teremos:
COMBATE a) universo.
b) vazio.
c) X∩Y e) X∩Y
d) X∩Y

01. (EPCAR 2012) Uma pessoa foi realizar um curso de 07. Sendo p e q proposições lógicas, pode-se afirmar que a proposição
aperfeiçoamento. O curso foi ministrado em x dias nos períodos da
manhã e da tarde desses dias. Durante o curso foram aplicadas 9
composta p  q  p  q é equivalente a:
avaliações que ocorreram em dias distintos, cada uma no período da a) p b) q c) p∨q d) p∧q e) p ⇒ θ
tarde ou no período da manhã, nunca havendo mais de uma avaliação
no mesmo dia. Houve 7 manhãs e 4 tardes sem avaliação. O número
08. (EN 1989) Dada a proposição p   q  r   p  q  p  r 
x é divisor natural de:
podemos afirmar que é:
a) 45 b) 36 c) 20 d) 18
a) logicamente falsa. d) equivalente a p  q  r
b) uma tautologia.
02. Quatro suspeitos de praticar um crime fazem as seguintes e) equivalente a p  q  r
declarações: c) equivalente a p  q  r
André: Carlos é o criminoso.
09. (CPII 2013) A sentença logicamente equivalente a “Se a
Bernardo: Eu não sou o criminoso.
matemática não é divertida, então eu sou triste” é:
Carlos: Danilo é o criminoso.
a) “Se eu não sou triste, então a matemática é divertida.”
Danilo: Carlos está mentindo.
b) “Se eu não sou triste, então a matemática não é divertida.”
Sabendo que apenas um dos suspeitos disse a verdade, o criminoso é:
c) “Se eu sou triste, então a matemática é divertida.”
a) André. d) Danilo.
d) “Se eu sou triste, então a matemática não é divertida.”
b) Bernardo. e) impossível determinar.
c) Carlos. 10. (EN) Neste ano de 2021, os sábados de fevereiro e março caíram
nos mesmos dias do mês: dia 6, dia 13, dia 20 e dia 27. Sejam X e Y
03. (CEFET 2007) Considere as seguintes afirmativas: os próximos dois anos em que novamente esse fato ocorrerá, ou seja,
• Todos os matemáticos são cientistas. que os sábados de fevereiro e março cairão nos dias 6, 13, 20 e 27, é
correto afirmar que o valor de X + Y é igual a:
• Alguns cientistas são filósofos.
a) 4062. c) 4064. e) 4066.
• Todos os filósofos são cientistas ou professores.
b) 4063. d) 4065.
• Nem todo professor é cientista.

31

PM_BOOK16 - MAT.indb 31 25/11/2022 19:06:44


LÓGICA

GABARITO

DESAFIO PRO EXERCÍCIOS DE FIXAÇÃO


01. B 05. D 09. B
02. B 06. C 10. C

1 Fazer a tabela-verdade das fórmulas abaixo.


03. B
04. C
07. D
08. D
a) ( (a ^ c) → a)
EXERCÍCIOS DE TREINAMENTO
b) ( (~~ a v c) ^ c)
01. C 09. C 17. E
c) ( ( p ^ ~q) v (~p v r) )
02. A 10. D 18. A
d) (p → q) ↔ (~q → ~p)
03. B 11. D 19. C

2 O 80˚ termo da sequência lógica (1, 1, 2, 1, 2, 3, 1, 2, 3, 4, 1,


2, 3, 4, 5, ...) é igual a:
04. A
05. A
12. A
13. D
20. E
21. A
a) 6 b) 3 c) 1 d) 7 e) 2 06. B 14. C 22. E
07. C 15. B 23. B
3 O produto de 417 por ⊗1∆⊗⊕ é 9⊕∇⊕057, sendo que os
símbolos representam números da base decimal. Assinale a
alternativa que apresenta o produto correto.
08. E
EXERCÍCIOS DE COMBATE
16. E 24. C

a) 9.131.057. 01. C 05. B 09. A


b) 9.343.057. 02. B 06. C 10. D
c) 9.242.057. 03. A 07. A
d) 9.121.057. 04. B 08. B
e) 9.141.057. DESAFIO PRO
01. a)

4 (EN) Chama-se conjunto-verdade de uma sentença aberta


p(X) em um conjunto A o conjunto de todos os elementos
a ∈ A, tais que p(a) é uma proposição verdadeira (V). Sejam V
a c
V
(a ^ c)
V
( (a ^ c) → c)
V
p(x), q(x) e r (x) sentenças abertas em um mesmo conjunto A. V F F V
Encontre o conjunto-verdade da sentença aberta composta F V F V
(p(x) → q(x))v ˜ r (x), em função de VP, VQ e Vr, assinale a opção F F F V
correta.
b)
a) CAVP∪(Vq∪CAVr)
a c (a ^ c) (~~ a v c) ( (~~ a v c) ^ c)
b) Vr∩(CAVq∪CAVq)
V V V V V
c) CAVq∪(Vp∩CAVr)
V F F F F
d) CAVr∪(Vq∩CAVp) F V F F F
e) Vp∩(CAVq∪CAVr) F F F F F
c)

5 A figura indica três cartas, A, B e C, cada uma com um


número inteiro positivo no verso.
V
p q
V
r
V
~q
F
~p
F
( p ^ ~q) (~p v r) ( ( p ^ ~q) v (~p v r) )
F F F
V V F F F F F F
V F V V F V F V
V F F V F V F V
F V V F V F V V
A respeito dos números no verso das cartas, sabe-se que: F V F F V F F F
I. os três números são diferentes; F F V V V F V V
II. a soma dos três números é igual a 13; F F F V V F F F
d)
III. os números estão em ordem crescente, da carta A para a C.
Alzira olhou apenas a carta A e disse que ainda não era possível p q ~p ~q p→q ~q → ~p (p→q) ↔ (~q → ~p)
saber os números das outras cartas. Cláudia olhou apenas a carta V V F F V V V
C e disse que ainda não era possível saber os números das outras V F F V F F V
cartas. Bruna olhou apenas a carta B e disse que ainda não era F V V F V V V
possível saber os números das outras cartas. Considerando que
F F V V V V V
cada uma ouviu o que foi dito por todas, e que todas utilizaram
raciocínio perfeito em suas deduções com as informações que 02. E
tinham até o momento em que olharam suas cartas, é correto 03. E
afirmar que a carta B tem, em seu verso, o número:
04. A
a) 2 c) 4 e) 6
05. C
b) 3 d) 5

32

PM_BOOK16 - MAT.indb 32 25/11/2022 19:06:44


CONJUNTOS

NOÇÕES PRIMITIVAS E NOTAÇÃO CONJUNTO UNITÁRIO


Conjunto, elemento e pertinência entre elemento e conjunto Um conjunto unitário é um conjunto que possui somente um
são noções primitivas, ou seja, conceitos iniciais para os quais não há elemento.
definição. Exemplos:
Um conjunto costuma ser denotado por uma letra maiúscula e os A = {1}
seus elementos por letras minúsculas entre chaves. Assim, o conjunto
A de elementos a, b e c é denotado por A = {a,b,c}. B = {x | x é um número primo par} = {2}

Um conjunto pode ser representado graficamente por um C = {{1,2,3}} é um conjunto unitário de elemento {1,2,3}.
diagrama de Venn, onde cada conjunto é representado por uma
ProBizu
curva fechada. Os elementos pertencentes ao conjunto localizam-se
no interior dessa curva e os elementos não pertencentes ao conjunto, O conjunto {∅} é um conjunto unitário de elemento ∅.
em seu exterior.

CONJUNTO UNIVERSO
Quando todos os conjuntos em análise são subconjuntos de um
mesmo conjunto, este recebe o nome de conjunto universo e denota-
se por U.

CONJUNTOS IGUAIS
Dois conjuntos são iguais se possuem os mesmos elementos.
A = B ⇔ (∀x)(x ∈ A ⇔ x ∈ B)

Exemplos:
{a,b,c} = {b,c,a}
{a,b,c} ≠ {a,b,c,d}
{a,b,c,a,c} = {a,b,c}, pois ambos os conjuntos possuem os mesmos
elementos a, b e c.

SUBCONJUNTOS
Um conjunto A é subconjunto de um conjunto B, denotado
por A ⊂ B, se, e somente se, todo elemento de A é também elemento
Se a é um elemento do conjunto A, diz-se que a pertence a de B.
A e denota-se a ∈ A.
A ⊂ B ⇔ (∀x)(x ∈ A ⇒ x ∈ B)
Se d não é elemento de A, diz-se que d não pertence a A e
denota-se d ∉ A. Exemplos:
Exemplo: {a,b} ⊂ {a,b,c,d}; {a,b} ⊄ {b,c,d}
Seja o conjunto A = {1,2}, então 1 ∈ A, 2 ∈ A e 3 ∉ A. O diagrama de Venn a seguir representa dois conjuntos A e B tais
que A ⊂ B.
Um conjunto pode ser descrito pela citação de seus elementos ou
por uma propriedade característica.
Exemplo:
A = {a,e,i,o,u} = {x | x é vogal}

CONJUNTO VAZIO
O conjunto vazio é o conjunto que não possui elementos e é
denotado por ∅ ou { }.
Exemplo:
A = {x | x é ímpar e múltiplo de 2} = ∅

33

PM_BOOK16 - MAT.indb 33 25/11/2022 19:06:45


CONJUNTOS

PROPRIEDADES DA INCLUSÃO NÚMERO DE ELEMENTOS DA UNIÃO


Para quaisquer conjuntos A, B e C, tem-se: O número de elementos da união de conjuntos pode ser calculado
∅⊂A com base no princípio da inclusão-exclusão. A seguir estão as relações
para o caso de dois e três conjuntos:
A ⊂ A (propriedade reflexiva)
(A ⊂ B ∧ B ⊂ A) ⇒ A = B (propriedade antissimétrica) #(A ∪ B) = #(A) + #(B) – #(A ∩ B)

(A ⊂ B ∧ B ⊂ C) ⇒ A ⊂ C (propriedade transitiva)
#(A ∪ B ∪ C) = #(A) + #(B) + #(C) – #(A ∩ B) –
– #(A ∩ C) – #(B ∩ C) + #(A ∩ B ∩ C)
• A é um subconjunto próprio de B quando A ⊂ B e A ≠ B.
Exemplo:
{1,2} é um subconjunto próprio de {1,2,3} INTERSEÇÃO DE CONJUNTOS
• O conjunto vazio não tem nenhum subconjunto próprio. Dados dois conjuntos A e B, a sua interseção é o conjunto formado
pelos elementos que pertencem a A e B, ou seja, pelos elementos
• Qualquer conjunto não vazio tem vazio como subconjunto comuns aos dois conjuntos.
próprio.
A ∩ B = {x | x ∈ A ∧ x ∈ B}
CONJUNTO DAS PARTES Exemplos:
(OU CONJUNTO POTÊNCIA)
{a,b,c,d} ∩ {c,d,e} = {c,d}; {a,b} ∩ {c,d} = ∅
O conjunto das partes ou conjunto potência de um determinado
conjunto A é o conjunto formado por todos os subconjuntos do O diagrama de Venn a seguir representa a interseção de dois
conjunto A e é denotado por P (A). conjuntos A e B.

Exemplo:
A = {a,b} ⇒ P(A) = {∅, {a}, {b}, {a,b}}
Se #(X) representa a quantidade de elementos do conjunto X, a
quantidade de elementos do conjunto das partes de um conjunto
A pode ser calculada pela expressão abaixo:
#(P(A)) = 2#(A)

OPERAÇÕES ENTRE CONJUNTOS


UNIÃO DE CONJUNTOS
Dados dois conjuntos A e B, a sua união é o conjunto formado por
CONJUNTOS DISJUNTOS
todos os elementos que pertençam a A ou a B.
Conjuntos disjuntos são conjuntos cuja interseção é o conjunto
A ∪ B = {x | x ∈ A ∨ x ∈ B} vazio, ou seja, não possuem elementos comuns.

Exemplo: A e B são disjuntos ⇔ A ∩ B = ∅


{a,b} ∪ {b,c} = {a,b,c}
Propriedades:
O diagrama de Venn abaixo representa a união dos conjuntos A
Sejam A, B e C conjuntos quaisquer, então:
e B.
• A ∩ A = A (idempotente)
• A∩∅=∅
• A ∩ B = B ∩ A (comutativa)
• (A ∩ B) ∩ C = A ∩ (B ∩ C) (associativa)

PROPRIEDADE DISTRIBUTIVA
DA UNIÃO E DA INTERSEÇÃO
A ∩ (B ∪ C) = (A ∩ B) ∪ (A ∩ C)
A ∪ (B ∩ C) = (A ∪ B) ∩ (A ∪ C)

Propriedades:
Sejam A, B e C conjuntos quaisquer, então: DIFERENÇA DE CONJUNTOS
• A ∪ A = A (idempotente) A diferença entre dois conjuntos A e B é o conjunto formado pelos
elementos que pertencem a A e não pertencem a B.
• A ∪ ∅ = A (elemento neutro)
A – B = {x | x ∈ A ∧ x ∉ B}
• A ∪ B = B ∪ A (comutativa)
Exemplos:
• (A ∪ B) ∪ C = A ∪ (B ∪ C) (associativa)
{a,b,c,d} – {c,d,e} = {a,b}; {a,b} – {a,b,c} = ∅; {a,b} – {c,d} = {a,b}
Os diagramas de Venn a seguir representam as diferenças A – B
e B – A.

34

PM_BOOK16 - MAT.indb 34 25/11/2022 19:06:47


CONJUNTOS

CONJUNTOS NUMÉRICOS
CONJUNTO DOS NÚMEROS NATURAIS
 = {0,1,2,3,...}
É fechado em relação à adição e multiplicação.
O conjunto dos naturais positivos  – {0} é denotado por.
* = {1,2,3,...}

Propriedades:
Tricotomia: dados dois naturais a e b quaisquer, tem-se que ou
a < b ou a = b ou a > b.
Princípio da boa-ordenação: todo subconjunto não-vazio dos
números naturais possui um menor elemento.

CONJUNTO DOS NÚMEROS INTEIROS


Além de ser fechado em relação à adição e multiplicação, o
conjunto dos números inteiros também é fechado em relação à
subtração.
 = {...,–3,–2,–1,0,1,2,3,...}

SUBCONJUNTOS IMPORTANTES
COMPLEMENTAR DE B EM A Conjunto dos inteiros não nulos
Dados dois conjuntos A e B, tais que B ⊂ A, chama-se
* = {...,–3,–2,–1,1,2,3,...}
complementar de B em relação a A o conjunto A – B.
Conjunto dos inteiros não negativos
B ⊂ A ⇒ CAB = A – B
+ = {0,1,2,3,...}
Exemplos:
Conjunto dos inteiros não positivos
A = {a,b,c} e B = {a,b}, B ⊂ A ⇒ CAB = A – B = {c}
- = {...,–3,–2,–1,0}
Se B não for um subconjunto de A, então CAB não está definido. Conjunto dos inteiros positivos
ProBizu *+ = {1,2,3,}

Leis de De Morgan Conjunto dos inteiros negativos

A B  A B *−= { , −3, −2, −1}

A B  A B
VALOR ABSOLUTO OU MÓDULO DE UM INTEIRO
C(A), A e A’ são notações que representam o complementar de A a, se a ≥ 0
em relação ao universo. a =
−a, se a < 0

DIFERENÇA SIMÉTRICA
Propriedades:
A diferença simétrica de dois conjuntos A e B é o conjunto dos
elementos que pertencem a um, e somente a um, dos conjuntos A e a) x ≥0
B, e denota-se por A∆B.
b) −x ≤x≤ x
A∆B = (A – B) ∪ (B – A) = (A ∪ B) – (A ∩ B)
c) x ⋅ y = x⋅y
Exemplo:
d) x2 = x
{a,b,c} ∆ {b,c,d} = {a,d}
O diagrama de Venn a seguir representa a diferença simétrica de
dois conjuntos A e B. CONJUNTO DOS NÚMEROS RACIONAIS
É o conjunto dos números que podem ser obtidos pela divisão de
dois inteiros.
a 
 =  a ∈ , b ∈ * 
b 
TODOS os números inteiros são também números racionais.

35

PM_BOOK16 - MAT.indb 35 25/11/2022 19:06:49


CONJUNTOS

OPERAÇÕES BEM DEFINIDAS RETA REAL


a c Existe uma correspondência biunívoca entre o conjunto dos
Igualdade: = ⇔ ad =bc
b d pontos de uma reta orientada e o conjunto dos números reais.
a c ad + bc
Adição: + = 
b d bd
a c ac
Multiplicação: ⋅ =
b d bd
a c a d O módulo de um número pode ser definido como a distância entre
Divisão: ÷ = ⋅
b d b c o ponto correspondente ao número na reta real e a origem da mesma.

REPRESENTAÇÃO DECIMAL DOS NÚMEROS INTERVALOS REAIS


RACIONAIS E NÚMEROS IRRACIONAIS Dados dois números reais a < b, define-se:
Todo número racional, pode ser representado na forma decimal.
Essa representação pode ser finita (decimal exato) ou infinita e Definição:
periódica (dízimas periódicas).
I. [a,b] = {x ∈ ;a ≤ x ≤ b}
Todo número que não pode ser escrito como fração de dois
números inteiros, isto é, que não são decimais exatos e nem são II. [a,b ) = {x ∈ ;a ≤ x < b}
dízimas periódicas é um número irracional.
III. ( a,b] = {x ∈ ;a < x ≤ b}
Exemplos:
a) 2 IV. ( a,b ) = {x ∈ ;a < x < b}
b) π V. [a, +∞=) {x ∈ ;a ≤ x}
c) 3
3
VI. ( −∞,a=] {x ∈ ;x ≤ a}
d) e
VII. ( −∞,a=) {x ∈ ;x < a}
GERATRIZ DE UMA DÍZIMA PERIÓDICA
Um intervalo em  é o próprio conjunto  ou dos subconjuntos
É fração ordinária cuja representação decimal é uma dízima acima. Os intervalos estão representados abaixo na reta rela.
periódica: O numerador é a parte inteira seguida de parte não-
periódica seguida do período, menos a parte inteira seguida da
parte não-periódica e o denominador é o número formado de tantos Intervalo fechado em a e b:
9 quantos forem os algarismos do período, seguidos de tantos 0 [a,b] = {x ∈  a ≤ x ≤ b}
quantos forem os algarismos da parte não-periódica após a vírgula.
Exemplos:
a) 0,333...
3 1 Intervalo fechado em a e aberto em b:
x = 0,333... ⇔ 10x = 3,333.. ⇔ 9x = 3 ⇔ x = =
9 3
[a,b[ = [a,b ) = {x ∈  a ≤ x < b}
b) 0,2777...
=
x 0,2777... ⇔ 10x = 2,777... ⇔ 100x= 27,777... ⇔
27 − 2
100x − 10x = 27 − 2 ⇔ 90x = 27 − x ⇔ x =
90
Intervalo aberto em a e fechado em b:
25 5
=
x =
90 18 ]a,b] = ( a,b] = {x ∈  a < x ≤ b}
c) 2,4333...
=x 2,4333.. ⇔ 10x
= 24,333... ⇔ 100x= 243,333...
243 − 24 219 73
100x − 10x= 243 − 24 ⇔ x= = =
90 90 30 Intervalo aberto em a e b:
]a,b[ = ( a,b ) = {x ∈  a < x < b}
CONJUNTO DOS NÚMEROS REAIS
O conjunto dos números reais  é a união do conjunto dos
números racionais com o conjunto dos números irracionais (dízimas
não-periódicas). Intervalo aberto em a até +∞:
=  ∪ 
]a, +∞=[ ( a, +∞=) {x ∈  x > a}
Além de ser fechado em relação à adição, subtração, multiplicação
e divisão, o conjunto dos números reais também é fechado em relação
à radiciação de números não negativos.
Intervalo fechado em a até +∞:
Nos conjuntos numéricos abordados valem as seguintes relações:
[a, +∞=[ [a, +∞=) {x ∈  | x ≥ a}
⊂⊂⊂
⊂ a

36

PM_BOOK16 - MAT.indb 36 25/11/2022 19:06:51


CONJUNTOS

Intervalo de –∞ até fechado em a: 08. (ESPCEX) Numa pesquisa feita junto a 200 universitários sobre
o hábito de leitura de dois jornais (A e B), chegou-se às seguintes
] − ∞,a] = ( −∞,a] = {x ∈  x ≤ a}
conclusões:
1) 80 universitários leem apenas um jornal;
2) o número dos que não leem nenhum dos jornais é o dobro do
número dos que leem ambos os jornais.
Intervalo de –∞ até aberto em a:
3) o número dos que leem o jornal A é o mesmo dos que leem
]−∞,a[ = ( −∞,a) = {x ∈  x < a} apenas o jornal B.
Com base nesses dados, podemos afirmar que o número de
universitários que leem o jornal B é:
a) 160 c) 120 e) 80
b) 140 d) 100
EXERCÍCIOS DE

FIXAÇÃO
09. Sendo M(0) o conjunto dos múltiplos de zero e D(0) o conjunto
dos divisores de zero, M(0) e D(0) são, respectivamente conjuntos:
a) unitário e infinito d) vazio e infinito
b) unitário e vazio e) infinito e vazio
01. Se A = ∅ e B = { ∅ }, então: c) vazio e unitário
a) A ⊂ B c) A=B e) B ⊂ A
b) A ∪ B = ∅ d) A ∩ B = B 10. Dados os conjuntos C = {15,25,30,35} e D = {15,25,40,50},
obtenha o n(A ∪ B) :
02. Em relação aos principais conjuntos numéricos, é correto afirmar que: a) 4 c) 6 e) 8
a) Todo número inteiro é natural, mas nem todo número natural é b) 5 d) 7
inteiro.
b) Todo número real é natural, mas nem todo número natural é real.
EXERCÍCIOS DE

TREINAMENTO
c) Todo número irracional é real.
d) Todo número racional é natural, mas nem todo número natural
é racional.
e) Todo número racional é inteiro, mas nem todo número inteiro é
racional. 01. (EEAR) “N” é o conjunto dos números naturais, = K {3x | x ∈ N},
=L {5x | x ∈ N
=} e M {15x | x ∈ N}. A afirmativa correta é:
03. Se um conjunto A possui 1024 subconjuntos, então o cardinal de a) K ∪L =M c) K–L=M
A é igual a:
a) 5 c) 7 e) 10 b) K ⊂ L d) K ∩L =M
b) 6 d) 9
02. (EEAR) Sejam os conjuntos A = {x ∈ N / x é múltiplo de 2},
B = {x ∈ Z / – 2 < x ≤ 9} e C = {x ∈ R / x ≥ 5}. A soma dos elementos
04. Os conjuntos A, B e A ∪ B tem respectivamente, 10, 9 e 15 que formam o conjunto (A ∩ B) – C é:
elementos. O número de elementos de A ∩ B é:
a) 9 b) 6 c) 3 d) 1
a) 2 c) 4 e) 8
b) 3 d) 6 03. (EEAR) Os elementos de um conjunto A são tais que 10 deles são
múltiplos de 4; 9 são múltiplos de 6; 8 são múltiplos de 12; e 4 são
05. Numa Universidade são lidos apenas os jornais X e Y, 80% dos números ímpares. Se A ⊂ (N = conjunto dos números naturais), então
alunos leem o jornal X e 60% o jornal Y. sabendo-se que todo aluno o número de elementos de A é:
é leitor de pelo menos um dos dois jornais, assinale a alternativa que a) 31 b) 25 c) 21 d) 15
corresponde ao percentual de alunos que leem ambos.
a) 80% d) 60% 04. (EEAR) Seja P o conjunto dos retângulos, Q o conjunto dos
b) 14% e) 48% quadrados e L o conjunto dos losangos. É correto afirmar que:
c) 40% a) L ∩ P = L – P c) L∩Q=P
b) L ∩ Q = L – Q d) L ∩ P = Q
06. Após um jantar, foram servidas as sobremesas X e Y. Sabe-se que
das 10 pessoas presentes, 5 comeram a sobremesa X, 7 comeram a 05. (CFOE) Em uma entrevista a um grupo de 979 sargentos da FAB,
sobremesa Y e 3 comeram as duas. Quantas não comeram nenhuma? indagou-se sobre o(s) idioma(s) que cada um fala: inglês e espanhol.
a) 1 c) 3 e) 0 Constatou-se que 527 falam inglês, 251 espanhol e 321 não falam
b) 2 d) 4 nenhum desses idiomas. Com base nessas informações, é INCORRETO
afirmar que dos sargentos entrevistados,
07. (ESA) Se A e B são conjuntos quaisquer, não vazios, podemos a) 120 falam os dois idiomas.
afirmar que a única opção falsa é: b) mais de 600 falam inglês ou espanhol.
a) A − B =∅ => B ⊂ A d) a∈ A e A ⊂ B =
> a∈ B c) 452 não falam inglês.
b) A ∩B =A=
> A ∪B =B e) a∈ A ∪ B =
> a∈ A ou a∈ B d) exatamente 130 falam somente espanhol.
c) a∈ A e a∈ B =
> a∈ A ∩ B

37

PM_BOOK16 - MAT.indb 37 25/11/2022 19:06:52


CONJUNTOS

06. (PETROBRÁS) Conversando com os 45 alunos da primeira série 14. Numa pesquisa de mercado, foram entrevistadas várias pessoas
de um colégio, o professor de educação física verificou que 36 alunos acerca de suas preferências em relação a três produtos, A, B e C. Os
jogam futebol, e 14 jogam vôlei, sendo que 4 alunos não jogam nem resultados foram:
futebol nem vôlei. O número de alunos que jogam tanto futebol • 210 pessoas compraram A
quanto vôlei é:
• 210 pessoas compraram B
a) 5 c) 9 e) 13
• 250 pessoas compraram C
b) 7 d) 11
• 20 pessoas compraram os três produtos
07. (ESPCEX) Sendo: • 100 pessoas não compraram nenhum produto
 + , o conjunto dos números reais não negativos, • 60 pessoas compraram A e B
 , o conjunto dos números racionais, • 70 pessoas compraram A e C
 , o conjunto dos números inteiros, • 50 pessoas compraram B e C

, o conjunto dos números naturais, Quantas pessoas foram entrevistadas?


a) 670 c) 870 e) 510
A intersecção dos conjuntos  + ,   (    ) e (    )   é igual a
b) 970 d) 610
a) ∅ c) * e) 
b) *+ d)  15. Numa classe de 30 alunos, 16 alunos gostam de matemática
e 20 de história. O número de alunos desta classe que gostam de
matemática e história é:
08. (EFOMM) Sejam os conjuntos U = {1, 2, 3, 4} e A = {1, 2} . O
conjunto B tal que B ∩ A ={1} e B ∪ A = U é: a) exatamente 16 d) exatamente 6
a) 0 c) {1,2} e) U b) exatamente 10 e) exatamente 18
b) {1} d) {1, 3, 4} c) no mínimo 6

09. (EFOMM) Numa companhia de 496 alunos, 210 fazem natação, 16. (CN) Considere os diagramas onde A, B, C, e U são conjuntos. A
260 musculação e 94 estão impossibilitados de fazer esportes. Neste região sombreada pode ser representada por:
caso, o número de alunos que fazem só natação é:
a) 116 c) 166 e) 194
b) 142 d) 176

10. (ESPCEX) Sejam os conjuntos A com 2 elementos, B com 3


elementos e C com 4 elementos. O número de elementos do conjunto
C − [(A ∩ B) ∩ C] pode variar entre:
a) 2 e 4 c) 0e4 e) 0 e 2
b) 2 e 3 d) 0 e 3

11. (AFA) Entrevistando 100 oficiais da AFA, descobriu-se que 20


deles pilotam a aeronave TUCANO, 40 pilotam o helicóptero ESQUILO
e 50 não são pilotos. Dos oficiais entrevistados, quantos pilotam o
TUCANO e o ESQUILO?
a) 5 c) 15
b) 10 d) 20

12. (AFA) Em um grupo de n cadetes da Aeronáutica, 17 nadam, 19


jogam basquetebol, 21 jogam voleibol, 5 nadam e jogam basquetebol,
2 nadam e jogam voleibol, 5 jogam basquetebol e voleibol e 2 fazem ( A ∩ B) ∪ ( A ∩ C) − (B ∩ C)
a)
os três esportes. Qual o valor de n, sabendo-se que todos os cadetes
desse grupo praticam pelo menos um desses esportes? b) ( A ∩ B) ∪ ( A ∩ C) − (B ∪ C)
a) 31 c) 47 c) ( A ∪ B) ∪ ( A ∩ C) ∪ (B ∩ C)
b) 37 d) 51 d) ( A ∪ B) − ( A ∪ C) ∩ (B ∩ C)
e) ( A − B) ∩ ( A − C) ∩ (B − C)
13. Em uma entrevista 52 pessoas discutem a preferência por dois
produtos A e B, entre outros e conclui-se que o número de pessoas 17. No cursinho α, onde se ensina somente Matemática, Física e
que gostavam de B era: Química, há um corpo docente composto de 24 professores, sendo que
I. O quádruplo do número de pessoas que gostavam de A e B; • 12 ensinam Matemática;
II. O dobro do número de pessoas que gostavam de A; • 10 ensinam Física;
III. A metade do número de pessoas que não gostavam de A nem de B. • 9 ensinam Química;
Nestas condições, o número de pessoas que não gostavam dos dois • 4 ensinam Matemática e Física;
produtos é igual a:
• e nenhum deles ensina Matemática e Química.
a) 48 d) 47
Se o número de professores que ensinam apenas Física é igual ao
b) 35 e) 37 número de professores que ensinam Física e Química, é INCORRETO
c) 36 afirmar que:

38

PM_BOOK16 - MAT.indb 38 25/11/2022 19:06:53


CONJUNTOS

a) 25% do corpo docente ensina apenas Química. 21. Considere o conjunto G e três de seus subconjuntos A, B e C. Se
b) o número de professores que ensinam apenas Matemática é 1/3 M também é subconjunto de G, considere MG o complemento de M
do número de professores que compõe o corpo docente. em relação à G.
c) o número de professores que ensinam apenas Física é a metade Dados:
dos que ensinam apenas Química. • B ∩ C = {7} • (A ∪ B ∪ C) G = {4, 6}
d) existem nesta Escola menos professores ensinando Matemática • A ∪ C = {1, 2, 3, 5, 7, 8, 9, 10} • BG = {3, 4, 5, 6, 8, 9}
ou Física do que professores ensinando Física ou Química. • A ∪ B = {1, 2, 7, 9, 10}

18. Para uma turma de 80 alunos do CPCAR, foi aplicada uma prova Sabendo-se que A e C são conjuntos disjuntos, é FALSO afirmar que:
de matemática valendo 9,0 pontos distribuídos igualmente em 3 a) A, B e C têm o mesmo número de elementos.
questões sobre: b) se S(A), S(B) e S(C) indicam respectivamente a soma dos elementos
1ª) FUNÇÃO de A, a soma dos elementos de B e a soma dos elementos de C,
2ª) GEOMETRIA então S(B) < S(A) < S(C)
3ª) POLINÔMIOS c) se A ∩ B = D, então D representa todos os divisores de 10
d) no conjunto G existem 4 números que são primos.
Sabe-se que:
• apesar de 70% dos alunos terem acertado a questão sobre 22. Analise as alternativas abaixo e marque a correta.
FUNÇÃO, apenas 1/10 da turma conseguiu nota 9,0;
• 20 alunos acertaram as questões sobre FUNÇÃO e GEOMETRIA;
a) Se B = { }
m ∈  | m2 < 40 , então o número de elementos do
conjunto B é 6
• 22 acertaram as questões sobre GEOMETRIA e POLINÔMIOS; e
1 1
• 18 acertaram as questões sobre FUNÇÃO e POLINÔMIOS. =
b) Se α + , então α ∈ (  −  )  (  −  ) 
2 −1 2 +1
A turma estava completa nessa avaliação, ninguém tirou nota zero, c) Se c = a + b e b é divisor de a, então c é múltiplo de a,
no critério de correção não houve questões com acertos parciais e o necessariamente.
número de acertos apenas em GEOMETRIA é o mesmo que o número
de acertos apenas em POLINÔMIOS. d) Se A = ]1, 5[ e B = ]−3, 3[ , então ] 3,1[ .
B − A =−
Nessas condições, é correto afirmar que:
a) o número de alunos que só acertaram a 2ª questão é o dobro do 23. (AFA) Se α = 2 ⋅ 2 + 2 ⋅ 2 + 2 + 2 ⋅ 2 − 2 + 2 , então
número de alunos que acertaram todas as questões. a) α ∈ ( − )
b) metade da turma só acertou uma questão. b) α pode ser escrito na forma=
α 2k, k ∈ 
c) mais de 50% da turma errou a terceira questão.
c) α ∈ (  −  )  (  −  ) 
d) apenas 3/4 da turma atingiu a média maior ou igual a 5,0
d) (    )  (  −  )  ⊃ α
19. Sendo  o conjunto dos números reais,  o dos racionais,  o
dos inteiros e  o dos complexos, considere os conjuntos: 24. (AFA) No conjunto universo S dado por
é definido o subconjunto
A=  −  , B=  −  e P = − (  −  )
 1
É correto afirmar que M (x,y) ∈  x  | 0 ≤ x ≤ 1 e 0 ≤ y ≤  . Pode-se afirmar que CMS é
=
 2
a) BP ≠ B igual a:
b) não existe k ∈  tal que k 5 17 ∈ B
a)
c) 7 ∈B
d) ( A  B)  P = B  1 1 
b) (x,y) ∈  x  | 0 < x ≤ e ≤ y ≤ 1
 2 2 
20. Em uma pesquisa realizada num grupo de 100 alunos do CPCAR,
constatou-se que 42 falam inglês, 12 falam inglês e francês, 18 falam  1 1
c) (x,y) ∈  x  | < x ≤ 1 e 0 ≤ y ≤ 
espanhol e inglês e 16 falam espanhol e francês. O número de alunos  2 2
que falam espanhol é, precisamente, 50% maior que o número
daqueles que falam francês. d)
Com base nessas informações, julgue os itens abaixo, classificando-os
em (V) verdadeiros ou (F) falsos.
25. Sejam os conjuntos A,B e C contidos em um conjunto universo
( ) O número de alunos que falam francês é igual a 0,6 do número U, sabendo que
dos que falam espanhol.
n C C − (B − A c )   =
c
25
( ) Se 9 dos alunos consultados falam as três línguas e 5 não falam  
nenhuma dessas línguas, então mais da metade dos alunos falam
n [C − B]= n C − A c  = 0
francês.
n ( A c  Bc =
)  19 e n B − A=
c
( ) Se 9 dos alunos consultados falam as três línguas, espanhol, inglês c
 5
e francês, enquanto 5 deles não falam nenhuma dessas línguas, 

{ }
então exatamente 24 desses alunos falam apenas inglês.
Onde 2n [C] = n ( A  B)  , calcule n ( A − B)  B  ( A  C) 
c c

A sequência correta é:  
a) V – V – F c) V–F–F a) 14 c) 12 e) 10
b) F – V – V d) F – F – V b) 13 d) 11

39

PM_BOOK16 - MAT.indb 39 25/11/2022 19:06:55


CONJUNTOS

EXERCÍCIOS DE 05. (ESPCEX 1996) Numa pesquisa feita junto a 200 universitários

COMBATE
sobre o hábito de leitura de dois jornais (A e B), chegou-se às seguintes
conclusões:
I. 80 universitários leem apenas um jornal.
II. O número dos que não leem nenhum dos jornais é o dobro do
01. (UFF 1999) Dado o conjunto P = {0,{0},∅,{∅}}, considere as número dos que leem ambos os jornais.
afirmativas: III. O número dos que leem o jornal A é o mesmo dos que leem
I. {0} ∈ P apenas o jornal B.
II. {0} ⊂ P Com base nesses dados, podemos afirmar que o número de
universitários que leem o jornal B é:
III. ∅ ∈ P
a) 160
Com relação a estas afirmativas conclui-se que:
b) 140
a) todas são verdadeiras.
c) 120
b) apenas a I é verdadeira.
d) 100
c) apenas a II é verdadeira.
e) 80
d) apenas a III é verdadeira.
e) todas são falsas. 06. (EEAR 2000) Os conjuntos S, T e P são tais que todo elemento
de S é elemento de T ou P. O diagrama que pode representar esses
02. (EFOMM 2010) Se X é um conjunto com um número finito de conjuntos é:
elementos, n(X) representa o número de elementos do conjunto X.
Considere os conjuntos A, B e C com as seguintes propriedades: a) c)
• n  A  B  C  25
• n  A  C  13
• n B  A   10
b) d)
• n  A  C  n C   A  B 
O maior valor possível de n (C) é igual a:
a) 9 d) 12
b) 10 e) 13
c) 11 07. (EFOMM 1995) Num grupo de 99 esportistas, 40 jogam vôlei;
20 jogam vôlei e “Futevôlei”; 22 jogam “Futevôlei” e basquete; 18
03. (EFOMM 2014) Denotaremos por n (X) o número de elementos jogam vôlei e basquete; 11 jogam as 3 modalidades. O número de
de um conjunto finito X. Sejam A, B, C conjuntos tais que pessoas que jogam “Futevôlei” é igual ao número de pessoas que
n  A  B  14 , n  A  C  14 e n B  C  15 , n  A  B  C  17 jogam basquete. O número de pessoas que jogam “Futevôlei” ou
basquete e não jogam vôlei é:
e n  A  B  C  3 . Então n  A   n B  n C é igual a:
a) 55
a) 18 d) 29
b) 56
b) 20 e) 32
c) 57
c) 25
d) 58
04. (ESPCEX 2014) Uma determinada empresa de biscoitos realizou e) 59
uma pesquisa sobre a preferência de seus consumidores em relação
a seus três produtos: biscoitos cream cracker, wafer e recheados. Os 08. (EPCAR 1986) Um conjunto A tem n elementos e p subconjuntos
resultados indicaram que: e um conjunto B tem 3 elementos a mais do que o conjunto A. Se q é
• 65 pessoas compram cream crackers. o número de subconjuntos de B, então:
• 85 pessoas compram wafers. a) q = 3p d) p 1
=
b) p = 8q q 8
• 170 pessoas compram biscoitos recheados.
c) p=q+8 e) q = p + 8
• 20 pessoas compram wafers, cream crackers e recheados.
• 50 pessoas compram cream crackers e recheados.
09. (EN 2010) Os 36 melhores alunos do Colégio Naval submeteram-
• 30 pessoas compram cream crackers e wafers. se a uma prova de 3 questões para estabelecer a antiguidade militar.
• 60 pessoas compram wafers e recheados. Sabendo que dentre esses alunos, 5 só acertaram a primeira questão,
• 50 pessoas não compram biscoitos dessa empresa. 6 só acertaram a segunda, 7 só acertaram a terceira, 9 acertaram
a primeira e a segunda, 10 acertaram a primeira e a terceira, 7
Determine quantas pessoas responderam essa pesquisa. acertaram a segunda e a terceira e, 4 erraram todas as questões,
a) 200 podemos afirmar que o número de alunos que não acertaram todas
as 3 questões é igual a:
b) 250
a) 6
c) 320
b) 8
d) 370
c) 26
e) 530
d) 30
e) 32

40

PM_BOOK16 - MAT.indb 40 25/11/2022 19:07:06


CONJUNTOS

10. Se A, B, C forem conjuntos tais que n(A ∪ B) = 23, n(B – A) = 12,


n(C – A) = 10, n(B ∩ C) = 6 e n(A ∩ B ∩ C) = 4, então n(A), n(A ∪ C), GABARITO
n(A ∪ B ∪ C), nesta ordem,
EXERCÍCIOS DE FIXAÇÃO
a) formam uma progressão aritmética de razão 6.
01. A 04. C 07. A 10. C
b) formam uma progressão aritmética de razão 2.
02. C 05. C 08. D
c) formam uma progressão aritmética de razão 8, cujo primeiro
03. E 06. A 09. A
termo é 11.
EXERCÍCIOS DE TREINAMENTO
d) formam uma progressão aritmética de razão 10, cujo último
termo é 31. 01. D 08. D 15. D 22. B
e) não formam uma progressão aritmética. 02. B 09. B 16. A 23. A
03. D 10. A 17. D 24. D
04. D 11. B 18. C 25. 5
05. C 12. C 19. D

DESAFIO PRO 06. C


07. D
13. A
14. D
20. C
21. C
EXERCÍCIOS DE COMBATE

1 (ITA) Sejam A um conjunto com 8 elementos e B um


conjunto tal que A ∪ B contenha 12 elementos. Então, o
número de elementos de P(B\A) ∪ P( ∅ ) é igual a:
01. A
02. D
04. B
05. D
07. E
08. D
10. D

03. D 06. C 09. D


a) 8. c) 20. e) 9.
DESAFIO PRO
b) 16. d) 17.
01. B 04. C
05. ( S1 ∩ S2 ) ⊂ (P1 ∩ P2 )
2 (ITA) Denotemos por n(X) o número de elementos de 02. D
um conjunto finito X. Sejam A, B e C conjuntos tais que 03. C
n(A ∪ B) = 8, n(A ∪ C) = 9, n(B ∪ C) = 10, n(A ∪ B ∪ C) = 11
e n(A ∩ B ∩ C) = 2. Então, n(A) + n(B) + n(C) é igual ANOTAÇÕES
a) 11. c) 15. e) 25.
b) 14. d) 18.

3 (ITA) Sejam A, B e C subconjuntos de um conjunto universo


U. Das afirmações:
I. (A \ Bc ) \ CC = A ∩ (B ∪ C);
II. (A \ Bc ) \ C = A ∪ (B ∩ CC )C ;
III. Bc ∪ CC =(B ∩ C)C ,

É(são) sempre verdadeira(s) apenas:


a) I.
b) II.
c) III.
d) I e III.
e) II e III.

4 (ITA) Sejam A, B e C subconjuntos de um conjunto universo


U. Das afirmações:
I. A \ (B ∩ =
C) (A \ B) ∪ (A \ C);
II. (A ∩ C) \ B = A ∩ BC ∩ C;
III. (A \ B) ∩ (B \ C) =
(A \ B) \ C,
É(são) verdadeira(s):
a) apenas I.
b) apenas II.
c) apenas I e II.
d) apenas I e III.
e) todas.

5 (IME) Sejam os conjuntos P1, P2, S1 e S2 tais que


(P2 ∩ S1 ) ⊂ P1, (P1 ∩ S2 ) ⊂ P2 ( S1 ∩ S2 ) ⊂ (P1 ∪ P2 ) . Demonstre
que S1 ∩ S2 ) ⊂ (P1 ∩ P2 ) .
(

41

PM_BOOK16 - MAT.indb 41 25/11/2022 19:07:07


CONJUNTOS

ANOTAÇÕES

42

PM_BOOK16 - MAT.indb 42 25/11/2022 19:07:07


TEORIA DOS NÚMEROS

DIVISIBILIDADE Resto na divisão por 5: resto do último algarismo na divisão por 5.


Neste momento inicial, nosso interesse será em determinar Exemplo:
quando a divisão entre dois números inteiros é exata, ou seja, quando 995 é múltiplo de 5, pois o último algarismo é 5, mas 1003 não é
o resto da divisão é 0. múltiplo de 5, pois o último algarismo é 3. O resto de 1003 na divisão
Antes de mais nada, vamos à definição crucial desta parte: por 5 é 3.

DEFINIÇÃO DIVISIBILIDADE POR 6


Um número é múltiplo de 6 se, e somente se, é múltiplo de 2 e
Dizemos que o inteiro a é divisível pelo inteiro b (ou ainda que a é
de 3.
múltiplo de b) se existe um inteiro c tal que a = bc.
Resto na divisão por 6: resto por 6 da soma do algarismo das
Exemplos:
unidades com o quádruplo da soma dos algarismos anteriores.
15 é múltiplo de 3
Exemplo:
33 é múltiplo de 11
6|120, mas 722 não é múltiplo de 6. O resto de 722 na divisão por
17 não é múltiplo de 2 6 é o resto de 2 + 4 (7 + 2) = 2 + 36 = 38 na divisão por 6, que é 2.
Vejamos agora alguns critérios de divisibilidade importantes que
ajudam a ganhar tempo em diversos problemas. DIVISIBILIDADE POR 7
Um número é múltiplo de 7 se, e somente se, a soma das classes
CRITÉRIOS DE DIVISIBILIDADE ímpares menos a soma das classes pares é múltipla de 7.
Resto na divisão por 7: resto da soma das classes ímpares menos
DIVISIBILIDADE POR 2 a soma das classes pares na divisão por 7.
Um número é múltiplo de 2 se, e somente se, seu último algarismo Exemplo:
é par. 7|1638931720888, pois a soma das classes ímpares é
Resto na divisão por 2: se o último algarismo é par, o resto é 0 e 888 + 931 + 1 = 1820 e a soma das classes pares é 720 + 638 = 1358,
se o último algarismo é ímpar, o resto é 1. donde a diferença é 462 = 66 ⋅ 7, que é múltipla de 7.
Exemplo:
2344 é múltiplo de 2, pois 4 é par, mas 31441 não é múltiplo de DIVISIBILIDADE POR 8
2, pois 1 é ímpar. O resto de 31441 na divisão por 2 é 1. Um número é múltiplo de 8 se, e somente se, o número formado
pelos três últimos algarismos é múltiplo de 8.
DIVISIBILIDADE POR 3 Resto na divisão por 8: resto do número formado pelos três
Um número é múltiplo de 3 se, e somente se, a soma de seus últimos algarismos na divisão por 8.
algarismos é múltipla de 3. Exemplo:
Resto na divisão por 3: resto da soma dos algarismos do número 271824 é múltiplo de 8, pois 824 é múltiplo de 8, mas 31442 não
na divisão por 3. é múltiplo de 8, pois 442 = 55 ⋅ 8 + 2, donde o resto de 31442 na
Exemplo: divisão por 8 é 2.
3459 é múltiplo de 3, pois 3 + 4 + 5 + 9 = 21 e 2 + 1 = 3, que é
múltiplo de 3, mas 121 não é múltiplo de 3, pois 1 + 2 + 1 = 4 não o DIVISIBILIDADE POR 9
é. O resto de 121 na divisão por 3 é 1. Um número é múltiplo de 9 se, e somente se, a soma de seus
algarismos é múltipla de 9.
DIVISIBILIDADE POR 4 Resto na divisão por 9: resto da soma dos algarismos do número
Um número é múltiplo de 4 se, e somente se, o número formado na divisão por 9.
por seus dois últimos algarismos é múltiplo de 4. Exemplo:
Resto na divisão por 4: resto do número formado pelos dois 1233 é múltiplo de 9, pois 1 + 2 + 3 + 3 = 9 é múltiplo de 9, mas
últimos algarismos na divisão por 4. 727 não é múltiplo de 9, pois 7 + 2 + 7 = 16 e 16 = 9 ⋅ 1 + 7, donde
Exemplo: o resto de 727 na divisão por 9 é 7.
15684 é múltiplo de 4, pois 84 é múltiplo de 4, mas 14234 não é
múltiplo de 4, já que 34 não o é. O resto de 14234 na divisão por 4 é 2. DIVISIBILIDADE POR 10
Um número é múltiplo de 10 se, e somente se, seu algarismo das
DIVISIBILIDADE POR 5 unidades é 0.
Um número é múltiplo de 5 se, e somente se, seu último algarismo Resto na divisão por 10: o resto de um número na divisão por 10
é 0 ou 5. é o algarismo das unidades.

43

PM_BOOK16 - MAT.indb 43 25/11/2022 19:07:08


TEORIA DOS NÚMEROS

Exemplo: Observação
880 é múltiplo de 10, pois 880 termina em 0, mas 1003 não é
RESTO DE POTÊNCIAS:
múltiplo de 10 e seu resto na divisão por 10 é 3.
Para calcular o resto de am na divisão por N, primeiramente
determina-se um expoente d tal que ad deixa resto 1 na divisão por
DIVISIBILIDADE POR 11
N. Assim, se m = dq + r, 0 ≤ r < d, o resto de am na divisão por N
Um número é múltiplo de 11 se, e somente se, a diferença entre será o resto de adq+r = adq . ar na divisão por N. Como ad deixa resto
a soma dos algarismos de ordem ímpar e a soma dos algarismos de 1, adq também deixará resto 1. Dessa forma, o resto de am é igual
ordem par é múltipla de 11. ao resto de ar na divisão por N.
Resto na divisão por 11: resto da diferença entre a soma dos Exemplos:
algarismos de ordem ímpar e a soma dos algarismos de ordem par
1. Calcular o resto de 15698 na divisão por 11.
na divisão por 11.
Como 156 deixa resto 2 na divisão por 11, basta calcular o resto
Exemplo:
de 298 na divisão por 11.
407 é múltiplo de 11, pois 7 + 4 – 0 = 11 é múltiplo de 11, mas
Temos que 21 deixa resto 2, 22 deixa resto 4, 23 deixa resto 8, 24
300 não é múltiplo de 11, pois 0 + 3 – 0 = 3 não o é. Seu resto na
deixa resto 5, 25 deixa resto 10, 26 deixa resto 9, 27 deixa resto 7, 28
divisão por 11 é 3.
deixa resto 3, 29 deixa resto 6 e 210 deixa resto 1. Assim, 298 deixará
o mesmo resto que 28, pois 98 = 10 . 9 + 8. Como 28 deixa resto 3,
DIVISIBILIDADE POR 2K temos que 15698 deixa resto 3 na divisão por 11.
Um número é múltiplo de 2k se, e somente se, o número formado
pelos seus k últimos algarismos é múltiplo de 2k. 2. Calcular o resto de 392012 na divisão por 7.
Resto na divisão por 2k: resto do número formado pelos k últimos Temos que 39 deixa resto 4 na divisão por 7. Então, basta calcular o
algarismos na divisão por 2k. resto de 42012 na divisão por 7. Temos que 41 deixa resto 4, 42 deixa
resto 2 e 43 deixa resto 1. Assim, 42012 deixará o mesmo resto que 42,
pois 2012 = 670 . 3 + 2. Assim, 392012 deixa resto 2 na divisão por 7.
DIVISIBILIDADE POR 5K
Um número é múltiplo de 5k se, e somente se, o número formado
pelos seus k últimos algarismos é múltiplo de 5k.
Resto na divisão por 5k: resto do número formado pelos k últimos Exercício Resolvido
algarismos na divisão por 5k .
01. (CN) O número 583ab é divisível por 9. O valor máximo da
soma dos algarismos a e b é:
DIVISIBILIDADE POR 10 K
a) indeterminado
Um número é múltiplo de 10k se, e somente se, seus k últimos
b) 20
algarismos são zeros.
c) 18
Resto na divisão por 10k: o resto é o número formado pelos k
últimos algarismos. d) 11
e) 2
PROPRIEDADES IMPORTANTES Resolução: D
I. O resto de uma soma na divisão por um determinado número
Para que 583ab seja divisível por 9, a soma de seus algarismos
N é igual ao resto da soma dos restos de cada parcela na
deve ser divisível por 9. Assim, temos que 5 + 8 + 3 + a + b – a + b
divisão por N.
+ 16 é divisível por 9. Veja ainda que a,b ≤ 9 (pois são algarismos
Exemplo: de um número). Desta forma, temos que a + b é, no máximo, 18.
Calcular o resto de 123 + 441 + 1829 na divisão por 5. Para a + b + 16 ser divisível por 9, podemos ter a + b igual a 2 ou
11 (qualquer outro valor passaria de 18). Então o valor máximo
Basta calcular o resto de 3 + 1 + 4 = 8 na divisão por 5, que é 3.
pedido é 11.
II. O resto de um produto na divisão por um determinado
número N é igual ao resto do produto dos restos de cada
fator na divisão por N.
Exemplo:
FATORAÇÃO EM NÚMEROS PRIMOS
Vejamos uma definição inicial:
Calcular o resto de 765 · 423 · 112 na divisão por 11.
Basta calcular o resto de 6 ⋅ 5 ⋅ 2 = 60 na divisão por 11, que é 5.
DEFINIÇÃO
III. Para achar o resto de uma potência na divisão por um Um número inteiro p é dito um número primo quando possui
determinado número N, calcula-se primeiramente o resto exatamente quatro divisores inteiros: ±p, ±1. Por outro lado, um
da base na divisão por N e depois eleva-se esse resto ao inteiro é dito composto quando não é primo.
expoente dado. Obtido esse número, calcula-se o resto na Exemplo:
divisão por N.
2,3,5,7,11 são números primos, mas 6,15,21,42 são números
Exemplo: compostos.
Calcular o resto de 349 na divisão por 7.
O resto de 34 na divisão por 7 é 6. Então, precisamos calcular Observação
o resto de 69 na divisão por 7. Mas 62 = 36 deixa resto 1 na O único número primo par é o 2.
divisão por 7. Assim, 68 deixa resto 1 na divisão por 7 e,
portanto 69 = 68 ⋅ 61 deixa resto 6 na divisão por 7.
O resultado mais importante referente a números primos é o
chamado teorema fundamental da Aritmética:

44

PM_BOOK16 - MAT.indb 44 25/11/2022 19:07:08


TEORIA DOS NÚMEROS

TEOREMA FUNDAMENTAL DA ARITMÉTICA Exercício Resolvido


Todo inteiro maior do que 1 pode ser expresso de maneira única
(a menos da ordem) como produto de fatores primos. 03. Seja N = 24 · 35 · 56. O número de divisores naturais de N que
são múltiplos de 10 é:
Exemplo:
a) 24
5544 = 23 ⋅ 32 ⋅ 7 ⋅ 11
b) 35
c) 120
DIVISORES DE UM NÚMERO
d) 144
Os divisores de um número inteiro n são todos os inteiros m tais
que n é múltiplo de m. e) 210

Observação Resolução: D
Um divisor natural genérico de N é da forma 2a ⋅ 3b ⋅ 5c, onde
Se a = p1α1p2α2...pkαk, um divisor natural de a é da forma p1θ1p2θ2...
0 ≤ a ≤ 4,0 ≤ b ≤ 5,0 ≤ c ≤ 6. Para que tal divisor seja múltiplo
pkθk, onde 0 < βi < αi para todo 1 ≤ i ≤ k. Este fato permite, através
de 10, a e c devem ser maiores ou iguais a 1. Desta forma, há
do princípio multiplicativo da Análise Combinatória, calcular a
4 possíveis valores para a (1, 2, 3, 4), 6 possíveis valores para b
quantidade de divisores de um determinado número.
(0,1,2,3,4,5) e 6 possíveis valores para c (1,2,3,4,5,6). Assim, o
Exemplo: total de divisores naturais de N múltiplos de 10, pelo princípio
Os divisores de 30 formam o conjunto {±1, ±2, ±3, ±5, ±6, ±10, multiplicativo, é 4 ⋅ 6 ⋅ 6 = 144.
±15, ±30}.

MÁXIMO DIVISOR COMUM (MDC)


Observação
Um número n é dito um quadrado perfeito se existe k ∈  tal que DEFINIÇÃO
n = k2. É fácil demonstrar que um número é um quadrado perfeito Dados dois inteiros a e b não nulos, o máximo divisor comum de
se, e somente se, o expoente de cada primo em sua fatoração é a e b (denotado por mdc(a,b)) é o maior inteiro positivo que divide a
um número par. e b simultaneamente.
Um número n é dito um cubo perfeito se existe k ∈  tal que Exemplo:
n = k3. É fácil demonstrar que um número é um cubo perfeito se, Os divisores naturais de 30 são: 1, 2, 3, 5, 6, 10, 15, 30
e somente se, o expoente de cada primo em sua fatoração é um Os divisores naturais de 12 são: 1, 2, 3, 4, 6, 12
número múltiplo de três. Veja que o maior inteiro que aparece nas duas listas é o 6. Assim,
mdc(30,12) = 6.

Observação
NÚMERO DE DIVISORES
Se n = p1α1p2α2...pkαk, o número de divisores naturais de n é dado Dois números a e b com mdc igual 1 a são ditos primos entre si.
por d(n) = (α1 + 1)(α2 + 1)...(αk + 1). Para achar o número de divisores
inteiros de n, basta multiplicar d(n) por 2. Você deve estar se perguntando agora: será que existe uma
Exemplo: maneira mais prática de calcular o mdc de dois números? A resposta
Consideremos o número 22680 = 23 ⋅ 34 ⋅ 5 ⋅ 7. é SIM. Vejamos agora como:
O número de divisores naturais é (3 + 1)(4 + 1)(1 + 1)(1 + 1) = 80.

Exercício Resolvido ALGORITMO DE EUCLIDES


Dados A e B (A > B), dividimos A por B, obtendo quociente q1 e
02. Se P é o produto de todos os números primos menores que resto r1. Colocamos q1 acima de B e r1 abaixo de A e ao lado de B (r1
1000, o algarismo das unidades de P é: será agora o novo divisor e o novo dividendo é B). Feito isso, dividimos
a) 0 B por r1, obtendo quociente q2 e resto r2. Colocamos q2 acima de r1
e r2 abaixo de B e ao lado de r1 (r2 será agora o novo divisor e o novo
b) 1
dividendo é r1). Repetimos este processo até obter resto 0. O último
c) 2 divisor, rn, será o mdc.
d) 5 O método acima normalmente é apresentado através do dispositivo
e) 9 de cálculo a seguir:

Resolução: A q1 q2 q3 ... qn qn+1


Como 2 e 5 são primos menores que 1000, estes dois números
estarão presentes no produto P. Desta forma, P é múltiplo de 10 e A B r1 r2 ... rn–1 rn
assim seu algarismo das unidades é 0.
r1 r2 r3 ... rn 0

Exemplo:
Calcular mdc (665,280).
2 2 1 2
665 280 105 70 35
105 70 35 0
Obtemos assim mdc (665,280) = 35.

45

PM_BOOK16 - MAT.indb 45 25/11/2022 19:07:08


TEORIA DOS NÚMEROS

MÉTODO DA DECOMPOSIÇÃO CANÔNICA Exercício Resolvido


Neste método, encontramos inicialmente a fatoração em primos
dos números de interesse: a = p1α1 · p2α2 · ... · pnαn e b = p1β1 · p2β2 · ... 04. Três satélites artificiais giram em torno da Terra, em órbitas
· pnβn (alguns expoentes podem ser NULOS!). constantes. O tempo de rotação do primeiro é de 42 minutos,
o do segundo de 72 minutos e o do terceiro é de 126 minutos.
Desta forma, temos que mdc(a,b) = p1min{α1,β1} · p2min{α2,β2} · ... Em dado momento, eles se alinham em um mesmo meridiano,
· pnmin{αn,βn}, ou seja, mdc(a,b) é o produto dos fatores primos comuns embora em latitudes diferentes. Eles voltarão em seguida, a passar
às duas decomposições tomados com seus menores expoentes. simultaneamente pelo mesmo meridiano, depois de:
Exemplo: a) 15 h 24 min
665 = 5 ⋅ 7 ⋅ 19 e 280 = 23 ⋅ 5 ⋅ 7. b) 7 h 48 min
Logo, mdc(665,280) = 5 ⋅ 7 = 35.
c) 126 min
ProBizu d) 8 h 24 min
Se mdc(a,b) = d, podemos escrever a = du, b = dv, com u,v primos Resolução: D
entre si.
O primeiro satélite passa pelo meridiano a cada 42 minutos, o
segundo a cada 72 minutos e o terceiro a cada 126 minutos. Desta
forma, temos o seguinte esquema para os tempos nos quais os
MÍNIMO MÚLTIPLO COMUM (MMC) satélites passarão por tal meridiano:
Satélite 1: 42, 84, 126, 168, ...
DEFINIÇÃO Satélite 2: 72, 144, 216, 288, ...
O mmc entre dois inteiros não nulos a e b é o menor inteiro Satélite 3: 126, 252, 378, 504, ...
positivo que é divisível por a e b.
Veja que os tempos nos quais os satélites passam são os múltiplos
Exemplo: de 42, os múltiplos de 72 e os múltiplos de 126. Desta forma,
Os múltiplos positivos de 30 são: 30, 60, 90, 120, 150, ... o primeiro tempo no qual eles passarão juntos novamente pelo
Os múltiplos positivos de 12 são: 12, 24, 36, 48, 60, 72, ... meridiano é o menor número comum às três listas, ou seja, é o
menor múltiplo comum entre 42, 72 e 126.
Veja que 60 é o menor número comum às duas listas. Desta
forma, mmc(30, 12) = 60. Vamos calcular então mmc(42,72,126). Para isso, veja que:
Assim como no mdc, vejamos um método de calcular o mmc 42 = 2 · 3 · 7
entre dois números. 72 = 2³ · 2²
126 = 2 · 3² · 7
Observação
Para calcular o mmc, devemos tomar os fatores primos comuns
O mmc de dois números primos entre si é igual ao produto entre e não comuns elevados ao MAIOR expoente, obtendo assim
estes números. Por exemplo, 3 e 5 são primos entre si e, portanto, mmc(42,72,126) = 23 ⋅ 32 ⋅ 7 = 504.
mmc (3,5) = 3 × 5 = 15.
Com isso, a cada 504 minutos, os satélites estarão juntos no
meridiano. Como 504 minutos equivalem a 8 h 24 min, depois de
8 h 24 min, eles estarão juntos no meridiano.
MÉTODO DA DECOMPOSIÇÃO CANÔNICA
Tal qual no mdc, encontramos primeiramente a fatoração dos
números de interesse: a = p1α1 · p2α2 · ... · pnαn e b = p1β1 · p2β2 · ... · pnβn Exercício Resolvido
(alguns expoentes podem ser NULOS!).
Desta forma, temos que mmc(a,b) = p1min{α1,β1} · p2min{α2,β2} · ... 05. Se mdc(a, b) = 4, mmc(a,b) = 80 e a + b = 36, então o valor
· pn , ou seja, mmc(a,b) é o produto dos fatores primos comuns
min{αn,βn} numérico de 2a – b, onde a > b é:
e não comuns às duas decomposições tomados com seus maiores a) 24
expoentes. b) 16
Exemplo: c) 20
665 = 5 ⋅ 7 ⋅ 19 e 280 = 23 ⋅ 5 ⋅ 7. d) 36
Logo, mmc(665,280) = 23 ⋅ 5 ⋅ 7 ⋅ 19 = 5320.
e) 12
Observação
Resolução: A
O seguinte resultado é bastante útil: dados inteiros a,b não nulos, Como mdc(a,b) = 4, podemos escrever a = 4u, b = 4v, com u e v
vale que ab = mmc(a,b) · mdc(a,b) primos entre si. Desta forma, temos que mmc(a,b) = mmc(4u,4v)
= 4uv = 80 ⇔ uv = 20. Além disso, a + b = 36 ⇔ 4u + 4v = 36 ⇔
u + v = 9. Como a > b, segue que u > v e assim u = 5, v = 4. Daí
a = 20, b = 16 e, portanto, 2a − b = 40 − 16 = 24.

46

PM_BOOK16 - MAT.indb 46 25/11/2022 19:07:08


TEORIA DOS NÚMEROS

EXERCÍCIOS DE 45 segundos, as lâmpadas verdes, a cada 60 segundos, as azuis, a

FIXAÇÃO cada 27 segundos, e as vermelhas só acendem quando as lâmpadas


das outras cores estão acesas ao mesmo tempo. De quantos em
quantos minutos, as lâmpadas vermelhas acendem?
a) 6 c) 12 e) 18
01. Se o resto da divisão do número inteiro positivo b por 7 é igual a 5, b) 9 d) 15
então, o resto da divisão do número b² + b + 1 por 7 é igual a
a) 2 c) 3 07. Sendo n um número natural, n ≠ 0, assinale a alternativa verdadeira.
b) 4 d) 5 a) O número n² + 3 é sempre um número ímpar.
b) O número n³ é sempre divisível por 3.
02. Maria adora séries de televisão e pretende assistir, durante um c) O número n · (n – 1) é sempre ímpar.
ano, a todos os episódios (de todas as temporadas e sem pular
d) O mínimo múltiplo comum entre n e 2n é sempre um número par.
nenhum episódio) das suas três séries preferidas. Para isso, ela assistirá
a três episódios por dia, sendo um de cada série. Sabe-se que cada e) O máximo divisor comum entre n e 2n é 2n.
temporada da série A tem 20 episódios, da série B tem 24 episódios
e da série C tem 18 episódios. Nenhuma das três séries tem mais 08. (FUVEST) Sejam a e b dois números inteiros positivos. Diz-se que
que 365 episódios ao todo. Ela decidiu que começará, hoje, a assistir a e b são equivalentes se a soma dos divisores positivos de a coincide
ao 1º episódio da 1ª temporada de cada uma dessas três séries. com a soma dos divisores positivos de b.
Maria também sabe que haverá um certo dia X em que conseguirá, Constituem dois inteiros positivos equivalentes:
coincidentemente, assistir ao último episódio de alguma temporada
a) 8 e 9. c) 10 e 12. e) 16 e 25.
das três séries. Ao final do dia X, Maria já terá assistido, ao todo,
b) 9 e 11. d) 15 e 20.
a) 12 temporadas completas das três séries.
b) 15 temporadas completas da série A.
09. Qual das alternativas abaixo indica uma afirmação verdadeira?
c) 18 temporadas completas da série B.
a) Todo múltiplo de 7 é um número ímpar.
d) 20 temporadas completas da série C.
b) Todo número ímpar é múltiplo de 7.
c) Todo número par é múltiplo de 8.
03. A respeito de um número natural, sabe-se que:
d) Todo múltiplo de 8 é um número par.
• divisível por 4;
• é múltiplo de 3 e de 7;
10. Sejam a e b algarismos. Existem exatamente N números naturais
• não é múltiplo de 5; de cinco algarismos, da forma 1a79b, que são divisíveis por 15.
• está localizado entre 400 e 550. Tendo isso em vista, o valor de N é
A soma dos algarismos desse número é igual a Lembre-se de que um número natural é divisível por:
a) 8 c) 10 • 3, quando a soma dos seus algarismos for divisível por 3;
b) 9 d) 11 • 5, quando o algarismo das unidades for 0 ou 5.
a) 15 c) 9 e) 2
04. (CMRJ) Uma professora do Colégio Militar do Rio de Janeiro tem
b) 12 d) 6
três filhas matriculadas regularmente numa escola. O produto da
idade da professora com as idades de suas três filhas é 26.455. Desta
forma, pode-se afirmar que a soma das idades da filha mais velha e EXERCÍCIOS DE

TREINAMENTO
da filha mais nova é um
a) número ímpar.
b) número primo.
c) número múltiplo de 3.
d) número múltiplo por 5. 01. Seja N o maior número formado por três algarismos distintos que,
e) número divisível por 7. dividido por 5, deixa resto 2. A soma dos algarismos de N é igual a:
a) 27 c) 25 e) 23
05. Roberto, ao escolher os números de sua aposta numa loteria, b) 26 d) 24
procedeu da seguinte forma:
• 1º Passo: escolheu os números 6, 12 e 20, que são as idades, em 02. (UFMG 99) Um número natural n tem três algarismos, todos não-
anos, de seus três filhos; nulos. A soma dos três algarismos de n é igual a 12 e o quadrado de
• 2º Passo: escolheu mais dois números, que são o MMC e o MDC um desses algarismos é igual à soma dos outros dois. Assinale a única
dos números escolhidos no 1º passo; afirmativa FALSA em relação a essa situação:
• 3º Passo: escolheu a média aritmética dos dois maiores números a) n é sempre múltiplo de 3.
já escolhidos nos dois passos anteriores. b) O produto dos três algarismos de n é sempre menor que 56.
A soma de todos os números escolhidos por Roberto é c) 3 é sempre um dos algarismos de n.
a) 100 c) 140 d) Existem 21 valores possíveis para n.
b) 120 d) 160
03. (CN 2002) Se a e b não forem múltiplos de 13 e 2a + b é divisível
06. Rodrigo estava observando o pisca-pisca do enfeite natalino de por 13, então um número que é múltiplo de 13 é:
sua casa. Ele é composto por lâmpadas nas cores amarelo, azul, verde a) 91a + b c) 93a + b e) 95a + b
e vermelho. Rodrigo notou que lâmpadas amarelas acendem a cada b) 92a + b d) 94a + b

47

PM_BOOK16 - MAT.indb 47 25/11/2022 19:07:09


TEORIA DOS NÚMEROS

04. (CN 2005) Um número natural N deixa: resto 2 quando dividido 13. (EPCAR 2003) Um aluno da EPCAR, indagado sobre o número de
por 3; resto 3 quando dividido por 7; e resto 19 quando dividido por exercícios de matemática que havia resolvido naquele dia respondeu:
41. Qual é o resto da divisão do número k = (N + 1) · (N + 4) · (N + 22) “Não sei, mas contando de 2 em 2 sobra um; contando de 3 em 3
por 861? sobra um; contando de 5 em 5 também sobra um; mas contando de
a) 0 c) 19 e) 43 7 em 7 não sobra nenhum. O total de exercícios não chega a uma
centena”. Então, o número de exercícios resolvidos é tal que a soma
b) 13 d) 33 de seus algarismos é igual a
a) 8 b) 9 c) 10 d) 11
05. (CN 2002) Se a é um número natural, a5 – 5a³ + 4a é sempre
divisível por:
14. (EPCAR 2001) Ao separar o total de suas figurinhas, em grupos de
a) 41 c) 50 e) 72 12, 15 e 24, uma criança observou que sobravam sempre 7 figurinhas.
b) 48 d) 60 Se o total de suas figurinhas está compreendido entre 240 e 360,
pode-se afirmar que a soma dos algarismos significativos desse total é:
06. (CN 2010) Estudando os quadrados dos números naturais, um a) 6 c) 10
aluno conseguiu determinar corretamente o número de soluções
b) 9 d) 13
inteiras e positivas da equação 5x² + 11y² = 876543. Qual foi o
número de soluções que este aluno obteve?
15. (CN 2002) O mínimo múltiplo comum entre dois números naturais
a) 0 c) 2 e) 4 a e b é 360 e ab = 3600. Qual o menor valor que a + b pode assumir?
b) 1 d) 3 a) 120 c) 150 e) 370
b) 130 d) 200
07. O número inteiro positivo N, de dois algarismos, quando dividido
por 15 dá quociente A e resto B e quando dividido por 8 dá quociente
B e resto A. A soma de todos os valores de N é igual a: 16. (EPCAR 2006) Três alunos A, B e C participam de uma gincana
e uma das tarefas é uma corrida em pista circular. Eles gastam para
a) 112 c) 160 e) 336 esta corrida, respectivamente, 1,2 minutos, 1,5 minutos e 2 minutos
b) 144 d) 255 para completarem uma volta na pista. Eles partem do mesmo local e
no mesmo instante. Após algum tempo, os três alunos se encontram
08. (ESPM 2015) O número natural N = 474747...47X possui 47 pela primeira vez no local de partida. Considerando os dados acima,
algarismos e é múltiplo de 9. O valor do algarismo X é: assinale a alternativa correta.
a) 4 c) 3 e) 5 a) Na terceira vez que os três se encontrarem, o aluno menos veloz
b) 7 d) 8 terá completado 12 voltas.
b) O tempo que o aluno B gastou até que os três se encontraram
pela primeira vez foi de 4 minutos.
n2 + 37
09. (CN 2013) O maior inteiro n tal que também é inteiro c) No momento em que os três alunos se encontraram pela segunda
n+5
tem como soma de seus algarismos um valor igual a vez, o aluno mais veloz gastou 15 minutos.
a) 6 c) 10 e) 14 d) A soma do número de voltas que os três alunos completaram
b) 8 d) 12 quando se encontraram pela segunda vez foi 24.

10. (ITA 2013) Seja n > 6 um inteiro positivo não divisível por 6. Se, 17. Três satélites artificiais giram em torno da Terra, em órbitas
na divisão de n2 por 6, o quociente é um número ímpar, então o resto constantes. O tempo de rotação do primeiro é de 42 minutos,
da divisão de n por 6 é o do segundo de 72 minutos e o do terceiro é de 126 minutos.
Em dado momento, eles se alinham em um mesmo meridiano,
a) 1. c) 3. e) 5. embora em latitudes diferentes. Eles voltarão em seguida, a passar
b) 2. d) 4. simultaneamente pelo mesmo meridiano, depois de:
a) 15 h 24 min c) 126 min
11. (EPCAR 2011) Sabe-se que x, y, z são números naturais distintos
b) 7 h 48 min d) 8 h 24 min
e x > y. Considere A = xy e B = (xyz)² e que o mdc (A,B) e o mmc
(A,B) são, respectivamente, 21 e 1764. Se W = x² + y² + z², então o
conjunto formado pelos divisores naturais de W possui 18. Os divisores comuns de 4512 e 4128 são em número de:
a) 4 elementos a) 8 c) 12 e) 96
b) 6 elementos b) 10 d) 48
c) 9 elementos
19. (CN 1977) O MMC de dois números é 300 e o MDC desses
d) 12 elementos números é 6. O quociente entre o maior e o menor desses números:
a) pode ser 2
12. (EPCAR 2007) Assinale a alternativa correta.
b) tem 4 divisores positivos
a) Se x ∈ , y ∈  e x ≠ y ≠ 1 e se x e y são divisíveis por p, então p
é o máximo divisor comum de x e y. c) é um número primo
b) O máximo divisor comum de dois números naturais divide o seu d) tem 6 divisores positivos
mínimo múltiplo comum. e) pode não ser um número inteiro
c) Se x e y são números primos, com x > y > 2, o máximo divisor
comum de x e y é igual a x. 20. (EN 1973) Quantas espécies distintas de polígonos regulares de
100 lados existem?
d) Se o conjunto dos múltiplos do número natural x é subconjunto
do conjunto dos múltiplos do número natural y, então x não é a) 1 c) 20 e) 40
múltiplo de y. b) 10 d) 30

48

PM_BOOK16 - MAT.indb 48 25/11/2022 19:07:09


TEORIA DOS NÚMEROS

21. (EPCAR 2005) Se o mínimo múltiplo comum entre os inteiros 04. (CN 1991) Considere as afirmativas:
a = 16 · 3k (k ≠ 0) e b = 2p · 21 for 672, então, pode-se concluir que I. O número 1147 não é primo.
a) p é divisor de 2p · 21. II. Todo número da forma abba, onde a e b são algarismos, é divisível
b) 3k é divisível por 2p. por 11.
c) pk é múltiplo de 3. III. Todo número múltiplo de 5 e 15 é múltiplo de 75.
d) p – k = 4k. IV. O número de divisores naturais de 576 é divisor de 63.
O número de afirmativas verdadeiras é:
22. (CN 2012) Qual é o menor valor positivo de 2160x + 1680y,
sabendo que x e y são números inteiros? a) 0 d) 3
a) 30 c) 120 e) 480 b) 1 e) 4
b) 60 d) 240 c) 2

23. (CN 2008) De uma determinada quantidade entre 500 e 1000 05. (CN 2014) Um número natural N, quando dividido por 3, 5, 7
DVDs, se forem feitos lotes de 5 DVDs sobram 2; se forem feitos lotes ou 11, deixa resto igual a 1. Calcule o resto da divisão de N por
com 12 DVDs sobram 9 e se forem feitos lotes com 14 DVDs sobram 1155, e assinale a opção correta.
11. Qual é a menor quantidade, acima de 5 DVDs por lote, de modo a) 17 d) 5
a não haver sobra? b) 11 e) 1
a) 6 c) 9 e) 15 c) 7
b) 8 d) 13
06. (EPCAR 2011) Um agricultor fará uma plantação de feijão
24. (CN 2007) Deseja-se revestir uma área retangular, de 198 cm de em canteiro retilíneo. Para isso, começou a marcar os locais onde
comprimento e 165 cm de largura, com um número exato de lajotas plantaria as sementes. A figura abaixo indica os pontos já marcados
quadradas, de tal forma que a medida do lado dessas lajotas, expressa pelo agricultor e as distâncias, em cm, entre eles.
por um número inteiro em cm, seja a maior possível. Quantas lajotas
deverão ser usadas?
a) 27 c) 33 e) 38
b) 30 d) 36
Esse agricultor, depois, marcou outros pontos entre os já existentes,
de modo que a distância d entre todos eles fosse a mesma e a maior
25. (CN 2003) Se x e y são números inteiros e positivos, representa-se possível. Se x representa o número de vezes em que a distância d foi
o máximo divisor comum de x e y por mdc(x,y); assim, o número de obtida pelo agricultor, então x é um número divisível por
x + y = 810
a) 4 c) 6
pares ordenados (x,y) que são soluções do sistema 
é igual a 
mdc ( x,y ) = 45 b) 5 d) 7
a) 6 c) 10 e) 18
b) 8 d) 16 07. (EPCAR 2004) Numa avenida que mede 15750 metros, a partir do
início, a cada 250 m há uma parada de ônibus e a cada 225 m uma de
bonde. A quantidade de pontos comuns de parada de ônibus e bonde
é dada por um número do intervalo
EXERCÍCIOS DE
a) [41, 65] c) [26,40]

COMBATE b) [66,80] d) [0,25]

08. (CN 2008) O mínimo múltiplo comum e o máximo divisor comum


entre os números naturais a, x e b, são respectivamente iguais a 1680
01. Determine a soma dos algarismos do menor número da forma e 120. Sendo a < x < b, quantos são os valores de x que satisfazem
aabbccc que é múltiplo de 836. essas condições?
a) 14 d) 20 a) nenhum
b) 16 e) 22 b) apenas um
c) 18 c) apenas dois
d) apenas três
02. O menor número N de 4 algarismos múltiplo de 12 tal que se ao
somar 3 unidades converte-se em um número múltiplo de 15 e se lhe e) apenas quatro
somarmos mais 3 unidades se converte em um múltiplo de 18. A soma
dos algarismos de N é 09. (CN 2005)
a) 9 d) 18 1 1 2
b) 12 e) 21 A B C 40
c) 15 D E 0

03. Se o número N = 5ab é múltiplo dos números ab mais um resíduo O algoritmo acima foi utilizado para o cálculo do máximo divisor
de 16, a soma dos valores dos números ab é comum entre os números A e B. Logo A + B + C vale
a) 33 d) 22 a) 400 d) 180
b) 44 e) 77 b) 300 e) 160
c) 66 c) 200

49

PM_BOOK16 - MAT.indb 49 25/11/2022 19:07:09


TEORIA DOS NÚMEROS

10. (CN 2005)


a,b,c 2
3 (CN) O número h tem 241 algarismos e h = (z · w)x. O MDC
(x,25), com x natural, resolvido pelo algoritmo das divisões
sucessivas de Euclides, gera o esquema a seguir:
a,x,x 2
a,x,x 2 y 1 4 ← quocientes
a,x,x 3
x 25 z w ← dividendos e divisores
x,x,x 3
x,x,x 3 z w 0 ← restos
x,x,x 5
Sendo assim, é correto afirmar que a soma x + y + z + w é igual a
x,x,1 7
a) 274
1,1,1
b) 224
No algoritmo acima, tem-se a decomposição simultânea em fatores c) 199
primos dos números a, b e c, onde x está substituindo todos os d) 149
números que são diferentes de a, b, c e 1. Analise as afirmativas
abaixo. e) 99
I. a certamente é múltiplo de 36.
II. b certamente é múltiplo de 30.
III. c certamente é múltiplo de 35.
4 (CN) Considere as divisões de números naturais, em que
D é o divisor. A soma de todos os restos possíveis e pares
dessas divisões é 182. Sabendo que D é ímpar e múltiplo de 3,
o resto da divisão de [(2 + 0 + 1 + 5) · 2015]2016 + [(2 + 0 + 1 + 6)
Assinale a opção correta. · 2016]2015 por D é
a) apenas a afirmativa I é falsa. a) 0
b) apenas a afirmativa II é falsa. b) 1
c) apenas a afirmativa III é falsa. c) 2
d) apenas as afirmativas II e III são falsas. d) 15
e) as afirmativas I, II e III são falsas. e) 16

5 (CN) Os números x e y pertencem ao conjunto


C = {17,20,23,29,...,2018} e são tais que x > y. Sendo assim,
pode-se concluir que 2017 · 2x + 8y, na divisão por 7, deixa resto

DESAFIO PRO a) 0
b) 1
c) 3

1 (CN) Considere que N seja um número natural formado


apenas por 200 algarismos iguais a 2, 200 algarismos iguais
a 1 e 2015 algarismos iguais a zero. Sobre N, pode-se afirmar
d) 4
e) 5
que:
a) se forem acrescentados mais 133 algarismos iguais a 1, e GABARITO
dependendo das posições dos algarismos, N poderá ser um EXERCÍCIOS DE FIXAÇÃO
quadrado perfeito.
01. C 04. C 07. D 10. D
b) independentemente das posições dos algarismos, N não é
um quadrado perfeito. 02. D 05. C 08. E
c) se forem acrescentados mais 240 algarismos iguais a 1, e 03. B 06. B 09. D
dependendo das posições dos algarismos, N poderá ser um EXERCÍCIOS DE TREINAMENTO
quadrado perfeito. 01. D 08. D 15. B 22. D
d) se os algarismos da dezena e da unidade não forem iguais a 02. B 09. D 16. D 23. C
1, N será um quadrado perfeito.
03. C 10. C 17. D 24. B
e) se forem acrescentados mais 150 algarismos iguais a 1, e
04. A 11. A 18. C 25. A
dependendo das posições dos algarismos, N poderá ser um
quadrado perfeito. 05. D 12. B 19. D
06. A 13. C 20. C

2 (CN) Sejam A = {1,2,3,...,4029,4030} um subconjunto dos


números naturais e B ⊂ A, tal que não existem x e y,
x ≠ y, pertencentes a B nos quais x divida y. O número máximo de
07. D 14. C
EXERCÍCIOS DE COMBATE
21. D

elementos de B é N. Sendo assim, a soma dos algarismos de N é 01. E 04. D 07. D 10. E
a) 8 02. B 05. E 08. C
b) 9 03. E 06. D 09. A
c) 10 DESAFIO PRO
d) 11 01. B 03. D 05. E
e) 12 02. A 04. B

50

PM_BOOK16 - MAT.indb 50 25/11/2022 19:07:10


FUNÇÃO - CONCEITOS

PAR ORDENADO E PLANO CARTESIANO Exemplos:


O conceito de par ordenado, denotado por (a,b), é um conceito A = {0,2} e B = {1,3,5}
primitivo sendo um conjunto de dois elementos ordenados. A × B = {(0,1); (0,3); (0,5); (2,1); (2,3); (2,5)}
Dois pares ordenados são iguais se, e somente se, as suas duas B × A = {(1,0); (1,2); (3,0); (3,2); (5,0); (5,2)}
coordenadas são iguais. n(A × B) = n(B × A) = 2 · 3 = 6
(a,b) = (c,d) ⇔ a = c ∧ b = d É possível representar o produto cartesiano graficamente através
de um diagrama de flechas.
Os pares ordenados podem ser representados graficamente em Sendo A = {1,2,3} e B = {1,2,3,4}, então A × B = {(1,1); (1,2);
um plano cartesiano ortogonal, onde o primeiro elemento do par (1,3); (1,4); (2,1); (2,2); (2,3); (2,4); (3,1); (3,2); (3,3); (3,4)} terá a
ordenado é representado no eixo horizontal Ox (eixo das abscissas) e representação abaixo.
o segundo elemento do par ordenado é representado no eixo vertical
Oy (eixo das ordenadas). Isso pode ser observado na figura a seguir:

D(R) ⊂ A e Im(R) ⊂ B

O produto cartesiano pode ser representado graficamente no


plano cartesiano ortogonal, através da representação dos pares
ordenados que o compõem.
A representação gráfica é útil também para apresentar o resultado
Assim, dizemos que o par ordenado (a,b) possui abscissa a e
do produto cartesiano entre intervalos reais.
ordenada b.
Exemplo 1:

PRODUTO CARTESIANO A = {1,2,3} e B = {1,2}


O produto cartesiano de dois conjuntos A e B é o conjunto
de todos os pares ordenados que têm o primeiro termo em A e o
segundo termo em B.
A × B = {(x,y) | x ∈ A ∧ y ∈ B}

Se um dos conjuntos for vazio, o produto cartesiano é vazio, ou


seja, ∅ × B = A × ∅ = ∅ × ∅ = ∅.
O produto cartesiano não é comutativo, assim A × B ≠ B × A,
quando A ≠ B.
O produto cartesiano de um conjunto por ele mesmo é denotado
por A² = A × A.
No caso de dois conjuntos finitos, o número de elementos do
produto cartesiano pode ser obtido multiplicando a quantidade de
elementos de cada um deles.
n(A × B) = n(A) · n(B)

51

PM_BOOK16 - MAT.indb 51 25/11/2022 19:07:10


FUNÇÃO - CONCEITOS

RELAÇÃO
Uma relação R de A em B é qualquer subconjunto de A × B.
Quando R é uma relação de A em A, diz-se apenas que R é uma
relação em A.
Numa relação de A em B, A é chamado conjunto de partida
e B, conjunto de chegada. O conjunto de todas as primeiras
coordenadas que pertencem a R é chamado domínio e o conjunto
de todas as segundas coordenadas que pertencem a R é chamado
imagem, ou seja, o domínio e a imagem são formados por elementos
que efetivamente estão em algum par ordenado da relação.

D(R) ⊂ A e Im(R) ⊂ B
Exemplo:
Sejam os conjuntos A = {1,2,3,4} e B = {2,4,5,6,8,10}, então o
conjunto R = {(1,2);(2,4);(2,8)} é uma relação de A em B, ou seja,
R: A → B, cujo domínio é D(R) = {1,2} e a imagem Im(R) = {2,4,8}.
Exemplo 2:
A = [1,3] e B = [1,5[

FUNÇÃO
Observação
Seja f uma relação de A em B, isto é, f ⊂ A × B, dizemos que f
é uma função de A em B se, e somente se, para todo elemento
x ∈ A existe um e apenas um elemento y ∈ B tal que (x,y) ∈ f, ou
seja, y = f(x).

Portanto, para que uma relação de A em B seja uma função,


exige-se que a cada x ∈ A esteja associado um único y ∈ B. Entretanto
pode existir y ∈ B que não esteja associado a nenhum elemento
pertencente ao conjunto A ou que esteja associado a mais de um
elemento de A.
Os dois diagramas seguintes representam relações de A em B,
mas não funções de A em B. O primeiro porque existe um elemento
de A que não está associado a nenhum elemento de B e o segundo
porque existe um elemento de A que está associado a mais de um
elemento de B.

Propriedades do produto cartesiano:


A × (B ∪ C) = (A × B) ∪ (A × C)
A × (B ∩ C) = (A × B) ∩ (A × C)
A × (B – C) = (A × B) – (A × C)

52

PM_BOOK16 - MAT.indb 52 25/11/2022 19:07:11


FUNÇÃO - CONCEITOS

Exemplo:
Qual o domínio das funções a seguir?
1
a) f  x  
x 1
b) g  x   x 1
Vamos identificar o domínio de validade dessas funções, ou seja, o
maior subconjunto dos reais para os quais as funções estão definidas.
a) x + 1 ≠ 0 ⇔ x ≠ 1 ⇒ Df =  – {–1}
b) x + 1 ≥ 0 ⇔ x ≥ –1 ⇒ Dg = [–1,+∞]
O diagrama de flechas a seguir representa uma relação de A em B
que também é uma função de A em B. As funções reais de variável real podem ser representadas
graficamente no plano cartesiano ortogonal. O gráfico da função
é composto por todos os pares ordenados que compõem a função.

Observação
Em virtude da definição de função, toda reta vertical, que passa
por um ponto do domínio, intersecta o gráfico da função em
exatamente um ponto.

Exemplo 1:
A relação dada por x2 + y2 = 9 não é função, pois há retas verticais
que intersectam o seu gráfico em dois pontos.

Domínio de f: D(f) = A
Contradomínio de f: B
Imagem de f: Im(f) ⊂ B

A análise do diagrama de flechas permite concluir que: f(a) = d,


f(b) = d e f(c) = e.
O conjunto A é o domínio da função f e os seus elementos são
os primeiros termos dos pares ordenados que constituem a função e a
origem das flechas no diagrama.
O conjunto B é o contradomínio de f e os seus elementos são
os segundos termos dos pares ordenados do produto cartesiano e os
possíveis destinos das flechas.
O conjunto imagem é um subconjunto de B formado pelos
elementos que são segundos termos dos pares ordenados da função
ou o conjunto dos elementos que são efetivamente destino de flechas.
No diagrama anterior se deve observar que deve partir exatamente
uma flecha de todo elemento do conjunto A. Já os elementos do Exemplo 2:
conjunto B podem receber uma ou mais flechas ou até não receber A relação y = x2 é uma função, pois as retas verticais traçadas a
nenhuma flecha. partir de pontos do domínio intersectam o gráfico em exatamente um
Assim, o fato de d ∈ B ser imagem de dois elementos a e b de A ponto, ou seja, cada elemento do domínio possui exatamente uma
não é empecilho para que f seja uma função de A em B. imagem.

Notação:
f:A →B ou f = {(x,y) ∈ A × B | y = f(x)}
x → f(x)

Chamam-se funções reais de variável real, aquelas cujo domínio e


contradomínio são subconjuntos dos reais, ou seja, os pares ordenados
(x,y) ∈ ².
Nesse caso, costuma-se por comodidade definir a função apenas
pela “regra de correspondência” e adota-se como domínio o maior
subconjunto possível de .

Observação
Em exercícios em que se pede o cálculo do domínio de uma função,
em geral, o que se deseja é que seja apresentado o domínio
máximo da função, ou seja, o maior conjunto para o qual aquela A análise do gráfico da função permite identificar o seu domínio e
função está definida. a sua imagem, como pode ser visto a seguir:

53

PM_BOOK16 - MAT.indb 53 25/11/2022 19:07:13


FUNÇÃO - CONCEITOS

y EXERCÍCIOS DE

FIXAÇÃO
r
d y = f(x)
01. (EAM) Sejam os conjuntos A = {x ∈ ;1 ≤ x ≤ 4} ,
B = {y ∈ ;3 ≤ x ≤ 7} . Considerando o conjunto A × B, (A cartesiano
B) pode-se afirmar que a diagonal do polígono formado por esse
conjunto é representada numericamente por:
a) 2 c) 4 e) 6
P b) 3 d) 5

02. (CMRJ) A alternativa que representa o gráfico do conjunto B × A


c
{
onde A = {2,3,4} e B = x ∈  2 ≤ x ≤ 4 :}
0 a b x
Domínio = [a,b] a) d)
Conjunto Imagem = [c,d]

Zero ou raiz da função f é o número x ∈ D(f), cuja imagem é nula,


isto é, f(x) = 0. Esses pontos são os pontos onde o gráfico da função f
intersecta o eixo das abscissas (Ox).
É possível também identificar o sinal da função em cada trecho do
domínio. Os pontos de imagem positiva encontram-se acima do eixo
das abscissas (parte positiva do eixo das ordenadas) e os de imagem
negativa abaixo (parte negativa do eixo das ordenadas).
b) e)
y

y = f(x)

yB B c)

+ +
xA
x0 yA x1 xB x
A –

No gráfico acima, x0 e x1 são raízes. O ponto A tem ordenada


yA < 0 e o ponto B ordenada yB > 0. Além disso, f(x) > 0 para x < x0 ou
x > x1, e f(x) < 0 para x0 < x < x1.
Uma função f pode ser definida por várias sentenças 03. (CMRJ) Cada figura abaixo, mostra uma relação binária de
abertas, cada uma das quais associada a um domínio Di ⊂ Df. A = {1,2,3,4} em B = {5,6,7,8}.
Exemplo:
1 para x  0

f:  →  tal que f  x   2 para 0  x  1
1 para x  1


Duas funções f e g são iguais se, e somente se, têm o mesmo


domínio e f(x) = g(x) para todo x no domínio. Isso é equivalente
a dizer que todos os pares ordenados que compõem as funções
são iguais.

x2  1
Assim, f(x) = x + 1 e g  x   são diferentes, pois Df =  ≠
x 1
x 1
2
, se x  1
 – {1} = Dg. Por outro lado, f(x) = x + 1 e h  x    x  1 são
 2 , se x  1

iguais, pois possuem o mesmo domínio  e o mesmo valor em todos
os pontos.

54

PM_BOOK16 - MAT.indb 54 25/11/2022 19:07:15


FUNÇÃO - CONCEITOS

Neste caso, podemos afirmar que: EXERCÍCIOS DE


a) (I), (II) e (III) são funções de A em B.
b) (I), (II) e (III) não são funções de A em B. TREINAMENTO
c) Somente (III) é uma função de A em B.
d) Somente (II) não é uma função de A em B. 01. (AFA) Sejam a e b números naturais diferentes de zero.
e) Somente (I) não é uma função de A em B. I. Se f é uma função tal que f(a + b) = f(a) + f(b), então f(ab) = af(b)
1 1
04. (EPCAR) Qual dos gráficos NÃO representa uma função? II. Se log (a + b) = log a + log b, então + =1
a) c) a b
III. Se para todo x real a função f(x – 1) =, então f   = f  
a b
b a
Considerando (V) verdadeiro e (F) falso, as assertivas acima são,
respectivamente:
a) V, V, V c) V, F, F
b) F, V, V d) V, V, F

02. (AFA) Os números inteiros do domínio da função real


b) d)
f(x) = (5 + 2x) ⋅ ( 2 − 3x ) são as raízes da equação g(x) = 0. Uma
expressão analítica da função g(x) é
a) x2 + x2 +2x c) x3 – 3x2 + 2x
b) x + x – 2x
3 2
d) x3 + 3x2 + 2x

1 1
03. (ESPCEX) O domínio da função =
real y − é
x+3 5−x

a) ]–3, 5[ c) ]–5, 3[
05. (CMRJ) Considere a função f :  → , tal que:
b) ]–3, +∞[ d) ]−∞,3[ ∪ ]5, + ∞[
 1, se x é racional 
f(x) =  
 −1, se x é irracional  04. (ESPCEX) Seja f : R → R uma função tal que –2 ≤ f(x) < 5 e
g : R → R dada por g(x) = 1 – f(x). Então o conjunto imagem da função
 1
O valor de f   + f ( π ) + f ( 2,1313...) − f
 2
( 2 ) + f (3,14 ) é: g(x) é:
a) ]–4, 3] c) ]–4, 3[ e) ]–3, 4]
a) 0 c) 2 e) 4 b) [–4, 3] d) [–3, 4[
b) 1 d) 3
x 1
05. Seja a função f definida por: f(x) = + . O domínio
06. (EEAR) Seja f: R → R uma função. Essa função pode ser x2 − 4 9 − x2
da função f é:
a) f(x) = √x b) f(x) = |x| c) f(x) = 1/x d) fx = 11 + x
a) {x ∈  : – 3 < x < – 2} ∪ {x ∈  : 2 < x < 3}
b) {x ∈  : x > 2}
07. (EEAR) Dada a função f(x – 1) = x + 3x – 2, considerando os
2

valores de f(1) e f(2), pode-se afirmar corretamente que Parte superior c) {x ∈  : x < 3}
do formulário d) {x ∈  : – 3 < x < – 2} ∩ {x ∈  : 2 < x < 3}
a) f(1) = f(2) + 4 c) f(2) = 2 f(1) e) {x ∈  : x < 3} ∪ {x ∈  : x > 2}
b) f(2) = f(1) – 1 d) f(1) = 2 f(2)
x2 − x − 6
2x + 2 06. (ESPCEX) O domínio da função f(x) = é
08. (EEAR) Considere a função f: R* → R definida por f(x) = . 3x − 6
Se f(2a) = 0, então o valor de a é: x
a) [ −2,2[ ∪ [3, + ∞[
a) – 1/2 b) 1/2 c) –1 d) 1
b) [ −2,0] ∪ ]2,3]

x −1 c) [0,2[ ∪ [3, + ∞[
09. (EEAR) O domínio da função real g(x) = e
D = {x ∈ R / _________}.
3
x −4
2
d) ]−∞, −2] ∪ ]2,3]
a) x ≥ 1 e x ≠ 2 c) –1≤x≤1 e) ]−∞,0] ∪ ]2,3]
b) x > 2 e x ≠ 4 d) – 2 ≤ x ≤ 2 e x ≠ 0
x+y x−y
=
07. (EFOMM) Dada a função f(x,y) − , o valor de
x +5 f(a + b, a – b) é: x−y x+y
10. (EEAR) Seja a função real f(x) = . A sentença que completa
x −1
corretamente a expressão do conjunto domínio D = {x ∈  _____} a2 − b2 a2 + b2
a) c) 1 e)
dessa função é ab 2ab
a) x > 1. c) x > 0. a2 − b2 a2 + b2
b) d)
b) x ≠ 1. d) x ≠ 0. 2ab ab

55

PM_BOOK16 - MAT.indb 55 25/11/2022 19:07:16


FUNÇÃO - CONCEITOS

08. (ESPCEX) Os gráficos abaixo representam duas funções reais “f ”


e “g”, cujas únicas raízes são –1 e 2, respectivamente.

O conjunto de todos os números reais tais que f(x) ⋅ g(x) < 0 é dado por
a) x > 0 ou x < -1 c) 0<x<2 e) x < –1 ou x > 2
b) –1 < x < 0 d) –1 < x < 2
f(x)
Sendo h(x) = assinale a alternativa que apresenta os intervalos
 1 1 g(x)
1 −   x −  onde h assume valores negativos.
x  x
09. (ESPCEX) A função f(x) =  , definida em R- {0, 1}
2 1
1− + 2 a) ]−3, −1] ∪ ]6,8] d) ]−∞, −3[ ∪ [ −1,2[ ∪ [7, +∞[
x x
tem, para o mesmo domínio, os mesmos valores numéricos que a b) ]−∞, −3[ ∪ ]−1,6[ ∪ ]8, +∞[
função
e) ]−3, −1] ∪ [2,4[ ∪ ]6,8]
a) f(x) = 1 c) f(x) = x2 1 c) ]−∞,2[ ∪ [ 4, +∞[
e) f(x) =
x (x + 1)2
b) f(x) = x + 1 d) f(x) = 14. (FGV) Observe o gráfico de uma função g, definida pela lei
x +1
y = g(x), com domínio no intervalo [0,6].
12
−x − 5 +
10. (AFA) Com relação à função real f definida por f(x) = x +1
é correto afirmar que: x+9 5
− +
x +1 x
a) o domínio de f é  – {–5,–1,0}.
b) f(x) =0⇔x=−1 ou x =7.
c) f(x) > 0 ⇔ −7 < x < −5 ou x > 0
d) f(x) < 0 ⇔ x < −7 ou − 5 < x < 0 e x ≠ −1

11. (ESPCEX) Sejam f e g funções reais, tais que:


• f(x) < 0 somente para x ∈ R | –3 < x < 4;
• g(x) < 0 somente para x ∈ R | x < 0 ou x > 6;
• f(x) ≠ 0 e g(x) ≠ 0 para todo x real
Nestas condições, pode-se afirmar que o conjunto-solução da Se f é uma função com domínio [0,3] tal que, para todo x no intervalo
f(x) [0,3], temos f(x) = 3g(2x), então o gráfico de f(x) será
inequação < 0 é:
g(x)
a) {x ∈ R | x < -3 ou 0 < x < 4 ou x > 6} a) d)
b) {x ∈ R | –3 < x < 0 ou 4 < x < 6}
c) {x ∈ R | x < –3 ou 0 < x < 4}
d) {x ∈ R | x < –3 ou 0 < x < 6}
e) {x ∈ R | x ≤ –3 ou 0 < x ≤ 4 ou x ≠ 6}

12. (ESPCEX) A função f, de domínio real mais amplo possível, é tal


ax + b − 5
que f(x) = . Sabendo que f(3) não existe e f(–1) = 1, o valor
ax + 3b
de a2 + b2 é:
50 c)
25 50
a) e)
16 2 9
25 50
b) d)
3 8

13. (ESPCEX) Na figura estão representados os gráficos das funções


reais f (quadrática) e g (modular) definidas em . Todas as raízes das
funções f e g também estão representadas na figura.

56

PM_BOOK16 - MAT.indb 56 25/11/2022 19:07:18


FUNÇÃO - CONCEITOS

b) e) a) c)

b) d)

c)
2x
18. A função f(5x) em função de f(x), sendo f(x) = , é?
x −5

19. (ESPCEX) Seja A o maior subconjunto de  no qual está definida


x 3 − 5x 2 − 25x + 125
a função real f(x) = . Considere, ainda, B o
x +5
conjunto das imagens de f. Nessas condições,
a) A=  − {−5} e B=  + − {10}
b) A=  − {−5} e B=  +
c) A=  − {−5} e B= 
d) A=  − {−5,5} e B=  +
e) A=  − {−5,5} e B=  + − {10}

20. (EN) Considere f uma função real de variável real tal que:
15. (FUVEST) Sejam Df e Dg os maiores subconjuntos de 
nos quais estão definidas, respectivamente, as funções reais I. f(x + y) = f(x)f(y)
x + 2x − 4x − 8
3 2
x + 2x − 4x − 8 . Considere,
3 2 II. f(1) = 3
f(x) = e g(x)
x−2 x−2 III. f ( 2) = 2
ainda, lf e lg as imagens de f e de g, respectivamente. Nessas condições,
a) Df = Dg e lf = lg. ( )
Então f 2 + 3 2 é igual a
b) tanto Df e Dg quanto lf e lg diferem em apenas um ponto. a) 108 c) 54 e) 12
c) Df e Dg diferem em apenas um ponto, lf e lg diferem em mais de b) 72 d) 36
um ponto.
d) Df e Dg diferem em mais de um ponto, lf e lg diferem em apenas 21. (EPCAR) Considere a função real cujo gráfico está representado
um ponto. abaixo.
e) tanto Df e Dg quanto lf e lg diferem em mais de um ponto.

4x
16. (FUVEST) Considere a função f(x) = 1 − , a qual está
(x + 1)2
definida para x ≠ –1. Então, para todo x ≠ 1 e x ≠ –1, o produto
f(x) ⋅ f(–x) é igual a
a) –1 d) x2 + 1
b) 1 e) (x – 1)2
c) x+1
Pode-se afirmar que os valores de x para os quais 0 < f(x) < h são:
a) ]a, g[ – ( ]c, d[ ∪ ]f, g[ ∪ ]a, b] ∪ {e} )
17. (EPCAR) Sabendo-se que f é uma função real definida por
f(x) = 10p, onde p − 1 < x ≤ p e p é um número natural entre zero e b) ]b, f] – {e}
cinco, pode-se dizer que a melhor representação de f é dada por c) ]a, g[ – ( [c, d] ∪ [f, g[ )
d) ]a, g[ – ( ]a, b] ∪ [f, g[ ∪ {e} )

57

PM_BOOK16 - MAT.indb 57 25/11/2022 19:07:19


FUNÇÃO - CONCEITOS

22. (EPCAR) Observe o gráfico da função real g abaixo. EXERCÍCIOS DE

COMBATE
01. (AFA 2001) O domínio da função real expressa pela lei
f( x )  x ( x  1)1  ( x  1)1 é x ∈  , tal que:
a) x  1 ou 0  x  1 c) x  1 ou 0 < x < 1
b) 1  x  0 ou x > 1 d) 1  x  0 ou x > 1

02. (ESPCEX 2003) Sejam f e g funções de A em , definidas por


x 1 x 1
f( x )  e g( x )  . Nessas condições, pode-se afirmar que
x 1 x 1
Analise as alternativas seguintes e marque a FALSA.
f = g se:
a) A função g não possui raiz negativa.
b) O conjunto imagem de g é R*– {y ∈ R | y < a ou y > d} a) A  x   | x  1 ou x  1 d) A  x   | x  1
c) g(x) < 0 {x ∈ R | 0 < x < p ou x > q} b) A  x   | x  1 e) A  x   | x  1
d) g cresce se, e somente se, x é real tal que x ≤ 0 ou 0 < x ≤ m ou c) A=
x ≥ s.
xa
5f(n) + 2 03. (FUVEST 2002) f(x)  para 1  x  3.
23. Uma função f é definida como f(n + 1) = . Sendo bx  c
f(1) = 5, o valor de f(101) é 5
a) 45 b) 50 c) 55 d) 65

24. (AFA) Considere os gráficos das funções reais f: A →  e g: B → .


Sabe-se que A = [–a, a]; B = ]-∞, t]; g(–a) < f(–a); g(0) > f(0); g(a) < f(a)
e g(x) = n para todo x ≤ −a .
A = [–a, a]; B = ]–∞, t]; g(–a) < f(–a); g(0) > f(0); g(a) < f(a)

Pode-se concluir que o valor de b é:


a) −2 b) −1 c) 0 d) 1 e) 2

04. (AFA 2005) Seja f a função real cujo gráfico se apresenta a seguir:
y
Analise as afirmativas abaixo e marque a FALSA. 3
2
a) A função f é par. 1
b) Se f :  →  e g :  →  , então f(x) ∙ g(x) < 0.
-0,5 0,5 2
c) =
Im(g) [n,r[ ∪ {s} . -2 -1 ,5 5 x
-0
−2 2
d) A função h: E →  dada por h(x) = está definida se -1
f(x) − g(x)

E = {x ∈  − a ≤ x < −d ou d < x ≤ a} Analisando o gráfico, é INCORRETO afirmar que:


a) f  f 1   f  0, 5 .
25. (FUVEST) Considere a função real definida por b) f  x   1  0, x   .
1 1 c) f  0   f  x  , x   .
f(x) = x− + 1− − x .
x x 5
d) se g  x   f  x  1 , então g( 2)  f   .
a) Qual é o domínio de f?  2
b) Encontre o(s) valor(es) de x para o(s) qual(is) f(x) = 0. 05. (AFA 2012) Considere a função real g : A →  tal que
x2  x
gx  2 . Sabendo-se que o conjunto A é o mais amplo possível,
x x
é verdade que:

58

PM_BOOK16 - MAT.indb 58 25/11/2022 19:07:35


FUNÇÃO - CONCEITOS

a) x  A tal que g  x   1
b) se h  x   1 g  x  , então h possui raiz real.
c) se 0 < x < 1 , então 1  g  x   0
DESAFIO PRO
d) x  , 2 tal que g  x   3

06. (ESPCEX 2012) Na Física, as leis de Kepler descrevem o movimento


1 Considere a função f : N* → N , tal que f(x) seja o número
máximo de interseções de x retas do plano. Assinale a única
afirmação FALSA entre as alternativas abaixo:
dos planetas ao redor do Sol. Define-se como período de um planeta
a) f(3) = 3
o intervalo de tempo necessário para que este realize uma volta
b) f(4) = 6
completa ao redor do Sol. Segundo a terceira lei de Kepler, “Os
c) f(x + 1) = 2 ∙ f(x) para qualquer x ∈ N *
quadrados dos períodos de revolução (T) são proporcionais aos cubos
d) f(x + 1) = f(x) + x para qualquer x ∈ N *
das distâncias médias (R) do Sol aos planetas”, ou seja, T2 = kR3, em
e) Não existe x ∈ N * tal que f(x) = 14
que k é a constante de proporcionalidade.

2
Sabe-se que a distância do Sol a Júpiter é 5 vezes a distância Terra- (AFA) No gráfico abaixo estão representadas as funções
Sol; assim, se denominarmos T ao tempo necessário para que a Terra
f :  →  e g:  → 
realize uma volta em torno do Sol, ou seja, ao ano terrestre, a duração
do “ano” de Júpiter será:
a) 3 5T c) 3 15 T e) 3 3T
b) 5 3T d) 5 5T

2 x
07. (ESPCEX 2011) O domínio da função real f(x)  é:
x 2  8x  12
a) ]2,∞[ c) ]–∞,6] e) ]–∞,2[
b) ]2,6[ d) ]–2,2]

 2x  1,se x for racional Sobre estas funções é correto afirmar que


08. (ESPCEX) Seja a função f(x)    2x ,se x for irracional . Assim o
4
a) A ∪ B, A ∩ C e B ∩ C tal que 0 ≤ x ≤ d
x 2  8,se x não for real
 b) f(x) > g(x) apenas para 0 < x < d

 2
  
valor de f  1   f i64  5i110   f f 2 , em que i² = –1, é:
f(a) + g(f(a))
a) 0 c) 2 c) >1
e) 4 g(c) + f(d)
b) 1 d) 3
d) f(x) ⋅ g(x) ≥ 0 ∀ x ∈  tal que x ≤ b ou x ≥ c
09. (ESPCEX) Na figura abaixo está representado o gráfico da
função polinomial f, definida no intervalo real [a, b]. Com base nas
informações fornecidas pela figura, podemos afirmar que: 3 (AFA) Considere o gráfico da função real g : A → A abaixo
e marque (V) verdadeiro ou (F) falso.

a) f é crescente no intervalo [a, 0].


b) f(x) ≤ f(e) para todo x no intervalo [d, b].
c) f(x) ≤ 0 para todo x no intervalo [c, 0].
d) a função f é decrescente no intervalo [c, e].
e) se x1 ∈ [a, c] e x2 ∈ [d, e] então f(x1) < f(x2). ( ) A função g possui exatamente duas raízes.
( ) g(4) = –g(–3)
10. (ESPCEX) Assinale a alternativa que representa o conjunto
( ) Im ( g) = {−3} ∪ −2,4 [
de todos os números reais para os quais está definida a função
x²  6x  5 . ( ) A função definida por h(x) = g(x) + 3 não possui raiz.
f(x)  ( ) (gogogo... og) (–2) = 2
3
x²  4
a) R – {–2,2} d) (–∞,1) ∪ (5,+∞) A sequência correta é:
b) (–∞,–2) ∪ (5,+∞) e) (–∞,–2] ∪ [2,+∞) a) F - V - F - F - V c) F - V - F - V - F
c) (–∞,–2) ∪ (–2,1]∪[5,+∞) b) F - F - V - F - V d) V - V - F - F - V

59

PM_BOOK16 - MAT.indb 59 25/11/2022 19:07:44


FUNÇÃO - CONCEITOS

4 (AFA) O gráfico abaixo descreve uma função f : A → B


GABARITO
EXERCÍCIOS DE FIXAÇÃO
01. D 05. D 09. A
02. D 06. B 10. A
03. C 07. C
04. C 08. A
EXERCÍCIOS DE TREINAMENTO
01. D 10. D 19. B
02. D 11. E 20. B
03. A 12. C 21. A
04. A 13. B 22. B
05. A 14. E 23. A
06. A 15. E 24. B
Analise as proposições que seguem. 07. A 16. B 1+ 5
25. x =
I. A = * 08. E 17. B 2
II. f é sobrejetora se B =  − [ −e,e] 09. B 5f(x)
18. f(5x) =
III. Para infinitos valores de x ∈ A , tem-se f(x) = –b 2f(x) + 1
IV. f(-c) - f(c) + f(-b) + f(b) = 2b EXERCÍCIOS DE COMBATE
V. f é função par 01. A 05. C 09. D
VI. Ζx ∈  f(x) =−d 02. D 06. D 10. C
03. D 07. E
São verdadeiras apenas as proposições 04. B 08. C
a) I, III e IV b) I, II e VI c) III, IV e V d) I, II e IV DESAFIO PRO
01. C 03. A 05. C

5 (ITA) Considere conjuntos A, B ⊂ R e C ⊂ A ∪ B.


Se A ∪ B, A ∩ C e B ∩ C são os domínios das funções
02. D 04. A

x−π
reais definidas por ln ( )
x − π , −x2 + 6 − 8 e
5−x
,
respectivamente, pode-se afirmar que:
a) C = ] π , 5[. d) C = [π, 4].
b) C = [2, π]. e) C não é intervalo.
c) C = [2, 5[.

ANOTAÇÕES

60

PM_BOOK16 - MAT.indb 60 25/11/2022 19:07:45


TIPOS DE FUNÇÃO

FUNÇÕES MONOTÔNICAS PARIDADE


Chama-se monotônica ou monótona a função que é sempre
crescente ou decrescente no seu domínio. FUNÇÃO PAR
Seja a função f: A → B, então: Seja A um conjunto tal que x ∈ A ⇒ –x ∈ A e a função f: A → B.
• f é crescente (não decrescente) se ∀x, y ∈ A tais que x < y ⇒ Diz-se que:
f(x) < f(y). f é par ⇔ f (x) = f (–x), ∀x ∈ A
• f é decrescente (não crescente) se ∀x,y ∈ A tais que x < y ⇒ Exemplo:
f(x) > f(y).
f(x) = x² e f(x) = cos x.
• f é estritamente crescente (crescente) se ∀x,y ∈ A tais que
O gráfico das funções pares é simétrico em relação ao eixo Oy,
x < y ⇒ f(x) < f(y).
pois (x,y) ∈ f ⇔ (–x,y) ∈ f.
• f é estritamente decrescente (decrescente) se ∀x,y ∈ A tais
que x < y ⇒ f(x) > f(y).
São funções crescentes f(x) = 3x – 1, f(x) = 2x e f(x) = x³.
x
 1
São funções decrescentes f(x) = –2x + 5, f  x     e f(x) = –x³.
 2
As funções f(x) = x² e f(x) = sen x não são crescentes e nem
decrescentes em i.
Esses conceitos são facilmente observados no gráfico da função.
Nas funções crescentes o gráfico “sobe” para a direita, enquanto nas
funções decrescentes o gráfico “desce” para a direita.

FUNÇÃO ÍMPAR
Seja A um conjunto tal que x ∈ A ⇒ –x ∈ A e a função f : A → B.
Diz-se que:
f é impar ⇔ f (–x) = – f(x), ∀x ∈ A
Exemplo:
f(x) = x³ e f(x) = sen x.
O gráfico das funções ímpares é simétrico em relação a origem,
pois (x,y) ∈ f ⇔ (–x,–y) ∈ f.

Já a função a seguir não é monótona, pois ela é decrescente numa


parte do domínio e crescente em outra.

Se uma função não é nem par nem ímpar dizemos que ela não
possui paridade.
Exemplo:
f(x) = x² + x – 1.

61

PM_BOOK16 - MAT.indb 61 25/11/2022 19:07:46


TIPOS DE FUNÇÃO

Para identificar a paridade de uma função devemos obter a A função f: A → B é injetora quando cada elemento de B está
expressão de f(–x). Caso essa expressão seja idêntica à de f(x), a função associado por f a no máximo um elemento de A, ou seja, elementos
é par, caso seja idêntica à de –f(x), a função é ímpar, e caso não seja distintos de A estão associados a elementos distintos de B.
idêntica a nenhuma das duas, a função não possui paridade. f ∅ injetora ⇔ ∀x1, x2 ∈ A, x1 ≠ x2 ⇒ f(x1) ≠ f(x2) ou
Note que devemos obter identidades, não basta que a igualdade ∀x1, x2 ∈ A, f(x1) = f(x2) ⇒ x1 = x2
ocorra para alguns pontos, ele tem que ocorrer em todo o domínio.
Exemplo: No diagrama de flechas, não há elemento em B que receba mais
de uma flecha.
Identifique a paridade de f  x   x5  sen3 x .
5 3
f   x     x   sen3   x    x5    sen x    x5  sen3 x   f  x  , x  
3
n3   x    x5    sen x    x5  sen3 x   f  x  , x  

Logo, a função é ímpar.

ProBizu
A soma, diferença, produto ou quociente de duas funções pares
é uma função par.
A soma ou diferença de duas funções ímpares é uma função
ímpar.
O produto ou o quociente de duas funções ímpares é função par.

No gráfico, retas horizontais traçadas a partir de pontos do


contradomínio intersectam o gráfico em no máximo um ponto.
TIPOLOGIA DAS FUNÇÕES
Exemplo:
A função f: A → B é sobrejetora quando todo elemento de B está
associado por f a pelo menos um elemento de A, ou seja, quando a A função f:  →  dada por f(x) = 2x é injetora, mas não é
imagem da função é igual ao seu contradomínio. sobrejetora. Observe que as retas horizontais traçadas intersectam o
gráfico em um ou nenhum ponto.
f : A → B é sobrejetora ⇔ ∀y ∈ B, ∃x ∈ A tal que (x,y) ∈ f ou y = f(x)
ƒ(x) = 2x
No diagrama de flechas, todo elemento do contradomínio B
y
recebe flecha.

No gráfico, retas horizontais traçadas a partir de pontos do x


contradomínio intersectam o gráfico em pelo menos um ponto.
Exemplo:
A função f:  →  dada por f(x) = x³ – 2x + 1 é sobrejetora, mas
não é injetora. Observe que as retas horizontais traçadas intersectam
o gráfico em um, dois ou três pontos. A função f: A → B é bijetora se, e somente se, é sobrejetora e
injetora, ou seja, todo elemento de B está associado por f a um único
ƒ(x) = x – 2x + 1
3

y
elemento de A.
No diagrama de flechas, todo elemento de B recebe exatamente
uma flecha.

62

PM_BOOK16 - MAT.indb 62 25/11/2022 19:07:48


TIPOS DE FUNÇÃO

No gráfico, retas horizontais traçadas a partir de pontos do a) f é injetora, pois para cada menina pertencente ao conjunto A
contradomínio intersectam o gráfico em exatamente um ponto. está associado um menino diferente pertencente ao conjunto B.
Exemplo: b) f é sobrejetora, pois cada par é formado por uma menina
A função f:  →  dada por f(x) = x³ é bijetora. Observe que as pertencente ao conjunto A e um menino pertencente ao conjunto
retas horizontais traçadas intersectam o gráfico em exatamente um B, sobrando um menino sem formar par.
ponto. c) f é injetora, pois duas meninas quaisquer pertencentes ao
conjunto A formam par com um mesmo menino pertencente ao
y conjunto B, para envolver a totalidade de alunos da turma.
ƒ(x) = x 3
d) f é bijetora, pois dois meninos quaisquer pertencentes ao conjunto
B formam par com uma mesma menina pertencente ao conjunto A.
e) f é sobrejetora, pois basta que uma menina do conjunto A forme
par com dois meninos pertencentes ao conjunto B, assim nenhum
menino ficará sem par.

02. (ESPCEX) Sabendo que c e d são números reais, o maior valor de d


− x + c , para x ≥ d

tal que a função f :  →  definida por f(x) =  2
seja injetora é x − 4x + 3, para x < d

x a) 0. b) 1. c) 2. d) 3. e) 4.

03. A função f definida por f(x) = 1 + x 2 é uma função bijetora, se os


conjuntos que representam o domínio (D(f)) e a imagem (Im(f)) são:
ProBizu
a) D(f) =  e lm(f)= [1, +∞[
Seja f: A → B, onde A e B são conjuntos finitos e suas quantidades
de elementos são dadas por #(A) e #(B), respectivamente, então b) D(f) = ] − ∞,0] e lm(f) = 
se f é sobrejetora, então #(A) ≥ #(B); c) D(f) =  e lm(f) = 
se f é injetora, então #(A) ≤ #(B); e d) = [0, +∞[ e
D(f)
se f é bijetora, então #(A) = #(B). e) = [0, +∞[ e lm(f)= [1, +∞[
D(f)

04. Considere que f :  →  é uma função bijetora. Dados a e b


FUNÇÃO LIMITADA números reais quaisquer, defina a função g, dada pela expressão
Uma função f é limitada se ∃K > 0 tal que ∀x ∈ Df ⇒ |f(x)| < K. g(x) = f(x + a) + b. É correto afirmar que para qualquer que seja a
função f temos
Exemplo:
A função f(x) = sen x é uma função limitada, pois ∀x ∈ , temos a) a imagem da função g é o conjunto [b, ∞ ) .
–1 ≤ sen x ≤ 1 ⇔ |sen x| ≤ 1. b) o domínio da função g é o conjunto [a, ∞ ) .
A função f(x) = x² não é limitada, pois ∀K > 0, ∃x ∈ Df tal que c) o gráfico da função g é uma reta.
f(x) = x² > K . b
d) para a ≠ 0, é uma raiz da função g.
a
e) g é uma função bijetora.
FUNÇÃO PERIÓDICA
Uma função f é periódica se, e somente se, ∃p > 0 tal que 05. Sejam
f(x) = f(x + p), ∀x ∈ Df.
x−2 2
Isso significa que os valores da função se repetem em intervalos I. f(x) = III. f(x) = , x≠0
x2 + 2 x
de tamanho p > 0. O menor número positivo p com essa propriedade
é chamado período da função. 1 IV. f(x) = (x + 1) + (x – 1)
II. f(x) = , x≠0
Os exemplos mais comuns de funções periódicas são as funções x2
trigonométricas. A função f(x) = sen x, por exemplo, é uma função Classificando cada uma das funções reais acima em par, ímpar ou nem
periódica de período 2π, pois sen(x + 2π) = senx, ∀x ∈ . par nem ímpar, temos, respectivamente:
a) par, par, ímpar, ímpar
EXERCÍCIOS DE b) nem par nem ímpar, par, ímpar, ímpar

FIXAÇÃO c) par, ímpar, par, ímpar


d) ímpar, par, ímpar, ímpar
e) par, par, ímpar, nem par nem ímpar

01. No primeiro ano do ensino médio de uma escola, é hábito os 06. Considerando os conjuntos: R = {0, 1, 3, 5, 7}, S = {2, 4, 6} e
alunos dançarem quadrilha na festa junina. Neste ano, há 12 meninas P = {1, 2}, assinale o que for correto.
e 13 meninos na turma, e para a quadrilha foram formados 12 pares
distintos, compostos por uma menina e um menino. Considere que 01) 1 ∈ (S – P).
as meninas sejam os elementos que compõem o conjunto A e os 02) Existe uma função f : S → P que é bijetora.
meninos, o conjunto B, de modo que os pares formados representem 04) (S ∩ P) ∪ R = R.
uma função f de A em B.
08) R ∩ S ∩ P= ∅.
Com base nessas informações, a classificação do tipo de função que
16) Nenhuma função f : S → R é sobrejetora.
está presente nessa relação é

63

PM_BOOK16 - MAT.indb 63 25/11/2022 19:07:50


TIPOS DE FUNÇÃO

07. Seja a um número real e uma função definida EXERCÍCIOS DE


por f(x) = m2x2 + 4mx + 1, com m ≠ 0. O valor de a para que a função
f seja sobrejetora é: TREINAMENTO
a) - 4 c) 3 e) 2
b) - 3 d) 0
01. Sejam a, b, c e d números reais com a ≠ b e c ≠ d. Suponha que
f:[a,b] → [c,d] é uma função estritamente crescente (isto é, x1 < x2 ⇔
08. Uma função f : R → R diz-se par quando f(−x) = f(x), para todo
f(x1) < f(x2)) e sobrejetiva. Então podemos afirmar corretamente que:
x ∈ R, e ímpar quando f(−x) = − f(x), para todo x ∈ R.
a) f[(a + b)/2] = (c + d)/2
a) Quais, dentre os gráficos exibidos, melhor representam funções
pares ou funções ímpares? Justifique sua resposta. b) f(a) = c e f(b) = d
c) f(a) + f(b) ∈ [c, d]
d) f(b) – f(a) ∈ [c, d]
e) |f(a)| < |f(b)|

02. Considere as funções f, g e h, todas definidas em [m, n] com


imagens em [p, q] representadas através dos gráficos a seguir:

Pode-se afirmar que:


a) f é bijetiva, g é sobrejetiva e h não é injetiva.
b) Dê dois exemplos de funções, y = f(x) e y = g(x), sendo uma par e
outra ímpar, e exiba os seus gráficos. b) f é sobrejetiva, g é injetiva e h não é sobrejetiva.
c) f não é injetiva, g é bijetiva e h é injetiva.
09. Há funções y = f(x) que possuem a seguinte propriedade: “a d) f é injetiva, g não é sobrejetiva e h é bijetiva.
valores distintos de x correspondem valores distintos de y”. Tais
e) f é sobrejetiva, g não é injetiva e h é sobrejetiva.
funções são chamadas injetoras.
Qual, dentre as funções cujos gráficos aparecem abaixo, é injetora? 03. Seja f a função de  em , dada pelo gráfico a seguir
a) d)

b) e)

É correto afirmar que:


a) f é sobrejetora e não injetora.
c)
b) f é bijetora.
c) f(x) = f(–x) para todo x real.
d) f(x) > 0 para todo x real.
e) o conjunto imagem de f é ]–∞; 2].

04. Sejam A e B conjuntos com m e n elementos respectivamente.


10. Sejam E o conjunto formado por todas as escolas de ensino médio Analise as seguintes afirmativas:
de Natal e P o conjunto formado pelos números que representam a
( ) Se f: A → B é uma função injetora então m ≤ n.
quantidade de professores de cada escola do conjunto E.
( ) Se f: A → B é uma função sobrejetora então m ≥ n.
Se f: E → P é a função que a cada escola de E associa seu número de
professores, então ( ) Se f: A → B é uma função bijetora então m = n.
a) f não pode ser uma função bijetora. ( ) Se f: A → B é uma função bijetora então o gráfico de f é um
subconjunto de A × B com m × n elementos.
b) f não pode ser uma função injetora.
( ) Se m = n o número de funções bijetoras f: A → B é m!
c) f é uma função sobrejetora.
d) f é necessariamente uma função injetora.

64

PM_BOOK16 - MAT.indb 64 25/11/2022 19:07:50


TIPOS DE FUNÇÃO

05. Dentre as curvas a seguir, qual pode ser o gráfico de uma função 10. Sendo f: R → R uma função periódica de período P > 0, classifique
injetora y = f(x)? as afirmações a seguir em V (verdadeira) ou F (falsa).
a) d) ( ) A função g(x) = f(2x) é periódica de período 2p.
p
) A função h ( x ) = f   é periódica de período
x
(
 2 2
( ) A função j(x) = f(x+q), onde q é uma constante positiva, não é
periódica.
A sequência obtida foi:
b) e) a) V, V, V c) F, V, V e) F, F, F
b) V, V, F d) F, F, V

11. (CMRJ) Observe o gráfico abaixo de uma função real f e, em


seguida, assinale a afirmativa FALSA, relativa a esse gráfico.

c)

n
 2 , se n é par
06. (EEAR) A função f : N → N definida por f(n) =  é:
 n + 1, se n é ímpar
 2
a) bijetora.
b) somente injetora. a) Os zeros da função são –2 e 5.
c) somente sobrejetora. b) A função é crescente para os valores de x que pertencem a
d) não injetora e não sobrejetora. ]−4,0[ .
c) f(2) = f(3) + f(4)
07. (AFA) Seja D = {1, 2, 3, 4, 5} e f : D → R, a função definida por d) f(x) > 0 se −2 ≤ x ≤ 5
f(x) = (x – 2)(x – 4). Então, pode-se afirmar que f
e) A soma das imagens dos elementos e 6 do domínio de f é –3.
a) é bijetora.
b) é somente injetora. 12. Sejam f e g duas funções cujos domínio e contradomínio são o
c) é somente sobrejetora. conjunto dos números reais. Considere as afirmações a seguir:
d) possui conjunto imagem com 3 elementos. I. Sempre que g é injetora, g  f :  →  é injetora.
II. Se f é decrescente e g também é decrescente, então, f  g
08. (UFRN) Sejam E o conjunto formado por todas as escolas de também é decrescente.
ensino médio de Natal e P o conjunto formado pelos números que
III. Se f é crescente, g é decrescente e g(x) > 0 para todo x real, então,
representam a quantidade de professores de cada escola do conjunto
E. Se f : E → P é a função que a cada escola de E associa seu número f é crescente.
de professores, então g
a) f não pode ser uma função bijetora. IV. Se f é decrescente e g é decrescente, então, f + g é decrescente.
b) f não pode ser uma função injetora. V. Se os gráficos de f e g não interceptam o eixo das abscissas, então,
c) f é uma função sobrejetora. o gráfico de f ∙ g também não intercepta o eixo das abscissas.
d) f é necessariamente uma função injetora. A quantidade de afirmações INCORRETAS é
a) 1 d) 4
09. Classifique as afirmações a seguir em verdadeira ou falsa. b) 2 e) 5
( ) O produto de duas funções ímpares é ímpar. c) 3
( ) O produto de duas funções pares é par.
( ) A soma de duas funções ímpares é par. 13. (EPCAR 3° ANO) Uma função f é definida em A e tem imagem
( ) A soma de duas funções pares é par. em B. Sabe-se que o conjunto A tem 2k – 2 elementos e o conjunto
B tem k + 3 elementos, com k > 1. Neste caso, é correto afirmar que
( ) Se uma função é ímpar e bijetora, sua inversa é ímpar.
a) se f é injetora, então k é estritamente igual a 5.
A diferença entre o número de afirmativas verdadeiras e o número de b) o número máximo de funções injetoras que podem ser definidas
afirmativas falsas é igual a de A em B é dado pelo arranjo de (k + 3) elementos tomados
a) 1 d) –3 (2k – 2) a (2k – 2) elementos.
b) –1 e) 5 c) se f é sobrejetora, então 1 < k ≤ 5 .
c) 3 d) f é bijetora para todo k

65

PM_BOOK16 - MAT.indb 65 25/11/2022 19:07:51


TIPOS DE FUNÇÃO

14. (EPCAR 3º ANO) Dados os gráficos de funções de R em R, assinale 17. (EFOMM) Seja a função f:  →  (sendo  o conjunto dos
aquele cuja função não é injetora e nem sobrejetora. números inteiros e  o conjunto dos números racionais) com a
a) c) f ( x − 1) − 1
seguinte propriedade definida por f ( x − 1) + 1 = . Sabendo-
f (x)
se que f(0) = 4, o valor de f(1007) é igual a:
1 3
a) –1 c) − e)
4 5
5
b) 4 d) −
3
b) d)
18. (ITA) Considere os conjuntos S = {0, 2, 4, 6}, T = {1, 3, 5} e
U = {0, 1} e as afirmações:
I. {0} ∈ S e S ∩ U ≠ ∅.
II. {2} ⊂ (S – U) e S ∩ T ∩ U = {0, 1}.
III. Existe uma função f : S → T injetiva.
IV. Nenhuma função g : T → S é sobrejetiva.
15. (EPCAR 3° ANO) Sejam as funções reais f1, f2 e f3 abaixo
Então, é(são) verdadeira(s):
representadas:
a) apenas I. d) apenas II e III.
b) apenas IV. e) apenas III e IV.
c) apenas I e IV.

19. (ITA) Seja D = R – {1} e f: D → D uma função dada por


f(x) = (x + 1) / (x – 1).
Considere as afirmações:
I. f é injetiva e sobrejetiva.
II. f é injetiva, mas não sobrejetiva.
III. f(x) + f(1/x) = 0, para todo x ∈ D, x ≠ 0.
IV. f(x) · f(–x) = 1, para todo x ∈ D.
Então, são verdadeiras
a) penas I e III.
b) apenas I e IV.
c) apenas II e III.
d) apenas I, III e IV.
e) apenas II, III e IV.

20. (FEI) Sejam f e g funções de  em , definidas por f(x) = x2 e


g(x) = 2x – 3. Pode-se afirmar que:
a) f ∘ g é injetora.
Considere as afirmações:
b) g ∘ g é decrescente para todo x real.
I. f1 admite inversa
c) f ∘ g(1) = –1.
II. f2 é uma função crescente
d) g-1 é decrescente para todo x real.
III. f3 é sobrejetora
e) g ∘ f(2) = 5.
Associe a cada uma delas V, se for verdadeiro, e F, caso seja falso.
Nesta ordem, tem-se:
a) V, V, F c) F, V, V
EXERCÍCIOS DE

COMBATE
b) V, F, V d) F, F, V

16. (EPCAR 3° ANO) Analise as funções abaixo quanto à tipologia e


assinale a opção correta
I. f :  →  tal que f(x) = x2 01. (EPCAR 1999 – 2º ano) Seja ƒ:  →  uma função injetora
II. f :  → + tal que f(x) = x2 definida por y = ƒ(x). Tem-se que ƒ(0) = –5 e ƒ(1) = 0 e ƒ(3) = 6.
III. f :  →  tal que f(x) = x3 Sabendo-se que ƒ(ƒ(a – 2)) = –5, então ƒ(a) é igual a:
IV. f :  →  tal que f(x) = |x| a) zero b) –5 c) 3 d) 6

a) III é bijetora 02. (EPCAR 2002 – 3º ano) Sejam A e B dois conjuntos com,
b) I e III são injetoras respectivamente, 19 e 92 elementos. Seja a função ƒ: A → B. Então,
c) II e IV são sobrejetoras a) existem sobrejeções de A em B.
d) Todas são bijetoras b) toda função de A em B é uma injeção.

66

PM_BOOK16 - MAT.indb 66 25/11/2022 19:07:52


TIPOS DE FUNÇÃO

c) não existem bijeções de A em B. 06. (EFOMM 2015) Sejam as funções ƒ:  →  e g:  → . Sabendo


d) o conjunto imagem de qualquer função, de A em B, tem 19 que ƒ é bijetora e g é sobrejetora, considere as sentenças a seguir:
elementos. I. g  f é injetora;
II. f  g é bijetora;
03. (EPCAR 2003 – 3º ano) Analise os itens abaixo, classificando-os
III. g  f é sobrejetora.
em V (verdadeiro) ou F (falso).
Assinalando com verdadeiro (V) ou falso (F) a cada sentença, obtém-se
I. Se ƒ(2 + x) = x · (x2 + 1)-1, então ƒ(3) = 0,5 .
a) V – V – V d) F – F – V
II. A função h :  → + representada no gráfico ao lado não é
injetora, mas é sobrejetora. b) V – V – F e) V – F – V
y
c) F–V–F

07. (EFOMM 2010) Seja ƒ :  →  uma função estritamente


decrescente, quaisquer x1 e x2 reais, com x1 < x2 tem-se
ƒ(x1) > ƒ(x2). Nessas condições, analise as afirmativas abaixo.
x I. ƒ é injetora.
II. ƒ pode ser uma função par.
III. Se ƒ possui inversa, então sua inversa é estritamente decrescente.
Assinale a opção correta.
1
III. Se g: R → R associa x à expressão , a) Apenas a afirmativa I é verdadeira.
1 + x2
( )
então g 4 7 =
1− 7
6
. A sequência correta é:
b) Apenas as afirmativas I e III são verdadeiras.
c) Apenas as afirmativas II e III são verdadeiras.
a) V - V - V c) V-F-V d) As afirmativas I, II e III são verdadeiras.
b) F - V - V d) V - V - F e) Apenas a afirmativa II é verdadeira.

04. (AFA 2013) O gráfico abaixo descreve uma função ƒ: A → B 08. (ESPCEX 2004) Analise os itens abaixo para a função ƒ :  → :
I. Se ƒ(x) + ƒ(–x) = 0 então ƒ é uma função par.
II. Se ƒ(x) é uma função constante, então ƒ é função par.
III. Se |ƒ(x)| = ƒ(x) então Im(ƒ) ⊂ +.
IV. Se |ƒ(x)| = ƒ(x), então f é função bijetora.
São corretas as afirmativas:
a) I e II.
b) II e IV.
c) II e III.
d) I e III.
e) III e IV.

09. (ITA 1979) Seja f uma função real definida para todo x real tal que:
ƒ é ímpar; ƒ(x + y) = ƒ(x) + ƒ(y); e ƒ(x) ≥ 0, se x ≥ 0. Definindo
f(x) - f(1)
g(x) = , se x ≠ 0. Sendo n um número natural, podemos
x
Analise as proposições que seguem.
afirmar que:
I. A = *
a) f é não decrescente e g é uma função ímpar.
II. ƒ é sobrejetora se B =  – [–e, e].
b) f é não decrescente e g é uma função par.
III. Para infinitos valores de x ∈ A, tem-se ƒ(x) = –b
c) f é não decrescente e 0 ≤ g(n) ≤ f(1).
IV. ƒ(–c) – ƒ(c) + ƒ(–b) + ƒ(b) = 2b
d) f não é monótona e 0 ≤ g(n) ≤ f(1).
V. ƒ é função par.
e) não é possível garantir que 0 ≤ g(n) ≤ f(1).
VI. ∃x ∈  | ƒ(x) = –d
São verdadeiras apenas as proposições: 10. (ESPCEX) Sabendo que “c” e “d” são números reais, o maior valor
a) I, III e IV. c) III, IV e V. de “d” tal que a função ƒ:  →  definida por ƒ(x)
-x + c, para x ≥ d
b) I, II e VI. d) I, II e IV.  2 seja injetora é:
x - 4x + 3 + c, para x < d
ax + b a) 0
05. (ITA 2002) Sendo par a função dada por ƒ(x) = ,
x +c b) 1
– c < x < c, então ƒ(x), para – c < x < c, é constante e igual a: c) 2
a) a + b d) b d) 3
b) a + c e) a e) 4
c) c

67

PM_BOOK16 - MAT.indb 67 25/11/2022 19:07:53


TIPOS DE FUNÇÃO

Podemos afirmar que:

DESAFIO PRO
a) g é uma função sobrejetora.
b) g é uma função injetora.
c) f é uma função sobrejetora.
d) f é uma função injetora.
1 (ITA) Mostre que toda função f:  / {0} → , satisfazendo
f(xy) = f(x) + f(y) em todo seu domínio, é par. e) g(2018) tem mais do que 4 divisores positivos.

2 (IME) Considere as alternativas: GABARITO


EXERCÍCIOS DE FIXAÇÃO
I. O inverso de um irracional é sempre irracional.
II. Seja a função f: A → B e X e Y dois subconjuntos quaisquer 01. A 03. E 05. B 07. B
de A, então f(X ∩ Y)= f(X) ∩ f(Y). 02. C 04. E 06. SOMA: 24
III. Seja a função f: A → B e X e Y dois subconjuntos quaisquer 08. a) As funções pares são I e III, pois f(–a) = f(a) para qualquer a real.
de A, então f(X ∪ Y)= f(X) ∪ f(Y). As funções ímpares são IV e V, pois f(–a) = –f(a) para qualquer a.
IV. Dados dois conjuntos A e B não vazios, então A ∩ B =A se, b) função y = x2 é par e a função y = x é ímpar.
e somente se, B ⊂ A.

Observação: f(Z) é a imagem de f no domínio Z.


São corretas:
a) I, apenas. d) I e IV, apenas.
b) I e III, apenas. e) II e III, apenas.
c) II e IV, apenas.

3 (ITA) Sejam x e y dois conjuntos finitos com X ⊂ Y e X ≠ Y.


Considere as seguintes afirmações: 09. E 10. C
I. Existe uma bijeção f : X → Y EXERCÍCIOS DE TREINAMENTO
II. Existe uma função injetora g: Y → X 01. B 06. C 11. D 16. A
III. O número de funções injetoras f : X → Y é igual ao número 02. A 07. D 12. B 17. D
de funções sobrejetoras g : Y → X 03. A 08. C 13. D 18. B
É (são) verdadeira(s) 04. V V V F V 09. A 14. A 19. A
a) nenhuma delas. d) apenas I e II. 05. E 10. E 15. C 20. E
b) apenas I. e) todas. EXERCÍCIOS DE COMBATE
c) apenas III. 01. D 04. A 07. B 10. C
02. C 05. E 08. C

4 (ITA) Seja f:  →  a função definida por f(x) = –3ax, onde


a é um número real, 0 < a < 1.
03. D
DESAFIO PRO
06. D 09. C

Sobre as afirmações: 01. x = z e y = z → f(z2) = f(z) + f(z) → f(z2) = 2f(z)


I. f(x + y) = f(x) f(y), para todo x, y, ∈ . x = –z e y = –z → f(z2) = f(–z) + f(–z) → f(z2) = 2f(–z)
II. f é bijetora. Logo, f(z2) = 2 f(z) = 2 f(–z), ∀z ∈  / {0} →
III. f é crescente e f (]0, +∞[) = ]–3, 0[ f(–z) = f(z),∀z ∈  / {0} → f é par, ∀z ∈  / {0}
Podemos concluir que:
02. B 03. A 04. E 05. E
a) Todas as afirmações são falsas.
b) Todas as afirmações são verdadeiras. ANOTAÇÕES
c) Apenas as afirmações (I) e (III) são verdadeiras.
d) Apenas a afirmação (II) é verdadeira.
e) Apenas a afirmação (III) é verdadeira.

5 (IME) Definimos a função f :  →  da seguinte forma:


f(0) = 0

f(1) = 1

=f(2n) f(n), n ≥ 1
f(2n + 1)
= n2 , n ≥ 1

Definimos a função g :  →  da seguinte forma:


g(n) = f(n)f(n + 1)

68

PM_BOOK16 - MAT.indb 68 25/11/2022 19:07:54


FUNÇÃO COMPOSTA E INVERSA

FUNÇÃO COMPOSTA TRANSLAÇÕES E TRANSFORMAÇÕES


A composição de funções consiste na aplicação sucessiva de DE GRÁFICOS
funções. Muitas vezes busca-se uma expressão única que substitua
essas aplicações sucessivas, em outros casos descobrir valores da
função composta ou das funções originais ou estudar seus domínios. TRANSLAÇÃO NA VERTICAL
Isso aparece em diversos exercícios e os conceitos a seguir vão nos Temos uma função f(x) tal que f:  →  e definimos a função
ajudar a lidar com cada uma dessas questões. g(x) = f(x) + a onde a ∈ . Dessa forma a função g(x) irá deslocar a
Dados os conjuntos A, B e C e as funções f: A → B, definida por função f(x), “a” unidades na vertical, sem alterar seu gráfico. Deslocará
y = f(x), e g: B → C, definida por z = g(y), chama-se função composta para cima quando a > 0 e deslocará para baixo quando a < 0.
de g com f a função h = (g o f); A → C, definida por: Exemplo:
Z = (g o f) (x) = g(f(x)) f(x) = x² + 4x – 5

Assim, a função (g o f) pode ser entendida como uma função


única que apresenta o mesmo resultado que as aplicações sucessivas
de f e g.
O diagrama de flechas a seguir ilustra esse conceito:

Ao fazermos a função g(x) = f(x) + 3 teremos a função


g(x) = (x² + 4x – 5) + 3 = x² + 4x – 2.

Na definição acima, adotamos o contradomínio de f coincidente


com o domínio de g. No caso geral, a condição para que a função (g o f)
esteja definida é que a imagem de f esteja contida no domínio de g.
A composição de funções não é comutativa: g o f ≠ f o g. Pode
acontecer, inclusive, de só uma das funções (f o g) ou (g o f) estar
definida.

ProBizu
A sentença aberta que define (g o f) (x) = g (f(x)) é obtida de g(x)
substituindo-se x pela expressão de f(x).

Exemplo:
Sejam as funções reais f(x) = x2 + 4x - 5 e g(x) = 2x - 3, calcule as
expressões de (f o g) e (g o f)
(f o g)(x) = f(g(x)) = f(2x – 3) = (2x – 3)2 + 4 · (2x – 3) – 5 = 4x2 – 4x – 8
(g o f)(x) = g(f(x)) = g(x2 + 4x – 5) = 2 · (x2 + 4x – 5) – 3 = 2x2 + 8x – 13 Percebemos que a função se movimentou 3 unidades para
“cima”.
Então dessa forma podemos dizer que em g(x) = f(x) + a onde
a ∈  temos que o gráfico “sobe” a unidades quando a > 0 e “desce”
a unidades quando a < 0.

69

PM_BOOK16 - MAT.indb 69 25/11/2022 19:07:55


FUNÇÃO COMPOSTA E INVERSA

TRANSLAÇÃO NA HORIZONTAL
Temos uma função f(x) tal que f:  →  e definimos a função
g(x) = f(x + b) onde b ∈ . Dessa forma a função g(x) irá deslocar
a função f(x), “b” unidades na horizontal, sem alterar seu gráfico.
Deslocará para esquerda quando b > 0 e deslocará para direita
quando b < 0.
Exemplo:
f(x) = x – 2

g(x) = |f(x)|
Ao se fazer g(x) = |f(x)| temos uma “modulação” da função
f(x). Isso quer dizer que a função f(x) passa assumir somente valores
positivos, sendo
 as partes negativas do gráfico sendo refletidas para
cima do eixo Ox .
Exemplo:

Agora quando definirmos a função g(x) = f(x + 2) devemos pensar f(x) = x² – 3x – 2


o seguinte:
Para x = 0 teremos f(0) = 0 – 2 = -2 e g(0) = f(0 + 2) = f(2) = 2 – 2 = 0.
Repare que a função f vem a ser 0 para x = 2, porém para a função g
isso acontece em x = 0.
g(0) = f(2) = 0

Podemos pensar que ao fazer g(x) = f(x + 2) estaremos adiantando


o que acontece com a função f em 2 unidades. Por exemplo a função
g zera 2 unidades antes da função f, conforme a figura abaixo. Dessa
forma:
g(2) = f(2 + 2) = f(4) = 4 – 2 = 2

O que para a função f acontece em x para a função g acontece


em (x + 2).
Como se vê no gráfico que a função se desloca para esquerda.

Fazendo g(x) = |f(x)| teremos

Então dessa forma podemos dizer que em g(x) = f(x + b) onde


b ∈  temos que o gráfico “anda para a direita” b unidades quando
b < 0 e “anda para a esquerda” b unidades quando b > 0.
Veja um outro exemplo para f(x) = x² – 3x – 2 e g(x) = f(x – 3)

70

PM_BOOK16 - MAT.indb 70 25/11/2022 19:07:56


FUNÇÃO COMPOSTA E INVERSA

Vemos que a parte negativa


 da parábola foi completamente ProBizu
rebatida para cima do eixo Ox mantendo toda a simetria.
Uma função só possui inversa se ela for bijetora.
No gráfico abaixo vemos as funções cos x e |cos x| no intervalo
de 0 a 4π.
No caso de funções que não são bijetoras, a relação inversa está
f(x) = cos x definida, mas ela não é uma função, pois pode haver elementos no
conjunto de partida que não estejam relacionados a ninguém ou a
mais de um elemento no conjunto de chegada.
O domínio da função inversa é a imagem da função original e a
imagem da função inversa é o domínio da função original.
D(f –1) = Im(f) e Im(f –1) = D(f)
Esses conceitos podem ser observados nos diagramas de flecha
seguintes:
f(x) = |cos x|

FUNÇÃO INVERSA
A função inversa está relacionada à ideia de relação inversa, ou
seja, se na função original a entrada x corresponde a uma saída y, ProBizu
na função inversa a entrada y corresponde à saída x. Isso é abordado Para encontrar a imagem de uma função bijetora, basta encontrar
em diversas situações, como encontrar a expressão ou valores da o domínio da sua função inversa.
função inversa, a forma do seu gráfico e suas características, inclusive
associada à composição de funções. Vamos agora ver como fazer tudo Exemplo:
isso nos conceitos apresentados a seguir e nos exercícios seguintes.
1
Se f é uma função de A em B, bijetora, a relação inversa de f é Encontre a imagem de f: i – {1} → i tal que f  x  
x 1
uma função de B em A, chamada função inversa de f e denotada por Vamos começar encontrando a inversa de f.
f -1 e também é bijetora.
1 1 1
A definição acima resulta nas seguintes expressões: y  f x   x  1   x   1
x 1 y y
Exemplo: y 1
(x, y) ∈ f ⇔ (y, x) ∈ f –1  f 1  y   x 
y
f(x) = y ⇔ f–1(y) = x
x 1
Substituindo-se a variável y por x, temos: f 1  x  
A função inversa é composta pelos pares ordenados obtidos pela x
inversão da ordem dos elementos dos pares ordenados da função original.
Observe agora que o domínio de f–1 é Df-1 = *, pois o denominador
Assim, se a função f: A → B associa cada elemento x ∈ A a um x deve ser não nulo.
elemento correspondente y ∈ B, a função f–1, inversa de f, associa a
cada elemento y ∈ B, o elemento correspondente x ∈ A. Mas sabemos que a imagem de uma função inversível é
igual ao domínio da sua função inversa, então a imagem de f é
Exemplo: Imf = Df-1 = *.
Seja a função bijetora f(x) = 2x + 1, encontre o valor de f –1 (7). A função inversa da função inversa é a função original.
Observe que para essa questão não é necessário obter a expressão
de f–1, apenas usar a definição de relação inversa. (f –1) –1 = f
f –1 (7) = x ⇔ f(x) = 7 O resultado da aplicação da função composta de f –1 com f sobre
Vamos usar a expressão de f(x) para encontrar o valor de x tal que um elemento x ∈ Df é o próprio x, ou seja, a composição da função
f(x) = 7: inversa com a função original é a função identidade.
f(x) = 2x + 1 = 7 ⇔ x = 3 (f–1 o f)(x) = f –1 (f(x)) = x, ∀x ∈ Df
Daí se conclui que:
O resultado da aplicação da função composta de f com f –1 sobre
f –1 (7) = 3.
um elemento x ∈ Df-1 é o próprio x, ou seja, a composição de uma
função com a sua inversa é a função identidade.
(f o f-1)(x) = f(f-1(x)) = x, ∀x ∈ Df-1

71

PM_BOOK16 - MAT.indb 71 25/11/2022 19:07:59


FUNÇÃO COMPOSTA E INVERSA

Como (x,y) ∈ f ⇔ (y, x) ∈ f–1, os gráficos de f e f–1 são simétricos Exercício Resolvido
em relação à bissetriz dos quadrantes ímpares (β13), como pode ser
visto no exemplo abaixo: 01. (ITA 1987) Considere x = g(y) a função inversa da seguinte
1
função: “y = f(x) = x² – x + 1, para cada número real x ≥ ”.
Nestas condições, a função g é assim definida: 2

1 3 3
a) g( y )   y  , para cada y ≥ .
2 4 4
1 1 1
b) g( y )   y  , para cada y ≥ .
2 4 4
3 3
c) g( y )  y  , para cada y ≥ .
4 4
1 1
d) g( y )  y  , para cada y ≥ .
4 4
3 1 1
e) g( y )   y  , para cada y ≥ .
4 2 2

OBTENÇÃO DA FUNÇÃO INVERSA


Diversos exercícios solicitam que se encontre a expressão da função
inversa, identificando o seu domínio. A seguir vamos apresentar duas
maneiras de fazer isso.

1º MÉTODO:

Na expressão y = f(x), efetuamos as operações algébricas


necessárias a fim de obter uma expressão de x em função de y.
Essa é a expressão da função inversa, ou seja, x = f-1(y).
Ao final, pode ser efetuada a substituição da variável x por y, e Resolução: A
vice-versa, resultando a expressão y = f-1(x), onde x ∈ Df-1.
y = f(x) = x² – x + 1 ⇔ x² – x + (1 – y) = 0
1 1 4 1 y  1 3
Exemplo: x x  y
Obtenha a expressão da função inversa da função bijetora f:  → ,
2 2 4
definida por y = 2x – 4. 1 1 3 3
y x  gy  x   y  , y 
y  2x  4  2x  y  4  x  2 2 2 4 4
2
1
Observe que  ,  é o vértice da parábola. A condição x ≥
1 3
y x
 f 1  y    2  f 1  x    2 2 4 2
2 2
define que se busca a função inversa do ramo direito da parábola.
1  3 
A função f :  ,     ,   é bijetora e, portanto possui
2º MÉTODO: inversa. 2  4 

Sabemos que f(f-1(x)) = x. Vamos, inicialmente, obter a expressão de


f(f-1(x)), substituindo x por f-1(x) na expressão de f(x). Na expressão
EXERCÍCIOS DE
encontrada, efetuamos as operações algébricas necessárias para
isolar f-1(x).
FIXAÇÃO
Exemplo:
Obtenha a expressão da função inversa da função bijetora f:  → , 01. Se f(x) = x3 e g(x) = x4, mostre que fog = gof.
definida por y = 2x – 4.
f  f 1  x    x  2  f 1  x   4  x  =
02. Sejam f(x) x − 1 e g(x) = 2x2 – 5x + 3. Determine os domínios
x das funções fog(x) e gof(x).
 2  f 1  x   x  4  f 1  x    2
2
03. Dadas as funções f(x) = 2x + m e g(x) = ax + a, qual a relação que
Muitas vezes, para obter a expressão da função inversa, é a e m devem satisfazer para que se tenha a igualdade fog(x) = gof(x)??
necessário identificar, dentre as possíveis expressões encontradas,
aquela que corresponde ao domínio adotado para a função original. 04. Sejam as funções reais f(x) = 3x – 5 e fog(x) = x2 – 3. Determine
a lei da função g.

72

PM_BOOK16 - MAT.indb 72 25/11/2022 19:08:11


FUNÇÃO COMPOSTA E INVERSA

05. O gráfico de uma função f é o segmento de reta que une os 05. (AFA) Seja f:  →  a função definida por f(x) = x2 + a, onde a é
pontos (–3,4) e (3,0). Se f-1 é a função inversa de f, determine f-1(2). um número real não nulo. Se f o f(1) = 1, o valor de a é:
a) 0 b) – 1 c) –2 d) –3
06. Seja f:  →  a função dada por f(x) = x2 e seja g:  →  a função
dada por: 06. (AFA) Se f e g são funções de  em  definidas por
f(x + h) − f(x) 3x − 2
=g(x) ,h ≠ 0. f(3x + 2) = e g(x – 3) = 5x – 2, então f(g(x)) é
h 5
x−4 c) 5x + 13
Nessas condições, g(x) é igual a: a)
5
a) h b) x c) 2x d) 2x + h e) x + h 5x + 11
5x + 9 d)
b) 5
5
07. O gráfico de uma função f(x) = ax + b é uma reta que corta os
eixos coordenados nos pontos (2, 0) e (0, 3). O valor de f (f–1 (0)) é:
1
15 10 5 07. (AFA) Considere a função real f(x) = , x ≠ −1. Se
a) c) − e) − 2x + 2
2 3 2
1 a 
10 f( −2 + a) + = f( − a), então f  − 1 + f(4 + a) é igual a:
b) 0 d) 5 2 
3
a) 1 b) 0,75 c) 0,5 d) 0,25
08. Sendo f e g funções de  em , tais que f(x) = 4x + 7 e
fog(x) = 8x + 55, qual das alternativas abaixo indica o valor de g(4)? 08. (ESPCEX 2010) Considere a função real g(x) definida por:
a) 12 b) 16 c) 18 d) 20
5x , se x ≤ 1

( x )  −3x + 3x + 17 , se 1 < x ≤ 3
2
09. Se f(x) = 3x – 5 e f[g(x)] = 3 x2 –14, o valor de g(1) é igual a: g=
 4 2 4
a) 0 b) 1 c) –2 d) 2 
 x + 1 , se x > 3
 2 2
10. (ESPCEX) Na função f(x) = 3x – 2, sabemos que f(a) = b – 2 e
f(b) = 2b + a. O valor de f(f(a)) é: O valor de g(g(g(1))) é:
a) 2 b) 1 c) 0 d) –1 e) –2 a) 0 c) 2 e) 4
b) 1 d) 3
EXERCÍCIOS DE

TREINAMENTO
09. (EPCAR 3° ANO) Dada a função real f, tal que f(5x + 3) = x. Sendo
f-1 a inversa de f, pode-se afirmar que
a) (f-1 o f)(5) = 28 c) (f o f)(–7) = 1
b) f-1 é função ímpar. d) (f o f-1)(x) = x
01. (EEAR) A função f: N → N definida por f(x) = 3x + 2,
a) é apenas injetora 10. (AFA) Dadas f e g, duas funções reais definidas por f(x) = x3 – x e
g(x) = senx, pode-se afirmar que a expressão de ( f  g) ( x ) é
b) é apenas sobrejetora
a) senxcos2x c) –senxcos2x
c) é injetora e sobrejetora
b) –sen(x3 – x) d) senx3 – senx
d) não é nem injetora nem sobrejetora
11. (AFA) Considere o conjunto A = {0,1,2,3} e a função f: A → A tal
02. (EEAR) Sejam f e g duas funções inversas entre si. Se f(x) = 3x – 2,
que f(3) = 1 e f(x) = x + 1, se x ≠ 3. A soma dos valores de x para os
então g(1) é
quais é
a) 9 b) 10 c) 11 d) 12
a) 2 b) 3 c) 4 d) 5
x+3
03. (EEAR) Sabe-se que a função f(x) = é invertível. Assim, 12. (EPCAR 3° ANO) Analise os itens abaixo, classificando-os em V
f-1 (3) é 5
(verdadeiro) ou F (falso).
a) 3 b) 4 c) 6 d) 12 I. Existem apenas três números inteiros que satisfazem ao domínio
2x + 3
04. (CPII) Duas funções reais f e g, ambas de domínio [0,4], estão da função g dada por g(x) = −1
3− x
representadas graficamente abaixo:
II. O trinômio (m – 1)x + mx + m, onde m ∈  é sempre negativo
2

se, e somente se, m ∈ ∗


III. Se f(x) = x + 1 e g(x) = |x|, então a composição fog não é par nem
ímpar.
A sequência correta é
a) F F V b) F V F c) VFV d) V V F

13. (EPCAR 3° ANO) Sejam A = {1, 2, 3} e f: A → A definida por


f(1) = 3, f(2) = 1 e f(3) = 2. O conjunto solução de (fof)(x) = 3 é
O número de elementos do conjunto solução da equação g [ f ( x )] = 1 é a) {1} c) {3}
a) 3 b) 4 c) 6 d) 7 b) {2} d) {1,2}

73

PM_BOOK16 - MAT.indb 73 25/11/2022 19:08:12


FUNÇÃO COMPOSTA E INVERSA

14. (EPCAR 3° ANO) Sejam os gráficos das funções f, g, h (g // h) x −1


08) Se f é dada por f(x) = , para todo x diferente de 2, então f-1
definidas em 2− x
2x + 1
(x) = , para todo x diferente de –1
x +1

A soma dos itens verdadeiros é:


a) 15 b) 14 c) 13 d) 8

18. (EPCAR 3° ANO) Observe o gráfico da função real g abaixo.

Analise as interseções de regiões do plano xOy, e assinale a alternativa


correta.
a) x ≥ 0 ⇒ f(x) < g(x) c) ∃ x ∈ R tal que, h(x) ≥ g(x)
h ( x ) > f(x) Analise as alternativas seguintes e marque a FALSA.
b) x ≤ 0 ⇒ g(x) ≤ h(x) d)  ⇒2< x ≤3
 h ( x ) ≤ 0 a) A função g não possui raiz negativa.
b) O conjunto imagem de g é *– {y ∈  | y < a ou y > d}
15. (AFA) Sejam as funções g e f definidas por g :  →  tal que c) g(x) < 0 ⇔ {x ∈  | 0 < x < p ou x > q}
 2, se x > 2 d) f cresce se, e somente se, x é real tal que x ≤ 0 ou 0 < x ≤ m ou
g(x) =  e f :  →  tal que f(x) = x – 2. Sobre a
−1, se x ≤ 2 x ≥s .
composta ( g  f ) ( x ) , é correto afirmar que
19. (EPCAR 3º ANO) Considere a figura abaixo, gráfico da função real
a) se x ≥ 1 , então ( g  f ) ( x ) = −1 . g tal que g : D → ]−4,6] .
b) se x < 0 , então ( g  f ) ( x ) = 2 .
c) se x ≤ −1 , então ( g  f ) ( x ) = −1 .
d) se x < 1 e x ≠ 0 , então ( g  f ) ( x ) = −1 .

16. (AFA) Analise o gráfico abaixo das funções f e g e marque a opção


correta.

É correto afirmar que:


a) os elementos do conjunto D, domínio de g, são os mesmos do
conjunto I, imagem de g.
b) o conjunto {x ∈  | 2 < x < 5 e g ( x ) =π} possui exatamente 2
elementos.

a) O gráfico da função h(x) = g(x) – f(x) é uma reta ascendente.


c) ( )
3 ⋅ g g ( g ( −2) )  < g ( 4 )

b) O conjunto imagem da função s(x) = f(g(x)) é . d) a função g é sobrejetora, mas não é injetora.
c) f ( x ) ⋅ g ( x ) ≥ 0 , ∀x ≥ t .
20. (EN) Considere f e g funções reais de variável real definidas por,
d) g ( f (=
x ) ) g ( x ) , ∀x ∈  1
f(x) = e g(x) = 2x2. Qual é o domínio da função composta (f o g)(x)?
4x − 1
17. (EPCAR 3° ANO) Analise os itens abaixo, referentes a funções reais.
a)   1 1 
01) A função f definida por f(x) = 3x2 + 4x é par d) x ∈  | x ≠ , x ≠ 
 1 1   4 2 2
02) Se g é definida por g(x) = 2x + 8, então g(x) > 0, ∀ x > – 4 b) x ∈  | x ≠ − ,x≠ 
 2 2 2 2  1 1 
x e) x ∈  | x ≠ − , x ≠ − 
04) O conjunto-imagem da função h, definida por h(x) = é 4
x −1  1  2 2
Im ={y ∈R | y ≠1} c) x ∈  | x ≠ 
 4

74

PM_BOOK16 - MAT.indb 74 25/11/2022 19:08:15


FUNÇÃO COMPOSTA E INVERSA

21. (AFA) Seja f a função real cujo gráfico se apresenta a seguir: 25. (AFA) Analise as proposições abaixo classificando-se em V
(verdadeiro) ou F (falso), considerando funções reais.
( =
) O domínio e a imagem da função g definida por g(x) 9 − x2
são, respectivamente [–3, 3] e [0, +∞[
( ) Se f(x) = x2 e g(x) = f(x + m) − f(x) então g(2) é igual a m(4 + m)
1
( ) Se h(x) =, então h-1(x) = h(x)
x
A sequência correta é:
a) F – V – V c) V–F–V
b) F – V – F d) V – V – F

Analisando o gráfico, é INCORRETO afirmar que: 26. (AFA) Sabendo-se que a função real f: D → B definida por
x
a) f(f(1)) = f(0,5) f(x) = é inversível e que D e B são conjuntos os mais amplos
1− x
b) f ( x ) + 1 > 0, ∀x ∈  possíveis, é FALSO afirmar que:
c) f ( 0 ) ≤ f ( x ) , ∀x ∈  a) f é crescente para todo x tal que x < 1 ou x > 1
5 b) a equação da assíntota horizontal de f é y = –1
d) se g(x) = f(x) – 1, então g( −2) =f 
 2 c) se g é tal que g(x) = |f-1(x)|, então não existe x real tal que g(x) = 1
 1
22. (AFA) Considere as funções reais f, g e h tais que: d) f −1(0) + f −1  −  < 0
 2
1
f(x) = mx2 – (m + 2)x + (m + 2), g(x) = e h(x) = x 27. (AFA) Analise o gráfico abaixo da função real g :  →  .
x
Para que a função composta h  g  f(x) tenha domínio D = , deve-
se ter:
2 2
a) m> c) 0<m<
3 3
2
b) −2 < m < d) - 2 < m < 0
3

23. (AFA) Considere as funções reais f e g tal que f(x) = x2 + 1 e que


Se h é uma função real tal que h(x) = g(x) + 2, então, marque
(x + 1) . Sobre a
2
existe a composta de g com f dada por ( gof )=
(x) 2
alternativa verdadeira.
função g, é incorreto afirmar que ela é:
a) (hohoh...oh)(0) = 4
a) par. b) (hohoh)(3) > (hohohoh)(2)
b) sobrejetora.    1
c) tal que g ( x ) ≥ 0∀x ∈  c) Se y=h  h  h     então y ∈ ]2,3[
   2 
d) crescente se x ∈ [1, +∞[
  3 
d) Se x=h  h  h     então x ∈ ]1,2[
   2 
24. (AFA) No gráfico abaixo está representada a função real f: A → B.
Classifique em (V) verdadeira ou (F) falsa cada proposição a seguir 28. (AFA) Considere as funções reais f :  →  e g :  →  cujos
sobre a função f. gráficos estão representados abaixo.

( ) No conjunto A existem apenas 15 números inteiros.


Sobre essas funções, é correto afirmar que
( ) Se B = [–4,4], então f é sobrejetora, mas não é injetora.
a) ∀ x ∈ [0 , 4], g(x) − f(x) > 0
( ) A composta ( f  f  f 
=  f ) ( 4 ) f ( 4 ) ou f ( −4 ) .
b) f(g(0)) – g(f(0)) > 0
( ) f é função par.
g(x) ⋅ f(x)
Tem-se, então, a sequência correta: c) ≤ 0 ∀ x ∈] − ∞ , 0 [ ∪ [4 , 9]
[f(x)]2
a) V – F – V – F c) F–F–V–V
b) F – V – F – V d) V – V – F – F d) ∀ x ∈ [0 , 3] tem-se g(x) ∈ [2 , 3]

75

PM_BOOK16 - MAT.indb 75 25/11/2022 19:08:16


FUNÇÃO COMPOSTA E INVERSA

29. (EN) Sejam f e g funções reais definidas por a) f (f (f (b))) – f (f (–c)) = 2f(–a) c) Se –e < x < –b, então f(x) > 0
4x − 3,se x ≥ 0 x + 1,se x > 2 b) f é uma função ímpar. d) Im(f ) = IR − ] − d, d [
f(x) =  2 e g(x) =  . Sendo assim,
1 − x ,se x ≤ 2
2
x − 3x + 2,se x < 0
32. Considere f e g funções reais. Considere também que existem as
pode-se dizer que (f  g)(x) é definida por
funções compostas fog e gof.
4x + 1, se x > 2 Observe a tabela a seguir.

a) (f  g)(x) = 1 − 4x 2 , se − 1 ≤ x ≤ 1
 4
x + x , se x < −1ou 1 < x ≤ 2
2
x f (x) g(x) f(g(x)) g(f(x))

4x − 1, se x > 2 1 1 b 5 –1

b) (f  g)(x) = 1 − 4x 2 , se − 1 ≤ x < 1 2 –1 0 3 c
 4
x + x , se x < −1ou 1 ≤ x ≤ 2
2

4x + 1, se x ≥ 2 A partir das informações acima, é correto afirmar que o valor de b - c


 é igual a
c) (f  g)(x) = 1 − 4x 2 , se − 1 < x < 1
 4 a) 0 b) 1 c) 2 d) 3
x + x , se x ≤ −1ou 1 ≤ x < 2
2

4x + 1, se x ≥ 2 33. A função real f :  →  está representada no gráfico abaixo.



d) (f  g)(x) = 1 − 4x 2 , se − 1 < x ≤ 1
 4
x + x , se x < −1ou 1 < x < 2
2

4x + 1, se x > 2

e) (f  g)(x) = −1 − 4x 2 , se − 1 ≤ x ≤ 1
 4
x − x , se x < −1ou 1 ≤ x ≤ 2
2

30. (AFA) Considere o gráfico da função real p: A → B

Analise as alternativas abaixo e marque a FALSA.


a) Im(f ) = ] − ∞ ,p]
b) f(x) = −p ⇔ x ∈] − ∞,n] ∪ {0}
c) f (f (f (f (...f (q))))) = p
d) se x ≤ 0 , então f(x) < 0

Analise as alternativas abaixo e, a seguir, marque a FALSA. 34. (AFA) Considere o gráfico da função real g: A → A abaixo e
a) p(x) ≤ 0 ⇔ { x ∈  | x < 0 ou c ≤ x ≤ r } marque (V) verdadeiro ou (F) falso.
b) p(p(p(p(p(r))))) = p(p(p(p(r ))))
c) Existe um único x ∈A tal que p(x) = c
d) Im(p) = { −r}∪] − c,c ]

31. O gráfico abaixo descreve a função real f :  → 

( ) A função g possui exatamente duas raízes.


( ) g(4) = –g(–3)
( ) Im(g) = { −3} ∪ −2,4[
( ) A função definida por h(x) = g(x) + 3 não possui raiz.
( ) (g  g  g    g)( −2) =
2

A sequência correta é
a) F - V - F - F - V c) F-V-F-V-F
Com base no gráfico da função f acima, analise as proposições e
marque a FALSA. b) F - F - V - F - V d) V - V - F - F - V

76

PM_BOOK16 - MAT.indb 76 25/11/2022 19:08:18


FUNÇÃO COMPOSTA E INVERSA

35. (UFJF) Seja f A → B uma função. A imagem inversa de um 05. (AFA 2005) Observe os gráficos abaixo, das funções f e g, definidas
subconjunto Y ⊂ B é o conjunto denotado por f-1(Y) e definido por no intervalo [0,1].
x
f-1(Y) = {x ∈ A | f(x) ∈ Y}. Dada a função f:  → , definida por f ( x ) = ,
o conjunto f-1(N) é igual a: 2
a) {x ∈  | x = 2k, k ∈ }
b) {x ∈  | x = 2k, k ∈ }
c) 
d) 
e) 

Com base nos gráficos, assinale a alternativa FALSA.


EXERCÍCIOS DE

COMBATE
a) g  f  0, 4    g  f  x   , x  0,1 .
b) g  f  0, 05   g  f  0,1 
c) g  g  x    x, x  0, 3; 0, 8
01. (ESPCEX) Considere a função real f(x), cujo gráfico está d) g  f  0, 6    g  f 1 
representado na figura, e a função real g(x), definida por g(x) = f(x - 1) + 1.
06. (AFA 2006) Dadas as funções reais f e g definidas por
x
Y f(x) = x 2 - 5x + 6 f(x) = g(x) = e , sabendo-se que existe
x
( g  f )(x), pode-se afirmar que o domínio de g  f é:
2 a) R – ]2,3[ c) R – {2,3}
b) R – [2,3] d) R* – [2,3]

07. (AFA 2012) Considere a figura abaixo que representa um esboço


X do gráfico da função real f:
-3 0

 1
O valor de g  −  :
 2
a) – 3 c) 0 e) 3
b) – 2 d) 2

02. (ESPCEX) Sejam as funções reais f(x) =


x 2 + 4x e g(x) =
x −1. O
domínio da função f(g(x)) é:
Sabe-se que g  x   f  x   3u, h  x   g  x  u e j  x   h  x  .
a) {x ∈ R | x ≤ −3 ou x ≥ 1}
D=
Um esboço do gráfico que melhor representa a função j é:
b) D = {x ∈ R | x ≤ 1}
a)
c) D = {x ∈ R | x ≤ 0 ou x ≥ 4}
d) D = {x ∈ R | −3 ≤ x ≤ 1}
e) D= {x ∈ R | 0 ≤ x ≤ 4}

03. (ESPCEX) Considere a função bijetora ƒ: [1, +∞) → (–∞, 3],


definida por ƒ(X) = - X2 + 2X + 2 e seja (a, b) o ponto de intersecção b)
de f com sua inversa. O valor numérico da expressão (a + b) é:
a) 2 c) 6 e) 10
b) 4 d) 8

04. (AFA 2004) Considere as funções reais


c)
4 x 2  6x  1 se x  1 e g(x) – 2x –3.
 f  g  x   
 4 x  3 se x  1
Com base nessas funções classifique as afirmativas abaixo em
VERDADEIRA(S) ou FALSA(S).
I. f(x) é par.
II. f(x) admite inversa em todo seu domínio.
d)
III. f(x) é crescente em x   | x  1 e em x   | x  1.
IV. se x < -6 então f(x) > -3.
A sequência correta é:
a) V, V, F, V b) F, F, V, F c) F, F, V, V d) F, V, V, F

77

PM_BOOK16 - MAT.indb 77 25/11/2022 19:08:27


FUNÇÃO COMPOSTA E INVERSA

08. (IFC 2009) Dada a função f  x   x  2x  3 , definida para x ≥ 1,


2

a expressão da sua função inversa f x  é:


 1  
5 (IME) Definimos a função f :  →  da seguinte forma:

f(0) = 0

f(1) = 1
a) f 1  x   1 x  4  x  4  
=f(2n) f(n), n ≥ 1
b) f 1  x   1 x  2  x  2 f(2n + 1)
 = n2 , n ≥ 1
c) f 1  x   2x  3x
d) f 1  x   1 x  4  x  4 
Definimos a função g :  →  da seguinte forma:
g(n) = f(n)f(n + 1)
09. (AFA 1997) Seja f  1,     3,   a função definida por
Podemos afirmar que:
f  x   3x 2  6x. Se g   3,    1,   é a função inversa de f, então
a) g é uma função sobrejetora.
g 6  g  32 é:
b) g é uma função injetora.
a) 5 c) 5−2 6
c) f é uma função sobrejetora.
b) 2 6 d) 5  2 6 d) f é uma função injetora.
e) g(2018) tem mais do que 4 divisores positivos.
10. (AFA 2007) A função f definida por
 x 2  4 x  7, se x  2

f x   2x  1, se  1  x  2. GABARITO
 2
  x  2x  4, se x  1
EXERCÍCIOS DE FIXAÇÃO
a) não admite inversa porque não é injetora. 01. DISCURSIVA
b) admite inversa e uma das sentenças que define a mesma é 02. Dom = R – [0,5; 2] e Dom = R – ( −∞,1)
y  1  x  3 se x  3. 03. m = 2 / (a – 1)
c) não admite inversa porque existem valores de x com várias imagens. (x 2 + 2)
04. g(x) =
d) admite inversa f tal que f  5  2.
−1 1
3
05. 0 07. B 09. C
06. D 08. D 10. B
EXERCÍCIOS DE TREINAMENTO

DESAFIO PRO
01. A 10. C 19. D 28. C
02. A 11. B 20. B 29. A
e) . 12. D 21. B 30. B
03. D
04. D 13. B 22. A 31. D

1 (IME) Sejam as funções f :  →  , g :  →  , h :  → . A


alternativa que apresenta a condição necessária para que
se f(g(x)) = f(h(x)), então g(x) = h(x) é:
05. D
06. B
14. D
15. B
23. B
24. A
32. C
33. B
07. D 16. D 25. B 34. A
a) f(x) = x
08. C 17. B 26. C 35. A
b) f(f(x)) = f(x)
09. D 18. B 27. C
c) f é bijetora
EXERCÍCIOS DE COMBATE
d) f é sobrejetora
01. D 04. D 07. C 10. A
e) f é injetora
02. A 05. B 08. A

2 (ITA) Considere as funções f, g :  →  dadas por


f(x) = ax + b e g(x) = cx + d, com a, b, c, d ∈  , . Se
03. B
DESAFIO PRO
06. C 09. E

f −1  g−1 = g−1  f −1 , então uma relação entre as constantes a, b, 01. E 04. log3 (x + x 2 + 1).
c e d é dada por: 02. A 05. E
a) b + ad = d + bc 1
03. −
b) d + ba = c + db 2015
c) a + db = b + cd
d) b + ac = d + ba ANOTAÇÕES
e) c + da = b + cd

3 (IME) Sejam as funções fn, para n ∈ {0, 1, 2, 3, ...}, tais


que: f0 (x) =
1
e fn (x) = f0 (fn −1(x)), para n ≥ 1. Calcule
1− x
f2016 (2016).
3x − 3− x
4 (ITA) Analise se a função f :  →  , f(x) = é bijetora
2
e, em caso afirmativo, determine a função inversa f-1.

78

PM_BOOK16 - MAT.indb 78 25/11/2022 19:08:39


FUNÇÃO AFIM

DEFINIÇÃO y y=-x-2

Uma função do 1º grau (ou função afim) é dada pela lei de


formação f(x) = ax + b , com a, b ∈  e a ≠ 0. O número a é chamado 4
de coeficiente angular de f e b é dito coeficiente linear.
3
Exemplo:
2
f1  x   5x  3, f2  x   3x  4 1

Se a for igual a 0, a função é chamada de função constante, 0


x
-3 -2 -1 1 2 3 4
exemplo y = 2. -1

O gráfico da função constante é sempre uma reta paralela ao eixo -2

x, ou o próprio eixo x, no caso da reta y = 0. -3

ProBizu -4

-5
O gráfico de qualquer função constante, com domínio R, é uma
reta horizontal. -6

Observação
Observe que, nessa função, quanto maior é o valor de x, maior
As raízes (ou zeros) de uma função f real são os valores que anulam o valor de y.
3 Você pode ver isso na tabela, no gráfico ou na lei y = 2x – 2. (Aqui
tal função. Por exemplo, a raiz de 5x + 3 é x   . De forma mais
5 indica-se que x é adicionado. Assim, aumentando x, aumenta-se o
b valor que será adicionado. Portanto o valor de y diminuirá).
geral, a ÚNICA raiz de f(x) = ax + b é x   . Dizemos que uma
a
Essa função do 1° grau dada por y = – x – 2, é, portanto,
função afim é linear se b = 0, ou seja, uma função é linear se é da
decrescente.
forma f(x) = ax, a ≠ 0.
Os exemplos que acabamos de ver são casos particulares de uma
situação geral em que valem as seguintes afirmações:
GRÁFICO Observação
Exemplo 1:
1º Chama-se função crescente aquela em que, aumentando o
Gráfico da função do 1° grau, com domínio R dada por y = 2x – 2. valor de x, sempre aumenta o valor de y.
2º Chama-se função decrescente aquela em que, aumentando o
x y = 2x – 2 valor de x, sempre diminui o valor de y.

–2 –6 Exemplo 2:
Gráfico da função do 1° grau, com domínio R dada por y = – x – 2.
–1 –4 x y=–x–2
–4 2
0 –2
–3 1
–2 0
1 0
–1 –1

2 2 0 –2
1 –3
3 4 2 –4

79

PM_BOOK16 - MAT.indb 79 25/11/2022 19:08:41


FUNÇÃO AFIM

y
COEFICIENTE ANGULAR E
COEFICIENTE LINEAR
4
y=-x-2
O coeficiente a é chamado coeficiente angular e representa a taxa
∆y
3 de variação média da função que é igual à tangente do ângulo de
∆x
2 inclinação da reta. Sendo θ o ângulo de inclinação da reta, tem-se
1
-1 1 2 3 4 y y 2  y1
tg    a (coeficiente angular)
-4 -3 -2
0
x x x 2  x1
-1

-2 a>0 θ é agudo função crescente


a<0 θ é obtuso função decrescente
-3

-4 O coeficiente b é chamado coeficiente linear e é o ponto onde a


reta cruza o eixo 0y, ou seja, a reta passa no ponto (0, b).

Observe que, nessa função, quanto maior é o valor de x, menor ESTUDO DO SINAL
o valor de y. Com isso, veremos agora como estudar o sinal de uma função afim,
Você pode ver isso na tabela, no gráfico ou na lei y = – x o que será extremamente útil na sequência do curso, quando estudarmos
os quadros de sinais. Mais uma vez, o resultado é bastante simples e pode
Observação ser expresso através de um esquema. Temos o seguinte resultado:
1º O gráfico de qualquer função do 1° grau com domínio R é uma TEOREMA: seja f(x) = ax + b, a ≠ 0, uma função afim. Então:
reta.  b
2º Qualquer função do 1° grau dada por y = ax + b, com a > 0, é x   a  f  x   0
uma função crescente. I. a > 0: 
x   b  f  x   0
3º Qualquer função do 1° grau dada por y = ax + b, com a < 0, é  a
uma função decrescente.
 b
x   a  f  x   0
II. a < 0: 
ProBizu x   b  f  x   0
 a
O gráfico de uma função afim é uma reta. Desta forma, para
efetuar a construção de tal gráfico, basta que conheçamos dois de
seus pontos. Em geral, escolhemos os pontos onde a reta corta o
eixo x e o eixo y.

Observação
No gráfico, o coeficiente angular é igual à tangente do ângulo
de inclinação da reta e o coeficiente linear é igual à ordenada do
ponto de interseção da reta com o eixo das ordenadas (eixo y).

EXERCÍCIOS DE

FIXAÇÃO
Vejamos agora um exemplo usando o nosso bizu:
Construir o gráfico de f(x) = 5x + 10.
A raiz desta função é x = –2. Assim, a nossa reta deve passar pelo
ponto (–2,0). Por outro lado, quando x = 0, a função assume valor
igual a 10. Desta forma, a reta também deve passar pelo ponto (0,10). 01. (CESGRANRIO) O valor de um carro novo é de R$ 9.000,00 e,
Estes dois pontos são suficientes para construirmos o gráfico: com 4 anos de uso, é de R$ 4.000,00. Supondo que o preço caia com
o tempo, segundo uma linha reta, o valor de um carro com 1 ano de
uso é:
a) R$ 8.250,00 c) R$ 7.750,00 e) R$ 7.000,00
b) R$ 8.000,00 d) R$ 7.500,00

02. Na figura a
seguir tem-se o
gráfico da função f,
onde f(x) representa o
preço pago em reais
por x cópias de um
mesmo original, na
Copiadora Reprodux.

80

PM_BOOK16 - MAT.indb 80 25/11/2022 19:08:44


FUNÇÃO AFIM

De acordo com o gráfico, é verdade que o preço pago nessa Copiadora 09. (ESPCEX) O crescimento de um vegetal, sob certas condições e a
por partir de uma determinada altura, segue a função do gráfico abaixo.
a) 228 cópias de um mesmo original é R$ 22,50.
b) 193 cópias de um mesmo original é R$ 9,65.
c) 120 cópias de um mesmo original é R$ 7,50.
d) 100 cópias de um mesmo original é R$ 5,00
e) 75 cópias de um mesmo original é R$ 8,00.

03. Uma pessoa, pesando atualmente 70 kg, deseja voltar ao peso


normal de 56 kg. Suponha que uma dieta alimentar resulte em um
emagrecimento de exatamente 200 g por semana. Fazendo essa
dieta, a pessoa alcançará seu objetivo ao fim de
Mantidas tais condições, pode-se afirmar que a função que representa
a) 67 semanas. d) 70 semanas. o crescimento do vegetal e sua altura no 12° dia são, respectivamente:
b) 68 semanas. e) 71 semanas. 1 12 1 17
c) 69 semanas.
a) h(t) =t − 5 e h = cm. d) h(t) = t + 1 e h = cm.
2 15 4 5
1 5 12 t −5 12
04. Sabendo que os pontos (2, -3) e (-1, 6) pertencem ao gráfico da b) h(t) =t − e h =cm. =
e) h(t) = e h cm.
3 3 5 5 15
função f: IR ∈ IR definida por f(x) = ax + b, determine o valor de b – a.
1 17
c) h(t) =t + 1 e h = cm.
05. (EPCAR) Para que o número x satisfaça simultaneamente as 5 5
desigualdades 3x + 2 < 7 − 2x, 48x ≤ 3x + 10 e 11− 2(x − 3) > 1− 3
(x − 5) é suficiente que 10. O único valor de “x” que verifica a equação, na incógnita “x”,
(x – 2)2 + (x + 1)·(x – 1) = 2(x + 5)2 – 167, é divisor de:
a) –1 < x ≤ 2/9 c) –1 < x < 1
a) 54 c) 97 e) 75
b) 2/9 ≤ x < 1 d) –1 < x < 2/9
b) 12 d) 33
06. Se f(x) = (k – 4)x + 2 é uma função do 1º grau decrescente, então
a) k < 4. c) k = 5. EXERCÍCIOS DE
b) k > 6. d) k = 8.

07. Utilizando o fato de a água congelar a 0° Celsius ou 32° Fahrenheit


TREINAMENTO
e ferver a 100º Celsius ou 212° Fahrenheit, e sabendo que existe uma
relação linear entre as duas escalas de temperaturas, conforme o 01. Uma produtora pretende lançar um filme em fita de vídeo e prevê
gráfico abaixo, pode-se completar adequadamente a tabela abaixo uma venda de 20.000 cópias. O custo fixo de produção do filme foi
com os seguintes valores aproximados ou exatos. R$150.000,00 e o custo por unidade foi de R$ 20,00 (fita virgem,
processo de copiar e embalagem). Qual o preço mínimo que deverá
Celsius –10°
ser cobrado por fita, para não haver prejuízo?
Fahrenheit 0° 68° a) R$ 20,00 d) R$ 27,50
b) R$ 22,50 e) R$ 35,00
c) R$ 25,00

02. Para que as funções f(x) = m(x – 1) + 2x + 3 e g(x) = mx + 1 + 2xm


sejam simultaneamente, crescente e decrescente temos,
respectivamente, que
a) –2 < m < 0 c) –2 ≤ m ≤ 0
b) –2 ≤ m < 0 d) –2 < m ≤ 0

03. O ônibus X parte da cidade A com velocidade constante de


80 km/h, à zero hora de certo dia. Às 2 horas da madrugada, o ônibus
Y parte da mesma cidade, na direção e sentido do ônibus X, com
a) –17,7°; 14°; 20° c) 32°; 90°; 100º velocidade constante de 100 km/h. O ônibus Y vai cruzar com o
ônibus X, pela manhã, às
b) –32°; 42°; 168° d) –18,8°; 50°; 112°
a) 6 horas. c) 10 horas. e) 12 horas.
08. Sabendo que a função y = ax + b, pode-se afirmar que: b) 8 horas. d) 11 horas.
a) O gráfico da função passa sempre pela origem.
x −1
b) O gráfico da função corta sempre o eixo das ordenadas. 04. Sobre a equação kx − =
1, na variável x, é correto afirmar
que: k
b
c) O zero da função é .
a a) admite solução única se k² = 1 e k ∈ *.
d) A função é crescente para a < 0. b) NÃO admite solução única se k = 1.
e) O gráfico da função nunca passa pela origem. c) admite mais de uma solução se k = –1.
d) admite infinitas soluções se k = 0.

81

PM_BOOK16 - MAT.indb 81 25/11/2022 19:08:44


FUNÇÃO AFIM

05. Uma pequena empresa fabrica camisas de um único modelo e as 11. A reta r contém os pontos (0,4) e (7,7). Dos pontos abaixo, qual é
vende por R$ 80,00 a unidade. Devido ao aluguel e a outras despesas o mais próximo da reta r?
fixas que não dependem da quantidade produzida, a empresa tem a) (1999, 858) c) (1999, 860) e) (1999, 862)
um custo fixo anual de R$ 96 000,00. Além do custo fixo, a empresa
tem que arcar com custos que dependem da quantidade produzida, b) (1999, 859) d) (1999, 861)
chamados custos variáveis, tais como matéria-prima, por exemplo; o
custo variável por camisa é R$ 40,00. Em 2009, a empresa lucrou 12. (EPCAR 2°ANO) A reta do gráfico abaixo indica a quantidade de
R$ 60 000,00. Para dobrar o lucro em 2010, em relação ao lucro de soro (em ml) que uma pessoa deve tomar, em função de seu peso
2009, a quantidade vendida em 2010 terá de ser x% maior que a de (dado em Kgf), num tratamento de imunização. A quantidade total de
2009. O valor mais próximo de x é: soro a ser tomada será dividida em 10 injeções idênticas. Quantos ml
de soro receberá um indivíduo de 65 Kgf em cada aplicação?
a) 120 c) 80 e) 40
b) 100 d) 60
a) 20

06. A prefeitura de uma cidade concede benefícios fiscais às indústrias b) 40


que lá se instalam. Para obter os benefícios, o número de empregados c) 2
que reside na cidade deve ser, no mínimo, o dobro mais 5% do d) 4
número de empregados que não residem nela. Uma indústria que
contratou 80 funcionários que residem fora da cidade deve contratar,
entre os moradores da cidade, um número mínimo de
a) 160 funcionários. d) 164 funcionários.
b) 166 funcionários. e) 178 funcionários.
c) 176 funcionários. 13. (EPCAR 3° ano) Em um jogo de futebol amistoso entre Brasil e
Argentina, no Mineirão, compareceram 90.000 torcedores. Quatro
portões foram abertos às 12 horas, e até as 14 horas entrou um
07. (FATEC) Na figura abaixo, a reta r tem equação x + 3y – 6 = 0,
2 número constante de pessoas por minuto. Entre 14 horas e 15
e a reta s passa pela origem e tem coeficiente angular . A área do horas não entrou ninguém. Às 15 horas, abriram mais 4 portões,
triângulo OAB, em unidades de área, é igual a: 3
aumentando o fluxo de pessoas e, às 17 horas, os portões foram
fechados. O gráfico abaixo indica o número de pessoas dentro do
a) 1 estádio em função do horário de entrada.
b) 2 nº de pessoas
c) 3
90.000
d) 4
78.000

78.000

08. (ESPCEX) Sabendo que a função y = ax + b, pode-se afirmar que: 12 14 15 17 horário


a) O gráfico da função passa sempre pela origem.
Com base nisso, pode-se dizer que, quando o número de pessoas
b) O gráfico da função corta sempre o eixo das ordenadas.
atingiu 78.000, o relógio marcava
b
c) O zero da função é . a) 15 horas e 30 minutos. d) 16 horas e 30 minutos.
a
d) A função é crescente para a < 0. b) 15 horas e 45 minutos. e) 16 horas e 45 minutos.
e) O gráfico da função nunca passa pela origem. c) 16 horas.

09. A variação de temperatura y = f(x) num intervalo de tempo x é 14. (AFA) Na figura abaixo, tem-se representado as funções f, g e
dada pela função f(x) = (m² – 9)x² + (m + 3)x + m – 3. Determine o h que indicam os valores pagos, respectivamente, às locadoras de
valor de m para que o gráfico da função seja uma reta e também para automóveis x ∈ ]m,+∞[, m ∈  para x quilômetros rodados por dia.
que f seja crescente. Uma pessoa pretende alugar um carro e analisa as três opções.
a) – 3 d) – 9
b) 9 e) 0
c) 3

10. (FUVEST) Um estacionamento cobra R$ 6,00 pela primeira hora


de uso, R$ 3,00 por hora adicional e tem uma despesa diária de
R$ 320,00. Considere-se um dia em que sejam cobradas, no total, 80
horas de estacionamento. O número mínimo de usuários necessário
para que o estacionamento obtenha lucro nesse dia é:
a) 25 d) 28
b) 26 e) 29
c) 27

82

PM_BOOK16 - MAT.indb 82 25/11/2022 19:08:45


FUNÇÃO AFIM

Após a análise, essa pessoa conclui que optar pela locadora α ao Assim, no instante t = 10 horas o móvel está a uma velocidade de
invés das outras duas locadoras, é mais vantajoso quando x ∈ ]m,+∞[, 55 km/h, por exemplo. Sabe-se que é possível determinar a distância
m ∈ . O menor valor possível para m é que o móvel percorre calculando a área limitada entre o eixo horizontal
a) 60 c) 80 t e a semirreta que representa a velocidade em função do tempo.
Desta forma, a área hachurada no gráfico fornece a distância, em km,
b) 70 d) 90 percorrida pelo móvel do instante 6 a 10 horas. É correto afirmar que
a distância percorrida pelo móvel, em km, do instante 3 a 9 horas é de
15. (AFA) Na figura abaixo, tem-se o gráfico da função real f em
a) 318 c) 256
que f(x) representa o preço, pago em reais, de x quilogramas de um
determinado produto. (Considere f(x) ∈ ) b) 306 d) 212

19. (ESPCEX) Considere a função real f(x), cujo gráfico está representado
na figura, e a função real g(x), definida por g(x) = f(x – 1) + 1.

O valor de g  −  é
1
 2
a) -3
b) -2
De acordo com o gráfico, é INCORRETO afirmar que
c) 0
a) o preço pago por 30 quilogramas do produto foi R$ 18,00.
d) 2
b) com R$ 110,00, foi possível comprar no máximo 55 quilogramas
do produto. e) 3
c) com R$ 36,00, foi possível comprar 72 quilogramas do produto.
d) com R$ 32,00, compra-se tanto 53,333... quilogramas, quanto 64
quilogramas do produto. 20. (EPCAR) João, ao perceber que seu carro apresentara um defeito,
optou por alugar um veículo para cumprir seus compromissos de
16. (CN 2012) Sejam A = [72011, 112011] e B = {x ∈  | x = (1 – t) · 72011 trabalho. A locadora, então, lhe apresentou duas propostas:
+ t · 112011 com t ∈ [0,1]}, o conjunto A – B é - plano A, no qual é cobrado um valor fixo de R$ 50,00 e mais
a) A ∩ B c) A – {72011} e) ∅ R$ 1,60 por quilômetro rodado.
b) B – {112011} d) A - plano B, no qual é cobrado um valor fixo de R$ 64,00 mais R$ 1,20
por quilômetro rodado.
17. (CMRJ) Considere a função afim f, representada no gráfico abaixo. João observou que, para certo deslocamento que totalizava k
Sabendo-se que A (3, 1); B(0, 1) e que C é o ponto de interseção do quilômetros, era indiferente optar pelo plano A ou pelo plano B, pois
gráfico de f com o eixo das ordenadas, a área do triângulo ABC é, em o valor final a ser pago seria o mesmo. É correto afirmar que k é um
unidades de área, igual a: número racional entre
a) 14,5 e 20. c) 25,5 e 31.
a) 10
b) 20 e 25,5. d) 31 e 36,5.
b) 9
c) 8,5 21. (EFOMM 08) Uma churrascaria cobra, num almoço, R$ 10,00 por
d) 7,5 pessoa. Após as 15h, esse valor cai para R$ 8,00. Estima-se que o
custo total de um almoço seja de R$ 6,00 por pessoa. Em certo dia, na
e) 6
churrascaria almoçaram 100 pessoas; x dos quais permaneceram até
as 15h. Assinale a alternativa que representa o intervalo de variação
de x a fim de que 300 < L(lucro) < 400.
a) maior que 100
b) menor que 50
c) entre 50 e 100
d) menor que 50 e maior que 100
18. (EPCAR) O gráfico a seguir é de uma função polinomial do 1º grau e) maior que 50
e descreve a velocidade v de um móvel em função do tempo t:
22. (EFOMM 09) Todos os anos uma fábrica aumenta a produção
em uma quantidade constante. No 5º ano de funcionamento, ela
produziu 1460 peças, e no 8º ano, 1940. Quantas peças, então, ela
produziu no 1º ano de funcionamento?
a) 475 c) 598 e) 820
b) 520 d) 621

23. (AFA 13) Dois corredores partem de um ponto ao mesmo tempo e


se deslocam da seguinte forma: o primeiro é tal, que sua velocidade y1
é dada em função da distância x por ele percorrida através de
 4,se x ≤ 200

y1 =  n n² + n − 8
x− ,se 200n < x ≤ 200 (n + 1)
 200 2

83

PM_BOOK16 - MAT.indb 83 25/11/2022 19:08:45


FUNÇÃO AFIM

Em que n varia no conjunto dos números naturais não nulos. O EXERCÍCIOS DE


segundo é tal que sua velocidade y2 é dada em função da distância
x por ele percorrida através de =
y2
x
100
+ 4. Tais velocidades são COMBATE
marcadas em km/h, e as distâncias, em metros. Assim sendo, ambos
estarão à mesma velocidade após terem percorrido
01. (ESPCEX) A quantidade de combustível gasto por um veículo
a) 800 m c) 1000 m
blindado, por quilômetro rodado, está indicada pelo gráfico abaixo.
b) 900 m d) 1100 m Qual a função que representa o consumo C(d) em relação à distância
d percorrida?
24. (AFA) Para angariar fundos para a formatura, os alunos do 3º ano
do CPCAR vendem bombons no horário do intervalo das aulas. a) C(d) = 0,75d.
Inicialmente, começaram vendendo cada bombom por R$ 4,00. b) C(d) = 0,25d.
Assim, perceberam que vendiam, em média, 50 bombons por dia.
c) C(d) = 1,75d.
A partir dos conhecimentos que os alunos tinham sobre função,
estimaram que para cada 5 centavos de desconto no preço de cada d) C(d) = 1,25d.
bombom (não podendo conceder mais que 70 descontos), seria e) C(d) = 1,20d.
possível vender 5 bombons a mais por dia.
Considere:
- p o preço de cada bombom;
02. (ESPCEX) A questão da reciclagem do alumínio ganha cada vez
- n o número de bombons vendidos, em média, por dia; mais importância nos dias atuais, principalmente pelo fato de que a
- x ∈  o número de reduções de 5 centavos concedidas no preço quantidade de energia necessária para se produzir 1 kg de alumínio
unitário de cada bombom; e por meio de reciclagem corresponde a apenas 5% da energia
- y a arrecadação diária com a venda dos bombons. necessária para obter-se esse mesmo kg de alumínio a partir do
minério. O gráfico a seguir mostra a quantidade de energia necessária
Com base nessas informações, analise as proposições abaixo. para obter-se certa massa de alumínio em função do percentual de
(02) O gráfico que expressa n em função de p está contido no alumínio reciclado existente nessa massa.
segmento AB do gráfico abaixo.

(04) A maior arrecadação diária possível com a venda dos bombons,


considerando os descontos de 5 centavos, ocorre quando concederem
35 descontos de 5 centavos. Identificando a energia consumida por E e a porcentagem de alumínio
(08) Se forem concedidos 20 descontos de 5 centavos, serão vendidos reciclado por P, pode-se afirmar que a função que representa esse
mais de 100 bombons por dia. processo, seu domínio e sua imagem são, respectivamente
A soma das proposições verdadeiras é igual a 19
a) E = - P + 200; [0,100]; [10,200]
10
a) 6
21
b) 10 b) E=- P + 200; [0,100]; [10,200]
10
c) 12
19
d) 14 c) E=- P + 200; [0,100]; [10,210]
10
21
25. (ITA) Considere as funções f, g:  → , f(x) = ax + m, g(x) = bx + n, d) E=- P + 200; [0,100]; [0,210]
em que a, b, m e n são constantes reais. Se A e B são as imagens de f e 10
de g, respectivamente, então, das afirmações abaixo: 21
e) E=- P + 200; [10,210]; [0,100]
I. Se A = B, então a = b e m = n; 10
II. Se A = , então a = 1;
III. Se a, b, m, n ∈ , com a = b e m = –n, então A = B 03. (ESPCEX) Dada uma função do 1º grau ƒ:  → , tal que
f(x) = ax + b; a ≠ 0; a, b ∈ . A função f é decrescente e seu gráfico
É (são) verdadeira(s)
corta o eixo das ordenadas no ponto (0, 4). Sabendo-se que a região
a) apenas I. delimitada pelos eixos coordenados e a representação gráfica de f tem
b) apenas II. área igual a 20 unidades de área, a soma de a + b é igual a
c) apenas III. a) – 2/5 d) 16/5
d) apenas I e II. b) 0 e) 18/5
e) nenhuma. c) 12/5

84

PM_BOOK16 - MAT.indb 84 25/11/2022 19:08:46


FUNÇÃO AFIM

04. (ESPCEX) Na figura a seguir, são fornecidas as coordenadas 09. (EFOMM 07) Uma empresa mercante A paga R$ 1.000,00 fixos
cartesianas dos pontos P1 e P2. Denomina-se θ o ângulo P1ÔP2. mais R$ 600,00 por dia de viagem e uma empresa B R$ 400,00 fixos
mais R$ 800,00 por dia de viagem. Sabe-se que Marcos trabalha na
empresa A e Cláudio na B e obtiveram o mesmo valor salarial. Quantos
dias eles ficaram embarcados?
a) 1 d) 7
b) 3 e) 9
c) 5

10. (AFA 2011) Luiza possui uma pequena confecção artesanal de


bolsas. No gráfico abaixo, a reta c representa o custo total mensal com
a confecção de x bolsas e a reta f representa o faturamento mensal de
Luiza com a confecção de x bolsas.

Com base nessas informações pode-se afirmar que o valor de cosθ é


4 3−3 c) 3 3−4 4+3 3
a) e)
10 10 10
13 3
b) d)
10 10

05. (ESPCEX) Na figura abaixo está representado o gráfico de uma


função real do 1º grau f(x). A expressão algébrica que define a função
inversa de f(x) é
Com base nos dados acima, é correto afirmar que Luiza obtém lucro
x se, e somente se, vender
a) y = +1
2 a) no mínimo 2 bolsas.
1 b) pelo menos 1 bolsa.
b) y = x+
2 c) exatamente 3 bolsas.
c) y = 2x – 2
d) no mínimo 4 bolsas.
d) y = –2x + 2
e) y = 2x + 2

06. Determine o valor de m para que a equação m2x – m + 1 = 3mx – 2x DESAFIO PRO
possua infinitas soluções.
a) 0 c) 2 e) 4
x −1 x −k
b) 1 d) 3
1 Na equação =
x−2 x−6
, na variável x, k é um parâmetro
real. O produto dos valores de k para os quais essa equação
07. (EFOMM 2002) Determine o coeficiente angular da reta cujas não apresenta solução real em x é
equações são dadas por x = 2t – 1 e y = t + 2, onde t é real.
a) 10. d) 24.
a) – 1 c) 2/5 e) 1
b) 12. e) 30.
b) – 1/2 d) 1/2
c) 20.

08. (EFOMM 2004) Calcule o coeficiente angular da reta s


representada no gráfico.
2 (IME) Um curso oferece as disciplinas A, B, C e D. Foram
feitas as matrículas dos alunos da seguinte forma:
a) – 1 – 6 alunos se matricularam na disciplina A;
b) 0 – 5 alunos se matricularam na disciplina B;
c) 1 – 5 alunos se matricularam na disciplina C; e

2 – 4 alunos se matricularam na disciplina D.


d)
Sabe-se que cada aluno se matriculou em, no mínimo, 3
e) 3 disciplinas. Determine a quantidade mínima de alunos que se
matricularam nas 4 disciplinas.
a) 0
b) 1
c) 2
d) 3
e) 4

85

PM_BOOK16 - MAT.indb 85 25/11/2022 19:08:47


FUNÇÃO AFIM

3 (EPCAR) Analise as afirmativas seguintes e classifique-as


em V (verdadeira) ou F (falsa). ANOTAÇÕES
( ) Considere dois números pares, consecutivos e não nulos. O
produto da soma dos inversos desses números pela metade
do maior entre eles é um quociente entre dois números
inteiros consecutivos.
( ) Para todo a ∈  e para todo b ∈  existe x ∈  tal que
3x – a = 5bx + 5b.
( ) Se m é um número inteiro, ímpar e m < –3, então o menor
valor para x, no conjunto solução da inequação m(m + x) ≤
–3(x – 3), é um número par positivo.
Tem-se a sequência correta em
a) V - F - V c) F - V - F
b) F - V - V d) V - F - F

4 (CN) A equação k²x – kx = k² – 2k – 8 + 12x, na variável


x, é impossível. Sabe-se que a equação na variável y dada
a − 114y 17b + 2
por 3ay + =admite infinitas soluções. Calcule o
2 2
ab + k
valor de , e assinale a opção correta.
4
a) 0 b) 1 c) 3 d) 4 e) 5

5 (IME) Sejam r, s, t e v números inteiros positivos tais que


r t
< . Considere as seguintes relações:
s v
I. (r + s) (t + v)
<
s v
r t
II. <
(r + s) (t + v)
r (r + t)
III. <
s s+v
(r + t) (r + t)
IV. <
s v
O número total de relações que estão corretas é:
a) 0 c) 2 e) 4
b) 1 d) 3

GABARITO
EXERCÍCIOS DE FIXAÇÃO
01. C 04. 6 07. A 10. E
02. B 05. A 08. B
03. D 06. A 09. C
EXERCÍCIOS DE TREINAMENTO
01. D 08. B 15. B 22. E
02. A 09. C 16. E 23. C
03. C 10. C 17. B 24. D
04. B 11. D 18. A 25. E
05. E 12. D 19. D
06. D 13. D 20. D
07. D 14. A 21. D
EXERCÍCIOS DE COMBATE
01. A 04. A 07. D 10. B
02. A 05. C 08. A
03. E 06. B 09. B
DESAFIO PRO
01. E 03. A 05. D
02. C 04. D

86

PM_BOOK16 - MAT.indb 86 25/11/2022 19:08:47


FUNÇÃO QUADRÁTICA

DEFINIÇÃO INTERSECÇÃO DA COM EIXO OY


É uma função da forma f(x) = ax² + bx + c, com a, b, c ∈  e a ≠ 0. 
Quando buscamos a intersecção com o eixo Oy estamos buscando
o ponto da forma (0,f0), ou seja, o valor da função f para x = 0. Dessa
Observação
forma temos f0 = a ⋅ 02 + 0⋅x + c, sendo f0 = c, portanto o ponto em
a é dito coeficiente líder da função quadrática. que a parábola corta o eixo das ordenadas é o ponto (0,c).
Exemplo:
f(x) = 4x² + 5x – 8

GRÁFICO
O gráfico de uma função quadrática é uma parábola.

Observação
Quando a > 0, a parábola tem concavidade voltada para CIMA e
quando a < 0, a parábola tem concavidade voltada para BAIXO.

DISCUSSÃO SOBRE AS RAÍZES REAIS


a>0 Dada uma equação do 2º grau com coeficientes reais, temos:
a<0 I. ∆ > 0: a equação possui duas raízes reais distintas.
II. ∆ = 0: a equação possui duas raízes reais iguais (raiz dupla).
III. ∆ < 0: a equação não possui raízes reais.
Cada sinal do determinante irá determinar uma
 maneira diferente
que a parábola pode ou não interceptar o eixo Ox.
I.
ProBizu
Para memorizar mais facilmente a concavidade da parábola,
podemos pensar no seguinte “macete”:
Quando a > 0, pensamos que “a parábola está feliz” e assim a
concavidade é para CIMA.
Quando a < 0, pensamos que “a parábola está triste” e assim a
concavidade é para BAIXO.

RAÍZES
Para resolver uma equação da forma ax² + bx + c = 0, usaremos a
técnica de completar quadrados, vista no nosso módulo de fatoração II.
ax² + bx + c = 0 ⇔ 4a²x² + 4abx + 4ac = 0
Somando b² - 4ac a ambos lados, temos que (2ax + b)² = b² – 4ac.
Sendo b² – 4ac = ∆ (discriminante), obtemos a (Fórmula de Bhaskara)
As raízes da equação do 2º grau ax² + bx + c = 0, a ≠ 0, são dadas por
−b ± ∆
, onde ∆ = b² – 4ac.
2a
Observação
A fórmula de Bhaskara também é verdadeira para equações com
coeficientes complexos.

87

PM_BOOK16 - MAT.indb 87 25/11/2022 19:08:48


FUNÇÃO QUADRÁTICA

III. II. Se a < 0, a função quadrática y = ax² + bx + c admite valor



máximo y V = − (“y do vértice”) e tal valor máximo ocorre para
4a
b
x = xV = − (“x do vértice”). Neste caso, o “x do vértice” é dito
2a
maximizante.

RELAÇÕES ENTRE
COEFICIENTES E RAÍZES
Dada uma equação do segundo grau ax² + bx + c = 0 de raízes x1
b c
e x2, temos que S =x1 + x 2 =
− ,P =x1x 2 =.
a a
Para demonstrarmos estas fórmulas, basta usar a fórmula de
Bhaskara:
−b + ∆ −b − ∆ Observação
=x1 = e x2
2a 2a
Sabemos que 2m + n = 8, com m e n números reais. Devemos
−b + ∆ + −b − ∆ 2b b determinar o par (m,n) que torna o produto m·n máximo.
Logo, S =
x1 + x 2 = =
− =
− .
2a 2a a Veja que podemos isolar n em função de m.
n = 8 – 2m
( −b + ∆ )( −b − ∆ ) b − ∆ 4ac c
2
=
Por fim, P x=
1x 2 = = = . Dessa forma vamos chamar o produto de P e assim
4a2 4a2 4a2 a
P = mn = m(8 – 2m) = –2m2 + 8m
ProBizu Acabamos
 de criar uma função
 a por P, onde m “fará papel de eixo
Ox“ e P “fará papel de eixo Oy”. Como nossa função possui a > 0
Fique atento às seguintes ideias que despencam em prova: teremos realmente um ponto máximo.
x12  x 22   x1  x 2   2x1x 2  S2  2P
2

x13  x 23   x1  x 2   3x1x 2  x1  x 2   S3  3SP .


3

MÁXIMOS E MÍNIMOS
Muitos problemas que aparecem nas provas militares envolvem
minimizar ou maximizar uma função quadrática. Vejamos agora como
calcular os máximos e mínimos de uma função quadrática:
I. Se a > 0, a função quadrática y = ax² + bx + c admite valor

mínimo y V = − (“y do vértice”) e tal valor mínimo ocorre para
4a
b
x = xV = − (“x do vértice”). Neste caso, o “x do vértice” é dito
2a
minimizante.
Usando a ideia de x do vértice encontraremos o valor de m que
torna o produto P máximo.
b 8
x v ==
m − = − =
2
2a 2 ( −2)
Assim m sendo igual a 2 teremos n = 8 – 2 · 2 = 4.
Assim o produto máximo será P = mn = 2 · 4 = 8.
Note que não calculamos yv pois não era nosso interesse o produto
máximo mas sim os valores que tornavam máximo esse produto. Se
precisássemos calcular o produto máximo poderíamos ter utilizado

yv = −

=−
( 82 − 4.( −2) .0) =−
64
= 8.
4a 4 ( −2) −8
Note que o valor máximo ou mínimo também pode ser encontrando
fazendo yv = f(x)v. Assim yv = f(2) = -2(2)2 + 8 · 2 = –2 · 4 + 16 = 8.

88

PM_BOOK16 - MAT.indb 88 25/11/2022 19:08:51


FUNÇÃO QUADRÁTICA

EIXO DE SIMETRIA
O gráfico da função quadrática admite um eixo de simetria perpendicular ao eixo Ox e (paralelo a y) que passa pelo vértice.

b 
xv  e yv 
2a 4a

FORMA FATORADA
Se f(x) = ax² + bx + c possui raízes r1 e r2, podemos fatorar f(x) = a(x – r1) (x – r2).
Exemplo:
1  1
Seja f(x) = 6x² – 5x – 1. As raízes desta função quadrática são 1 e − . Assim, podemos fatorar tal função como 6 ( x − 1)  x +  =( x − 1)( 6x + 1) .
6  6

ESTUDO DO SINAL
Seguem os tipos de gráficos que podemos obter.

Esses foram os gráficos para a > 0, agora veremos para a < 0.

89

PM_BOOK16 - MAT.indb 89 25/11/2022 19:08:52


FUNÇÃO QUADRÁTICA

Exercício Resolvido Exercício Resolvido

01. Determine o valor de k para que as raízes da equação do 03. O gráfico de um trinômio do 2º grau y tem concavidade para
segundo grau (k – 5)x² – 4kx + k – 2 = 0 sejam o seno e o cosseno cima e intersecta o eixo das abscissas em dois pontos à direita da
de um mesmo arco. origem. O trinômio –y tem um valor
a) mínimo e raízes positivas
Resolução:
b) mínimo e raízes negativas
Sendo x1 = senα e x2 = cosα as raízes da equação, temos pela
relação fundamental da trigonometria que x1 + x 2 =
2 2
1. c) máximo e raízes positivas
d) máximo e raízes negativas
Vimos no PROBIZU que x12 + x 22 = ( x1 + x 2 ) − 2x1x 2 = S2 − 2P.
2

e) máximo e raízes de sinais opostos


4k k−2
Veja agora=
que S = eP .
k −5 k −5 Resolução: C
2
 4k  k−2 Se y possui concavidade para cima, –y possui concavidade para
Logo   − 2⋅ =1. Multiplicando tudo por (k – 5)²,
k −5 k −5 baixo e assim admite máximo. Por outro lado, as raízes de –y são as
obtemos: mesmas que as raízes de y. Como o enunciado disse que as raízes
16k² – 2(k – 2)(k – 5) = (k – 5)² ⇔ 13k² + 24k – 45 = 0 estão à direita da origem, estas são positivas.
Resolvendo a equação do segundo grau, obtemos k = –3 ou Assim, –y admite valor máximo e possui raízes positivas.
15
k= .
13
Devemos verificar ainda se para os valores obtidos para k, as raízes
Exercício Resolvido
encontradas são reais ou não (lembre que seno e cosseno são
números reais!). 04. O produto de dois números reais x e y é igual a 150. Assim
O discriminante da equação original é (4k)² – 4 · (k – 5)(k – 2) = sendo, x + y NÃO pode ser igual a
12k² + 28k – 40. É fácil ver que para k = –3, o discriminante é a) 31,71
15
negativo e para k = , o discriminante é positivo. b) 28,27
13
15 c) 25,15
Assim o único valor de k é .
13 d) 24,35
e) –26,95

Exercício Resolvido Resolução: D


Sendo S a soma dos números x e y, temos que a equação do
02. O conjunto de todos os valores de m para os quais a função segundo grau t² – St + 150 = 0 admite x e y como raízes.
x 2 + (2m + 3)x + (m2 + 3) Como x e y são números reais, o discriminante deve ser não
f(x) = está definida e é não negativa
x 2 + (2m + 1)x + m2 + 2 negativo:
para todo x real é: S² – 4 · 1 · 150 ≥ 0 ⇔ S² ≥ 600
1 7 Assim S ≤ − 600 ou S ≥ 600. Como 600 ≅ 24,495, temos que
a)  4 , 4  x + y NÃO pode ser igual a 24,35.
1 
b)  4 , +∞ 

c)
 7 EXERCÍCIOS DE
 0, 4 

d)
 1
 −∞, 4 
FIXAÇÃO
1 7
e)  4 , 4  01. Determinar os zeros reais das funções.
a) f(x) = x² – 3x + 2 c) f(x) = x² + 4x + 4
Resolução: D
b) f(x) = –x² + 7x – 12 d) f(x) = –3x² + 6
Para que a função esteja definida e seja não negativa para todo x
real, devemos ter:
02. Determinar os valores de m para que a função f(x) = mx² + (m + 1)x
I) NUMERADOR não negativo para todo x real + (m – 1) tenha um zero real duplo.
II) RADICANDO DO NUMERADOR positivo para todo x real
Assim, temos: 03. Na equação do segundo grau 2x² – 5x – 1 = 0 de raízes x1 e x2,
calcular.
I) ∆NUMERADOR ≤ 0 :
a) x1 + x2
1
Logo (2m + 3)2 − 4 (m2 + 3=
) 12m − 3 ≤ 0 ⇔ m ≤ b) x1 · x2
4
II) ∆DENOMINADOR < 0: c) 1/x1 + 1/x2
7
Logo (2m + 1)2 − 4 (m2 + 2)= 4m − 7 < 0 ⇔ m < 04. (CFT) Uma função real f tem a propriedade f(x + 1) = x² + 2, para
4
1 x ∈ R. O valor de f(3) é
Juntando as inequações obtidas, segue que m ≤ .
4 a) 3. b) 4. c) 5. d) 6.

90

PM_BOOK16 - MAT.indb 90 25/11/2022 19:08:53


FUNÇÃO QUADRÁTICA

05. (CFT) Considerando-se a função f(x) = ax² + bx + 1/3, cujos zeros 04. (EEAR) Seja a função quadrática f(x) = ax² + bx + 1. Se f(1) = 0 e
são – 1/3 e 1/3, pode-se afirmar que f(–1) = 6, então o valor de a é
a) a ≠ 0 e b ≠ 0. c) a ≠ 0 e b = 0. a) 5 b) 4 c) 3 d) 2
b) a = 0 e b ≠ 0. d) a = 0 e b = 0.
05. (EEAR) Seja a função f(x) = 2x² + 8x + 5. Se P(a,b) é o vértice do
06. (CFT) Sejam a função f(x) = 2x² – 5x + 2 e o intervalo A = ]0, 2[. gráfico de f, então |a + b| é igual a
Se x ∈ A, a função f. a) 5 b) 4 c) 3 d) 2
a) é crescente para x < 1/2 e decrescente para x > 1/2.
b) é sempre crescente. 06. (AFA) Para que o valor mínimo da função y = x2 – 4x + k seja igual
a –1, o valor de k é:
c) tem uma raiz real.
a) 1 b) 2 c) 3 d) 4
d) tem duas raízes reais.
07. (EFOMM) Examine a função real f(x) = 2x – 3x² quanto à existência
07. (CFT) Seja a parábola que representa a função y = kx² – x + 1. Os de valores e pontos de máximos e mínimos. Analise o problema e
valores de k, para os quais essa parábola não intercepta o eixo das assinale a alternativa CORRETA.
abscissas, são tais que
a) A função atinge o valor máximo de 2/3, no ponto x = 1/3.
a) k > 1/4.
b) A função atinge o valor mínimo de 1/3, no ponto x = 1/3.
b) k > –4.
c) A função atinge o valor máximo de 1/3, no ponto x = 2/3.
c) –4 < k < 1/4.
d) A função atinge o valor mínimo de 2/3, no ponto x = 1/3.
d) –1/4 < k < 4.
e) A função atinge o valor máximo de 1/3, no ponto x = 1/3.
08. A equação do 2º grau, (m + 1)x² – 2mx + (m – 1), de incógnita x,
08. (EFOMM) Considere a função real f(x) = 1 + 4x + 2x². Determine
1 1
possui + =3 . Sendo assim o valor de m² é o ponto x* que define o valor mínimo dessa função.
x1 x 2
a) x* = -2 c) x* = -1/2 e) x* = 1
a) 1 c) 16 e) 9
b) x* = -1 d) x* = zero
b) 4 d) 25

09. (EFOMM) Numa equação, encontramos o valor de 884. Para


09. (EEAR) O vértice da parábola y = –x² + 6x – 5 é o ponto cuja
chegar a esse resultado, somamos os quadrados de dois números
ordenada é
pares, consecutivos e positivos. Determine o quociente da divisão do
a) –2 c) 4 maior pelo menor
b) –1 d) 6 a) 0,87. c) 1,03. e) 1,10.
b) 0,95. d) 1,07.
10. Na parábola descrita pela função de  →  f(x) = x² + bx + 1,
b ∈ , o vértice tem coordenadas (–0,5; –0,75). Então:
10. (EFOMM) Seja f(k) = k² + 3k + 2 e seja W o conjunto de inteiros
a) b < –1. {0,1,2,...,25}. O número de elementos de W, tais que f(W) deixa resto
b) –1 ≤ b < 0. zero, quando dividido por 6, é:
c) 0 ≤ b < 1. a) 25 c) 21 e) 17
d) 1 ≤ b < 2. b) 22 d) 18

11. (AFA) Corta-se um pedaço de arame de comprimento 98 cm


EXERCÍCIOS DE em duas partes. Com uma, faz-se um quadrado, com a outra, um

TREINAMENTO retângulo com base e altura na razão de 3 para 2. Se a soma das áreas
compreendidas pelas duas figuras for mínima, o comprimento, em
cm, do arame destinado à construção do quadrado será
a) 36 c) 50
01. (EEAR) Resolvendo a inequação (2x – 6)(4x + 8) ≤ 0, para x ∈ , b) 48 d) 54
obtemos
a) –2 < x < 3 c) –6 < x < 1 12. (AFA) A função y = ax2 + bx + c assume valor máximo de 8 para
b) –2 ≤ x ≤ 3 d) –6 ≤ x ≤ 1 x = 2 e valor 6 para x = 0. O valor de a + b + c é:
15 13
a) b) 8 c) d) –8
02. (EEAR) A função do 2º grau que descreve o gráfico abaixo é 2 2

a) f(x) = x² – x + 6 13. (ESPCEX) Seja a função real f(x) = (m² – 4)x² – (m + 2)x + 1. Das
b) f(x) = x² + 5x - 6 afirmações abaixo:
c) f(x) = -x² – 5x + 6 I. f é função afim para m = 2.
d) f(x) = x² – 5x + 6 II. f é função constante para m = –2.
III. f é função quadrática para m ≠ 2 e m ≠ –2.
IV. f tem uma raiz igual a 1 para m = 3.
03. (EEAR) O menor valor inteiro positivo que pertence ao conjunto- Estão corretas apenas as afirmações
solução da inequação (–3x² + 12)(x² – 6x + 8) < 0 é o a) I, II e IV. c) II, III e IV. e) I, II, III.
a) 2 b) 3 c) 4 d) 5 b) I e III. d) III e IV.

91

PM_BOOK16 - MAT.indb 91 25/11/2022 19:08:53


FUNÇÃO QUADRÁTICA

b 2
14. (AFA) Seja f(x) = ax² + bx + c (a ≠ 0) uma função real definida
para todo número real. Sabendo-se que existem dois números x1 e x2,
20. (AFA) O polinômio do segundo grau =
y
2
( x + 1) + ax , com
distintos, tais que f(x1) · f(x2) < 0, pode-se afirmar que coeficientes reais, não possui raiz real, se e somente se:

a) f passa necessariamente por um máximo a) a – b < 0 c) b² - 4a > 0

b) f passa necessariamente por um mínimo b) a² - b² < 0 d) b² - 2ab < 0

c) x1 · x2 é necessariamente negativo
21. (ESPCEX) Um curral retangular será construído aproveitando-se
d) b² – 4ac > 0 um muro pré-existente no terreno, por medida de economia. Para
cercar os outros três lados, serão utilizados 600 metros de tela de
− x2 + 6x − 9
arame. Para que a área do curral seja a maior possível, a razão entre as
15. (ESPCEX) O menor valor que a função real y =  
1
pode
 2  suas menor e maior dimensões será:
assumir, é
a) 0,25 c) 0,75 e) 1,25
a) 1 c) 1/2 e) 1/8
b) 0,50 d) 1,00
b) 2 d) 1/4
22. (ESPCEX) Se um retângulo tem base x e perímetro 100, então a
16. (EFOMM) Uma aluna do 3º ano da EFOMM, responsável pelas
área A do retângulo é dada em função de sua base por
vendas dos produtos da SAMM (Sociedade Acadêmica da Marinha
Mercante), percebeu que, com a venda de uma caneca a R$ 9,00, a) A(x) = x² - 50x; 0 < x < 50
em média 300 pessoas compravam, quando colocadas as canecas à b) A(x) = -x² + 50x; 0 < x < 50
venda em um grande evento. Para cada redução de R$ 1,00 no preço c) A(x) = -x² + 100x; 0 < x < 100
da caneca, a venda aumentava em 100 unidades. Assim, o preço da
caneca, para que a receita seja máxima, será de d) A(x) = 2x(x - 50); 0 < x < 50
a) R$ 8,00. d) R$ 5,00. e) A(x) = x(x - 100); 0 < x < 100
b) R$ 7,00. e) R$ 4,00.
23. (AFA) Considere o polinômio p(x) = ax2 + bx + c, satisfazendo as
c) R$ 6,00. condições a < 0, c < 0 e p(1) > 0. Se as suas raízes forem reais, então
elas serão:
17. (EFOMM) De acordo com conceitos administrativos, o lucro de a) nulas c) positivas
uma empresa é dado pela expressão matemática L = R – C, onde
L é o lucro, C o custo da produção e R a receita do produto. Uma b) negativas d) de sinais contrários
indústria produziu x peças e verificou que o custo de produção era
dado pela função C(x) = x² – 500x + 100 e a receita representada por 24. (EPCAR 3° ANO) A soma dos quadrados das raízes da equação
R(x) = 2000x – x². Com base nessas informações, determine o número 104
5x² – 2px – p = 0 é . Vale então dizer que o valor positivo de p é
de peças a serem produzidas para que o lucro seja máximo. 25
um número
a) 625 c) 1000 e) 375
a) par c) divisor de 15
b) 781150 d) 250
b) primo maior que 10 d) menor que 1
18. (ESPCEX) O projétil disparado por um canhão, posicionado num
25. (ESPCEX) Considere m, n e p números reais não nulos e as funções
ponto de altitude igual a 200 metros, atinge um alvo localizado num
f e g de variável real, definidas por f(x) = mx² + nx + p, e g(x) = mx + p.
ponto de altitude igual a 1200 metros.
A alternativa que melhor representa os gráficos de f e g é
Considerando-se que:
8 4
I. A trajetória descrita pelo projétil é dada pela equação=
y x − x2
3 3
com x e y em quilômetros, e referenciada a um sistema cartesiano
com origem no canhão.
II. O alvo é atingido quando o projétil encontra-se no ramo
descendente da sua trajetória.
Nas condições acima descritas, pode-se afirmar que a distância
horizontal entre as posições do canhão e do alvo é:
a) 0,5 km. d) 2,0 km.
b) 1,0 km. e) 2,5 km.
c) 1,5 km.

19. (AFA) O retângulo, com base no eixo das abscissas, está inscrito
numa parábola, conforme figura abaixo. O valor de x que faz esse
retângulo ter perímetro máximo é

a) 1
b) 0,5
c) 0,25
d) 0,125

92

PM_BOOK16 - MAT.indb 92 25/11/2022 19:08:54


FUNÇÃO QUADRÁTICA

26. (ESPCEX) A temperatura T de aquecimento de um forno, em °C, 32. (AFA) No plano cartesiano abaixo estão representados o gráfico
varia com o tempo t, em minutos, segundo a função abaixo: da função real f definida por f(x) = –x² – x + 2 e o polígono ABCDE.

20 + 28t, se t ≤ 10

t + 5t + 150, se t > 10
2

O tempo necessário para que a temperatura do forno passe de


160 °C para 564 °C é:
a) 5 minutos. d) 18 minutos.
b) 12 minutos. e) 23 minutos.
c) 13 minutos.

Considere que:
27. (ESPCEX) O conjunto solução da inequação 2x + 3x − 2 ≤ 0 está
2

contido em: 2 − 3x - o ponto C é vértice da função f.


- os pontos B e D possuem ordenadas iguais.
 2 1  e) ]−∞, −2]
a)  −∞, 3  c)  2 , +∞  - as abscissas dos pontos A e E são raízes da função f.

]−2, +∞[ Pode-se afirmar que a área do polígono ABCDE, em unidades de área, é
b) d) ]−3, +∞[
1 1 1 1
a) 8 b) 4 c) 4 d) 8
28. (AFA) As raízes da equação 2x² – px – 1 = 0 são senθ e cosθ. Sendo 16 8 4 2
θ um número real, o valor de p é:
a) 0 c) 4 33. (ESPCEX) Um portal de igreja tem a forma de um arco de parábola,
conforme figura abaixo. A medida da sua base AB é 4 m e da sua
b) 2 d) 5
altura é 5 m. Um vitral foi colocado 3,2 m acima da base. Qual a
medida CD da base, em metros?
29. (EPCAR 3° ANO) Num terreno em forma de um triângulo
retângulo com catetos medindo 20 e 30 metros, deseja-se construir
a) 1,44
uma casa retangular de dimensões x e y, conforme figura abaixo. O
perímetro da casa, em metros, tendo a mesma ocupado a maior área b) 1,80
possível, é igual a c) 2,40
d) 3,00
a) 100
e) 3,10
b) 150
c) 50
d) 25

34. (ESPCEX) Considere as funções reais f e g, tais que f(x)= x +4


e f(g(x)) = x² – 5, onde g(x) é não negativa para todo x real. Assinale a
alternativa cujo conjunto contém todos os possíveis valores de x, que
satisfazem os dados do enunciado.
30. (ESPCEX) Na criação de um determinado animal para abate, o
criador dispõe de estudos que lhe informam que a)  – ]– 3,3[ c)  − 5, 5  e)  − ]− ∞, 3[
- o custo da criação evolui no tempo segundo a relação b)  −  − 5, 5  d) ]– 3,3[
2 2
PC = t + 2 2t + 200 2 ;
120
- o preço obtido pelo criador ao vender o produto evolui no tempo 35. (AFA) Considere as funções reais f, g e h tais que f(x) = mx² –
1
2 2 (m + 2)x + (m + 2), g(x) = e h(x) = x . Para que a função composta
segundo a relação PV = − t + 3 2t + 200 2 x
120 h  g  f(x) tenha domínio D = , deve-se ter
onde PC e PV são respectivamente os preços de custo e de venda da 2 2
arroba de carne, em reais, e t, o tempo de engorda, em dias. Nestas a) m> c) 0<m<
3 3
condições pode-se afirmar que o tempo de engorda que fornece
maior lucro (PV – PC) é de 2 d) –2 < m < 0
b) −2 < m <
a) 20 dias. c) 90 dias. e) 45 dias. 3
b) 30 dias. d) 60 dias.
36. (AFA) Uma fábrica produz casacos de determinado modelo. O
preço de venda de um desses casacos é de R$ 200,00, quando são
x +x+3
2
vendidos 200 casacos. O gerente da fábrica, a partir de uma pesquisa,
31. (AFA) A solução da inequação ≤ 3 é dada pelo
conjunto: x +1 verificou que, para cada desconto de R$ 2,00 no preço de cada casaco,
o número de casacos vendidos aumenta de 5. A maior arrecadação
a) {x ∈ R / 0 ≤ x ≤ 2} possível com a venda dos casacos acontecerá se a fábrica vender cada
b) {x ∈ R / x ≤ –1 ou 0 < x ≤ 2} casaco por um valor, em reais, pertencente ao intervalo
c) {x ∈ R / x > –1 ou 0 ≤ x ≤ 2} a) [105,125[ c) [145,165[
d) {x ∈ R / x < –1 ou 0 ≤ x ≤ 2} b) [125,145[ d) [165,185[

93

PM_BOOK16 - MAT.indb 93 25/11/2022 19:08:55


FUNÇÃO QUADRÁTICA

37. (AFA) Dada a função real f(x) = x², considere a função real Sabendo-se que h é a função definida por h(x) = (ax + b)(cx + d),
g(x) = f(x + m) + k, sendo m, k reais. É INCORRETO afirmar que: pode-se dizer que
a) o gráfico da função g em relação ao gráfico da função f é a) o gráfico de h é uma parábola com a concavidade voltada para
deslocado k unidades para cima, se k > 0, e m unidades para a cima.
direita, se m < 0. b) h não tem raízes reais.
b) a equação do eixo de simetria da parábola que representa g é c) h intercepta o eixo de Oy num ponto de ordenada negativa.
dada por x = m.
d) a abscissa do vértice do gráfico que representa a função h é um
c) se m = 0 e k = 1, então o conjunto imagem de g é dado por número real negativo se |ad| > |bc|.
Im = {y ∈  | y ≥ 1}.
d) se m = -2 e k = -3, então as coordenadas do vértice da parábola 41. (EPCAR 3° ANO) Num laboratório, a temperatura obtida em
que representa g são (–m,k). determinada experiência, em graus centígrados, é dada pela função
t2
38. (AFA) As funções f:  →  do primeiro grau e g:  → [b,+∞[ do f(t) =− + t + 20, onde t é o tempo em segundos (t ≥ 0).
8
segundo grau estão representadas no gráfico abaixo. É correto afirmar que a temperatura
a) é sempre positiva.
b) máxima é 20 graus.
c) máxima ocorre para t = 4 segundos.
d) nunca será igual a zero.

42. (EPCAR 3° ANO) Alguns alunos do 3° ano da EPCAR desejam


fazer uma viagem durante um recesso e para isso precisam fretar um
ônibus. Duas empresas, α e β, candidatam-se para fazer a viagem.
Sabendo-se que as duas empresas possuem ônibus de 50 lugares e
que: se for contratada a empresa α, o custo da viagem terá uma parte
fixa de R$ 300,00, mais um custo por passageiro de R$ 15,00; se for
contratada a empresa β, o custo terá um valor fixo de R$ 250,00, mais
um custo(C), por passageiro, dado por C(n) = 35 – 0,5n, onde n é o
número de passageiros que fará a viagem. Com base nisso, é correto
afirmar que
a) se todos os lugares forem ocupados, será menos vantajoso
contratar a empresa β.
b) existe um determinado número n de passageiros para o qual o
custo na empresa α é o mesmo da empresa β.
c) o custo máximo da viagem na empresa β é de R$ 862,50.
d) para um custo de R$ 750,00, a empresa β levará um número de
Com base nas informações acima, é correto afirmar que passageiros 50% maior que a empresa α.
a) o menor valor de b que torna a função g sobrejetora é um número
inteiro. 43. (AFA) Considere que g:  → B, definida por g(x) = –bx² + cx – a é
função par e possui como gráfico o esboço abaixo.
b) ( g  g  f )  52  > 0
−1

 f ( x )  > 0 ⇔ x ∈  | x < 1 ∨ x > 4


2

c) { }
g(x )
d) f ( x ) − g ( x ) ≤ 0 ⇔ {x ∈  | x ≤ 0 ∨ x ≥ 6}

39. (EPCAR) Seja y = (3x + 2)(ax + b) onde a > 0 e b < 0. O conjunto


de todos os valores reais de x, para os quais y é positivo é
b 2 b
a) x < 0 ou x > − c) − <x<−
a 3 a
b 2 2 b
b) x<− ou x > − d) x<− ou x > −
a 3 3 a Marque a alternativa INCORRETA.
a) A função dada por t(x) = g(x) + a é positiva ∀x ∈ .
40. (EPCAR 3° ANO) Considere as funções definidas por f(x) = ax + b b) Se B = [–a,+∞[, então a função g é sobrejetora.
e g(x) = cx + d e os respectivos gráficos.
c) b<c<a
d) A função h:  →  dada por h(x) = –g(x) – a possui um zero real
duplo.

44. (AFA) Alguns cadetes da AFA decidiram programar uma viagem


de férias à cidade de Natal para janeiro de 2009. Fizeram pesquisa de
preços das diárias de alguns hotéis e verificaram que as duas melhores
propostas seriam as dos hotéis Araújo’s e Fabiano’s, que foram as
seguintes:

94

PM_BOOK16 - MAT.indb 94 25/11/2022 19:08:56


FUNÇÃO QUADRÁTICA

EXERCÍCIOS DE

COMBATE
Hotel Araújo’s: possui 40 quartos disponíveis, todos individuais, sem
direito a cama extra. A diária de cada quarto é dada por uma taxa
fixa de R$ 200,00 mais R$ 10,00 por quarto não ocupado.

Hotel Fabiano’s: possui 50 quartos disponíveis, todos individuais,


sem direito a cama extra. O valor da diária de cada quarto é 0, 6 de 01. (AFA 2013) O gráfico de uma função polinomial do segundo grau
6 décimos de 125 milésimos de R$ 6000,00 y = f(x), que tem como coordenadas do vértice (5,2) e passa pelo
ponto (4,3), também passará pelo ponto de coordenadas
A viagem citada foi programada para x cadetes (x ≤ 40) e, no período c) (6,4)
a) (1,18)
em que eles estiverem hospedados, os hotéis só receberão como
b) (0,26) d) (–1,36)
hóspedes esses x cadetes.
Com base nisso, marque a alternativa INCORRETA.
02. (EFOMM 2012) O lucro obtido pela venda de cada peça de roupa é
a) Se forem viajar menos de 30 cadetes, então é mais vantajoso para x – 10, sendo x o preço da venda e 10 o preço do custo. A quantidade
os cadetes optarem pelo Hotel Fabiano’s. vendida por mês é igual a 70 – x. O lucro mensal máximo obtido com
b) O faturamento diário do Hotel Araújo’s será de R$ 8000,00 se, a venda do produto é:
e somente se, o número de cadetes que forem à viagem for 20 a) 1200 reais. c) 900 reais. e) 600 reais.
c) Se 15 cadetes forem viajar, então o valor da diária do Hotel b) 1000 reais. d) 800 reais.
Fabiano’s é 0,6 do valor da diária do Hotel Araújo’s.
d) O maior faturamento diário que o Hotel Araújo’s poderá ter, caso 03. (EFOMM 2012) Um professor escreveu no quadro-negro uma
os cadetes optem por hospedarem nele, acontecerá quando 10 equação do segundo grau e pediu que os alunos a resolvessem. Um
quartos não forem ocupados. aluno copiou errado o termo constante da equação e achou as raízes
–3 e –2. Outro aluno copiou errado o coeficiente do termo do primeiro
45. (AFA) Para angariar fundos para a formatura, os alunos do 3º grau e achou as raízes 1 e 4. A diferença positiva entre as raízes da
ano do CPCAR vendem bombons no horário do intervalo das aulas. equação correta é:
Inicialmente, começaram vendendo cada bombom por R$ 4,00. Assim, a) 1 c) 3 e) 5
perceberam que vendiam, em média, 50 bombons por dia. A partir
dos conhecimentos que os alunos tinham sobre função, estimaram b) 2 d) 4
que para cada 5 centavos de desconto no preço de cada bombom
(não podendo conceder mais que 70 descontos), seria possível vender 04. (EN 1994) O conjunto solução da inequação é:
5 bombons a mais por dia. x4  1
Considere: 0
 x  3x 3  2x 2
4

- p o preço de cada bombom;


a) (-∞,–1] ∪ (2,+∞)
- n o número de bombons vendidos, em média, por dia;
b) (-∞,–1] ∪ (1,2)
- x ∈  o número de reduções de 5 centavos concedidas no preço
unitário de cada bombom; c) (-∞,–1) ∪ (0,2)
- y a arrecadação diária com a venda dos bombons. d) (-∞,–1] ∪ [1,2]
Com base nessas informações, analise as proposições abaixo.
e) (-∞,–1) ∪ (-1,0)
(02) O gráfico que expressa n em função de p está contido no
05. (CN 2005) As raízes do trinômio do 2º grau y = ax² + bx + c são
segmento AB do gráfico abaixo.
1000 e 3000. Se quando x vale 2010, o valor numérico de y é 16, qual
é o valor numérico de y quando x vale 1990?
a) 64 c) 16 e) 4
b) 32 d) 8

06. (ESPCEX) Dada a função f:  → , tal que f(x) = x² – 7x + 10, a


única afirmação verdadeira a respeito de f(x) é
a) f(–2) = –28.
b) a menor ordenada que f atinge é 2,25.
c) a função se anula para x = –2 ou para x = –5.
d) para x > 5, enquanto x cresce, f(x) também cresce.
e) dobrando x, f(x) também dobra.
(04) A maior arrecadação diária possível com a venda dos bombons,
considerando os descontos de 5 centavos, ocorre quando concederem
35 descontos de 5 centavos. 07. (ESPCEX) Em uma determinada função quadrática, –2 e 3 são suas
raízes. Dado que o ponto (–3,12) pertence ao gráfico dessa função,
(08) Se forem concedidos 20 descontos de 5 centavos, serão vendidos pode-se concluir que
mais de 100 bombons por dia.
a) o seu valor máximo é –12,50.
A soma das proposições verdadeiras é igual a b) o seu valor mínimo é 0,50.
a) 6 c) o seu valor máximo é 6,25.
b) 10 d) o seu valor mínimo é –12,50.
c) 12 e) o seu valor máximo é 0,50.
d) 14

95

PM_BOOK16 - MAT.indb 95 25/11/2022 19:08:57


FUNÇÃO QUADRÁTICA

08. (ESPCEX) Na figura abaixo, estão representados um sistema de


eixos coordenados com origem O, o gráfico de uma função real do
tipo f(x) = ax2 + bx + c e o quadrado OMNP, com 16 unidades de área.
Sabe-se que o gráfico de f(x) passa pelos pontos P e N, vértices do
quadrado, e pelo ponto de encontro das diagonais desse quadrado.
DESAFIO PRO
Assim, o valor de a + b + c é

a)
y
1 (ITA) O conjunto de todos os valores de m para os quais

a função f(x) =
x 2 + (2m + 3)x + (m2 + 3)
está definida e é
x 2 + (2m + 1)x + (m2 + 2)
não-negativa para todo x real é:
b)
1 7
c) P
a)  4 , 4 
N
1 
2 b)  , ∞ 
d) 4 
2
 7
e) 5 2 c)  0, 4 
2
 1
d)  −∞ , 
 4
x
O M  1 7
e)  4 , 4 

09. (EFOMM 2010) O gráfico das três funções polinomiais do 1º


grau a, b e c definidas, respectivamente, por a(x), b(x) e c(x) estão 2 (IME) Seja a, b e c números reais e distintos.
Ao simplificar a função real, de variável real,
( x − b )( x − c ) ( x − c )( x − a) ( x − a)( x − b )
representadas abaixo. ( )
f x = a2 + b2 + c2 ,
( a − b )( a − c ) (b − c )(b − a) ( c − a)( c − b )
obtém-se f(x) igual a:
a) x² – (a + b + c)x + abc
b) x² + x – abc
c) x²
d) –x²
e) x² – x + abc

a b
3 (ITA) Considere a equação −
1 − x2 x − 1 / 2
números inteiros positivos. Das afirmações:
=
5, com a e b

I. Se a = 1 e b = 2, então x = 0 é uma solução da equação.


1
II. Se x é solução da equação, então x ≠ , x ≠ 1 e x ≠ 1.
2
2
III. x = não pode ser solução da equação.
3
É (são) verdadeira(s)
a) apenas II.
b) apenas I e II.
Nestas condições, o conjunto solução da inequação é:
c) apenas I e III.
 a  x    b  x  
5 6
d) apenas II e III.
0
c  x 
3
e) I, II e III.
a) (–4,–1) ∪ [3,+∞)
b)
c)
(–4,–1] ∪ [3,+∞)
(–∞,–4) ∪ [–1,+∞)
4 (ITA) Considere um número real a ≠ 1 positivo, fixado, e a
equação em x a2x + 2βax – β = 0,β ∈ . Das afirmações:
I. Se β < 0, então existem duas soluções reais distintas;
d) [4,+∞)
II. Se β = –1, então existe apenas uma solução real;
e)  – {4}
III. Se β = 0, então não existem soluções reais;
10. (ESPCEX) Um fabricante de poltronas pode produzir cada peça IV. Se β > 0, então existem duas soluções reais distintas,
ao custo de R$ 300,00. Se cada uma for vendida por x reais, este é (são) sempre verdadeira(s) apenas
fabricante venderá por mês (600 – x) unidades, em que 0 ≤ x ≤ 600.
a) I.
Assinale a alternativa que representa o número de unidades vendidas
mensalmente que corresponde ao lucro máximo. b) I e III.
a) 150 d) 450 c) II e III.
b) 250 e) 550 d) II e IV.
c) 350 e) I, III e IV.

96

PM_BOOK16 - MAT.indb 96 25/11/2022 19:08:58


FUNÇÃO QUADRÁTICA

GABARITO
EXERCÍCIOS DE FIXAÇÃO
01. a) x1 = 1; x2 = 2 1 1
c) + =
−10
b) x1 = –4; x2 = –3 x1 x 2
c) x1 = x2 = –2 04. D
d) x1 = √2; x2 = –√2 05. C
2 3 06. D
02. m  1 
3 07. A
5 08. E
03. a) x1 + x 2 =
2 09. C
1 10. D
b) x1 − x 2 =−
2
EXERCÍCIOS DE TREINAMENTO
01. B 13. E 25. E 37. B
02. D 14. D 26. C 38. B
03. B 15. A 27. D 39. D
04. D 16. C 28. A 40. D
05. A 17. A 29. C 41. C
06. C 18. C 30. B 42. C
07. E 19. B 31. C 43. A
08. B 20. B 32. B 44. B
09. E 21. B 33. C 45. D
10. E 22. B 34. A
11. B 23. C 35. A
12. A 24. A 36. B
EXERCÍCIOS DE COMBATE
01. A 04. A 07. D 10. D
02. C 05. C 08. C
03. C 06. D 09. C
DESAFIO PRO
01. D 03. E
02. C 04. C

ANOTAÇÕES

97

PM_BOOK16 - MAT.indb 97 25/11/2022 19:08:59


FUNÇÃO QUADRÁTICA

ANOTAÇÕES

98

PM_BOOK16 - MAT.indb 98 25/11/2022 19:09:00


FUNÇÃO MODULAR

DEFINIÇÃO
Sendo x ∈ , define-se módulo ou valor absoluto de x que se
indica através da relação.
1) |x| < k ⇔ -k < x < k
= | x | x, se x ≥ 0 Devemos pensar em todos os números reais que possuem o
 módulo menor que um número k, que é o mesmo que pensar em
 |x| = −x, se x < 0
todos os números que possuem a distância até a origem da reta real
1) o módulo de um número real não negativo é igual ao próprio menor que a distância do número k. Assim vemos que se trata da
número. parte interna entre k e -k.
2) o módulo de um número real negativo é igual ao oposto desse
número.

2) |x| > k ⇔ x < -k ou x > k


Exemplos:
De maneira análoga iremos pensar agora em todos os números
|2| = 2, |–7| = 7, |0| = 0
reais que possuem a distância até a origem da reta real maior do que
o número k.
PROPRIEDADES
I. x2 = x
II. xy = x y Exemplos:
2
III. x=
2
x= x 2 (Esta propriedade é muito importante!) 1) Resolver em R, |2x + 1| < 3

x 2x +1 < 3
x
IV. = , se y ≠ 0
y y − 3 < 2x +1< 3
− 2 < x < 1.
EQUAÇÕES MODULARES S = {x ∈R | − 2 < x <1}
1° Tipo 2) Resolver em R, |4x – 3| 〉 5.
x =k⇔x=k ou x =−k |4x – 3| > 5 ⇒ 4x – 3 < –5 ou 4x – 3 > 5
2° Tipo 1
x<− ou x > 2
2
x =k ⇔ x =k ou x =−k
 1 
S= x ∈ R / x < − ou x > 2
Exemplos:  2 
1) Resolver |2x – 1| = 3
 2x − 1 = 3 ⇒ x = 2 OUTROS TIPOS DE EQUAÇÕES E
2x − 1= 3 ⇒ 
2x − 1 =−3 ⇒ x =−1
S = {2,1}
INEQUAÇÕES MODULARES
Existem outros tipos de equações ou inequações modulares,
geralmente quando há alguma operação entre 2 ou mais módulos
2) Resolver |3x – 1| = |2x + 3| distintos, onde precisaremos decompor cada módulo individualmente
 3x − 1= 2x + 3 ⇒ x = 4 e arrumar em uma tabela para melhor visualização dos intervalos. Mas
 antes vamos aprender a decompor módulos corretamente.
 2
3x − 1 =−2x − 3 ⇒ x =− 5
 x, se x ≥ 0
Por terem visto a seguinte decomposição x =
=

S 4, − 
2 −x, se x < 0
 5 muitos copiam e colam para os demais módulos e acabam errando,
por exemplo:
 x + 1, se x ≥ 0
INEQUAÇÕES MODULARES x +1 = , veja que está errado pois o correto é
−x − 1, se x < 0
Podemos interpretar o módulo de um número como sua distância
 f(x), se f(x) ≥ 0
a origem na retal real. Dessa forma sempre que tivermos um número o seguinte f(x) =  , assim o que devemos fazer
real k haverá o seu simétrico –k que terá a mesma distância a origem −f(x), se f(x) < 0
da reta real e dessa forma consequentemente o mesmo módulo.

99

PM_BOOK16 - MAT.indb 99 25/11/2022 19:09:01


FUNÇÃO MODULAR

realmente é estudar o sinal da função que está dentro do módulo 1


para depois decompô-lo. Se x ≥
2
Veja: 2
3x + 2 = 4 ⇒ 3x = 2 ⇒ x =
|x + 1| ⇒ f(x) = x + 1, fazendo o zero da função x + 1 = 0 ⇒ x = –1 3
2
A solução x = é solução para este intervalo.
3
 2
Dessa forma teremos duas soluções S = 0,  .
 3
Exemplo 2:
2x − 4 − x + 1 ≥ 3

2x − 4, se x ≥ 2  x + 1, se x ≥ −1
=2x − 4  = e x +1 
 x + 1, se x ≥ −1 4 − 2x, se x < 2 −x − 1, se x < −1
Sendo assim o correto x + 1 =
−x − 1, se x < −1
–1 2
Depois de visto isso vamos ver nossas equações e inequações:
|2x – 4| 2x – 4 2x – 4 4 – 2x

Exemplo 1: |x + 1| x+1 –x – 1 –x – 1
|x + 3| + |1 – 2x| = 4
|2x – 4| – |x – 1| x–5 3x – 3 –x + 5
|x + 3| ⇒ f(x) = x + 3, fazendo o zero da função x + 3 = 0 ⇒ x = –3
Se x < –1
x–5≥3⇒x≥8
Teremos de fazer ]−∞, −1[  [8, ∞[ .
]−∞, −1[  [8, ∞[ = ∅
Se –1 ≤ x < 2
3x – 3 ≥ 3 ⇒ 3x ≥ 6 ⇒ x ≥ 2
Teremos de fazer [ −1, 2[  [2, ∞[ .
 x + 3, se x ≥ −3
x+3 =
 [ −1, 2[  [2, ∞[ =∅
−x − 3, se x < −3
Se x ≥ 2
1 –x + 5 ≥ 3 ⇒ –x ≥ –2 ⇒ x ≤ 2
|1 – 2x| ⇒ g(x) = 1 – 2x, fazendo o zero da função 1 − 2x = 0 ⇒ x =
2 Teremos de fazer [2, ∞[  ]−∞,2] = {2} .
Dessa forma dentre todos os intervalos temos apenas um conjunto
unitário como solução, S = {2}.

GRÁFICO DA FUNÇÃO MODULAR


f :  → 
A função modular  possui o gráfico bem simples de
 x→ x
ser traçado, veremos como podemos traçar gráficos das diversas
funções modulares, não só de f(x) = |x|.
1− 2x, se x < 1
 2 Exemplo 1:
1− 2x = 
2x − 1, se x ≥ 1 f(x) = |x|
2
Podemos proceder da seguinte forma:
Assim iremos montar uma tabela: 1°) traçamos o gráfico da função que está dentro do módulo.
–3 1/2 2°) “Rebatemos” a parte negativa do gráfico (se houver) para cima.
|x + 3| –x – 3 x+3 x+3 f(x) = |x| ⇒ f(x) = |g(x)| onde g(x) = x, assim faremos o gráfico de
g(x) = x.
|1 – 2x| 1 – 2x 1 – 2x 2x – 1

|x + 3| + |1 – 2x| –3x – 2 –x + 4 3x + 2

Se x < –3
–3x – 2 = 4 ⇒ –3x = 6 ⇒ x = –2
Mas repare que estamos no intervalo x < –3, assim –2 ∉ x < –3,
então ∃ x ∈ .
1
Se −3 ≤ x <
2
–x + 4 = 4 ⇒ x = 0
1
A solução x = 0 está dentro do intervalo −3 ≤ x < .
2

100

PM_BOOK16 - MAT.indb 100 25/11/2022 19:09:02


FUNÇÃO MODULAR

Verificamos que a simetria da parte negativa do gráfico deve ser Vamos ver outro exemplo.
mantida depois de rebatermos para cima. Exemplo 2:
f(x) = |2x + 1| + 2
Daí podemos dizer que f(x) = |g(x)| + 2 onde g(x) = 2x + 1.
Assim traçamos primeiro g(x) = 2x + 1.

Fazendo f(x) = |g(x)|

Depois modulamos f(x) = |g(x)|

Por fim

101

PM_BOOK16 - MAT.indb 101 25/11/2022 19:09:03


FUNÇÃO MODULAR

E agora iremos deslocar 2 unidades para cima: f(x) = |g(x)| + 2

Também podemos traçar o gráfico de funções modulares


decompondo os módulos e criando assim funções de sentenças
abertas.
Exemplo 3:
f(x) = x + 2 − 1 − x

 x + 2, se x ≥ −2  1− x, se x < 1
x+2 e 1− x 
−x − 2, se x < −2 −1 + x, se x ≥ 1

-2 1
|x + 2| –x – 2 x+2 x+2

|1 – x| 1–x 1–x –1 + x

|x + 2| – |1 – x| –3 2x + 1 3

 −3, se x < −2

Assim teremos f(x)= 2x + 1, se − 2 ≤ x < 1
 3, se x ≥ 1

102

PM_BOOK16 - MAT.indb 102 25/11/2022 19:09:04


FUNÇÃO MODULAR

EXERCÍCIOS DE

FIXAÇÃO
01. Resolva as equações modulares abaixo:
a) 2x – 3= 6
b) 4 – 3x = 11
c) 3x – 4=x – 3

02. Resolva a equação modularx + 3–2x – 1= 3x + 1:

03. (PUC) Os pesos aceitáveis do pãozinho de 50 g verificam a


desigualdade | x – 50 | ≤ 2, em que x é medido em gramas. Então,
assinale o peso mínimo aceitável de uma fornada de 100 pãezinhos,
em quilogramas.
a) 4,50 c) 5,20
b) 4,80 d) 5,50

04. A soma das raízes distintas da equação x² – 5x + 6 = x – 3 é:


a) 10 c) 0 e) 4
Exemplo 4:
b) 7 d) 3
f(x)= | 2x + 1| −1
05. (EEAR) Seja a função ƒ: R → R, definida por ƒ(x) = |2x² – 3|. O
 2x + 1, se x ≥ − 1 valor de 1 + ƒ(–1) é:
 2 , daí teremos
2x + 1 
−2x − 1, se x < − 12 a) – 1 b) 0 c) 1 d) 2

1 06. (EEAR) Seja a inequação |x – 2| ≤ 3. A soma dos números inteiros


Se x < − ⇒ f(x) = −2x − 1 − 1 = 2x − 2 e
2 que satisfazem essa inequação é:
1 a) 8 b) 7 c) 5 d) 4
se x ≥ − ⇒ f(x)
= 2x + 1 − 1
= 2x
2
x
07. (EEAR) No conjunto solução da inequação 1   5 , quantidade
de números inteiros pares é: 3
a) 14 b) 12 c) 10 d) 8

08. A soma das raízes que a equação modular ||x – 2| –7| = 6 é:


a) 15 c) 4 e) 8
b) 30 d) 2

09. (PUC) Qual dos gráficos abaixo representa a função real ƒ(x) = |3x – 1|?

a)
c)

Se quiséssemos poderíamos ter divido mais, veja:


 2x + 2, se x ≥ −1  2x, se x ≥ 0
2x + 2  e 2x 
 −2x − 2, se x < −1 −2x, se x < 0

 −2x − 2, se x < −1

2x + 2, se − 1 ≤ x < − 12 b)
f(x) =  d)
 −2x, se − 12 ≤ x < 0

 2x, se x ≥ 0

103

PM_BOOK16 - MAT.indb 103 25/11/2022 19:09:06


FUNÇÃO MODULAR

EXERCÍCIOS DE

TREINAMENTO
e)
01. (CFT) Seja ƒ(x) = |x – 6| uma função real. A soma dos valores de x
para os quais ƒ(x) = 5 é:
a) 10 b) 12 c) 14 d) 16

02. (CFOE) Seja a inequação |x + 4| < 2 em . Podemos afirmar que o


menor valor inteiro que x pode assumir é:
10. (PUC) Considere a função real ƒ(x) = |–x + 1| O gráfico que a) – 6 b) – 5 c) –4 d) – 3
representa a função é:
03. (CFOE) A solução da equação |2x – 7| = 8x – 1 é:
 4
a) S = {–1} c) S = −1, 
 5
a) 4   4
b) S=  d) S= 1, − 
5   5

04. (EEAR) Seja f(x) = |x – 3| uma função. A soma dos valores de x para
os quais a função assume o valor 2 é:
a) 3 b) 4 c) 6 d) 7

05. (EEAR) Sendo S o conjunto-solução da equação em  |3x – 1| =


–3x + 1, pode-se afirmar que:
b) 1 2 3 1 1 2
a) ∈S b) ∈S c)  , ⊂ S d)  , ⊂ S
2 3 5 3 5 7

06. (EEAR) A equação |x²| + |x| – 6 = 0


a) só tem uma solução.
b) tem duas soluções, tais que seu produto é = –6.
c) tem duas soluções, tais que seu produto é = –4.
d) tem duas soluções, tais que seu produto é igual a 0.
c)
07. (ITA 80) Considere a equação |x| = x – 6. Com respeito à solução
real desta equação podemos afirmar que:
a) a solução pertence ao intervalo [1,2]
b) a solução pertence ao intervalo [–2,–1]
c) a solução pertence ao intervalo (–1,–1)
d) a solução pertence ao complementar da união dos intervalos anteriores
e) a equação não tem solução

d) 08. (ESPCEX) O conjunto solução da equação |x – 3| = |x – 3|², em :


a) Possui somente 4 elementos
b) Possui somente 3 elementos
c) Possui somente 2 elementos
d) Possui somente 1 elemento
e) É vazio

09. (ESPCEX) Dos gráficos abaixo, o que melhor representa a função


|4x² – 16x + 7| é:
e)

a) b)

104

PM_BOOK16 - MAT.indb 104 25/11/2022 19:09:07


FUNÇÃO MODULAR

14. (ESPCEX) Sabendo que o gráfico a seguir representa a função real


f(x) = |x – 2| + |x + 3|, então o valor de a + b + c é igual a:
c) e)
a) –7
b) –6
c) 4
d) 6
e) 10

d)

15. (AFA) Dados A = {x ∈ R, 0 <x + 2< 5} e B = {x ∈ Z,x + 3≤ 5},


10. (ESPCEX) O domínio e a imagem da função f(x) = |2x² – 2x| + 4, tem-se que:
são respectivamente:
a) A ∩ B = {–1, 0, 1, 2} c) A∩B=A
a)  e [4,5; +∞[ d)  e ]–∞; 4,5]
b) A ∩ B = [–6, 2] d) A ∩ B = B – {–8, –7, –6}
b)  e [4; +∞[ e) + e [4; +∞[
c) + e ]–∞; 4] 16. (ESPCEX) O conjunto solução da inequação ||x – 4| + 1| ≤ 2 é um
intervalo do tipo [a, b]. O valor de a + b é igual a:
1
=
11. (AFA) O gráfico que melhor representa a função f(x)
2
( x − x ) é: a) –8 b) –2 c) 0 d) 2 e) 8

17. (AFA) Durante 16 horas, desde a abertura de certa confeitaria,


observou-se que a quantidade q(t) de unidades vendidas do doce
a) c) “amor em pedaço”, entre os instantes (t – 1) e t, é dada pela lei
q(t) = ||t – 8| + t – 14|, em que t representa o tempo, em horas, e
t ∈ {1, 2, 3,..., 16}.
É correto afirmar que:
a) entre todos os instantes foi vendida, pelo menos, uma unidade de
“amor em pedaço”.
b) a menor quantidade vendida em qualquer instante corresponde
b) d) a 6 unidades.
c) em nenhum momento vendem-se exatamente 2 unidades.
d) o máximo de unidades vendidas entre todos os instantes foi 10.

18. (ESPCEX) O conjunto solução da inequação |x² + x + 1| ≤ |x² + 2x – 3| é:


1,se 0 ≤ x ≤ 2  1 
12. (AFA) Considere a função f(x) =  . A função a) x ∈  | − ≤ x ≤ 2 ou x ≥ 4
g(x) = |f(x)| – 1 terá o seguinte gráfico: −2,se − 2 ≤ x < 0  2 
 1 
b) x ∈  | −2 ≤ x ≤ ou x ≥ 4
 2 
a) c)  1 
c) x ∈  | x < − ou 2 ≤ x ≤ 4
 2 
 1 1 
d) x ∈  | x < − ou ≤ x ≤ 4
 2 2 
 1 
e) x ∈  | − ≤ x ≤ 4
 2 

b) d) 19. (ESPCEX) O gráfico que melhor representa a função real definida


4 − | x − 4 |, se 2 < x ≤ 7
por  2 é:
x − 2x + 2, se x ≤ 2

a)
13. (ESPCEX) Dada a equação |2x – 3| + |x| – 5 = 0, a soma de todas
as suas soluções é igual a:
a) 3 c) 2 e) 2/3
b) 8/3 d) 4/3

105

PM_BOOK16 - MAT.indb 105 25/11/2022 19:09:08


FUNÇÃO MODULAR

23. (EPCAR 3° ANO) A função ƒ(x) = |x + 2| + |x| + |x – 5|, definida


de  em *,
b) a) possui conjunto imagem Im = {y ∈  | y ≥ 7}
b) é sobrejetora.
c) possui um máximo que se localiza em cima do eixo das ordenadas.
d) é decrescente para todo x do intervalo [5, +∞[.

24. (EPCAR 3° ANO) Considerando a função real ƒ, definida por


f(x) = |x – 2| + |2x + 1| – x – 6 é INCORRETO afirmar que:
c)
a) seu conjunto-imagem é [–3, +∞[
b) ƒ é bijetora ⇔ x ≥ 2
5
c) se ƒ(x) = 5, então x = −ou x = 6
2
 1
d) o intervalo em que ƒ decresce é  −∞, − 
 2
d)
25. (EPCAR 3° ANO) Em , o conjunto solução da inequação
|x + 1| – |x| ≤ x + 2 é:
a) [–3, 0]
b) {x ∈  | x = 0}  [3,5]
c) [–3, +∞[
d) [-3,0]  {x ∈  | x = 1}
e)
26. (EPCAR 3° ANO) Os valores de x ∈ , para os quais a função real
dada por f(x) = 5 − 2x − 1 − 6 está definida, formam o conjunto:

a) [0, 1] c) [-5, 0]  [1, 6]


b) [–5, 6] d) [-5, 0]  [1, +∞[
20. (EPCAR 3° ANO) O gráfico que MELHOR representa a função
ƒ:  → , definida por f(x) = |x –1| + |x| – 1, corresponde à 27. (EPCAR 3° ANO) Sobre as funções reais ƒ e g, definidas por
alternativa: 2x + 2x
f(x) = e g(x) = |–x|–1, é FALSO dizer que:
4
a) a equação ƒ(x) = g (x) tem conjunto-solução unitário.
b) ƒ(x) ≥ g(x) ∀x ∈ 
a) c)
c) x ≥ 0 então ƒ(x) ≥ 0
d) se x < 0 então ƒ(x) = 0 e g(x) > -1

28. (AFA) As funções reais ƒ e g são tais que ƒ(x) = |x| –2 e g(x) =
ƒ(2x) + ƒ(|x|).A melhor representação gráfica de g é:

b) d) a) c)

21. (EN) A soma das raízes reais distintas da equação ||x – 2| – 2| = 2 d)


é igual a: b)

a) 0 d) 6
b) 2 e) 8
c) 4
29. (AFA) Sobre a função real definida por
22. (EFOMM) Determine a imagem da função f, definida por 2x 2 + x − 3, se x ≤ −1 ou x ≥ 1
f(x) =  , pode-se dizer que:
f(x) = ||x + 2| – |x – 2||, para todo x ∈ , conjunto dos números reais.
(1 − x ) , se − 1 < x < 1
2

a) Im(f) = 
b) Im(f) = {y ∈  | y ≥ 0} a) ƒ(x) ≥ 7 ⇔ x ≥ 2 ou x ≤ -2
c) Im(f) = {y ∈  | 0 ≤ y ≤ 4} b) tem valor máximo igual a 1
d) Im(f) = {y ∈  | y ≤ 4} c) ƒ(x) ≥ 0 ∀x ∈ 
e) Im(f) = {y ∈  | y > 0} d) se –1 < x < 1, então 0 < y ≤ 1

106

PM_BOOK16 - MAT.indb 106 25/11/2022 19:09:09


FUNÇÃO MODULAR

30. (EN 1990) A equação |2x + 3| = ax + 1: 02. (EFOMM 2013) Os valores de x ∈  para os quais a função real
a) não possui solução para a < -2; dada por f  x   4  2x  1  6 está definida formam o conjunto:
b) possui duas soluções para a > 2;
 1 3
c) possui solução única para a < 2 ; a)   3 , 2 
3
d) possui solução única para –2 < a < 2 ;
3  9 5 3 7 
b)   2 ,  2    2 , 2 
e) possui duas soluções para –2 < a < 2 .
3
 5 1   7 11
31. Dadas as funções ƒ:  →  e g:  →  definidas por ƒ(x) = |1 – x²| c)   ,    , 
 2 2 2 2 
e g (x) = |x|, o número de pontos na interseção do gráfico de f com o
gráfico de g é igual a:  5   7
a) 5 b) 4 c) 3 d) 2 e) 1
d)   2 , 0   0, 2 

32. (EFOMM 2011) A área entre o gráfico de y = ||3x + 2| – 3| e a reta  9 1   3 11


y = 3, em unidades de área, vale:
e)   2 ,  2    2 , 2 
a) 6 b) 3 c) 1,5 d) 2 e) 0,5
03. O produto das raízes reais da equação |x² - 3x + 2| = |2x – 3| é igual a:
33. Para x ∈ , o conjunto solução de |53x - 52x+1 + 4 · 5x| = |5x – 1| é:
a) –5 c) 1 e) 5
a) {0,2 ± 5,2 ± 3 } b) –1 d) 2

b) {0,1,log (2 +
5 5 )} 04. (EN 2013) A soma das raízes reais distintas da equação ||x – 2| – 2| = 2
é igual a:
 1 1  2   a) 0 c) 4 e) 8
c) 0, log5 2, log5 3,log5   
 2 2  2   b) 2 d) 6
d) {0,log (2 + 5 ) ,log (2 + 3 ) ,log (2 − 3 )}
5 5 5
05. (EN 2010) Considere a equação x² + bx + c = 0, onde c representa
e) A única solução é x = 0 a quantidade de valores inteiros que satisfazem a inequação
|3x – 4| ≤ 2. Escolhendo-se o número b, ao acaso, no conjunto
{–4,–3,–2,–1,0,1,2,3,4,5}, qual é a probabilidade de a equação acima
34. Sobre a equação na variável real x, |||x – 1|– 3|– 2| = 0, podemos
ter raízes reais?
afirmar que:
a) 0,50 c) 0,75 e) 1
a) ela não admite solução real
b) 0,70 d) 0,80
b) a soma de todas as suas soluções é 6
c) ela admite apenas soluções positivas 06. (ESPCEX) Sejam x e y números reais não nulos. Das seguintes
d) a soma de todas as soluções é 4 afirmações:
e) ela admite apenas duas soluções reais I. Se |x| = |y| então x = y
II. |x + y| ≥ |x| + |y|
35. (EN 2012) Sejam A e B conjuntos de números reais tais que
III. Se 0 < x < 1 então x2 < x
seus elementos constituem, respectivamente, o domínio da função
f(x) = In (2 + x + 3|x| – |x + 1|) e a imagem da função IV. Se x < 0 então x = √x2
2( x + x − 2 ) Pode-se concluir que:
g ( x ) =−2 + . Pode-se afirmar que:
2 a) todas são verdadeiras.
a) A = B c) A⊃B e) A – B = - b) somente a IV é falsa.
b) A ∩ B = ∅ d) A ∩ B = + c) somente I e III são verdadeiras.
d) somente II e IV são falsas.
e) somente a III é verdadeira.
EXERCÍCIOS DE

COMBATE 07. (ESPCEX) Observando o gráfico abaixo, que representa a função


real f(x) = |x – k| – p , pode-se concluir que os valores de k e p são,
respectivamente,

01. (ITA 1988) Sabendo-se que as soluções da equação |x|² – |x| – 6 = 0 a) 2 e 3


são raízes da equação x² – ax + b = 0, podemos afirmar que: b) –3 e –1
a) a = 1 e b = 6 c) –1 e 1
b) a = 0 e b = –6 d) 1 e –2
c) a = 1 e b = –6 e) –2 e 1
d) a = 0 e b = –9
e) não existem a e b tais que x² – ax + b = 0 contenha todas as raízes
da equação dada.

107

PM_BOOK16 - MAT.indb 107 25/11/2022 19:09:13


FUNÇÃO MODULAR

08. (ESPCEX) Considerando a função Real f(x) = (x – 1) · |x – 2|, o


intervalo real para o qual f(x) ≥ 2 é: GABARITO
a) {x ∈  | x ≥ 3} EXERCÍCIOS DE FIXAÇÃO
b) {x ∈  | x ≤ 0 ou x ≥ 3} 01. a) x = 9/2 ou x = –3/2 06. D
c) {x ∈  | 1 ≤ x ≤ 2} b) x = –7/3 ou x = 5 07. A
c) x = 1/2 ou x = 7/4
d) {x ∈  | x ≥ 2} 08. E
02. x = 3/4
e) {x ∈  | x ≤ 1} 09. D
03. B
10. A
09. (ESPCEX) Se Y = {y ∈  tal que |6y – 1| ≥ 5y – 10}, então: 04. E
 1 05. D
a) Y =  −∞, 
 6 EXERCÍCIOS DE TREINAMENTO

b) Y = {–1} 01. B 10. B 19. C 28. D

c) Y= 02. B 11. B 20. C 29. A

d) Y = ∅ 03. B 12. A 21. D 30. D


04. C 13. C 22. C 31. B
1 
e)
 6 , +∞  05. D 14. C 23. A 32. A
06. C 15. A 24. B 33. D
07. E 16. E 25. C 34. D
10. (ESPCEX) O número de soluções da equação 1 | x | ⋅ | x − 3 | = 2 ⋅ x − 3 , 08. B 17. D 26. C 35. C
no conjunto , é: 2 2
09. B 18. B 27. B
a) 1 b) 2 c) 3 d) 4 e) 5
EXERCÍCIOS DE COMBATE
01. D 04. D 07. E 10. D
02. E 05. A 08. A

DESAFIO PRO
03. A 06. E 09. C
DESAFIO PRO
01. A
02. C
1 (ITA) O produto das raízes
|x2 – 3x + 2| = |2x – 3| é igual a:
reais da equação
03. a) S = − , .
9 3
 2 2
a) –5 b) f(0) = 3
b) –1
c) S = −4, .
7
c) 1  3
d) 2
e) 5 04.

2 (ITA) O número de soluções inteiras da inequação


0 ≤ x² – |3x² + 8x| ≤ 2 é:
a) 1
b) 2
c) 3
d) 4
e) 5

 1+ 3 1+ 3 
3 (ITA) Considere as funções f1, f2, f:  → , sendo
f1=
(x)
1
x + 3, f2 (x)
=
3
x + 1 e f(x) igual ao maior valor
05. S = x ∈  : −2 < x < −
 2
ou
2
< x < 2 .

2 2
entre f1(x) e f2(x), para cada x ∈ . Determine: ANOTAÇÕES
a) Todos os x ∈  tais que f1(x) = f2(x).
b) O menor valor assumido pela função f.
c) Todas as soluções da equação f(x) = 5.

4 (ITA) Esboce o gráfico da função f:  →  dada por


1
= 2− | x | − .
f(x)
2

5 (IME) Resolva a inequação abaixo, onde x é uma variável real.


2 |x³| – 6x² + 3| x | +2 < 0

108

PM_BOOK16 - MAT.indb 108 25/11/2022 19:09:14


FUNÇÃO EXPONENCIAL

DEFINIÇÃO OPERAÇÕES COM POTÊNCIAS


A potência de expoente n, n natural e n > 1, do número a, (I) Para multiplicar potências de mesma base, basta conservar a
representa-se por an, é o produto de n fatores iguais a “a”. base e somar os expoentes.
Representa-se por am x an = am+n onde a é denominado base e n é Assim,
denominado expoente. am × an = am+n
Quando n = 2, lê-se a2 como “a ao quadrado” e quando n = 3,
lê-se a3 como a ao cubo”. (II) Para dividir potências de mesma base, conserva-se a base e
subtraem-se os expoentes.
Exemplos:
Assim,
I. 210  2

2
2 
 2  1024
am
10 fatores  am n ; m  0 e a  0
an
 3   3   3     3  6561
8
II.
  
8 fatores
(III) Para calcular uma potência de uma potência, conserva-se a
 5   5   5     5  15625
6
III. base e multiplicam-se os expoentes.
  
6 fatores Assim,
IV. 4
7
4 4 4 –16384 (am)n = am×n = (an)m
7 fatores p p
Não devemos confundir an com (an) pois a primeira é uma
V. 65 = 6 × 6 × 6 × 6 × 6 = 7776
potência cujo expoente é uma potência enquanto que a segunda é
VI. 04 = 0 × 0 × 0 × 0 = 0 uma potência de uma potência.
     7 3    7 3    7 3   343 27
3
VII.  7
3
DEFINIÇÃO
DEFINIÇÕES ESPECIAIS Potências semelhantes são aquelas que possuem o mesmo
expoente.
Estende-se a definição de potência pondo-se
(IV) Para multiplicar potências semelhantes conserva-se o
(I) a1 = a, para todo a real e a0 = 1, para todo a real diferente de zero.
expoente e multiplicam-se as bases.
(II) Dado um número real, não nulo, e um número natural n
Assim,
define-se a potência a-n pela relação:
n an × bn = (a × b)n
 1 1
a –n
   n
a a (V) Para dividir potências semelhantes conserva-se o expoente
Exemplos: e dividem-se as bases.

 18 1 1 Assim,
2
I. 20060 = 1 IV.    64
 18
n
2
1 an a
II. (–7) = –7
1 64 ; b 0
bn b
1 1
56  65 
3 3
III. ( 2)1   V.  216125
( 2)1 2

Observação
RADICIAÇÃO
Das definições anteriores decorre que a base a das potências pode DEFINIÇÃO
ser um número real positivo, negativo ou nulo e, com base nisto,
podemos fazer as seguintes observações: O número real b, é a raiz de índice n (raiz enésima), sendo n um
número natural, do número real a se, e somente se n a  b  b  a
n
(1) toda potência de expoente par é positiva;
onde, se n é par, devemos ter a ≥ 0 e b ≥ 0.
(2) toda potência de expoente ímpar tem sempre o sinal da base; 2
Quando n = 2 escreve-se a ao invés de a para representar a
(3) toda potência de 0 com expoente positivo é igual a zero. Não raiz quadrada positiva de a.
se definem 00 nem 0n com n negativo;
Quando n = 3, 3 a é chamada de raiz cúbica de a.
(4) as potências de (−1) são iguais +1 ou (−1) conforme o expoente
seja par ou ímpar respectivamente. O número real a é chamado radicando e n a é chamado radical
de índice n.
Assim,
I. 81 = 9 porque 9² = 81.
( 1)n 1 se n é par II. 3
125  5 porque (–5)³ = –125.
1 se n é ímpar III. −8 , 4 −8 e 6
−8 não são definidas.

109

PM_BOOK16 - MAT.indb 109 25/11/2022 19:09:24


FUNÇÃO EXPONENCIAL

CONSEQUÊNCIA DA DEFINIÇÃO Portanto definiremos,


1
Decorre imediatamente da definição da raiz enésima de a que an = n a
 
n
a n  a.
Observemos que se n é par é necessário que a ≥ 0.
 6
3
Assim, 3
 6. Finalmente, definiremos a potência para todo expoente racional
m
Observação irredutível onde m e n são inteiros e n > 0.
n
Se n a existe como um número real (e este será o caso para todo
=
(I) Para todo natural n não nulo, n
0 0=
e n 1 1. número real a se n for ímpar e para a ≥ 0 se n for par), definimos
m m
(II) Para todo natural n e quaisquer reais não nulos a e b tem- a n = ( n a )m ou equivalentemente a n = n am .
se que a  b  n a  n b e a  b  n a  n b. Em particular,
a  1  a  n a  1 e a  1  1  n a  a.
DEFINIÇÃO
Sendo a > 0, a ≠ 1, um número real, definimos a função
f:  → +* como sendo f(x) = ax. Tal função é dita função exponencial
LEIS DOS RADICAIS de base a.
Para todos os naturais não nulos n e p e todos os reais não nulos
a e b valem as seguintes fórmulas.
GRÁFICO
(I) Para multiplicar raízes de mesmo índice conserva-se o índice e
multiplicam-se os radicandos. Como vimos, a base da função exponencial precisa estar presente
no intervalo a > 0 e a ≠ 1.
n
a  n b  n a b

(II) Para dividir raízes de mesmo índice conserva-se o índice e


dividem-se os radicandos.
n
a a Vamos dividir e analisar separadamente duas situações para a
 n , b0 base.
n
b b

(III) Para elevar uma raiz a um expoente eleva-se o radicando ao


expoente e conserva-se o índice da raiz.

 a
p
n
 n ap

(IV) Para se extrair a raiz de uma raiz conserva-se o radicando e FUNÇÃO CRESCENTE (BASE > 1)
multiplicam-se os índices.
Exemplo: f ( x ) = ( 2)
x

n p np
a= a
Para montar o gráfico da função y = 2x, podemos descobrir pontos
(V) Um radical não se altera quando multiplicamos o seu índice e o substituindo valores de x e realizando as contas.
expoente do radical por um mesmo natural diferente de zero. Se x = −∞ → y = zero
... ...
np
n
a ap , p  0 Se x = −2 → y = 1
4
Se x = −1 → y = 1
2
Observação
Se x = 0 → y = 1
(I) Quando a > 0 e n é par, a notação n a ⊗
representa também a raiz Se x = +1 → y = 2
n-ésima positiva de a (a negativa é representada por − n a).
Se x = +2 → y = 4
 a se a  0 Se x = +3 → y = 8
Em particular, 2k a2k  a  
a se a  0 ... ...
Assim, a2 = a . Se x = +∞ → y = +∞

(II) Quando a < 0 e n for ímpar as leis das raízes podem ser aplicadas Montando o gráfico a partir dos pontos acima, temos:
para − n a (ou − n −a ) mas não para n a . Assim, por exemplo
para reduzirmos 3 −2 a um radical de índice 6 devemos fazer
3
2   3 2   6 22   6 4 e não 3 2  6 ( 2)2  6 4 .

POTÊNCIA DE EXPOENTE RACIONAL


Para dar significado às potências de expoentes fracionárias
precisamos do uso de radicais. A razão para isto, é que precisamos
1
definir an de modo que ela seja consistente com as leis dos expoentes
isto é, devemos ter:

 
 1
1 n  xn
an  a n   a1  a

110

PM_BOOK16 - MAT.indb 110 25/11/2022 19:09:41


FUNÇÃO EXPONENCIAL

FUNÇÃO DECRESCENTE (0 < BASE < 1) • f(x) = 2x

( )
x
Exemplo: f ( x ) = 1
2
Para montar o gráfico da função y = 2x , podemos descobrir pontos
substituindo valores de x e realizando as contas.
Se x = −∞ → y = +∞
... ...
Se x = −2 → y = 8
Se x = −1 → y = 4
Se x = 0 → y = 2
Se x = +1 → y = 1
Se x = +2 → y = 1
2
Se x = +3 → y = 1
4
... ...
Se x = +∞ → y = zero

Montando o gráfico a partir dos pontos acima, temos:

• f(x) = 2x + 2

Nos gráficos acima, podemos perceber algumas propriedades


importantes:
– A função exponencial da forma y = ax só existe nos dois
primeiros quadrantes. Isso ocorre pois não importa o
valor de x,y será sempre positivo.
– Nos dois casos as funções tendem a encostar no eixo x,
mas nunca de fato o tocam. Por conta disso, dizemos
que o eixo x é uma assíntota horizontal.
– O conjunto imagem da função exponencial y =ax é igual
a +*. • f(x) = –(2x)
– A função do tipo y = ax também será injetora, pois para
diferentes valores do domínio encontraremos diferentes
valores no contradomínio.
– Nas funções do tipo y = ax, teremos sempre f(0) = 1.
Portanto, o gráfico da função exponencial sempre
interceptará o eixo das abscissas no ponto (0,1).
Mas tudo isso é relativo à função f(x) = ax, como vimos no estudo
das funções há diversas operações que que podem ser aplicadas
a ax fazendo com que seu gráfico se modifique, vamos ver alguns
exemplos.
Vamos utilizar como base a função f(x) = 2x

111

PM_BOOK16 - MAT.indb 111 25/11/2022 19:09:45


FUNÇÃO EXPONENCIAL

• f(x) = –(2x) + 4 • f(x) = –|– (2x) + 4| + 2

Conseguimos muitas coisas interessantes.

• f(x) = |– (2x) + 4| 1. Reforçar que quando estamos falando de funções, pelo fato das
transformações que podem ocorrer, é difícil que tenhamos verdades
imutáveis.
2. Conseguimos aqui revisar pontos importantes sobre as
transformações que as funções podem sofrer inclusive no que diz
respeito as funções modulares.

EQUAÇÃO EXPONENCIAL
A regra básica é tentar igualar as bases pois daí teremos que 2
potências de mesma base serão iguais quando os expoentes também
forem. Basicamente teremos 3 tipos de equações exponenciais.
Tipo 1
x −1
 x7−1 
1
=
7
49x −1 ⇒ 7=
7 −1
 49  ⇒ 7= ( 7)
 
−1
( )
2 7

2x − 2
2x − 2 5
7 =( 7)
−1
7 ⇒ =−1 ⇒ 2x − 2 =−7 ⇒ x =−
7 2
Tipo 2
• f(x) = –|– (2x) + 4|
3x-1 – 3x+1 + 3x+2 = 306
Usando as propriedades de potências am+n = am · an teremos
3x ⋅ 3−1 − 3x + 3x ⋅ 31 + 3x ⋅ 32 =
306
3x
− 3x + 3x ⋅ 3 + 3x ⋅ 32 =
306
3
Podemos colocar 3x em evidência ou fazer uma substituição,
como 3x = a
a
− a + a⋅3 + a⋅9 =306
3
Multiplicando toda a equação por 3, para reduzirmos os
denominadores
a – 3a + 9a + 27a = 306 · 3
34a = 306 · 3
306 ⋅ 3 306 ⋅ 3
=a = = 9 ⋅ 3 = 27
34 34

Voltando a variável x
3x = 27
3x = 33 ⇒ x = 3
S = {x ∈ / x = 3}

112

PM_BOOK16 - MAT.indb 112 25/11/2022 19:09:46


FUNÇÃO EXPONENCIAL

Tipo 3 Vamos ver mais alguns exemplos de inequações


4x – 2x – 2 = 0 Exemplo: Se 5x+2, calcule o valor de 52x.
Também usando as propriedades de potência (am) = (an) = am · n
n m
Sabemos que 5x+2 = 100. Utilizando as propriedades das potências,
(2²) – 2x – 2 = 0
x temos que:
(2x)² – 2x – 2 = 0 5x ⋅ 52 = 100 ∴ 5x ⋅ 25 = 100 ∴ 5x = 4
Novamente podemos fazer uma substituição, 2 = a x
Precisamos descobrir o valor de 52x. Segue que:
a² – a – 2 = 0 52x = (5x)2 = 42 = 16
Resolvendo a equação do 2º grau Portanto 52x = 16
−( −1) ± ( −1)² − 4 ⋅ (1) ⋅ ( −2)
a=
2 ⋅1 Exemplo: Encontre o conjunto solução da equação exponencial
1± 1+ 8 1± 9 1± 3 (4x – 2x) = 56
=a = =
2 2 2 4x – 2x = 56
a = 2 ou a = −1 (22)x – 2x – 56 = 0
(2x)2 – 2x – 56 = 0
Voltando a variável x
2x = 2 ⇒ x =1 ou 2x = −1 ⇒ ∃ x ∈  Fazendo y = 2x temos
S =∈
{x  / x = 1} y 2 − y − 56 =0

∆ = ( −1)2 − ( 4 ) ⋅ (1) ⋅ ( −56 ) = 1 + 224 = 225


INEQUAÇÕES EXPONENCIAIS 1 ± 15
y= , assim, y = 8 ou y = −7
1º caso: {Se a > 1, função crescente, ax > ay então x > y (mantém 2
a desigualdade). O resultado y = –7 não convém, pois 2x é sempre positivo, assim:
2º caso: {Se 0 < a < 1, função decrescente, ax > ay então x < y
2x = 8 ↔ 2x = 23 ↔ x = 3 ↔ s = 3
(inverte a desigualdade).
Exemplo 1:
Resolver 2x > 128. 2
−14 1
Exemplo: A equação 2x = tem duas soluções reais.
Como a base (2) é maior que devemos manter a desigualdade 1024
para os expoentes Encontre a soma das duas soluções.
2
−14
2x > 27 ⇒ x > 7 2x =2−10 ⇔ x 2 − 14 =−10 ⇔ x 2 =4 ⇔ x=±2.
Logo, x1 + x 2 =
0.
Exemplo 2:
x 2x − 2
Resolver   ≥
3 125
.  3 1
Exemplo: Calcule o valor de x na equação   = .
5 27
 9  27
Como a base   é um número maior que 0 e menor que 1,
3 2x − 2
2x − 2  21  2x − 2
5  3 1 3   1 −2 
 9  = ⇒  2 
3−3 ⇒  32 
= = 3−3
devemos inverter a desigualdade para os expoentes 27
  3   
x x 3  
 3   5³   3   5 
  ≥ ⇒  ≥  3
 5   3³   5   3  − ( 2x − 2) =−3 ⇒ 2x − 2 =2 ou x =2
2
x≤3

Exercício Resolvido Exemplo: Resolver 2x > 128


Como a base (2) é maior que devemos manter a desigualdade
01. Resolva a equação 23x-4 = 4x+1.
para os expoentes
Resolução: 2x > 27 ⇔ x > 7
A ideia nesse tipo de problema é ‘igualar’ as bases:
23x-4 = (2²)x+1 = 22x+2. Daí, temos 3x – 4 = 2x + 2, logo x = 6.
x
 3  125
Exemplo: Resolver   ≥
5 27
 3
Exercício Resolvido Como a base  5  é um número maior que 0 e menor que 1,
 
02. Resolva a inequação 25x – 23 · 5x – 50 < 0. devemos inverter a desigualdade para os expoentes
x x 3
 3 5   3 5
3
Resolução:   ≥ 3→  ≥ 
5  3  5  3
Fazendo 5x = t (t > 0), temos t² – 23t – 50 < 0. As raízes da
expressão quadrática são 25 e –2 e, como a concavidade é voltada x≤3
para cima, tem-se que –2 < t < 25. No entanto, repare que t é
positivo (pois é potência de positivo), logo, 0 < t < 25. Voltando Vamos ver mais alguns exemplos?
à variável original, temos 0 < 5x < 25 ⇔ 0 < 5x < 52, o que nos
3x − 5 x
Exemplo: A desigualdade  
dá x < 2. 1  1
>  tem como conjunto
solução  2 4

113

PM_BOOK16 - MAT.indb 113 25/11/2022 19:09:47


FUNÇÃO EXPONENCIAL

a) S =
{x ∈  | x > 1} ⇒ S= {x ∈  | x ≤ −3 ou x ≥ 2}
b) S ={x ∈  | x < 5}
c) S = {x ∈  | x > 5} 2
− 5x + 7
d) S = {x ∈  | 1 < x < 5} Exemplo: Em relação à desigualdade: 3x <3
a) Encontre os valores de x, no conjunto dos reais, que satisfaçam
1 essa desigualdade;
Sendo 0 < < 1, temos
2 b) Encontre a solução da desigualdade para valores de x no
3x − 5 x 3x − 5 2x conjunto dos inteiros.
 1  1  1  1
  >  ⇒  >  ⇒
 2 4  2  2 2
− 5x + 7
a) 3x <3
3x − 5 < 2x ⇒ x < 5
Por conseguinte, o conjunto solução da inequação é Por isso, x 2 − 5x + 7 < 1 ⇒ x 2 − 5x + 6 < 0 ⇒ ( x − 2)( x − 3) < 0 o
S=
{x ∈  | x < 5}. que ocorre para 2 < x < 3

x2 − 6x +1
 1 1
Exemplo: Resolvendo a inequação   > em , a soma
das soluções inteiras é igual a: 4 4

a) 10
b) 12
c) 15
d) 21 b) A solução é 2 < x < 3 para números reais. Nesse intervalo,
e) 28 ]2, 3[ , não há inteiros, logo, não há solução nos inteiros.
x2 − 6x +1
 1 1
> ⇒ x 2 − 6x + 1 < 1 ⇒ x 2 − 6x < 0 ou
 
4 4 EQUAÇÕES E INEQUAÇÕES DO TIPO
ƒ(x)g(x)
Vamos falar mais sobre algumas equações e inequações
exponenciais intrigantes?
2
− 5x + 6
Vamos ver como resolvemos a equação exponencial x x =1
por exemplo.
2
− 5x + 6
xx para resultar em 1 temos a seguintes possibilidades.
S = {x ∈  | 0 < x < 6} I. Que tenhamos o expoente (x² – 5x + 6) igual a 0, lembrando
da restrição que o caso de 00 não nos serve por ser
indeterminado.

Soluções inteiras: 1 + 2 + 3 + 4 + 5 = 15 II. Que tenhamos a base da potência (x) igual a 1, pois 1n = 1.
Sendo assim vamos verificar se conseguimos ter x 2 − 5x + 6 =0
com x ≠ 0.
Exemplo: Dada a inequação , o conjunto verdade V, considerando
o conjunto universo como sendo o dos reais, é dado por x 2 − 5x + 6 = 0 ⇒ S = 5 e P = 6 ⇒ x = 2 ou x = 3
a) V= {x ∈ R | x ≤ −3 ou x ≥ 2}. Então por enquanto temos que x = 2 ou x = 3 são soluções pois
b) V= {x ∈ R | x ≤ −3 e x ≥ 2}. resultam em 20 = 1 e 30 = 1 .
c) V = {x ∈ R | −3 ≤ x ≤ 2}.
d) V= {x ∈ R | x ≤ −3}. Agora veremos se a com a base da potência sendo igual teremos
solução.
e) V ={x ∈ R | x ≥ 2}.
2
1 ⇒ 11 −5⋅1+ 6 =
x= 12 =
1
x −1 x −3
 x
  3 3 1
 3 2
 ≥  , temos que = = 3−1 , ou seja, Então vemos que x = 1 também é solução da nossa equação e
  9 9 3 dessa forma S = {1, 2, 3}.
x −1
 2x  x
( x −1)
≥ ( 3−1 )
x −3
 3  ⇒ 32 ≥ 33 − x , como 3 é uma base positiva maior 2
  Agora vamos ver a seguinte equação x 2x − 7x + 4
= x.
x
que 1 temos que: ( x − 1) ≥ 3 − x Nessa nova equação temos que ver o seguinte, o lado direito da
2 igualdade não é uma potência e sim somente x. Dessa forma pode
haver a igualdade para o seguinte.
x2 x x2 x
⇒ − ≥ 3− x ⇒ + − 3 ≥ 0 ⇒ x2 + x − 6 ≥ 0
2 2 2 2 =0n 0, com n ≠ 0
 k
Para a equação de 2º grau: ∆ = 1 − 4 ⋅ 1⋅ ( −6 ) = 25 daí, para as  1 = 1, ∀ k ∈ 
−1 ± 25 2
=
raízes: x = assim, para que x 2 + x − 6 ≥ 0 devemos
2 −3 Então vemos que nesses tipos de equações fazer o teste para que
ter: x ≤ −3 ou x ≥ 2 as bases das potências sejam iguais a 0 ou a 1 é uma boa pedida.

114

PM_BOOK16 - MAT.indb 114 25/11/2022 19:09:50


FUNÇÃO EXPONENCIAL

1
2 Agora fazendo a intersecção de x > 1 e < x < 4 teremos
x =0 ⇒ 02⋅0 − 7⋅0 + 4
=0 ⇒ 04 =0 (V) 2
2 1
1 12⋅1 − 7⋅1+ 4 =⇒
x =⇒ 1 1−1 =1 (V) <x<4
2
Então temos x = 0 ou x = 1 como soluções.
Agora repare em um detalhe na nossa equação.
2
x 2x − 7x + 4 = x possui a mesma base de potência em ambos os lados
da igualdade e dessa forma se tivermos x1 = x teremos também mais
soluções e então devemos verificar se é possível que 2x 2 − 7x + 4 = 1.
2x 2 − 7x + 4 =1 ⇒ 2x 2 − 7x + 3 =0 ⇒

− ( −7) ± ( −7)
2
− 4 ⋅ 2 ⋅ 3 7 ± 49 − 24 7 ± 25 7 ± 5
=x = = = ⇒
]1, +∞[ ∩ 
2⋅2 4 4 4 1 
, 4 = ]1, 4[
1  2 
x = 3 ou x =
2
Sendo assim teremos III. Agora pensaremos ao contrário, se a base da potência for um
número entre 0 e 1 devemos ter o expoente maior que zero para que
3 =3
1
(V) o resultado da potência seja menor que 1.
1
 1 1 0 < x < 1 e 2x 2 − 9x + 4 > 0
  = (V)
 2 2 Sobre 2x 2 − 9x + 4 > 0 já sabemos pois se trata do mesmo esboço
 1  acima.
Sendo assim nosso conjunto solução é S = 0, ,1, 3
 2  1
2x 2 − 9x + 4 > 0 ⇒ x < ou x > 4
Então vimos que nossas resoluções irão se basear praticamente 2
1
no teste do 0 e do 1 como soluções e em outros casos estudar se há Agora fazendo intersecção de 0 < x < 1 e x < ou x > 4 teremos
2
outras maneiras de haver a igualdade
Vamos agora falar das inequações dessa maneira. Iremos resolver
2
− 9x + 4
a inequação x 2x < 1.
Vamos considerar 3 casos.
I. Devemos verificar se 0 ou 1 são soluções.
2
x = 0 ⇒ 02⋅0 − 9⋅0 + 4
< 1 ⇒ 04 < 1 ⇔ 0 < 1 ⇒ x = 0 é solução (V)
2⋅12 − 9 ⋅1+ 4 −3
x = 1⇒ 0 < 1 ⇒ 1 < 1 ⇒ 0 < 1 ⇒ x = 1 não é solução (F)
Por enquanto somente x= 0 é solução.

]0,1[ ∩  −∞,
1  1
∩ ]4, +∞[ =  0, 
II. Agora vamos ver o seguinte, se a base da potência for maior  2   2
1, que expoente devemos ter para que o resultado seja menor que 1?
Um expoente negativo. Assim
Então por fim a solução da nossa inequação será todos os
x > 1 e 2x 2 − 9x + 4 < 0 intervalos encontramos por nós.
Vamos encontrar as raízes de 2x 2 − 9x + 4 igualando a 0.  1
S1 = {0}, S2 = {1 < x < 4} e S3 = 0 < x < 
 2
− ( −9 ) ± ( −9 )
2
− 4 ⋅2⋅ 4 9 ± 49 9 ± 7
2x 2 − 9x + 4 = 0 ⇒ x = = = ⇒  1 
2⋅2 4 4 S1 ∩ S2 ∩ S3 = 0 ≤ x < ou1 < x < 4
 2 
1
x = 4 ou x =
2
EXERCÍCIOS DE

FIXAÇÃO
Agora vamos fazer 2x 2 − 9x + 4 < 0

( 2) ( )
3x − 1 2x −1
01. Calcule o valor de x na igualdade: = 3
16 .

02. Calcule (x + 1)6, sabendo que (0,0625)x+ 2 = 0,25.

03. Calcule se houver o valor de x na equação 5x-2 – 5x + 5x+1 = 505.

04. Encontre o valor x na equação 23x + 23x+1 + 23x+2 + 23x+3 = 240.

05. 9x + 3x = 90, calcule o valor x.


1
<x<4
2 06. Encontre o valor de x na equação 102x-1 – 11 · 10x-1 + 1 = 0.

115

PM_BOOK16 - MAT.indb 115 25/11/2022 19:09:52


FUNÇÃO EXPONENCIAL

07. Resolva a inequação (0,1)3-4x < 0,0001. a) ímpar múltiplo de 9. c) par múltiplo de 15.
b) par divisor de 10.000. d) ímpar múltiplo de 25.
08. Resolva a inequação 25 < 1252x-1 < 125.
1
09. (ESA) Quantos algarismos são necessários para escrever o produto 09. (ESPCEX) O domínio da função f(x) = é
−x −2 1
(16)13 · (25)25? a) *- 3 −
9
a) 50 d) 52 b) -
b) 51 e) 53 c) +
c) 54 d) *+
e) 
10. (ESA) O conjunto solução da equação exponencial 4x – 2x = 56 é
a) {–7, 8} d) {2, 3} 10. (ESPCEX) A soma e o produto das raízes da equação (2x+6)x²-6x+5 = 1
b) {3, 8} e) {8} são, respectivamente:

c) {3} a) –5 e 6 d) 0 e –6
b) 11 e 30 e) –11 e 0
c) 0 e –30
EXERCÍCIOS DE

TREINAMENTO 11. (EEAR) O valor da raiz da equação 2x+1 + 2x-1 = 40 é um número


a) inteiro positivo. c) inteiro negativo.
b) irracional. d) imaginário puro.
3x − 5 x
01. (EEAR) A desigualdade  1   1
>  tem como conjunto 12. (AFA) Se x ∈  e 75x = 243, então 7–3x é igual a
solução:  2 4 a) 1/3 c) 1/27
a) S = {x ∈  | x > 1} c) S = {x ∈  | x > 5} b) 1/9 d) 1/81
b) S = {x ∈  | x < 5} d) S = {x ∈  | 1 < x < 5}
− x2

02. (AFA) A cada ano que passa, o valor de uma máquina diminui 13. (EEAR) O conjunto solução da inequação  1  ≥ 2, sendo
10% em relação ao do valor do ano anterior. Se V for o valor da U = , é  2
máquina no ano da compra, após 10 anos será a) {x ∈  / x ≤ –1 ou x ≥ 1}.
a) (0,9)10 V c) (0,1)9 V b) [–1,1].
b) (0,5)9 V d) (0,1)10 V c) ∅.
d) .
03. (AFA) A equação 2x+1 + 4x = 80 para x real:
a) tem duas soluções c) não admite solução 14. O produto das raízes da equação 4x-2 – 17 · 2x-4 + 1 = 0 é igual a
b) tem uma única solução d) admite x = 4 como solução a) 0 c) 16 e) 18
b) 4 d) 17
04. (ESPCEX) Se f(x) = 5x, com x ∈ , o valor de f(x + 2) – f(x + 1) é:
a) 30 · f(x) c) 20 · f(x) e) 5 · f(x) 15. (ESPCEX) A fórmula N = 6 · 108 · V-3/2 relaciona, numa dada
b) 24 · f(x) d) 9 · f(x) sociedade, o número N de indivíduos que possuem renda anual
superior ao valor V, em reais. Nessas condições, pode-se afirmar que,
para pertencer ao grupo dos 600 indivíduos mais ricos dessa sociedade
05. (ESPCEX) A soma e o produto das raízes da equação
x2 − x − 9
é preciso ter no mínimo uma renda anual de
 3 243
a) R$ 10.000,00.
9  = são, respectivamente
5 125
b) R$ 100.000,00.
a) 1 e –12 c) –2 e –8 e) 7 e 10
c) R$ 1.000.000,00.
b) 7 e 12 d) –1 e 12
d) R$ 10.000.000,00.
06. (AFA) O conjunto-solução da inequação (0,5)x(x-2) < (0,25)x – 1,5 é e) R$ 100.000.000,00.
a) {x ∈  l x < 1}. c) {x ∈  l 1 < x < 3}.
16. (AFA) Sabe-se que o isótopo do carbono, C14, tem uma meia vida
b) {x ∈  l x > 3}. d) {x ∈  l x < 1 ou x > 3}. de 5760 anos, isto é, o número N de átomos de C14 na substância
é reduzido a N/2 após um espaço de tempo de 5760 anos. Essa
07. (EEAR) Se (0,0625)x+2 = 0,25, então (x + 1)6 vale substância radioativa se degrada segundo a sequência N = N0 · 2-t,
3 1 1 {0,1,2,...} em que N0 representa o número de átomos de C14 na
a) − b) c) 64 d)
2 32 64 substância no instante t = 0 e t é o tempo medido em unidades de
5760 anos. Com base nas informações acima, pode-se dizer que
x a) o número de átomos quando t = 1 era 5760.
08. (AFA) Sejam as funções f:  →  e g:  →  definidas por f(x) =
e g(x) = 2 . Considere os números A e B tais que
-x 2 b) após 11520 anos haverá a quarta parte do número inicial de
átomos.
A = f(1) + f(2) + ... + f(50)
c) o número de átomos será igual a um terço de N0 quando
B = 1 + g(1) + g(2) + ... + g(n) + ... decorridos 1920 anos.
Se o produto de A por B tende ao número α, então, α é d) Quando t = 5760 haverá metade do número inicial de átomos.

116

PM_BOOK16 - MAT.indb 116 25/11/2022 19:09:52


FUNÇÃO EXPONENCIAL

17. (AFA) No intervalo [–1, 100], o número de soluções inteiras da O gráfico que MELHOR representa A em função de t é:
inequação 3x – 8 > 32–x é
a) 97 b) 98 c) 99 d) 100

18. (EN) O elemento químico Califórnio, Cf251, emite partículas alfa,


transformando-se no elemento Cúrio, Cm247. Essa desintegração
obedece à função exponencial N(t) = N0e-αt, onde N(t) é quantidade
de partículas de Cf251 no instante t em determinada amostra; N0 é
a quantidade de partículas no instante inicial; e α é uma constante,
chamada constante de desintegração. Sabendo que em 898 anos a
concentração de Cf251 é reduzida à metade, pode-se afirmar que o
tempo necessário para que a quantidade de Cf251 seja apenas 25% da
quantidade inicial está entre
a) 500 e 1000 anos. d) 2000 e 2500 anos.
b) 1000 e 1500 anos. e) 2500 e 3000 anos.
c) 1500 e 2000 anos.

4x − x2
19. (AFA) Dada a expressão  1  , em que x é um número real
 3 1 x
a − 1(a ∈  e a > 1).
24. (AFA) Seja f:  → B a função definida por f(x) =
qualquer, podemos afirmar que 2
a) o maior valor que a expressão pode assumir é 3. Analise as afirmativas abaixo, classificando-as em (V) verdadeira(s) ou
(F) falsa(s).
b) o menor valor que a expressão pode assumir é 3.
( ) f(p + q) = f(p) – f(q), ∀p, q ∈ .
1
c) o menor valor que a expressão pode assumir é . ( ) f é crescente ∀x ∈ .
81
1  3 
d) o maior valor que a expressão pode assumir é . ( ) Se x ∈ ]–∞,0[, então y ∈  − , −1 .
27  2 
1 ( ) Se B = ]–∞,–1[, então f é bijetora.
e) o menor valor que a expressão pode assumir é .
9 A sequência correta é
a) F - F - V - V c) V-F-F-F
20. (AFA) A função real f definida por f(x) = a · 3x + b, sendo a e b
constantes reais, está graficamente representada abaixo. b) F - V - F - V d) F - V - V - V

Pode-se afirmar que o ex + 4


produto 25. (EPCAR 3° ANO) Se o domínio de f(x) = é
e2 − x 2
(a · b) pertence Obs.: e = 2,7182...
ao intervalo real a) {x ∈  / x < –e ou x > e} c) {x ∈  / x < –1 ou x > 1}
a) [-4,-1[ b) {x ∈  / –e < x < e} d) {x ∈  / –2 < x < 2}
b) [-1,2[
c) [2,5[ 26. (AFA) Considere a função real f:  →  definida por f(x) = ax – b,
d) [5,8] em que 0 < a < 1 e b > 1. Analise as alternativas abaixo e marque a
falsa.
a) Na função f, se x > 0, então –b < f(x) < 1 – b.
100
x b) Im(f) contém elementos menores que o número real –b.
21. (EN) Dadas as funções reais f ( x ) = e g ( x ) = 2 , pode-se
2
1 + 2− x c) A raiz da função f é um número negativo.
afirmar que (g  f-1)(90) é igual a
d) A função real h, definida por h(x) = f(|x|) não possui raízes.
a) 10 c) 1 3
e)
b) 3 1 10
d) 27. (EN) Após acionado o flash de uma câmera, a bateria imediatamente
3 começa a recarregar o capacitor do flash, que armazena uma carga
 −t

=
22. (AFA) Todos os valores reais de x para os quais existe f(x) x 4x −1 − x elétrica dada por =
Q(t) Q0 1 − e 2  , onde Q0 é a capacidade limite de
são tais que  
carga e t é medido em segundos. Qual o tempo, em segundos, para
a) x > 1 c) 0 < x < 1/2 recarregar o capacitor de 90% da sua capacidade limite?
b) 0 < x < ½ ou x ≥ 1 d) 0 < x < ½ ou x > 1 a) n10 c)  n10 e)  n(10)2
b) n(10)² d) (  n10) −1
23. (EPCAR 3° ANO) Leia atentamente as seguintes afirmações:
- Em radioatividade, define-se atividade A de uma amostra radioativa 28. (AFA) Considere a função real f definida por f(x) = ax com
como sendo a velocidade de desintegração de seus átomos. a ∈ ]0,1[. Sobre a função real g definida por g(x) = |–b – f(x)| com b ∈
- A constante de desintegração λ representa a probabilidade de que ]–∞,–1[, é correto afirmar que
um átomo do elemento se desintegre na unidade de tempo. a) possui raiz negativa e igual a loga (–b).
- A0 é a atividade de uma amostra no instante t0 e A é a atividade da b) é crescente em todo o seu domínio.
amostra no instante t.
c) possui valor máximo.
- A função A = f(t) é representada por A = A0 · e-λt, onde t é o tempo
e e = 2,7182... d) é injetora.

117

PM_BOOK16 - MAT.indb 117 25/11/2022 19:09:53


FUNÇÃO EXPONENCIAL

29. Considere a equação 22x – t · 2x + 4 = 0. Qual das opções representa a) A função v tem conjunto imagem Im = ]1, + ∞[.
o conjunto de todos os valores de t para os quais a equação admite 2 b) O gráfico da função h é uma reta.
raízes reais distintas?
c) O domínio da função v é *+.
a) t > 4 c) t > 4 ou t < -4 e) t = 6
d) Os gráficos das funções f e g se interceptam num ponto de
b) t < -4 d) t = 5 abscissa menor que 1.

30. (AFA) Analise os itens abaixo classificando-os em V (verdadeiro)


ou F (falso).
( ) Em , o conjunto solução da inequação 8 · (0,5)x – 1 ≤ 0 é dado EXERCÍCIOS DE
por [4, +∞[.
( ) A função real y = e1-x é crescente ∀x ∈  (considere e a base dos
logaritmos neperianos).
COMBATE
( ) Se f(x) = 2x, então f(a) · f(b) é sempre igual a f(a + b), onde a e b
são reais quaisquer 01. (EFOMM 2014) O valor de x para resolver a equação 4x + 6x – 2 · 9x é
A sequência correta é a) 0 d) 3
a) F – F – V c) F–V–V b) 1 e) 4
b) V – V – F d) V – F – F c) 2

31. (AFA) Assinale a alternativa INCORRETA. 02. (EN 2006) No universo U = +, o conjunto solução da inequação
a) O conjunto solução da inequação (2 − 3)x > −1 é . x2x²-9x+4 < 1 é
x −2 2
b) O número real que satisfaz a sentença (3 ) = 52 − x
é divisor
 1 1 
de 1024. a) 0, 2   1, 4 c)  2 ,1  0 e) [0,1[ ∪ ]1,4[
   
c) A função exponencial definida por f(x) = –(a – 4)x é decrescente
1  1 
 2 ,1  4,   2 , 4   0
se 4 < a < 5. b) d)
d) Se y = 10x é um número entre 10.000 e 100.000, então x está
entre 4 e 6.
03. A soma de todos os números reais x tais que (2x – 4)³ + (4x – 2)³ –
(4x + 2x – 6)³ é igual a:
32. (AFA) Sabendo-se que b é um número real tal que b > 1 e que
a função real f:  → B é tal que f(x) = 2 – b|x|, analise as alternativas 3 c) 5 7
a) e)
abaixo e marque a FALSA. 2 2 2
b) 2 d) 3
a) A função f admite valor mínimo.
1
b) x ≤ 1 ⇔ 2 − ≤ f(x) < 2. 04. O número de raízes reais da equação ( 4  15 )x  ( 4  15 )x  62
b
é igual a:
c) A função f é par.
a) 1 d) 4
d) Se B = [0, 2[ então f é sobrejetora.
b) 2 e) 5

33. (EPCAR 3° ANO) Se f:  → B é tal que f(x) = –ab , em que a ∈ +


x * c) 3
e 0 < b < 1, então
05. Um acidente de carro foi presenciado por 1/65 da população
a) f(x + y) = f(x) · f(y), ∀x, y ∈ .
de Votuporanga (SP). O número de pessoas que souberam do
b) f é decrescente ∀x ∈ . acontecimento t horas após é dado por:
c) se x ∈ ]–∞,0], então, y ∈ ]–∞,–a]. B
f t 
d) f é bijetora se B = -. 1 Cekt

34. (EPCAR 3° ANO) A população de certo tipo de bactéria triplica a onde B é a população da cidade. Sabendo-se que 1/9 da população
cada meia hora. Em uma experiência, colocou-se, inicialmente, uma soube do acidente 3 horas após, então o tempo que passou até que
amostra de 1000 bactérias. Com base nisso, é correto afirmar que se 1/5 da população soubesse da notícia foi de:
Dado: log 2 = 0,3 e log 3 = 0,4. a) 4 horas.
b) 5 horas.
e) ao final da experiência, obtém-se um total de 6,561 x 106 bactérias,
então, o tempo total do experimento foi maior que 6 horas. c) 6 horas.
a) o tempo da experiência foi de 2 horas, então, o número de d) 5 horas e 24 minutos.
bactérias obtidas foi menor que 7,5 x 104. e) 5 horas e 30 minutos.
b) a população de bactérias, ao final da experiência, chegou a
4 x  x²
06. (ESPCEX) Dada a expressão  
80.000, então, o tempo da experiência foi inferior a 2h30min. 1
, em que x é um número real
c) um cientista deseja obter um número de bactérias entre 20.000 qualquer, podemos afirmar que  3 
e 40.000, então, o tempo do experimento deverá estar entre
a) o maior valor que a expressão pode assumir é 3.
3h25min e 4 horas.
b) o menor valor que a expressão pode assumir é 3.
35. (EPCAR 3° ANO) Considere as funções reais f, g, h e v tais que c) o menor valor que a expressão pode assumir é 1/81.
f:  → *+ é dada por f(x) = ax (0 < a < 1), g é a inversa de f, h é d) o maior valor que a expressão pode assumir é 1/27.
definida por h(x) = f(g(x)) e v é definida por v(x) = h(x) + 1 e, a seguir,
e) o menor valor que a expressão pode assumir é 1/9.
assinale a alternativa FALSA.

118

PM_BOOK16 - MAT.indb 118 25/11/2022 19:10:00


FUNÇÃO EXPONENCIAL

 3x  27y  9
 (a x
− a− x )
07. (ESPCEX) O conjunto solução do sistema  2
y³  xy²  0
 3
é formado
2 (ITA) Considere a equação
+ a− x )
= m, na variável real x,
(a x

com 0 < a ≠ 1. O conjunto de todos os valores de m para os


por dois pontos, cuja localização no plano cartesiano é
quais esta equação admite solução real é
a) ambos no primeiro quadrante.
a) (–1, 0) ∪ (0, 1)
b) um no quarto quadrante e o outro no eixo X.
b) (–∞, –1) ∪ (1, +∞)
c) um no segundo quadrante e o outro no terceiro quadrante.
c) (–1, 1)
d) um no terceiro quadrante e o outro no eixo Y.
d) (0, ∞)
e) um no segundo quadrante e o outro no eixo X.
e) (–∞, +∞)

08. (ESPCEX) Na pesquisa e desenvolvimento de uma nova linha de


defensivos agrícolas, constatou-se que a ação do produto sobre a
população de insetos em uma lavoura pode ser descrita pela expressão
3 (ITA) Se x é um número natural com 2015 dígitos, então o
número de dígitos da parte inteira de 7 x é igual a
N(t) = N 0 · 2kt, sendo N 0 a população no início do tratamento, N(t), a) 285.
a população após t dias de tratamento e k uma constante, que b) 286.
descreve a eficácia do produto. Dados de campo mostraram que, c) 287.
após dez dias de aplicação, a população havia sido reduzida à quarta
parte da população inicial. Com estes dados, podemos afirmar que o d) 288.
valor da constante de eficácia deste produto é igual a e) 289.
a) 5–1 c) 10 e) –10–1
b) –5–1 d) 10–1
4 (ITA) Determine todos os valores reais de x que satisfazem
a inequação 43x-1 > 34x.
09. (ESPCEX) A inequação 10x + 10x+1 + 10x+2 + 10x+3 + 10x+4 < 11111,
em que x é um número real,
5 (ITA) Encontre o conjunto solução S ⊂  da inequação
4
a) não tem solução.
b) tem apenas uma solução.
exponencial: 3x −2 +
∑3
k =1
x +k

1081
18
.

c) tem apenas soluções positivas.


d) tem apenas soluções negativas. GABARITO
e) tem soluções positivas e negativas.
EXERCÍCIOS DE FIXAÇÃO
10. (ESPCEX) Um jogo pedagógico foi desenvolvido com as seguintes 01. DISCURSIVA 04. DISCURSIVA 07. DISCURSIVA 10. C
regras: 02. DISCURSIVA 05. DISCURSIVA 08. DISCURSIVA
– os alunos iniciam a primeira rodada com 256 pontos; 03. DISCURSIVA 06. DISCURSIVA 09. B
– faz-se uma pergunta a um aluno. Se acertar, ele ganha a metade EXERCÍCIOS DE TREINAMENTO
dos pontos que tem. Se errar, perde metade dos pontos que tem;
01. B 10. C 19. C 28. A
– ao final de 8 rodadas, cada aluno subtrai dos pontos que tem os
02. A 11. A 20. A 29. B
256 iniciais, para ver se “lucrou” ou “ficou devendo”.
03. B 12. C 21. B 30. A
O desempenho de um aluno que, ao final dessas oito rodadas, ficou
devendo 13 pontos foi de 04. C 13. A 22. D 31. C
a) 6 acertos e 2 erros. 05. A 14. C 23. C 32. D
b) 5 acertos e 3 erros. 06. C 15. A 24. A 33. C
c) 4 acertos e 4 erros. 07. D 16. C 25. B 34. C
d) 3 acertos e 5 erros. 08. D 17. B 26. B 35. B
e) 2 acertos e 6 erros. 09. A 18. C 27. B
EXERCÍCIOS DE COMBATE
01. A 04. B 07. E 10. B
02. A 05. A 08. B

DESAFIO PRO 03. E


DESAFIO PRO
06. C 09. D

01. C 04. x < log8 2

1
9
(ITA) Considere a função f:  – {0} → , 02. C  1
1 1 05. S =
x ∈  : x ≤ log3 
3x −2 ( 92x +1 ) 2x − ( 32x +5 ) x + 1. A soma de todos os
03. D  2
f :  − {0} =
→  ,f(x)
valores de x para os quais a equação y + 2y + f(x) = 0 tem raiz
2
ANOTAÇÕES
dupla é:
a) 0
b) 1
c) 2
d) 4
e) 6

119

PM_BOOK16 - MAT.indb 119 25/11/2022 19:10:01


FUNÇÃO EXPONENCIAL

ANOTAÇÕES

120

PM_BOOK16 - MAT.indb 120 25/11/2022 19:10:01


FUNÇÃO LOGARÍTMICA

DEFINIÇÃO PROPRIEDADES
Sejam a e b números reais positivos e a ≠ 1, define-se logaritmo de Sejam a, b, c ∈  +∗ e a ≠1 e α, β ∈  e n ∈ N, n ≥ 2, então:
b na base a como o expoente x que satisfaz ax = b. a. (Log do produto) loga (b · c) = loga b + loga c
loga b = x ⇔ ax = b b
b. (Log da divisão) loga    loga b  loga c
c
onde b é chamado logaritmando, a é a base e x é o logaritmo.
c. (“Regra do peteleco”) loga (b α) = α · loga b
O logaritmando também é chamado antilogaritmo e indicado por:
b = antiloga x ⇔ x = loga b ⇔ ax = b. Assim, antilog2 3 = 2³ = 8. 1
d. (“Regra do peteleco invertido”) log( a ) b   loga b

Convenciona-se que a omissão da base, escrevendo-se apenas
log b, indica que trata-se dos logaritmos decimais cuja base é a = 10. Exemplos:
Exemplos: log10 2 + log10 5 = log10 10 = 1
log2 8 = 3, pois 23 = 8  12 
log2 12  log2 3  log2    log2 4  2
1 1  3
log3  4 , pois 34 
81 81 log7 32 = log7 (25) = 5 · log7 2
2
 1 1
log 1 25  2, pois    25 log27 2  log( 33 ) 2   log3 2
5 5 3
1
1 5
log49 7 = , pois 49 2 = 7 = =
log81 32 log (25 ) log3 2
2 ( 34 )
4
CONSEQUÊNCIAS IMEDIATAS:
CONDIÇÃO DE EXISTÊNCIA  1
O logaritmo de b na base a somente é definido quando a. loga     loga b
b
a  0 e a  1 b. log 1  b   loga b

b  0  
a
1
Exemplos: c. loga n b   loga b
n
Para que valores de x está definido log(x+1) (3 – x).
d. logn a b  n  loga b
logaritmando: 3 −x > 0 ⇔ x < 3
A expressão –loga b é comumente chamada cologaritmo de b
base: x +1 > 0 ⇔ x >−1 na base a, o que significa que o cologaritmo é o oposto do logaritmo.
x +1 ≠ 1 ⇔ x ≠ 0
 1
co loga b   loga b  loga    log 1  b
O logaritmo está definido para x ∈ ]−1, 3[ − {0}. b  
a

CONSEQUÊNCIAS IMEDIATAS
Sejam a, b, c ∈  +∗ e a ≠1 e k ∈  , então: MUDANÇA DE BASE
a. loga 1 = 0 Sejam a, b, c ∈  +∗ e a, c ≠ 1, temos:
b. loga a = 1 logc b
loga b =
c. loga ak = k logc a
d. alogab = b Exemplo:
e. loga b = loga c ⇔ b = c log2 8 3 3
log14 8   
Exemplos: log2 14 log2 2  log2 7 1 log2 7
log2 1 = 0 CONSEQUÊNCIAS:
log 7
7 =1 a. loga b =
1
logb a
log3 3 5
 5
b. logc a · loga b = logc b
2 log227
= 27
c. loga b · logb c · logc d · ... · logy z = loga z

121

PM_BOOK16 - MAT.indb 121 25/11/2022 19:10:14


FUNÇÃO LOGARÍTMICA

Exemplos:
2 5
loga b   logb a 
5 2
log2 7 · log7 4 = log2 4 = 2

FUNÇÃO LOGARÍTMICA
É dita função logarítmica, toda e qualquer função de domínio e
contradomínio real e que tenha lei de formação na forma y = loga x.
*+ →
f:
→ f (x) =
loga x

Como podemos observar, existem restrições para o domínio


da função logarítmica. Essas restrições vêm da inversa da função
logarítmica. Observe abaixo.
Invertendo a função exponencial, temos:
y = ax
x = ay
loga x = loga ay
Temos nesse caso o eixo y como uma assíntota pois nunca teremos
loga x= y ⋅ loga a x = 0 em f ( x ) = loga x .
loga x = y Temos como raiz x = 1 pois f ( x ) = 0 ⇒ loga x = 0 ⇒ x = a0 = 1.
Uma função f ( x ) = loga x com a > 1 é uma função crescente.
Invertendo a função logarítmica, temos:
y = loga x BASE ENTRE ZERO E 1: (0 < a < 1)
x = loga y Agora utilizaremos o mesmo raciocínio anterior. Para exemplificar
vamos utilizar a função abaixo.
ax = y f ( x ) = log 1 x
2
Ou seja, ao invertemos a função exponencial, obtemos a função
logarítmica e vice-versa. Por conta disso, o domínio da função y
 1
exponencial será igual ao contradomínio da função logarítmica e o Temos que= y log 1 x ⇒= x   e podemos pensar o seguinte:
contradomínio da exponencial será igual ao domínio da logarítmica. 2  2
y
Sendo assim temos que o domínio de uma função logarítmica deve 1  1
para uma potência cuja base é e o expoente é y e x =   quanto
obedecer 2  2
 x>0 mais aumentarmos o valor de x (resultado da potência) devemos
f ( x ) loga x ⇒ Domf : 
= diminuir o expoente y pois temos uma base que está entre 0 e 1 e se
a > 0 e a≠1
quisermos diminuirmos o resultado x devemos aumentar o expoente.
Que é a condição de existência da nossa operação de logaritmo.

GRÁFICO
Para construir o gráfico da função logarítmica, precisaremos
estudar duas possibilidades de valores para a base do log.

BASE MAIOR DO QUE 1: (a > 1)


Uma das maneiras de construir o gráfico é achar pontos através
da substituição de valores em x, por exemplo.
Lembra que falamos que log seria a mesma coisa que expoente?
Veja só a função abaixo por exemplo.
f ( x ) = log2 x

Temos que= y log2 x ⇒= x 2y e podemos pensar o seguinte: para


uma potência cuja base é 2 e o expoente é y e x = 2y quanto mais
aumentarmos o valor de x (resultado da potência) mas também deve-
se se aumentar o expoente y e vice versa.

Uma função f ( x ) = loga x com 0 < a < 1 é uma função decrescente.

122

PM_BOOK16 - MAT.indb 122 25/11/2022 19:10:16


FUNÇÃO LOGARÍTMICA

Nos gráficos acima, podemos perceber algumas propriedades • f ( x ) = − (log2 x )


importantes
– A função logarítmica da forma y = loga x só existe no 1º e
4º quadrantes. Isso quer dizer que não está definida para x
negativo.
– Nos dois casos as funções tendem a encostar no eixo y, mas
nunca de fato o tocam. Por conta disso, dizemos que o eixo y
é uma assíntota vertical.
– O conjunto imagem da função logarítmica da forma y = loga x
é igual a .
– A função do tipo y = loga x também será injetora, pois para
diferentes valores do domínio encontraremos diferentes
valores no contradomínio.
– Nas funções do tipo y = loga x , teremos sempre f(1) = 0.
Portanto, o gráfico da função exponencial sempre interceptará
o eixo das abscissas no ponto (1,0)

Mas tudo isso é relativo à função f ( x ) = loga x, vimos isso inclusive


no módulo de função exponencial. Vamos brincar aqui também?

Vamos utilizar como base a função f ( x ) = log2 x


• f ( x ) = log2 x
 
Veja que o gráfico de − (log2 x ) é igual ao gráfico de  log 1 x  pois
 2 
 
( )
podemos fazer  log 1 x  = log −1 x =
  2
1
−1
. (log2 x ) = − (log2 x ) .
 2 

• f (x) =
− (log2 x ) + 4

• f ( x ) log2 x + 2
=

• f ( x ) = log2 x

123

PM_BOOK16 - MAT.indb 123 25/11/2022 19:10:17


FUNÇÃO LOGARÍTMICA

• f ( x ) = − log2 x EQUAÇÕES LOGARÍTMICAS


1º tipo: Se a > 0; a ≠ 0 e α ∈ 
loga x = b → x = ab
Exemplo:
log2 ( 3x + 1) =
4
Usando a definição de logaritmo
2=
4
3x + 1
3x + 1 =16
3x = 15 ⇒ x = 5

2º tipo: log=
ax loga N ⇒=
x N
Exemplo:
log2 ( 3x − 5) =
log2 7
Igualando os logaritmandos
3x − 5 = 7 → 3x = 12 → x = 4

• f (x) =
− log2 x + 2 3º tipo: incógnita auxiliar
Exemplo:

(log2 x )
2
− log2 x =
2
Fazendo log2 x = y teremos
y2 − y = 2 ⇒ y2 − y − 2 = 0

Resolvendo a equação do 2º grau

− ( −1) ± ( −1) − 4 (1)( −2) 1 ± 1 + 8 1 ± 9 1 ± 3


2

=y = = =
2 ⋅1 2 2 2
4 −2
y= = 2 ou y = = −1
2 2
Voltando a variável x
log2 x = 2 ⇔ 22 = x ⇔ x = 4
ou
1
log2 x =−1⇔ 2−1 = x ⇔ x =
2
f ( x ) log2 ( x + 3)
• = Temos que ter atenção a um detalhe muito importante, a condição
de existência do logarítmo, lembra?
 a>0
logb a ⇒ 
b > 0 e b ≠1
Toda vez que temos um logarítmo isso deve ser primordial.
No exemplo log2 ( 3x − 5) =
log2 7 temos que ter a preocupação de
3x – 5 > 0, porém como na nossa resolução já iremos ter que igualar
3x – 5 = 7 e 7 > 0 não será um problema se não fizermos antes a
condição 3x – 5 > 0 que diz pra gente que essa equação logarítmica
5
só admite soluções tai que x > .
3
Já no exemplo (log2 x ) − log2 x =
2
2 ao fazemos a condição de
existência x > 0 antecipadamente é muito bom pois previne que
no final encontremos soluções que não satisfazem a condição de
existência do logaritmo.
Vamos ver mais um exemplo e ver como isso fica bem claro.
log2 ( x + 1) + log2 ( x − 1) =
3
Antes de mais nada temos que satisfazer os dois logs e assim

Tivemos mais uma vez oportunidade de reforçar que quando x + 1 > 0 x > −1
 ⇒ ⇒ x >1
estamos falando de funções, pelo fato das transformações que podem x − 1 > 0 x >1
ocorrer, é difícil que tenhamos verdades imutáveis e conseguimos Primeiramente vemos que temos a soma de 2 logs de mesma
revisar pontos importantes sobre as transformações que as funções base e devemos lembrar que log= logc a + logc b e assim podemos
c ab
podem sofrer.

124

PM_BOOK16 - MAT.indb 124 25/11/2022 19:10:18


FUNÇÃO LOGARÍTMICA

também voltar a soma dos logs em um único log. INEQUAÇÕES LOGARÍTMICAS


log2 ( x + 1) + log2 ( x −=
1) log2 ( x + 1)( x − 1) As inequações logarítmicas são totalmente ligadas ao fato de
log2 ( x + 1)( x − 1) =
3 termos uma função crescente (base da potência maior que 1) ou uma
função decrescente (base da potência maior que 0 e menor que 1)
( x + 1)( x − 1) = 23 ⇒ x 2 − 1 = 8 ⇒ x 2 = 9 ⇒ x = ±3 assim como as inequações exponenciais.
Porém como fizemos de antemão a condição de existência Vamos ver da mesma maneira um exemplo prático.
conjunta para ambos os logaritmos sabemos que só podemos ter Digamos que trabalhamos todas as propriedades necessárias e
soluções tais que x > 1 e assim –3 não pode ser uma solução. chegamos a log3 x ≥ 3.
Sendo assim S = {3}. Primeiramente vamos a condição de existência de log3 x.
Quer ver o que aconteceria se x = –3? log3 x ⇒ x > 0
log2 ( −3 + 1) + log2 ( −3 −=
1) log2 ( −2) + log2 ( −4 ) Vamos pensar no caso que já conhecemos, vamos descobrir
Que não é definido. quando há a igualdade.
Já imaginou ficar tendo que testar todas as soluções que encontrar log3 x =3 ⇒ 33 =x
uma a uma? Por isso é sempre melhor determinarmos a condição de x = 27
existência previamente.
Então como já sabíamos a igualdade se dará para x = 27 mas
ProBizu o que nos foi perguntado foi “quais os valores de x para que log3 x
seja maior que 3” e assim podemos pensar o seguinte, a igualdade
A utilização dos logaritmos em equações muitas vezes se dá a partir acontece para x = 27, sim, isso nós vimos. Agora vamos pensar o
de uma equação exponencial que não é possível igualar as bases. seguinte: x é o resultado de uma potenciação de base de potência 3
Geralmente essas questões trazem dados de aproximação para onde o logarítmo (resultado) é o expoente.
utilizarmos aliadas as propriedades aqui explicitadas, por exemplo. Assim como nossa base é 3 quanto maior o nosso expoente maior
Imagine que a partir de um problema chegaremos na seguinte será o resultado e assim quanto maior for o logarítmo maior será x.
equação exponencial. log3 81 > log3 50 > log3 30 > log3 28 > log3 27 > 3
5x = 360
Ou seja, basta termos x ≥ 27 para que a log3 x ≥ 3.
Logicamente 5 e 360 não poderão ser colocados na mesma base,
nesse tipo de questão, geralmente a questão nos traz informações. x>0
Fazendo a intersecção  ⇒ x ≥ 27
Digamos que ela nos fornece os seguintes dados, use log2 = 0,3 x ≥ 27
e log3 = 0,48.
A primeira coisa que temos que ter em mente é que podemos Se fizermos a análise para uma base que está entre 0 e 1 teremos
aplicar logaritmo nos dois lados de uma equação, desde que na o pensamento inverso em relação ao logarítmo e x, assim como nas
mesma base. inequações exponenciais.
Teremos que saber em que base devemos aplicar esses logaritmos. Sendo assim teremos também 2 casos
O ideal seria aplicar a base 5, pois aplicando como base do Se a > 1, ( função crescente ) , então
log a mesma base da potência que temos a incógnita temos nossa 
 loga x ≥ k ⇒ x ≥ ak > 0
intenção se dando de forma mais direta. 
1º caso:  loga x > k ⇒ x > ak > 0 (mantém a desigualdade)
=5 (5 ) log5 ( 360 ) ⇒ x ⋅ log
= 5 (5) log5 ( 360
= ) ⇒ x log5 ( 360 ) 
x
log loga x ≤ k ⇒ 0 < x ≤ ak

Porém a suposta questão só nos forneceu dados de logaritmos na  loga x < k ⇒ 0 < x < ak
base 10, podemos até seguir por esse caminho, mas teríamos que
passar todas as informações fornecidas pela questão da base 10 Se 0 < a < 1, ( função decrescente ) , então
para a base 5. 
 loga x ≥ k ⇒ 0 < x ≤ ak
Sendo assim, a maior parte das vezes aplicamos como base do log 
2º caso:  loga x > k ⇒ 0 < x < ak (inverte
a mesma base dos dados que nos foram fornecidos no problema, a desigualdade) 
loga x ≤ k ⇒ x ≥ ak > 0
sendo assim aplicaremos os logs na base 10. 
360 ⇒ log (5x =  loga x < k ⇒ x > ak > 0
5=
x
) log ( 360) ⇒ x ⋅ log (5=) log ( 360)
Vamos ao que interessa! Isso tudo sem esquecermos da condição de existência de um
logaritmo.
 10 
x ⋅ log  =  log ( 9 ⋅ 4 ⋅ 10 ) ⇒ x ⋅ (log10 − log2=
) log9 + log4 + log10 ⇒
 2
Vamos ver mais um exemplo.
x ⋅ (log10 − log2) =
log32 + log22 + log10 =
2 ⋅ log3 + 2 ⋅ log2 + log10 ⇒
log2 ( 2x − 1) < log2 6
Vamos agora substituir os valores que nos foram fornecidos,
log2 = 0,3, log3 = 0,48 e não esqueçamos que log10 = 1. Como a base do logaritmo é maior que 1, devemos manter a
desigualdade
x ⋅ (1 − 0,3) = 2 ⋅ 0,48 + 2 ⋅ 0,3 + 1 ⇒ 0,7x = 0,96 + 0,6 + 1 ⇒
log2 ( 2x − 1) < log2 6 ⇔ 2x − 1 < 6 e 2x − 1 > 0
2,56
=x ≅ 3,65 0 < 2x − 1 < 6
0,7
1 < 2x < 7
1 7
<x<
2 2
 1 7
S = x ∈  | < x < 
 2 2

125

PM_BOOK16 - MAT.indb 125 25/11/2022 19:10:20


FUNÇÃO LOGARÍTMICA

ProBizu LOGARITMO NATURAL


Repare que as próprias propriedades dos logarítmos nos ajudam. (LOGARITMO NEPERIANO)
1 Os logaritmos neperianos ou logaritmos naturais são os que têm
log 1 x ≥ 3 ⇒ log3−1 x ≥ 3 ⇒ log3 x ≥ 3 ⇒ − log3 x ≥ 3 ⇒ log3 x ≤ −3 ⇒
3
−1 como base o número irracional e ≅ 2,7182.
1 ln b = log e b
x ≤ 3−3 ⇒ x ≤
27
Da definição temos que ln e = 1.
Podemos ter também inequações onde a substituição será
O número e também pode ser definido como:
necessária primeiro.
n
log23 x − 3log3 x + 2 > 0  1
e  lim 1 
n  n
Mas antes log3 x ⇒ x > 0
Fazendo log3 x = m teremos
m² – 3m + > 0 FUNÇÃO LOGARÍTMICA × FUNÇÃO
As raízes por soma e produto temos S = 3 e P = 2 onde m = 1 ou m = 2. EXPONENCIAL
Assim como temos o a da nossa parábola positivo, teremos. A função logarítmica é a função inversa da função exponencial e
dessa forma quando traçamos uma função f e f-1 num mesmo plano
cartesiano temos a simetria em relação a reta y = x.

m < 1 ou m > 2
Como não queremos m mas sim x temos que voltar em log3 x .
log3 x < 1 ⇒ x < 31 ⇒ x < 3
Mas como temos também que x > 0 teremos 0 < x < 3
log3 x > 2 ⇒ x > 32 ⇒ x > 9
S = {x ∈ |0 < x < 3 ou x > 9}
 

Gostou né? Mais um exemplo então.


log2 ( x 2 + x − 2) ≤ 2

Da condição de existência temos que x² + x – 2 > 0 e nossas raízes serão


S = – 1 e P = – 2 onde x = – 2 ou x = 1

x < –2 ou x > 1

log2 ( x 2 + x − 2) ≤ 2 ⇒ x 2 + x − 2 ≤ 22 ⇒ x 2 + x − 2 ≤ 4 ⇒ x 2 + x − 6 ≤ 0

Encontrando as raízes por soma e produto S = –1 e P = –6 teremos


x = –3 ou x = 2.

x ≤ −3 ou x ≥ 2
x < −2 ou x > 1
Agora precisamos fazer a intersecção 
x ≤ −3 ou x ≥ 2

S= {x ∈
 | −3 ≤ x < −2 ou 1 < x ≤ −2}

126

PM_BOOK16 - MAT.indb 126 25/11/2022 19:10:21


FUNÇÃO LOGARÍTMICA

ENCONTRANDO A FUNÇÃO INVERSA DE UMA


FUNÇÃO EXPONENCIAL
Quando temos uma exponencial simples como por exemplo
f ( x ) = 5x é bem simples encontrar f −1 ( x ) da maneira que conhecemos
que é trocar x por y e trocar y por x.
y = 5x ⇒ x = 5y ⇒ aplicando log na base 5 em ambos os lados da
igualdade teremos
log5 =
5y log5 x ⇒ y ⋅ log=
55 log5 x ⇒=y log5 x
Mas e nos casos mais complexos?
f ( x ) =9x − 3x +1 − 4
Vamos lá como sabemos
y = 9x − 3x +1 − 4 ⇒ x = 9y − 3y +1 − 4 ⇒

=
x (3 )
2 y
− 3y ⋅ 3 − 4 ⇒ =
x 32y − 3 ⋅ 3y − 4 ⇒ =
x (3 )
y 2
− 3 ⋅ 3y − 4 ⇒

3y = m
x =m2 − 3m − 4 ⇒ m2 − 3m − 4 − x =0 ⇒ m2 − 3m − ( x + 4 ) =0 ⇒

− ( −3) ± ( −3) − 4 ⋅ 1⋅ ( − ( x + 4 ) ) 3 ± 9 + 4x + 16 3 ± 4x + 25
2

=m = = ⇒
2 ⋅1 2 2

+ 4 )) 3 ± 9 + 4x + 16 3 ± 4x + 25 3 ± 4x + 25 Logicamente não precisamos ter o conhecimento para esboço


= ⇒ m= mas como 3y = m temos
2 2 2 desses gráficos, para isso precisaríamos do conhecimento de cálculo
3 ± 4x + 25 3 ± 4x + 25 estudado no nível superior.
=
3y ⇒ log
=33
y
log3 ⇒
2 2
Vamos ver, por curiosidade, o gráfico de f ( x ) =9x − 3x +1 − 4 .
3 ± 4x + 25 3 ± 4x + 25
=
y ⋅ log3 3 log3 = ⇒ f −1 ( x ) log3
2 2
25
1º ponto: 4x + 25 ≥ 0 ⇒ 4x ≥ −25 ⇒ x ≥ − para que
4
4x + 25 exista.
3 + 4x + 25 3 − 4x + 25
2º ponto: f −1 ( x ) = log3 ou f −1 ( x ) = log3
2 2
3 + 4x + 25
Para f −1 ( x ) = log3 não teremos problemas pois
2
3 + 4x + 25 será sempre maior que zero e assim sempre teremos a
condição de existência do logaritmo satisfeita. E assim nossa função
 25
somente terá o domínio Domf−1 =  − , +∞  .

 4 

Para f −1 ( x ) = log3 3 − 4x + 25 temos que ter 4x + 25 < 3 pois


2
assim 3 − 4x + 25 > 0 e assim Do gráfico de f(x) vemos que quando definida de  →  ela não
4x + 25 < 3 ⇒ 4x + 25 < 9 ⇒ x < −4 é bijetora e com esse domínio e contradomínio não admitiria função
inversa. Então precisamos utilizar subconjuntos de seu domínio e
 25 contradomínio para que admita inversa.
Então temos Dom −1 =x ≥ −  25 
 4 ⇒ Domf−1 = − 4 , −4 
f
 x < −4 Quando definimos para a função os seguintes domínio e
contradomínio.
 3   25 
log , +∞  →  − , +∞ 
f :  3 2   4 
x → 9x − 3x +1 − 4

Conseguimos assim determinar uma função inversa, pois a função


f passa a ser bijetora e assim definimos a sua função inversa.

127

PM_BOOK16 - MAT.indb 127 25/11/2022 19:10:23


FUNÇÃO LOGARÍTMICA

05. Dado log5 = P, calcule o valor de log200 em função de P


 25   3 
 − 4 , +∞  → log3 2 , +∞  a) 5P b) 200P c) P–3 d) 3 – P e) 5 – P
f −1 :
3 + 4x + 25
x → log3 06. (ESA) Aumentando-se um número x em 75 unidades, seu
2
logaritmo na base 4 aumenta em 2 unidades. Pode-se afirmar que x
Vamos ver ambas no mesmo plano cartesiano. é um número:
a) Irracional. c) Múltiplo de 3. e) Maior que 4.
b) Divisor de 8. d) Menor que 1.

07. (ESA) Se log2 3 = a e, log2 5 = b então o valor de log0,5 75 é


a) α+β c) α–β e) –α – 2β
b) –α + 2β d) α – 2β

1
08. (ESA) Sabendo que log P = 3 ⋅ log a − 4 ⋅ log b + ⋅ log c, assinale a
2
alternativa que representa o valor de P. (dados: a = 4, b = 2 e c = 16)
a) 12 b) 52 c) 16 d) 24 e) 73

09. (ESA) O logaritmo de um produto de dois fatores é igual à soma


dos logaritmos de cada fator, mantendo-se a mesma base. Identifique
a alternativa que representa a propriedade do logaritmo anunciada.
a) logb(a · c) = logb a + logb c
b) logb(a · c) = logb (a + c)
c) logb (a + c) = logb a · logb c
d) logb (a + c) = logb (a·c)
e) loge (a·c) = logb a + logf c

10. (ESA) Dados log 3 = a e log 2 = b a solução de 4x = 30.


Também poderíamos utilizar outros subconjuntos para a função f.
(2a + 1) (2b + 1) (b + 2)
 3  25  a) c) e)
−∞, log3  →  − , +∞  b a a
f :  2  4 
(a + 2) (a + 1)
x → 9x − 3x +1 − 4 b) d)
b 2b
E dessa forma, para essa definição de função f temos a seguinte
função inversa.
EXERCÍCIOS DE

TREINAMENTO
 25   3
 − 4 , +∞  →  −∞, log3 2 
f −1 :
3 − 4x + 25
x → log3
2
01. (EEAR) Sejam m, n e b números reais positivos, com b ≠ 1. Se logb
n
m = x e se logb n = y, então logb (m ⋅ n) + logb   é igual a
EXERCÍCIOS DE m

FIXAÇÃO a) x b) 2y c) x + y d) 2x – y

02. (EEAR) Se M = log2 32 + log1 3 3 − log 2 8 , então M vale


a) –1 b) 1 c) –2 d) 2
01. Calcule o valor dos seguintes logaritmos:
a) log16 64 c) log5 0,000064 03. (EEAR) O gráfico abaixo representa a função y = loga x. Dentro das
b) log625 5 d) log49 7 3 condições de existência para que a operação de logarítmo seja sempre
possível e de resultado único, a base “a” é

02. O número real x, tal que logx   = , é


9 1 a) 0 < a < 1
4 2 b) a = 0
a) 81 b) −
3 c) 1 d)
3
e) −
81
c) a>1
16 2 2 2 16
a
d) a < 0
03. (PUCRS) Escrever blogb = b-2, equivale a escrever
a) a = 1 c) a = b² 1
e) b =
b² d) b² = -a a²
b) b = a² 04. (EEAR) Um número, seu logaritmo 2 e a base do logarítimo
formam, nessa ordem, uma P.A. Esse número é
04. (UCS) Se log2 = a e log3 = b, então log12 vale 9 − 17 9 + 17 −1 + 17 −1 − 17
a a) b) c) d)
a) a + b b) 2a + b c) a + 2b d) a · b e) 2 2 2 2
b

128

PM_BOOK16 - MAT.indb 128 25/11/2022 19:10:24


FUNÇÃO LOGARÍTMICA

05. (EEAR) A curva da figura representa o gráfico da função y = loga x, 12. (ESPCEX) A curva do gráfico abaixo representa a função y = log4 x.
(a > 1). Dos pontos B(3,0) e C(9,0) saem perpendiculares ao eixo das
abscissas, as quais interceptam a curva em D e E, respectivamente. Se
a área do trapézio retângulo BCED vale 9, a área do triângulo ABD,
onde A(1,0) vale
E
1. y
a)
2
D y = loga x
b) 2.

c) 3.
2
d) 1. O A B C x A área do retângulo ABCD é
3
a) 12. b) 6. c) 3. d) 6log4 . e) log4 6.
06. (ESPCEX) Considerando logm 10 = 1,4 e logm 50 = 2,4, pode-se 2
afirmar, com base nesses dados, que o valor do logarítmo decimal de 5 é:
13. (ESPCEX) A equação log3 x = 1 + 12logx2 3 tem duas raízes reais.
a) 3/7 b) 1/2 c) 5/7 d) 7/3 e) 7/5 O produto dessas raízes é

07. (ESPCEX) Há números reais para os quais o quadrado de seu a) 0. b) c) d) 3. e) 9.


logaritmo decimal é igual ao logaritmo decimal de seu quadrado. A
soma dos números que satisfazem essa igualdade é
14. (AFA) O domínio mais amplo da função real f definida por
a) 90 b) 99 c) 100 d) 101 e) 201
=f(x) loga (x 2 − 3),
em que a ∈ ]0,1[, é
08. (ESPCEX) Acrescentando 48 unidades a um número, seu logaritmo a) [–2,2] c) ]–∞,–2 ∪ [2,+∞[
na base 5 aumenta de 2 unidades. Esse número é b) ]–2,2[ d) [−2, − 3[∪] 3, 2]
a) 1 b) 2 c) 3 d) 6 e) 12
15. (AFA) O domínio da função definida por f(x) = log (x³ – 3x² + 2x)
09. (ESPCEX) Considerando o gráfico abaixo, onde: é o conjunto:
a) ]0,1[ ∪ ]2,∞[ c) ]2,∞[
b) ]–∞,0[ ∪ ]1,2[ d) ]–∞,1[

8
16. (AFA) O valor máximo da expressão log24 x + 12 ⋅ log22 x ⋅ log2  
para 1 ≤ x ≤ 64 é: x
a) 1 c) 9
b) 3 d) 81

I - A curva é a representação da função y = log x, para x ≥ 1. a− y


17. (AFA) Seja = x, com a ∈ , a > 0 e a ≠ 1. Determinando-se
1 + a− y
II - Os retângulos sombreados têm um dos vértices sobre a curva. y em função de x, o domínio da função assim definida é
Nas condições apresentadas acima, a área da região sombreada é: a) {x ∈  l x ≥ 0}. c) {x ∈  l x < 1}.
a) log 24 c) log 12 e) log 6 b) {x ∈  l x ≥ 1}. d) {x ∈  l 0 < x < 1}.
b) log 18 d) log 9
18. (AFA) Se log10 ≤ (log2 4 · log4 6 · log6 8) – 1, então
10. (ESPCEX) A figura abaixo fornece a representação gráfica da a) 0 < x ≤ 102 c) 104 < x ≤ 106
função y = logb x.
b) 10 < x ≤ 10
2 4
d) 106 < x ≤ 108

8
19. (AFA) O valor máximo da expressão log24 x + 12 ⋅ log22 x ⋅ log2  
para 1 ≤ x ≤ 64 é:  x

a) 1 b) 3 c) 9 d) 81

20. (ESPCEX) Uma epidemia ocorre quando uma doença se desenvolve


num local, de forma rápida, fazendo várias vítimas num curto intervalo
de tempo. Segundo uma pesquisa, após t meses da constatação da
Nestas condições, o valor de b é existência de uma epidemia, o número de pessoas por ela atingida é
a) 1/4 c) 3 e) 10 20000
N(t) = .
b) 2 d) 4 2 + 15 ⋅ 4 −2t
Considerando que o mês tenha 30 dias, log 2 ≅ 0,30 e log 3 ≅ 0,48,
2000 pessoas serão atingidas por essa epidemia, aproximadamente, em
1− x 
11. (ESPCEX) A função f(x) = log   tem por domínio a) 7 dias. d) 7 meses.
 x + 2
a) ]–2,1[ c)  – {–2,1} e)  b) 19 dias. e) 1 ano.

b)  – {–2} d) ]–∞,–2[ ∪ [1,+∞[ c) 3 meses.

129

PM_BOOK16 - MAT.indb 129 25/11/2022 19:10:25


FUNÇÃO LOGARÍTMICA

21. (AFA) A soma de todos os valores reais que satisfazem a equação 30. (AFA) Pesquisas realizadas verificaram que, no planeta Terra, no
xlog4 x = 16x, x > 0, início do ano de 2013, a população de pássaros da espécie A era 12
17 33 65 129 vezes a população de pássaros da espécie B. Sabe-se que a população
a) b) c) d) de pássaros da espécie A cresce a uma taxa de 5% ao ano, enquanto
4 4 4 4
que a população de pássaros da espécie B cresce a uma taxa de 20%
ao ano. Com base nesses dados, é correto afirmar que, essas duas
22. (AFA) Se x1 e x2 são as raízes da equação: logbx b + 2logx b + 3logb²x populações de pássaros serão iguais
b = 0, b ∈ *+, b ≠ 1, então (x1 · x2)6 vale:
(Considere: log7 = 0,85; log6 = 0,78; log2 = 0,3)
a) b11 b) b–11 c) b6 d) b–6
a) no 1º semestre do ano de 2034.
23. (EPCAR 3° ANO) Num sistema de logaritmos, o logaritmo de b) no 2º semestre do ano de 2034.
101,44 supera em 5 o logaritmo de 3,17. É correto afirmar que se c) no 1º semestre do ano de 2035.
trata do sistema de logaritmos d) no 2º semestre do ano de 2035.
a) de base menor que 3 c) decimais
b) de base igual a 5 d) neperianos 31. (AFA) No plano cartesiano, seja P(a,b) o ponto de interseção entre
x
as curvas dadas pelas funções reais f e g definidas por f ( x ) =   e
1
a  2
24. (ESPCEX) Fazendo x = n5 temos que y =ex − e− x = , a ∈  e ( )
g x = log x.1
b ∈ *, a e b primos entre si. Logo a + b é igual a b 2

a) 28 b) 29 c) 40 d) 51 É correto afirmar que


 1    1
a) a = log2  c) a = log 1  log 1   
25. (ESPCEX) O número N de bactérias de uma cultura é dado  1   a 
 log2    2
em função do tempo t (em minutos), pela fórmula N(t) = (2,5)1,2t. 2
 a
Considere log10 2 = 0,3, o tempo (em minutos) necessário para que a
cultura tenha 1084 bactérias é
b) a = log2 (log2 a) (
d) a = log2 log 1a
2
)
a) 120 d) 185 32. (MACK)
b) 150 e) 205
c) 175

26. (AFA) Considere os números A, B e C a seguir.


A= log25 27 ⋅ log4 5 ⋅ log3 2

B = logn (logn n n
n ) (n é natural maior que 2)
log c log a log b
a b c
C=   ⋅  ⋅  {a, b, c} ⊂ *+
b c a

A correta relação de ordem entre os números A, B e C é Se na figura temos os esboços dos gráficos das funções f(x) = log2x e
a) A < B < C c) B<C<A   1
g(x) = ax² + bx + c, então g  f    é igual a
b) B < A < C d) C < A < B   8 
a) 14 d) 17
27. (ESPCEX) Resolvendo a equação a seguir, obtém-se b) 15 e) 18
log3 (x 2 − 2x − 3) + log 1 (x −=
1) log3 (x + 1), c) 16
3

a) S = {–1}. c) S = {6}. e) S = {4}. 33. (EPCAR 3° ANO) Se A = log(1 + cotg² x) + log (1 + cos x) + log(1 – cos x),
π
b) S = {4,5}. d) S = {∅}. sendo 0 < x < , então A é igual a
2
1 1
a) log b) log c) log 1 d) log 10
28. (EFOMM) Um atleta de tiro ao prato tem probabilidade de 0,9 10 2
de acertar o prato a cada novo lançamento. Analisando esse jogador
antes do início da competição, após quantos lançamento de pratos, a 34. (EPCAR 3° ANO) Analise as proposições classificando-as em V
probabilidade de ele não ter acertado todos os tiros se tornará maior (verdadeira) ou F (falsa).
que a probabilidade de acertar todos? 2
 1 
a) 9 b) 8 c) 7 d) 6 e) 5 I. (
O produto das raízes da equação 2 1 + logx2 10 =

 log x 
)
−1 
é
1
igual a .
29. (EPCAR 3° ANO) Um boato alastra-se com determinada rapidez 10
entre os habitantes de uma metrópole. Após x horas (x > 0), o número II. Se 5 = 2, então log2 100 = 2(1 + n-1).
n

n III. Observando o gráfico de f(x) = logn x abaixo pode-se afirmar que


de pessoas que já sabiam do boato é dado por f(x) =  n 
, onde
− x o valor de f(256) = n.
1 + 2e  2 
e é a base do sistema de logaritmos neperianos e n o número de
habitantes da metrópole (em milhões). Sabendo-se que após 2 horas
do início da propagação do boato, 80% da população já estava ciente
do caso e considerando ln 2 = 0,69 pode-se dizer que o número de
habitantes da metrópole pertence ao intervalo
a) [1, 2] b) ]2, 3] c) ]3, 4] d) ]4, 5]

130

PM_BOOK16 - MAT.indb 130 25/11/2022 19:10:26


FUNÇÃO LOGARÍTMICA

 
A sequência correta é 
02. O conjunto verdade da desigualdade log2  log 1 x 2  2x  1
    0
a) V – V – V b) V – F – F c) F–V–V d) F – V – F é:  4 
a) (0,1/2) ∪ (3/2,2)
ex + 4 b) (–2,0) ∪ (3/2,2)
35. (EPCAR 3° ANO) Se o domínio de f(x) = é
e2 − x 2 c) (1/2,3/2)
Obs.: e = 2,7182...
d) (–∞,1/2) ∪ (3/2,∞)
a) {x ∈  / x < –e ou x > e} c) {x ∈  / x < –1 ou x > 1}
e) ∅
b) {x ∈  / –e < x < e} d) {x ∈  / –2 < x < 2}
03. Acrescentando 16 unidades a um número, seu logaritmo na base
36. (EPCAR 3° ANO) Uma população de bactérias cresce conforme
3 aumenta de 2 unidades. Esse número é:
a função definida por N(t) = N0 · eαt, onde t é o tempo (dado em
horas), N0 é o número inicial de bactérias e α é uma constante a) 5 d) 4
positiva. Sabendo-se que o número de bactérias triplica ao final das b) 8 e) 3
três primeiras horas, pode-se dizer que α pertence ao intervalo c) 2
Dados: adote e = 2,7 (e é a base do sistema de logaritmos neperianos)
log 3 = 0,48 04. O valor de y ∈  que satisfaz a igualdade logy 49 = logy² 7 + log2y 7, é:
a) [0,3;0,4] b) ]0,4;0,5] c) ]0,5;0,6] d) ]0,6;0,7] a) 1/2 d) 1/8
b) 1/3 e) 7
37. (EPCAR 3° ANO) Assinale a alternativa correta. c) 3
a) Se x > 0 e a > 1, os gráficos das funções reais f e g, dadas por
x
 1 05. (EFOMM 2010) Sabendo-se que log30 3 = a e log30 5 = b, que
f(x) = loga x e g(x) =   , se interceptam em um ponto cuja
a opção representa log10 2?
abscissa é maior que zero e menor que um.
1 a  b 1− a − b
b) Para qualquer que seja x real, o intervalo ]–4,+∞[ é o conjunto- a) d)
x +1
2a 2−a
 1 1− a − b 1− a − b
imagem da função real h, definida por= h(x)   − 4 . b) e)
 2 a −1 1− a
c) Se log9 (log3(logx 3)) = 0, então x é um número racional.
1 a  b
d) Se f é uma função real definida por f(x) = 3 + 2-x, então c)
a 1
1
f (log4 9 ) = .
3 06. (ESPCEX) Considerando log 2 = 0,30 e log 3 = 0,48, o número real
x, solução da equação 5x-1 = 150, pertence ao intervalo:
10− y + 1
38. (EPCAR 3° ANO) Seja a relação − y = x . Ao determinar y em a) ]-∞,0] d) [0,2[
10 + 2
função de x, obtém-se uma função cujo o domínio é b) [4,5[ e) [5,+∞[
a)  *
 1 c) ]1,3[
c)  −∞, 2   ]1, ∞[
+

1 
b)  2 ,1 d)  − {1} 07. (ESPCEX) Na figura abaixo, dois vértices do trapézio sombreado
estão no eixo x e os outros dois vértices estão sobre o gráfico da
função real f(x) = logk x, com k > 0 e k = 1. Sabe-se que o trapézio
39. (EPCAR 3° ANO) O domínio da função real definida por
sombreado tem 30 unidades de área; assim, o valor de k + p – q é
f(x) = log1 loga (loga x )  é
a

a) a < x ≤ aa se 0 < a < 1. c) a < x ≤ aa se a > 1.


b) 0 < x < 1 e x ≥ aa se a > 1. d) x ≥ aa se 0 < a < 1.

40. (EN) Qual é o domínio da função real de variável real, definida por
= n(x 2 − 3x + 2) + e2x −1 − 1 ?
f(x)

a) [1,2[ 1 
d)  2 ,1 ∪ ]2, +∞[
b)  ,2 ∪ ]3, +∞[
1
2  1 
c) ]2,+∞[
e)  2 , +∞ 

a) –20 d) 15

EXERCÍCIOS DE
b) –15 e) 20

COMBATE
c) 10

6 − loga m
08. (ESPCEX) Se = 2, com a > 0, a ≠ 1 e m > 0, então o
m
1+ loga² m
01. O valor da expressão log3 2 · log4 3 · ... · log10 9 é: valor de é
a+ m
a) 0 c) log4 3 e) 1 a) 4 c) 1 e) 1/2
b) log10 2 d) log3 4 b) 1/4 d) 2

131

PM_BOOK16 - MAT.indb 131 25/11/2022 19:10:31


FUNÇÃO LOGARÍTMICA

09. (ESPCEX) Na figura abaixo, está representado o gráfico da função


y = log x. Nesta representação estão destacados três retângulos cuja
soma das áreas é igual a:
4 (IME) Seja a equação ylog3 3y = ylog3 3y − 6, y > 0.
O produto das raízes reais desta equação é igual a:
1
a)
3
3
b)
4
3
c)
4
d) 2
e) 3

5 (IME) Sejam a, b, c e d números reais positivos diferentes de


1. Temos que loga d, logb d e logc d são termos consecutivos
de uma progressão geométrica e que a, b e c formam uma
progressão aritmética em que a < b < c.
a) log2 + log3 + log5 d) 1 + 2log15 Sabendo-se que b = bloga b – a, determine:
b) log30 e) 1 + 2log30 a) Os valores de a, b e c;
c) 1 + log30 b) As razões das progressões aritmética e geométrica, r e q,
respectivamente.
1 log10 3
10. (ESPCEX) Seja β= ⋅ . O conjunto solução da
2 log10 3 − log10 7
β
 3 GABARITO
desigualdade 3cos x ≤   no intervalo [0,2π(, é igual a
7
EXERCÍCIOS DE FIXAÇÃO
 π π   3π  01. a) 3/2 02. A 06. E 10. D
a) 0, 3  c)  3 ,2π  e)  2 ,2π 
b) 1/8 03. A 07. E
 π 5π  π  c) –6 04. B 08. C
b)  3 , 3  d)  3 ,2π 
d) 1/6 05. D 09. A
EXERCÍCIOS DE TREINAMENTO
01. B 11. A 21. C 31. A

DESAFIO PRO 02. C


03. A
12. B
13. D
22. D
23. A
32. C
33. B
04. A 14. D 24. B 34. A
05. C 15. A 25. C 35. B

1 (ITA) Seja f a função definida por f(x) = logx+1 (x² – 2x – 8).


Determine:
06. C
07. D
16. A
17. D
26. B
27. D
36. A
37. B
a) O domínio Df da função f.
08. B 18. A 28. C 38. B
b) O conjunto de todos os valores de x ∈ Df tais que f(x) = 2.
09. A 19. A 29. B 39. C
c) O conjunto de todos os valores de x ∈ Df tais que f(x) > 1.
10. D 20. A 30. B 40. D
EXERCÍCIOS DE COMBATE
2 (ITA) Sejam a, b, c, d números reais positivos e diferentes
de 1. Das afirmações: 01. B
02. A
04. D
05. E
07. B
08. E
10. B
I. a (logc b)
=b .
(logc a)

logd c logd a logd b 03. C 06. B 09. D


II. a b c
      = 1. DESAFIO PRO
b c a
III. logab (bc) = loga c 01. a) D = ]4,+∞[.
é (são) verdadeira(s) b) S = φ.
a) apenas I. c) apenas I e II. e) todas. 3 + 3 5 
=c) S  , +∞ 
b) apenas II. d) apenas II e III.  2 
02. C

3 (ITA) Se log2 π = a e log5 π = b, então

1 1 1 3 1 1
03. E
04. A
log 3 2 log 3 2 log 3 2
a) + ≤ . d) < + ≤ 2.
a b 2 2 a b 05. a) a = 2 4
,b=
2⋅2 4 e c = 3⋅2 4

1 1 1 1 1 log 3 2
b) < + ≤ 1. e) 2 < + . =b) r 2=
4
e q log3 2.
2 a b a b 2
1 1 3
c) 1 < + ≤ .
a b 2

132

PM_BOOK16 - MAT.indb 132 25/11/2022 19:10:32


RESOLUÇÃO DE INEQUAÇÕES

PRINCIPAIS PROPRIEDADES DAS


DESIGUALDADES EM 
1 1
 a > b se ab > 0
1. a < b ⇔ a + c < b + c e a < b ⇔ 
 1 < 1 se ab < 0
 a b
ac < bc se c > 0
2. a < b ⇔ 
ac > bc se c < 0
3. 0 < a < 1 ⇒ am < a e a > 1 ⇒ am > a
4. a < b ⇔ a < b, ∀a,b ∈ *+
5. 0 < a < 1 ⇔ a < a < 1 e a > 1 ⇔ a < a
a < a'
b < b'
6.  ⇒ a + b +  + l < a'+ b'+  + l'
 
l < l' 

INEQUAÇÕES SIMULTÂNEAS
A dupla desigualdade f(x) < g(x) , h(x), se decompõem em 2
inequações simultâneas
 f(x) < g(x)

f(x) < g(x) < h(x) ⇔  e
g(x) < h(x)

Exemplo:
 II 
3x
+ 2
< − x +3 ≤ x + 4 e 3x + 2 < − x + 3 ≤ x + 4
I

Temos que resolver as 2 inequações


1) 3x + 2 < –x + 3 ⇒ 4x < 1 ⇒ x < 1/4
2) –x + 3 ≤ x + 4 ⇒ –2x ≤ 1 ⇒ x ≥ –1/2
A interseção desses dois conjuntos é

 1 1
S = x ∈ R | − ≤ x ≤ 
 2 4

SINAIS DAS PRINCIPAIS FUNÇÕES

133

PM_BOOK16 - MAT.indb 133 25/11/2022 19:10:33


RESOLUÇÃO DE INEQUAÇÕES

f(x).g(x) = 0

f(x).g(x) ≥ 0 ⇔  ou
f(x).g(x) > 0

Exemplo: Resolver a inequação (3x + 1)·(2x – 5) ≥ 0 em .
Fazemos o estudo de f(x) = (3x + 1) e g(x) = (2x – 5).
Sinal de f(x) = 3x + 1
1
Zero da função: f(x) = 0 ⇒ 3x + 1 = 0 ⇒ x = −
3

Sinal de g(x) = 2x - 5
5
Zero da função: g(x) = 0 ⇒ 2x – 5 = 0 ⇒ x =
2
INEQUAÇÕES PRODUTO
Sendo f(x) e g(x) duas funções na variável x, as inequações
f(x) · g(x) > 0, f(x) · g(x) < 0, f(x)·g(x) ≥ 0 e f(x)·g(x) ≤ 0 são denominadas
inequações produto.
Exemplo: (x + 2)·(2x – 1) > 0. Um processo prático é fazer o
estudo de cada função separadamente.
Fazemos o estudo de f(x) = (x + 2) e g(x) = (2x + 1).
Sinal de f(x) = x + 2
Zero da função: f(x) = 0 ⇒ x + 2 = 0 ⇒ x = –2 Fazendo o quadro de sinais teremos

Sinal de g(x) = 2x + 1  1 5
S= x ∈ R | x ≤ − ou x ≥ 
Zero da função: g(x) = 0 ⇒ 2x + 1 = 0 ⇒ x = –½  3 2

Dentre as inequações produto são importantes as inequações do


tipo (f(x))n > 0, (f(x))n < 0, (f(x))n ≥ 0 e (f(x))n ≤ 0. Para resolver essas
inequações vamos relembrar as regras de potências
1) toda potência de base real e expoente ímpar conserva o sinal
da base
a2n+1 > 0 ⇔ a > 0
a2n+1 = 0 ⇔ a = 0
a2n+1 < 0 ⇔ a < 0 (n ∈ )
Agora usamos o auxílio do quadro abaixo onde usamos a regra
dos sinais para a multiplicação. 2) toda potência de base real e expoente positivo resulta num
número não negativo
a2n ≥ 0, ∀a ∈ , ∀n ∈ 
Assim sendo temos
f(x) > 0 se n é ímpar
[(f(x))]
n
>0⇔
 f(x) ≠ 0 se n é par

f(x) < 0 se n é ímpar


[(f(x))]
n
<0⇔
 ∃/ x ∈  se n é par
S = {x ∈  | x < –2 ou x > ½}

A inequação f(x)·g(x) ≥ 0 tem como soluçao a união das soluções


de f(x) · g(x) > 0 com as soluções de f(x) · g(x) = 0.

134

PM_BOOK16 - MAT.indb 134 25/11/2022 19:10:34


RESOLUÇÃO DE INEQUAÇÕES

f(x) ≥ 0 se n é ímpar INEQUAÇÕES DE GRAU SUPERIOR AO


[(f(x))]
n
≥0⇔
∀ x ∈ D(f) se n é par SEGUNDO E INEQUAÇÕES RACIONAIS
( x − a1 ) ( x − a2 )  ( x − ak )
n1 n2 nk
f(x) ≤ 0 se n é ímpar Consideremos a função f ( x ) =
[(f(x))] ≤ 0 ⇔  f(x) = 0 se n é par
n
( x − b1 ) ( x − b2 )  ( x − bp )
m
1 m
2
m p


onde n1, n2, ..., nk e m1, m2, ..., mp são números naturais e os números
Exemplos: a1, a2, ..., ak e b1, b2, ..., bp são números reais quaisquer tais que
ai ≠ bj onde i = 1,2,...,k e j = 1,2,...,p. Uma inequação da forma f(x) > 0
1º) ( 3x − 2)3 > 0 ⇒ 3x − 2 > 0 ⇒ S = x ∈ R | x > 2
3 é chamada uma inequação racional. Nos pontos x = ai a função f(x)

se anula(estes pontos são chamados zeros da função). Os pontos
2º) ( 4x − 3)6 > 0 ⇒ 4x − 3 ≠ 0 ⇒ S = x ∈ R | x ≠ 3  x = bj são os pontos de descontinuidade da função f(x). Marcando-
 4
se todos os zeros e pontos de descontinuidade da função em uma
3º) ( 2x + 1) < 0 ⇒ 2x + 1 < 0 ⇒ S = x ∈ R | x < − 

5 1
reta numérica, dividimos esta reta em k + p + 1 intervalos e para
 2 determinar o sinal de f(x) basta determinar o sinal de f(x) em um ponto
4º) (x – 2)4 < 0 ⇒ S = ∅ pertencente a cada um dos intervalos. O método geral de resolução de
uma inequação da forma f(x) > 0 (ou <, ≥, ≤) é baseado no seguinte:
INEQUAÇÕES QUOCIENTE I. Se c é o maior dos números ai, bj então a função f(x) é positiva
As regras para se resolver as inequações quociente são as mesmas no intervalo (c, ∞).
para as inequações produtos, apenas devemos atentar para as II. Se ai(ou bj) é um ponto tal que o expoente ni da função (x – ai)n i
restrições de não se poder deixar zerar o denominador da fração pois ou (x – bj)m j é um número ímpar então, à direita e à esquerda
nao existe divisão por zero. de ai(ou bj) isto é, em intervalos adjacentes, a função possui
sinais desiguais e o ponto ai(ou bj) é chamado ponto simples.
Exemplo: Resolver em R a inequação 3x + 4 ≤ 2, temos Isto significa que quando a função f(x) passa por um ponto
1− x
simples ela muda de sinal.
3x + 4 3x + 4 3x + 4 − 2(1 − x) 5x + 2 III. Se ai(ou bj) é um ponto tal que o expoente ni da função (x – ai)n i
≤2⇒ −2≤0⇒ ≤0⇒ ≤0
1− x 1− x 1− x 1− x ou (x – bj)m j é um número par então, à direita e à esquerda
 
f( x ) de ai(ou bj) isto é, em intervalos adjacentes, a função possui
5x + 2 sinais iguais e o ponto ai(ou bj) é chamado ponto duplo. Isto
−x
1 significa que quando a função f(x) passa por um ponto duplo
g( x ) ela não muda de sinal.

Sinal de f(x) = 5x + 2 Exemplo:


2
x 2 ( x − 2) ( x + 3)
3
Zero da função: f(x) = 0 ⇒ 5x + 2 = 0 ⇒ x = −
5 Resolver a inequação >0
(x − 4)
7

Resolução:
x 2 ( x − 2) ( x + 3)
3

A função f ( x ) = anula-se nos pontos x1 = 0,


(x − 4)
7

x2 = 2, x3 = –3 e possui um ponto de descontinuidade em x4 = 4.


Estes quatro pontos dividem a reta real numérica em cinco intervalos,
a saber, (–∞ –3),(–3, 0), (0,2), (2,4) e (4,∞). Assim, os pontos x = 2,
x = –3, x = 4 são simples enquanto que x = 0 é um ponto duplo e, os
sinais da função f(x) são mostrados abaixo
Sinal de g(x) = 1 - x
Zero da função: g(x) = 0 ⇒ 1 – x = 0 ⇒ x = 1

Este método, baseado nos três princípios enunciados acima para


a resolução de uma inequação do tipo f(x) > 0, ou f(x) < 0 pode ser
sintetizado no seguinte:
1. Todos os zeros e pontos de descontinuidade da função f(x)
são marcados na reta real numérica, levando em consideração
que se a desigualdade for estrita, os zeros são marcados com
círculos vazios e com círculos cheios em caso contrário.
Fazendo o quadro de sinais 2. Da direita para a para a esquerda, começando acima da
reta numérica uma curva sinuosa que passa pelos pontos
marcados é traçada de modo que quando ela passa por
um ponto simples, a curva intersecta a reta e quando passa
por um ponto duplo, ela permanece no lado em que está
localizada na reta numérica.
3. Os intervalos apropriados são escolhidos de acordo com o
sinal da inequação (a função é positiva quando a curva estiver
acima da reta numérica e é negativa se a curva se encontrar
abaixo da reta numérica) a união dos intervalos representa a
 2  solução da inequação.
S= x ∈ R | x ≤ − ou x > 1
 5 

135

PM_BOOK16 - MAT.indb 135 25/11/2022 19:10:35


RESOLUÇÃO DE INEQUAÇÕES

TEOREMA DE BOLZANO
Se um polinômio P(x) apresenta valores P(a) e P(b) tais que
P(a) ⋅ P(b) < 0,então a equação admite um número ímpar (pelo menos I.
uma) de raízes reais entre a e b.

II.

Assim x < –2 ou x > 4 ou seja, o menor valor inteiro positivo é 5.


Exemplo:
P(x) = x3 − 3x2− x + 3
P(0) = 3 e P(2) = 23− 3 ⋅ 22− 2 + 3 = −3 Exercício Resolvido
Pelo Teorema de Bolzano existe pelo menos uma raiz entre 0 e 2.
02. (CN) A quantidade de soluções inteiras da inequação
1 2 ≥ 1 é:
+
x² − 4 x + 2
a) 0
b) 1
c) 2
d) 3
e) 4

Resolução: B
1 2 1.1 − 2(x − 2)
+ ≥ 1⇒ − 1≥ 0 ⇒
Na verdade, 1 é raiz de P(x). x2 − 4 x+2 x2 − 4
1 (x − 2)
1 − 2x + 4 − (x 2 − 4) −3 + 2x − x 2 + 4
Exercício Resolvido ≥0⇒ ≥0⇒
x2 − 4 x2 − 4
01. (EEAR) O menor valor inteiro positivo que pertence ao −x + 2x + 1
2
≥ 0 ⇒ f(x) =−x 2 + 2x + 1 e g(x) =x 2 − 4
conjunto-solução da inequação (–3x² + 12)(x² – 6x + 8) < 0 é o: x2 − 4
a) 2
Encontrando as raízes
b) 3 f(x) = 0 ⇒ −x 2 + 2x + 1= 0 ⇒ x 2 − 2x − 1= 0 ⇒
c) 4 −( −2) ± ( −2)2 − 4.1.( −1)
=x ⇒
d) 5 2.1
2± 8 2±2 2
x= = = 1± 2
Resolução: D 2 2
Vamos estudar o sinal das 2 funções e posteriormente fazer uso g(x) = 0 ⇒ x − 4 = 0 ⇒ x 2 = 4 ⇒ x = ±2
2

do quadro de sinais.
(
−3x + 12)( x − 6x + 8 ) < 0 ⇒
2


2

 
I II f(x)
Raízes de I ⇒ −3x 2 + 12 = 0 ⇒ x 2 = 4 ⇒ x = ±2
−( −6) ± ( −6)2 − 4.1.8
Raízes de II ⇒ x =
2.1
6 ± 36 − 32 6 ± 2
⇒ = ⇒=x 2 ou= x 4
2 2

136

PM_BOOK16 - MAT.indb 136 25/11/2022 19:10:36


RESOLUÇÃO DE INEQUAÇÕES

1
06. A solução da inequação x > é:
x
a) x > 0
g(x) b) x > 1
c) x < –1 ou x > 1
d) –1 < x < 0 ou x > 1

Quadro de sinais (2x + 2)


07. Considere a função f(x) = . O conjunto dos valores de x
(x − 3)
para os quais f(x) ∈ {y ∈ : 0 < y ≤ 4} é:
a) {x ∈ : x ≥ 7}
b) {x ∈ : x < –1 ou x ≥ 7}
c) {x ∈ : –1 < x ≤ 7}
d) {x ∈ : x < –1}

08. Quantos números inteiros e estritamente positivos satisfazem a


1 1
 2,1 − 2  ∪  2, 1 + 2 
S =− sentença ≤ ?
    (x − 20) (12 − x)
a) Dezesseis.
Assim o único inteiro que faz parte da solução é x = –1.
b) Quinze.
c) Quatorze.

EXERCÍCIOS DE
d) Treze.

FIXAÇÃO
e) Menos que treze.

2x + 6
09. O número de soluções inteiras da inequação ≥ 0 é:
14 − 2x
a) 8
01. (EEAR) O conjunto dos valores reais de x para os quais a expressão
x −1 b) 9
é estritamente positiva é:
| x² − 10x + 21| c) 10
a) {x ∈  / x > 1} d) 11
b) {x ∈  / x > 3 e x ≠ 7} e) infinito
c) {x ∈  / x < 1 ou 3 < x < 7}
(4x − 3)
d) {x ∈  / x > 1, x ≠ 3 e x ≠ 7} 10. O número real x satisfaz > 2. Assinale a alternativa em
(x + 1)
que estão incluídas TODAS as possibilidades para x.
02. (EEAR) A inequação (x² – 5x + 6)(x – 3) ≥ 0 tem para conjunto solução:
a) –1 < x < 5/2
a) {x ∈  / x ≤ 3}
b) x < –1 ou x > 5/2
b) {x ∈  / x ≥ 2}
c) x > 5/2
c) {x ∈  / 2 ≤ x ≤ 3}
d) x < –1
d) {x ∈  / x ≤ 2 ou x ≥ 3}

03. Considerando as funções f(x) = – x2 – 2x + 8 e g(x) = 2 – x, o EXERCÍCIOS DE

TREINAMENTO
f(x)
intervalo solução de ≥ 1 é:
g(x)
a) ]– 3, 2 [ ∪ ] 2, + ∞[
b) [– 3, 2 [ ∪ ] 2, + ∞ [
c) ]– 3, ∞[ x2 + 3
01. O conjunto solução da inequação ≤ 0 é:
d) [– 3, ∞[ 2x − 1
a) R

(x² − 2x − 3) b) {x ∈ R | x < 0}
04. Os valores de x que satisfazem à inequação ≥ 0, c) {x ∈ R | x > 1}
pertencem a: (x − 2)
d) {x ∈ R | x ≥ 1}
a) [- 1, 2) ∪ [3, ∞).
b) (- 1, 2] ∪ (3, ∞). 02. A soma dos quadrados dos números naturais que pertencem ao
c) [1, 3]. (3 − x) ⋅ (x 2 − 1)
conjunto solução de ≥ 0 é igual a:
d) [– 3, 2). x+2
a) 13
e) [– 3, – 2] ∪ (2, ∞). b) 14
c) 15
05. O número de soluções inteiras da inequação (1 – x) (x – 8)2 (x + 4)3 > 0, é:
d) 19
a) 0 c) 4
e) 20
b) 2 d) 6

137

PM_BOOK16 - MAT.indb 137 25/11/2022 19:10:37


RESOLUÇÃO DE INEQUAÇÕES

x +1
03. Considere a inequação ≤ 0, com x ∈ . Qual é o conjunto 10. Sobre a inequação-produto (–4x² +2x – 1)(x² – 6x + 8) ≥ 0, em ,
solução da inequação? − x −5 é correto afirmar que:
a) (–∞, 1] ∪ [5, ∞) a) não existe solução em .
b) (–∞, -5) ∪ [–1, ∞) b) o conjunto admite infinitas soluções em .
c) [0, ∞) c) o conjunto solução é S = {x ∈  / 2 ≤ x ≤ 4}.
d) [–5, ∞) d) o conjunto solução é S = {x ∈  / x ≤ 2 ou x ≥ 4}.
e) (–1, ∞)
11. Com relação ao número de soluções inteiras da equação
−x + 3 (5 − x 2 )(x 2 − 2)
04. A soma das soluções da inequação > 0 onde x pertence ao > 0, podemos garantir que existem:
conjunto dos números naturais é: 2x − 1 x 2 − 2x + 5
a) infinitas
a) 3
b) quatro
b) 4
c) três
c) 5
d) seis
d) 6
e) duas
e) 8
9 − x2
(2x² + 5x − 3) 12. (MACKENZIE) A função f(x) = tem como domínio o
05. Os números reais x e y são tais que: y = . Nessas x +x−2
2
(1 − 5x) conjunto solução:
condições, tem-se y < 0 se, e somente se, x satisfizer a condição: a) S = {x ∈ / –3 < x ≤ –2 ou 1 ≤ x < 3}
a) – 3 < x < – 1/2 ou x > – 1/5 b) S = {x ∈ / –3 ≤ x < –2 ou 1 < x ≤ 3}
b) – 3 < x < 1/2 ou x > 1/5 c) S = {x ∈ / –3 ≤ x < –2 ou 1 ≤ x ≤ 3}
c) – 3 < x < 1/5 ou x > 1/2 d) S = {x ∈ / –2 < x ≤ –1 ou 1 ≤ x ≤ 3}
d) 1/5 < x < 1/2 ou x > 3 e) S = {x ∈ / –2 ≤ x < –1 ou 1 < x ≤ 3}
e) x < – 3 ou 1/5 < x < 1/2
13. A soma de todos os números inteiros que satisfazem
06. Considere a inequação (x – 1)(x – 4) ≤ 0. Considerando os números simultaneamente a inequação-produto (3x – 7) · (x + 4) < 0 e a
inteiros que a satisfazem. É correto concluir que: 2x + 1
inequação-quociente > 0 é:
a) Só dois deles são positivos. 5−x
a) 3
b) A soma de todos eles, é dez. b) 5
c) O maior deles é múltiplo de 3. c) 6
d) O produto de todos eles, é zero. d) 7
e) O produto de todos é um número negativo.
14. Dadas as desigualdades, em :
07. O conjunto solução S ⊂  da inequação (5x² – 6x – 8)(2 – 2x) < 0 é: I. 3x + 1 < –x + 3 ≤ –2x + 5
 4  4x − 1
a) S =  − ,2 ∪ ]−∞,1[ . II. ≤1
 5  x−2
O menor intervalo que contém todos os valores de x que satisfazem,
]2, +∞[ ∪  −
4 
b) =
S ,1 .
 5  simultaneamente, às desigualdades I e II é:

 4  a)  1 3
S =  − ,2 ∪ ]1, +∞[ . ,
c)
 5   3 5 

 4  3
d) S =  −∞, −  ∪ ]1,2[ . b)  −2, − 2 
 5
 4   3
e) S =  − ,1 ∪ ]2, +∞[ . c)  −∞, 5 
 5 
 1 1
(x³ − x² + −1) d)  − 3 , 2 
08. Considere a seguinte inequação: ≤ 0 . O conjunto
solução em  é: (x³ − 2x² + x − 2)
e)  4 3
a) [1, – 2[  3 , 5 
b) [– 1, 2[
c) [2, 3] x 2 − 4x + 3
15. A desigualdade 2 > 0 se verifica para todos os números
reais x tais que: x − 7x + 10
d) [1, 2[
a) –1 < x ou –3 < x < –2 ou < –5
09. Se n é um número inteiro, então a quantidade de números b) x < 1 ou 2 < x < 3 ou x > 5
2n 7
racionais da forma , que são estritamente menores que , é: c) 1 < x < 2 ou 3 < x < 5
3n + 15 13
a) 21 c) 20 e) 27 d) x > 1 ou 2 < x < 5
b) 25 d) infinita e) 1 < x < 3 ou 2 < x < 5

138

PM_BOOK16 - MAT.indb 138 25/11/2022 19:10:38


RESOLUÇÃO DE INEQUAÇÕES

16. Uma função consiste na associação de dois conjuntos A e B de 22. A amplitude do menor intervalo que contém as soluções inteiras
números reais, por meio de uma lei f. O subconjunto dos elementos de x 2 − 3x − 18
da inequação ≥ 0 é igual a:
A que corresponde a um, e somente um, elemento de B é denominado 13x − x 2 − 42
domínio da função D(f). a) 2
b) 4
(2x 2 − 8)(x 2 + x − 6)
Considerando que a expressão f(X) = é uma c) 6
x 2 + 2x − 3
função, determine o domínio de f(x). d) 8
a) D = {x ∈  | x > 1; x ≤ –2 e x ≠ –3} e) 10
b) D = {x ∈  | x > 1; x < –2 e x ≠ –3}
c) D = {x ∈  | x > 1; x ≥ –2 e x = –3} x 4 + 3x 3 − 4x 2
23. A função f ( x ) = é negativa em dois intervalos de
d) D = {x ∈  | x ≥ 1; x ≤ –2 e x = 3} x 2 + 4x − 4
números reais. A soma dos comprimentos desses dois intervalos é:
e) D = {x ∈  | x ≤ 1; x > –2 e x ≠ 3}
1
a)
2
17. (CN) Seja S a soma dos valores inteiros que satisfazem a inequação
(5x − 40)2 b) 1
≤ 0. Sendo assim, pode-se afirmar que:
x 2 − 10x + 21 c) 2
a) S é um número divisível por 7.
d) 2
b) S é um número primo.
e) 2 2
c) S² é divisível por 5.
d) S é um número racional. 24. (EFOMM 2010) O gráfico das três funções polinomiais do 1°
e) 3S+1 é um número ímpar. grau a, b e c definidas, respectivamente, por a(x), b(x), e c(x) estão
representadas abaixo.
18. O conjunto solução da inequação (x2 + x – 1)/(9 – x2) ≥ 1/(3 – x)
é dado por:
a) [-3, 3[
b) ]–∞, –2] ∪ [2, ∞[
c) ]–3, –2] ∪ [2, 3[
d) [–2, 2]
e) [2, ∞[

13x − 9
19. Dadas as funções f(x) = x + 3 e g(x) = , determine o maior
x+2
subconjunto dos números reais tal que f(x) > g(x).
a) ]5, +∞[
b) ]–2,5[
( ( )) ( ( ))
Nessas condições, o conjunto solução da inequação a x ⋅ b3 x
5 6

c) ]–∞,3[ ∪ ]5,+∞[ ≥ 0 é:
a) (–4;–1) ∪ [3;+∞)
(c ( x ))
d) ]–∞,3[
e) ]–2,3[ ∪ ]5,+∞[ b) [–4;–1] ∪ [3;+∞)
c) (–∞;–4) ∪ [–1;+∞)
20. (MACKENZIE) Se f e g são funções reais definidas por f(x) = x d) [4;+∞)
x
e g(x) = 2 , então o domínio da função composta f º g é o e)  – {4}
2x − 5x + 2
conjunto:
25. Seja a função
a) x ∈  | 0 ≤ x ≤ ∨ x ≥ 2
1
 2  f ( x=
) ( x 2 − 2x + 2) ( −x 2 + 3x + 10 ) − ( −x 2 + x − 1) ( x 2 + x − 6 ) , cujo
b) x ∈  | 0 ≤ x < ∨ x > 2
1
domínio é D = [a, b + c]. O valor de a + b + c é:
3 3 3

 2  abc
a) –3
c) x ∈  | 0 < x < ∨ x > 2
 1
 2  b) 1
d) x ∈  | x < ∨ x > 2
 1 3
 2  c) 0

e) x ∈  | x ≤ ∨ x ≥ 2
1 d) − 1
 2  3
e) 3
21. O número de inteiros negativos compreendidos entre as soluções
x 2 − ax
( x − 1) ( x + 2) ( x − 3) ( x + 6 ) ≤ 0
3 4 5
26. A solução da inequação ≥ 0, onde a < 0 é:
da inequação é igual a: −x + ax + 2a2
2
x 2 ( x − 7)
3

a) 1 a) ]-∞,2a[ ∪ [a,0] ∪ ]-a,+∞[


b) 3 b) ]-∞,2a] ∪ ]a,0[ ∪ [-a,+∞[
c) 5 c) ]-a,0] ∪ [a,2a[
d) 7 d) [2a,a] ∪ [0,-a]
e) 9 e) ]2a,a] ∪ [0,-a[

139

PM_BOOK16 - MAT.indb 139 25/11/2022 19:10:39


RESOLUÇÃO DE INEQUAÇÕES

27. A solução da inequação ax² – 2x + 4 > 0 é: x 4 − 2x 2 − 8


03. O número de soluções inteiras da inequação < 0 é igual a:
x2 + x − 1
a) x > 2, se a = 0 a) 0
b) +∞ < x < –∞, se a ≤ 1/4 b) 1
1 + 1 − 4a 1 − 1 − 4a c) 2
c) <x< , se a < 0
a a d) 3
1 + 1 − 4a
d) x > , se 0 < a ≤ 1 e) infinitas
a 4
1 − 1 − 4a 1
e) x < , se 0 < a ≤ 04. Dada a inequação: (3 − 5x) ⋅ (3 − 7x) ⋅ (7 − 5x) ≤ 0 . Resolvendo, em
a 4
, tem-se o conjunto solução:
28. A solução da inequação x − 1 < x + 2 é dada por:  3 7
a) S = x ∈  | ≤ x ≤ 
 1  7 5
a)  −∞, 
 2  3 7 3
b) S = x ∈  | ≤ x ≤ ou x ≤ 
 5 5 7
1 
b)  ,2
2   5 7 7
c) S = x ∈  | ≤ x ≤ ou x ≥ 
 3 5 5
1 
c)  2 , +∞  d)  7 3 7
S = x ∈  | ≤ x ≤ ou x ≤ 
 5 5 5
d) [2,+∞)
 3 3 7
e) (–∞,+∞) e) S = x ∈  | ≤ x ≤ ou x ≥ 
 7 5 5
3x 2 + 2x
29. (AFA) Sobre a inequação ≥ x 3 , considerando o conjunto x4 − 1
x 05. (EN) O conjunto solução da inequação ≤ 0, é:
universo U ⊂ , é INCORRETO afirmar que possui conjunto solução: −x + 3x 3 − 2x 2
4
a) ]–∞,–1] ∪ ]2,+∞[
a) unitário se U = {x ∈  | x > 0 e x = 2k, k ∈ *+}
b) ]–∞,–1[ ∪ ]1,2[
b) vazio se U = [2,+∞[
c) ]–∞,–1[ ∪ ]0,2[
c) com infinitas soluções se U = {x ∈  | x = 2k + 1, k ∈ }
d) ]–∞,1[ ∪ ]1,2[
d) com infinitas soluções se U = {x ∈ * | x ≤ 2}
e) ]–∞,1[ ∪ ]–1,+∞[

30. (IME) O sistema de inequações abaixo admite k soluções inteiras.


06. (CMRJ) Seja D o domínio da função
 x 2 − 2x − 14
 > 3. Pode-se afirmar que: ( 2x 2 − 7x + 6 )( 2x 2 − 7x + 5)
 x f (x) = . O complementar de D em
x ≤ 12 x 2 − 5x − 6
 relação a , onde  é o conjunto dos números reais, é:
a) 0 ≤ k < 2
 3  5 
b) 2 ≤ k < 4 a) ]−∞, −1[ ∪ 1,  ∪ 2,  ∪ ]6, +∞[
 2  2
c) 4≤k<6 5 
d) 6 ≤ k < 8 b) ]−∞,1] ∪  , +∞ 
2 
e) k ≥ 8 3  5 
c) [ −1,1[ ∪  ,2 ∪  ,6
2  2 
 3
d)  −∞,  ∪ [2, +∞[
 2
EXERCÍCIOS DE
 3  5

COMBATE
e)  −1,  ∪  2,  ∪ [6, +∞[
 2  2

x 2 − ax − 2a2
07. (ITA) Em qual dos casos vale a desigualdade < 0?
x − (a + 2)x + 2a
2

01. O número de inteiros positivos compreendidos entre as soluções a) a < 0, x < 2a


( x + 5) ( x − 3 )( x + 2 ) < 0 é igual a: b) a = 0, x > a
da inequação
( 2x − 3)( 4x + 5) c) a > 2, 2 < x < a
a) 0 d) a > 2, –a < x < 2
b) 1 e) a > 2, x > 2a
c) 2
d) 3 x 2 − 4x + 3
08. O conjunto solução da inequação ≤ 1 − x é igual a:
3 − 2x
e) 4 a) ( −∞, 0]  1, 3
 2 )
02. (CN) O número de soluções inteiras da inequação
x − 6x + 10
2
< 0 é:
b) [0,1]   3 , +∞
 2 )
a) 0 d) infinito
x2 − 1
c) ( −∞, 1]   3 2 , +∞ )
b) 3 e) 2 (
d) 0, 3 2   [3, +∞ )
c) 1 e) (–∞,+∞)

140

PM_BOOK16 - MAT.indb 140 25/11/2022 19:10:41


RESOLUÇÃO DE INEQUAÇÕES

09. O conjunto solução da inequação 1 + 2 < 3 é igual a:


x −1 x − 2 x − 3
( )
a) 1, 3 2  ( 3, +∞ )

(
b) ( −∞, 1)  3 , +∞
2 )
c) (–∞,1)  (3,+∞)

d) ( −∞, 1)  ( 3 2 ,2)  ( 3, +∞ )
e) (–∞,+∞)

10x 2 − 3x − 2
10. O conjunto solução da inequação −1 < < 1 é igual a:
−x 2 + 3x − 2
( ) (
a) − 2 ,0  6 , 2
3 11 3 )
( ) (
b) −∞, 611  2 3 , +∞ )
( ) (
c) −∞, − 2 3  2 3 , +∞ )
(
d) − 2 3 , 611 )
(
e) − 2 , 2
3 3 )

GABARITO
EXERCÍCIOS DE FIXAÇÃO
01. D 04. A 07. B 10. B
02. B 05. C 08. B
03. B 06. D 09. C
EXERCÍCIOS DE TREINAMENTO
01. B 09. B 17. B 25. E
02. B 10. C 18. C 26. E
03. B 11. E 19. E 27. C
04. A 12. B 20. B 28. C
05. C 13. A 21. E 29. B
06. B 14. D 22. E 30. D
07. E 15. B 23. B
08. D 16. A 24. A
EXERCÍCIOS DE COMBATE
01. A 04. E 07. D 10. A
02. C 05. A 08. A
03. B 06. C 09. D

ANOTAÇÕES

141

PM_BOOK16 - MAT.indb 141 25/11/2022 19:10:42


RESOLUÇÃO DE INEQUAÇÕES

ANOTAÇÕES

142

PM_BOOK16 - MAT.indb 142 25/11/2022 19:10:42


ANÁLISE COMBINATÓRIA:
ELEMENTOS DISTINTOS

PRINCÍPIOS DE CONTAGEM distintas de se ir de B até C e 3 maneiras distintas de se ir de C até D


então, pelo MP, o número de maneiras de se ir de A até D passando
por B e C é dado por 2 × 5 × 3 = 30.
INTRODUÇÃO
Frequentemente, no nosso dia a dia, precisamos enumerar Observação
“eventos” tais como, arrumação de objetos de certa maneira, 1. Como afirmações matemáticas, tanto o AP quanto o MP são
particionar coisas sob uma certa condição, distribuições para certos triviais e por esta razão são frequentemente negligenciados por
fins etc. Para fazermos isto, antes de tudo, precisamos enunciar dois aqueles que começam a estudar combinatória o que é uma
teoremas que são fundamentais em todos os problemas de contagem. pena pois, na realidade, eles são fundamentais na resolução de
problemas de contagem. Como veremos, nos exemplos, um dado
O PRINCÍPIO ADITIVO (AP) problema de contagem independente de quão complicado seja ele
Suponha que existam pode sempre ser decomposto em alguns mais simples que podem
ser contados usando AP e/ou MP.
n1 maneiras para o evento E1 ocorrer,
2. As palavras “e” e “ou” indicam geralmente quando um ou outro
n2 maneiras para o evento E2 ocorrer, princípio é mais apropriado para a resolução de um problema. A
 palavra “e” sugere o princípio multiplicativo (MP) e a palavra “ou”
nk maneiras para o evento Ek ocorrer, onde k ≥ 1. Se estas maneiras o princípio aditivo (AP).
para as ocorrências dos eventos distintos forem disjuntas duas a duas
então o número de maneiras nas quais pelo menos um dos eventos E1,
E2, ..., ou Ek pode ocorrer é ARRUMAÇÕES E ESCOLHAS SIMPLES
k
n1  n2  ...  nk  n
i1
i
DEFINIÇÕES
Uma permutação de n objetos distintos é qualquer agrupamento
Assim, por exemplo, se podemos ir de uma cidade P a uma
ordenado desses n objetos isto é, uma arrumação em ordenação dos
cidade Q por vias aérea, marítima e rodoviária, e supondo que existam
n objetos.
2 companhias marítimas, 3 companhias aéreas e 2 companhias
rodoviárias que fazem o trajeto entre P e Q, então pelo AP o número Uma k-permutação ou uma permutação de classe k dos n
total de se fazer o trajeto de P a Q pelo mar, pelo ar ou por rodovia é objetos distintos é uma arrumação utilizando k dos n objetos (uma
2 + 3 + 2 = 7. k-permutação é o que os livros tradicionais chamam de “arranjo de
k coisas dentre n”). Usaremos P (n, k) · A (n, k) ou Akn para denotar o
Uma forma equivalente do AP usando a terminologia dos
número de k-permutações de um conjunto de n objetos.
conjuntos onde |X| representa o número de elementos do conjunto
X é o seguinte: Do princípio multiplicativo obtemos:
Sejam A1, A2, ..., Ak conjuntos finitos quaisquer onde k ≥ 1. Se os P(n,2) = n · (n – 1); P(n,3) = n · (n – 1) · (n – 2) e P(n,n) = n · (n – 1)
conjuntos dados são distintos dois a dois, então · (n – 2) ... 3 · 2 · 1.
k k Consideremos os n objetos x1, x2, x3, ..., xn e as n posições:
A
i1
i  A1  A 2  ...  Ak  A
i1
i ___ ___ ___ ... ___
p1 p2 p3 ... pn
Enumerando todas as permutações dos n objetos x1, x2, x3, ...,
O PRINCÍPIO MULTIPLICATIVO (MP) xn temos que o número de tais permutações é igual ao número de
Supondo que um evento E possa ser decomposto em r eventos modos possíveis de se ocupar com esses n objetos as n posições p1,
ordenados E1, E2, ..., Er e que existam. p2, p3, ..., pn. Para a posição p1 existem n escolhas na arrumação.
n1 maneiras para o evento E1 ocorrer Após o preenchimento de p1 existem n – 1 escolhas (os n – 1 objetos
remanescentes) para a posição p2. Existem n – 2 maneiras diferentes de
n2 maneiras para o evento E2 ocorrer
ser preenchida a posição p3 após terem sido preenchidas as posições
 p1 e p2, ..., é finalmente uma escolha para a última posição pn, após
nr maneiras para o evento Er ocorrer terem sido preenchidas as posições p1, p2, p3, ..., pn-1. Portanto, pelo
Então o número de maneiras do evento E ocorrer é dado por princípio multiplicativo e utilizando a notação n! = n(n – 1)(n – 2)(n – 3)
r
... 3 · 2 · 1, temos que:
n1  n2  ...  nr  n
i1
1 O número de modos de ordenar n objetos distintos é n(n – 1)(n – 2)
(n – 3) ... 3 · 2 · 1 = n!
Assim, por exemplo, se para irmos de uma cidade A até uma Assim, temos as fórmulas P(n,n) = n! ou simplesmente Pn = n! e
cidade D devemos passar pelas cidades B e C, nesta ordem, e supondo n!
Akn  n(n  1)(n  2) ... (n  (k  1)) 
que existam duas maneiras distintas de ir de A até B, 5 maneiras (n  k )!

143

PM_BOOK16 - MAT.indb 143 25/11/2022 19:10:46


ANÁLISE COMBINATÓRIA: ELEMENTOS DISTINTOS

Por extensão, define-se P0 = 0! = 1 e P1 = 1! = 1 03. (EEAR) As atuais placas de automóveis possuem três letras do
Uma k-combinação ou uma combinação de classe k de n objetos alfabeto latino (incluindo K, W, Y) e quatro algarismos. O número de
distintos é uma escolha não ordenada ou um subconjunto de k dos placas que não repetem nem letras e nem algarismos é
o objetos. 26!10! c) 26! 10!
a)
Representaremos o número de combinações de n objetos distintos 23! 6! 26!10!
d)
de classe k ou tomados k a k por um dos símbolos b) 263 · 104 4! 3!
C(n, k ) ou 
n
k
04. (EEAR) O número de anagramas da palavra ESCOLA que começam
É padronizado ler qualquer um dos dois símbolos como “n escolhe por S e terminam por L, é
k”. (Outra notação comumente utilizada é Ckn.
a) 720 b) 120 c) 24 d) 12

TEOREMA 05. (EEAR) Com os algarismos 1, 2, 4, 5 e 7, a quantidade de números


Se 0 ≤ k ≤ n, então o número de subconjuntos de k elementos de de três algarismos distintos que se pode formar é
um subconjunto com n elementos ou o número de combinações de n a) 100 b) 80 c) 60 d) 30
objetos distintos e classe k é dado por

   k!(nn! k)!
n
k
06. (EEAR) Com os algarismos 1, 2, 3, 4 e 5, sem repeti-los, podemos
escrever x números de 4 algarismos, maiores que 2400. O valor de x é
Prova. O conjunto de todas as permutações simples de k a) 68 b) 72 c) 78 d) 84
elementos selecionados de um conjunto com n elementos contém
n! 07. (EEAR) Se existem k maneiras possíveis de pintar uma parede com
permutações. Entretanto, cada subconjunto de k elementos 3 listras verticais, de mesma largura e de cores distintas, dispondo de
(n − k )!
pode ser ordenado de k! maneiras assim, o número de maneiras de 12 cores diferentes, então o valor de k está compreendido entre
primeiro escolher um subconjunto e depois ordenar os elementos a) 1315 e 1330 c) 1345 e 1360
deste subconjunto é pelo princípio multiplicativo igual a b) 1330 e 1345 d) 1360 e 1375
  . k!
n
k
08. (CFOE) Com os dígitos 1, 2, 3, 6 e 0, podemos formar x números
Entretanto, cada uma dessas ordenações é uma diferente de 4 algarismos distintos. Então, x é igual a:
permutação de k elementos selecionados dentre todos os n elementos a) 160 b) 96 c) 180 d) 108
e cada permutação de k elementos distintos surge da escolha de um
subconjunto e a seguir, assim
09. (ESA) Com as letras da palavra SARGENTO foram escritos todos
  . k!  (n n!k)!
n
k
os anagramas iniciados por vogais e com as consoantes todas juntas.
Quantos são esses anagramas?
a) 120960 c) 2160 e) 120
o que produz
b) 40320 d) 720
 n
k 
n!
k !(n  k )!
10. (EFOMM) O código Morse, desenvolvido por Samuel Morse, em
1835, é um sistema de representação que utiliza letras, números e
EXERCÍCIOS DE sinais de pontuação através de um sinal codificado intermitentemente

FIXAÇÃO por pulsos elétricos, perturbações sonoras, sinais visuais ou sinais


de rádio. Sabendo-se que um código semelhante ao código Morse
trabalha com duas letras pré-estabelecidas, ponto e traço, e codifica
com palavras de 1 a 4 letras, o número de palavras criadas é:
01. (CFT) A quantidade de números de quatro algarismos distintos a) 10 c) 20 e) 30
que podem ser formados com os algarismos 3, 4, 5, 7 e 9 é b) 15 d) 25
a) 120 c) 160
b) 140 d) 210
EXERCÍCIOS DE

02. (CFT) Deseja-se colorir os seis triângulos da figura com cores


diferentes. TREINAMENTO
01. (EEAR) Uma classe tem 10 meninos e 9 meninas. Seu professor
necessita formar comissões de 7 crianças, sendo 4 meninos e 3
meninas, que incluam obrigatoriamente o melhor aluno dentre os
meninos e a melhor aluna dentre as meninas. O número possível de
comissões é
a) igual a 2300 c) menor que 2300
Dispondo-se de sete cores, o número de maneiras diferentes de b) maior que 2400 d) igual a 2352
conseguir o que se deseja é
a) 3200 02. (EEAR) Com os algarismos 2, 3, 4, 5, 6 e 7 posso escrever ____
b) 4700 números pares de quatro algarismos distintos.
c) 5040 a) 120 c) 240
d) 6090 b) 180 d) 360

144

PM_BOOK16 - MAT.indb 144 25/11/2022 19:10:50


ANÁLISE COMBINATÓRIA: ELEMENTOS DISTINTOS

03. (EEAR) Em um campeonato de tênis estão inscritos 10 militares. 13. (AFA) Em quantos números de 7 algarismos significativos, os
Para disputar o campeonato, esses militares podem formar _____ algarismos 5 e 8 aparecem exatamente 2 e 3 vezes, respectivamente?
duplas diferentes. a) 10270 b) 10280 c) 10290 d) 10300
a) 34 b) 35 c) 44 d) 45
14. (AFA) Sobre os lados de um triângulo, marcam-se, respectivamente,
04. (EEAR) Considere os algarismos 1, 2, 3, 4, 5 e 6. A partir deles, 4, 5 e 6 pontos distintos dos vértices. O número total de triângulos
podem ser criados _____ números pares de quatro algarismos distintos. com vértices nos pontos marcados é:
a) 60 b) 120 c) 180 d) 360 a) 301 b) 355 c) 421 d) 432

05. (ESPCEX) Um tabuleiro possui 16 casas distribuídas em 4 linhas e


15. (AFA) Se Amm + 2 = 360 , então o valor de
(m − 1) ! - m ! é:
4 colunas. De quantas maneiras diferentes é possível colocar 4 peças
(m-1) !
iguais nesse tabuleiro de modo que, em cada linha e cada coluna, seja
colocada apenas uma peça. a) –1 b) –3 c) –5 d) –7
a) 4096 c) 256 e) 16
16. (AFA) A quantidade de pares de retas reversas que contêm as
b) 576 d) 64
arestas de um cubo é

06. As antigas placas para automóveis, formadas por duas letras a) 12 b) 24 c) 36 d) 48


seguidas de quatro algarismos, como por exemplo MY – 7406 foram
substituídas por placas com três letras seguidas de quatro algarismos, 17. (AFA) Colocam-se em ordem crescente todos os números com 5
como por exemplo DWK – 2179. Utilizando um alfabeto de 26 letras e algarismos distintos, sem repetição, formados com 2, 4, 5, 7 e 8. A
supondo que qualquer sequência de letras e algarismos seja permitida posição do número 72584 é
(na realidade algumas sequências não são permitidas) quantos veículos a) 76ª b) 78ª c) 80ª d) 82ª
a mais podem ser emplacados?
18. (AFA) Seja An,p o número de arranjos simples de n elementos
07. (ESPCEX) Sobre um plano α tomam-se 8 pontos distintos dos distintos, tomados p a p. A equação An,3 = 6n tem como solução
quais não existem 3 na mesma reta, e fora de α toma-se um ponto A. a) uma raiz nula.
O número de pirâmides de base quadrangular com vértice em A que
pode se obter a partir desses pontos é b) uma raiz positiva.
a) 64 c) 72 e) 96 c) duas raízes positivas.
b) 70 d) 82 d) uma raiz positiva e outra negativa.

08. (ESPCEX) Entre duas cidades A e B há dois postos de pedágio, sendo 19. (AFA) A palavra que não muda o seu sentido, quer se leia da
o primeiro com 5 cabines e o segundo com 4 cabines. Há também esquerda para a direita ou da direita para a esquerda, é chamada
10 pontos de abastecimento. Um viajante realizará o percurso entre palíndromo (Ex.: ovo, asa, acaiaca, serres, etc.). Considerando-se
essas duas cidades passando pelos dois pedágios e parando três vezes as 23 letras do nosso alfabeto, quantos anagramas de 6 letras com
para abastecimento. Entendendo por “formas diferentes de realizar o características de um palíndromo, pode-se formar?
percurso” cada uma das opções de passar pelas cabines de pedágio a) 236 b) 233 c) 323 d) 623
e parar nos postos de abastecimento, o número de formas diferentes
como ele poderá realizar o percurso da cidade A para a cidade B é 20. (ESPCEX) Considere o conjunto de números naturais {1, 2, ..., 15}.
a) 60 c) 1200 e) 14400 Formando grupos de três números distintos desse conjunto, o número
de grupos em que a soma dos termos é ímpar é
b) 600 d) 2400
a) 168. c) 224. e) 231.
09. (EFOMM) De quantas maneiras diferentes podemos escolher seis b) 196. d) 227.
pessoas, incluindo pelo menos duas mulheres, de um grupo composto
de sete homens e quatro mulheres? 21. (ESPCEX) Duas instituições financeiras fornecem senhas para seus
a) 210 c) 371 e) 756 clientes, construídas segundo os seguintes métodos:
b) 250 d) 462 1ª instituição: 5 caracteres distintos formados por elementos do
conjunto {1,2,3,4,5,6,7,8,9};
10. (AFA) Um baralho é composto por 52 cartas divididas em 4 naipes 2ª instituição: 6 caracteres distintos formados por duas letras, dentre
distintos (copas, paus, ouros e espadas). Cada naipe é constituído por as vogais, na primeira e segunda posições da senha, seguidas por 4
13 cartas, das quais 9 são numeradas de 2 a 10, e as outras 4 são 1 algarismos dentre os elementos do conjunto {3,4,5,6,7,8,9}.
valete (J), 1 dama (Q), 1 rei (K) e 1 ás (A). Ao serem retiradas desse Para comparar a eficiência entre os métodos de construção das senhas,
baralho duas cartas, uma a uma e sem reposição, a quantidade de medindo sua maior ou menor vulnerabilidade, foi definida a grandeza
sequências que se pode obter em que a primeira carta seja de ouros e “força da senha”, de forma que, quanto mais senhas puderem ser
a segunda não seja um ás é igual a criadas pelo método, mais “forte” será a senha.
a) 612 b) 613 c) 614 d) 615 Com base nessas informações, pode-se dizer que, em relação à 2ª
instituição, a senha da 1ª instituição é
11. (AFA) Lançando-se 4 dados, sucessivamente, o número de a) 10% mais fraca. d) 20% mais fraca.
maneiras de se obter soma 7 é
b) 10% mais forte. e) 20% mais forte.
a) 20 b) 24 c) 72 d) 216
c) De mesma força.
12. (AFA) A quantidade de números naturais de 4 algarismos 22. (ESPCEX) Um grupo é formado por oito homens e cinco mulheres.
distintos, formados por 1, 2, 3, 4, 5 e 6, que contém o algarismo 3 Deseja-se dispor essas oito pessoas em uma fila, conforme figura
ou o algarismo 4 é abaixo, de modo que as cinco mulheres ocupem sempre as posições
a) 196 b) 286 c) 336 d) 446 1, 2, 3, 4 e 5, e os homens as posições 6, 7 e 8.

145

PM_BOOK16 - MAT.indb 145 25/11/2022 19:10:51


ANÁLISE COMBINATÓRIA: ELEMENTOS DISTINTOS

a) o segredo do cadeado da primeira mala, deverá fazer no máximo


(52 × 83) tentativas para abri-lo.
b) o segredo do cadeado da segunda mala, o número máximo de
tentativas para abri-lo será de 1.890.
c) apenas os três dígitos consecutivos em ordem crescente do
Quantas formas possíveis de fila podem ser formadas obedecendo a
cadeado da primeira mala, ela conseguirá abri-lo com, no máximo,
essas restrições?
8 tentativas.
a) 56 c) 40.320 e) 8.648.640
d) apenas os dois primeiros dígitos do cadeado da segunda mala,
b) 456 d) 72.072 deverá tentar no máximo 10 vezes para abri-lo.

23. (EN) Qual a quantidade de números inteiros de 4 algarismos 27. (FUVEST) Doze pontos são assinalados sobre quatro segmentos de
distintos, sendo dois algarismos pares e dois ímpares que podemos reta de forma que três pontos sobre três segmentos distintos nunca
formar, usando algarismos de 1 a 9? são colineares, como na figura.
a) 2400 c) 1840 e) 1200
b) 2000 d) 1440

24. (AFA) Com base no conhecimento sobre análise combinatória, é


correto afirmar que
(01) existem 2160 possibilidades de 8 pessoas ocuparem um veículo
com 3 lugares voltados para trás e 5 lugares voltados para frente,
sendo que 2 das pessoas preferem bancos voltados para trás, 3
delas preferem bancos voltados para frente e as demais não têm
preferência.
(04) com os algarismos 0, 1, 2, 3, 4 e 5, pode-se formar 525 números O número de triângulos distintos que podem ser desenhados com os
ímpares com 4 algarismos e que não tenham zeros consecutivos. vértices nos pontos assinalados é
(08) podem ser formados 330 paralelogramos a partir de 7 retas a) 200. c) 208. e) 220.
paralelas entre si, interceptadas por outras 4 retas paralelas b) 204. d) 212.
entre si.
A soma das alternativas corretas é 28. (AFA) Uma pessoa deve escolher (não importando a ordem) sete,
dentre dez cartões numerados de 1 a 10, cada um deles contendo
a) 05 b) 09 c) 12 d) 13
uma pergunta diferente. Se nessa escolha houver, pelo menos três,
dos cinco primeiros cartões, ela terá n formas de escolha. Sendo
25. (AFA) No ano de 2017, 22 alunos da EPCAR foram premiados assim, pode-se afirmar que n é um número
na Olimpíada Brasileira de Matemática das Escolas Públicas (OBMEP).
a) quadrado perfeito. c) ímpar.
Desses alunos, 14 ganharam medalhas, sendo 3 alunos do 3º
esquadrão, 9 do 2º esquadrão e 2 do 1º esquadrão. Os demais b) múltiplo de 11. d) primo.
receberam menção honrosa, sendo 2 alunos do 3º esquadrão, 4 do 2º
esquadrão e 2 do 1º esquadrão. 29. (AFA) As senhas de acesso a um determinado arquivo de um
microcomputador de uma empresa deverão ser formadas apenas
Para homenagear os alunos premiados, fez-se uma fotografia para ser
por 6 dígitos pares, não nulos. Sr. José, um dos funcionários dessa
publicada pela Nascentv em uma rede social.
empresa, que utiliza esse microcomputador, deverá criar sua única
Admitindo-se que, na fotografia, os alunos que receberam menção senha. Assim, é INCORRETO afirmar que o Sr. José
honrosa ficaram agachados, sempre numa única ordem, sem alteração
a) poderá escolher sua senha dentre as 212 possibilidades de
de posição entre eles, à frente de uma fila na qual se posicionaram os
formá-las.
alunos medalhistas, de modo que, nesta fila:
b) terá 4 opções de escolha, se sua senha possuir todos os dígitos
- As duas extremidades foram ocupadas somente por alunos do 2º
iguais.
esquadrão que receberam medalha;
c) poderá escolher dentre 120 possibilidades, se decidir optar por
- Os alunos do 1º esquadrão, que receberam medalha, ficaram um
uma senha com somente 4 dígitos iguais.
ao lado do outro; e
d) terá 480 opções de escolha, se preferir uma senha com apenas 3
- Os alunos do 3º esquadrão, que receberam medalha, ficaram,
dígitos iguais.
também, um ao lado do outro.
Marque a alternativa que contém o número de fotografias distintas 30. (AFA) Para evitar que João acesse sites não recomendados na
possíveis que poderiam ter sido feitas. Internet, sua mãe quer colocar uma senha no computador formada
a) (72) · 9! c) (288) · 9! apenas por m letras A e também m letras B (sendo m par). Tal senha,
quando lida da esquerda para a direita ou da direita para a esquerda,
b) (144) · 9! d) (864) · 9!
não deverá se alterar (Ex.: ABBA)
26. (AFA) Uma pessoa fará uma viagem e em cada uma de suas Com essas características, o número máximo de senhas distintas que
duas malas colocou um cadeado contendo um segredo formado por ela poderá criar para depois escolher uma é igual a:
cinco dígitos. Cada dígito é escolhido dentre os algarismos 0, 1, 2,
a)
( 2m)! ( 2m)!
3, 4, 5, 6, 7, 8 e 9. Na primeira mala, o segredo do cadeado começa m!m! c)  m   3m 
e termina com dígito par e os demais são dígitos consecutivos em  !  !
2  2 
ordem crescente. Na segunda mala, o segredo do cadeado termina  m! 
2

em dígito ímpar e apenas o 1° e o 2° dígitos são iguais entre si. Dessa  m   m   m!


b)
maneira, se ela esquecer:   !  ! d) m m
 2   2    !  !
2 2

146

PM_BOOK16 - MAT.indb 146 25/11/2022 19:10:51


ANÁLISE COMBINATÓRIA: ELEMENTOS DISTINTOS

31. (AFA) Sr. José deseja guardar 4 bolas – uma azul, uma branca, 03. (AFA 2013) Num acampamento militar, serão instaladas três
uma vermelha e uma preta – em 4 caixas numeradas: barracas: I, II e III. Nelas, serão alojados 10 soldados, dentre eles o
soldado A e o soldado B, de tal maneira que fiquem 4 soldados na
barraca I, 3 na barraca II e 3 na barraca III.
Se o soldado A deve ficar na barraca I e o soldado B NÃO deve ficar
na barraca III, então o número de maneiras distintas de distribuí-los
é igual a:
a) 560
b) 1120
O número de maneiras de Sr. José guardar todas as 4 bolas de forma c) 1680
que uma mesma caixa NÃO contenha mais do que duas bolas, é igual a d) 2240
a) 24 c) 144
04. (EN) Entre os dez melhores alunos que frequentam o grêmio de
b) 36 d) 204
informática da Escola Naval, será escolhido um diretor, um tesoureiro
e um secretário. O número de maneiras diferentes que podem ser
32. Calcule a soma de todos os números de cinco algarismos distintos feitas as escolhas é:
formados com os algarismos 1, 2, 3, 4 e 5.
a) 720 d) 120
33. (ITA) Seja A um conjunto com 14 elementos e B um subconjunto b) 480 e) 60
de A com 6 elementos. O número de subconjuntos de A com um c) 360
número de elementos menor ou igual a 6 e disjuntos de B é:
a) 28 – 9 c) 28 – 26 e) 28 05. (AFA 2011) Um colecionador deixou sua casa provido de R$ 5,00,
b) 28 – 1 d) 214 – 28 disposto a gastar tudo na loja de miniaturas da esquina. O vendedor
lhe mostrou três opções que havia na loja, conforme a seguir.
34. (ITA) Determine quantos números de 3 algarismos podem ser • 5 diferentes miniaturas de carros, custando R$ 4,00 cada miniatura;
formados com 1, 2, 3, 4, 5, 6 e 7, satisfazendo à seguinte regra: O • 3 diferentes miniaturas de livros, custando R$ 1,00 cada miniatura;
número não pode ter algarismos repetidos, exceto quando iniciar com • 2 diferentes miniaturas de bichos, custando R$ 3,00 cada miniatura.
1 ou 2, caso em que o 7 (e apenas o 7) pode aparecer mais de uma
vez. Assinale o resultado obtido. O número de diferentes maneiras desse colecionador efetuar a compra
a) 204 d) 210 das miniaturas, gastando todo o seu dinheiro, é:
b) 206 e) 212 a) 15 c) 42
c) 208 b) 21 d) 90

35. De quantas maneiras podemos distribuir n objetos diferentes em 06. Permutam-se de todas as formas possíveis os algarismos 1, 3, 5,
duas caixas diferentes, de modo que nenhuma caixa fique vazia? 7, 9 e, escrevem-se os números assim formados em ordem crescente.
A soma de todos os números assim formados é igual a
a) 2n-1 – 1 c) 2n – 2 e) 2n
a) 1 000 000
b) 2n – 1 d) 2n-1 – 2
b) 1 111 100
c) 6 000 000
d) 6 666 000
EXERCÍCIOS DE
e) 6 666 600

COMBATE 07. Os alunos de uma escola realizam experiências no laboratório


de Química utilizando 8 substâncias diferentes. O experimento
consiste em misturar quantidades iguais de duas dessas substâncias
01. Dispondo das cores verde, amarelo, azul e branco, o número de e observar o produto obtido. O professor recomenda, entretanto,
maneiras distintas que podemos pintar 7 casas enfileiradas de modo que as substâncias S1, S2 e S3 não devam ser misturadas entre si, pois
que cada casa seja pintada de uma só cor e duas casas vizinhas não produzem como resultado o gás metano, de odor muito ruim. Assim,
sejam pintadas com a mesma cor é igual a o número possível de misturas diferentes que se pode obter, sem
a) 2916 d) 1960 produzir o gás metano é

b) 2520 e) 1800 a) 16 d) 28

c) 2400 b) 24 e) 56
c) 25
02. Uma senha numérica é formada por 5 algarismos. Sabe-se que o
primeiro algarismo é ímpar, os dois últimos são iguais e os demais são 08. Sete livros didáticos, cada um de uma disciplina diferente, devem
distintos. Os quatro primeiros algarismos estão em ordem crescente ser posicionados lado a lado em uma estante, de forma que os livros
(da esquerda para a direita), como exemplos abaixo. de Física, de Química e de Matemática estejam sempre juntos, em
12344 e 35799 qualquer ordem. O número de maneiras diferentes em que esses livros
podem ser posicionados é
A quantidade de senhas possíveis com essas características é
a) 720 d) 2880
a) 22680 d) 160
b) 1440 e) 5040
b) 11340 e) 80
c) 2160
c) 3780

147

PM_BOOK16 - MAT.indb 147 25/11/2022 19:10:52


ANÁLISE COMBINATÓRIA: ELEMENTOS DISTINTOS

09. Para se ter acesso a um arquivo de computador, é necessário que o


usuário digite uma senha de 5 caracteres, na qual os três primeiros são GABARITO
algarismos distintos, escolhidos de 1 a 9, e os dois últimos caracteres
EXERCÍCIOS DE FIXAÇÃO
são duas letras, distintas ou não, escolhidas dentre as 26 do alfabeto.
Assim, o número de senhas diferentes, possíveis de serem obtidas por 01. A 04. C 07. A 10. E
esse processo, é 02. C 05. C 08. B
a) 327650 03. A 06. D 09. C
b) 340704 EXERCÍCIOS DE TREINAMENTO
c) 473805 01. D 10. A 19. B 28. B
d) 492804 02. B 11. A 20. C 29. C
e) 501870 03. D 12. C 21. A 30. B
04. C 13. B 22. C 31. D
10. Num determinado setor de um hospital, trabalham 4 médicos e 8
05. B 14. C 23. D 32. 3.999.960
enfermeiras. O número de equipes distintas, constituídas cada uma de
1 médico e 3 enfermeiras, que podem ser formadas nesse setor é de 06. 2916 15. B 24. A 33. A
a) 60 07. B 16. B 25. D 34. E
b) 224 08. D 17. A 26. C 35. C
c) 495 09. C 18. B 27. D
d) 1344 EXERCÍCIOS DE COMBATE
e) 11880 01. A 04. A 07. C 10. B
02. E 05. B 08. A
03. B 06. E 09. B
DESAFIO PRO

DESAFIO PRO 01. 220


02. E
03. 2620
04. E
05. 473

ANOTAÇÕES

1 (ITA) Determine quantos paralelepípedos retângulos


diferentes podem ser construídos de tal maneira que a
medida de cada uma de suas arestas seja um número inteiro
positivo que não exceda 10.

2 (ITA) Pintam-se N cubos iguais utilizando-se 6 cores


diferentes, uma para cada face. Considerando que cada
cubo pode ser perfeitamente distinguido dos demais, o maior
valor possível de N é igual a
a) 10 d) 25
b) 15 e) 30
c) 20

3 (IME) Um professor dá um teste surpresa para uma turma


de 9 alunos, e diz que o teste pode ser feito sozinho ou
em grupos de 2 alunos. De quantas formas a turma pode se
organizar para fazer o teste? (Por exemplo, uma turma de 3
alunos pode se organizar de 4 formas e uma turma de 4 alunos
pode se organizar de 10 formas).

4 (ITA) Sobre duas retas paralelas r e s são tomados 13


pontos, m pontos em r e n pontos em s, sendo m > n. Com
os pontos são formados todos os triângulos e quadriláteros
convexos possíveis. Sabe-se que o quociente entre o número
de quadriláteros e o número de triângulos é 15/11. Então, os
valores de n e m são, respectivamente,
a) 2 e 11.
b) 3 e 10.
c) 4 e 9.
d) 5 e 8.
e) 6 e 7.

5 (ITA) Quantos pares de números inteiros positivos (A,B)


existem cujo mínimo múltiplo comum é 126 × 10³? Para
efeito de contagem, considerar (A,B) ≡ (B,A).

148

PM_BOOK16 - MAT.indb 148 25/11/2022 19:10:52


ANÁLISE COMBINATÓRIA:
ELEMENTOS COM REPETIÇÃO

PERMUTAÇÃO COM REPETIÇÃO uma escolha sem repetição. Suponhamos que as três variedades sejam
sem molho, com molho e completo e que a atendente tenha anotado
Discutiremos agora arrumações de uma coleção de objetos com o seguinte pedido:
objetos repetidos, tais como a coleção {b, a, n, a, n, a} e escolhas
de objetos de um conjunto onde um objeto pode ser selecionado sem molho com molho completo
mais de uma vez, tais como pedir seis cachorros quentes a serem x xxxx x
escolhidos dentre três variedades. Determinaremos as fórmulas para
esses problemas de contagem através de dois exemplos: Se cada s representa um cachorro quente, então o pedido acima
significa um sem molho, quatro com molho e um completo. Uma vez
Exemplo:
que todos os atendentes saibam que esta é a sequência dos pedidos
Quantas arrumações podem ser feitas com as seis letras b, a, n, de cachorros quentes (sem molho, com molho, completo) podemos
a, n, a? omitir os nomes das variedades escrevendo apenas x | xxxx | x. Assim,
Formaremos as arrumações escolhendo primeiro as três posições qualquer pedido de k cachorros quentes consiste numa sequência

onde os a’s ficarão, isto é 63  20 maneiras. Agora, escolhamos as de k x’s e dois |’s. Reciprocamente, toda sequência de k x’s e dois |’s
representa um pedido: os x’s antes do primeiro | representa o número
duas posições (entre as três remanescentes) onde os n’s ficarão, isto

é 32  3 maneiras. Finalmente, na última posição, fica o b. Assim, de cachorros sem molho: os x’s entre os dois |’s representa o número
de cachorros com molho e os x’s finais representam o número de
existem 20 · 3 · 1 = 60 arrumações.
cachorros completos. Deste modo, existe uma correspondência um
a um entre pedidos e tais sequências, mas o número de sequências
TEOREMA de seis x’s e dois |’s é simplesmente o número de escolhas de duas
Se existem n objetos dos quais k1 são do tipo 1, k2 são do tipo 2, 
posições na sequência para os |’s. Assim, a resposta é 82  28.
..., e km são do tipo m onde k1 + k2 + ... + kn = n então o número de
arrumações destes n objetos denotado por P(n; k1, k2, ..., km) é: TEOREMA
P(n; k1, k 2 , ..., km )        ...    = k !k n!...
n
k1
n k1
k2
km
km
!
k
O número de escolhas com repetição de k objetos dentre n tipos
 k  n  1
1 2 m!
de objetos é  .
Prova com efeito, além do argumento utilizado no exemplo acima  k 
a saber, escolhendo as posições para um dos tipos dentre aquelas que Prova. Com efeito, faremos um “pedido” para uma escolha
vão restando, podemos provar o teorema acima da seguinte forma: como fizemos no exemplo anterior com um x para cada objeto
escolhido. Como fizemos anteriormente, os x’s antes do primeiro |
Suponhamos que para cada tipo dos ki objetos do tipo i sejam
conta o número de objetos do primeiro tipo, os x’s entre o primeiro
dados índices 1, 2, 3, ..., m tornando-os distintos. Existem então n!
e o segundo |’s conta o número de objetos do segundo tipo, ..., e os
arrumações destes n objetos distintos. Enumeremos agora estas n!
x’s após o (n – 1) – ésimo| conta o número de objetos do enésimo
arrumações de objetos distintos enumerando todas as P(n; k1, k2, ...,
tipo (n – 1 traços são necessários para separar n tipos). O número de
km) disposições (sem índices) dos objetos e então, para cada disposição
 k  (n  1) 
colocando os índices de todos os modos possíveis. Por exemplo, da sequências com k x’s e (n – 1) |’s é  .
disposição banana os índices podem ser colocados nos a’s de 3!  k 
maneiras:
b a1 a2 n n a3 b a2 a1 n n a3 b a3 a1 n n a2 DISTRIBUIÇÕES
b a3 a2 n n a1 b a1 a3 n n a2 b a2 a3 n n a1 Geralmente um problema de distribuição é equivalente a um
problema de arrumação ou de escolha com repetição. Problemas
Para cada uma dessas 3! maneiras para indexar os a’s, existem especializados de distribuição devem ser divididos em subcasos que
2! maneiras para indexar os n’s. Assim, em geral, uma disposição possam ser contados por intermédio de permutações e combinações
qualquer terá k1! maneiras de indexar os k1 objetos do tipo 1, k2! simples. Um roteiro geral para modelar problemas de distribuição
maneiras para o tipo 2, ..., km! maneiras para o tipo m. Então: é, distribuições de objetos distintos correspondem a arrumações e
distribuições de objetos idênticos correspondem a escolhas.
n! .
P(n; k1, k 2 , ..., km ) = Assim, distribuir k objetos distintos em n urnas diferentes é
k1! k 2 ! ... km !
equivalente a colocar os objetos em linha e atribuir o nome de
6! cada uma das n diferentes urnas em cada objeto. Assim, existem
Assim sendo, a palavra banana tem P63,2,1   60 anagramas.
3! 2! 1! n . n . n . ... n = nk distribuições.
  
k vezes
COMBINAÇÕES COM REPETIÇÃO Por outro lado, o processo de distribuir k objetos idênticos em
Exemplo: n urnas distintas é equivalente a escolher um subconjunto (não
De quantos modos distintos podemos comprar seis cachorros ordenado) de k nomes de urnas, com repetição, entre as n escolhas de
quentes podendo escolher entre 3 variedades distintas?  k  n  1 (k  n  1)!
urnas. Assim, existem   distribuições.
Para resolver problemas de escolhas com repetição, precisamos  k  k !(n  1)!
fazer uma correspondência um a um com um problema relacionado a

149

PM_BOOK16 - MAT.indb 149 25/11/2022 19:11:00


ANÁLISE COMBINATÓRIA: ELEMENTOS COM REPETIÇÃO

Observação distinguíveis se, e somente se, uma não pode ser obtida da outra por
qualquer rotação como, por exemplo, duas permutações quaisquer
Os problemas de escolhas com repetição podem ser formulados de em linhas diferentes do quadro acima.
três formas equivalentes a saber: Portanto, no cálculo das permutações circulares interessa apenas a
1. o número de maneiras de escolhermos k objetos com repetição posição relativa dos objetos entre si, isto é, o número de permutações
dentre n tipos de objetos distintos; circulares distinguíveis.
2. o número de maneiras de distribuir k objetos idênticos em n Generalizando temos que o número de permutações circulares de
urnas distintas; n objetos, denotado por (PC)n, é igual a n!/n, isto é:
3. o número de soluções inteiras não negativas da equação x1 + x2
n!
+ ... + xn = k. (PC)n   (n 1)!
n

PERMUTAÇÕES CIRCULARES EXERCÍCIOS DE


Consideremos n objetos distintos e disponhamos esses n objetos
em torno de um círculo. FIXAÇÃO
Se n > 3, podemos imaginar esses objetos situados nos vértices de
um polígono, por exemplo, um polígono regular.
O quadro abaixo apresenta as disposições dos objetos A, B, C, D 01. (ENEM) Um brinquedo infantil caminhão-cegonha é formado por
em torno de um círculo. uma carreta e dez carrinhos nela transportados, conforme a figura.

A D C B

B D A C D B C A

C B A D
A D B C

C D A B D C B A
No setor de produção da empresa que fabrica esse brinquedo, é feita
a pintura de todos os carrinhos para que o aspecto do brinquedo fique
B C A D
mais atraente. São utilizadas as cores amarelo, branco, laranja e verde,
A B D C e cada carrinho é pintado apenas com uma cor. O caminhão-cegonha
tem uma cor fixa. A empresa determinou que em todo caminhão-
C B A D B C D A cegonha deve haver pelo menos um carrinho de cada uma das quatro
cores disponíveis. Mudança de posição dos carrinhos no caminhão-
D C A B cegonha não gera um novo modelo do brinquedo. Com base
nessas informações, quantos são os modelos distintos do brinquedo
A C D B caminhão-cegonha que essa empresa poderá produzir?
a) C6,4 c) C10,4 e) 46
B C A D C B D A
b) 69,3 d) 6 4

D B A C
02. O número de anagramas que se pode formar com a palavra
A C B D ARRANJO é igual a
a) 21 c) 5.040 e) 1.260
D C A B C D B A
b) 42 d) 2.520
B D A C
03. (PUCRJ) Seja n a quantidade de anagramas da palavra FILOSOFIA
A B C D que possuem todas as vogais juntas. Temos que n vale:
a) 1.800 c) 4.800 e) 362.880
D B A C B D C A
b) 3.600 d) 181.440
C D A B
04. O número de anagramas distintos que podemos formar com o
termo DIREITO é
Observamos então que:
a) 5.040 b) 2.520 c) 120 d) 7
A 1ª coluna do quadro foi obtida fixando-se o objeto A e
permutando-se os objetos B, C, D de todos os modos possíveis, isto é, 05. O número de anagramas da palavra TAXISTA, que começam com
3! = 6 modos. Em cada linha uma disposição pode ser obtida de outra a letra X, é
por uma rotação conveniente e dadas duas disposições em linhas
diferentes, nenhuma pode ser obtida da outra por qualquer rotação. a) 180. c) 720. e) 10080.
Assim, chama-se permutação circular de n objetos distintos b) 240. d) 5040.
qualquer disposição desses objetos em torno de um círculo e duas
permutações circulares são indistinguíveis se, e somente se, uma 06. A quantidade de anagramas da palavra CONCURSO é:
pode ser obtida a partir da outra por uma rotação conveniente como, a) 2520 c) 10080 e) 40320
por exemplo, duas permutações quaisquer de uma mesma linha do b) 5040 d) 20160
quadro acima. Diremos ainda que duas permutações circulares são

150

PM_BOOK16 - MAT.indb 150 25/11/2022 19:11:02


ANÁLISE COMBINATÓRIA: ELEMENTOS COM REPETIÇÃO

07. (ESA) Quantos anagramas da palavra CONSOANTES podem ser 05. Uma urna contém 10 bolas, sendo 3 bolas pretas iguais, 3 bolas
formados com as vogais juntas e em ordem alfabética? brancas iguais, 2 bolas verdes iguais e 2 bolas azuis iguais. Quantas
10! 10! 7! são as maneiras diferentes de se extrair, uma a uma, as 10 bolas da
a) c) e) urna, sem reposição?
2!.2!.2! 7!.3! 2!.2!
a) 25.200 c) 144 e) 72.000
10! 7!
b) d) b) 10! d) 3.600
2!.2! 2!.2!.2!

08. (ESA) Assinale a alternativa cuja palavra possui 60 anagramas. 06. A capital dos gaúchos, oficialmente fundada em 26 de março de
a) A M E I X A c) BANANA e) P A T E T A 1772, já foi chamada de Porto de Viamão. Atualmente, a também
capital dos Pampas recebe o nome de PORTO ALEGRE. Adicionando o
b) B R A N C O d) P A R Q U E
número de anagramas formados com as letras da palavra ALEGRE ao
de anagramas formados com as letras da palavra PORTO em que as
09. (ESA) O número de anagramas diferentes com as letras da palavra consoantes aparecem juntas, obtemos __________ anagramas.
MILITAR que não possuem consoantes consecutivas que se pode obter
a) 378 c) 738 e) 840
é:
b) 396 d) 756
a) 60 c) 120 e) 224
b) 72 d) 186
07. Daniela tem 5 pulseiras diferentes e as utiliza necessariamente
colocando-as uma após a outra. Ela pode usar todas as pulseiras em
10. Na figura a seguir, as linhas horizontais e verticais representam apenas um braço ou distribuí-las entre os braços direito e esquerdo.
ruas e os quadrados representam quarteirões. A quantidade de Daniela considera como um arranjo diferente tanto o braço em que
trajetos de comprimento mínimo ligando A a B é: as pulseiras são colocadas quanto a ordem como elas são distribuídas.
As figuras mostram três arranjos diferentes que Daniela pode fazer.

a) 40.320
b) 6.720
c) 256
d) 120
e) 56 O número de arranjos diferentes que Daniela pode fazer usando todas
essas pulseiras é
EXERCÍCIOS DE a) 240. c) 480. e) 720.

TREINAMENTO b) 360. d) 600.

08. (FATEC) No Boxe, um dos esportes olímpicos, um pugilista tem


à sua disposição quatro golpes básicos: o jab, o direto, o cruzado e
o gancho. Suponha que um pugilista, preparando-se para os Jogos
01. Os alunos do curso de Computação Gráfica do campus Olinda
Olímpicos do Rio, em 2016, queira criar uma sequência com 6 golpes,
estão desenvolvendo um vídeo com todos os anagramas da palavra
empregando necessariamente dois jabs, dois diretos, um cruzado e
CARNAVAL. Se cada anagrama é mostrado durante 0,5 s na tela, a
um gancho. Assim, o número máximo de sequências que ele poderá
animação completa dura
criar será de
a) menos de 1 minuto. d) menos de 10 minutos.
a) 180. c) 140. e) 100.
b) menos de 1 hora. e) mais de 1 hora.
b) 160. d) 120.
c) menos de meia hora.
09. Uma criança possui 6 blocos de encaixe, sendo 2 amarelos, 2
02. O número de ternos (x, y, z) de números inteiros positivos, maiores vermelhos, 1 verde e 1 azul.
do que cinco, que cumprem a condição x + y + z = 30 é
a) 71. b) 91. c) 61. d) 81.

03. Uma senha de internet é constituída de seis letras e quatro


algarismos em que a ordem é levada em consideração. Eis uma
senha possível: (a, a, b, 7, 7, b, a, 7, a, 7). Quantas senhas diferentes
podem ser formadas com quatro letras “a”, duas letras “b” e quatro Usando essas peças, é possível fazer diferentes pilhas de três blocos. A
algarismos iguais a 7? seguir, são exemplificadas quatro das pilhas possíveis.
a) 10! c) 3 150 10!
e)
b) 2 520 d) 6 300 4!6!

04. (UERJ) Uma criança ganhou seis picolés de três sabores diferentes:
baunilha, morango e chocolate, representados, respectivamente, pelas
letras B, M e C. De segunda a sábado, a criança consome um único picolé
por dia, formando uma sequência de consumo dos sabores. Observe
estas sequências, que correspondem a diferentes modos de consumo: Utilizando os blocos que possui, o total de pilhas diferentes de três
(B, B, M, C, M, C) ou (B, M, M, C, B, C) ou (C, M, M, B, B, C) blocos, incluindo as exemplificadas, que a criança pode fazer é igual a
O número total de modos distintos de consumir os picolés equivale a: a) 58. c) 42. e) 72.
a) 6 b) 90 c) 180 d) 720 b) 20. d) 36.

151

PM_BOOK16 - MAT.indb 151 25/11/2022 19:11:02


ANÁLISE COMBINATÓRIA: ELEMENTOS COM REPETIÇÃO

10. (CEFET MG) Como prêmio pela vitória em uma competição,


serão distribuídas 12 moedas de ouro idênticas entre as três pessoas a) 60
da equipe vencedora, e cada uma deverá receber, pelo menos, b) 120
duas moedas. O número de maneiras distintas de efetuarmos essa
distribuição é c) 8400

a) 12. c) 38. e) 120. d) 12600

b) 28. d) 40. e) 15120

11. Existe quantidade ilimitada de bolas de três cores diferentes 16. De quantos modos diferentes podemos distribuir 10 bombons
(branca, preta, azul) em um depósito, sendo que as bolas se diferenciam idênticos em 4 caixas diferentes?
apenas pela cor. Oito dessas bolas serão colocadas em uma caixa. A
quantidade de caixas diferentes que podem ser compostas com oito 17. Dispondo de 4 cores diferentes, de quantas maneiras distintas
bolas é igual a podemos pintar 5 objetos idênticos? (Cada objeto deve se pintado
com uma única cor).
a) 38. c) 56. e) 25.
b) 336. d) 45. 18. De quantos modos podemos distribuir as 52 cartas de um baralho
comum entre 4 jogadores de modo que um deles fique com todas
12. Um projeto piloto desenvolvido em um curso de Engenharia as cartas de espadas? Quantas são as distribuições nas quais cada
Mecânica prevê a construção do robô “Eddie”, cujos movimentos jogador fique com um ás?
estão limitados apenas a andar para frente (F) e para a direita (D).
Suponha que Eddie está na posição A e deseja-se que ele se desloque 19. (UFLA 2008) Um problema clássico em combinatória é calcular o
até chegar à posição B, valendo-se dos movimentos que lhe são número de maneiras de se colocar bolas iguais em caixas diferentes.
permitidos. Admita que cada movimento feito por Eddie o leve a uma Calcule o número de maneiras de se colocar 7 bolas iguais em 3 caixas
posição consecutiva, conforme ilustra um esquema a seguir, em que diferentes, sem que nenhuma caixa fique vazia.
foram realizados 10 movimentos (as posições possíveis estão marcadas
por pontos e o percurso executado de A até B, é representado pela
20. Quantas soluções inteiras possui a equação x1 + x2 + x3 + x4 = 12
sequência ordenada de movimentos D F D D F F D F F D).
com x1 ≥ 0? Quantas são as soluções com x1 ≥ 2, x2 ≥ 2, x3 ≥ 4 e x4 ≥ 0?

21. (EFOMM) Considere uma loja que vende cinco tipos de


refrigerantes. De quantas formas diferentes podemos comprar três
refrigerantes desta loja?
a) Dez. c) Vinte. e) Sessenta.
b) Quinze. d) Trinta e cinco.

22. (EFOMM) Um decorador contemporâneo vai usar quatro “objetos”


perfilados lado a lado como decoração de um ambiente. Ele dispõe de
4 copos transparentes azuis, 4 copos transparentes vermelhos, duas
bolas amarelas e 3 bolas verdes. Cada “objeto” da decoração pode
ser um copo vazio ou com uma bola dentro. Considerando que a cor
altera a opção do “objeto”, quantas maneiras distintas há de perfilar
esses quatro “objetos”, levando-se em conta que a posição em que
ele se encontra altera a decoração?
a) 1.296 c) 1.152 e) 1.008
b) 1.248 d) 1.136
Com base nas informações acima, o número de maneiras possíveis
de Eddie se deslocar de A até B, sem passar pelo ponto C, é igual a
23. (AFA) O número de anagramas da palavra ALAMEDA que não
a) 192 c) 15 apresenta as 4 vogais juntas é
b) 60 d) 252 a) 96 b) 744 c) 816 d) 840

13. (EEAR) O número de anagramas formados com as letras da 24. Considere que, em uma empresa, seja utilizado sistema de
palavra ROMA de modo que não apareçam vogais ou consoantes códigos com apenas dois tipos de símbolos (1 e 2), sendo cada
juntas é igual a código formado por uma sequência desses símbolos, cuja ordem é
a) 4! c) 8 igual à soma dos algarismos que formam o código, a exemplo dos
códigos distintos 1, 11, 12 e 121, que são de ordem 1, 2, 3 e 4,
b) 4 d) 2
respectivamente. Considere, ainda, que S(0) = 1 e que S(n) é igual
ao número de códigos distintos de ordem n, onde n > 1. O número
14. (EN) A Escola Naval irá distribuir 4 viagens para a cidade de máximo de códigos distintos de ordem menor ou igual a 10 é
Fortaleza, 3 para a cidade de Natal e 2 para a cidade de Salvador. De
quantos modos diferentes podemos distribuí-las entre 9 aspirantes, a) 230 c) 120 e) 55
dando somente uma viagem para cada um? b) 180 d) 72
a) 288 c) 60800 e) 120960
25. (AFA) Dez vagas de um estacionamento serão ocupadas por seis carros,
b) 1260 d) 80760 sendo: 3 pretos, 2 vermelhos e 1 branco. Considerando que uma maneira
de isso ocorrer se distingue de outra tão somente pela cor dos carros, o
15. (EFOMM - Adaptada) Quantos são os anagramas da palavra total de possibilidades de os seis carros ocuparem as dez vagas é igual a
MERCANTE que possuem a letra M da 1ª posição (no caso, a posição a) 12.600 c) 21.600
de origem), ou a letra E na 2ª posição, ou a letra R na 3ª posição?
b) 16.200 d) 26.100

152

PM_BOOK16 - MAT.indb 152 25/11/2022 19:11:03


ANÁLISE COMBINATÓRIA: ELEMENTOS COM REPETIÇÃO

EXERCÍCIOS DE 08. (EPCAR 3º ANO) De quantos modos 3 casais podem sentar-se ao

COMBATE redor de uma mesa circular de tal modo que marido e mulher não
fiquem juntos?
a) 12 c) 72
b) 120 d) 32
01. Quantos anagramas da palavra ARATACA começam por
consoante? 09. (AFA 2007) Assinale a alternativa correta.
a) Pode-se codificar quinhentos pacientes, por uma palavra de duas
02. (ITA 2014) Determine quantos paralelepípedos retângulos letras quando as letras são escolhidas de um alfabeto de 25 letras.
diferentes podem ser construídos de tal maneira que a medida de
b) Nas calculadoras, os algarismos são frequentemente
cada uma de suas arestas seja um número inteiro positivo que não
representados, iluminando-se algumas das sete barras reunidas
exceda 10.
na forma padrão. O número de diferentes símbolos que podem
ser expressos pelas sete barras é igual a 7! (fatorial de 7).
03. (AFA 2010) Numa sala de aula, estão presentes 5 alunos e 6 alunas.
Para uma determinada atividade, o professor deverá escolher um grupo c) Entre 10 machos e 7 fêmeas de gatos experimentais, foi escolhida
de 3 dessas alunas e 3 dos alunos. Em seguida, os escolhidos serão uma amostra de dois machos e duas fêmeas. O número de
dispostos em círculo de tal forma que alunos do mesmo sexo não maneiras que isto pode ser feito é igual a 945.
fiquem lado a lado. Isso poderá ocorrer de n. O número n é igual a: d) O número de anagramas da palavra ASTRONAUTA é igual a 10!
a) 24000 b) 2400 c) 400 d) 200 (fatorial de 10).

04. A figura representa o mapa de uma cidade, na qual há 7 avenidas 10. (AFA 2001) Numa demonstração de paraquedismo, durante a
na direção norte-sul e 6 avenidas na direção leste-oeste. queda livre, participam 10 paraquedistas. Em um certo momento, 7
deles devem dar as mãos e formar um círculo. De quantas formas
distintas eles poderão ser escolhidos e dispostos nesse círculo?
a) 120 c) 86400
b) 720 d) 151200

DESAFIO PRO
1 (EFOMM) A quantidade de anagramas da palavra
MERCANTE que não possui vogais juntas é

a) Quantos são os trajetos de comprimento mínimo ligando o ponto a) 40320. d) 7200.


A ao ponto B? b) 38160. e) 3600.
b) Quantos desses trajetos passam por C? c) 37920.

05. (EN 1997) A Escola Naval (EN) receberá 20 novos Oficiais, entre
Fuzileiros, Intendentes e Oficiais da Armada. De quantos modos pode
ser preenchido o efetivo da EN se deve haver entre os 20 novos Oficiais
2 (ITA) Determine quantos paralelepípedos retângulos
diferentes podem ser construídos de tal maneira que a
medida de cada uma de suas arestas seja um número inteiro
pelo menos dois Fuzileiros, pelo menos dois Intendentes e pelo menos positivo que não exceda 10.
dois do Corpo da Armada?
a) 40
b) 80
c)
d) 120
100 e) 420
3 (EFOMM) Quantos anagramas são possíveis formar com a
palavra CARAVELAS, não havendo duas vogais consecutivas
e nem duas consoantes consecutivas?

06. Se todos os anagramas da palavra ESPCEX forem colocados em a) 24 d) 1.920


ordem alfabética, a palavra ESPCEX ocupará, nessa ordenação, a b) 120 e) 3.840
posição c) 480
a) 144 c) 206 e) 215
b) 145 d) 214
4 (EN) Calcule o número de soluções inteiras não negativas
de x1 + x2 + x3 + x4 + x5 + x6 = 20, nas quais pelo menos 3
incógnitas são nulas, e assinale a opção correta.
07. Um gerente de um hotel após fazer alguns cálculos chegou à
conclusão de que para atingir a meta de economia de energia elétrica a) 3.332 d) 3.678
bastava apagar duas lâmpadas de um corredor com 10 lâmpadas b) 3.420 e) 3.711
alinhadas. Para manter um mínimo de claridade ao longo do corredor
c) 3.543
o gerente determinou que duas lâmpadas adjacentes não poderiam
ficar apagadas e as 2 lâmpadas das extremidades deveriam ficar
acesas. Sendo assim o número de maneiras que esse gerente pode
apagar duas lâmpadas é: 5 (EN) Quantos são os anagramas de MARINHA, em que
somente uma vogal apareça em sua posição de origem?
a) 24 d) 12 a) 1512 d) 720
b) 21 e) 10 b) 1152 e) 480
c) 15 c) 1008

153

PM_BOOK16 - MAT.indb 153 25/11/2022 19:11:03


ANÁLISE COMBINATÓRIA: ELEMENTOS COM REPETIÇÃO

GABARITO
EXERCÍCIOS DE FIXAÇÃO
01. B 04. B 07. E 10. E
02. E 05. C 08. C
03. A 06. C 09. B
EXERCÍCIOS DE TREINAMENTO
01. B 14. B
02. B 15. C
03. B 16. 286
04. A 17. 56
05. A 39! 48!
18. e 4!⋅ .
(13!) (12!)
3 4
06. A
07. E 19. 8
08. A 20. 455 e 35
09. C 21. D
10. B 22. D
11. D 23. B
12. A 24. E
13. C 25. A
EXERCÍCIOS DE COMBATE
01. DISCURSIVA 04. DISCURSIVA 07. B 10. C
02. DISCURSIVA 05. D 08. D
03. B 06. B 09. C
DESAFIO PRO
01. D 03. C 05. B
02. 220 04. E

ANOTAÇÕES

154

PM_BOOK16 - MAT.indb 154 25/11/2022 19:11:04


BINÔMIO DE NEWTON E
POLINÔMIO DE LEIBNIZ

FATORIAL Somando 2 elementos consecutivos de uma mesma linha obtemos


o elemento localizado abaixo da segunda parcela.
Definimos fatorial de um número natural n, tradicionalmente
denotado por n!, ao número definido indutivamente por: 0! = 1 e Prova: considere o número de comissões de p + 1 pessoas que
n! = n(n – 1)! podemos formar a partir de um grupo de n + 1 pessoas, sabendo que
uma destas n + 1 pessoas é Pedro.
Temos que n!, logo tem-se que 5! = 5 · 4 · 3 · 2 · 1 = 120;
3! = 3 · 2 · 1 = 6; etc... O total dessas comissões é igual a Cpn++11, por outro lado podemos
contar este total separando em dois casos; primeiro caso comissões
que Pedro participa que são Cpn e o segundo caso, as comissões na
COEFICIENTES BINOMIAIS qual Pedro não participa que são Cp−1
n .

DEFINIÇÃO TEOREMA DAS LINHAS


Dados os naturais n e k, sendo n ≥ k chama-se coeficiente A soma dos termos da linha n é igual a 2n.
n
binomial n sobre k e se indica   ao número definido por: Cn0  C1n  Cn2  ...  Cnn1  Cnn  2n.
n n n! k 
se 1 ≤ k ≤ n. Prova: seja o conjunto A = {1,2,3,...,n} o total de subconjuntos
  1 e   
0  k  k !(n  k )! é igual a Cn0  C1n  Cn2  ...  Cnn1  Cnn . Por outro lado, para formar um
subconjunto de A, cada elemento de A tem duas opções, participar
Da definição que para 1 ≤ k ≤ n tem-se: ou não do subconjunto.
n n! n(n  1)(n  2)...(n  k  1) Concluímos que o total de subconjuntos é igual a 2n.
   onde o numerador da
k
  k !(n  k )! k(k  1)(k  2)...1
fração contém k fatoriais.
TEOREMA DAS COLUNAS
Exemplo:
A soma dos elementos de uma coluna do triângulo, começando
(n  1)! no primeiro termo da coluna, é igual ao elemento que está na linha e
Calcule n, sabendo-se que 7.
n! na coluna posteriores ao último elemento da soma.
Solução: Temos que (n + 1)! = (n + 1) · n · (n – 1) · ... · 3 · 2 · 1 =
Cpp  Cpp 1  Cpp  2  ...  Cpp n  Cpp 1n1
(n + 1) · n!
n!(n  1) Prova: considere o conjunto A = {1,2,...,p,...,n+p,n+p+1} com
Logo,  7  n 1 7  n  6 . p +1
n! p < n, o total de subconjuntos de p+1 elementos de A é igual a Cn+p +1.
Por outro lado cada um destes subconjuntos tem um elemento
TRIÂNGULO DE PASCAL que é o máximo (maior elemento do conjunto).
O triângulo de Pascal é um triângulo aritmético formado por • Primeiro caso: p + 1 é o máximo, os outros p elementos são
números que têm muitas relações entre si. Algumas dessas relações escolhidos a partir de {1,2,...,p}.
foram descobertas por Pascal. Total de subconjuntos = Cpp .
Ckn
Onde representa combinação de n escolhe k, n é o número da • Segundo caso: p + 2 é o máximo; agora devemos escolher os
linha e k representa o número da coluna. outros p elementos de p + 1 elementos {1,2,...,p + 1}.
Os quadros abaixo representam o triângulo de Pascal: Total de subconjuntos = Cpp +1.
1 ...
C00
• Caso N: n + p + 1 é o máximo. Agora devemos escolher os outros
C10 C11 1 1
p elementos a partir do conjunto {1,2,3,...,n + p}.
C02 C12 C22 1 2 1 Total de subconjuntos = Cn+p .
p

C03 C13 C23 C33 1 3 3 1 Igualando as duas formas, o teorema está provado.
C04 C14 C24 C34 C44 1 4 6 4 1
C50 C15 C52 C53 C54 C55 1 5 10 10 5 1 OUTRAS PROPRIEDADES
C60 C16 C62 C63 C64 C56 C66 1 6 15 20 15 6 1 a. Cn0  C1n1  Cn2 2  ...  Cnpp  Cpn1p 1 ; teorema das diagonais.
... ...
b. Cpn  Cnnp relação das combinações complementares.
n 1
Cpn  Cpn 1 se p<
RELAÇÃO DE STIFEL c.
2
.

Trata-se de uma propriedade que permite construir o Triângulo de


Pascal dada pela seguinte fórmula: Cpn  Cpn 1  Cpn11.

155

PM_BOOK16 - MAT.indb 155 25/11/2022 19:11:17


BINÔMIO DE NEWTON E POLINÔMIO DE LEIBNIZ

6
2 4
BINÔMIO DE NEWTON Exemplo: Encontrar o coeficiente de x4 na expansão de  − x 
x 
Consideremos a igualdade: Esse talvez seja o tipo de problema que mais iremos encontrar.
(x + a)n = (x + a)(x + a)...(x + a) (n fatores)(1) Devemos primeiro utilizar nossos conhecimentos de propriedades de
potência e arrumar nossos termos a e b do binômio.
Para se formar um termo do produto (x + a)(x + a)...(x + a)
6 6
devemos escolher uma das duas parcelas em cada um dos n fatores x 2 4  1 4
( 2x −1 − x 4 )
6
+ a e efetuar o produto das mesmas.  −x  =2⋅ − x  =
x   x 
Por exemplo, se escolhermos p letras a em p dos n binômios, e
n – p letras x dos n – p binômios restantes, então um termo genérico  n  n −p p
 n  Para utilizarmos na nossa fórmula do termo geral Tp +1 =   ⋅ x ⋅ a
do desenvolvimento de (x + a)n é da forma: a a ... a x x ... x  ap xnp podemos ainda pensar o seguinte p

   
  
com p = 0,1,2,...,n (2).
( 2x ) (2x + ( −x )=
) (2x + ( −1⋅ x ))
p n.p 6 6 6
−1 −1 −1
− x 4= 4 4

O número de termo da forma (2) é, então é igual ao número de


modos de escolhermos p letras a em p dos n binômios x + a, isto é, Cpn. Sendo assim temos a = 2x −1, b =( −1) ⋅ ( x 4 ) e n = 6
Portanto, reduzindo todos os termos da forma a x , encontramos p n-p Pela nossa fórmula
um único termo, Cpnap xn−p. n 6
(
Tp +1=   ⋅ an −p ⋅ bp=   ⋅ ( 2x −1 ) ⋅ ( −1) ⋅ ( x 4 ) )
6 −p p

Finalmente, fazendo p variar de 0 até n, encontramos todos os


p p
termos do desenvolvimento de (x + a)n.
6 6
  ⋅ ( 2) ( x ) ⋅ ( −1)p ( x 4 )=  p  ⋅ 26 −p ⋅ xp −6 ⋅ ( −1)p ⋅ x 4p
6 −p −6 + p p
n
Tp +=
 Cpnap xnp .
1
Então, ( x  a)n  p
   
p0
 6  6 −p  6  6 −p
  ⋅ 2 ⋅ ( −1) ⋅ x ⋅ x =   ⋅ 2 ⋅ ( −1) ⋅ x
p p −6 p 5p − 6
Expandindo o somatório acima, temos: Tp +1 = 4p

p p
n n  n  n1
( x  a)n  xn    a xn1    a2xn2  ...   n
 a x  a (I)
1
  2
   n  1 Nosso termo geral possui como termo algébrico x5p–6 de onde
deveremos igualar 5p – 6 a 4, para termos o que a questão pediu, o
termo x4.
TERMO GERAL DO 5p − 6 = 4 → 5p = 10 → p = 2
DESENVOLVIMENTO DE (x + a)n Sendo p = 2
Todos os termos do desenvolvimento de (x + a)n são obtidos de 6 6 6!
T2 +1 =   ⋅ 26 − 2 ⋅ ( −1) ⋅ x5⋅2 − 6 → T3 =   ⋅ 24 ⋅ 1⋅ x10 − 6 =
2
Cpnap xn−p
quando fazemos neste termo, p variar de 0 a n. ⋅ 16 ⋅ x 4
2
   2 2!⋅ 4!
Por este motivo, Cpnap xn−p é chamado de termo geral.
Chamando o 1º, 2º, 3º, ...termos do desenvolvimento de (x + a)n Então o termo que possui x4 é o 3º Termo do desenvolvimento
respectivamente por T1, T2, T3,..., podemos observar que: (p + 1) e
6 ⋅ 5 ⋅ 4!
Para p = 0 obtemos T1 = Cn0a0 xn T3 = ⋅ 16 ⋅ x 4 = 15 ⋅ 16 ⋅ x 4 = 240x 4
2 ⋅ 4!
Para p = 1 obtemos T2  C1na1xn1
Logo o coeficiente que procuramos é 240.
Para p = 2 obtemos T3  Cn2a2xn 2
Observação
Para p = 3 obtemos T4  Cn3a3xn 3
Toda vez que nos for pedida a soma dos coeficientes de um
... polinômio (uma expansão binomial não deixa de resultar em um
Isto é, a ordem de cada termo é igual à taxa da combinação polinômio) basta fazermos x = 1.
correspondente mais 1. Como a taxa da combinação do termo geral é
Sendo assim, qual a soma dos coeficientes da expansão ( x + 1) ?
10
p, segue-se que este termo é de ordem p + 1. Isto é, Tp 1  Cpnap xnp.
Responder essa pergunta é fácil, pois temos a tendência de achar
Exemplo: Na expansão de ( x + 2)
4
que só podemos respondê-la depois de totalmente expandido o
Pela linha do triângulo de Pascal temos os números binomiais binômio, o que não é verdade.
( x + 1) = x10 + 10x 9 + 45x 8 + 120x7 + 210x6 + 252x5 + 210x 4 + 120x 3 + 45x 2 + 10x + 1
10

1 4 6 4 1
( x + 1) = x10 + 10x 9 + 45x 8 + 120x7 + 210x6 + 252x5 + 210x 4 + 120x 3 + 45x 2 + 10x + 1
10

Mas teremos também a regra dos expoentes aplicada aos termos


Podemos tanto fazer x = 1 do lado direito da igualdade (o que é
do binômio a ser expandido, x e 2.
bem mais trabalhoso porque muitas vezes expandir não será fácil)
1⋅ x 4 ⋅ 20 4 ⋅ x 3 ⋅ 21 6 ⋅ x 2 ⋅ 22 4 ⋅ x1 ⋅ 23 1⋅ x 0 ⋅ 24 como fazer x = 1 no lado esquerdo da igualdade.
Então a soma de todos os coeficientes da expansão de
1⋅ x ⋅ 1
4
4⋅x ⋅2
3
6⋅x ⋅4 2
4⋅x ⋅8
1
1⋅ 1⋅ 16
( x + 1) =(1 + 1) =( 2) =1024.
10 10 10

x4 8x 3 24x 2 32x 16 Agora confira você se 1024 não é realmente a resposta só que
fazendo x = 1 do lado direito da igualdade.
Como nossa expansão é ( x + 2) teremos todos os sinais positivos,
4

Estando preparado para não cair mais em pegadinhas diga pra


assim
mim qual a soma dos coeficientes da expansão de ( 3x − 1) .
7

( x + 2)
4
=x 4 + 8x 3 + 24x 2 + 32x + 16

( x − 2)
4
Se tivéssemos a expansão de iremos ter os sinais
alternantes
POLINÔMIO DE LEIBNIZ
Existe uma generalização do binômio de Newton onde com ele
( x − 2)
4
=x 4 − 8x 3 + 24x 2 − 32x + 16 podemos fazer uma expansão de qualquer quantidade de termos,
chama-se Polinômio de Leibniz.

156

PM_BOOK16 - MAT.indb 156 25/11/2022 19:11:25


BINÔMIO DE NEWTON E POLINÔMIO DE LEIBNIZ

n! 04. (UERN) A soma dos algarismos do termo independente de x no


(x + x 2 + x 3 + … + xp ) =
n α
∑ x1α1x 2α2 … xp p 8
desenvolvimento do binômio de Newton  + x  é
1
α1!α2 !…αp ! 2
x 
Estendendo-se o somatório a todos os valores inteiros não- a) 3 c) 6
negativos de α1, α2 ,…, αp tais que α1 + α2 + … + αp =n b) 4 d) 7

Vamos ver um exemplo prático com um exemplo? Claro que com n

∑  p  = 256, então o valor de n vale


n
3 termos! 05. (MACKENZIE) Sabendo que

Vamos desenvolver ( x 2 − x + 2) .
6 p=0

a) 8 c) 6 e) 4
6! 6!
( x − x + 2) =∑ α !α !⋅ α ! ( x ) ⋅ ( −x ) ⋅ ( 2) =∑ α !α !⋅ α ! ( −1) 2 x
2 α1 α2 α3 b)2α1 +7 d) 5
2 6 α2 α3 α2

1 2 3 1 2 3

! 6! 06. (IFAL) O termo independente no desenvolvimento do binômio


( x 2 ) 1 ⋅ ( − x ) α2
α
⋅ ( 2) =∑ ( −1) 2 x
α3 α2 α3 2α1 + α2
5
!⋅ α3 ! α1!α2 !⋅ α3 !  2 3
 2x − 3  é
 x 
Vamos escolher agora um expoente e ver qual será o coeficiente a) –720. c) 0. e) 720.
que irá acompanhar este termo. b) –360. d) 360.
Por exemplo, vamos escolher o expoente 3.
α1 + α2 + α3 =6 07. (FGV) O coeficiente de x12 na expansão de (1 + x4 + x5)10 é igual a
 a) 120. c) 81. e) 54.
 2α1 + α2 =3
b) 90. d) 60.
Vamos utilizar uma tabela auxiliar
08. (ESPCEX) O coeficiente de x5 no desenvolvimento de (x + 2)9 é:
α1 α3 α3
a) 64 c) 524 e) 2016
0 3 3 b) 126 d) 1024

1 1 4
09. (ESPCEX) O termo independente de x no desenvolvimento de
18
1 4 
Para α1 = 0, α2 = 3 e α3 =3  2 − x é
x 
6! 6! 6 ⋅5⋅ 4
( −1) 2 2α3 x 2α1+ α2 = ( −1) 23 x 2⋅0 + 3 ( −1) ⋅ 8 ⋅ x 3 =a)
α 3
= −160x 3
153 c) 149 e) 361
α1!α2 !⋅ α3 ! 0!3!⋅ 3! 6
6! 6 ⋅5⋅ 4 b) 261 d) 457
( −1) 23 x 2⋅0 + 3 = ( −1) ⋅ 8 ⋅ x 3 =−160x 3
α2 3
=
0!3!⋅ 3! 6 n
  2 
10. (PUCPR) Sabendo que o desenvolvimento de 2x 2 −    possui
Para α1 = 1, α2 = 1 e α3 =4   3x  
6! 6! 7 termos e que um deles é 240 ax6, acharemos para “a” o valor:
( −1) 2 x ( −1) 2 x =⋅ 6 5 ⋅ ( −1) ⋅ 16 ⋅ x =
α2 α3 2α1 + α2 1 4 2⋅1+1
= 3
−480x 3

α1!α2 !⋅ α3 ! 1!1!⋅ 4! a) 4/9 c) 1/9 e) 5/3


6!
( −1) 24 x 2⋅1+1 =⋅
6 5 ⋅ ( −1) ⋅ 16 ⋅ x 3 =
α1 + α2 1
= −480x 3 b) 2/9 d) 2/3
1!1!⋅ 4!
Nosso total é – 160x³ – 480x³ = – 640x³
EXERCÍCIOS DE

EXERCÍCIOS DE
TREINAMENTO
FIXAÇÃO
01. (AFA) O termo independente de x no desenvolvimento de
7
01. (FGV) Desenvolvendo-se o binômio P(x) = (x + 1)5, podemos dizer  4 1  é
x +
 
que a soma de seus coeficientes é  x3 
a) 16 c) 32 e) 48 a) 4 c) 21
b) 24 d) 40 b) 10 d) 35

02. (MACKENZIE) O número de valores de x, para os quais os  1


65

02. Determine o termo máximo do desenvolvimento de 1 + 


6 6  3
coeficientes binomiais   e  2  sejam iguais, é
 2x  x  03. Qual é a soma dos coeficientes do desenvolvimento de (x3 – 2x 2)15?
a) 1 d) 4
b) 2 e) 5 04. (ESPCEX) Determine o algarismo das unidades da seguinte soma
c) 3 2016

S= ∑n!, em que n! é o fatorial do número natural n.


n =1
03. (UECE) No desenvolvimento de x(2x + 1)10 o coeficiente de x³ é
a) 0 c) 2 e) 4
a) 480. c) 260.
b) 1 d) 3
b) 320. d) 180.

157

PM_BOOK16 - MAT.indb 157 25/11/2022 19:11:26


BINÔMIO DE NEWTON E POLINÔMIO DE LEIBNIZ

n
05. No desenvolvimento  x 2 +  , n ∈ , os coeficientes binominais
3 n
 k 
 x 13. (UEPG) No desenvolvimento do binômio  x 2 + 3  , onde n e k
do quarto e do décimo terceiro termos são iguais. Então, o termo são números reais, o 4º termo vale 280x .
7  x
independente de x é o: Nesse contexto, analise as afirmativas.
a) décimo. d) décimo-segundo. (01) n é um número primo.
b) décimo-primeiro. e) oitavo. (02) n + k > 10.
c) nono. (04) O desenvolvimento não tem um termo independente de x.
(08) A soma de seus coeficientes é 81.
06. (FGV) O termo independente de x do desenvolvimento de
12 (16) O coeficiente do 3º termo vale 84.
 1
x + 3  é O somatório das afirmativas corretas é
 x 
a) 26. d) 280. 6

∑  p  x
6
b) 169. e) 310. 14. (AFA) Sendo p(x) =
6 −p
. 2p , a soma das raízes de p(x) é um
c) 220. número do intervalo p=0

a) ]–13,0[ c) ]60,70[
07. (UERN) Qual é o valor do termo independente de x do binômio b) ]11,15[ d) ]–3,3[
n
2 
 2 + x  , considerando que o mesmo corresponde ao sétimo termo
x  15. (AFA) No desenvolvimento de (xr + x-r)n, ordenado pelas potências
de seu desenvolvimento?
decrescentes de x, sendo r > 0 e n natural, o coeficiente de 5º termo
a) 435 c) 543 que é independente de x é igual a:
b) 672 d) 245 a) 252 b) 70 c) 10 d) 8

08. (ESPCEX) O valor da expressão E = (999)5 + 5 · (999)4 + 10 · (999)3 16. (UECE) O coeficiente de x6 no desenvolvimento de
+ 10 · (999)² + 5 · (999) + 1 é igual a 
3
1  2 1
3

a) 9 · 10³ d) 999.999  2x + 2  ⋅  x +  é
 x   2x 
b) 9 · 1015 e) 999.1015 a) 18. c) 34.
c) 10 15
b) 24. d) 30.

09. (UECE) Se n é um número natural maior do que dois, ao 17. Analise as seguintes proposições:
n
ordenarmos o desenvolvimento de  x 2 +  segundo as potências
1
n n n  n 
 2x  (02) O valor de n na igualdade   +   +   + ... +  =510 é
decrescentes de x, verificamos que os coeficientes dos três primeiros igual a 9.  1  2   3   n − 1
termos estão em progressão aritmética. Nessas condições, o valor de (04) A expressão (x + 1)5 – 5(x + 1)4 + 10(x + 1)3 – 10(x + 1)2 + 5(x + 1) – 1
né 5

a) 8. b) 6. c) 4. d) 10. corresponde a ∑  5  ( x + 1)
p=0
5 −p
.( −1)p
.
p
10. (ESPCEX) Determine o valor numérico do polinômio p(x) = x4 + 4x3  
+ 6x2 + 4x + 2017 para x = 89. (08) No desenvolvimento do binômio (x – y)n, a soma de todos os
a) 53 213 009. c) 61 342 008. e) 67 302 100. coeficientes positivos é 256. O valor de n é então igual a 9.
b) 57 138 236. d) 65 612 016. 10   10  11
(16) Se   +  =  , então n é o número 5.
 4   n + 1  4 
11. (UEPG) Analise as afirmativas. A soma dos números que correspondem às proposições verdadeiras
(n + 4)!− 20(n + 2)! é igual a
(01) Simplificando a expressão obtém-se n – 1.
(n + 8) ⋅ (n + 2)! a) 06 c) 20
4
 a b) 14 d) 22
(02) No desenvolvimento do binômio  3x +  , o termo independente
27 1  x
de x é . Então a2 = . 18. No desenvolvimento do binômio (x + p · y)n, com p e n naturais,
2 4
o termo 112x6y2 é o terceiro quando feito com potências crescentes
(04) Permutando os algarismos 1, 1, 3, 3, 3, 5 podem ser formados 20
de y e o sétimo quando feito com potências crescentes de x. O valor
números maiores que 500.000.
de p + n é igual a:
 20   20   20   20  a) 10 c) 9 e) 13
(08)   +   +   +  +   = 220 − 211.
3 4
      5  20  b) 12 d) 11
(16) Num estádio há 12 portas de entrada e saída. Existem 132
possibilidades de uma pessoa entrar por uma porta e sair por outra  (x − 1)(5x − 7) 
diferente. 19. (EN) Seja m a menor raiz inteira da equação 
 3 ! = 1.
O somatório das afirmativas corretas é Pode-se afirmar que o termo médio do desenvolvimento de
( y − z3 )12m é
7
2  12! 3 18
12. (EN) O coeficiente de x5 no desenvolvimento de  + x 3  é 12! 18 2
3
30! 2 45
15
e)
x  a) y z c) y z yz
6!6! 15!15! 6!6!
a) 30 c) 120 e) 560
−12! 3 18 −30! 2 45
15
b) 90 d) 270 b) yz d) y z
6!6! 15!15!

158

PM_BOOK16 - MAT.indb 158 25/11/2022 19:11:28


BINÔMIO DE NEWTON E POLINÔMIO DE LEIBNIZ

20. Determine o coeficiente de x4 no desenvolvimento de (x2 – x + 2)6. 06. (ESPCEX) O termo independente de x no desenvolvimento de
10
 3 1  é igual a
21. Qual o coeficiente de x7 na expansão de (2 + 3x + x²)4? x  2 
 x 
a) 18 b) 16 c) 14 d) 12 e) 10
a) 110 c) 310 e) 510

22. O coeficiente de x na expansão de (1 + x + x ) é igual a


12 4 5 10 b) 210 d) 410
9
a) 120. c) 81. e) 54.  k 
07. (ESPCEX) No desenvolvimento do binômio  x 
2
 , o termo
b) 90. d) 60.  x4 
independente de x é igual a 672. Então k é um número
23. (AFA) O menor dos possíveis coeficientes do termo em x8, no a) primo. d) inteiro quadrado perfeito.
desenvolvimento de (2 + x2 + 3x3)10 é igual a
b) divisível por 3. e) inteiro cubo perfeito.
a) 11.240 c) 13.440
c) múltiplo de 5.
b) 12.420 d) 14.720
08. (EPCAR 3º ANO) No desenvolvimento de (a + b)6, segundo as
24. (EN) O par ordenado (x,y) de números reais, x ≠ 0 e y ≠ 0, satisfaz potências decrescentes de a, a razão do coeficiente binomial de certo
1 1 3
 x + y =
termo para o termo seguinte é 4/3. Então a posição do 1º desses
4 termos é
ao sistema  em que x é o menor elemento do par. Se
 +1 1 5 a) 1ª c) 3ª
=
 x
2 2
y 16
b) 2ª d) 4ª
p = 3x + y, encontre o termo de ordem (p + 1) do binômio
15
 x 2z 
5 − y 2  e assinale a opção correta. 09. (EPCAR 3ª SÉRIE) Se n é o número de termos do desenvolvimento
 143 
 
55
5
de x  10 y que não contém radical, então n vale:
a) –21x10z5y20 c) –21x10z5y10 e) 2x10z5y20
a) 4 c) 6
b) 21x z y
5 10 20
d) 21x z y 32 10 20

b) 5 d) 7
25. (ITA) O termo independente de x no desenvolvimento do binômio
 33 x
12 10. (EPCAR 3º ANO) De acordo com o triângulo de Pascal pode-se
5x 
 −3  é afirmar que
 5x 3 x 
  a  2   a  1  a  1
a) 729 453
5
a)     em que a > b + 3.
d) 3 376    b  1  b   b  1
b) 972 3 15  3
 m   m 
e) 165 3 75 b)    somente para m = 2p.
 3
c) 891  
3  p  q   p  1
5 10
10 
c)    1024
i 1  i 

EXERCÍCIOS DE  c  1  c  1  c  2 
d)     em que c ≥ d + 2.

COMBATE  d  2  d  3  d  3

8
1 1

DESAFIO PRO
01. Calcule o 5º termo do desenvolvimento de:  x 2y  
2 x

02. (AFA 1994) No desenvolvimento (x2 + 3x)12, o coeficiente de x20 é:

1
a) 32 ⋅ 110 c) 35 ⋅ 110 (ITA) O coeficiente de x4y4 no desenvolvimento de
b) 36 ⋅ 55 d) 35 ⋅ 55 (1 + x + y)10 é
a) 3150 d) 81900
7
2 3 b) 6300 e) 151200
03. (EN 2013) O coeficiente de x5 no desenvolvimento de   x  é:
x  c) 75600
a) 30 c) 120 e) 560
b) 90 d) 270

(n  2)! (n  1)!
2 (ITA) Para os inteiros positivos k e n, com k ≤ n,
sabe-se que
n + 1  n   n + 1
 =  . Então, o valor de
04. Simplifique: k + 1  k   k + 1
(n  2)! (n  1)!
n 1 n 1 n 1 n
 +   +   + ... +   é igual a
0
  2 1
  3 2
  n + 1 n
05. (ITA 2010) A expressão  2 3  5    2 3  5  é igual a:
5 5
a) 2n + 1. n +1
d) 2 − 1.
a) 2630 5 d) 1584 15 b) 2n+1 + 1. n+1
b) 2690 5 e) 1604 15 n
c)
2n +1 + 1
. e) 2 − 1.
c) 2712 5 n n

159

PM_BOOK16 - MAT.indb 159 25/11/2022 19:11:35


BINÔMIO DE NEWTON E POLINÔMIO DE LEIBNIZ

3 (IME) O valor da soma abaixo é:


 2016   2017   2018   2019   2020   2016 
ANOTAÇÕES
 + + + + + 
 5   5   5   5   5   6 

 2020 
a)  
 6 
 2020 
b)  
 7 
 2021
c)  
 5 
 2021
d)  
 6 
 2022 
e)  
 5 
10
 1 
4 (IME) No desenvolvimento de  x ⋅ sen 2β + cos 2β 
 x
o valor do termo independente de x é igual a 63/256.

Considerando que β é um número real, com 0 < β < π/8 e x ≠ 0,


o valor de β é:
a) π/9
b) π/12
c) π/16
d) π/18
e) π/24

5 (ITA) Sejam a e b números inteiros positivos. Se a e b são,


nessa ordem, termos consecutivos de uma progressão
12
e o termo independente de  ax −
1 b 
geométrica de razão 
2  x
é igual a 7.920, então a + b é
a) 2. d) 5.
b) 3. e) 6.
c) 4.

GABARITO
EXERCÍCIOS DE FIXAÇÃO
01. C 04. B 07. A 10. A
02. B 05. A 08. E
03. D 06. E 09. A
EXERCÍCIOS DE TREINAMENTO
01. D 08. C 15. B 22. A
02. 17 09. A 16. B 23. C
03. –1 10. D 17. B 24. E
04. D 11. SOMA: 27 18. A 25. E
05. B 12. E 19. E
06. C 13. SOMA: 17 20. 780
07. B 14. A 21. D
EXERCÍCIOS DE COMBATE
01. DISCURSIVA 04. DISCURSIVA 07. A 10. D
02. B 05. B 08. D
03. E 06. B 09. C
DESAFIO PRO
01. A 03. D 05. B
02. D 04. E

160

PM_BOOK16 - MAT.indb 160 25/11/2022 19:11:36


PROBABILIDADE

PROBABILIDADE TEOREMA - PROBABILIDADE DO


Consideremos um experimento com resultados imprevisíveis e EVENTO COMPLEMENTAR
mutuamente exclusivos, ou seja, cada repetição desse experimento P(AC) = 1 – P(A)
é impossível prever com certeza qual o resultado que será obtido,
e além disso, a ocorrência de um deles exclui os demais. É o caso
do lançamento de uma moeda, cujos possíveis resultados são: cara,
coroa; ou então, o lançamento de um dado, com resultados: 1, 2, 3, PROBABILIDADE CONDICIONAL
4, 5, 6. Todo experimento desta natureza chama-se de experimento Para iniciar o conceito do que é probabilidade condicional, vamos
aleatório, e seus possíveis resultados, mutuamente exclusivos, são considerar o seguinte problema.
chamados de eventos simples. Suponha que um redator de um jornal recebeu 120 cartas de
Diremos que um é determinístico quando repetido em condições leitores sobre uma demissão polêmica de um professor universitário,
iguais conduz a resultados idênticos. Os experimentos que repetidos mas o redator só pode publicar uma destas cartas, que será escolhida
sob as mesmas condições produzem resultados geralmente diferentes ao acaso.
são chamados experimentos aleatórios. As cartas foram escritas por alunos ou por pais, algumas apoiam
O espaço amostral de um experimento aleatório é o conjunto de o reitor que demitiu o professor, outras apoiam o professor, e estão
todos os seus eventos simples. No caso do lançamento do dado, o dividias conforme o quadro abaixo:
espaço é igual a U = {1,2,3,4,5,6}.
Os elementos do espaço amostral são chamados eventos ESTUDANTES PAIS
elementares. Os subconjuntos do espaço amostral serão chamados
Apoiam o professor 16 44
eventos. Por exemplo, o subconjunto A = {2,3,5} é o evento que
ocorre se o número mostrado na face de cima é primo. Apoiam o reitor 8 52
Para calcular a probabilidade de um evento A, iremos considerar
o caso do evento A = {2,3,5} do nosso exemplo. É claro intuitivamente A situação pode também ser representada pelo diagrama de
que se repetimos o experimento um grande número de vezes Venn, onde E representa o conjunto das cartas que foram escritas
obteremos um número primo em aproximadamente a metade dos pelos estudantes e P representa o conjunto de cartas que foram
casos. Se os eventos elementares são todos equiprováveis e se o escritas em apoio ao professor.
número de elementos de A é a metade dos elementos do espaço
amostral.
A probabilidade do evento A, será calculada da seguinte forma:
#(A) 3 1
probabilidade de A= = = .
#( Ω) 6 2
Suponha que um experimento aleatório tem as seguintes
características:
a) Há um número finito (digamos n) de eventos elementares
(casos possíveis). A união de todos os eventos elementares é
o espaço amostral U.
b) Os eventos elementares são igualmente prováveis.
c) Todo evento A é uma união de m eventos elementares onde Como podemos ver, a probabilidade de retirar uma carta escrita
m ≤ n. 16 2
por um estudante que apoia o professor é P (E ∩ P )= = .
Definimos, então: 120 15
número de casos favoráveis Para calcular qual a probabilidade de uma carta apoiar o professor dado
Probabilidade de = =
A P(A) 16
número de casos possíveis
#(A) m P (P ∩ E ) 120 16
= = . que foi escrita por um estudante, temos P (P= | E) = = ,
#(U) n P (E ) 24 24
n° de elementos de P ∩ E 16
=
São consequências imediatas desta definição as seguintes ou P (P | E ) = . 120
n° de elementos de E 24
propriedades:
Generalizando a partir do nosso exemplo, vamos definir
1. Para todo evento A, 0 ≤ P(A) ≤ 1; probabilidade condicional.
2. P(U) = 1; Se A e B são eventos de um espaço amostral S e P(B) ≠ 0, então a
3. P(∅) = 0 (porque #(∅) = 0; probabilidade do evento A ocorrer dado que ocorreu B é representada
4. Se A ∩ B = ∅ então P(A ∪ B) = P(A) + P(B). P ( A ∩ B)
P (A|B) e é dada por P ( A | B) = , P(B) ≠ 0.
5. Se A ∩ B ≠ ∅ então P(A ∪ B) = P(A) + P(B) – P(A ∪ B). P (B)

161

PM_BOOK16 - MAT.indb 161 25/11/2022 19:11:37


PROBABILIDADE

REGRA DO PRODUTO DE 05. Dentre os números inteiros de 1 a 50, um número é escolhido


aleatoriamente. Qual a probabilidade de ele divisível por 5?
PROBABILIDADES a) 1/50 b) 1/5 c) 1/2 d) 3/4
Dois eventos A e B são independentes quando a ocorrência, ou
não ocorrência de um deles não influi na ocorrência do outro, isto é: 06. A probabilidade de um jogador de futebol marcar o gol ao
P(A|B) = P(A) cobrar um pênalti, é de 80%. Se esse jogador cobrar dois pênaltis
ou ainda, consecutivos, a probabilidade dele fazer o gol, em ambas as cobranças,
é igual a:
P(A ∩ B) = P(A) · P(B)
a) 16% c) 32% e) 80%
b) 20% d) 64%
PROPOSIÇÕES
Sejam A, B e C eventos de algum espaço amostral S. Então, 07. Um aluno da ESA tem uma habilidade muito boa nas provas de
• P(∅ | A) = 0, P(S | A) = 1 e 0 ≤ P(B | A) ≤ 1 tiro com pistola, possuindo um índice de acerto no alvo de quatro em
• P((B ∪ C) | A) = P(B | A) + P(C | A), se B ∩ C = ∅ cada cinco tiros. Se ele atirou duas vezes, a probabilidade de que ele
tenha errado os dois tiros é:
a) 16/25 c) 1/5 e) 1/25
TEOREMA DE BAYES b) 8/25 d) 2/5
p

Sejam A1, A2, ..., Ap eventos disjuntos e B ⊂


 A ,então
i =1
i
08. Com os dígitos 1, 2, 3, 4 e 5 são formados números de 4
p algarismos distintos. Um deles é escolhido ao acaso. A probabilidade

P(Ak ) . P(B | Ak )
P(B) = P(Ai ).P(B | Ai ) e, P(Ak | B) = . desse número ser par é:
P(A1) . P(B | A1) + ... + P(Ap ) . P(B | Ap )
i =1
a) 1/3 c) 3/5 e) n.r.a.
b) 2/5 d) 2/3
DISTRIBUIÇÃO BINOMINAL
A probabilidade de ocorrerem exatamente k sucessos em uma 09. Uma urna contém 3 bolas verdes e 4 amarelas. Ao retirar, sem
sequência de n provas independentes, na qual a probabilidade de reposição, duas bolas, a probabilidade delas serem amarelas é
n a) 2/7 b) 3/7 c) 4/7 d) 6/7
sucesso em cada prova é p, igual a   pk (1 − p ) .
n −k

 k
Exemplo:
10. No lançamento de 2 dados, qual a probabilidade de que a soma
Jogamos uma moeda não viciada 10 vezes. Qual é a probabilidade dos dois seja 8, supondo que no 1º resultado tenha sido menor que 5?
de obtermos exatamente 5 caras?
a) 10% b) 12,5% c) 14% d) 16,5%
Solução:
Pondo sucesso = cara, temos p = 1/2 em cada prova e as provas
são independentes. Queremos achar a probabilidade de k = 5 sucessos EXERCÍCIOS DE
em n = 10 provas. Pelo teorema binominal, a resposta é
10   1 
  
5
 1
5

1 −  =
252
=
63
.
TREINAMENTO
5   2   2  1024 256

01. (EEAR) Uma urna contém bolas verdes e azuis. Sabe-se que a
6
EXERCÍCIOS DE probabilidade de se retirar uma bola azul é de . A probabilidade de

FIXAÇÃO
11
ser retirada, em uma única tentativa, uma bola verde é de
1 2 4 5
a) b) c) d)
11 11 11 11
01. Dois dados são lançados simultaneamente. Qual a probabilidade 02. (EFOMM) Um atirador, em um único tiro, tem probabilidade de
da soma ser menor do que 4? 80% de acertar um específico tipo de alvo. Num exercício ele dá seis
a) 1/6 b) 1/8 c) 1/12 d) 1/16 tiros seguidos nesse mesmo tipo de alvo. Considerando-se que os tiros
são independentes, em cálculo aproximado, qual é a probabilidade de
o atirador errar o alvo exatamente duas vezes?
02. Um número inteiro é escolhido ao acaso entre 1 e 20 inclusive.
Qual a probabilidade de o número escolhido ser um quadrado a) 4,12% c) 24,58% e) 40,25%
perfeito? b) 18,67% d) 27,29%
1 1 3 1
a) b) c) d) 03. (EFOMM) Considere uma urna contendo cinco bolas brancas, duas
20 10 20 5
pretas e três verdes. Suponha que três bolas sejam retiradas da urna,
03. Uma urna contém bolas verdes e azuis. Sabe-se que a probabilidade de forma aleatória e sem reposição. Em valores aproximados, qual é a
de se retirar uma bola azul é de 6/11 A probabilidade de ser retirada, probabilidade de que as três bolas retiradas tenham a mesma cor?
em uma única tentativa, uma bola verde é de
a) 7,44% c) 9,17% e) 27,51%
a) 1/11 b) 2/11 c) 4/11 d) 5/11
b) 8,33% d) 15,95%

04. Jogando-se um dado comum de seis faces e não viciado, a


04. (EFOMM) Um garoto dispõe de um único exemplar de cada
probabilidade de ocorrer um número primo e maior que 4 é de
poliedro de Platão existente. Para brincar, ele numerou cada vértice,
a) 1/3 c) 1/6 e) 5/6 face e aresta de cada poliedro sem repetir nenhum número. Em
b) 1/2 d) 2/3 seguida, anotou esses números no próprio poliedro. Se ele sortear um

162

PM_BOOK16 - MAT.indb 162 25/11/2022 19:11:38


PROBABILIDADE

dos números usados, aleatoriamente, qual será a probabilidade de o proibidos, bancas de alguns desses jogos são comumente encontradas
número sorteado representar um vértice? em festas populares Brasil afora.
5 1 1 Exemplo desses jogos é aquele em que o jogador tem 1 bolinha para
a) c) e)
9 3 10 lançar sobre uma rampa, levemente inclinada, e deverá acertar uma
das “casinhas” numeradas de 1 a 6. Geralmente, o dono da banca de
5 5
b) d) jogo impõe condições para que o jogador ganhe um prêmio.
14 19
Suponha que uma condição de sorte seja, desconsiderando quaisquer
05. (EFOMM) Um programa de auditório tem um jogo chamado outras influências, lançar a bolinha três vezes sucessivas de modo
“Porta Premiada”, que funciona da seguinte maneira: que, ao final dos três lançamentos, seja observado que a soma dos
1º. há três portas: uma tem prêmios e duas estão vazias; números das casinhas é igual a 12. Desse modo, a probabilidade de se
ter sorte nesse jogo é
2º. o apresentador pede ao convidado que escolha uma das portas;
a) menor que 3%.
3º. após a escolha, o apresentador abre uma das duas portas não
escolhidas. Como ele sabe qual é a premiada, abre uma vazia; b) maior que 8% e menor que 10%.
4º. depois de aberta uma das portas, ele pergunta ao convidado se c) maior que 11% e menor que 13%.
deseja trocar de porta; d) superior a 13%.
5º. finalmente, abre a porta do convidado para verificar se ganhou
ou perdeu. 10. (AFA) Durante o desfile de Carnaval das escolas de samba do
Analisando o jogo de forma puramente probabilística, verifique qua(l) Rio de Janeiro em 2017, uma empresa especializada em pesquisa de
(is) das estratégias abaixo tem a maior probabilidade de vencer o jogo. opinião entrevistou 140 foliões sobre qual agremiação receberia o
prêmio de melhor do ano que é concedido apenas a uma escola de
I. Após escolher a porta, não trocá-la até o final do jogo. samba. Agrupados os resultados obtidos, apresentaram-se os índices
II. Todas as probabilidades são iguais; não há estratégia melhor que conforme o quadro a seguir:
a outra, ou seja, tanto faz trocar ou não a porta.
III. A melhor estratégia é sempre trocar a porta. Agremiação
A B C AeB AeC BeC A, B e C
escolhida
Sobre as estratégias I, II e III apresentadas, é correto afirmar que
a) somente a alternativa I está correta. Nº de foliões
77 73 70 20 25 40 5
que escolheram
b) somente a alternativa II está correta.
c) somente a alternativa III está correta. A respeito dos dados colhidos, analise as proposições a seguir e
d) nenhuma alternativa está correta. classifique-as em V (VERDADEIRA) ou F (FALSA).
e) todas as alternativas apresentam circunstâncias com a mesma ( ) Se A for a agremiação vencedora em 2017 e se um dos foliões que
probabilidade de vencer. opinaram for escolhido ao acaso, então a probabilidade de que ele
NÃO tenha votado na agremiação que venceu é igual a 45%.
06. (EFOMM) Um atleta de tiro ao prato tem probabilidade de 0,9 ( ) Escolhido ao acaso um folião, a probabilidade de que ele tenha
de acertar o prato a cada novo lançamento. Analisando esse jogador indicado exatamente duas agremiações é de 50%.
antes do início da competição, após quantos lançamento de pratos, a ( ) Se a agremiação B for a campeã em 2017, a probabilidade de que
probabilidade de ele não ter acertado todos os tiros se tornará maior o folião entrevistado tenha indicado apenas esta como campeã é
que a probabilidade de acertar todos? menor que 10%.
a) 9 c) 7 e) 5 A sequência correta é
b) 8 d) 6 a) V – V – F c) F–V–F
b) F – V – V d) V – F – V
07. (EFOMM) Seis alunos da EFOMM – três paranaenses, dois cariocas
e um alagoano – são colocados em uma fila aleatoriamente. Qual é a 11. (AFA) Num auditório da Academia da Força Aérea estão presentes
probabilidade, então, de que nenhum conterrâneo fique ao lado do 20 alunos do Curso de Formação de Oficiais Aviadores dos quais
outro? apenas 10 usam agasalho. Estão presentes, também, 25 alunos do
3 1 1 Curso de Formação de Oficiais Intendentes dos quais apenas 15 usam
a) c) e)
31 24 6 agasalho. Um dos alunos presentes é escolhido ao acaso. É correto
1 1 2
b) d) afirmar que é igual a a probabilidade de que o aluno escolhido
36 12 9
a) seja do Curso de Formação de Oficiais Intendentes ou use
08. (EFOMM) Um dado cúbico, não viciado, com faces numeradas de agasalho.
1 a 6, é lançado três vezes. Em cada lançamento, anota-se o número b) use agasalho, sabendo que é do Curso de Formação de Oficiais
obtido na face superior do dado, formando-se uma sequência (a,b,c). Intendentes.
Qual é a probabilidade de que b seja sucessor de a e que c seja
c) seja do Curso de Formação de Oficiais Aviadores que não use
sucessor de b ou que a, b e c sejam primos?
agasalho.
4 108 10
a) c) e) d) não use agasalho, sabendo que é do Curso de Formação de
216 216 216 Oficiais Aviadores.
27 31
b) d)
216 216 12. (AFA) Em uma mesa há dois vasos com rosas. O vaso A contém 9
rosas das quais 5 tem espinhos e o vaso B contém 8 rosas sendo que
09. (AFA) Pela legislação brasileira, atualmente, os ditos “Jogos
exatamente 6 não tem espinhos.
de Azar” estão proibidos. Tais jogos são, na maioria das vezes,
sustentados pelas perdas dos jogadores que financiam os que vão ter Retira-se, aleatoriamente, uma rosa do vaso A e coloca-se em B. Em
sorte. Esses jogos têm por condição de existência que, na diferença seguida, retira-se uma rosa de B. A probabilidade de essa rosa retirada
entre as probabilidades de sorte e azar, predomine o azar. Ainda que de B ter espinhos é

163

PM_BOOK16 - MAT.indb 163 25/11/2022 19:11:39


PROBABILIDADE

8 15 18 23 Com base nos dados registrados nesse gráfico, é correto afirmar que,
a) b) c) d) escolhido um aluno ao acaso, a probabilidade de ele ter 20 anos ou
81 81 81 81
21 anos é igual a
a) 20% b) 25% c) 30% d) 35%
13. (AFA) Um jogo é decidido com um único lançamento do dado cuja
planificação está representada abaixo. 17. (AFA) Considere que:
I. Em uma urna encontram-se p bolas vermelhas e q bolas azuis;
II. Duas bolas são retiradas dessa urna, sucessivamente e com reposição.
Sabe-se que x é a variável que indica o número de bolas azuis
observadas com as retiradas, cuja distribuição de probabilidade está
de acordo com a tabela a seguir.
___________________
x 0 1 2

P(x) 0,36 0,48 0,16


Nessas condições, é correto afirmar que
Participam desse jogo quatro pessoas: Carlos, que vencerá o jogo se
ocorrer face preta ou menor que 3; José vencerá se ocorrer face branca a) a probabilidade de se observar no máximo uma bola azul é 64%;
e número primo; Vicente vencerá caso ocorra face preta e número b) se p = 6, então q = 9;
par; Antônio vencerá se ocorrer face branca ou número menor que 3.
c) se p = 18, então q = 12;
Nessas condições, é correto afirmar que
d) p + q é necessariamente menor ou igual a 100.
a) Vicente não tem chance de vencer.
b) Carlos tem, sozinho, a maior probabilidade de vencer. 18. (ESPCEX) Enrico guardou moedas em um cofrinho por um certo
c) a probabilidade de José vencer é o dobro da de Vicente. período de tempo e, ao abri-lo, constatou que:
d) a probabilidade de Antônio vencer é maior do que a de Carlos. I. o cofrinho contém apenas moedas de R$ 0,25, R$ 0,50 e R$ 1,00.
II. a probabilidade de retirar uma moeda de R$ 0,25 é o triplo da
14. (AFA) Distribuiu-se, aleatoriamente, 7 bolas iguais em 3 caixas probabilidade de retirar uma moeda de R$ 0,50.
diferentes. Sabendo-se que nenhuma delas ficou vazia, a probabilidade
III. se forem retiradas 21 moedas de R$ 0,25 desse cofrinho, a
de uma caixa conter, exatamente, 4 bolas é
9
a) 25% probabilidade de retirar uma moeda de R$ 0,50 passa a ser .
40
b) 30% IV. se forem retiradas 9 moedas de R$ 0,50 desse cofrinho, a
c) 40% 1
probabilidade de retirar uma moeda de R$ 1,00 passa a ser .
4
d) 48%
Diante dessas constatações, podemos afirmar que a quantidade de
15. (AFA) Um dado cúbico tem três de suas faces numeradas com moedas de R$ 0,25 nesse cofrinho era
“0”, duas com “1” e uma com “2”. Um outro dado, tetraédrico, tem a) 27. c) 33. e) 108.
duas de suas faces numeradas com “0”, uma com “1” e uma com b) 32. d) 81.
“2”. Sabe-se que os dados não são viciados.
Se ambos são lançados simultaneamente, a probabilidade de a soma 19. (ESPCEX) Em uma população de homens e mulheres, 60% são
do valor ocorrido na face superior do dado cúbico com o valor ocorrido mulheres, sendo 10% delas vegetarianas. Sabe-se, ainda, que 5%
na face voltada para baixo no tetraédrico ser igual a 3 é de dos homens dessa população também são vegetarianos. Dessa forma,
a) 12,5% c) 37,5% selecionando-se uma pessoa dessa população ao acaso e verificando-
se que ela é vegetariana, qual é a probabilidade de que seja mulher?
b) 16,6% d) 67,5%
a) 50% c) 75% e) 85%
16. (AFA) Suponha que a distribuição das idades dos cadetes do 1º b) 70% d) 80%
ano da Academia da Força Aérea no ano de 2011 esteja representada
pelo gráfico seguinte. 20. (ESPCEX) A probabilidade de um casal ter um filho de olhos azuis
1
é igual a . Se o casal pretende ter 4 filhos, a probabilidade de que no
3
máximo dois tenham olhos azuis é
1 8 1
a) c) e)
9 9 2
7 2
b) d)
9 3
21. (ESPCEX) De uma caixa contendo 50 bolas numeradas de 1 a
50 retiram-se duas bolas, sem reposição. A probabilidade do número
da primeira bola ser divisível por 4 e o número da segunda bola ser
divisível por 5 é
12 59 11
a) . c) . e) .
245 2450 545
14 59
b) . d) .
245 1225

164

PM_BOOK16 - MAT.indb 164 25/11/2022 19:11:40


PROBABILIDADE

22. (ESPCEX) Se escolhermos, ao acaso, um elemento do conjunto 30. (EN) Há 10 postos de gasolina em uma cidade. Desses 10,
dos divisores inteiros positivos do número 360, a probabilidade de exatamente dois vendem gasolina adulterada. Foram sorteados
esse elemento ser um número múltiplo de 12 é: aleatoriamente dois desses 10 postos para serem fiscalizados. Qual
1 1 3 é a probabilidade de que os dois postos infratores sejam sorteados?
a) c) e)
2 3 8 1 1 1
a) c) e)
3 2 45 15 30
b) d)
5 3 1 2
b) d)
90 45
23. (ESPCEX) A probabilidade de se obter um número divisível por 2
31. (EN) Uma caixa contém 4 pistolas e 4 fuzis, sendo uma pistola e 2
na escolha ao acaso de uma das permutações dos algarismos 1, 2, 3,
fuzis defeituosos. Duas armas são retiradas da caixa sem reposição. A
4, 5 é
probabilidade de pelo menos uma arma ser defeituosa ou ser pistola
1 3 1 é igual a
a) c) e)
5 4 2 27 6 5
a) c) e)
2 1 28 7 7
b) d)
5 4 13 11
b) d)
24. (ESPCEX) Pesquisas revelaram que, numa certa região, 4% dos 14 14
homens e 10% das mulheres são diabéticos. Considere um grupo 32. (EN) Considere como espaço amostral (Ω), o círculo no plano xy
formado por 300 homens e 700 mulheres dessa região. Tomando-se de centro na origem e raio igual a 2. Qual a probabilidade do evento
ao acaso uma pessoa desse grupo, a probabilidade de que essa pessoa A = {(x,y) ∈ Ω ||x| + |y| < 1}?
seja diabética é
2 1 e) π
a) 4% c) 5,4% e) 8,2% a) c)
π π
b) 5% d) 7,2% 1
b) 4π d)

25. (ESPCEX) Se forem tomadas ao acaso duas arestas de um prisma
reto de bases triangulares, a probabilidade de que elas estejam em 33. (FUVEST) Uma seta aponta para a posição zero no instante inicial.
retas-suporte reversas é A cada rodada, ela poderá ficar no mesmo lugar ou mover‐se uma
1 1 1 unidade para a direita ou mover‐se uma unidade para a esquerda,
a) c) e) cada uma dessas três possibilidades com igual probabilidade.
3 6 2
2 1
b) d)
3 4
26. (EN) Um exame de laboratório tem eficiência de 90% para detectar
uma doença quando essa doença existe de fato. Entretanto, o teste
aponta um resultado “falso positivo” (o resultado indica doença, mas Qual é a probabilidade de que, após 5 rodadas, a seta volte à posição
ela não existe) para 1% das pessoas sadias testadas. Se 1,5% da inicial?
população tem a doenças, qual a probabilidade de uma pessoa ter a
1 1 125
doença dado que seu exame foi positivo? a) c) e)
9 3 243
95 270 73
a) c) e) 17 51
294 467 255 b) d)
81 125
160 75
b) d)
433 204 34. (FUVEST) Em uma urna, há bolas amarelas, brancas e vermelhas.
Sabe-se que:
27. (EN) Um atirador, em um único tiro, tem probabilidade de 80%
I. A probabilidade de retirar uma bola vermelha dessa urna é o
de acertar um específico tipo de alvo. Se ele realiza seis tiros seguidos
dobro da probabilidade de retirar uma bola amarela.
nesse mesmo tipo alvo, considerando-se que os tiros são realizados de
forma independente, qual a probabilidade de o atirador errar o alvo II. Se forem retiradas 4 bolas amarelas dessa urna, a probabilidade
duas vezes? 1
de retirar uma bola vermelha passa a ser .
a) 4,12% c) 40,25% e) 18,67% 2
III. Se forem retiradas 12 bolas vermelhas dessa urna, a probabilidade
b) 24,58% d) 27,29% 1
de retirar uma bola branca passa a ser .
2
28. (EN) Considere uma urna contendo cinco bolas brancas, duas A quantidade de bolas brancas na urna é
pretas e três verdes. Suponha que três bolas sejam retiradas da urna, a) 8. c) 12. e) 16.
de forma aleatória e sem reposição. Qual é, aproximadamente, a b) 10. d) 14.
probabilidade de que as três bolas retiradas tenham a mesma cor?
a) 9,17% c) 7,44% e) 8,33% 35. (FUVEST) Cláudia, Paulo, Rodrigo e Ana brincam entre si de
b) 27,51% d) 15,95% amigo-secreto (ou amigo-oculto). Cada nome é escrito em um pedaço
de papel, que é colocado em uma urna, e cada participante retira um
29. Considere os conjuntos A = {0, 1, 2, 3,4} e B = {1,2,3,4,5,6,7,8,9,10}. deles ao acaso.
Seja F o conjunto de funções cujo domínio é A e cujo contradomínio A probabilidade de que nenhum participante retire seu próprio nome é
B. Escolhendo-se ao acaso uma função f de F, a probabilidade de f ser 1 1 5
a) c) e)
estritamente crescente ou ser injetora é? 4 3 12
a) 0,00252 c) 0,25200 e) 0,55440 7 3
b) d)
b) 0,00462 d) 0,30240 24 8

165

PM_BOOK16 - MAT.indb 165 25/11/2022 19:11:42


PROBABILIDADE

36. (FUVEST) Em um experimento probabilístico, Joana retirará EXERCÍCIOS DE


aleatoriamente 2 bolas de uma caixa contendo bolas azuis e bolas
vermelhas. Ao montar-se o experimento, colocam-se 6 bolas azuis na
caixa.
COMBATE
Quantas bolas vermelhas devem ser acrescentadas para que a
1 01. (ITA) Retiram-se 3 bolas de uma urna que contém 4 bolas verdes,
probabilidade de Joana obter 2 azuis seja ?
3 5 bolas azuis e 7 bolas brancas. Se P1 é a probabilidade de não sair
a) 2 c) 6 e) 10
bola azul e P2 é a probabilidade de todas as bolas saírem com a mesma
b) 4 d) 8 cor, então a alternativa que mais se aproxima de P1 + P2 é
a) 0,21. c) 0,28. e) 0,40.
37. (FUVEST) Considere todos os pares ordenados de números
b) 0,25. d) 0,35.
naturais (a,b), em que 11 ≤ a ≤ 22 e 43 ≤ b ≤ 51. Cada um desses
pares ordenados está escrito em um cartão diferente. Sorteando-se
um desses cartões ao acaso, qual é a probabilidade de que se obtenha 02. (ITA) Considere o conjunto D = {n ∈ N; 1 ≤ n ≤ 365} e
um par ordenado (a,b) de tal forma que a fração a/b seja irredutível e H ⊂ P(D) formado por todos os subconjuntos de D com 2 elementos.
com denominador par? Escolhendo ao acaso um elemento B ∈ H, a probabilidade de a soma
de seus elementos ser 183 é
7 6 5
a) c) e) a) 1/730 c) 1/365 e) 91/730
27 27 27
11 b) 46/33215 d) 92/33215
13
b) d)
54 54
03. (ITA) Considere uma população de igual número de homens e
38. (FUVEST) Um dado cúbico, não viciado, com faces numeradas de mulheres, em que sejam daltônicos 5% dos homens e 0,25% das
1 a 6, é lançado três vezes. Em cada lançamento, anota-se o número mulheres. Indique a probabilidade de que seja mulher uma pessoa
obtido na face superior do dado, formando-se uma sequência (a, b, daltônica selecionada ao acaso nessa população.
c). Qual é a probabilidade de que b seja sucessor de a ou que c seja a) 1/21 b) 1/8 c) 3/21 d) 5/21 e) 1/4
sucessor de b?
4 7 23 04. (ITA) Um palco possui 6 refletores de iluminação. Num certo
a) c) e)
27 27 54 instante de um espetáculo moderno os refletores são acionados
11 10 aleatoriamente de modo que, para cada um dos refletores, seja de
b) d) 2
54 27 a probabilidade de ser aceso. Então, a probabilidade de que, neste
3
39. (ITA) Uma urna de sorteio contém 90 bolas numeradas de 1 a 90, instante, 4 ou 5 refletores sejam acesos simultaneamente, é igual a
sendo que a retirada de uma bola é equiprovável à retirada de cada 16 151
uma das demais. a) . c) . 24 25
e) + .
27 243 34 35
a) Retira-se aleatoriamente uma das 90 bolas desta urna. Calcule a
49 479
probabilidade de o número desta bola ser um múltiplo de 5 ou b) . d) .
de 6. 81 729
b) Retira-se aleatoriamente uma das 90 bolas desta urna e, sem 05. (ITA) Numa caixa com 40 moedas, 5 apresentam duas caras, 10
repô-la, retira-se uma segunda bola. Calcule a probabilidade de o são normais (cara e coroa) e as demais apresentam duas coroas. Uma
número da segunda bola retirada não ser um múltiplo de 6. moeda é retirada ao acaso e a face observada mostra uma coroa. A
probabilidade de a outra face desta moeda também apresentar uma
40. (IME) Cada um dos quatro quadrados menores da figura abaixo é coroa é
pintado aleatoriamente de verde, azul, amarelo ou vermelho. 7 5 5 3 3
a) . b) . c) . d) . e) .
8 7 8 5 7
06. (ITA) Dois atiradores acertam o alvo uma vez a cada três disparos.
Se os dois atiradores disparam simultaneamente, então a probabilidade
do alvo ser atingido pelo menos uma vez é igual a
2 4 2
a) c) e)
9 9 3
1 5
b) d)
3 9
Qual é a probabilidade de que ao menos dois quadrados, que possuam 07. (ITA) Considere os seguintes resultados relativamente ao
um lado em comum, sejam pintados da mesma cor? lançamento de uma moeda:
1 I. Ocorrência de duas caras em dois lançamentos.
a)
2 II. Ocorrência de três caras e uma coroa em quatro lançamentos.
5 III. Ocorrência de cinco caras e três coroas em oito lançamentos.
b)
8
Pode-se afirmar que
7
c) a) dos três resultados, I é o mais provável.
16
b) dos três resultados, II é o mais provável.
23
d) c) dos três resultados, III é o mais provável.
32
43 d) os resultados I e II são igualmente prováveis.
e)
64 e) os resultados II e III são igualmente prováveis.

166

PM_BOOK16 - MAT.indb 166 25/11/2022 19:11:44


PROBABILIDADE

08. (ITA) Seja p uma probabilidade sobre um espaço amostral


1
finito Ω. Se A e B são eventos de Ω tais que p ( A ) = , p (B) =
1 3 (IME) Um menino, na cidade do Rio de Janeiro, lança uma
moeda. Ele andará 1 m para leste se o resultado for cara ou
1 m para oeste se o resultado for coroa. A probabilidade deste
1 2 3
e p ( A ∩ B) =, as probabilidades dos eventos A \ B, A ∪ B e AC ∪ menino estar a 5 m de distância de sua posição inicial, após 9
4 lançamentos da moeda, é
BC são, respectivamente,
9 2 9!
1 5 1 1 5 1 a) c) e)
a) , e . d) , e . 26 9! 29
4 6 4 3 6 3
2 35
1 5 1 1 7 3 b) d)
b) , e . e) , e . 9! 29
6 6 4 4 12 4

c)
1 7 3
, e .
6 12 4 4 (ITA) Um atirador dispõe de três alvos para acertar. O
primeiro deste encontra-se a 30 m de distância; o segundo,
a 40 m; o terceiro alvo, a 60 m. Sabendo que a probabilidade
09. (IME) O time de futebol “X” irá participar de um campeonato de o atirador acertar o alvo é inversamente proporcional ao
no qual não são permitidos empates. Em 80% dos jogos, “X” é o quadrado da distância e que a probabilidade de ele acertar
favorito. A probabilidade de “X” ser o vencedor do jogo quando ele 2
é o favorito é 0,9. Quando “X” não é o favorito, a probabilidade de o primeiro alvo é de , então a probabilidade de acertar ao
3
ele ser o vencedor é 0,02. Em um determinado jogo de “X” contra menos um dos alvos é
“Y”, o time “X” foi o vencedor. Qual a probabilidade de “X” ter sido
120 110 119
o favorito nesse jogo? a) . c) . e) .
160 144 144
a) 0,80 c) 180/181 e) 170/181
119 105
b) 0,98 d) 179/181 b) . d) .
154 135

10. (ITA) Com os elementos 1, 2, ..., 10 são formadas todas as


sequências (a1, a2, ..., a7). Escolhendo-se aleatoriamente uma dessas
sequências, a probabilidade de a sequência escolhida não conter
5 (IME) Os inteiros n e m são sorteados do conjunto
{1,2,3,...,2016}, podendo haver repetição. Qual a
probabilidade do produto n x m ser múltiplo de 12?
elementos repetidos é
5 5 5
7! 3! 10! a) c) e)
a) . c) . e) . 12 24 144
107 ⋅ 3! 107 ⋅ 7! 107
5 5
10! 10! b) d)
b) . d) . 18 36
107 ⋅ 3! 103 ⋅ 7!

GABARITO
EXERCÍCIOS DE FIXAÇÃO

DESAFIO PRO 01. C


02. D
04. C
05. B
07. E
08. B
10. B

03. D 06. D 09. A

1 (IME) Em um jogo de RPG “Role-Playing Game” em que


os jogadores lançam um par de dados para determinar a
vitória ou a derrota quando se confrontam em duelos, os dados
EXERCÍCIOS DE TREINAMENTO
01. D 15. A 29. D
são icosaedros regulares com faces numeradas de 1 a 20. Vence 02. C 16. B 30. A
quem soma mais pontos na rolagem dos dados e, em caso de
03. C 17. C 31. A
empate, os dois perdem. Em um confronto, seu adversário
somou 35 pontos na rolagem de dados. É sua vez de rolar os 04. D 18. D 32. D
dados. Qual sua chance de vencer este duelo? 05. C 19. C 33. B
a) 1/2 06. C 20. C 34. C
b) 3/76 07. E 21. D 35. D
c) 9/400 08. D 22. C 36. B
d) 1/80 09. C 23. B 37. E
e) 3/80 10. A 24. E 38. C
11. C 25. A 39. a) P(A ∪ B) = 1/3

2 (ITA) As faces de dez moedas são numeradas de modo


que: a primeira moeda tem faces 1 e 2; a segunda, 2 e 3; a
terceira, 3 e 4, e assim sucessivamente até a décima moeda, com
12. D
13. C
26. C
27. B
b) P = 5/6
40. E
faces 10 e 11. As dez moedas são lançadas aleatoriamente e os 14. C 28. A
números exibidos são somados. Então, a probabilidade de que EXERCÍCIOS DE COMBATE
essa soma seja igual a 60 é
01. E 04. A 07. D 10. B
63 180
a) . d) . 02. A 05. B 08. E
128 512
03. A 06. D 09. C
63 189
b) . e) . DESAFIO PRO
512 1024
01. E 03. A 05. B
63
c) . 02. B 04. E
512

167

PM_BOOK16 - MAT.indb 167 25/11/2022 19:11:45


PROBABILIDADE

ANOTAÇÕES

168

PM_BOOK16 - MAT.indb 168 25/11/2022 19:11:46


ESTATÍSTICA

CONCEITOS BÁSICOS Exemplo:


Um pesquisador, contratado pela empresa de cervejas, deseja
estudar quantas cervejas por semana seus clientes bebem. A amostra
ESTATÍSTICA
com 10 clientes resultou nos seguintes números: 2, 3, 7, 1, 10, 11,
É a ciência que utiliza a coleta de dados, sua classificação, sua 5, 2, 8, 9.
apresentação, sua análise e sua interpretação para se tomar algum
A amplitude desta amostra é igual a 11 – 1 = 10.
tipo de decisão.

ESTATÍSTICA DESCRITIVA ROL


Os dados coletados em uma amostra podem ser organizados
É o ramo da Estatística que se ocupa em organizar e descrever os
em tabelas ou gráficos. Para isso, antes devemos organizá-los em
dados que podem ser expressos por tabelas e gráficos.
sequências crescentes ou decrescentes denominadas Rol.
Exemplo:
ESTATÍSTICA INFERENCIAL
No exemplo anterior organizando em ordem crescente temos: 1,
É o ramo da Estatística que utiliza técnicas de análise e interpreta- 2, 2, 3, 5, 7, 8, 9, 10, 11.
ção de dados, a partir de uma amostra de uma população, e fornece
conclusões sobre este conjunto.
DADOS BRUTOS
POPULAÇÃO Podemos considerar como dados brutos aqueles que não estão
numericamente organizados.
Na coleta de dados sobre determinado assunto, chama-se
população estatística, o conjunto formado por todos os elementos Exemplo:
que possam oferecer dados relativos ao assunto em questão. 2, 3, 7, 1, 10, 11, 5, 2, 8, 9.
Podemos dizer que população é qualquer conjunto que reúna
todos os elementos que tenham pelo menos uma característica VARIÁVEIS
comum, objeto de estudo.
Uma outra definição que aparece na análise de dados estatísticos
é o conceito de variável.
AMOSTRA Uma variável é quantitativa quando seus valores podem ser
É um subconjunto de uma população. A seleção da amostra pode representados por contagem (variáveis quantitativas discretas) ou
ser feita de várias maneiras, dependendo, entre outros fatores, do grau mensuração (variáveis quantitativas contínuas).
de conhecimento que temos da população, da quantidade de recursos Uma variável é qualitativa quando apresentam como resultado um
disponíveis e outros fatores. A seleção da amostra deve fornecer um atributo, qualidade ou preferência de um entrevistado.
subconjunto de valores mais parecido possível com a população original.
Exemplo:
SÉRIES ESTATÍSTICAS
Uma pesquisa típica de audiência na televisão utiliza uma amostra
Uma série estatística é um conjunto de dados estatísticos que
de 5000 lares e, com base nestes dados, formula conclusões acerca de
fazem referência aos seguintes fatores: tempo, local e fenômeno.
uma população de todos os milhões de lares no país.
Os exemplos mais comuns de séries estatísticas são:

CENSO • a série temporal, cronológica ou Hhistórica;

É o método utilizado para um trabalho com uma população. • varia o tempo;


• a série geográfica, territorial ou espacial;
DADOS ESTATÍSTICOS • varia o local;
Os dados são denominados quantitativos quando são representa- • a série específica ou especificativa;
dos por números ou medidas, como por exemplo as alturas de uma • varia o fenômeno.
população, o número de filhos e o salário bruto. Quando os dados
representam contagens são discretos e quando representam mensu-
rações são contínuos. Elementos essenciais em uma tabela:
Os dados são chamados de qualitativos ou nominais quando • Título – é a indicação contida na parte superior da tabela,
são definidos por categorias tais como: cor dos olhos, sexo, nível de onde deve estar definido o fato observado;
escolaridade, naturalidade. • Corpo – é constituído por linhas e colunas, que fornecem o
conteúdo das informações prestadas;
AMPLITUDE DE UMA AMOSTRA • Cabeçalho – é a parte da tabela que apresenta a natureza do
A amplitude total dos dados é a diferença entre o valor máximo e que contém cada coluna.
o valor mínimo da amostra.

169

MAT085 - ESTATÍSTICA.indd 169 25/11/2022 19:32:18


ESTATÍSTICA

Há ainda os elementos que complementam a tabela como, por a) calcular a amplitude da amostra; 60 – 48 = 12 cm
exemplo: b) dividir o intervalo em subintervalos de mesmo comprimento.
• Fonte – designação da instituição que forneceu os dados Como exemplo, tomemos 4 subintervalos de comprimento igual
estatísticos. a 3: [48, 51[; [51,54[; [54,57[; [57,60]. Esses subintervalos são
Exemplo: chamados de classes e o comprimento de cada um é chamado de
amplitude da classe;
Datafolha, IBOPE, IBGE, INPE e etc.
c) conte quantas observações se situam em cada classe, respeitando
• Notas – esclarecimentos de natureza geral. os intervalos fechados à esquerda e abertos à direita, e coloque as
observações numa tabela do tipo abaixo.
TABELAS DE FREQUÊNCIAS
Um processo que possibilita uma leitura mais sucinta dos dados é COMPRIMENTO Fi Fri Fac Frac
a construção de uma tabela de frequências.
Exemplo 1: 5
[48, 51[ 5 = 0=
, 25 25% 5 25%
Uma entrevista com 20 pessoas é realizada no estado do Rio de 20
Janeiro. O objetivo da pesquisa era saber qual o time do entrevistado.
Dos 20 entrevistados foram encontrados os seguintes resultados 6
[51, 54[ 6 = 0=
, 3 30% 11 55%
para a frequência absoluta dos entrevistados: 20
Flamengo (f1 = 10)
Vasco (f2 = 6) 4
[54, 57[ 4 = 0=
, 20 20% 15 75%
Fluminense (f3 = 2) 20
Botafogo (f4 = 1)
5
Note que f1 + f2 + f3 + f4 = 20. [57, 60] 5 = 0=
, 25 25% 20 100%
20
Definimos frequência relativa absoluta assumido por uma variável
como a razão entre a frequência absoluta e o número total de dados. Total 20
fi
fri = Onde,
n
fi (frequência absoluta): frequência que o intervalo aparece na
distribuição;
FREQUÊNCIA FREQUÊNCIA fri (frequência relativa): é a percentagem do valor dos dados em
TIME ABSOLUTA RELATIVA PORCENTAGEM relação ao total da amostra;
(FI) (FRI)
fac (frequência acumulada): é a soma das frequências absolutas
começando pelo menor valor;
10
Flamengo 10 50% frac (frequência relativa acumulada): é a porcentagem do valor
20
das frequências acumuladas em relação ao total da amostra.

6
Vasco 6
20
30% GRÁFICOS

3 GRÁFICO EM LINHA
Fluminense 3 15% Esse tipo de gráfico é usado sobretudo quando temos observações
20
temporais de uma variável em estudo e desejamos representá-la
no tempo (abscissa) a fim de reconhecer possíveis tendências e/ou
1
Botafogo 1 5% sazonalidade (comportamento periódicos repetidos). O exemplo a
20 seguir ilustra bem a utilidade do gráfico em linha para a evolução do
preço da ação da empresa PETRO S.A.
TOTAL 20 1 100%

Exemplo 2:
Para avaliar o tempo de permanência em um supermercado,
o gerente mensurou em minutos o tempo de permanência de 20
clientes na loja. Os tempos estão representados na tabela abaixo.

49 52 56 52 50

54 57 60 48 59

48 49 57 53 55

51 53 52 55 57

GRÁFICO EM BARRAS HORIZONTAIS


Para representar esses dados em uma tabela de frequência
devemos: Os dados que estejam organizados em colunas ou linhas em uma
tabela podem ser representados em um gráfico de barras horizontais.

170

MAT085 - ESTATÍSTICA.indd 170 25/11/2022 19:32:25


ESTATÍSTICA

Gráficos de barras ilustram comparações entre itens individuais. Voltando ao exemplo 2:


Considere a utilização de um gráfico de barras horizontais quando:
• os rótulos dos eixos forem longos; TEMPO FI
• os valores mostrados forem durações.
[48, 51[ 5

[51, 54[ 6

[54, 57[ 4

[57, 60] 5

TOTAL 20

O Histograma referente à tabela é

GRÁFICO DE COLUNAS
A ideia é expressar informações individualizadas, e representadas
por barras cuja altura representa a frequência nas categorias. Vejamos
o exemplo a seguir, representando em barras o número de cópias de
jornais (em milhares de exemplares) em alguns países.

POLÍGONO DE FREQUÊNCIA
O polígono de frequência é obtido unindo-se os pontos médios
da parte superior de cada retângulo do histograma com segmentos de
reta. É importante notar que tanto o histograma quanto o polígono de
frequência indicam a frequência absoluta de cada classe.
Voltando ao exemplo 2 temos:
GRÁFICO EM SETORES
É utilizado quando se deseja mostrar partes do total, conforme
ocorre em produções, vendas e orçamentos de países e etc.

O gráfico em linhas representativo de uma distribuição de


frequências acumuladas é chamado polígono de frequências
acumuladas ou ogiva de Galton. No caso de dados agrupados em
intervalos de classe, os pontos do gráfico são os pontos correspondentes
aos limites superiores das classes das bases superiores dos retângulos.

HISTOGRAMA CARTOGRAMA
Quando as classes são intervalos reais, a interpretação da Um cartograma é um mapa que mostra informação quantitativa
distribuição de frequências em um sistema de eixos é feita por um mantendo um certo grau de precisão geográfica das unidades
tipo de gráfico chamado Histograma. espaciais mapeadas.

171

MAT085 - ESTATÍSTICA.indd 171 25/11/2022 19:32:26


ESTATÍSTICA

% de Prefeitos Eleitos em 2012: PT Exemplo:


12  13  14  14  14  15  15  16  16  17
x  14, 6
10

PROPRIEDADES DA MÉDIA
1. Ao adicionarmos um mesmo valor a cada um dos valores
assumidos pela variável, a média aritmética fica adicionada
desse valor.
2. Ao multiplicarmos cada um dos valores assumidos pela variável
por um mesmo valor, a média aritmética fica multiplicada por
esse valor.
Seja x uma variável quantitativa e x1, x2, x3, ..., xk os valores assumidos
por essa variável, com frequências absolutas respectivamente iguais
a n1, n2, n3, ..., nk. A média aritmética (x) desses valores é igual à
soma de cada um os valores assumidos pela variável multiplicado
pela sua frequência dividida pela soma das frequências, ou seja,
k

 x n
i1
i i
x1  n1  x 2  n2    xk  nk .
x k

n1  n2    nk
n i1
i

A média aritmética é muito influenciada por valores discrepantes


(“outliers”).
PICTOGRAMA
É comum em jornais e revistas ilustrar os vários tipos de gráficos
com figuras relacionadas ao assunto, tornando-os mais atraentes.
MEDIANA
Esses são os pictogramas. Seja x1 ≤ x2 ≤ x3 ≤ ... ≤ xn os valores ordenados assumidos pela
variável quantitativa x. A mediana (Me) é dada por:
x  n+1 , se n é ímpar
  2 

Me =  x n + x n
   
  2   +1
2 
 , se n é par
 2

Dessa forma, a mediana é tal que a quantidade de valores


menores ou iguais à mediana é igual à quantidade de valores maiores
ou iguais à mediana.
A mediana é uma medida de centralidade menos sensível a
valores discrepantes.
Exemplo:
x5  x 6 14  15
No nosso exemplo a mediana é   14, 5.
2 2

MODA
A moda de um conjunto é o valor que ocorre mais vezes ou de
maior frequência simples (absoluta ou relativa) numa distribuição de
MEDIDAS DE CENTRALIDADE: MÉDIA frequências.
ARITMÉTICA, MÉDIA ARITMÉTICA No nosso exemplo a moda é igual a 14.

PONDERADA, MEDIANA, MODA A moda pode também não existir ou não ser única.

Para apresentar os conceitos a seguir, vamos considerar um grupo Exemplo:


de 10 estudantes com as seguintes idades: 0, 1, 1, 2, 2, 3 tem modas 1 e 2 (bimodal).
12, 13, 14, 14, 14, 15, 15, 16, 16, 17.
MEDIDAS DE DISPERSÃO: DESVIO
MÉDIA ARITMÉTICA E MÉDIO, VARIÂNCIA, DESVIO PADRÃO
MÉDIA ARITMÉTICA PONDERADA Considere uma turma de 5 alunos em que todos tiraram nota 5 e
Seja x uma variável quantitativa e x1, x2, x3, ..., xn os valores outra com a mesma quantidade de alunos com as seguintes notas: 1, 3,
assumidos por essa variável. A média aritmética (x) de x é igual a soma 5, 7 e 9. Essas duas turmas têm a mesma média aritmética e a mesma
de todos os valores assumidos pela variável dividida pelo número de mediana que é 5. Mas a dispersão dos valores é completamente
valores, ou seja, diferente e pode ser calculado.
n

x
i1
i
x1  x 2    xn
DESVIO MÉDIO ABSOLUTO (DMA)
x  O desvio em relação à média aritmética é a diferença entre cada
n n
valor e a média aritmética.

172

MAT085 - ESTATÍSTICA.indd 172 25/11/2022 19:32:29


ESTATÍSTICA

δi = xi – x PROPRIEDADES DO DESVIO PADRÃO


Se adicionarmos um mesmo valor a cada um dos valores assumidos
A soma de todos os desvios em relação à média aritmética é
pela variável, o desvio padrão não se altera.
sempre nula.
Se multiplicarmos cada um dos valores assumidos pela variável
O desvio médio absoluto (DMA) é a média aritmética dos desvios
por um mesmo valor, o desvio padrão fica multiplicado por esse valor.
em módulo.
n n
Observação
   x x
i1
i
i1
i
Se o desvio padrão for calculado sobre uma amostra em vez
DMA  
n n de sobre toda a população, teremos então o chamado desvio
n n
O desvio médio absoluto da segunda turma do nosso exemplo é
4  1 0  1 4  2
i  x  x 
i
2

DMA   2. padrão amostral que é dado por S  i1


 i1
5 n 1 n 1
VARIÂNCIA POPULACIONAL k k

A variância populacional (σ²) é a média aritmética da soma



i1
2
i  fi  x  x 
i1
i
2
 fi
ou para dados agrupados por S   .
dos quadrados dos desvios em relação à média de um conjunto de n 1 n 1
números.
n
n n A razão entre o desvio padrão amostral e o populacional é
  x  x  n −1
2
i2 i chamado fator de correção de Bessel.
2  i1
 i1
n n Se os valores se distribuem de acordo com uma distribuição
normal (unimodal, gaussiana, simétrica, de afunilamento médio)
Outra fórmula para o cálculo da variância populacional é
podemos dizer que:
  n  
2

1

n

 
x 
i


• 68% dos valores encontram-se a uma distância da média
inferior a um desvio padrão;
2

   xi 
n  i1
2  i1
n

.
 • 95% dos valores encontram-se a uma distância da média
inferior a duas vezes o desvio padrão;
PROPRIEDADES DA VARIÂNCIA • 99,7% dos valores encontram-se a uma distância da média
inferior a três vezes o desvio padrão.
Se adicionarmos um mesmo valor a cada um dos valores assumidos
pela variável, a variância não se altera. Esta informação é conhecida como a regra dos “68-95-99,7”.
Se multiplicarmos cada um dos valores assumidos pela variável
por um mesmo valor, a variância fica multiplicada pelo quadrado desse
valor.
Se a variância for calculada sobre uma amostra em vez de sobre
toda a população, teremos então a chamada variância amostral que
n

2
 x  x 
i1
i
2

é dada por S  .
n 1

DESVIO PADRÃO POPULACIONAL


O desvio padrão populacional (σ) é a média quadrática dos
Fonte: Wikipédia
desvios em relação à média de um conjunto de números ou a raiz
quadrada da variância.
Exemplo:
n n


i1
i2   xi  x 2
i1
Sejam 10 estudantes com as seguintes idades: 13, 13, 14, 14, 14,
14, 15, 15, 15, 16.
 
n n
IDADE Ni δi δ²i
Outra fórmula para o cálculo do desvio padrão populacional é
 13 2 −1,3 1,69
 
2
 n

1

 n
  x  

i
14 4 −0,3 0,09
 
 xi 
n  i1
2  i1
n
.
 15 3 0,7 0,49
Se os dados forem agrupados em intervalos de classe, então o 16 1 1,7 2,89
quadrado de cada desvio deve ser multiplicado pela sua frequência
absoluta (ni) ou relativa (fi).
k O desvio padrão populacional é
 x  x 
2
 fi
i k 2  1, 69  4  0, 09  3  0, 49  1 2, 89
 i1
n
  x  x 
i1
i
2
 fri 
2  4  3 1
 0, 81  0, 9.

173

MAT085 - ESTATÍSTICA.indd 173 25/11/2022 19:32:40


ESTATÍSTICA

EXERCÍCIOS DE a) todas. c) apenas três.

FIXAÇÃO b) apenas quatro. d) apenas duas.

10. (EEAR) Os salários mensais, em reais, dos 24 funcionários de uma


empresa são
01. (ESA) Numa sala de aula, a média das idades dos 50 alunos era
800 840 880 880 1000 1050 1060 1060
de 22,5 anos. No cálculo da média, foram consideradas idades com
anos completos. Transcorridas algumas semanas, houve a desistência 1100 1150 1200 1210 1230 1250 1280 1300
de um aluno e a média das idades caiu para 22 anos. Considerando-se 1340 1380 1450 1480 1500 1500 1520 1550
que nesse período nenhum dos alunos da turma fez aniversário, então
a idade do aluno que desistiu é igual a: O salário mensal mediano dessa empresa, em reais, é
a) 47 anos. c) 37 anos. e) 27 anos. a) 1200 b) 1210 c) 1220 d) 1230
b) 45 anos. d) 35 anos.
EXERCÍCIOS DE

TREINAMENTO
02. (ESA) A média aritmética das notas de Matemática em uma turma
de 25 alunos em um dos doze Colégios Militares existentes no Brasil
diminui em 0,1, se alterarmos uma das notas para 6,8. A referida nota
sem ser alterada é:
a) 4,3 c) 4,8 e) 9,8
01. (EEAR) Numa prova de matemática, três classes obtiveram as
b) 8,8 d) 9,3 seguintes médias e desvios:
classe A: x = 4,5 e δ = 2,5
03. (ESA) Em uma turma a média aritmética das notas é 7,5. Sabe-se
classe B: x = 4,5 e δ = 3,1
que a média aritmética das notas das mulheres é 8 e das notas dos
homens é 6. Se o número de mulheres excede o de homens em 8, classe C: x = 4,5 e δ = 2,8
pode-se afirmar que o número total de alunos da turma é Se for sorteado um aluno em cada classe, em qual delas é mais
a) 4. c) 12. e) 20. provável que a nota desse aluno esteja entre 3,0 e 6,0?
b) 8. d) 16. a) Classe A c) Classe C
b) Classe B d) Classes B e C
04. (ESA) A média aritmética de n números é 29. Retirando-se o
número 12 a média aumenta para 30. Podemos afirmar que o valor 02. (EEAR) Os resultados da prova de Ciências aplicada a uma turma
de n será de um certo colégio estão apresentados no gráfico. Baseado neste
a) 17. c) 42. e) 18. gráfico, podemos afirmar que a porcentagem de alunos dessa turma
com nota inferior a 5,0, nessa prova de Ciências, foi de
b) 11. d) 41.
14
05. (ESA) A média aritmética de todos os candidatos de um concurso
12
número de alunos

foi 9,0, dos candidatos selecionados foi 9,8 e dos eliminados foi 7,8.
Qual o percentual de candidatos selecionados? 10
a) 20% c) 30% e) 60% 8
b) 25% d) 50%
6
06. (ESA) Qual é a média de idade de um grupo em que há 6 pessoas 4
de 14 anos, 9 pessoas de 20 anos e 5 pessoas de 16 anos? a) 37,5%
2
a) 17,2 anos c) 17,0 anos e) 19,4 anos b) 42,5%
0
b) 18,1 anos d) 17,5 anos c) 47,5%
1 2 3 4 5 6 7 8 9
d) 52,5% notas
07. (ESA) Identifique a alternativa que apresenta a frequência absoluta
(fi) de um elemento (xi) cuja frequência relativa (fr) é igual a 25 % e 03. (EEAR) A altura de 80 homens de uma comunidade está distribuída
cujo total de elementos (N) da amostra é igual a 72. de acordo com a tabela. A porcentagem de homens com altura maior
a) 18. c) 9. e) 45. ou igual a 1,80 m é
b) 36. d) 54. altura (m) número de homens
1,60 |–– 1,65 04
08. (ESA) O exército realizou um concurso de seleção para contratar
sargentos e cabos. A prova geral foi igual para ambos. Compareceram 1,65 |–– 1,70 12
500 candidatos para sargento e 100 para cabo. Na prova, a média de 1,70 |–– 1,75 18
todos os candidatos foi 4, porém, a média apenas entre os candidatos
1,75 |–– 1,80 26
a sargento foi 3,8. Desse modo, qual foi a média entre os candidatos
a cabo? 1,80 |–– 1,85 10
a) 3,9 c) 6,0 e) 5 1,85 |–– 1,90 08
b) 1,0 d) 4,8 1,90 |–– 1,95 02

09. (EEAR) Os alunos da 6ª série A de um colégio foram pesquisados em Total 80


cinco diferentes objetos de estudo: sexo, idade, cor dos olhos, disciplina a) 25% c) 60%
favorita e estatura. Desses cinco objetos, são variáveis qualitativas
b) 30% d) 75%

174

MAT085 - ESTATÍSTICA.indd 174 25/11/2022 19:32:41


ESTATÍSTICA

04. (EEAR) Um teste de inteligência, aplicado aos alunos das 4as Os valores de M, P, R e S são, respectivamente:
séries do Ensino Fundamental da Escola A, apresentou os seguintes a) 0,35; 5; 7 e 0,35. c) 0,4; 7; 5 e 0,25.
resultados:
b) 0,4; 7; 5 e 0,4. d) 0,25; 5; 7 e 0,25.
Pontos nº de alunos Pontos nº de alunos
11. (EEAR) A tabela apresenta as frequências acumuladas das notas
90 |–– 95 40 115 |–– 120 140 de 70 alunos, obtidas em uma avaliação. A frequência absoluta da 2ª
classe é
95 |–– 100 60 120 |–– 125 120
Notas Frequência acumulada
100 |–– 105 140 125 |–– 130 30 2,0 |− 3,5 12
105 |–– 110 160 130 |–– 135 20 3,5 |− 5,0 26

110 |–– 115 180 135 |–– 140 10 5,0 |− 6,5 43


6,5 |− 8,0 57
A frequência relativa da classe modal é
8,0 |− 9,5 70
a) 0,2 b) 0,22 c) 0,25 d) 0,5
a) 14 c) 16
05. (EEAR) Na distribuição a seguir, as 6 classes foram justapostas,
b) 15 d) 17
isto é, o limite superior de uma classe é o inferior da classe seguinte.
Se fi é frequência absoluta, então a frequência acumulada absoluta
da 4ª classe é 12. (EEAR) Na tabela de dados brutos tem-se as massas, em
quilogramas, de 15 clientes de uma clínica médica. Organizando os
Classes 5 |−− 10 10 |−− 15 15 |−− 20 20|−− 30 25|−− 30 30 |−− dados desta tabela pode-se verificar que a amplitude do rol, em kg, é
fi 8 12 21 16 6 3
83 72 86 74 88
a) 57 b) 20 c) 41 d) 16 57 81 91 65 82
59 55 49 73 74
06. (EEAR) A média de um conjunto de quatro valores é 4,25. Se
aumentarmos de 5 unidades o menor desses valores, e diminuirmos a) 36 c) 51
de 3 unidades o maior deles, a nova média será
b) 42 d) 55
a) 4,75 b) 5,25 c) 5 d) 5,5
13. (EEAR) A média da distribuição representada pelo seguinte
07. (EEAR) A tabela traz as idades, em anos, dos filhos de 5 mães. Histograma é
Nome
Ana Márcia Cláudia Lúcia Eloisa
da Mãe
Idade
dos 7, 10, 12 11, 15 8, 10, 12 12, 14 9, 12, 15, 16, 18
filhos a) 8
A idade modal desses 15 filhos é inferior à idade média dos filhos de b) 7
Eloísa em _______ anos(s). 56
c)
a) 4 b) 3 c) 2 d) 1 9
d) 61
08. (EEAR) Na distribuição dos salários de 800 empregados de uma 9
empresa, o ponto médio da 4ª classe é R$ 1400,00. Se as 8 classes
dessa distribuição têm a mesma amplitude de R$ 200,00 e são do tipo 14. (EEAR) A tabela seguinte informa a quantidade de pessoas que
[a, b[, então a 6ª classe não inclui, com certeza, o salário de R$ compraram ingressos antecipados de um determinado show, cujos
a) 1900,00 b) 1850,00 c) 1800,00 d) 1750,00 preços eram modificados semanalmente.

09. (EEAR) Sejam x1, x2, x3, ..., x81 os valores ordenados de uma variável Valor do ingresso (R$) Número de pessoas
X. A mediana desse conjunto de valores é igual a
50 |− 75 300
x 40 + x 41 x 41 + x 42
a) x41 b) x40 c) d)
2 2 75 |− 100 640
10. (EEAR) Os resultados de uma pesquisa realizada com 20 alunos 100 |− 125 500
de uma escola, a respeito da área da carreira pretendida, estão
apresentados na tabela: 125 |− 150 1.310

150 |− 175 850


Área Frequência Absoluta Frequência Relativa
Σ = 3.600
Humanas 8 M

Biológicas P 0,35 O percentual de pessoas que adquiriram o ingresso por menos de


R$ 125,00 foi
Exatas R S a) 40% c) 50%
Total 20 1,00 b) 45% d) 55%

175

MAT085 - ESTATÍSTICA.indd 175 25/11/2022 19:32:42


ESTATÍSTICA

15. (EBMSP) Com base nos dados apresentados, a mediana da distribuição pertence à
a) 2ª classe b) 3ª classe c) 4ª classe d) 5ª classe

19. (EPCAR) “Demanda Crescente


O consumo de energia elétrica no Brasil nunca foi tão alto. Na
quinta-feira passada, atingiu seu recorde histórico. O valor é muito
superior ao registrado em anos anteriores”
(revista Veja – 10/02/10 – p. 71)

O gráfico abaixo indica o pico de consumo de energia (em megawatts)


na primeira quinta-feira de fevereiro dos anos de 2002 a 2010.

O gráfico ilustra o número percentual de pessoas que, atendidas em


um posto de saúde, em determinado período, apresentou problemas
cardíacos.
Com base nos dados do gráfico e considerando-se M o número
de mulheres e H o número de homens atendidos, nesse período, é
correto afirmar:
a) H = M – 10 c) H=M+5 e) H = 2M
b) H = M d) H = M + 10

16. (EEAR) A distribuição dos salários dos 20 funcionários de uma


empresa está representada no quadro a seguir.
Salário (em reais) Número de funcionários (fi) fia fr (%)
860 2 2 10
950 6 8 _____
1.130 _____ 16 40
1.480 3 ____ 15
2.090 1 20 5
Os valores que completam corretamente as lacunas do quadro são Analisando-se o gráfico acima e supondo-se que em 2011, na
primeira quinta-feira do mês de fevereiro, haverá um crescimento do
a) fi = 10; fia = 13; fr = 30 c) fi = 8; fia = 11; fr = 20
pico de consumo de energia, proporcional ao crescimento ocorrido
b) fi = 10; fia = 13; fr = 20 d) fi = 8; fia = 19; fr = 30 na primeira quinta-feira do mês de fevereiro do ano de 2009 ao ano
de 2010, é correto afirmar que x é um número compreendido entre
17. (EEAR) Ao calcular a média aritmética das notas dos Testes Físicos a) 76000 e 77000 c) 78000 e 79000
(TF) de suas três turmas, um professor de Educação Física anotou os
seguintes valores: b) 77000 e 78000 d) 79000 e 80000

Turma Nº de alunos Média do TF 20. (EPCAR) Para as eleições para a Presidência da República do Brasil
A 20 9 foi feita uma pesquisa com 2400 pessoas sobre suas preferências em
relação aos candidatos A, B e C. Sabe-se que cada pessoa optou por
B 40 7,5
um único candidato, ou votou em branco, ou votou nulo, e que o
C 30 8 diagrama abaixo indica os resultados da pesquisa.
A média aritmética das notas do TF dos 90 alunos das turmas A, B e

a) 8,0 b) 8,1 c) 8,2 d) 8,3

18. (EEAR) A distribuição de frequência abaixo refere-se à exportação


de soja realizada por uma Cooperativa no mês de abril.
xi Toneladas exportadas fi
1 10 |− 20 3
2 20 |− 30 2
3 30 |− 40 8 Dados: Os ângulos a, b, c
4 40 |− 50 10 e d são tais que:
c = 90º
5 50 |− 60 7 a + b = 90º
∑fi = 30 a = 2b
Dados fictícios Em cada região do diagrama tem-se:

176

MAT085 - ESTATÍSTICA.indd 176 25/11/2022 19:32:44


ESTATÍSTICA

I. Nº de pessoas que votou no candidato A. 23. (AFA) No Atlas de Desenvolvimento Humano no Brasil 2013
II. Nº de pessoas que votou no candidato B. constam valores do Índice de Desenvolvimento Humano Municipal
(IDHM) de todas as cidades dos estados brasileiros.
III. Nº de pessoas que votou no candidato C.
O IDHM é um número que varia entre 0 e 1. Quanto mais próximo
IV. Nº de pessoas que votou em branco. de 1, maior o desenvolvimento humano de um município, conforme
V. Nº de pessoas que votou nulo. escala a seguir.
Sabe-se que a diferença entre o número de pessoas que votou nulo
e o número de pessoas que votou em B é y. Então, y representa a/o
a) quarta parte do total de entrevistados.
b) metade do total de entrevistados.
c) terça parte do total de entrevistados.
d) dobro do número de pessoas que votou em C. Abaixo estão relacionados o IDHM de duas cidades de Minas Gerais
em condições extremas, Monte Formoso e Uberlândia, e uma em
21. (FUVEST) Examine o gráfico. situação intermediária, Barbacena.

Analisando os dados acima, afirma-se que


I. o município de maior crescimento do IDHM, nos períodos
considerados, é Monte Formoso.
Com base nos dados do gráfico, pode-se afirmar corretamente que
a idade II. na última década, Barbacena apresentou maior evolução do
IDHM que Uberlândia.
a) mediana das mães das crianças nascidas em 2009 foi maior que
27 anos. III. uma tabela que relaciona cidade, época e faixa de IDHM pode ser
representada corretamente como:
b) mediana das mães das crianças nascidas em 2009 foi menor que
23 anos. Monte Formoso Barbacena Uberlândia
c) mediana das mães das crianças nascidas em 1999 foi maior que 1991 Muito baixo Baixo Baixo
25 anos.
2000 Muito baixo Alto Alto
d) média das mães das crianças nascidas em 2004 foi maior que 22
anos. 2010 Baixo Alto Alto
e) média das mães das crianças nascidas em 1999 foi menor que São corretas
21 anos. a) apenas I e II. c) apenas I e III.
b) apenas II e III. d) I, II e III.
22. (MACKENZIE) Um pesquisador fez um conjunto de medidas em
um laboratório e construiu uma tabela com as frequências relativas 24. (PUC) O ponto forte das políticas públicas de conservação de água
(em porcentagem) de cada medida, conforme se vê a seguir: da cidade de Campinas está relacionado a um amplo programa de
Valor medido Frequência relativa (%) educação ambiental, em especial no que diz respeito à recuperação
da qualidade dos cursos d’água urbanos.
1,0 30 Na tabela a seguir, têm-se dados sobre a utilização de água em
1,2 7,5 Campinas no período de 1993 a 2003.

1,3 45 Consumo diário


Volume mensal Volume mensal
per capta para
1,7 12,5 Ano captado utilizado
todos os usos
(milhões de m³) (milhões de m³)
1,8 5 (litros por dia)
Total = 100 1993 9,2 5,4 211
Assim, por exemplo, o valor 1,0 foi obtido em 30% das medidas 1994 9,7 5,7 219
realizadas. A menor quantidade possível de vezes que o pesquisador 1995 9,7 6,0 227
obteve o valor medido maior que 1,5 é
a) 6 d) 9 1996 9,6 6,2 231
b) 7 e) 10 1997 9,9 6,2 227
c) 8

177

MAT085 - ESTATÍSTICA.indd 177 25/11/2022 19:32:46


ESTATÍSTICA

1998 9,6 6,3 227


1999 9,3 6,5 230
2000 9,6 6,6 230
2001 9 6,3 216
2002 9,1 6,4 216
2003 x y z
(Adaptado da Revista Saneamento Ambiental. Ano XIV. n. 105.
São Paulo: Signus. p. 39)
Os demais alunos fizeram a avaliação B e todos tiveram 4 acertos.
Sobre a tabela, é correto afirmar que Assim, o desvio padrão obtido a partir do gráfico acima ficou reduzido
à metade ao ser apurado o resultado da turma inteira. Essa turma do
a) a diferença entre o volume médio captado e o volume médio cursinho de inglês tem
utilizado, no período 1993-2002, foi de 33,1 milhões de m3.
a) mais de 23 alunos. c) 21 alunos.
b) a média de consumo diário per capta nos 5 primeiros anos (1993-
1997) foi maior que nos 5 anos de 1998 a 2002. b) menos de 20 alunos. d) 22 alunos.
c) se o volume médio captado, de 1993 a 1997, foi igual ao que
28. (AFA) Em uma turma de 5 alunos, as notas de um teste de
ocorreu de 1998 a 2003, então o volume x captado em 2003 é de
matemática são números inteiros tais que a média aritmética e a
11,12 milhões de m3.
mediana são iguais a 5, e nenhum aluno errou todas as questões.
d) se o volume y utilizado em 2003 correspondeu a 85% do volume Sabendo que esse conjunto de notas é unimodal, com moda igual a
médio utilizado no período 1993-2002, então y é maior que 5,5 8, então a diferença entre a maior nota e a menor nota é um número
milhões de m3. que é divisor de
e) o volume médio utilizado é ligeiramente inferior a 60% do volume a) 14 b) 15 c) 16 d) 18
médio captado no período 1993-2002.
29. (AFA) As notas de oito alunos numa prova de matemática foram
25. (FUVEST) Cada uma das cinco listas dadas é a relação de notas escritas pelo professor numa tabela como a que segue:
obtidas por seis alunos de uma turma em uma certa prova.
Assinale a única lista na qual a média das notas é maior do que a Aluno A B C D E F G H
mediana. Nota 6,5 10 8 9,4 8 6,4 x 7,4
a) 5, 5, 7, 8, 9, 10 d) 5, 5, 5, 7, 7, 9
Sabe-se que a média aritmética dessas notas é 8,2. Considerando as
b) 4, 5, 6, 7, 8, 8 e) 5, 5, 10, 10, 10, 10 notas dos oito alunos, é correto afirmar que a nota do aluno G é
c) 4, 5, 6, 7, 8, 9 a) igual à moda.
b) inferior a 9,8.
26. (PUC) Nas principais concentrações urbanas do país, trabalhadores
de baixa renda percorrem grandes distâncias a pé. Outros pedalam c) superior à mediana.
muitos quilômetros para usar uma condução a menos, deixando a d) inferior à média aritmética das outras sete notas.
bicicleta em estacionamentos próprios.
A tabela a seguir mostra os resultados de uma pesquisa sobre a faixa 30. (AFA) Na tabela a seguir estão relacionados os salários de todos os
salarial dos funcionários de uma empresa que usam bicicleta para ir funcionários das classes A, B e C de uma empresa cuja média salarial
ao trabalho. é R$ 1.680,00.
Classes Salários Quantidade de funcionários
Faixa salarial em reais Número de funcionários
A 900 |− 1.500 20
350 |− 450 380
B 1.500 |− 2.100 X
450 |− 550 260
C 2.100 |− 2.700 10
550 |− 650 200
Se a mediana para a distribuição de frequências obtida acima é m,
650 |− 750 180 então a soma dos algarismos de m é igual a
750 |− 850 120 a) 10 b) 12 c) 15 d) 18
850 |− 950 60
Total 1 200
EXERCÍCIOS DE

COMBATE
O salário médio desses trabalhadores é
a) R$ 400,00
b) R$ 425,00
c) R$ 480,00
01. (EEAR 2006) Sendo fi as frequências absolutas, a classe mediana
d) R$ 521,00 da distribuição é a:
e) R$ 565,00
classe [10,20[ [20,30[ [30,40[ [40,50[ [50,60[ [60,70[ [70,80[

27. (AFA) Um cursinho de inglês avaliou uma turma completa sendo fi 25 18 10 05 09 12 15


que parte dos alunos fez a avaliação A, cujo resultado está indicado
no gráfico abaixo. a) 2ª c) 4ª
b) 3ª d) 5ª

178

MAT085 - ESTATÍSTICA.indd 178 25/11/2022 19:32:47


ESTATÍSTICA

02. (EEAR 2006) A tabela mostra as idades dos alunos matriculados a) 12 e 12%
no Centro de Educação Infantil “X”, em 2005. b) 12 e 20%
Idade (anos) Número de alunos c) 18 e 36 %
2 3 d) 18 e 20%
3 3
4 5 07. (EEAR 2008) A revista Época publicou, em janeiro de 2000,
5 14 os resultados de uma pesquisa por ela realizada em setembro de
6 25 1999. Cada participante indicava o nome de uma personalidade
mundialmente conhecida, do século XX, da qual ele mais se lembrava.
Total 50 O gráfico a seguir traz o percentual de pessoas que indicaram cada
uma dessas personalidades.
A média das idades dos alunos dessa escola, em anos, é, aproxima-
damente:
a) 4,1 c) 5,1
b) 4,5 d) 5,6

03. (EEAR 2007) A tabela a seguir traz o resultado de uma prova


de Ciências. Nela, xi são as notas e fi são as frequências absolutas.
Agrupando os dados em 5 classes do tipo [a,b[, de amplitude 1,5,
sendo o limite inferior da 1ª classe a nota 1,5, a frequência absoluta
da 3ª classe da nova tabela será igual a

xi 1,5 2,0 2,5 3,0 3,5 4,0 4,5 5,0 5,5 6,0 6,5 7,0 7,5 8,0 8,5

fi 1 2 2 3 5 6 7 8 9 7 6 5 4 3 2

Sabendo que participaram dessa pesquisa 60 mil pessoas, Ayrton


a) 14 Senna foi indicado por ______ pessoas.
b) 19 a) 12.800
c) 24 b) 15.300
d) 29 c) 16.900
d) 18.600
04. (EEAR 2007) Os dados de uma pesquisa, cujo objetivo era saber
o número de filhos, por família, realizada em uma certa comunidade,
08. (EPCAR 2017) Uma consulta pública realizada pelo Instituto
estão na tabela:
que organiza a aplicação do Exame Nacional do Ensino Médio, em
Nº de filhos 0 1 2 3 4 5 fevereiro de 2017, visou conhecer a preferência sobre os possíveis
modelos de aplicação do Exame:
Nº de famílias 2 8 10 14 18 15
- Modelo A: testes em apenas 1 dia.
É correto afirmar que o número - Modelo B: testes no sábado e no domingo.
a) modal de filhos é maior que o número médio. - Modelo C: testes em dois domingos consecutivos.
b) médio de filhos coincide com o número modal. Suponha que tenham sido consultadas um total de x pessoas entre
moradores da capital e do interior. Desse total, 40 pessoas do interior
c) mediano e o número modal de filhos são iguais. e 60 da capital não manifestaram preferência pelos Modelos A, B ou
d) modal, o mediano e o número médio de filhos são iguais. C. O gráfico a seguir mostra os resultados dos que manifestaram sua
preferência:
05. (EEAR 2007) Quando o objetivo de uma pesquisa é comparar o
comportamento de uma mesma variável em populações com números
diferentes de elementos, a frequência mais conveniente é a
a) total.
b) relativa.
c) absoluta.
d) acumulada.

06. (EEAR 2007) Feito um levantamento sobre a altura dos 50 alunos


da 5ª série A de um colégio, chegou-se aos seguintes resultados:
Baseado nestas informações, é correto afirmar que
Altura (cm) nº de alunos Altura nº de alunos a) 20% das pessoas consultadas, exatamente, preferem a aplicação
150  154 6 162  166 8 do Exame em um único dia.
b) o número total das pessoas consultadas no interior e na capital
154  158 12 166  170 6
é o mesmo.
158  162 14 170  174 4 c) 5/7 das pessoas que manifestaram preferência pelos Modelos
optaram pela realização do Exame em dois dias.
Nessas condições, o número de alunos da 5ª A que não atingem d) exatamente 12% das pessoas consultadas não manifestaram
1,58 m de altura, e a porcentagem de alunos cuja altura é maior ou opinião.
igual a 1,62 m são, respectivamente,

179

MAT085 - ESTATÍSTICA.indd 179 25/11/2022 19:32:49


ESTATÍSTICA

09. (EPCAR) Durante os meses de janeiro e fevereiro de 2020, as 10. (EPCAR 2014) A tabela e os gráficos abaixo são referentes aos
notícias foram alarmantes, especialmente na China, em virtude do candidatos do Concurso CPCAR 2012.
surto do Novo Coronavírus.
DISTRIBUIÇÃO POR REGIÃO DO BRASIL
Em 2002 e 2003, esse mesmo país sofreu com outro surto. Àquela
época o vírus foi chamado de Sars. REALIZARAM APROVADOS NO
A cobertura feita pelas diversas formas de mídia – televisiva, CONCURSO CONCURSO
REGIÃO
escrita e internet, dentre tantas – deu informações acerca da evolução N° DE N° DE
% %
de cada um desses vírus à sua época. CANDIDATOS CANDIDATOS
Em 28/01/2020, o portal de notícias G1, na internet, publicou Norte 477 5,4 33 4,2
matéria sob o título: “Nas primeiras semanas do surto, casos do novo Nordeste 710 8,0 59 7,2
coronavírus superam os da epidemia Sars de 2003”. Centro-Oeste 554 6,3 39 4,8
Junto aos dados apresentados naquele portal, apareceu a Sudeste 6605 74,8 659 80
reprodução de dois infográficos, cuja fonte era a Organização Mundial Sul 482 5,5 31 3,8
da Saúde. Nesses, estavam comparações do surgimento de casos de
Total 8828 100 821 100
ambos os vírus e, também, do número de mortes causadas por eles.
As figuras a seguir reproduzem esses dois infográficos, com
alterações no intuito de facilitar possíveis cálculos, nos quais as
quantidades numéricas tanto de casos quanto de mortes correspondem
ao acumulado no período.

Analisando as informações acima, afirma-se sobre o Concurso CPCAR


2012:
I. Os candidatos da região Sudeste, além do maior número na
realização do concurso, também tiveram maior percentual entre
os aprovados.
II. Dentre os aprovados que vieram de Escola Pública Estadual, é
possível não haver nenhum da Região Sudeste.
III. Dentre os aprovados que não foram motivados pelo ensino
oferecido, é possível que só haja candidatos vindos da Região
Sudeste.
Julgue cada afirmativa em (V) verdadeira ou (F) falsa e marque a
alternativa que contém a sequência correta.
a) V – V – V b) V – F – F c) F–F–V d) V – F – V

GABARITO
EXERCÍCIOS DE FIXAÇÃO
01. A 04. E 07. A 10. C
02. D 05. E 08. E
03. D 06. A 09. C
EXERCÍCIOS DE TREINAMENTO
01. A 09. A 17. A 25. D
02. B 10. A 18. C 26. E
A partir da análise desses dois infográficos é correto afirmar que 03. A 11. A 19. D 27. A
a) até o 18º dia, o crescimento no número de casos do Novo 04. A 12. B 20. A 28. A
Coronavírus foi maior que o crescimento do número de casos da
05. A 13. D 21. D 29. C
Sars, no mesmo período.
06. A 14. A 22. B 30. B
b) levando-se em consideração apenas o número de mortes até o
17º dia, o Novo Coronavírus foi 50% mais letal que a Sars. 07. C 15. B 23. A
c) o número de mortes pelo Novo Coronavírus até o 18º dia foi 08. A 16. D 24. C
superior ao número de mortes pela Sars em menos de 50% no EXERCÍCIOS DE COMBATE
mesmo período. 01. B 04. A 07. B 10. B
d) entre o 16º e o 17º dia, o número de casos do Novo Coronavírus 02. C 05. B 08. C
diminuiu.
03. C 06. C 09. A

180

MAT085 - ESTATÍSTICA.indd 180 25/11/2022 19:32:51


NÚMEROS COMPLEXOS

INTRODUÇÃO PROPRIEDADES
O fato de equações como x2 + 1 = 0 não terem soluções no 1. z  z  z  R.
conjunto dos números reais, levou à definição dos números complexos. 2. z  z  x 2  y 2.
Para resolver equações como a supracitada, definimos a unidade
imaginária i, tal que 3. z1  z2  z1  z2.
4. z1.z2 =z1  z2.
i² = –1
5. z 1  ( z )1
DEFINIÇÃO z  z
6.  1   1 , z 2  0
Um número complexo z é um número da forma z = x + yi.  z2  z2
A expressão x + yi é chamada forma algébrica de um número z z zz
7. Re(z)= e Im(z)= .
complexo z = (x, y), podemos escrever o conjunto dos complexos da 2 2i
seguinte forma:
 = {x + yi | x ∈ , y ∈ , i2= –1} POTÊNCIAS DE I
x = Re(z); parte real do complexo z. Observe o cálculo das primeiras potências de i:
y = Im(z); parte imaginária do complexo z. i0 = 1;
Se x = 0 e y ≠ 0, z é um número imaginário puro. i1 = i;
i2 = –1;
Se y = 0, z é um número real. i3 = i2 .i = –i;
i4 = i3. I = 1;
OPERAÇÕES i5 = i4 .i = i;
i6 = i5. i = –1;
Sejam z1 = x1 + y1i e z2 = x2 + y2i dois números complexos. i7 = i6. i = –i.
Percebe-se que as potências de i repetem-se de 4 em 4, iremos
IGUALDADE provar isto agora:
z1 = z2 ⇔ Re(z1) = Re(z2) e Im(z1) = Im(z2).
TEOREMA
ADIÇÃO in = i4q+r = (i4)q · ir = ir

z1 + z2 = (x1 + y1i) + (x2 + y2i) = (x1 + x2) + (y1 + y2)i. Com n = 4 ⋅ q + r


Onde r é o resto da divisão de n por 4.
SUBTRAÇÃO Portanto para calcular uma potência de i basta observar o resto da
divisão de n por 4, e para isso basta utilizar o número formado pelos
z1 – z2 = (x1 + y1i) – (x2 + y2i) = (x1 – x2) + (y1 – y2)i.
dois últimos algarismos de n.

MULTIPLICAÇÃO
z1 · z2 = (x1 + y1i) · (x2 + y2i) = (x1 · x2 – y1 · y2) + (x1 · y2 + x2 · y1)i
PLANO DE ARGAND-GAUSS, MÓDULO,
PROPRIEDADES, DISTÂNCIA
DEFINIÇÃO DO CONJUGADO
DE UM NÚMERO COMPLEXO MÓDULO DE UM NÚMERO COMPLEXO
Seja z um número complexo tal que z = x + yi. O número z = x – yi Seja z = x + yi então definimos seu módulo | z | x 2  y 2 .
é chamado o conjugado do complexo z.
PROPRIEDADES
DIVISÃO a. |z| ≥ 0, ∀z ∈ C
b. |z| = |–z| – |z|
Sejam z = a + bi e w = c + di.
z a  bi c  di ac  bd bc  ad c. z · z = |z|²
= .   i d. |z · w| = |z| · |w|
w c  di c  di c2  d2 c2  d2
e. |z| = |w| ≤ |z + w| ≤ |z| + |w|
1 1 a  bi a  bi z
z 1      2. f. |z-1| = |z|-1, z ≠ 0
z a  bi a  bi a2  b2 z
g. z |z|,w≠0
=
w |w|

181

PM_BOOK16 - MAT.indb 181 25/11/2022 19:12:11


NÚMEROS COMPLEXOS

AFIXO DE UM COMPLEXO EXERCÍCIOS DE

Um número complexo z cuja forma algébrica é x + yi pode ser


representado no plano cartesiano R × R pelo ponto M = (x,y). O
ponto M é então chamado de afixo do complexo z. Desta
FIXAÇÃO
 forma,
podemos enxergar o complexo z como sendo o vetor OM e assim
fica perfeitamente plausível a definição que demos para o módulo do
 01. O valor de i11 – i21 – i38 é
complexo z pois OM | z | x 2  y 2 . a) 1 – 2i. c) –2.
b) 2 – i. d) 1.

1
02. O inverso do número complexo z = –2i, z’ = é
z
i 1 c) –2 d) 2i
a) b)
2 2

03. Seja o número complexo z = 1 + i. Se z’ é o conjugado de z, então


o produto |z| · |z’| é igual a
a) 1. b) 2. c) 3. d) 2 3.

04. O número complexo z = (a – 4) + (b – 5)i será um número


imaginário puro se
a) a = 4 e b = 5. c) a ≠ 4 e b = 5.
b) a = 4 e b ≠ 5. d) a ≠ 4 e b ≠ 5.
LUGARES GEOMÉTRICOS
Sejam z, z1, z2 ∈ , então o módulo |z2 – z1| representa a distância
05. Dado x ∈  para que o número z = (2 – xi) (x + 2i) seja real, o
do afixo z1 ao afixo z2.
valor de x pode ser
a) 4. c) –1.
b) 0. d) –2.

06. O módulo do complexo z = –3 + 4i é


a) 3. c) 5.
b) 4. d) 6.

1
07. (ESA) A parte real do número complexo é:
( 2i)
2
1 c) 0
a) −
4 d) 2
b) –2

08. (ESA) O número complexo i102, onde i representa a unidade


a) O conjunto dos pontos tais que |z – z1| = r é uma circunferência de imaginária,
centro em z1 e raio z2. a) é positivo.
b) é imaginário puro.
c) é real.
d) está na forma trigonométrica.
e) está na forma algébrica.

09. (ESA) Com relação aos números complexos Z1 = 2 + i e Z2 = 1 – i,


onde i é a unidade imaginária, é correto afirmar
a) Z1 · Z2 = –3 + i. d) z1 ⋅ z2 =10
b) z1 = 2 e) z1 + z2 =3
c) z2 = 5

10. Se z = 4 – i, então o inverso de z2 é


15 8
a) + i
b) O conjunto dos pontos tais que |z – z1| = |z – z2| é a mediatriz do 289 289
segmento z1z2 . 15 8
b) + i
c) O conjunto dos pontos tais que |z – z1| + |z – z2| = 2a, com 161 161
2a > |z1 – z2| é uma elipse com focos em z1 e z2 e eixo maior igual
15 8
a 2a. c) − i
289 289
d) O conjunto dos pontos tais que |z – z1| – |z – z2| = 2a, com
15 8
2a < |z1 – z2| é um ramo de hipérbole com focos em z1, z2. d) − i
161 161

182

PM_BOOK16 - MAT.indb 182 25/11/2022 19:12:14


NÚMEROS COMPLEXOS

EXERCÍCIOS DE 11. Considere as afirmações sobre as soluções da equação

TREINAMENTO z² – z = 0, com z ∈ :
I. Possui exatamente duas soluções.
II. A soma de todas as soluções é igual a 1.
III. O módulo de todas as soluções é menor ou igual a 1.
01. (EEAR) A parte real das raízes complexas da equação
x² – 4x + 13 = 0, é igual a É(são) verdadeira(s) a(s) afirmação(ões):
a) 1 b) 2 c) 3 d) 4 a) I. c) I, II. e) I, II, III.
b) III. d) II, III.
02. (EEAR) Se i é a unidade imaginária, então 2i³ + 3i² + 3i + 2 é um
número complexo que pode ser representado no plano de Argand- 3 + 2i
12. (MACKENZIE) O resultado da expressão na forma x + yi é
Gauss no __________ quadrante. 1 − 4i
11 14 11 14 1
a) primeiro c) terceiro a) + i c) − i e) 3 − i
17 17 17 17 2
b) segundo d) quarto
5 14 11 14
b) − + i d) − i
17 17 15 15
03. (EEAR) Sabe-se que os números complexos Z1 = [2m(3 + m)] +
(3n + 5)i e Z2 = (2m² + 12) + [4(n + 1)]i são iguais. Então, os valores de
m e n são, respectivamente 13. O módulo de a + bi , para a e b reais, é
a) 3 e 1 c) 2 e –1 a − bi
a) a² + b² c) 1 e) 0
b) 2 e 1 d) 3 e –1
b) 2 d) a² – b²

04. Sendo “i” a unidade imaginária, o resultado de ( 3 + 2i) (6 − 4i) é 14. (MACKENZIE) A solução da equação │z│ + z – 18 + 6i = 0 é um
−1 + 3i
a) –1 – 3i. 13 39 i complexo z de módulo:
d) + .
b) –13 – 39i. 5 5 a) 6 b) 8 c) 18 d) 12 e) 10
13 39 i e) 1 + i.
c) − − . 1+ i
5 5 15. (MACKENZIE) Se y = 2x, sendo x = e i= −1 , o valor de
(x + y)2 é 1− i
05. Sendo i a unidade imaginária, a potência de [(1 – i)² – (1 + i)²]³ é
igual a a) 9i c) –9 e) 9 – i
a) 64 c) 64i e) –32 b) –9 + i d) 9
b) –64 d) –64i
2+i
16. (MACKENZIE) Se tem parte imaginária igual a zero, então
06. Dentro do conjunto dos números complexos, a equação β + 2i
x4 – x2 – 2 = 0 tem como soluções o número real β é igual a
a) ±2 e ± i. a) 4 b) 2 c) 1 d) –2 e) –4
c) ±1 e i 2.
b) ± 2 e ± i. d) ±1 e ± i.
17. (MACKENZIE) Se 3 + 4i é raiz cúbica de um complexo z, então o
produto das outras raízes cúbicas de z é:
07. (FGV) Sendo a unidade imaginária do conjunto dos números
a) –7 + 24 i c) 24 + 7 i e) –7 – 24 i
complexos, o valor da expressão (i + 1)6 – (1 – i)6 é:
b) 7 – 24 i d) –24 – 7 i
a) 0 c) –16 e) –16i
b) 16 d) 16i 18. (MACKENZIE) Se w é um número complexo, satisfazendo
Re(w) > 0 e (w + i)² + |w + i|² = 6, então w é igual a
08. (FGV) Sendo i a unidade imaginária, então (1 + i)20 – (1 – i)20 é
a) –1 – i b) –1+ i c) 1–i d) –1 e) –i
igual a
a) –1024. c) 0. e) 1024i. 19. (FGV) O número complexo z = a + bi, com a e b reais, satisfaz
b) –1024i. d) 1024. z + |z| = 2 + 8i, com a + bi = a2 + b2 . Nessas condições, |z|² é igual a

1+ i a) 68. c) 169. e) 289.


09. Sendo um número complexo, seu conjugado vale
i b) 100. d) 208.
1− i c) 1 + i e) 1 – i
a)
i 20. (AFA) Sobre as raízes da equação ix2 – x + 2i = 0, onde i é a
i
b) 1+ i d) unidade imaginária, podemos afirmar que:
− 1+ i
i a) a soma é 1.
10. Uma das raízes da equação 2x³ + x² – 7x – 6 = 0 é x1 = 2. Pode-se b) o produto e 2i.
afirmar que: i
c) a soma dos inversos é −
.
a) as outras raízes são números imaginários puros. 2
d) são complexos conjugados.
b) as outras raízes são – 3 e – 2.
c) só uma das outras raízes é real. 21. (AFA) Sabendo-se que n é múltiplo de 4, o valor da soma 1 – i +
d) as outras raízes estão entre – 2 e 0. i2 – i3 + i4 – ...... + (–1)n · in é:
e) o produto das raízes é 6. a) 0 b) 1 c) –1 d) i

183

PM_BOOK16 - MAT.indb 183 25/11/2022 19:12:15


NÚMEROS COMPLEXOS

x 3 − 3xy 2 =
1
22. (AFA) A soma dos treze primeiros termos da progressão geométrica 30. (ITA) Sejam x e y números reais tais que:  2 . Então, o
3x y − y =
3
(2i, –2, ...), onde i = −1, é 1
a) 0 c) –2i número complexo z = x + iy é tal que z3 e │z│ valem, respectivamente:
b) 2i d) 2i – 2 a) 1 – i e 6
2 c) ie1 e) 1 + i e 6
2
b) 1 + i e 6
2 d) -i e 1
23. (EFOMM) A solução da equação |z| + z = 1 + 3i é um número
complexo de módulo:
5 c) 5 5
a) e) EXERCÍCIOS DE

COMBATE
4 2
5
b) 5 d)
2

24. (EN) Se z é o conjugado do número complexo z, então o número


de soluções da equação z² = z é
01. (EEAR 2015) Se i é a unidade imaginária, pode-se afirmar que i7 é
a) 0 c) 2 e) 4
a) i c) i³
b) 1 d) 3
b) i² d) i4
25. (AFA) Considere no plano complexo, o conjunto dos números
z = x + yi; {x,y} ⊂  e i² = -1 que satisfazem a condição |z| ≥ |2z + 1|. 02. (ESPCEX 2014) De todos os números complexos z que satisfazem
É FALSO afirmar que a condição |z – (2 – 2i)| = 1, existe um número complexo z1 que fica
mais próximo da origem. A parte real desse número complexo z1 igual
a) este conjunto pode ser representado por um círculo de raio igual
a:
1
a .
3 4− 2 4− 2 2
a) c) e)
b) z = -1 é o elemento de maior módulo, neste conjunto. 2 4 2
1 4+ 2 4+ 2
c) z= − é o elemento de maior argumento, neste conjunto. b) d)
3 2 4
d) não existe z, neste conjunto, que seja imaginário puro.
03. Calcule 3 + 4i :
26. (EFOMM) Resolvendo 1 + i + i² + ... + in, com n = 4k + 1 e k ∈ 
(nos inteiros), obtemos
04. (ITA 2007) Assinale a opção que indica o módulo do número
a) in. c) 1. e) 1 + i.
1
b) 1 + in+1. d) 1 + i². complexo , x  k, k  Z.
1 i cot g x
a) |cos x| d) |cos sec x|
27. (EN) O conjunto S formado por todos os números complexos z que
satisfazem a equação |z – 1| = 2|z + 1| é representado geometricamente b) (1 + sen x) / 2 e) |sen x|
por uma c) cos² x
a) reta vertical. n
5  4
b) circunferência de centro  , 0  e raio .
3  3
05. (AFA 2012) O valor de n tal que  1 i
j1
j
 31 i, sendo i a
unidade imaginária, é
c) parábola com vértice na origem e eixo de simetria 0x. a) par menor que 10. c) ímpar menor que 7
d) elipse de centro (–3,0) e eixo maior horizontal. b) primo maior que 8. d) múltiplo de 9.
e) circunferência de centro  − , 0  e raio .
5 4
 3  3 06. (ESPCEX) A representação geométrica, no Plano de Argand-
Gauss, do conjunto de pontos que satisfazem a condição |z + 2 – 3i| =
| z − 2 |=| z + 4 | |z – 1 + 4i|, com z = x + yi, sendo x e y números reais, é reta de equação
28. (EFOMM) Resolvendo o sistema  , para z
complexo, encontramos como solução | z − 3 | + | z + 3 |=
10 a) 2x – 3y + 7 = 0. d) 4x – 3y + 3 = 0.
8 6 8 6 6 8 6 8 b) 3x – 7y – 2 = 0. e) 2x – y = 0.
a) −1 + i; − 1 − i d) +1 + i; + 1 − i
5 5 5 5 c) 2x – 3y + 3 = 0.

b) 8 6 8 6 8 6 8 6
+1 + i; + 1 − i e) +1 − i; − 1 − i 07. Considere a família de curvas do plano complexo, definida por
5 5 5 5
 1
6 8 6 8 Re    C, onde z é um complexo não nulo e C é uma constante real
c) −1 + i; − 1 − i z
5 5 positiva. Para cada C temos uma
29. (ITA) Seja S o conjunto dos números complexos que satisfazem, a) circunferência com centro no eixo real e raio igual a C.
simultaneamente, às equações: |z – 3i| = 3 e |z + i| = |z – 2 – i|. O 1
b) circunferência com centro no eixo real e raio igual a .
produto de todos os elementos de S é igual a C
a) −2 + i 3 1
c) circunferência tangente ao eixo real e raio igual a .
2C
b) 2 2 + 3i 3 1
d) circunferência tangente ao eixo imaginário e raio igual a .
c) 3 3 − 2i 3 2C
e) circunferência com centro na origem do plano complexo e raio
d) –3 + 3i
1
e) –2 + 2i igual a .
C

184

PM_BOOK16 - MAT.indb 184 25/11/2022 19:12:23


NÚMEROS COMPLEXOS

2(a + bi)
08. (ESPCEX) Sendo z o conjugado do número complexo Z e i a
unidade imaginária, o número complexo Z que satisfaz à condição
z + 2z = 2 – zi é
5 (ITA) Considere a equação (a − bi)501 =
(a2 + b2 )250 + 1
número de pares ordenados (a,b) ∈ ² que satisfazem a
. O

equação é
a) z = 0 + 1i c) z =1 + 0i e) z = 1 – i
a) 500.
b) z = 0 + 0i d) z = 1 + i
b) 501.
x  yi c) 502.
09. Seja o número complexo z  , com x e y reais e i² = -1. Se
3  4i d) 503.
x² + y² = 20 , então o módulo do complexo z é igual a:
e) 504.
a) 0 2 5 d) 4 e) 10
b) 5 c)
5
10. (ITA 1987) Considerando z e w números complexos arbitrários e GABARITO
u = z · w + z · w, então o conjugado de u será necessariamente:
EXERCÍCIOS DE FIXAÇÃO
a) igual à |z| |w|.
01. A 04. B 07. A 10. A
b) um número imaginário puro.
02. A 05. D 08. C
c) igual ao dobro da parte real de z + w.
03. B 06. C 09. D
d) igual ao dobro da parte real do número z · w.
EXERCÍCIOS DE TREINAMENTO
e) diferente de u.
01. B 09. C 17. A 25. C
02. B 10. D 18. C 26. E
03. B 11. B 19. E 27. E

DESAFIO PRO
04. C 12. B 20. C 28. A
05. C 13. C 21. B 29. D
06. B 14. E 22. B 30. B

1 (IME) Sejam os complexos z = a + bi e w = 47 + ci, tais que 07. E 15. C 23. D


z³ + w = 0. Determine o valor de a, b e c, sabendo que esses 08. C 16. A 24. E
números são inteiros e positivos. EXERCÍCIOS DE COMBATE
01. C 04. E 07. D 10. D
2 (ITA) O lugar geométrico das soluções da equação
x² + bx + 1 = 0, quando |b| < 2, b ∈ , é representado no
plano complexo por
02. A 05. D
03. DISCURSIVA 06. B
08. D
09. C
a) dois pontos. DESAFIO PRO
b) um segmento de reta. 01. a = 1 → b = 4 02. C 05. D
c) uma circunferência menos dois pontos. a = 47 → b ∉  03. B
d) uma circunferência menos um ponto. c = 52 04. C
e) uma circunferência.
ANOTAÇÕES

3I.
(ITA) Considere as afirmações a seguir:

Se z e w são números complexos tais que z – iw = 1 – 2i e


w – z = 2 + 3i, então z² + w² = –3 + 6i.
II. A soma de todos os números complexos z que satisfazem
2|z|² + z² = 4 + 2i é igual a zero.
III. Se z = 1 – i, então z59 = 229 (–1 + i).
É (são) verdadeira(s)
a) apenas I.
b) apenas I e II.
c) apenas I e III.
d) apenas II e III.
e) I, II e III.

4 (ITA) Sejam A, B e C os subconjuntos de  definidos por


A = {z ∈ : |z + 2 – 3i| < 19 }, B = {z ∈ : |z + i| < 7/2} e
C = {z ∈ : z² + 6z + 10 = 0}. Então, (A\B) ∩ C é o conjunto
a) {–1 - 3i, –1 + 3i}.
b) {–3 – i, –3 + i}.
c) {–3 + i}.
d) {–3 – i}.
e) {–1 + 3i}.

185

PM_BOOK16 - MAT.indb 185 25/11/2022 19:12:25


NÚMEROS COMPLEXOS

ANOTAÇÕES

186

PM_BOOK16 - MAT.indb 186 25/11/2022 19:12:25


COMPLEXOS NA FORMA
TRIGONOMÉTRICA

FORMA TRIGONOMÉTRICA CONJUGADO NA FORMA


DE COMPLEXOS TRIGONOMÉTRICA
Seja z1 = r1 · cis θ1 = r1 · (cosθ1 + isenθ1), o conjugado de z, na
forma trigonométrica, é z1 = r1 · (cosθ1 – isenθ1) = r1 · (cos(–θ1) + isen
(–θ1)) = r · cis(–θ1).

DIVISÃO NA FORMA
TRIGONOMÉTRICA
Sejam z1 = r1 · cis θ1 e z2 = r2 · cisθ2 com z1 ≠ 0.
z2 z2.z1 rr cis  2  1  r2cis  2  1  r2
  12   .cis  2  1 
11cis  1  1 
z1 z1.z1 rr r1cis0 r1

CONCLUSÃO: dividem-se os módulos e subtraem-se os


argumentos.
r2.cisq2 r2
 .cis  2  1 
r1.cisq1 r1

PRIMEIRA FÓRMULA DE MOIVRE


Consideremos o complexo z = r · (cosθ + isenθ) = r · cisθ e seja
dado o número natural n, temos:
Seja z = (a, b) = a + bi.
zn = rn · (cos nθ + isen nθ) = rn · cisnθ
r  a2  b2 módulo do complexo z.
Exemplo:
a b
cos = ; sen =  a  r cos  e b = rsen z = 3 + i ⇒ z = 2[cos (π/6) + i.sen (π/6)]
r r
z4 = 24[cos (4π/6) + i · sen (4π/6)] = 16[cos (2π/3) + i · sen (2π/3)] =
z = r · (cosθ + isenθ) = r · cisθ. 16[–1/2 + 3 i/2] = –8 + 8 3 i
Com θ ∈ [0,2π], θ é o argumento principal.
Isto é, a forma trigonométrica do número complexo z é: SEGUNDA FÓRMULA DE MOIVRE
z = r (cosθ + i · senθ), que se abrevia z = rcisθ. Nos números reais sabemos que 4 16 = 2..
ProBizu No corpo dos números complexos temos que
24 = 16; (–2)4 = 16, (2i)4 = 16; (–2i)4 = 16.
Se b = 0, z é um número real.
Então o número 16 em C tem 4 raízes quartas.
Se b ≠ 0 e a = 0, z é um imaginário puro.
Dados complexo z = r · (cosθ + isenθ) = r · cisθ e o número natural
n (n≥ 2), então existem n raízes enésimas de z que são da forma:
MULTIPLICAÇÃO DE COMPLEXOS NA    2k 
wk  n z  n r . cos  
  2k  
  i.sen   
n n  n n  
FORMA TRIGONOMÉTRICA 
Considere os complexos de módulos r1 e r2 com argumentações com k ∈  variando de 0 até n–1.
θ1 e θ2. Como n r é constante e os argumentos diferem de 2π/n (para
r1.cis1  . r2.cis2   r1  cos 1  i sen1  .r2  cos 2  i sen2  valores consecutivos de n), conclui-se que as imagens das n raízes de
um número complexo são vértices de um polígono regular de n lados,
 2
1 2 cos 1 cos 2  i sen1sen2  i cos 1sen2  isen1 cos 2
rr  inscrito numa circunferência de centro na origem e raio n r , tendo
uma das raízes o argumento θ/n.
1 2  cos 1 cos 2  sen1sen2  i  cos 1sen2  sen1 cos 2  
rr
Exemplo:
1 2  cos  1  2   i sen  1  2  
rr
Determinar as raízes cúbicas de z = 8
CONCLUSÃO: multiplicam-se os módulos e somam-se os 2 2
z | = 8 + 0 = 8 ∴ cosθ = 1 senθ = 0 ⇒ θ = 0 ∴ z = 8(cos 0 +
argumentos. i · sen 0)
r1.cis1  . r2.cis2   rr1 2.cis  1  2   0  2k 0  2k   2k 2k 
wk  3 8 cos  i.sen  2 cos  i.sen
 3 3   3 3 

187

PM_BOOK16 - MAT.indb 187 25/11/2022 19:12:36


COMPLEXOS NA FORMA TRIGONOMÉTRICA

O número k deve variar entre 0 e 2. 04. (EFOMM) Seja o número complexo z =−1 − 3i, onde i é a unidade
k = 0 ⇒ w0 = 2(cos 0 + i · sen 0) = 2(1 + 0i) = 2 imaginária. O valor de z8 é:
k = 1 ⇒ w1 = 2(cos 2π/3 + i · sen 2π/3) = 2(–1/2 + 3i/3) = – 1 + 3i =  4π 4π   2π 2π 
a) z 256  cos + isen = d) z 256  cos + isen 
k = 2 ⇒ w2 = 2(cos 4π/3 + i · sen 4π/3) = 2(–1/2 – 3i/2) = – 1 – 3i  3 3   3 3 
 π π e) z 256 ( cos 2π + isen2π )
=
=
b) z 256  cos + isen 
RAÍZES ENÉSIMAS DA UNIDADE  3 3
As raízes da equação zn– 1=0 são chamadas as raízes da unidade. =  5π 5π 
c) z 256  cos + isen 
As raízes da unidade são:  3 3 

2k 2k
k  cos  isen , k  {0,1,2,...,n-1}.  π π
n n 05. (ESPCEX) Se (1 + i)  cos + isen  = x + iy, em que i é a unidade
 12 12 
imaginária e x e y são números reais, o valor de 3 ⋅ x + y é

EXERCÍCIOS DE a) 6 d) 3 6

FIXAÇÃO b)

c)
2
3
e)
2
3

01. Passe o complexo Z = 1 + 3i para a forma trigonométrica. 06. Um quadrado ABCD está inscrito num círculo com centro na
origem do plano de Gauss. O vértice “A” é imagem do complexo
3 + 4i. Os afixos dos outros três vértices são os complexos:
3 i
02. Qual o argumento do número complexo=
Z − ? a) –3 + 4i; –3 – 4i; 3 – 4i. d) –3 + 4i; –3 – 4i; 4 – 3i.
2 2
b) –4 + 3i; –3 – 4i; 4 – 3i. e) i, – i, 1 + i.
03. Passe para a forma trigonométrica o número complexo Z = 1 – i.
c) –4 + 3i; – 3 – 4i; 3 – 4i.

1 3 07. (AFA) Seja P o produto dos fatores (sen nO + cos nO), onde n = 45,
04. Sendo Z1= + i e Z2 = –1 – i, calcule Z1 · Z2.
2 2 46, 47, ..., 149, 150. Pode-se afirmar que

1 3 a) P = 0 c) 1≤P<8
05. Sendo Z1= + i e Z2 = –1 – i, calcule (Z1)³. b) P = 290 d) 8 ≤ P ≤ 290
2 2

(Z )
3
1 3
08. (AFA) Considere no campo complexo uma curva tal que Im   ≥ k,
06. Sendo Z1= + i e Z2 = –1 – i, calcule 1 . 2
2 2 Z2 z
onde z é um complexo não nulo. Se k = 2, tem-se sua representação
07. Sendo Z = 1 + i, qual deve ser o menor número n, com n ∈ *,
gráfica dada pelo
para que Zn seja um número real positivo?
1
a) círculo de raio e tangente ao eixo real.
3 i 4
6
08. Para o complexo Z =
− + encontre todas as raízes de Z. 1
2 2 b) círculo de raio e tangente ao eixo imaginário.
2
09. Passe para forma trigonométrica o complexo −1 − 3i . c) conjunto de pontos do plano complexo exterior ao círculo de raio
1  1 
e centro  − , 0  .
10. Encontre o argumento do número complexo – 1 – i. 2  2 
1
d) círculo de raio e tangente ao eixo real.
2
EXERCÍCIOS DE
09. (AFA) Dado o número complexo z tal que z + 2z – 9 = 3i, é correto

TREINAMENTO afirmar que:


a) z = 3 10 c) z = 9 – 3i
 7π 7π  d) 3i
=
b) z 3 2  cos + isen 
01. Determine a forma trigonométrica do número complexo dado:  4 4 
a) z = 1 + i c) z = –1 + i e) z = –3 4
a b  π π
b) z= 1+ i 3 d) z = 2i 10. (ESPCEX) Seja a igualdade − =i  cos + isen  , onde i é a
3 5  6 6
unidade imaginária. Se a e b são números reais, então o quociente
02. Dados z = 2 · cis50° e w = 3 · cis40°, calcule: a
é igual a
a) z · w b
b) w3 3 3
a) . d) − .
c) z6 5 5
3 3 15 3
03. (FUVEST) Seja z um número complexo de módulo 2 e argumento b) . e) .
principal 120°. O conjugado de z é: 5 4
a) 2 − 2i 3 c) −1 − i 3 e) 1+ i 3 3 3
c) − .
b) 2 + 2i 3 5
d) −1 + i 3

188

PM_BOOK16 - MAT.indb 188 25/11/2022 19:12:40


COMPLEXOS NA FORMA TRIGONOMÉTRICA

11. (ESPCEX) Sejam z e v números complexos onde |z| = 1 e v tem


a) 2 3 c) 1+ 2 2
 2 2
coordenadas no plano de Argand-Gauss  ,  . Sobre o número b) 3+ 3 d) 2+2 2
 2 2 
complexo z e v (resultante da multiplicação dos complexos z e v),
17. (AFA) Seja z um número complexo não nulo e i a unidade
podemos afirmar que
imaginária (i² = –1), z ≠ i. O conjunto de todos os valores de z, para os
a) sempre é um número real. r +i
quais é um número real, representa um(a)
b) sempre tem módulo igual a 2. 1 + ir
c) sempre é um número imaginário puro. a) elipse. c) hipérbole.
d) pertence à circunferência x² + y² = 1. b) circunferência. d) círculo.
π
e) sempre tem argumento igual a .
4 18. (AFA) Considere no Plano de Argand-Gauss os números complexos
z1 = –x – 2i, z2 = –2i, z3 = –2 + 3i e z4 = x + yi, onde x e y são
12. Sendo “w” um número complexo e z1 = 2 · (cos 10° + i · sen 10°)
números reais quaisquer e (i² = –1). Sobre o conjunto desses números
a sua raiz oitava de menor argumento, a soma dos argumentos
complexos que atendem simultaneamente às condições
principais de todas as raízes oitavas de “w” é:
I. Re(z1 · z2) ≤ Im(z1 · z2)
a) 256° c) 1180° e) 2680°
II. |z3 + z4| ≤ 2
b) 1600° d) 1340°
é correto afirmar que
13. (AFA) Analise as alternativas e marque a correta. a) representa uma região plana cuja área é menor que 6 unidades
a) Dado o complexo z = m + mi, onde m ∈ * e i é a unidade de área.
imaginária, pode-se dizer que o afixo de (z)² é, em relação à b) possui vários elementos que são números imaginários puros.
origem, simétrica do afixo (–2m², 0). c) possui vários elementos que são números reais.
b) No plano de Argand-Gauss os complexos z, tais que |z – 1| ≤ 1, d) seu elemento z de menor módulo possível possui afixo que
são representados pelos pontos do círculo de centro (0, 1) e raio pertence à reta (r) 3x + 2y = 0.
unitário.
c) Se n ∈  e i a unidade imaginária, então (in+1 + in)8 é um número 19. (ESPCEX) No plano complexo, temos uma circunferência λ de raio
real maior do que zero. 2 centrada na origem. Sendo ABCD um quadrado inscrito à λ, de
d) Se z = a + bi (a ∈ *, b ∈  e i é a unidade imaginária) é acordo com a figura abaixo, podemos afirmar que o número complexo
um complexo, então z – z é sempre um número complexo que representa o vértice B é
imaginário puro.

14. (AFA) Analise as sentenças abaixo, classificando-as em


VERDADEIRA(S) ou FALSA(S), considerando i= −1. A seguir, assinale
a alternativa que apresenta a sequência correta.
I. A representação geométrica dos números complexos z tais que
z − (1 − i) ≤ 2 é um círculo de centro C (1, −1) e raio 2.
1+ i  7π 7π  1 3
II. A forma trigonométrica de z == éz 2  cos + isen  . a) − + i.
i  4 4  2 2
Z cosα + isenα , então z ⋅ z = −i , ∀α ∈ .
III. Se =
2
b) − 3 − i.
a) V,V,V b) V,V,F c) F,F,V d) V,F,V c) −1 + 3i.
1 3
15. (AFA) O número complexo z = a + bi é vértice de um triângulo d) − − i.
2 2
equilátero, como mostra a figura abaixo.
3 1
e) − + i.
2 2
20. (AFA) Considere todos os números complexos z = x + yi, onde
2
x ∈ , y ∈  e i= −1, tais que z − −1 ≤ .
1+ i
Sobre esses números complexos z, é correto afirmar que
a) nenhum deles é imaginário puro.
b) existe algum número real positivo.
c) são todos imaginários.
É correto afirmar que o conjugado de z² tem afixo que pertence ao
d) apenas um é número real.
a) 1º quadrante. c) 3º quadrante.
b) 2º quadrante. d) 4º quadrante. 21. (AFA) Sejam z = x + yi (x ∈ *, x ∈ * e i a unidade imaginária),
z o conjugado de z e λ o lugar geométrico dos pontos P(x,y) do plano
16. (AFA) Considerando os números complexos z1 e z2, tais que: z = 2x + 3. Se A e B são os pontos de
cartesiano para os quais z · 
• z1 é a raiz cúbica de 8i que tem afixo no segundo quadrante. de λ com o eixo Oy e se A’ é o ponto de intersecção de λ
intersecção 
com o eixo Ox que possui a menor abscissa, então a área do triângulo
• z2 é raiz da equação x4 + x2 – 12 = 0 e Im(z2) > 0.
A’AB é, em unidades de área, é igual a
Pode-se afirmar que |z1 + z2| é igual a
a) 2 3 b) 2 2 c) 3 d) 2

189

PM_BOOK16 - MAT.indb 189 25/11/2022 19:12:41


COMPLEXOS NA FORMA TRIGONOMÉTRICA

22. (EFOMM) O número complexo, z = |z| · (cos θ + i · sen θ), sendo i 29. (EN) Qual valor de n, n inteiro maior que zero, para que (1 + i)n
a unidade imaginária e 0 ≤ θ ≤ 2π, que satisfaz a inequação |z + 3i| ≤ 2 seja um número real?
e que possui o menor argumento θ, é a) 2 c) 4 e) 6
5 2 5 2 5 5 e) z=
−2 5 + 5i b) 3 d) 5
a) z =− − i c) z=
− − i
3 3 3 3
2 5 5 30. (EN) Seja p a soma dos módulos das raízes da equação
5 2 5
b) z =− + i d) z=
− + i x³ + 8 = 0 e q o módulo do número complexo Z, tal que ZZ = 108,
3 3 3 3
onde Z é o conjugado de Z. Uma representação trigonométrica do
número complexo p + qi é
23. (AFA) Considere, no plano de Argand-Gauss, os números
complexos A e B, sendo A = x – 2i, x ∈  e B = 1 + i. Se no produto  π π  π π
a) 12  cos + i sen  d) 20 2  cos + i sen 
A · B tem-se Re (A · B) ≥ Im(A · B), então, sobre todos os números  3 3  6 6
complexos A, é correto afirmar que  π π  π π
b) 20  cos + i sen  e) 10  cos + i sen 
a) seus afixos formam uma reta.  3 3  3 3
b) nenhum deles é imaginário puro.  π π
c) 12  cos + i sen 
c) o que possui menor módulo é o que tem o maior argumento principal.  6 6
d) existe A tal que |A| = |B|.
31. (EN) Seja z um número complexo e i a unidade imaginária.
Determine z de forma que o triângulo de vértices i, z e iz seja equilátero
24. (AFA) Resolva a equação z3 – 1 = 0 no conjunto dos números
e assinale a opção correta.
complexos. Considerando as raízes encontradas, analise as proposições
−5 πi πi
abaixo e classifique-as em V (VERDADEIRA) ou F (FALSA).
( 3 − 2)e 4
( 3 + 2)e 4
( ) A equação possui três raízes de multiplicidade 1. a) z= ou z = −
2 2
( ) Os afixos das raízes formam um triângulo equilátero cuja área é πi −πi

3 3 ( 5 + 3)e 6 ( 5 − 3)e 6
unidades de área. b) z = ou z = −
2 2 2
−3πi πi
( ) Duas das raízes são conjugadas.
( 6 + 3)e 4 ( 6 − 3)e 4
( ) Todas as raízes têm o mesmo módulo. = c) z = ou z
2 2
A sequência correta é πi 5 πi

a) V – F – V – V c) F – F – V – F ( 6 − 2)e 4 ( 6 + 2)e 4
= d) z = ou z
b) V – V – F – V d) V – F – V – F 2 2
11πi πi

25. (AFA) Considere no Plano de Argand-Gauss os números complexos ( 3 + 2)e 6


( 3 − 2)e 6
e) z= ou z = −
z = x + yi, onde i= −1 e cujos afixos são os pontos P(x,y) ∈ ² 2 2
Dada a equação (z – 1 + i)4 = 1, sobre os elementos que compõem seu 32. (ITA) Seja z um número complexo satisfazendo Re(z) > 0 e (z + i)2
conjunto solução, é INCORRETO afirmar que + |z’ + i|2 = 6, onde z’ é o conjugado de z. Se n é o menor natural para
a) apenas um deles é imaginário puro. o qual zn é um imaginário puro, então n é igual a:
b) todos podem ser escritos na forma trigonométrica. a) 1 c) 3 e) 5
c) o conjugado do que possui maior argumento é 1 + 2i. b) 2 d) 4
d) nem todos são números imaginários.
93
 2
33. (ITA) O valor da potência  é:
1 3  1 + i 
26. (EN) Sabendo que z é o número complexo z= + i, qual o  
2 2
−1 + i −1 − i
( 2)
93
menor inteiro positivo n, para o qual o produto z · z² · z³ ... zn é um a) c) e) +i
real positivo? 2 2
1+ i
( )
93
a) 1 c) 3 e) 5 b) d) 2
b) 2 d) 4 2

27.
(EN) Considere os números complexos da forma
34. (ITA) Considere os números complexos z =2 +i 2 e w =
1 + i 3. ( )
(w + 3z 4 + 4i)
2
6

 π  Se m = , então m vale
=zn pcis  (17 − n),  , com n ∈ *. O menor número natural n, tal
 50  (z 2
+ w 3 + 6 − 2i)
que o produto z1 · z2 · ... · zn é um número real positivo, é igual a
a) 34 c) 16 e) 1
a) 8 c) 25 e) 50
b) 26 d) 4
b) 16 d) 33
35. (ITA) O conjunto de todos os números complexos z, z ≠ 0, que
28. (EN) Desenha-se no plano complexo o triângulo T com vértices satisfazem à igualdade |z + 1 + i| = | |z| – |1 + i||é:
nos pontos correspondentes aos números complexos z1, z2, z3, que
a) {z ∈ C: arg z = (5π/4) + 2kπ, k ∈ Z}
são raízes cúbicas da unidade. Desenha-se o triângulo S, com vértices
nos pontos correspondentes aos números complexos w1, w2, w3, que b) {z ∈ C: arg z = (π/4) + 2kπ, k ∈ Z}
são raízes cúbicas de 24 3. Se A é a área de T e B é a área de S, então c) {z ∈ C: │z│ = 1 e arg z = (π/6) + kπ, k ∈ Z}
a) B = 12A c) B = 24A e) B = 42A d) {z ∈ C: arg z = (π/4) + kπ, k ∈ Z}
b) B = 18A d) B = 36A

190

PM_BOOK16 - MAT.indb 190 25/11/2022 19:12:42


COMPLEXOS NA FORMA TRIGONOMÉTRICA

EXERCÍCIOS DE 1 i 3
07. (AFA) Considere o número complexo z= − e calcule Zn. No

COMBATE
2 2
conjunto formado pelos quatro menores valores naturais de n para os
quais Zn é um número real,
a) existem números que estão em progressão aritmética de razão
igual a 4.
01. (EFOMM 2000) Escrevendo-se na forma trigonométrica o
b) há elementos cuja soma é igual a 30.
3  3i
complexo z  , encontra-se: c) existem um único número ímpar.
i
d) existe apenas um elemento que é número primo.
      7   7 
a) 2 3 cos    isen    d) cos    isen  
 6
   6  6
   6  08. Analise as afirmativas abaixo referentes aos números complexos
 4   4  4   4  3 1
b) 2 3 cos    isen   e) cos    isen   =
z + e w = 1 – i.
  3   3   3   3  2 2
(01) | z | · w10 é um número imaginário puro.
  7   7  
c) 2 3 cos    isen   
 6   6   1 1
(02) O afixo de w-1 é o ponto  ,  .
 2 2
02. (AFA 99) A representação trigonométrica do conjugado do 11π 11π
=
(04) A forma trigonométrica de z cos + isen .
número complexo z = (1 + 3i)5, sendo i a unidade imaginária e 6 6
k ∈ Z, é:
(08) As raízes quartas de w são vértices de um quadrado inscrito numa
a) 32cos (π/3 + 2kπ) – 32isen (π/3 + 2kπ). circunferência de centro na origem e raio r = 4 2.
b) 32cos (5π/4 + 10kπ) – 32isen (5π/4 + 10kπ). Somando-se os números associados às afirmativas verdadeiras obtém-
c) 32cos (5π/6 + 10kπ) – 32isen (5π/6 + 10kπ). se um total t, tal que:
d) 32cos (5π/3 + 10kπ) – 32isen (5π/3 + 10kπ). a) t ∈ [1,4] c) t ∈ [9,12]
b) t ∈ [5,8] d) t ∈ [13,15]
03. (AFA 2014) Considere os números complexos z1 = x – 1,
1 09. (ESPCEX) A figura geométrica formada pelos afixos das raízes
z2 = i , z = –1 + 2i e z4 = x + yi em que x ∈ , y  * e complexas da equação x3 – 8 = 0 tem área igual a
2 3
i² = –1 e as relações: a) 7 3 c) 5 3 e) 3 3

I. Re  z1  z2   Im  z1  z2  b) 6 3 d) 4 3

II. z3  z 4  5 10. (ESPCEX) Sendo z o número complexo obtido na rotação de 90º,


O menor argumento de todos os complexos z4 que satisfazem, em relação à origem, do número complexo 1 + i, determine z3:
simultaneamente, as relações I e II é a) 1 – i c) –2i e) 2 + 2i
π π π b) –1 + i d) –1 – 2i
a) b) 0 c) d)
6 2 3

04. (ITA 2012) Seja z = n² (cos 45° + isen 45°) e w = n(cos 15° + isen 15°),

DESAFIO PRO
z
em que n é o menor inteiro positivo tal que (1 + i)n é real. Então,
é igual a w

a) 3 +i . c) 2  2  i . e) 2  3  i .
2  3  i .
10
b) 2  2  i .  1 + 3i 
1
d) (ITA) Se z =   , então o valor de 2 arcsen (Re(z)) + 5
 1 − 3i 
05. (ITA) O número complexo z a seguir possui argumento igual a 45°. arctg (2Im(z)) é igual a
Determine o valor de a. 2π 2π 5π
a) − . c) . e) .
1 cos a 1 2 cos a  sena 3 3 3
z  i. ; a   0.  2 
sena.cos a sen2a π 4π
b) − . d) .
3 3
06. (AFA) Os pontos A, B e C são afixos das raízes cúbicas do número
complexo z. Se n é o menor natural não nulo para o qual zn é um real
positivo, então n é igual a
2 (ITA) Sabe-se que -2 + 2i é uma das raízes quartas de um
número complexo z. Então, no plano de Argand-Gauss, a
área do triângulo, cujos vértices são as raízes cúbicas de z, é
igual a
a) 8
a) 4( 3 + 1).
b) 6
c) 4 b) 6 3.
d) 2 c) 8( 3 − 1).
d) 10 3.
e) 12 3.

191

PM_BOOK16 - MAT.indb 191 25/11/2022 19:12:55


COMPLEXOS NA FORMA TRIGONOMÉTRICA

2Z
3 (IME) Seja Z um número complexo tal que
argumento igual a
3π Zi
possui
e log3 (2Z + 2Z + 1) = 2. Determine
03. C
04. D
12. D
13. C
21. C
22. C
30. A
31. D
o número complexo Z. 4 05. A 14. A 23. C 32. B
06. B 15. C 24. A 33. A

4 (IME) Seja z um número complexo tal que z12 ∈ , Re(z) = 1


 π
e arg(z) ∈  0,  . A soma dos inversos dos possíveis valores
07. A
08. D
16. A
17. B
25. C
26. C
34. B
35. A
 2
de |z| está no intervalo: 09. B 18. D 27. A
 1 3 10. A 19. C 28. A
a)  , 
2 2 11. D 20. D 29. C
3 5 EXERCÍCIOS DE COMBATE
b)  , 
2 2 01. C 04. B 07. D 10. E
5 7 02. D 05. a = π/6 08. B
c)  ,  03. D 06. C 09. E
2 2
7 9 DESAFIO PRO
d)  , 
2 2 01. D 03. DISCURSIVA
 9 11 02. E 04. C
e)  , 
2 2 
ANOTAÇÕES

GABARITO
EXERCÍCIOS DE FIXAÇÃO
π
01. z 2 ( cos60° + isen60°=
= ) ou z 2cis
3
11π
02. =
θ ou=
θ 330°
6

=
03. z 2 ( cos 315° + isen315
= ° ) ou z 2cis
4
19π
=
04. z 2 ( cos 285° + isen285
= ° ) ou z 2cis
15
05. z = 1( cos180° + isen180° ) = −1

2 2
=
06. z
2
( cos ( −45°) + isen ( −45
= °))
2
( cos 315° + isen315° )
07. n = 8
08. cis25°, cis85°, cis145°, cis205°, cis265°, cis325°

09. z 2 ( cos 240° + isen240=
= ° ) ou z 2cis
3

10. =
θ ou=
θ 225°
4
EXERCÍCIOS DE TREINAMENTO
01.
 π π
=
a) z 2  cos + i sen 
 4 4
π π
b) z 2  cos + i sen 
=
 3 3
 3π 3π 
=
c) z 2  cos + i sen 
 4 4 
 π π
=
d) z 2  cos + i sen 
 2 2
e)=z 3 ( cos π + i sen π )
02.
a) z=
⋅ w 6cis90
= 0
6i.
27 27 3
b) w 3 =27cis1200 =− + i.
2 2
c) z=
6
64cis300=
0
32 − 32 3i.

192

PM_BOOK16 - MAT.indb 192 25/11/2022 19:12:57


POLINÔMIOS: CONCEITOS E
ALGORITMO DA DIVISÃO

DEFINIÇÃO Quando dois polinômios são idênticos, os seus coeficientes são


ordenadamente iguais.
Polinômios de uma variável são expressões que podem ser escritas
como soma finita de monômios do tipo: aktk onde k ∈ , ak podem ser
números reais ou complexos. RAÍZES
Exemplos: Chamam-se raízes do polinômio P(x) os valores de x ∈ C tais que
a) 3t³ - 2t² + 5t - 7 é um polinômio. P(x) = 0.
Um polinômio de grau n possui exatamente n raízes complexas.
b) ( 3  1)t2  7 é um polinômio. Desta forma, a quantidade de raízes reais é no máximo igual a n.
c) 9t + it + (2 + i)t² - 5t + 4 é um polinômio
5 3
Exemplo:
d) x  8x 2  5x  1 não é um polinômio. O polinômio P(x) = x2− x − 2 é um polinômio completo de grau 2
e) 1 e possui duas raízes reais: −1 e 2.
+ t não é um polinômio.
t
FUNÇÃO POLINOMIAL GRAU
Definição: Uma função de uma variável x é uma função polinomial Dado um polinômio P(x) com pelo menos um termo de coeficiente
complexa se pudermos escrevê-la na forma: não nulo, o grau de P, indicado por gr(P) é o maior dos expoentes da
p(x) = a0 + a1x + a2x² + ... + anxn ∀x ∈ C. variável x nos termos com coeficientes não nulos.
p:  → . Se P tem todos os coeficientes nulos, não se define o grau de P.
Onde Exemplo:
a0, a1, a2, ..., an são números complexos chamados de coeficientes P(x) = 2x3 − 2x2 + 4x +1 ⇒ gr(P) = 3
da função polinomial. P(x) = 5x ⇒ gr(P) = 1
se an ≠ 0, dizemos que p tem grau n. P(x) = 7 ⇒ gr(P) = 0
Se α é um número complexo e p(α) = 0, α é raiz de p(x).
Um polinômio completo é aquele que não possui coeficientes OPERAÇÕES COM POLINÔMIOS
nulos. Um polinômio completo de grau n possui n+1 termos.
ADIÇÃO E SUBTRAÇÃO DE POLINÔMIOS
VALOR NUMÉRICO A adição e a subtração de polinômios são feitas somando-se ou
O valor numérico de p(x) em a (a ∈ C) é a imagem de a pela função p subtraindo-se os coeficientes dos termos de mesmo grau em todas
as variáveis.
Exemplos:
Exemplos:
P(x) = 2x4 − 5x3 + 2x2 −x + 1⇒ P(2) = 2 ⋅ 24 − 5 ⋅ 23 + 2 ⋅ 22 − 2+1= −1
1. (4x2− 3x) − (x2 − 4x − 3) = 3x2 + x + 3
P(x) = x3 − 2ix2 −x +(3i − 2) ⇒ P(i) = i3 −4i ⋅ i2 −i +(3i − 2) = 5i − 2
2. (x3 − 1) + (x4 − x3 +1) = x4
P(x) = x3 + 3x2 + 2x ⇒ P(−1) = (−1)3 + 3 ⋅ (−1)2 + 2 ⋅ (−1) = 0

Observação MULTIPLICAÇÃO DE POLINÔMIOS


(1) = a0 + a1 + a2 + ... + an−1 + an é a soma dos coeficientes. Para multiplicar polinômios basta aplicar a distributividade da
multiplicação.
P(0) = a0 é o termo independente.
Exemplo 1:
(x3 +2x −1)⋅(x2 + x + 2) =
x5+ x4 + 2x3 + 2x3 + 2x2 + 4x − x2 − x − 2 =
POLINÔMIO IDENTICAMENTE NULO
x5 + x4 + 4x3 + x2 + 3x − 2
Chama-se polinômio identicamente nulo o polinômio p(x) que
tem todos os seus coeficientes nulos: Note que se o produto de dois polinômios é nulo, pelo menos um
dos polinômios deve ser nulo.
p(x) = 0 + 0 · x + 0 · x 2 + … + 0 · xn , isto é que assume valor zero
para todos os valores de x. p ⋅ q = 0 ⇔ p = 0 ou q = 0
Indicamos por p(x) ≡ 0. Observação
O grau do produto é a soma dos graus dos fatores.
IDENTIDADE DE POLINÔMIOS gr(p ⋅ q) = gr(p) + gr(q)
Dois polinômios são ditos idênticos quando têm sempre o mesmo
valor qualquer que seja o valor atribuído à variável.

193

PM_BOOK16 - MAT.indb 193 25/11/2022 19:12:59


POLINÔMIOS: CONCEITOS E ALGORITMO DA DIVISÃO

DIVISÃO DE POLINÔMIOS 04. (UECE) Se o polinômio p(x) = x5 + ax³ + x é divisível pelo polinômio
d(x) = x³ + bx, onde a e b são números reais, então, a relação entre a e b é:
Dados dois polinômios P(x) e D(x), de graus p e q, respectivamente,
dividir P(x) por D(x) significa é encontrar dois polinômios Q(x) e R(x), a) a² + ab + b² = 0
denominados quociente e resto, respectivamente, que satisfazem b) b² – ab + 1 = 0
P(x) = D(x) ⋅ Q(x) + R(x) c) a² – ab + 1 = 0
onde o grau de R(x) deve ser menor que o grau de D(x) ou R(x) = 0. d) b² – ab + b = 0
Se gr(P) ≥ gr(D), a divisão pode ser efetuada pelo seguinte
algoritmo denominado Método da Chave. 05. (UECE) O resto da divisão do polinômio D(x) = x5 – 5x³ + 4 pelo
Ordenam-se P(x) e D(x) segundo as potências decrescentes de x, polinômio d(x) = x³ – x² – 4x + 1 é o polinômio do segundo grau r(x).
inclusive com os termos do dividendo que possuem coeficiente 0. A solução real, não nula, da equação r(x) = 0 pertence ao intervalo:
Divide-se o primeiro termo de P(x) pelo primeiro termo de D(x), a) [0,1] c) [3,4]
obtendo-se o primeiro termo do quociente. b) [2,3] d) [–1,0]
Multiplica-se D(x) pelo primeiro termo do quociente e subtrai-se o
resultado de P(x), obtendo-se o primeiro resto parcial. 06. (EEAR) Considere P(x) = 2x³ + bx² + cx, tal que P(1) = -2 e P(2) = 6.
Assim, os valores de b e c são, respectivamente,
Com o primeiro resto parcial e o divisor D(x) repetem-se as
operações, obtendo-se o segundo termo do quociente e assim a) 1 e 2
sucessivamente até se encontrar um resto de grau menor que o divisor. b) 1 e –2
Exemplo: c) –1 e 3
Calcular (x3 + 2x – 1) ÷ (x2 + x + 2) d) –1 e -3
x 3 + 0x 2 + 2x − 1 x2 + x + 2
07. (UECE) O resto da divisão de (x² + x + 1)² por x² – x + 1 é:
x−1 a) 4x c) 4(x – 2)
−x − x − 2x
3 2

b) 4(x – 1) d) 4(x – 3)
−x 2 + 0x − 1

x2 + x + 2 08. Se x² – x – 1 é um dos fatores da fatoração de mx³ + nx² + 1, com


m e n inteiros, então, n+m é igual a:
x+1
a) –2 d) 1
Q(x) = x − 1 e R(x) = x + 1 b) –1 e) 2
Observação c) 0

O grau do quociente é a diferença dos graus do dividendo e do divisor. 09. Se o polinômio P(x) = x4 – 2x² + mx + p é divisível por D(x) = x² + 1,
gr(Q) = gr(P) −gr(D) o valor de m – p é:
a) –3 d) 2
No exemplo acima, o quociente tem grau 1 = 3 − 2. b) –1 e) 3
c) 0

EXERCÍCIOS DE
10. - Divisor: x² + x;

FIXAÇÃO - Resto: 1 – 7x; e,


- Quociente: 8x² – 8x + 12.
Logo, o dividendo dessa operação é:
01. (UECE) Considerando o polinômio P(x) = 4x³ + 8x² + x + 1, é a) 8x4 + 4x² + 5x + 1
 1 b) 6x4 + 4x² + 4x + 3
correto afirmar que o valor da soma P( −1) + P  −  é um número
localizado entre:  3 c) 8x4 + 4x² + 4x + 1
a) 5,0 e 5,5 c) 4,5 e 5,0 d) 6x4 + 8x² + 5x + 1
b) 4,0 e 4,5 d) 5,5 e 6,0

EXERCÍCIOS DE
02. A soma dos coeficientes do polinômio P(x) = (1 – x + x² – x³ + x4)1.000 é:
a) 1
b) 5
c) 100
d) 500
e) 1.000
TREINAMENTO
2 01. Qual é o polinômio que ao ser multiplicado por g(x) = 3x³ + 2x² + 5x – 4
03. Sendo x um número real maior que , a área de um retângulo é
3 tem como resultado o polinômio h(x) = 3x6 + 11x5 + 8x4 + 9x³ - 17x² + 4x?
dada pelo polinômio 3x² + 19x – 14. Se a base desse retângulo é dada
pelo polinômio x + 7, o quadrado da diagonal do retângulo é expresso a) x³ + x² + x
pelo polinômio: b) x³ + x² - x
a) 10x² + 26x + 29 d) 4x² + 2x + 53 c) x³ + 3x² + x
b) 10x² + 53 e) 10x² + 2x + 53 d) x³ + 3x² + 2x
c) 10x² + 65 e) x³ + 3x² – x

194

PM_BOOK16 - MAT.indb 194 25/11/2022 19:13:00


POLINÔMIOS: CONCEITOS E ALGORITMO DA DIVISÃO

02. (EPCAR) Sejam Q(x) e R(x) o quociente e o resto, respectivamente, 09. Dado o polinômio p(x) = ax³ + bx² + cx + d com a, b, c e d
da divisão do polinômio x³ - 6x² + 9x – 3 pelo polinômio x² – 5x + 6, números reais. Qual deve ser a relação entre os números a, b, c e d
em que x ∈ . para que o polinômio p(x) seja divisível pelo polinômio x² + 1?
O gráfico que melhor representa a função real definida por a) a = –d; c = d
P(x) = Q(x) + R(x) é: b) a = c; b = d
c) a = –c; b = –d
d) a = d; c = –b
e) a = b = c = d
a) c)
10. (EN) Sejam F(x) = x³ + ax + b e G(x) = 2x² + 2x – 6 dois polinômios
na variável real x, com a e b números reais. Qual valor de (a + b) para
F(x)
que a divisão seja exata?
G(x)
a) –2 c) 0 e) 2
b) –1 d) 1

b) d) 11. (CEFET MG) Perdeu-se parte da informação que constava em uma


solução de um problema, pois o papel foi rasgado e faz-se necessário
encontrar três dos números perdidos que chamaremos de A, B e C na
equação abaixo.
Ax − 2 B Cx 2 − 9x − C
+ =
03. (MACKENZIE) Os valores de R, P e A para que a igualdade x 2 + x + 3 2x − 1 2x 3 + x 2 + 5x − 3
2x 2 + 5x − 1 R P A O valor de A + B + C é:
=+ + seja uma identidade são,
x3 − x x x +1 x −1 a) –3 b) –2 c) 4 d) 5 e) 7
respectivamente,
a) 3, 1 e –2 d) 1, 3 e –2 12. Seja r(x) o resto da divisão do polinômio p(x) = 4x² + 3x + 5 por
b) 1, –2 e 3 e) –2, 1 e 3 q(x) = 2x² – x – 1. Se f(x) = 2x + k e f(g(x)) = r(x), então o valor da
c) 3, –2 e 1 constante k para que o conjunto solução da inequação g(x) ≥ 10 seja
{x ∈  | x ≥ 3} é:
04. (UECE) O termo independente de x no desenvolvimento da a) –12 d) 2
expressão algébrica (x² – 1)³ · (x² + x + 2)² é: b) –2 32
e) –
a) 4 c) 8 c) 12 5
b) –4 d) –8
13. Sejam q(x) e r(x), respectivamente, o quociente e o resto da divisão
x 2 + 2x + 2
05. É possível demonstrar que o polinômio P(x) = é uma de f(x) = 6x4 – x3 – 9x2 – 3x + 7 por g(x) = 2x2 + x + 1. O produto entre
2 todas as raízes de q(x) e r(x) é igual a:
boa aproximação da função f(x) = ex para valores de x próximos de
zero. Usando essa informação, o valor aproximado de 10 e é: 7
a) −
a) 1,105 c) 0,781 e) 0,553 3
b) 3
b) 1,061 d) 0,610
3
c)
06. (EEAR) Dado o polinômio: ax³ + (2a + b)x² + cx + d – 4 = 0, os 5
valores de a e b para que ele seja um polinômio de 2º grau são: d) 5
a) a = 0 e b = 0 5
e)
b) a = 1 e b ≠ 0 3
c) a=0eb≠0 14. O valor de n para que a divisão do polinômio p(x) = 2x3 + 5x2 + x + 17
d) a = –1 e b = 0 por d(x) = 2x2 + nx + 4 tenha resto igual a 5 é um número:
a) menor que – 6.
07. Na divisão do polinômio 6x4 – 2x³ – 8x² + 10x – 2 pelo divisor
b) negativo e maior que – 4.
x² + 3x – 2, o resto multiplicado por 2 é:
c) positivo e menor que 5.
a) –222x² + 252
d) par e maior que 11.
b) 444x² + 252
c) –444x + 252 15. Quais são os polinômios que representam o quociente q(x) e o
d) 222x + 252 resto r(x) da divisão do polinômio p(x) = x³ + 5x² + 6 pelo polinômio
e) –444x² – 252 d(x) = x² – 3?
a) q(x) = – (x + 5) e r(x) = 3x + 21.
08. (UECE) Se a expressão algébrica x² + 9 se escreve identicamente b) q(x) = x + 5 e r(x) = – (3x + 21).
como a(x + 1)² + b(x + 1) + c onde a, b e c são números reais, então
c) q(x) = x – 5 e r(x) = – 3x + 21.
o valor de a – b + c é:
d) q(x) = – (x + 5) e r(x) = 3x – 21.
a) 9 c) 12
e) q(x) = x + 5 e r(x) = 3x + 21.
b) 10 d) 13

195

PM_BOOK16 - MAT.indb 195 25/11/2022 19:13:00


POLINÔMIOS: CONCEITOS E ALGORITMO DA DIVISÃO

16. Calcular a, b e c de modo que se tenha, ∀x ∈ R, 03. (AFA 1999) O coeficiente de x³ no polinômio P(x) do terceiro grau
ax4 +(b +1)x2 + (2c −1) = x2 +1. que se anula para x = –1 e tal que dividido separadamente por x – 1,
x + 2 e x + 3 deixa sempre resto 20 é
1 A B para todo a) 5 c) 1
17. Obtenha A e B de forma que = +
x ≠ 0 e x ≠ −1. x(x + 1) x x +1 b) 10 d) −5

18. Dividir P(x) = x4 +2x3 +3x2 +4x +5 e D(x) = x3 +1. 04. (AFA 2001) Seja P(x) um polinômio de grau 4 com coeficientes
reais. Na divisão de P(x) por x – 2, obtém-se um quociente Q(x) e resto
igual a 26. Na divisão de P(x) por x² + x – 1, obtém-se um quociente
x 4 + x2 + 1
19. (UFF) Considere o polinômio p(x) = , x ≠ 1 e x ≠ −1. H(x) e resto 8x – 5. Se Q(0) = 13 e Q(1) = 26, então H(2) + H(3) é igual a
x2 − 1
Determine o polinômio q(x) e as constantes A, B e C tais que a) 0 c) –47
A A B C b) 16 d) –28
= q(x) + 2
p(x) e 2= + , x ≠ 1 e x ≠ −1.
x −1 x −1 x −1 x +1
20. (ESPCEX) Na divisão do polinômio 7x6 + ... + 8x – 12 por x + 2 x2 Ax  B Cx  D
05. (EN 2010) Ao escrevermos  
encontrou-se o quociente 7x5 + ... + 4. Qual o resto dessa divisão? x  1 a1x 2  b1x  c1 a2x 2  b2x  c2
4

onde ai, bi, ci (1 ≤ i ≤ 2) e A, B, C e D são constantes reais, podemos


21. (AFA) Um polinômio P(x) do terceiro grau que, para todo número afirmar que A2 + C2 vale:
real, satisfaz a expressão P(x) = P(x – 1) + x2 é: 3 1
a) x3/3 + x2/2 – x/6 c) x3/3 + x2/2 + x/6 a) d)
8 8
b) x3/3 – x2/2 + x/6 d) x3/3 – x2/2 – x/6
1 e) 0
b)
22. (ITA) Determine os valores de a e b, tais que os polinômios 2
P(x) = x3 – 2ax2 + (3a + b)x – 3b e Q(x) = x3 – (a + 2b)x + 2a sejam 1
divisíveis por x + 1? c)
4
23. Determine as constantes a, b e c na identidade: 06. (MACKENZIE) Os valores de R, P e A para que a igualdade
a·(x + 5y – 3z) + b·(2x – 2y + 5z) + c·(7x + 11y + 3z) + x – 2y = 0 2x²  5x  1 R P A
   seja uma identidade são, respectivamente,
x³  x x x 1 x 1
24. A identidade abaixo é válida para todo x real, diferente de – 1. a) 3, 1 e –2 d) 1, 3 e –2
Determine o valor de a + b + c. b) 1, –2 e 3 e) –2, 1 e 3
x3 + 4 a b.x + c c) 3, –2 e 1
≡ 1+ +
x3 + 1 x + 1 x2 − x + 1
07. (ESPCEX) Dadas as funções f(x) = x³ – 9x² + 27x – 27 e g(x) = x² – 6x + 9.
f(x)
25. (ITA) A divisão de um polinômio P(x) por x – x resulta no quociente
2
O gráfico que melhor representa h(x) = é
6x2 + 5x + 3 e resto –7x. O resto da divisão de P(x) por 2x + 1 é igual a: g(x)
a) 1 c) 3 e) 5 a) d)
y y
b) 2 d) 4

EXERCÍCIOS DE 3 x –3 x

COMBATE
b) e)
y y
01. Seja p(x) = ax + bx³ + cx² + dx + e um polinômio com coeficientes
4

reais. Sabendo que:


I. P(x) é divisível por x² – 4;
II. A soma das raízes de p(x) é igual a 1;
3 x 3 x
III. O produto das raízes de p(x) é igual a 3;
IV. P(–1) = –15/4;
Então, p(1) é igual a
a) –17/2 d) 9/4 c)
b) –19/4 e) 9/2 y
c) –3/2

02. (ESPCEX 2015) O polinômio f(x) = x5 – x³ + x² + 1, quando dividido


por q(x) = x³ – 3x + 2 deixa resto r(x).
3 x
Sabendo disso, o valor numérico de r(–1) é:
a) –10 c) 0 e) 10
b) –4 d) 4

196

PM_BOOK16 - MAT.indb 196 25/11/2022 19:13:04


POLINÔMIOS: CONCEITOS E ALGORITMO DA DIVISÃO

08. (ESPCEX) Os polinômios A(x) e B(x) são tais que A(x) = B(x) + 3x³ +
2x² + x + 1. Sabendo-se que –1 é raiz de A(x) e 3 é raiz de B(x), então ANOTAÇÕES
A(3)–B(–1) é igual a:
a) 98
b) 100
c) 102
d) 103
e) 105

09. (ESPCEX) Um polinômio q(x), do 2º grau, é definido por


q(x) = ax2 + bx + c, com a, b e c reais, a 0. Dentre os polinômios a
seguir, aquele que verifica a igualdade q(x) = q(1–x), para todo x real, é
a) q(x) = a(x² + x) + c
b) q(x) = a(x² – x) + c
c) q(x) = a²(x² – x) + c
d) q(x) = a²(x² + x) + c
e) q(x) = a² x + c

10. (ITA) No desenvolvimento de (ax2 – 2bx + c + 1)5 obtém-se um


polinômio p(x) cujos coeficientes somam 32. Se 0 e –1 são raízes de
p(x), então a soma a + b + c é igual a
a) – 1/2
b) – 1/4
c) 1/2
d) 1
e) 3/2

GABARITO
EXERCÍCIOS DE FIXAÇÃO
01. A 04. B 07. B 10. A
02. A 05. D 08. B
03. E 06. D 09. E
EXERCÍCIOS DE TREINAMENTO
01. E 14. B
02. A 15. E
03. B 16. a = 0, b = 0 e c = 2
04. B 17. A = 1 e B = –1
05. A 18. Q(x) = x + 2 e R(x) = 3x² + 3x + 3
06. C 19. A = 3, B = 3/2 e C = –3/2
07. C 20. r = -20
08. D 21. C
09. B 22. a = 3 e b = –2
10. B 37 13
23. a = − ;b = −5;c =
11. D 6 6
24. a + b + c = 2
12. D
25. E
13. D
EXERCÍCIOS DE COMBATE
01. D 04. B 07. A 10. A
02. A 05. C 08. C
03. A 06. B 09. B

197

PM_BOOK16 - MAT.indb 197 25/11/2022 19:13:05


POLINÔMIOS: CONCEITOS E ALGORITMO DA DIVISÃO

ANOTAÇÕES

198

PM_BOOK16 - MAT.indb 198 25/11/2022 19:13:05


POLINÔMIOS: TEOREMAS E
DIPOSITIVOS LIGADOS A DIVISÃO

TEOREMA DE D’ALEMBERT 2º. o coeficiente de cada termo é igual à soma do coeficiente


de mesma ordem do dividendo com o coeficiente do termo
O resto da divisão de um polinômio P(x) por x − a é igual a P(a). anterior multiplicado por a;
Seja, com efeito, P(x) = amxm + am-1xm-1 + ... + a0, um polinômio de 3º. o resto da divisão é igual à soma do coeficiente do último
x, ordenado segundo as potências decrescentes de x. Designemos o termo do dividendo com o coeficiente do último termo do
quociente dessa divisão por Q(x) e o resto por R. quociente multiplicado por a.
O resto tendo grau inferior ao divisor, que é do primeiro grau, será
de grau zero, isto é, independente de x. Exercício Resolvido
Podemos, pois, estabelecer a seguinte identidade:
03. Calcular o quociente e o resto da divisão
P(x) = (x – a) · Q(x) + R (x³ + 5x² + x – 1) : (x + 5)
Substituindo nesta identidade x por a, teremos:
P(a) ≡ (a – a) · Q(a) + R Resolução:
P(a) ≡ 0 · Q(a) + R b2 = 1
P(a) = R b1 = 1(–5) + 5 = 0
Este resultado nos mostra que R é uma constante, isto é, equivale b0 = 0(–5) + 1 = 1
ao valor numérico P(a) de Polinômio P(x), para x = a.
R = 1(–5) – 1
Observação Q(x) = x² + 1 e R = –6
Quando o polinômio divisor é da forma x + a, devemos substituir
no polinômio P(x), x por – a, visto que: x + a = x – (– a). Exercício Resolvido

04. Calcular o quociente e o resto da divisão


CONSEQUÊNCIA 1: para que um polinômio em x seja divisível por (2x4 – 3x3 + 2x2 – x + 6) : (x – 3)
x – a, é condição necessária e suficiente que ele se anule para x = a.
CONSEQUÊNCIA 2: para que um polinômio em x seja divisível por Resolução:
x + a, é condição necessária e suficiente que ele se anule para x = – a.
b3 = 2
Exercício Resolvido b0 = 2 · 3 – 3 = 3
b1 = 3 · 3 + 2 = 11
01. Calcular o resto da divisão (5x4 – 8x3 + 3x2 – 10x + 7) : (x – 2)
b0 = 11 · 3 – 1 = 32
R = 32 · 3 + 6 = 102
Resolução:
Q(x) = 2x³ + 3x² + 11x + 32
R = 5 · 2 - 8 · 2 + 3 · 2 - 10 · 2 + 7
4 3 2

R = 80 – 64 + 12 – 20 + 7
Exercício Resolvido
R = 15
05. Usando o dispositivo prático, dividir 2x3 − 5x2 + 3x − 4 por
Exercício Resolvido x−2

02. Calcular o resto da divisão (x³ + 12x² + 15x + 10) : (x + 5) Resolução:


Inicialmente alocar no dispositivo os coeficientes do dividendo e o
Resolução: segundo termo do binômio com o sinal trocado e então proceder
R = (–5)³ + 12 · (–5)² + 15 · (–5) + 10 como acima:
R = –125 + 300 – 75 + 10 = 110 2 −5 3 −4
2 2 −1 1 −2
REGRA DE RUFFINI ↓ ↓ ↓
O quociente da divisão de um polinômio completo e ordenado em
2 ⋅ 2+(−5) 2 ⋅ (−1)+3 2 ⋅ 1+(−4)
relação a x do grau m por um binômio da forma x – a, é um polinômio
e, ordenado em relação a x, é do grau m – 1, no qual:
Q(x) = 2x2 − x + 1 e R = −2
1º. o coeficiente do primeiro termo é o mesmo do primeiro termo
do polinômio dividendo;
Vamos detalhar a seguir o dispositivo.

199

PM_BOOK16 - MAT.indb 199 25/11/2022 19:13:05


POLINÔMIOS: TEOREMAS E DISPOSITIVOS LIGADOS A DIVISÃO

DISPOSITIVO PRÁTICO DE BRIOTT-RUFFINI Exemplo I:


Para dividir um polinômio P(x) = anxn + an-1xn-1 + ... + a2x2 + a1x + a0 P(x) = x³ – x² + 2x – 1 e G(x) = x4 + x² + x são congruentes para o
por x – a, devemos seguir o seguinte algoritmo: mesmo divisor D(x) = x² + 1 pois ambos geram o mesmo resto R(x) = x
quando divididos por D(x).
1º. na primeira linha do diagrama, dispomos a raiz a do divisor na
coluna à esquerda e a seguir os coeficientes de P(x), inclusive P(x) = x³ – x² + 2x – 1 = Q1(x) · D(x) + R(x) = (x – 1)(x² + 1) + x
os nulos; G(x) = x4 + x² + x = Q2(x) · D(x) + R(x) = (x²)(x² + 1) + x
a an an-1 an-2 ... a2 a1 a0 Então escrevemos que P(x) ≡ G(x)
... (mod(D(x))) ⇒ x³ – x² + 2x – 1 ≡ x4 + x2 + x (mod(x² + 1))
Vamos ver um exemplo de como utilizar a congruência em divisões
de polinômios de graus elevados.
2º. na segunda linha do diagrama, dispomos o coeficiente
do primeiro termo do dividendo que será o coeficiente do Exemplo II:
primeiro termo do quociente; Sendo R(x) o resto da divisão de
a an an-1 an-2 ... a2 a1 a0 P(x) = 100x100 + 99x99 + 98x98 + 5x5 + 4x4 + 3x3 + 2x2 + x + 1 por
an ... D(x) = x² – x + 1

qn1 O valor de R(–1) será:
Muitos tentam resolver este tipo de questão pelo método das
3º. à direita do termo anterior colocamos a·qn-1 + an-1 = qn-2, chaves encontrando um padrão de decrescimento nos expoentes nas
divisões sucessivas, mas esse padrão nem sempre aparece.
coeficiente do segundo termo do quociente;
Um outro fato que é importante de ser observado é que o
a an an-1 an-2 ... a2 a1 a0
divisor geralmente não pode ser decomposto em outros fatores, mas
an a  qn1  an1 a  qn2  an2 ...
  
geralmente é fator de um outro polinômio conhecido.
qn1 qn2 qn3
D(x) = x² – x + 1 não pode ser decomposto em outros fatores
de polinômios mas lembrando que x³ + 1 = (x + 1)(x² – x + 1)
4º. repete-se a operação descrita no item anterior até atingirmos sabemos que x³ + 1 é divisível por x² – x + 1, assim x³ + 1 ≡ 0
q0 P(x) = x³ + kx² + px – 9; mod(x² – x + 1) ⇒ x³ ≡ –1 mod(x² – x + 1).
a an an-1 an-2 ... a2 a1 a0 Então faremos aparecer x3 no polinômio
an a  qn1  an1 a  qn2  an2 ... a  q2  a2 a  q1  a1
     P(x) = 100x100 + 99x99 + 98x98 + 5x5 + 4x4 + 3x3 + 2x2 + x + 1
qn1 qn2 qn3 q1 q0

P(x) = 100(x³)33 · x +99(x³)33 +98(x³)32·x² +4(x³)· x +3x³ +2x² +x +1


5º. repetindo o procedimento mais uma vez obtemos o resto como x³ ≡ –1 mod(x² – x + 1) então
r = a · q0 + a0 da divisão.
100(–1)33 · x +99(–1)33 +98(–1)32 · x² +5(–1)⋅ x² +4(–1)· x +3(–1) +2x² +x +1
a an an-1 an-2 ... a2 a1 a0
⇒ –100x –99 +98x² –5x² –4x –3 +2x² + x + 1 = 95x² – 103x –101
an a  qn1  an1 a  qn2  an2 ... a  q2  a2 a  q1  a1 a  q0  a0
 
qn2
    ⇒ 100x100 +99x99 +98x98 +5x5 +4x4 +3x3 +2x2 +x +1 ≡ 95x2 –103x –101
qn1 qn3 q1 q0 r
mod(D(x))
Como o grau de R(x) ainda está igual ao grau de D(x) devemos
dividir R(x) por D(x)
TEOREMA FUNDAMENTAL DA
ÁLGEBRA 95x 2 – 103x – 101 x2 + 1
Todo polinômio de grau maior ou igual a 1 admite pelo menos
uma raiz real ou complexa. –95x 2 –95 95
Logo
TEOREMA DA DECOMPOSIÇÃO R(x) = –95x² – 95 = –95(x² + 1) ⇒
Se o número complexo α é raiz de um polinômio P, então P(x) é R(–1) = –95((–1)²+1) = –95 · 2 = –190
divisível por (x – α).’
Todo polinômio P(x) de grau n ≥ 1 P(x) = anxn + an-1xn-1 + ... + a2x2
EXERCÍCIOS DE
+ a1x + a0 (an ≠ 0) pode ser decomposto em n fatores do primeiro grau
de maneira única, a menos da ordem, como segue:
P(x) = an(x – r1)(x – r2) · ... · (x – rn)
FIXAÇÃO
onde r1, r2, ..., rn são as raízes (complexas) do polinômio.
TEOREMA: 01. O resto da divisão do polinômio p(x) = xn + x + 2 pelo polinômio
q(x) = x – 1 é:
Um polinômio de grau n possui exatamente n raízes complexas.
Desta forma, a quantidade de raízes reais é no máximo igual a n. a) 2 c) 4 e) –2
b) 0 d) –1
CONGRUÊNCIA POLINOMIAL
02. O resto da divisão de um polinômio do segundo grau P pelo binômio
A congruência entre polinômios se dá da mesma forma dos (x + 1) é igual a 3. Dado que P(0) = 6 e P(1) = 5, o valor de P(3) é:
números inteiros. Todos os polinômios que ao serem divididos por um
polinômio D(x) gerarem um mesmo resto R(x) serão ditos congruentes a) –7 c) 7
e formarão um conjunto de polinômios. b) –9 d) 9

200

PM_BOOK16 - MAT.indb 200 25/11/2022 19:13:09


POLINÔMIOS: TEOREMAS E DISPOSITIVOS LIGADOS A DIVISÃO

03. O quociente e o resto da divisão do polinômio x² + x – 1 pelo Dessa forma, r + s é:


binômio x + 3 são, respectivamente: a) 0 b) 10 c) 127 d) 137 e) 161
a) x – 2 e 5 c) x–3e2 e) x – 1 e –2
b) x + 2 e 6 d) x + 1 e 0 04. O resto da divisão de um polinômio do terceiro grau p(x) por (x – 3)
é igual a 24. Sabendo que as raízes do polinômio p(x) são –3, 1 e 2,
04. O trinômio x² + a + b é divisível por x + 2 e por x – 1. O valor de a – b é: o valor de p(0) é:
a) 0 c) 2 e) 4 a) 12 b) 15 c) 18 d) 21 e) 24
b) 1 d) 3
05. O resto da divisão do polinômio p(x) = x10 – 1 pelo polinômio
q(x) = x – 20,2 é:
05. (FUVEST) O polinômio p(x) = x3 + ax2 + bx, em que a e b são
números reais, tem restos 2 e 4 quando dividido por x – 2 e x – 1, a) 0 c) 2 e) 4
respectivamente. Assim, o valor de a é: b) 1 d) 3
a) – 6 c) –8 e) – 10
b) – 7 d) – 9 06. Considere o polinômio P(x) = 4x³ – x² – (5 + m)x + 3.
Sabendo que o resto da divisão de P pelo monômio x + 2 é 7,
06. O polinômio p(x) é divisível por x + 3, por x – 1 e por x + 5. determine o valor de m.
Podemos dizer que o seu grau g é: a) 0 c) 2 e) 21
a) g > 3 c) g≥3 e) g ≤ 3 b) 15 d) 7
b) g < 3 d) g = 3
07. (UNICAMP) Considere o polinômio p(x) = xn + xm + 1, em que
07. Se os polinômios p(x) = 2x + 9x + 3bx – (b – 9) e
3 2 n > m ≥ 1. Se o resto da divisão de p(x) por x + 1 é igual a 3, então
q(x) = x3 – bx2 + 7x + 3b, quando divididos por x + 1 fornecem restos a) n é par e m é par. c) n é par e m é ímpar.
iguais, então, o valor de b é: b) n é ímpar e m é ímpar. d) n é ímpar e m é par.
a) – 4 b) 0 c) 1 d) 4
08. O gráfico a seguir, que passa pelos pontos A, B, C e D, representa
08. O resto da divisão de P(x) = ax3 – 2x + 1 por Q(x) = x – 3 é 4. Nessas o polinômio P(x).
condições, o valor de a é:
a) 1/3 c) 2/3
b) 1/2 d) 3/2

09. A divisão do polinômio p(x) = x5 – 2x4 – x + m por q(x) = x – 1 é


exata. O valor de m é:
a) –2 c) 0 e) 2
b) –1 d) 1

10. Se o polinômio P(x) = x3 – kx2 + 6x – 1 for divisível por (x–1), ele


também será divisível por:
a) x2 – 5x + 1 c) x2 + 5x + 1 e) x2 – 5x + 5
b) x2 – 5x + 3 d) x2 + 5x + 3

EXERCÍCIOS DE

TREINAMENTO
I. O polinômio P(x) é um polinômio do segundo grau.
01. Seja p(x) um polinômio de grau três tal que p(0) = 6, p(1) = 1, 3
II. O polinômio D(x) = − x − 3 é divisor de P(x).
p(2) = 4 e p(3) = 9. É correto afirmar que p(4) é igual a: 4
III. A reta que passa pelos pontos A e C intercepta o eixo das
a) 0 c) 10 e) 8  11
b) 16 d) 14 ordenadas no ponto  0, −  .
 2
 1
= P− 
IV. P(2)
02. O polinômio P(x) = 6x² – 5x + k², em que k é uma constante  2
pertencente ao conjuntos dos números complexos, tem 3x – 4 como Todas as afirmações corretas estão em:
um de seus fatores. Assim, necessariamente, k será um número a) I – II – III c) III – IV
a) imaginário puro. d) inteiro. b) II – III – IV d) II – III
b) racional não inteiro. e) positivo.
c) irracional. 09. Considerando-se que o polinômio P(x) = x³ + ax² + bx + c tem 1
como raiz dupla e 3 como raiz simples, é correto afirmar que o resto
1 3 1 da divisão de P(x) por (x + 1) é:
03. Os restos da divisão do polinômio p(x) = 2x 4 − x + 2x 2 − x +1
2 2 a) –20 c) –16 e) –2
pelos polinômios q(x)= x − 2 e h(x)= x − 8 são r e s, respectivamente.
b) –18 d) –14

201

PM_BOOK16 - MAT.indb 201 25/11/2022 19:13:10


POLINÔMIOS: TEOREMAS E DISPOSITIVOS LIGADOS A DIVISÃO

10. (UECE) O resto da divisão de (264 + 1) por (232 + 1) é igual a: 21. (CMRJ) Dividindo o trinômio x² – x + 2 por x + 3a, obtém-se
a) 1 b) 0 c) 4 d) 2 quociente x – b e resto 2a + 3b, com a e b inteiros. A soma desses
valores inteiros de a e b é:
11. (FGV) Um dos fatores do polinômio P(x) = x³ + 2x² – 5x – 6 é a) 5 c) 1 e) –3
(x + 3). Outro fator desse polinômio é: b) 3 d) –2
a) (x + 8) c) (x + 4) e) (x + 1)
b) (x – 5) d) (x – 1) 22. Quando o polinômio x4 + ax³ – 7x² + bx – 49 é dividido por (x – 3) o
resto é 53, e quando é dividido por (x + 2) o resto é −87. Calcule a · b.
12. (PUCRS) O polinômio p(x) = ax³ + bx² + cx, em  é divisível por a) 3 c) 5 e) 9
(x – 1). Podemos afirmar que p(p(1)) é: b) 4 d) 6
a) –1 c) 1 e) –a + b – c
b) 0 d) a + b + c 23. (CN) Sabendo-se que a equação x²(x² + 13) – 6x(x² + 2) + 4 = 0
pode ser escrita como um produto de binômios do primeiro grau, a
soma de duas das suas raízes reais distintas é igual a
13. Um polinômio p(x) dividido por x + 1 deixa resto 16; por x – 1
deixa resto 12, e por x deixa resto –1 Sabendo que o resto da divisão a) –3 c) –1 e) 3
de p(x) por (x + 1)(x – 1)x é da forma ax² + bx + c, então o valor b) –2 d) 2
numérico da soma das raízes do polinômio ax² + bx + c é:
3 c) 2 e) –2 24. (ITA) Sendo c um número real a ser determinado, decomponha o
a)
5 15 polinômio 9x² – 63x + c, numa diferença de dois cubos (x + a)³ – (x + b)³.
Neste caso, |a +|b|– c| é igual a:
b) 2 d) 4
a) 104 c) 124 e) 144
14. (PUCPR) Se (x – 2) é um fator do polinômio x³ + kx² + 12x – 8, b) 114 d) 134
então, o valor de k é igual a:
a) –3 c) 3 e) –6 25. (ITA) Considere Q(x) e R(x), respectivamente, o quociente e o resto
da divisão de um polinômio A(x) pelo trinômio B(x) = –x² + 5x – 6.
b) 2 d) 6 Admita que o grau de A(x) é quatro e que os restos da divisão de A(x)
por x + 1 e x – 2 são, respectivamente, 3 e −1. Supondo também que
15. O polinômio P(x) = a · x³ + 2 · x + b é divisível por x – 2 e, quando Q(x) é divisível por x + 1, podemos afirmar que R(x) é igual a:
divisível por x + 3, deixa resto –45. Nessas condições, os valores de a
a) –4x + 5 4 5
e b, respectivamente, são: d) x−
b) 4x – 5 3 3
a) 1 e 4 c) –1 e 12 e) 1 e –12
4 5 e) 2 5
b) 1 e 12 d) 2 e 16 c) − x+ x+
3 3 3 6
16. Dividindo-se o polinômio p(x) = 2x4 – 5x³ + kx – 1 por (x – 3) e (x + 2), 26. O produto (1 + x + x2 + … + x100)(1 + x + x2 + … + x25) é um
os restos são iguais. Neste caso, o valor de K é igual a: polinômio na variável x. O coeficiente de x50 é?
a) 10 c) 8 e) 6
b) 9 d) 7 27. Determinar a e b para que o polinômio x3 − ax2 + bx − 10 seja
divisível por (x + 2)(x − 1).
17. Seja P(x) = x3 – 2x2 + 3x – 5 um polinômio. O resto da divisão de
1 28. (ITA) Se 1 é uma raiz de multiplicidade 2 da equação
P(x) pelo binômio B(x)= x − é:
2 x4 + x2 + ax + b = 0, com a, b ∈  , então a² – b³ é igual a:
a) um número natural.
a) –64 c) –28 e) 27
b) um número inteiro negativo.
b) –36 d) 18
c) um número racional positivo.
d) um número racional negativo.
(x − 6x 2 + 12x − 8 ) + 2x 2 − 8x + 1 + K
3 16

e) um número irracional. 29. (CN 1984) Se a divisão é


exata, o valor de K é: x 2 − 4x + 4
18. Dividindo-se o polinômio p(x) = 3x4 – 2x3 + mx + 1 por (x – 1) ou a) 3 c) 6 e) 8
por (x + 1), os restos são iguais. Nesse caso, o valor de m é igual a: b) 5 d) 7
a) –2 c) 1 e) 3
b) –1 d) 2 30. (ITA 2002) Com base no gráfico da função polinomial y = f(x)
esboçado abaixo, calcule o resto da divisão de f(x) por  x −  ( x − 1).
1
 2
19. Dividindo o polinômio p(x) pelo polinômio (x – 2)(x – 4)(x – 5)
obtém-se resto x + 3. Se os restos das divisões de p(x) por x – 2, x – 4 a) x + 1
e x - 5 são, respectivamente, os números A, B e C, então ABC vale 1
b) −x +
a) 100. c) 200. e) 360. 2
b) 180. d) 280. c) x 1

2 4
20. Na divisão do polinômio P(x) = 4x3 + mx2 – 3x + 4 por x – 2 o resto x 1
é 18. Nessas condições, o valor de m é: d) − +
4 2
a) –6 c) –3 e) –5
x 1
b) 3 d) 6 e) − +
4 4

202

PM_BOOK16 - MAT.indb 202 25/11/2022 19:13:10


POLINÔMIOS: TEOREMAS E DISPOSITIVOS LIGADOS A DIVISÃO

EXERCÍCIOS DE
GABARITO
COMBATE EXERCÍCIOS DE FIXAÇÃO
01. C 04. D 07. D 10. A
02. B 05. A 08. A
01. O resto da divisão de 16101 + 8101 + 4101 + 2101 + 1 por 2100 + 1 é:
03. A 06. C 09. E
a) 0 c) 4 e) 10
EXERCÍCIOS DE TREINAMENTO
b) 2 d) 11
01. C 09. C 17. D 25. C

02. (EN 1991/1992) O resto da divisão de 1 + x + x2 + … + x100 02. A 10. D 18. D 26. 26
por x2 – 1 é: 03. D 11. E 19. D 27. b = –3 e
a) 0 d) 50x + 51 04. A 12. B 20. C a = –6

b) x + 1 e) 51x + 50 05. D 13. C 21. A 28. –28

c) 50x + 50 06. B 14. E 22. A 29. 7


x 1
07. A 15. E 23. E 30. − +
03. (ITA) Se P(x) é um polinômio do 5° grau que satisfaz as condições 4 4
08. D 16. B 24. B
1 = P(1) = P(2) = P(3) = P(4) = P(5) e P(6) = 0, então temos:
EXERCÍCIOS DE COMBATE
a) P(0) = 4 c) P(0) = 9 e) P(0) = 0
01. D 04. B 07. C 10. E
b) P(0) = 3 d) P(0) = 2
02. D 05. C 08. D
04. (EFOMM 2012) Sabendo que o polinômio P(x) = x³ + kx² + px – 9 03. D 06. B 09. A
é divisível por D(x) = x² – 3, podemos afirmar que:
ANOTAÇÕES
a) p + k = -3 c) p + k = -9 k
e) p =
4
3
p d) p ∈  e k ∈
b)  1
k
05. (IME 2012) Considere o polinômio 5x³ - 3x² - 60x + 36 = 0.
Sabendo que ele admite uma solução da forma n , onde n é um
número natural, pode-se afirmar que:
a) 1 ≤ n < 5 c) 10 ≤ n < 15 e) 20 ≤ n < 30
b) 6 ≤ n < 10 d) 15 ≤ n < 20

06. (AFA) Sendo P(x) = x + 3x³ + 5x5 + 7x7 .. + 999x999, o resto da


divisão de P(x) por (x – 1) é
a) 249.500 c) 250.500
b) 250.000 d) 251.000

07. Seja P(x) um polinômio divisível por x −1. Dividindo-o por x2 + x,


obtêm-se o quociente Q(x) x2 – 3 e o resto R(x). Se R(4) = 10, então o
coeficiente do termo de grau 1 de P(x) é igual a:
a) −5 c) −1 e) 3
b) −3 d) 1

08. O resto da divisão x + x + 1 é igual a:


199

x −1
5

x −1
a) x2 (x – 1) d) –x2 (x + 1)
b) x3 (x – 1) e) x4 (x + 1)
c) x (x + 1)

09. Sabendo que g(x) = x5 + x4 + x3 + x2 + x + 1. Calcule o resto da


divisão de g(x12) por g(x) é
a) 6 c) 4 – x + x2 e) 2
b) 5 – x d) 4 – x + x2 – x³

10. (EN) Seja P(x) = x6 + bx5 + cx4 + dx3 + ex² + fx + g


um polinômio de coeficientes inteiros e que P( 2 + 3 3 ) = 0. O
polinômio R(x) é o resto da divisão de P(x) por x3 – 3x – 1. Determine a
soma dos coeficientes de R(x) e assinale a opção correta.
a) –51 c) –53 e) –55
b) –52 d) –54

203

PM_BOOK16 - MAT.indb 203 25/11/2022 19:13:13


POLINÔMIOS: TEOREMAS E DISPOSITIVOS LIGADOS A DIVISÃO

ANOTAÇÕES

204

PM_BOOK16 - MAT.indb 204 25/11/2022 19:13:13


EQUAÇÕES POLINOMIAIS

EQUAÇÃO POLINOMIAL OU ALGÉBRICA an(x–r1).(x–r2)...(x–rn) = anxn + (r1+r2+...+rn)xn-1 + (r1r2+r1r3+...+


Denominamos equação polinomial ou equação algébrica de grau rn-1rn)xn-2 + ... + (–1)nanr1r2...rn
n a toda equação da forma: Igualando as duas formas temos:
p(x) = anxn + an−1 xn−1 +an−2xn−2 + ... + a1x + a0 = 0 an 1
r1  r2  ....  rn  -
onde a0, a1, ..., an são chamados coeficientes e podem ser números an
reais ou complexos e an≠ 0 é chamado coeficiente dominante. an  2
O conjunto solução ou conjunto verdade de uma equação 1 2  rr
rr 1 3  ...  rn 1rn 
an
algébrica, no conjunto universo U, é o subconjunto de U que contém
as raízes da equação. ....
Duas equações são ditas equivalentes em U, quando apresentam a0
1 2 ..... rn   1 
n
rr
o mesmo conjunto solução nesse domínio. an
No caso da equação do 2° grau ax² + bx + c = 0, de raízes r1 e r2, a
TEOREMA DA DECOMPOSIÇÃO soma das raízes é S  r1  r2   1 
1 b b
  e o produto das raízes
Teorema Fundamental da Álgebra: todo polinômio de grau n ≥ 1 a a
c c
admite ao menos uma raiz complexa. é P  r1  r2  ( 1)2   .
a a
Corolário 1: toda equação polinomial de grau n admite
Exemplo:
exatamente n raízes complexas.
Sendo o polinômio P(x) = x³ + 6x² + 11x + 6 cujas raízes são –1,
Corolário 2: todo polinômio P(x) = anxn + an−1 xn−1 + an−2xn−2 + ... +
–2 e –3, então temos:
a1x + a0 de grau n pode ser colocado na forma fatorada:
6
P(x) = an (x −r1)⋅(x −r2)⋅...⋅(x −rn) 1   1  ( 2)  ( 3)  (  1)1   6
onde r1, r2, ..., rn são as raízes de P(x).
1
11
Corolário 3: se um polinômio de grau n possuir mais de n raízes, 2  ( 1)( 2)  ( 1)( 3)  ( 2)( 3)  ( 1)2  11
1
então ele é identicamente nulo. 6
3  ( 1)( 2)( 3)  ( 1)3  6
Exemplo: 1
Verificar que uma raiz da equação x3 − 3x2 + 4x − 2 = 0 é o número 1,
obter as outras raízes e obter a forma fatorada de P(x).
RAÍZES COMPLEXAS DE EQUAÇÕES
Podemos aplicar diretamente o algoritmo de Ruffini: COM COEFICIENTES REAIS
Se um complexo z = a + bi, a ∈ R e b ∈ R, é raiz de uma equação
1 −3 4 −2
algébrica de coeficientes reais, então o conjugado z  a  bi também
1 1 −2 2 0 é raiz da equação.
Como o resto da divisão por x − 1 é 0, então 1 é raiz de P(x). Corolários:
O quociente é q(x) = x2 − 2x + 2, cujas raízes são 1 ± i. 1. toda equação algébrica de coeficientes reais e grau ímpar
Raízes: 1, 1 + i e 1 − i. P(x) = (x −1)⋅(x −1 − i)⋅(x −1 + i) admite pelo menos uma raiz real;
2. se o complexo z é raiz de multiplicidade m de uma equação
algébrica de coeficientes reais, então o conjugado z também
MULTIPLICIDADE é raiz de multiplicidade m da equação.
Dizemos que r é raiz de multiplicidade m (m ≥ 1) da equação
P(x) = 0 se, e somente se, Exemplo:
P(x) = (x − r) ⋅ Q(x) e Q(r) ≠ 0
m Resolver a equação x4 + 4x3 − 17x2 + 26x − 14 = 0 sabendo que
1 − i é uma de suas raízes.
ou seja, r é raiz de multiplicidade m de P(x) = 0 quando o polinômio
P é divisível por (x − r)m e não é divisível por (x − r)m+1. Como trata-se de uma equação de coeficientes reais, se 1 − i é
raiz, então 1 + i também é raiz.
Quando m = 1 dizemos que r é uma raiz simples; quando m = 2,
dupla; tripla quando m = 3, etc. Aplicando o algoritmo de Briot-Ruffini:

1 4 −17 26 −14
RELAÇÕES DE GIRARD 1 1 5−i −13 7 + 7i 0
Seja a equação algébrica −i −6i
anxn + an-1xn-1 + an-2xn-2 + ... + a2x2 + a1x + a0 = 0 1 1 6 −7 0
escrevendo a equação na forma fatorada +i

205

PM_BOOK16 - MAT.indb 205 25/11/2022 19:13:18


EQUAÇÕES POLINOMIAIS

⇒ x2 + 6x − 7 = 0 ⇒ raízes: x = 1 ou x = −7 1
⇒ mdc(p, q) = (10x − 10) = x −1
⇒ S = {1, −7, 1+i, 1−i} 10
vale notar que a divisão por 10 se faz necessária para que o mdc
RAÍZES RACIONAIS DE EQUAÇÕES seja um polinômio unitário.
COM COEFICIENTES INTEIROS O mínimo múltiplo comum entre polinômios é o polinômio
unitário formado por todos os fatores que aparecem nos polinômios,
p
Se r = , p e q inteiros primos entre si, é uma raiz racional da comuns ou não, elevados ao seu maior expoente, de forma que ele é
q o polinômio de menor grau que é múltiplo de todos aqueles.
equação de coeficientes inteiros Todas as raízes dos polinômios são raízes do seu mmc.
p(x) = anxn + an−1 xn−1 +an−2xn−2 + ... + a1x + a0 = 0 Exemplos:
então p é divisor de a0 e q é divisor de an. P(x) = x(x – 1)2(x – 2)3 e Q (x) = x3(x – 1)(x – 3)2.
Exemplo: mdc (P, Q) = x(x – 1)
Verificar se a equação 2x + x + x − 1 = 0 admite raízes racionais.
3 2 mmc (P, Q) = x3(x – 1)2(x – 2)3(x – 3)2
p
r = ⇒ p ∈ {1, −1} e q ∈ {1, −1, 2, −2}
q p FÓRMULA DE NEWTON
⇒ r = ∈ {1, −1, 1 , − 1 }
q 2 2 Seja o polinômio P(x) = anxn + an−1 xn−1 + ... + a2x2 + a1x + a0, onde an ≠ 0,
e r1, r2, r3, ..., rn as suas raízes, definimos Sk  r1k  r2k  r3k    rnk .
p(x) = 2x3 +x2 +x −1
Assim, para k ∈ , temos:
p(1) = 3 p(−1) = −3 p(1/2) = 0 p(−1/2) = −3/2 an · Sk + an-1· Sk-1 + ...+ a1 · Sk-n-1+a0 · Sk-n = 0
Logo, a única raiz racional da equação é ½ Observe ainda que S0  r10  r20  r30    rn0  n e
a
S1  r11  r21  r31    rn1   n1 .
REGRA DE EXCLUSÃO DE NEWTON an
Suponhamos que uma equação polinomial P(x), com coeficientes
inteiros, admita a raiz x = a. Devemos ter:
P( x ) ≡ ( x − a) ⋅ Q( x ) EXERCÍCIOS DE
sendo Q(x) um polinô io inteiro, do grau m−1
P( x ) ≡ −(a - x ).Q( x ) ⇔
1
FIXAÇÃO
P( x )
= −Q( x ) 01. Uma equação polinomial do 3° grau que admite as raízes –1,
a−x
1
Substituindo x = 1 e x = − 1 temos − e 2 é:
2
P(1) a) x³ – 2x² – 5x – 2 = 0 d) 2x³ – x² – 2x – 2 = 0
= −Q(1)
a −1 b) 2x³ – x² – 5x + 2 = 0 e) 2x³ – x² – 5x – 2 = 0
P( −1)
= −Q( −1) c) 2x³ – x² + 5x – 2 = 0
a +1
Concluímos toda raiz inteira positiva a, a ≠ 1, diminuída de uma 02. Uma das raízes da equação 2x³ + x² – 7x – 6 = 0 é x1 = 2. Pode se
unidade deve dividir P(1) e aumentada de uma unidade deve dividir P(–1). afirmar que:
a) as outras raízes são números imaginários puros.
M.M.C. E M.D.C. DE POLINÔMIOS b) as outras raízes são – 3 e – 2.
O máximo divisor comum (M.D.C.) entre polinômios é o polinômio c) só uma das outras raízes é real.
unitário formado pelos fatores comuns aos polinômios elevados aos
d) as outras raízes estão entre – 2 e 0.
seus menores expoentes, de forma que ele é o polinômio de maior
grau que divide todos aqueles.
03. Dado P(x) = x³ –(2m + 4)x² + 9x + 13, o valor de m, para que 3i
As raízes comuns aos polinômios são também raízes de seu MDC, seja raiz de P(x), é
com a menor multiplicidade.
49 25 25 23
Se o MDC de dois polinômios é 1, diz-se que eles são primos entre si. a) − b) − c) − d)
18 6 6 18
Quando os polinômios não estão na forma fatorada, o seu MDC
pode ser obtido pelo método das divisões sucessivas. 04. Se o polinômio x³ – 9x² + 14x + 24 tem uma raiz igual a 6,
Exemplo: decompondo-o em fatores, obtém-se
Obtenha o MDC dos polinômios a) (x – 6)(x – 4)(x + 1) c) (x + 6)(x – 4)(x + 1)
p(x) = x −3x +3x −3x +2 e q(x) = x −4x +3.
4 3 2 2 b) (x – 6)(x + 4)(x – 1) d) (x + 6)(x + 4)(x – 1)

1 3 05. Para que o polinômio P(x) = 2x4 + x³ – 6x + αx + β tenha como


x2 +x +4 x− ← quocientes raiz dupla o número 1, os valores de α e β devem ser, respectivamente,
10 10
a) 1 e 2 b) 2 e 1 c) –2 e 1 d) 1 e –2
x4 −3x3 +3x2 −3x +2 x2 −4x +3 10x−10
10x −10 0 ← restos 06. A equação, cujas raízes são + 2 , + + 2, − 5 e + 5, é
x4 + ax² + b = 0. O valor de |a + b| é:
a) 2 b) 3 c) 4 d) 5

206

PM_BOOK16 - MAT.indb 206 25/11/2022 19:13:24


EQUAÇÕES POLINOMIAIS

07. Se 3 e –3 são duas das raízes da equação x4 – 5x² – 36 = 0, as 09. Considere o polinômio p(x) = x² + bx = 3 e assinale a alternativa correta.
outras raízes são: a) O polinômio tem pelo menos uma raiz real para todo b ∈ .
a) 3i e 2i b) 2i e –2i c) –i e –3i d) 3i e –3i b) O polinômio tem exatamente uma raiz real para b = 12.
c) O polinômio tem infinitas raízes reais para b = 0.
08. Uma equação polinomial de coeficientes reais admite como raízes
1
os números 3 + i, 7 e 2 – 3i. Essa equação tem, no mínimo, grau d) O polinômio não admite raiz real para b= 1 + .
3
a) 6 b) 5 c) 4 d) 3 e) O polinômio tem exatamente três raízes reais para b = π.

09. Se 3, 5 e –2, são as raízes da equação 4(x – a)(x – b)(x - 5) = 0 o 10. (FGV) A equação algébrica x³ – 7x² + kx + 216 = 0, em que k é
valor de a + b é: um número real, possui três raízes reais. Sabendo-se que o quadrado
a) 0 b) 1 c) 2 d) 3 de uma das raízes dessa equação é igual ao produto das outras duas,
então o valor de k é igual a:
10 Se a maior das raízes da equação x³ – 6x² + 11x – 6 = 0 é igual à a) –64 b) –42 c) –36 d) 18 e) 24
soma das outras duas, então seu valor é divisor de:
a) 10 b) 16 c) 18 d) 49 11. (MACKENZIE) Seja P(x) = 2x³ – 11x² + 17x – 6 um polinômio do 3º
grau e 2x – 1 um de seus fatores. A média aritmética das raízes de P(x) é:
7 9 11
a) c) e)
EXERCÍCIOS DE 2 2 6

TREINAMENTO b)
8
2
d)
10
2

12. (EEAR) Seja a equação polinomial x³ + bx² + cx + 18 = 0. Se –2 e 3


01. Sabendo-se que uma das raízes da equação algébrica são suas raízes, sendo que a raiz 3 tem multiplicidade 2, o valor de “b” é:
1 a) 8 b) 6 c) -3 d) –4
2x³ – 3x² – 72x – 35 = 0 é − , a soma das outras duas raízes é igual a:
2
a) –3 c) –2 e) 2 13. O polinômio de menor grau, com coeficientes inteiros, divisível
b) 3 d) 1 por 2x – 3, que admite x = 2i como uma das raízes e P(0) = –12 é:
(Dado: i é o número complexo cujo quadrado é igual a –1)
02. Com relação a equação 2x² + x – 1 = 0 é correto afirmar que: a) P(x) = 2x³ – 3x² – 8x – 12 c) P(x) = –2x³ – 3x² – 8x – 12
a) Não possui raízes reais. b) P(x) = 2x³ + 3x² – 8x – 12 d) P(x) = 2x³ – 3x² + 8x – 12
b) A soma das raízes é zero.
c) Possui duas raízes inteiras e distintas. 14. Sabendo-se que 1+i é uma das raízes do polinômio
p(x) = x5 – 2x4 + 2x³ – x² + 2x – 2, é correto afirmar que:
d) Possui uma raiz racional não inteira.
a) O polinômio não possui raízes reais.
e) O Produto das raízes é zero.
b) O polinômio possui exatamente duas raízes racionais.
5 c) O polinômio possui exatamente duas raízes distintas.
03. (EFOMM) Sabendo que é uma raiz do polinômio
2 d) O polinômio possui quatro raízes complexas não reais.
P(x) = 2x³ – 3x² – 9x + 10, a soma das outras raízes é igual a: e) O polinômio possui exatamente quatro raízes distintas.
a) –2 c) 10 e) –1
b) 0 d) 1 15. Se 3 e 1 são as raízes da equação ax² – 6x + p = 0, então o valor
3
de a + p é:
04. A soma dos quadrados dos números complexos que são as raízes a) –5 c) 0 e) 4
da equação x4 – 1 = 0 é igual a:
−9 18
a) 8 b) 0 c) 4 d) 2 b) d)
5 5

05. O polinômio P(x) = ax³ + bx² + cx + d é tal que as raízes da 16. Sabendo-se que o polinômio p(x) = 8x³ – 4x² – 66x – 63 tem uma
equação P(x) = 0 são os números –1, 1 e 2. Se P(0) = 24, então, o valor raiz simples x1 > 3 e uma raiz dupla x2, é correto afirmar que
do coeficiente a é igual a: 7 5 5
a) x2 < − c) x2 < − e) x2 >
a) 10 b) 8 c) 12 d) 6 2 4 2
3
06. Considere o polinômio p(x) = x³ – x² + ax – a, onde a é um número b) x2 < –3 d) x2 >
4
real. Se x = 1 é a única raiz real de p(x), então podemos afirmar que:
a) a < 0 b) a < 1 c) a>0 d) a > 1 17. (CEFET MG) Os polinômios A(x) = x² – 3x + 2 e
B(x) = 4x – 2x³ + kx² – 3x – 2tem uma única raiz em comum. Os valores
07. Se uma das raízes do polinômio P(x) = x4 – 8x² + ax + b é 2 e possíveis para k são números:
P(1) = 9, então o valor de a5 – 4b é: a) pares. c) inversos. e) simétricos.
a) –64 b) –28 c) 16 d) 24 b) primos. d) ímpares.

08. Considere a equação a·x² + b·x + c = 0, a ≠ 0. Sabemos que 18. Se uma das raízes do polinômio p(x) = x³ + x² + 4x + 4 é o número
a + b + c = 0 e que x = 3 é raiz da equação. Quanto vale o produto das complexo z = –2i, as outras raízes são:
duas raízes da equação? a) 1 e –1 c) –1 e 2 e) 2 e 2i
a) –6 b) –3 c) 3 d) 6 e) 9 b) –1 e 2i d) –1 e 3

207

PM_BOOK16 - MAT.indb 207 25/11/2022 19:13:25


EQUAÇÕES POLINOMIAIS

x
19. (FGV) Considere o polinômio P(x) tal que P   = x + x + 1. A
2
K H
 3 a) − c) −
soma de todas as raízes da equação P(3x) = 7 é igual a: L L
1 c) 0 5 H K
a) − e) b) d)
9 5 3 L L
d)
1 9 30. Sabe-se que 1 + i é uma das raízes da equação x4 – 2x³ + 4x – 4 = 0.
b) −
3 Pode-se afirmar, dessa forma, que essa equação
20. Sabe-se que 1 é uma raiz de multiplicidade 3 da equação a) possui raízes racionais e iguais.
x5 – 3 · x4 + 4 · x³ – 4 · x² + 3 · x – 1 = 0. As outras raízes dessa equação, b) possui raízes racionais e diferentes.
no Conjunto Numérico dos Complexos, são: c) possui raízes irracionais e iguais.
a) (–1–i) e (1+i) c) (–i) e (+i) e) (1–i) e (1+i) d) não possui raízes reais.
b) (1–i)² d) (–1) e (+1) e) possui raízes irracionais e diferentes.

21. Se o coeficiente do termo de maior grau de um polinômio do 4º 31. (FUVEST) Considere o polinômio P(x) = xn + an −1xn −1 +  + a1x + a0 ,
grau é 1 e suas raízes são x1 = 2i, x2 = –2i, x3 = 3 e x4 = 4, então o em que a0, ..., an-1 ∈ . Sabe-se que as suas n raízes estão sobre a
polinômio em questão é: circunferência unitária e que a0 < 0.
a) x4 – 7x³ + 16x² – 28x + 48 c) x4 = 16x³ + 4x² – x + 18 O produto das n raízes de P(x), para qualquer inteiro n ≥ 1, é:
b) x – 2ix³ + 2ix² + 3x + 4
4
d) x4 – 28x³ + 7x² + 48x – 28 a) –1 c) in+1 e) (–1)n+1
b) i n
d) (–1) n
22. (FGV) O número 1 é raiz de multiplicidade 2 da equação polinomial
x4 – 2x³ – 3x² + ax + b = 0. O produto a · b é igual a:
32. (FUVEST) O polinômio P(x) = x³ – 3x² + 7x – 5 possui uma raiz
a) –8 b) –4 c) –32 d) 16 e) –64 complexa ξ cuja parte imaginária é positiva. A parte real de ξ³ é igual a:
a) –11 c) 9 e) 12
23. (ESPCEX) Os polinômios A(x) e B(x) são tais que A(x) = B(x) + 3x³ +
2x² + x + 1. Sabendo-se que –1 é raiz de A(x) e 3 é raiz de B(x), então b) –7 d) 10
A(3) – B(–1) é igual a:
a) 98 c) 102 e) 105 33. Um polinômio de quinto grau tem 2 como uma raiz de
multiplicidade 3. A razão entre o coeficiente do termo de quarto grau
b) 100 d) 103 e o coeficiente do termo de quinto grau é igual a –7. A razão entre o
termo independente e o coeficiente do termo de quinto grau é igual a
24. A equação polinomial x³ – x² – 16x – 20 = 0 tem raízes x1, x2, x3. 96. A menor raiz desse polinômio vale:
1 1 1
O valor da expressão + + é: a) 0 b) –1 c) –2 d) –3
x1 x 2 x 3
a) 1 4 4 34. (MACKENZIE) A equação 2x³ + 3x² – 3x – 2 = 0 tem como raízes
c) e) −
3 5 5 1
b) − − , m e n. Então, mn é igual a:
4 3 2
d)
4 a) –1 ou 0 c) –2 ou –1 e) –2 ou 1
25. (FGV) O polinômio P(x) = x – 5x + 3x + 5x – 4 tem o número 1
4 3 2 1 1 1
b) − ou 2 d) ou −
como raiz dupla. 2 2 2
O valor absoluto da diferença entre as outras raízes é igual a: 35. Considere as afirmações:
a) 5 c) 3 e) 1 I. O polinômio p(x) = 2x5 – 8x4 + x + 1 possui, pelo menos, uma raiz
b) 4 d) 2 racional.
II. Se r é raiz do polinômio t(x) = x³ + 2x² + x + 15, então 2r é raiz do
26. Considere os polinômios m(x) = x² – 3x + 2, n(x) = x² – 4x + 3 polinômio q(x) = 2x³ + 4x² + 2x + 30.
e q(x) = x³ – x² – 4x + 4, que têm como fator comum o polinômio III. O polinômio s(x) = x + 1 é fator do polinômio u(x) = 7x8 + 2x4 – 4x² +
f(x) = x – 1. Se P(x) = m(x) · n(x) · q(x), a soma das raízes distintas da 6x + 1
equação polinomial P(x) = 0 é igual a
É CORRETO afirmar que:
a) 16 b) 6 c) 10 d) 4
a) Apenas a afirmação I é verdadeira.
27. Se as raízes do polinômio P(x) = x³ – 12x² + 47x – 60 são reais, b) Apenas a afirmação II é verdadeira.
distintas e formam uma progressão aritmética, então, a soma dos c) Apenas a afirmação III é verdadeira.
cubos dessas raízes é igual a: d) Apenas as afirmações II e III são verdadeiras.
a) 236 b) 206 c) 226 d) 216 e) Apenas as afirmações I e II são verdadeiras.

28. (UNICAMP) Sabendo que a e b são números reais, considere o 36. Um polinômio do terceiro grau, cujo coeficiente do termo
polinômio cúbico p(x) = x³ + ax² + x + b. Se a soma e o produto de dominante é igual a 1, admite apenas raízes reais e distintas que
duas de suas raízes são iguais a –1, então p(1) é igual a: quando multiplicadas resultam em 15 e quando somadas resultam em
a) 0 b) 1 c) 2 d) 3 1. Se o resto da divisão desse polinômio por g(x) = x + 2 é igual a 7,
então o quociente dessa divisão é igual a:
29. Se os três números primos distintos p1, p2 e p3 são as raízes a) x² – 3x – 11 d) x² – x – 11
do polinômio p(x) = x³ + Hx² + Kx + L, então, a soma dos inversos b) x² + 3x – 7 e) x² + x + 15
multiplicativos desses números é igual a:
c) x² – x – 15

208

PM_BOOK16 - MAT.indb 208 25/11/2022 19:13:26


EQUAÇÕES POLINOMIAIS

37. (ESPCEX) Sabendo que o número complexo i (sendo i a unidade 45. (ITA 2011) Se 1 é uma raiz de multiplicidade 2 da equação
imaginária) é raiz do polinômio p(x) = x5 – 2x4 – x + 2, podemos afirmar x4 + x² + ax + b = 0, com a, b ∈ , então a² – b³ é igual a:
que p(x) tem: a) –64 c) –28 e) 27
a) duas raízes iguais a i, uma raiz racional e duas raízes irracionais. b) 36 d) 18
b) i e -i como raízes complexas e três raízes irracionais.
c) uma raiz complexa i e quatro raízes reais.
d) i e -i como raízes complexas e três raízes inteiras. EXERCÍCIOS DE
e) três raízes simples e uma raiz dupla.
COMBATE
25 11 2 3 25 11 2
38. (ESPCEX) O número real 3 + + − pertence ao
conjunto: 8 4 8 4
01. (UFRJ 1999) Encontre as raízes de x3 +15x2 +66x +80 = 0, sabendo
a) [–5,–3) c) [–1,1) e) [3,5) que são reais e estão em progressão aritmética.
b) [–3,–1) d) [1,3)
02. (FGV 2002) Responda as questões abaixo:
39. (ESPCEX) As três raízes da equação x³ – 6x² + 21x – 26 = 0 são m, a) Sejam a, b e c as raízes da equação x3 −4x2 +6x −1 = 0. Calcule o
n e p. Sabendo que m e n são complexas e que p é uma raiz racional, 1 1 1
o valor de m² + n² é igual a: valor da expressão: + + .
ab ac bc
a) –18 c) 0 e) 8
b) Resolva a equação x3−2x2−5x +6 = 0, sabendo que a soma de
b) –10 d) 4
duas raízes vale 4.
40. (ESPCEX) Considere o polinômio p(x) = x6 – 2x5 + 2x4 – 4x³ + x² - 2x.
Sobre as raízes de p(x) = 0, podemos afirmar que: 03. (AFA) Considere a equação x3 + px2 + qx + r = 0, de coeficientes
reais, cujas raízes estão em progressão geométrica. Qual das relações
a) quatro raízes são reais distintas.
é verdadeira?
b) quatro raízes são reais, sendo duas iguais.
a) p2 = rq d) p3 = rq3
c) apenas uma raiz é real.
b) 2p + r = q e) q3 = rp3
d) apenas duas raízes são reais e iguais.
c) 3p2 = r2q
e) apenas duas raízes são reais distintas.
04. (EFOMM 2012) O valor de λ na equação y³ –61y² + λy – 5832 = 0
41. (EFOMM) Seja o polinômio p(x) = x6 – 26x4 – 32x³ - 147x² - 96x – 180. de modo que suas raízes estejam em progressão geométrica, é:
A respeito das raízes da equação p(x) = 0, podemos afirmar que:
a) 1017 c) 1078 e) 1121
a) todas as raízes são reais.
b) 1056 d) 1098
b) somente duas raízes são reais, sendo elas distintas.
c) somente duas raízes são reais, sendo elas iguais. 05. (ITA 2011) Se 1 é uma raiz de multiplicidade 2 da equação
d) somente quatro raízes são reais, sendo todas elas distintas. x4 + x2 + ax + b = 0, com a, b e , então a² – b³ é igual a:
e) nenhuma raiz é real. a) –64 c) –28 e) 27
b) –36 d) 18
11x + 6
42. (ESPCEX) Sendo R a maior das raízes da equação = x2 ,
então o valor de 2R – 2 é: x−4 06. (ESPCEX) Temos as funções f(x) = x + 1, g(x) = x³ + ax² + bx + c
e h(x) = g(f(x)). Considerando que as raízes de h(x) são {–1; 0; 1}
a) 2 c) 6 e) 10
determine h(– 2).
b) 4 d) 8
a) 0 c) 4 e) –6

43. (ESPCEX) A função f: → definida por f(x) = x – 5x³ + 5x² + 5 - 6


4 b) –3 d) 5
tem como algumas de suas raízes os números –1 e 1. Assinale a
alternativa que representa o conjunto de todos os números reais para 07. (ESPCEX) As medidas em centímetros das arestas de um bloco
os quais a função f(x) é positiva. retangular são as raízes da equação polinomial x³ – 14x² + 64x – 96 = 0.
Denominando-se r, s e t essas medidas, se for construído um novo
a) (-∞, -1) ∪ (0,1)
bloco retangular, com arestas medindo (r – 1), (s – 1) e (t – 1), ou seja,
b) (-∞, -1) ∪ (2, +∞) cada aresta medindo 1 cm a menos que a do bloco anterior, a medida
 1 1 do volume desse novo bloco será
c) ( −∞ , − 1) ∪  − ,  ∪ [2, + ∞ )
 2 2 a) 36 cm³ c) 54 cm³ e) 80 cm³
1  5 
d) ( −∞ , − 3) ∪  ,2  ∪  , + ∞  b) 45 cm³ d) 60 cm³
2  2 
e) (-∞, -1) ∪ (1,2) ∪ (3, +∞) 08. (ESPCEX) Seja a função complexa P(x) = 2x³ – 9x² + 14x – 5.
Sabendo-se que 2 + i é raiz de P, o intervalo I de números reais que faz
44. Um polinômio P(X) de grau maior que 3 quando dividido por x – 2, P(x) < 0, para todo x ∈ I é
x – 3 e x – 5 deixa restos 2,3 e 5, respectivamente. O resto da divisão
 1 1  e) ]0,+∞[
de P(X) por (x – 2)(x – 3)(x – 5) é a)
 −∞, 2  c)  4 ,2
a) 1 d) x – 1
 1 3
b) x e) x – 30 b) ]0,1[ d)  − 4 , 4 
c) 30

209

PM_BOOK16 - MAT.indb 209 25/11/2022 19:13:27


EQUAÇÕES POLINOMIAIS

09. (ESPCEX) A figura a seguir apresenta o gráfico de um polinômio


P(x) do 4º grau no intervalo ]0,5[. 4 (ITA) Seja p o polinômio dado por p(x) = x8 + xm – 2xn,
em que os expoentes 8, m, n formam, nesta ordem, uma
progressão geométrica cuja soma dos termos é igual a 14.
Considere as seguintes afirmações:
I. x = 0 é uma raiz dupla de p.
II. x = 1 é uma raiz dupla de p.
III. p tem quatro raízes com parte imaginária não nula.
Destas, é(são) verdadeira(s):
a) apenas I.
b) apenas I e II.
c) apenas I e III.
O número de raízes reais da equação P(x) + 1 = 0 no intervalo ]0,5[ é: d) apenas II e III.
a) 0 c) 2 e) 4 e) I, II e III.
b) 1 d) 3

10. (ESPCEX) Dado o polinômio q (x) que satisfaz a equação


5I.
(ITA) Considere as seguintes afirmações:

se x1, x2 e x3 são as raízes da equação x³ – 2x² = x + 2 = 0,


x³ + ax² – x + b = (x – 1)⋅q(x) e sabendo que 1 e 2 são raízes da equação então y1 = x2x3, y2 = x1x3 e y3 = x1x2 são as raízes da equação
x³ + ax² – x + b = 0, determine o intervalo no qual q(x) < 0: y³ – y² – 4y – 4 = 0.
a) [– 5, – 4] c) [– 1, 2] e) [6, 7] II. a soma dos cubos de três números inteiros consecutivos é
b) [– 3, – 2] d) [3, 5] divisível por 9.

3 + 5 1+ 5
III. = .
2 2
É(são) VERDADEIRA(S):

DESAFIO PRO a) apenas I.


b) apenas II.
15 c) apenas III.

1 (ITA) Seja p o polinômio dado por p(x) = ∑ aj x j , com aj ∈ ,


j= 0
j = 0,1,...,15, e a15 ≠ 0. Sabendo-se que i é uma raiz de p e
d) apenas II e III.
e) todas.
que p(2) = 1, então o resto da divisão de p pelo polinômio q,
dado por q(x) = x³ – 2x² + x – 2, é igual a:
1 2 1 GABARITO
a) x − .
5 5
EXERCÍCIOS DE FIXAÇÃO
1 2 1
b) x + . 01. E 04. A 07. B 10. C
5 5
2 2 2 02. D 05. A 08. B
c) x + .
5 5 03. A 06. B 09. B
3 2 3 EXERCÍCIOS DE TREINAMENTO
d) x + .
5 5 01. E 13. D 25. A 37. D
e) 3 2 1 02. D 14. D 26. D 38. D
x + .
5 5 03. E 15. D 27. D 39. B

2 (IME) Sejam x1, x2 e x3 raízes da equação x³ - ax – 16 = 0. 04. B 16. C 28. D 40. E


Sendo a um número real, o valor de x1³ + x2³ + x3³ é igual a: 05. C 17. A 29. A 41. B
a) 32 - a 06. C 18. B 30. E 42. E
b) 48 – 2a 07. A 19. A 31. E 43. E
c) 48 08. C 20. C 32. A 44. B
d) 48 + 2a 09. D 21. A 33. D 45. C
e) 32 + a 10. B 22. C 34. E
11. E 23. C 35. C

3 (IME) O polinômio P(x) = x³ – bx² + 80x – c possui três raízes


inteiras positivas distintas. Sabe-se que duas das raízes do
polinômio são divisoras de 80 e que o produto dos divisores
12. D 24. E
EXERCÍCIOS DE COMBATE
36. A

positivos de c menores do que c é c². Qual é o valor de b? 01. DISCURSIVA 04. D 07. B 10. C
a) 11 02. DISCURSIVA 05. C 08. A
b) 13 03. E 06. E 09. C
c) 17 DESAFIO PRO
d) 23 01. B 03. E 05. E
e) 29 02. C 04. C

210

PM_BOOK16 - MAT.indb 210 25/11/2022 19:13:29


PROGRESSÕES

SEQUÊNCIAS 1 1 3
III. 1, , 0, − , −1, − é uma progressão aritmética com 6 termos
Antes de começarmos o estudo das progressões, veremos uma 2 2 2
definição um pouco mais geral: estudaremos o que é uma sequência. 1
e cuja razão é − .
2
Intuitivamente, uma sequência é uma lista de elementos que
estão escritos em uma determinada ordem. CLASSIFICAÇÃO
Formalmente, uma sequência é uma função cujo domínio é o CRESCENTES: aquelas onde cada termo é maior que o anterior,
conjunto dos números inteiros positivos. ou seja, onde a razão é positiva.
Exemplo:
Vejamos alguns exemplos de sequências:
I. 1, 2, 3, 4, 5, 6, ... (sequência (infinita) dos inteiros positivos); 1,2,3,4,5,6,...
II. 2, 4, 6, 8, 10, ... (sequência (infinita) dos inteiros positivos CONSTANTES: aquelas onde cada termo é igual ao anterior, ou
pares); seja, aquelas PA’s com razão nula.
III. 2, 3, 5, 7, 11, 13, 17, ... (sequência (infinita) dos números Exemplo:
primos positivos);
1,1,1,1,1
IV. 5, 10, 15, 20, 25 (sequência (finita) dos inteiros positivos
múltiplos de 5 que são menores que 30); DECRESCENTES: aquelas onde cada termo é menor que o
V. 1, 1, 2, 3, 5, 8, 13, 21, .... (sequência (infinita) de Fibonacci anterior, ou seja, onde a razão é negativa.
– cada termo, a partir do segundo, é a soma dos dois termos
anteriores). CONDIÇÃO PARA QUE 3 TERMOS EM
Uma notação muito útil para representar sequências é a seguinte: SEQUÊNCIA FORMEM UMA P.A.
a1 representa o primeiro termo da sequência; (...,an −1,an ,an +1,...) ⇒ an +1 − an = an − an −1 ⇒
a2 representa o segundo termo da sequência; an +1 + an −1
a3 representa o terceiro termo da sequência; 2an = an +1 + an −1 ⇒ an =
2
...
an representa o n-ésimo termo da sequência. Exemplo:
Encontre o valor de x para que os termos (x + 1, 2x – 3, 4x – 2)
Com efeito, no exemplo V, teríamos: formem, nessa ordem, uma progressão aritmética.
a1 = 1, a2 = 1, a3 = 2, a4 = 3, a5 = 5, a6 = 8, a7 = 13, a8 = 21
(x + 1,2x − 3,4x − 2) ⇒ (4x − 2) − (2x − 3) = (2x − 3) − (x + 1) ⇒
4x − 2 − 2x + 3 =2x − 3 − x − 1 ⇒ 2x + 1 =x − 4 ⇒ x =−5
PROGRESSÃO ARITMÉTICA (PA) ( −4, −13, −22) ⇒ r =−9
Agora que já vimos o que é uma sequência, estamos aptos para
4x − 2 − 2x + 3 =2x − 3 − x − 1 ⇒ 2x + 1 =x − 4 ⇒ x =−5
introduzir a nossa primeira progressão.
( −4, −13, −22) ⇒ r =−9
Antes de mais nada, veja o seguinte exemplo: a Copa do Mundo
da França ocorreu em 1998, a Copa do Mundo da Coreia do Sul e do
Japão ocorreu em 2002, a Copa do Mundo da Alemanha ocorreu em TERMO GERAL
2006, a Copa do Mundo da África do Sul ocorreu em 2010 e a última Vamos deduzir agora fórmulas para encontrar um termo de uma
Copa do Mundo, realizada no Brasil, ocorreu em 2014. Repare que a PA se conhecermos outro termo e a razão.
Copa do Mundo acontece de 4 em 4 anos. Pela definição que veremos
a seguir, os anos em que ocorrem a Copa do Mundo formam uma
progressão aritmética. N-ÉSIMO TERMO EM FUNÇÃO DO PRIMEIRO
Como a diferença entre dois termos consecutivos é sempre igual
à razão da PA, podemos escrever:
DEFINIÇÃO
Uma sequência a1, a2, a3, ... é dita uma PROGRESSÃO ARITMÉTICA a2  a1  r
(PA) se a diferença entre quaisquer dois termos consecutivos é a  a  r
 3 2
constante, isto é, se an1  an  r , para todo n inteiro positivo e onde a4  a3  r
r é uma constante. Neste caso, dizemos que r é a razão desta PA. 
 
Exemplos:
an1  an2  r
I. 1,1,1,1,1 é uma progressão aritmética com 5 termos e cuja 
razão é igual a 1 – 1 = 0. an  an1  r
II. 1,2,3,4,5,6,... é uma progressão aritmética infinita com razão Somando estas n – 1 equações, repare que há cancelamento de
1. vários termos:

211

PM_BOOK16 - MAT.indb 211 25/11/2022 19:13:32


PROGRESSÕES

a2  a1  a3  a2  a4  a3    an1 an2  an  an1  n  1 r Observação

Assim, sobrarão apenas os termos an e a1 e então obtemos que 1°) A soma de 2 termos equidistantes numa P.A. é sempre constante
an – a1 – (n – 1)r. e para verificar quais são os pares de termos equidistantes em
qualquer PA basta lembrar que a1 e an são sempre equidistantes e
an  a1  n  1 r seus índices somam “n + 1” assim qualquer outro par de termos
que os índices somarem “n + 1” também serão equidistantes.
N-ÉSIMO TERMO EM FUNÇÃO DO P-ÉSIMO PA de 28 termos: {a1,a2,...,a27,a28} ⇒ n + 1 = 29, assim por exemplos
todos os termos que os índices somarem 29 serão equidistantes,
Com um procedimento análogo ao do item anterior, podemos como a5 e a24, a11 e a18, como tantos outros.
deduzir que:
2°) Outro fato importante quanto a equidistância de termos se
an  ap  n  p  r dá na fórmula da soma dos termos de uma PA. A soma (a1 + an)
pode ser substituída por qualquer outra soma de pares de termos
Veja que esta fórmula permite relacionar quaisquer dois termos equidistantes, veja o exemplo.
de uma PA. Além disso, a fórmula deduzida em I é um caso particular
desta.
Exemplo:
Numa P.A. de 15 termos o 8° termo é igual a 12, encontre a soma
INTERPOLAÇÃO ARITMÉTICA dos 15 termos dessa PA.
Interpolar, inserir ou intercalar k meios aritméticos entre os ( a + a ) .15
extremos a e b significa construir uma progressão aritmética de k + 2 Pela fórmula da soma teremos S15 = 1 15 , onde a soma
2
termos, sendo que o primeiro termo é igual a a e o segundo termo é a1 + a15 pode ser substituída pela soma de quaisquer 2 outros pares
igual ab. Vejamos um exemplo: de termos equidistantes, pela nossa regra todos os termos que os
Exemplo: índices somarem 16 (n + 1) serão equidistantes, dessa forma são
Intercalar 10 meios aritméticos entre os extremos 2 e 57. equidistantes: a1 e a16, a2 e a15, a3 e a15,..., a8 e a8 (numa PA de
quantidade de termos ímpar o termo central é equidistante de
Solução:
si mesmo) assim podemos usar na fórmula da soma dos termos
Queremos construir uma PA a1, a2, ..., a12 de forma que a1 = 2 e
= S15
( a8 +=
a8 ) .15 ( 2a8 ) .15 2.12.15
= = 180 .
a12 = 57. Com base nisto, vamos determinar a razão r da PA: pela
fórmula do termo geral, temos que a12 = a1 + (12 – 1)r. Substituindo 2 2 2
a1 = 2 e a12 = 57, segue que 57 = 2 + 11r ⇔ r = 5. Desta forma, os
meios que devemos inserir são: 7,12,17,22, 27,32,37,42,47,52. Vejamos agora alguns exercícios resolvidos antes de darmos
continuidade à matéria.
SOMA DOS TERMOS
Exercício Resolvido
Agora, estaremos interessados em calcular a soma dos termos de
uma progressão aritmética: Sn = a1 + a2 + ... + an-1 + an. Para isso, 01. De 1995 a 2004, a população de uma cidade vem aumentando
precisaremos do seguinte: anualmente em progressão aritmética. Em 2004 constatou-se
que o número de habitantes era 8% maior que no ano anterior.
ProBizu Pode-se concluir que, de 1995 a 2004, a população dessa cidade
Podemos representar PA’s da seguinte forma bastante útil: aumentou em:
3 termos: (x – r,x,x + r) a) 200% c) 160% e) 80%
r b) 180% d) 100%
4 termos: (x – 3y,x – y,x + y,x + 3y), onde y =
2
5 termos: (x – 2r,x – r,x,x + r,x + 2r) Resolução: A
Para mais termos, o procedimento é completamente análogo. De 1995 a 2004, há 10 anos a serem considerados. Seja a1 a
população no ano de 1995 e a10 a população no ano de 2004.
Para calcular Sn, utilizaremos o seguinte artifício (escreveremos a Da informação “Em 2004 constatou-se que o número de habitantes
soma desejada na ordem inversa) era 8% maior que no ano anterior”, podemos concluir que:
 8 
Sn  a1  a2    an1  an a10  1  a9  1, 08a9 (*)
  100 
Sn  an  an1    a2  a1 Como a10 = a9 + r, substituindo a9 = a10 – r na equação (*), temos
que:
Somando estas duas últimas equações, temos que
a10  1, 08  a10  r   1, 08r  0, 08a10 
2Sn   a1  an    a2  an1      an1  a2    an  a1  .
27 2 27r
r a10  a10 
Utilizando o ProBizu, segue que 2Sn = n(a1 + an) e então obtemos 25 25 2
a seguinte fórmula para a soma dos termos de uma PA: Finalmente, temos que a10 = a1 + (10 – 1)r = a1 + 9r ⇔ a10 – a1 = 9r.
27r 9r
Com isso, a1   9r  .
 a1  an  n 2 2
Sn 
2 Assim, o aumento percentual da população de 1995 até 2004 é de
Eu gosto de gravar esta soma com a seguinte frase: “a soma dos  a10  a1  9r .
   100%  9r  100%  200%
termos de uma PA é igual ao primeiro termo mais o último termo  a1 
vezes a quantidade de termos; isso tudo dividido por 2”. 2

212

PM_BOOK16 - MAT.indb 212 25/11/2022 19:13:39


PROGRESSÕES

Exercício Resolvido CLASSIFICAÇÃO


02. Determine quatro números em progressão aritmética crescente, CRESCENTES
sabendo que sua soma é 8 e que a soma de seus quadrados é 36.
Uma progressão geométrica é crescente quando o termo
Resolução: subsequente é maior que o termo anterior.
Sejam (x – 3y,x – y,x + y,x + 3y) os quatro números. Desta forma, an +1 > an
temos:
x  3y  x  y  x  y  x  3y  8 Caso 1: a1 > 0
 Para que uma P.G. de primeiro termo positivo seja crescente
 x  3y    x  y    x  y    x  3y   36
2 2 2 2
devemos ter q > 1 .
Simplificando as equações utilizando produtos notáveis, temos:
Caso 2: a1 < 0
4 x  8
 2 2 Para que uma P.G. de primeiro termo negativo seja crescente
4 x  20y  36 devemos ter 0 < q < 1 .
Da primeira equação, x = 2. Substituindo este valor na segunda
equação, obtemos que 16 + 20y² = 36 ⇔ y = ±1. Como a
Exemplos:
progressão é crescente, temos que y > 0. Logo y = 1 e a progressão
desejada é (–1,1,3,5). ( 2,6,18,54,…) é uma P.G. crescente pois o primeiro termo é
positivo ( a1 = 2) e a razão é maior que 1 ( q = 3).
 1 1 
 −4, −2, −1, − , − ,… é uma P.G. crescente pois o primeiro
PROGRESSÃO GEOMÉTRICA (PG)  2 4 
Veja a história a seguir: termo é negativo ( a1 = −4 ) e a razão é um número compreendido
 1
“O grão-vizir, principal conselheiro do rei, tinha inventado um novo entre 0 e 1  q =  .
 2
jogo. Era jogado com peças móveis sobre um tabuleiro quadrado que
consistia em 64 quadrados vermelhos e pretos. O objetivo era capturar
o rei inimigo, e por isso o jogo era chamado, em persa, shahmat – DECRESCENTES
shah para rei, mat para morto. Morte ao rei. O jogo, claro, é o xadrez.
Uma progressão geométrica é decrescente quando o termo
Mas reza a história que o rei ficou tão encantado com a invenção que
subsequente é menor que o termo anterior.
mandou o grão-vizir determinar sua própria recompensa. O grão-vizir
já tinha a resposta na língua: era um homem modesto, disse ao xá. an +1 < an
Desejava apenas uma recompensa simples. Apontando as oito colunas
e as oito filas de quadrados no tabuleiro que tinha inventado, pediu Caso 1: a1 > 0
que lhe fosse dado um único grão de trigo no primeiro quadrado, o Para que uma P.G. de primeiro termo positivo seja decrescente
dobro dessa quantia no segundo, o dobro dessa quantia no terceiro e devemos ter 0 < q < 1.
assim por diante, até que cada quadrado tivesse o seu complemento
de trigo. Não protestou o rei, era uma recompensa demasiada
Caso 2: a1 < 0
modesta para uma invenção tão importante.
Para que uma P.G. de primeiro termo negativo seja decrescente
No entanto, quando o mestre do Celeiro Real começou a contar
devemos ter q > 1.
os grãos, o rei se viu diante de uma surpresa desagradável. O número
de grãos começa bem pequeno: 1, 2, 4, 8, 16, 32, 64, 128, 256, 512,
1024... mas quando se chega ao 64° quadrado, o número se torna Exemplos:
colossal, esmagador. O número final chega a quase 18,5 quintilhões
 2 
(se cada grão tivesse o tamanho de um milímetro, todos os grãos 18, 6, 2, ,… é uma P.G. decrescente pois o primeiro termo é
juntos pesariam cerca de 75 bilhões de toneladas!).”  3 
 1
positivo ( a1 = 18 ) e a razão está entre 0 e 1  q = .
Este é um exemplo de progressão geométrica, que é o assunto  3
que estudaremos a seguir. ( −1, −2, −4, −8, −16,…) é uma P.G. decrescente pois o primeiro
termo é negativo ( a1 = −1) e a razão é maior que 1 ( q = 2).
DEFINIÇÃO
Uma sequência a1, a2, a3, ... é dita uma PROGRESSÃO
ALTERNANTES
GEOMÉTRICA (PG) se cada termo é igual ao anterior vezes uma
constante, isto é, se an1  an  q , para todo n inteiro positivo e onde Uma progressão geométrica é alternante quando o termo
subsequente possui o sinal contrário do termo anterior.
q é uma constante. Neste caso, dizemos que q é a razão desta PG.
an +1 ⋅ an < 0
Exemplos:
I. 1, 1, 1, 1, 1 é uma progressão geométrica com 5 termos e Para que uma P.G. seja alternante basta que a sua razão seja
1 negativa (q < 0).
cuja razão é igual a = 1.
1
II. 1,2,4,8,16,... é uma progressão geométrica infinita com
Exemplo:
razão 2.
1 1 1 1 1 ( 3, −6,12, −24, 48,…) é uma P.G. alternante pois a sua razão é
III. 1, − , , − , , − é uma progressão aritmética com 6 negativa ( q = −2).
2 4 8 16 32 1
termos e cuja razão é − .
2

213

PM_BOOK16 - MAT.indb 213 25/11/2022 19:13:42


PROGRESSÕES

CONSTANTE SOMA DOS TERMOS DE UMA PG FINITA


Uma progressão geométrica é constante quando todos os termos Agora, estaremos interessados em calcular a soma dos termos de
são iguais. uma progressão geométrica:
an +1 = an Sn  a1  a2    an1  an .
Utilizando a fórmula do termo geral, podemos reescrever
Para que uma P.G. seja constante basta que a razão seja igual a
1 ou o caso particular de primeiro termo igual a 0 e qualquer razão. Sn  a1  a1q  a1q2    a1qn1 (1)
O truque agora é multiplicar ambos os lados pela razão q, obtendo
assim:
Exemplo:
( 4, 4, 4, 4, 4,…) é uma P.G. constante pois a razão é igual a 1 Snq  a1q  a1q2  a1q3    a1qn (2)
( q = 1). Fazendo (2) – (1), obtemos, após cancelar os termos comuns:
( 0, 0, 0, 0,…) é uma P.G. constante e não é possível determinarmos 
Sn  q  1  a1 qn  1 
a razão.
Supondo q ≠ 1, obtemos a seguinte fórmula:
A P.G. ( 0, 0, 0, 0,…) é chamada de P.G. constante de razão
indeterminada.
Sn 

a1 qn  1 
q 1
ESTACIONÁRIA
Uma progressão geométrica é estacionária quando sua razão é Por outro lado, se q = 1, todos os termos da PG são iguais ao
igual a 0 ( q = 0 ) e o primeiro termo é diferente de 0 ( a1 ≠ 0 ). primeiro e obtemos:
( 4, 0, 0, 0, 0,…) é uma P.G. estacionária pois q = 0 e a1 = 4. Sn = na1

TERMO GERAL
Vamos deduzir agora fórmulas para encontrar um termo de uma PRODUTO DOS TERMOS DE UMA PG
PG se conhecermos outro termo e a razão. Estamos agora interessados em calcular Pn = a1a2...an. Substituindo
a2, a3, ..., an pelas expressões obtidas através do termo geral, temos:
N-ÉSIMO TERMO EM FUNÇÃO DO PRIMEIRO Pn  a1  a1q  a1q2    a1qn1 .
Podemos escrever:
Logo obtemos Pn = a1nq1+2+...+n-1 e como 1 + 2 + ... + n – 1 =
a2  a1q 1  n  1 n  1 n n  1
a  a q 1 2    n  1   , segue que:
2 2
 3 2
a4  a3q nn 1

  Pn  a1nq 2

an1  an2q

an  an1q SOMA DOS TERMOS DE UMA PG INFINITA
Multiplicando todas as equações e cancelando os termos, obtemos: Consideraremos agora PG’s infinitas cuja razão q é tal que
n 1
|q| < 1, ou seja, –1 < q < 1. Estamos interessados em calcular
an  a1q S = a1 + a2 + ... + an + ... . Para isso, note que as somas Sn = a1 + a2 +
... + an se aproximam cada vez mais de S, quando n vai ficando cada
vez maior. Pela fórmula da soma dos termos de uma PG finita, temos
N-ÉSIMO TERMO EM FUNÇÃO DO P-ÉSIMO
que Sn 

a1 qn  1  . Repare agora que quando n fica cada vez maior,
Com um procedimento análogo ao do item anterior, podemos
deduzir que:  q  1
temos que qn se aproxima cada vez mais de 0. Formalmente, temos
an  apqnp a   0  1 a
que S  lim Sn  1  1 . Com isso, obtemos a seguinte
Veja que esta fórmula permite relacionar quaisquer dois termos de
n q 1 1 q
uma PG. Além disso, a fórmula deduzida em I é um caso particular desta. fórmula para a soma dos termos de uma PG infinita:

a1
S (se |q| < 1)
INTERPOLAÇÃO GEOMÉTRICA 1 q
Interpolar, inserir ou intercalar k meios geométricos entre os extremos
a e b significa construir uma progressão geométrica de k + 2 termos,
sendo que o primeiro termo é igual a a e o segundo termo é igual a b. Observação
Vejamos um exemplo: Para três termos consecutivos a, b, c de uma PG, vale que b² = ac.
Intercalar 8 meios geométricos entre os extremos 5 e 2560.
Solução: Vejamos agora mais exercícios resolvidos antes de prosseguirmos
Queremos construir uma PG a1, a2, ..., a10 de forma que a1 = 5 e com a matéria:
a10 = 2560. Com base nisto, vamos determinar a razão q da PG: pela
fórmula do termo geral, temos que a10 = a1q9. Substituindo a1 = 5 e
a10 = 2560, segue que 2560 = 5q9 ⇔ q9 = 512 ⇔ q = 2. Desta forma,
os meios que devemos inserir são:
10,20,40,80,160,320,640,1280

214

PM_BOOK16 - MAT.indb 214 25/11/2022 19:13:52


PROGRESSÕES

Exercício Resolvido PROGRESSÃO ARITMÉTICA-


03. O terceiro termo de uma progressão geométrica é 10 e o sexto GEOMÉTRICA (PAG)
termo é 80. Então a razão desta progressão é:
a) 1 DEFINIÇÃO
b) – 1 Uma PAG é uma sequência cujo termo geral é da forma
an = unvn, onde un é uma progressão aritmética e vn é uma progressão
c) –2
geométrica. Vejamos alguns exemplos para esclarecer a definição.
d) 2
Exemplos:
e) 3
I. 1·1,2·2,3·4,4·8,5·16,... é uma PAG, pois seus termos são os
Resolução: D produtos dos termos da PA 1, 2, 3, 4, 5, ... pelos termos da
PG 1, 2, 4, 8, 16, ...
Temos que a3 = 10 e a6 = 80. Pela fórmula do termo geral, temos
que a6 = a3q6-3 = a3q3. Substituindo os valores, obtemos que: 3 6 9 12 15
II. , , , , é uma PAG, pois seus termos são os
80 = 10q3 ⇔ q3 = 8 ⇔ q = 2. 2 4 8 16 32
produtos dos termos da PA 3, 6, 9, 12, 15 pelos termos da PG
1 1 1 1 1
, , , , .
Exercício Resolvido 2 4 8 16 32

04. A soma dos cinco primeiros termos de uma PG, de razão SOMA DOS TERMOS DE UMA PAG
negativa, é 1/2. Além disso, a diferença entre o sétimo termo e
o segundo termo desta PG é igual a 3. Determine a razão da PG. Não desenvolveremos aqui uma fórmula geral para o cálculo da
soma dos termos de uma PAG, pois não é de grande valia guardar
Resolução: esta fórmula. O mais importante, na verdade, é saber o procedimento
Seja a1, a2, ..., an, ... a PG. Pelo enunciado, temos: para se calcular a soma dos termos de uma PAG. Este procedimento
1 é completamente similar ao método utilizado para o cálculo da soma
S5 = dos termos de uma PG.
2
a7 – a2 = 3
Pela fórmula da soma dos termos de uma PG, temos que Observação

S5 
 5
a1 q  1   1 (1) A ideia que deve ser gravada é: para calcular a soma dos termos de
uma PAG, multiplique a soma desejada pela razão da PG e subtraia
q 1 2 as duas relações encontradas.
Pela fórmula do termo geral, temos que a7 = a1q6 e
a2 = a1q. Substituindo na segunda equação, segue que Vejamos um exemplo para que as coisas fiquem ainda mais claras:
  
3
q

a1q q5  1  3  a1 q5  1  . Substituindo esta última na
Calcule S 
1 2 3
 
4
 
igualdade (1), obtemos: 3 9 27 81
3 Solução:
q 1 3 1 Esta é a soma de uma PAG infinita. Para calcular esta soma,
    q2  q  6  0 multiplicaremos S inicialmente pela razão da PG, que é 1/3. Assim, obtemos:
q 1 2 q  q  1 2
Resolvendo esta equação do segundo grau, levando em conta que S 1 2 3 4
      (1)
q é negativo, temos que q = –2. 3 9 27 81 243
1 2 3 4
S      (2)
3 9 27 81
Exercício Resolvido Fazendo (2) – (1), obtemos:
S 1 2 1 3  2 4  3 5  4
05. A soma dos termos de uma PG infinita é 3. Sabendo que o S       
3 3 9 27 81 243
primeiro termo é igual a 2, então o quarto termo desta PG é:
2S 1 1 1 1 1
2 1      
a) d) 3 3 9 27 81 243
27 27
Caímos agora na soma dos termos de uma PG infinita de termo
1 3 inicial 1/3 e razão 1/3. Assim, usando a fórmula da soma dos termos
b) e)
4 8 de uma PG infinita, temos:
2 1
c)
3 2S 3 1 3
  S
3 1 1 2 4
Resolução: A 3
a1
Temos que a1 = 2 e S   3 (usando a fórmula da soma dos
1 q PROGRESSÕES DE ORDEM SUPERIOR
infinitos termos de uma PG). Substituindo a primeira igualdade na
Considere a seguinte imagem:
2 1
segunda, chegamos a  3  q  . Pela fórmula do termo
1 q 3
3
 1
geral, a4  a1q3  2    
2
.
 3 27

215

PM_BOOK16 - MAT.indb 215 25/11/2022 19:14:03


PROGRESSÕES

Contando o número de bolinhas em cada figura, temos: SOMA DOS TERMOS


• Figura 1 – 1 bolinha
Mais uma vez, não demonstraremos o resultado a seguir.
• Figura 2 – 3 bolinhas
• Figura 3 – 6 bolinhas TEOREMA 2: A soma dos termos de uma PA de ordem k é
• Figura 4 – 10 bolinhas um polinômio de grau k + 1, sem termo independente.
• Figura 5 – 15 bolinhas Vejamos agora um exercício resolvido:
• Figura 6 – 21 bolinhas
A sequência do número de bolinhas em cada figura é dada por Exercício Resolvido
1, 3, 6, 10, 15, 21.
06. Supondo que uma sequência (an)n∈ seja uma PA de ordem 2
Agora, vamos observar as diferenças entre o número de bolinhas e que seus três primeiros termos sejam dados por a1 = 5, a2 = 8 e
entre uma figura e a próxima: a3 = 13, obtenha o polinômio de grau 2 que represente o termo
3–1=2 geral an.
6–3=3
10 – 6 = 4 Resolução:
15 – 10 = 5 Como (an)n∈ é uma PA de ordem 2, pela Proposição 1, devemos ter
21 – 15 = 6 an = P(n) = an2 + bn + c. Assim, P(1) = 5, P(2) = 8 e P(3) = 13, ou seja,
Veja que a sequência formada por estas diferenças constitui  a+b+c = 5
uma progressão aritmética. Pela definição que veremos a seguir, a 
sequência do número de bolinhas em cada figura constituirá uma  4a + 2b + c =8
9a + 3b + c =13
progressão aritmética de segunda ordem. 

Observação Verifica-se que a única solução desse sistema linear é a = 1, b = 0


e c = 4. Portanto, an = n2 + 4
Os números desta sequência que consideramos são chamados de
números triangulares.
Exercício Resolvido

DEFINIÇÃO 07. Calcule 1² + 2² + 3² + ... + n².

A definição de uma PA de ordem superior é recursiva, ou seja, a Resolução:


definição depende das definições anteriores. Começaremos definindo
O termo geral da soma que estamos buscando é da forma k², que
o que é uma PA de 2ª ordem, depois o que é uma PA de 3ª ordem e
é um polinômio de grau 2. Desta forma, queremos calcular a soma
em seguida generalizaremos a definição.
dos termos de uma PA de segunda ordem. Pelo teorema 2, esta
1. PA de 2ª ordem: uma sequência é dita uma PA de segunda soma é um polinômio de grau 3, sem termo independente.
ordem se as diferenças entre seus termos consecutivos constituem
Assim, temos 1² + 2² + 3² + ... + n² = an³ + bn² + cn. Para encontrar
uma PA.
as constantes a,b,c, devemos substituir alguns valores para n:
Exemplos:
n = 1: a + b + c = 1
a) 1, 3, 6, 10, 15, 21, 28 é uma PA de segunda ordem, pois as
n = 2: 8a + 4b + 2c = 1² + 2² = 5
diferenças entre os termos consecutivos formam a sequência
2, 3, 4, 5, 6, 7, que é uma PA. n = 3: 27a + 9b + 3c = 1² + 2² + 3² = 14
b) 1, 3, 7, 13, 21, 31 é uma PA de segunda ordem, pois as Temos então o sistema:
diferenças entre os termos consecutivos formam a sequência a  b  c  1
2, 4, 6, 8, 10, que é uma PA. 
4a  2b  c  5
2. PA de 3ª ordem: uma sequência é dita uma PA de segunda 9a  3b  c  14
ordem se as diferenças entre seus termos consecutivos constituem 
uma PA de segunda ordem. 1 1 1
Resolvendo este sistema, encontramos=a = ,b ,c =
Exemplos: 3 2 6

Assim, a soma pedida é n  n  n  


c) 1, 8, 27, 64, 125, 216 é uma PA de terceira ordem, pois as
3 2
n n  1  2n  1 .
diferenças entre os termos consecutivos formam a sequência 3 2 6 6
7, 19, 37, 61, 91, que é uma PA de segunda ordem, uma vez
que as diferenças entre os termos consecutivos desta última
sequência são 11, 18, 24, 30, que é uma PA.
TERMO GERAL DE UMA P.A. DE 2ª ORDEM
3. PA de k-ésima ordem: uma sequência é dita uma PA de ordem
k se as diferenças entre seus termos consecutivos constituem uma PA Vamos definir o operador diferença (∆an) = an+1 – an que será uma
de ordem k – 1. nova sequência e (∆²an) = an+1 – an da nova sequência encontrada.

( 3,7,13, 21, ...) ⇒ ( ∆a1 ) = 7 − 3 = 4, ( ∆a2 ) = 13 − 7 = 6 e ( ∆a3 ) = 21 − 13 = 8


TERMO GERAL
Não demonstraremos o resultado a seguir neste material, pois a Resultando na nova sequência (4,6,8,...) que é uma PA. Aplicando
demonstração foge ao escopo dos concursos para os quais estamos ∆²an teremos
nos preparando (a prova utiliza indução forte e algumas manipulações
algébricas).
( 4,6, 8,10, ...) ⇒ ( ∆2a1 ) =−
6 4=2, ( ∆ 2a2 ) =−
8 6=2 e ( ∆ 2a3 ) =2

TEOREMA 1: O termo geral de uma PA de ordem k é um


Assim teremos a PA constante (2,2,2,2,...) de onde concluímos que
polinômio de grau k.
(3,7,13,21,...) é uma P.A. de 2ª ordem. Assim podemos definir a fórmula.
 n − 1 n − 2 1 n − 2 2
an =   a1 +   ∆ a1 +   ∆ a1
 0   1   2 

216

PM_BOOK16 - MAT.indb 216 25/11/2022 19:14:05


PROGRESSÕES

Assim para a sequência (3,7,13,21,...) que temos a1 = 3, ∆¹a1 = 4 09. (ESA) Em um treinamento de condicionamento físico, um soldado
e ∆²a1 = 2 teremos inicia seu primeiro dia correndo 800 m. No dia seguinte corre 850 m.
No terceiro 900 m e assim sucessivamente até atingir a meta diária de
 n − 1 n − 2 n − 2 2.200 m. Ao final de quantos dias, ele terá alcançado a meta?
an =  ⋅3+  ⋅4 + ⋅2
 0   1   2  a) 31 c) 27 e) 23
=an
(n − 1)! ⋅ 3 + (n − 1)! ⋅ 4 + (n − 1)! ⋅ 2 b) 29 d) 25
0!(n − 1)! 1!(n − 2)! 2!(n − 3)!

an = 3 +
(n − 1) ⋅ (n − 2)! ⋅ 4 + (n − 1) ⋅ (n − 2) ⋅ (n − 3)! ⋅ 2 10. (EEAR) Seja a PG (a1, a2, a3 · a4) de razão q = 2. Se a1 + a5 = 272,
1⋅ (n − 2)! 2 ⋅ (n − 3)! o valor de a1 é
a) 8 b) 6 c) 18 d) 16
an = 3 + 4n − 4 + n2 − 3n + 2
an = n + n + 1
2

EXERCÍCIOS DE

EXERCÍCIOS DE TREINAMENTO
FIXAÇÃO 01. (ESPCEX) Numa progressão geométrica (PG) crescente de 5
termos, o primeiro e o último correspondem, respectivamente, às
raízes da equação x² – 51x + 144 = 0. O valor da soma do segundo,
01. (EEAR) Considere esses quatro valores x, y, 3x, 2y em PA crescente.
terceiro e quarto termos dessa PG é
Se a soma dos extremos é 20, então o terceiro termo é
a) 12 c) 28 e) 42
a) 9 c) 15
b) 24 d) 36
b) 12 d) 18

02. (ESPCEX) Numa modalidade de corrida, ganha a equipe que


02. (EEAR) A progressão aritmética, cuja fórmula do termo geral é
percorre uma determinada distância em menor tempo, revezando seus
dada por an = 5n – 18, tem razão igual a
atletas a cada 800 metros. A equipe Verde utilizou a tática de organizar
a) –5 b) –8 c) 5 d) 8 seus atletas na ordem crescente de suas velocidades. Sabe-se que o
atleta menos veloz dessa equipe gastou 5 minutos no revezamento e
03. (EEAR) Quatro números estão dispostos de forma tal que que a diferença de tempo entre dois atletas consecutivos foi sempre
constituem uma PG finita. O terceiro termo é igual a 50 e a razão é de 30 segundos. Sabendo que a equipe Verde realizou a prova em 26
igual a 5. Desta maneira, o produto de a1 · a4 vale minutos, a distância total percorrida foi de
a) 10 b) 250 c) 500 d) 1.250 a) 4000 metros. c) 6400 metros. e) 20800 metros.
b) 4160 metros. d) 10400 metros.
04. (EEAR) As casas de uma rua foram numeradas em ordem
crescente segundo as regras: os números formam uma PA de razão 5;
03. (ESPCEX) Considere a soma S = log   + log   + log   + ... + log 
3 4 5 n 
cujo primeiro termo é 1; as casas à direita são ímpares e as à esquerda, 
pares. Assim, se Tiago mora na 3ª casa do lado esquerdo,  2   3   4   n − 1
 3  o4 nº da  5   n 
casa dele é S = log   + log   + log   + ... + log   em que n é um número natural. O menor valor de n
 2 3 4  n − 1
a) 26 b) 31 c) 36 d) 41 para o qual S > 1 é
a) 20 c) 22 e) 29
05. (EEAR) Dada a equação 20x + 10x + 5x + ... = 5, em que o
b) 21 d) 25
primeiro membro representa a soma dos termos de uma progressão
geométrica infinita, o valor de 1/x é
04. (ESPCEX) Sendo a, b e c, nesta ordem, termos de uma progressão
a) 12 b) 10 c) 8 d) 5 aritmética em que a · c = 24 e A, B e C, nesta ordem, termos de uma
progressão geométrica em que A = a, B = c e C = 72, então o valor de b é
06. (EEAR) Os quatro primeiros termos da sequência definida por
a) 4 c) 6 e) 8
an = (–1)n · n + 1, n ∈ *, são tais que
b) 5 d) 7
a) formam uma PA de razão 4
b) formam uma PG de razão 2 05. (AFA) Quanto devemos adicionar a cada um dos números k + 3,
c) a1 + a3 = a2 + a4 k, k – 2 para que, nesta ordem, formem uma Progressão Geométrica?
d) a1 + a2 = a3 + a4 a) 6 – k b) 6 + k c) 1 – 6k d) 1 + 6k

07. (ESA) Em uma progressão aritmética cujo primeiro termo é 1,87 e 06. (AFA) Seja f uma função real que satisfaz as seguintes propriedades:
a razão é 0,004, temos que a soma dos seus dez primeiros é igual a: I. f(0) = 1;
a) 18,88 c) 9,5674 e) 18,99 II. 0 < f(1) < 1; e
b) 9,5644 d) 18,9 III. f(x + y) = f(x)f(y) x, y ∈ .
Então, a expressão f(0) + f(1) + f(2) + f(3) + ... + f(9) é equivalente a
08. (ESA) Em uma Progressão Aritmética com 6 termos, temos que a
soma de seus termos é igual a 102 e seu último termo é 27. Com base a) [f(1)]9 − 1 c) [f(1)]9 − f(1)
nessas informações, a razão dessa progressão é: f(1) − 1 f(1) − 1
a) 3 c) 11 e) 7 [f(1)]10 − 1 [f(1)]10 − f(1)
b) d)
b) 5 d) 4 f(1) − 1 f(1) − 1

217

PM_BOOK16 - MAT.indb 217 25/11/2022 19:14:06


PROGRESSÕES

π π π  16. (AFA) Considere {a,b,c,d} ⊂  e as funções reais f e g tais que


07. (AFA) O valor de sen  + + ... + n + ...  , n ∈ , é f(x) = a + b · cos (cx + d) e g(x) = a + b · tg (cx + d). Sabendo-se que a,
2 4 2 
20
1 b, c e d formam, nessa ordem, uma P.G. cuja soma dos termos é −
a) -1 b) 0 c) d) 1 1 9
2 e primeiro termo , é correto afirmar:
9
08. (AFA) Um oficial que comanda 1540 soldados, quer formá-los em a) o período da função é 2π
triângulo, de modo que a primeira fila tenha um soldado, a segunda
3 ( π + 2)
dois, a terceira três, e assim diante. Quantas filas ele formará? b) a função g está definida para x =
2
a) 75 b) 55 c) 100 d) 45
 4 4
c) o conjunto imagem da função f é  − , 
09. (AFA) Numa P.A., a soma dos 15 primeiros termos é 300 e a soma  9 9
dos 15 últimos é 1200. Sabendo-se que a sucessão tem 30 termos,
 3π + 6 5π + 6 
concluímos que a razão dessa P.A é: d) a função g é crescente para x ∈  ,
 2 2 
a) 1 b) 2 c) 3 d) 4
17. (AFA) Um cão e um gato, ambos parados, observam-se a uma
10. (AFA) Dada a sequência: 10, 102 , 103 ,..., determine o distância de 35 m. No mesmo instante, em que o cão inicia uma
número mínimo de termos consecutivos que se deve multiplicar, a perseguição ao gato, esta parte em fuga. O cão percorre 2 m no
partir do primeiro, para que o produto tenha 12 algarismos na parte primeiro segundo, 4 m no seguinte, 6 m no terceiro segundo e,
inteira, pelo menos. assim, sucessivamente. O gato, apavorado, percorre 3 m no primeiro
a) 4 b) 5 c) 6 d) 7 segundo, 4 m no seguinte, 5 m no terceiro segundo e, assim,
sucessivamente. Considerando que os dois animais se deslocam
sempre sem interrupção em seu movimento e numa trajetória retilínea
11. (AFA) Se a soma dos n primeiros termos de uma progressão
de mesmo sentido, assinale a alternativa INCORRETA.
3n² + n
aritmética (PA) é dada pela fórmula Sn = , então a soma do a) Até o quinto segundo, o cão terá percorrido uma distância igual
quarto com o sexto termo dessa PA é 2
àquela que o separa do gato naquele instante.
a) 25 c) 31 b) Ao final dos três primeiros segundos, o cão ainda está 35 m
b) 28 d) 34 distante do gato.
c) Em dez segundos, o cão alcançará o gato.
12. (AFA) Uma P.A. cujo primeiro termo é zero e uma P.G. cujo d) No oitavo segundo, o gato percorre 14 metros.
primeiro termo é 1 possuem a mesma razão. O nono termo da P.G. é
igual ao quadrado do nono termo da P.A. Então
18. (AFA) Sejam as sequências de números reais (–3,x,y,...) que é uma
a) uma das razões comum é –2. progressão aritmética de razão r, e (x,y,24,...) que é uma progressão
b) a razão comum é –1. geométrica de razão q.
c) a razão comum é 1. r
O valor de pertence ao intervalo
d) não existem as duas progressões. q
 1
13. (AFA) Considere uma P.G. onde o 1º termo é a, a > 1, a razão é
a) 0, 2 
q, q > 1, e o produto dos seus termos é c. Se loga b = 4, logq b = 2 e
logc b = 0,01 então a soma dos termos da P.G. é 1 
b)  2 ,1
a) a41 − a c) a41 − 1 c) [1,2[
a2 − 1 a2 − 1
d) [2,3[
b) a40 − a d) a40 − 1
a2 − 1 a2 − 1 19. (AFA) Considere as proposições abaixo.
n
I. A soma dos infinitos termos da sequência cujo termo geral é n ,
14. (AFA) Num certo jogo de azar, apostando-se uma quantia x, tem- 3
3
se uma das duas possibilidades seguintes: n ∈ *, converge para .
4
1°) perde-se a quantia x apostada;
 2kπ 
2°) recebe-se a quantia 2x. II. Se ak = cos   , k ∈ *, o valor de a1 + a2 + ... + a97 é zero.
 3 
Uma pessoa jogou 21 vezes da seguinte maneira: na 1ª vez, apostou 1 III. Se (3,a,b) formam uma progressão geométrica de razão q e
centavo; na 2ª vez, apostou 2 centavos; na 3ª vez, apostou 4 centavos (a,b,45), uma progressão aritmética de razão r, com a,b ∈ ,
e assim por diante, apostando em cada vez o dobro do que havia r
apostado na vez anterior. Nas 20 primeiras vezes, ela perdeu. Na 21ª então = 6.
q
vez, ela ganhou. Comparando a quantia total T perdida e a quantia Q
recebida, tem-se que Q é igual a Pode-se afirmar que, entre as proposições,
a) T/2 b) 2(T + 1) c) 2T d) T + 1 a) apenas uma é falsa.
b) apenas duas são falsas.
15. (AFA) São dadas uma progressão aritmética e uma progressão
c) todas são falsas.
geométrica alternante com primeiro termo igual a 1. Multiplicando-se
os termos correspondentes das duas sequências obtém-se a sequência d) todas são verdadeiras.
(–1,1,3,...). A soma dos 5 primeiros termos desta sequência é
a) 61 b) 97 c) 103 d) 111

218

PM_BOOK16 - MAT.indb 218 25/11/2022 19:14:07


PROGRESSÕES

20. (AFA) João Victor e Samuel são dois atletas que competem numa Admitindo que a quantidade de tratores produzidos evolua nos anos
mesma maratona. Num determinado momento, João Victor encontra- seguintes segundo a mesma razão de crescimento do período 2010-
se no ponto M, enquanto Samuel encontra-se no ponto N, 5 m à sua 2017, é possível concluir que a meta prevista
frente. a) deverá ser atingida, sendo superada em 80 tratores.
A partir desse momento, um observador passa a acompanhá-los b) deverá ser atingida, sendo superada em 150 tratores.
registrando as distâncias percorridas em cada intervalo de tempo de 1
segundo, conforme tabelas abaixo. c) não deverá ser atingida, pois serão produzidos 1.850 tratores a
menos.
João Victor Samuel
d) não deverá ser atingida, pois serão produzidos 150 tratores a
Intervalo Distância (m) Intervalo Distância (m) menos.
1 3 e) não deverá ser atingida, pois serão produzidos 80 tratores a
1° 1°
2 4 menos.

3 3
2° 2° π π π 
4 4 23. (EN) Sabendo que =
b cos  + + + ...  então o valor de
log2 |b| é  3 6 12 
9
3° 3° 1,0 a) 1 c) -1 e) 3
8
b) 0 d) -2
   
24. (EFOMM) Seja um quadrado de lado 2. Unindo os pontos médios
Sabe-se que os números da tabela acima que representam as distâncias de cada lado, temos um segundo quadrado. Unindo os pontos
percorridas por João Victor formam uma progressão geométrica, médios do segundo quadrado, temos um terceiro quadrado, e assim
enquanto os números da tabela acima que representam as distâncias sucessivamente. O produto das áreas dos dez primeiros quadrados é
percorridas por Samuel formam uma progressão aritmética. −
9

45
a) 2 2 c) 2 2 e) 2-25
Com base nessas informações, é INCORRETO afirmar que ao final do
25

a) 5º segundo, João Victor já terá atingido o ponto N. b) 2 2 d) 2-45
b) 5º segundo, Samuel percorreu uma distância igual à que os
separava nos pontos M e N. 25. (ESPCEX) Considere o triângulo ABC abaixo, retângulo em C, em
c) 6º segundo, João Victor terá alcançado Samuel.  = 30°. Nesse triângulo está representada uma sequência de
que BAC
d) 8º segundo, João Victor estará mais de 8 metros à frente de Samuel. segmentos cujas medidas estão indicadas por L1, L2, L3, ..., Ln, em que
cada segmento é perpendicular a um dos lados do ângulo de vértice A.
21. (AFA) De um dos lados de uma avenida retilínea, estão dispostos
L9
alguns postes nos pontos P1, P2, ..., Pi, i ∈ . Do outro lado dessa O valor é
mesma avenida, estão dispostas algumas árvores nos pontos A1, A2, L1
..., Aj, i ∈ . Sabe-se que: 27 3
a)
• P1P2 = 3 dam 128
• P1Pi = 63 dam 1
b)
• (P1P2, P2P3, ...), ... é uma progressão aritmética finita de razão 3 128
• A1Aj = P1Pi 81
c)
• (A1A2, A2A3, ...), ... é uma progressão geométrica finita de razão 2 256
• i=j 27
d)
Com base nessas informações, é correto afirmar que a maior distância 64
entre duas árvores consecutivas é, em dam, igual a 1
e)
a) 63 b) 32 c) 18 d) 16 256

22. (ESPCEX) Uma fábrica de tratores agrícolas, que começou a 26. (ESPCEX) Se x é um número real positivo, então a sequência (log3
produzir em 2010, estabeleceu como meta produzir 20.000 tratores x, log3 3x, log3 9x) é
até o final do ano de 2025. O gráfico abaixo mostra as quantidades
de tratores produzidos no período 2010-2017. a) Uma Progressão Aritmética de razão 1.
b) Uma Progressão Aritmética de razão 3.
c) Uma Progressão Geométrica de razão 3.
d) Uma Progressão Aritmética de razão log3 x.
e) Uma Progressão Geométrica de razão log3 x.

27. (EFOMM) Um garrafão contém 3 litros de vinho. Retira-se um litro


de vinho do garrafão e acrescenta-se um litro de água, obtendo-se
uma mistura homogênea. Retira-se, a seguir, um litro da mistura e
acrescenta-se um litro de água, e assim por diante. A quantidade de
vinho, em litros, que resta no garrafão, após 5 dessas operações, é
aproximadamente igual a
a) 0,396 d) 0,693
b) 0,521 e) 0,724
c) 0,676

219

PM_BOOK16 - MAT.indb 219 25/11/2022 19:14:08


PROGRESSÕES

28. (ESPCEX) Um menino, de posse de uma porção de grãos de arroz, 37. (EN) Um grande triângulo equilátero será construído com palitos
brincando com um tabuleiro de xadrez, colocou um grão na primeira de fósforos a partir de pequenos triângulos equiláteros congruentes
casa, dois grãos na segunda casa, quatro grãos na terceira casa, oito e dispostos em linhas. Por exemplo, a figura abaixo descreve um
grãos na quarta casa e continuou procedendo desta forma até que os triângulo equilátero (ABC) construído com três linhas de pequenos
grãos acabaram, em algum momento, enquanto ele preenchia a décima triângulos equiláteros congruentes (a linha da base do triângulo ABC
casa. A partir dessas informações, podemos afirmar que a quantidade possui 5 pequenos triângulos equiláteros congruentes). Conforme
mínima de grãos de arroz que o menino utilizou na brincadeira é o processo descrito, para que seja construído um triângulo grande
a) 480 c) 512 e) 1024 com linha de base contendo 201 pequenos triângulos equiláteros
congruentes é necessário um total de palitos igual a
b) 511 d) 1023

a) 15453
3 5 9 1.025
=
29. (ESPCEX) A sequência (a1, a2, ..., a10), onde a1 =, a2 = , a3 , =
, a10 b) 14553
3 5 9 1.025 2 2 2 2
a1 = , a2 = , a3 , =
, a10 é de tal forma que para cada n ∈ {1,2,...,10} temos c) 13453
2 2 2 que a = b 2 + c , onde (b , b , ..., b ) é uma PG com b1 ≠ 0 e de
n n n 1 2 10 d) 12553
razão q ≠ ±1 e (c1,c2,...,c10) é uma PA constante. Podemos afirmar que
a + a + ... + a é igual a e) 11453
1 2 10

a) 98 c) 260 e) 1.028
b) 172 d) 516

π π π 
=
30. (EN) Sabendo que b sec3  + + + ...  então, o valor de 38. (AFA) Uma escultura de chapa de aço com espessura desprezível
log2 |b| é  3 6 12  foi feita utilizando-se inicialmente uma chapa quadrada de 1 metro de
a) 8 b) 4 c) 3 d) 1 e) 0 lado apoiada por um de seus vértices sobre um tubo cilíndrico.
A partir desse quadrado, a escultura foi surgindo nas seguintes etapas:
x − y x − y x − y x − y 1ª) Em cada um dos 3 vértices livres do quadrado foi construído um
 − + − + =
−1
31. (AFA) A solução do sistema  2 6 18 54 1
3x − y =−2 quadrado de lado metro.
é tal que x + y é igual a 2
2ª) Em cada um dos vértices livres dos quadrados construídos
11 7 1
a) c) − anteriormente, construiu-se um quadrado de lado de metro.
3 3 4
10 8 E assim, sucessivamente, em cada vértice livre dos quadrados constru-
b) d) − ídos anteriormente, construiu-se um quadrado cuja medida do lado é
3 3
a metade da medida do lado do quadrado anterior.
32. (EFOMM) Numa progressão geométrica crescente, o 3° termo é A figura seguinte esquematiza a escultura nas etapas iniciais de sua
igual à soma do triplo do 1° termo com o dobro do 2° termo. Sabendo confecção.
que a soma desses três termos é igual a 26, determine o valor do 2°
termo.
26
a) 6 c) 3 e)
7
b) 2 d) 1

33. (EN) A soma dos três primeiros termos de uma P.G. crescente
vale 13 e a soma dos seus quadrados 91. Justapondo-se esses termos,
obtém-se um número de três algarismos. Pode-se afirmar que o resto
da divisão desse número pelo inteiro 23 vale
a) 1 b) 4 c) 8 d) 9 e) 11

34. (EN) O quinto termo da progressão aritmética 3 – x; –x; 9 − x  ,


x∈é Considerando que a escultura ficou pronta completadas sete etapas,
é correto afirmar que a soma das áreas dos quadrados da 7ª etapa é
a) 7 c) –2 e) –18
igual a
b) 10 d) − 14 7 8
a)  1 c)  1
   
35. (EFOMM) Resolvendo 1 + i + i² + ... + in, com n = 4k + 1 e k ∈  4 4
(números inteiros), obtemos 3
8
3
7
b)   d)  
a) in. c) 1. e) 1 + i. 4 4
b) 1 + in+1. d) 1 + i².
39. (ESPCEX) João e Maria iniciam juntos uma corrida, partindo de
1 1+ 2 1+ 2 + 3 um mesmo ponto. João corre uniformemente 8 km por hora e Maria
36. (EN) Considere a sequência x1 = ; x 2 = ; x3 = ;
1+ 2 + 3 + 4 2 1+ 2 1+ 2 + 4 corre 6 km na primeira hora e acelera o passo de modo a correr mais
x4 = ; . O valor de xn é 1
1+ 2 + 4 + 8 km cada hora que se segue. Assinale a alternativa correspondente
2
n +1 n(n + 1) n(n + 1) ao número de horas corridas para que Maria alcance João.
a) c) e)
2 2n − 1 2(2n − 1) a) 3 d) 10
n(n − 1) n(n + 1) b) 5 e) 11
b) d)
2n 2n
c) 9

220

PM_BOOK16 - MAT.indb 220 25/11/2022 19:14:10


PROGRESSÕES

40. (AFA) Constrói-se um monumento em formato de pirâmide 42. (EN) Considere a sequência (a,b,2) uma progressão aritmética e a
utilizando-se blocos cúbicos: sequência (b,a,2) uma progressão geométrica não constante, a, b ∈ .
A equação da reta que passa pelo ponto (a, b) e pelo vértice da curva
y² – 2y + x + 3 = 0 é
a) 6y – x – 4 = 0 d) x + 2y = 0
b) 2x – 4y – 1 = 0 e) x – 2y = 0
c) 2x – 4y + 1 = 0
Para a formação piramidal os blocos são dispostos em uma sequência
de camadas, sendo que na última camada, no topo da pirâmide, 43. (AFA) Sejam (1, a2, a3, a4) e (1, b2, b3, b4) uma progressão aritmética
haverá um único bloco, como mostra a figura a seguir. e uma progressão geométrica, respectivamente, ambas com a mesma
soma dos termos e ambas crescentes. Se a razão r da progressão
aritmética é o dobro da razão q da progressão geométrica, então, o
produto r · q é igual a
a) 15 c) 21
b) 18 d) 24

44. (AFA) Considere, no plano cartesiano, a figura  abaixo, em que


os segmentos horizontais são paralelos ao eixo Ox e os segmentos
verticais são paralelos ao eixo Oy.

Sabe-se que:
Na disposição total, foram utilizados 378 blocos, do topo à base da
pirâmide. Havendo necessidade de acrescentar uma nova camada - os comprimentos de segmentos consecutivos da poligonal, que
de blocos abaixo da base da pirâmide, obedecendo à sequência já começa na origem O(0,0) e termina em Q, formam uma progressão
estabelecida, serão gastos x blocos nesta camada. aritmética decrescente de razão r e primeiro termo a1, em que
 1 
A quantidade total de divisores positivos do número x é igual a − < r < 0;
 15 
a) 2
- dois comprimentos consecutivos da poligonal são sempre perpen-
b) 3
diculares;
c) 4
- OA = a1, AB = a2, BC = a3, e, assim sucessivamente, até PQ = a16.
d) 5
Suponha que uma formiga parta da origem O(0,0), e percorra a
trajetória descrita pela poligonal até chegar ao ponto Q.
41. (ESPCEX) Considere o seguinte procedimento: em uma
circunferência de diâmetro 2R, inscreve-se um hexágono regular para, Com base nas informações acima, analise as proposições abaixo.
em seguida, inscrever neste polígono uma segunda circunferência. 1
I. Se a1 = 1 e r = − , então a distância d percorrida pela formiga
Tomando esta nova circunferência, o processo é repetido gerando 16 17
uma terceira circunferência. Caso este procedimento seja repetido até chegar ao ponto Q é tal que d = a1.
2
infinitas vezes, a soma dos raios de todas as circunferências envolvidas
II. Quando a formiga estiver na posição do ponto L(x,y), então x = 6r.
nesse processo é igual a:
III. Se a1 = 1, então de A até C, a formiga percorrerá a distância
 d = 2 + 3r.
3
a) 2R 1 + 
 2  Quanto a veracidade das proposições, tem-se
a) apenas uma delas é verdadeira.
 3 b) apenas duas são verdadeiras.
b) 4R 1 + 
 2  c) todas são verdadeiras.
d) nenhuma delas é verdadeira.
 3
c) 4R 1 + 
 4  45. (AFA) Considere as expressões A = 26² – 24² + 23² – 21² + 20² –
18² + ... + 5² – 3² e B = 2 2 ⋅ 4 2 ⋅ 8 2 ⋅ 16 2...
d) (
R 2+ 3 ) A
O valor de é um número compreendido entre
B
 3
e) 2R 1 +  a) 117 e 120 c) 111 e 114
 4  b) 114 e 117 d) 108 e 111

221

PM_BOOK16 - MAT.indb 221 25/11/2022 19:14:11


PROGRESSÕES

EXERCÍCIOS DE 05. (ESPCEX) Na figura abaixo temos uma espiral formada pela união

COMBATE
de infinitos semicírculos cujos centros pertencem ao eixo das abscissas.
Se o raio do primeiro semicírculo (o maior) é igual a 1 e o raio de cada
semicírculo é igual à metade do semicírculo anterior, o comprimento
da espiral é igual a
01. (ESPCEX) Considere a progressão aritmética representada pela
7π 47π 59π  a) π
sequência  , , ,...  . Se todos os termos dessa PA forem
 12 60 60  b) 2π
representados num círculo trigonométrico, eles determinarão nesse
c) 3π
círculo os vértices de um
d) 4π
a) pentágono (5 lados). d) decágono (10 lados).
e) 5π
b) hexágono (6 lados). e) dodecágono (12 lados).
c) octógono (8 lados).

02. (ESPCEX) Em uma progressão aritmética, a soma Sn de seus n


primeiros termos é dada pela expressão Sn = 5n2 – 12n, com n ∈ N*.
A razão dessa progressão é
a) –2 c) 8 e) 12 06. (AFA 2000) Se a soma dos 6 primeiros termos de uma progressão
b) 4 d) 10 aritmética é 21 e o sétimo termo é o triplo da soma do terceiro com o
quarto termo, então o primeiro termo dessa progressão é
03. (ESPCEX) Um fractal é um objeto geométrico que pode ser dividido a) –7 c) –9
em partes, cada uma das quais semelhantes ao objeto original. Em mui- b) –8 d) –10
tos casos, um fractal é gerado pela repetição indefinida de um padrão.
A figura abaixo segue esse princípio. Para construí-la, inicia-se com uma
07. (AFA 2010) Sejam as funções f:  →  e g:  →  definidas por
faixa de comprimento m na primeira linha. Para obter a segunda linha, x
uma faixa de comprimento m é dividida em três partes congruentes, f  x   e g(x) = 2-x. Considere os números A e B tais que
2
suprimindo-se a parte do meio. Procede-se de maneira análoga para a A = f(1) + f(2) + ... + f(50)
obtenção das demais linhas, conforme indicado na figura.
B = 1 + g(1) + g(2) + ...
Se o produto de A por B tende para o número α, então α é
a) ímpar múltiplo de 9.
b) par divisor de 10000.
c) par múltiplo de 15.
d) ímpar múltiplo de 25.

 8
08. (AFA 2013) A sequência  x, 6, y, y   é tal que os três primeiros
 3
termos formam uma progressão aritmética e os três últimos formam
uma progressão geométrica. Sendo essa sequência crescente, a soma
de seus termos é
Se, partindo de uma faixa de comprimento m, esse procedimento
for efetuado infinitas vezes, a soma das medidas dos comprimentos a) 92/3 c) 86/3
de todas as faixas é b) 89/3 d) 83/3
a) 3 m
b) 4 m 09. (AFA 1999) Se a sequência de inteiros positivos (2, x, y) é uma
Progressão Geométrica e (x + 1, y, 11) uma Progressão Aritmética,
c) 5m então, o valor de x + y é:
d) 6 m a) 11
e) 7 m b) 12
c) 13
04. (ESPCEX) Os números naturais ímpares são dispostos como
mostra o quadro d) 14
e) 15
1º linha: 1
2º linha: 3 5 10. (EN 2011) Três números inteiros estão em PG. A soma destes
3º linha: 7 9 11 números vale 13 e a soma de seus quadrados vale 91. Chamando de n
o termo do meio desta PG, quantas comissões de n elementos a Escola
4º linha: 13 15 17 19 Naval pode formar com 28 professores do Centro Técnico Científico?
5º linha: 21 23 25 27 29 a) 2276
... ... ... ... ... ... b) 3176
c) 3276
O primeiro elemento da 43ª linha, na horizontal, é:
d) 19656
a) 807 c) 1307 e) 1807
e) 19556
b) 1007 d) 1507

222

PM_BOOK16 - MAT.indb 222 25/11/2022 19:14:13


PROGRESSÕES

GABARITO

DESAFIO PRO EXERCÍCIOS DE FIXAÇÃO


01. B 04. A 07. A 10. D
02. C 05. C 08. D

1 (ITA) Sejam a, b, c, d ∈ . Suponha que a, b, c, d formem,


nesta ordem, uma progressão geométrica e que a, b/2,
c/4, d – 140 formem, nesta ordem, uma progressão aritmética.
03. C 06. D
EXERCÍCIOS DE TREINAMENTO
09. B

Então, o valor de d – b é 01. E 13. A 25. C 37. A


a) –140 02. C 14. C 26. A 38. D
b) –120 03. B 15. D 27. A 39. C
c) 0 04. D 16. A 28. C 40. C
d) 120 05. A 17. D 29. E 41. B
e) 140 06. B 18. C 30. C 42. D
07. B 19. A 31. B 43. B

2 (IME) Sejam uma progressão aritmética (a1,a2,a3,a4,...)


e uma progressão geométrica (b1,b2,b3,b4,...) de termos
inteiros, de razão r e razão q, respectivamente, onde r e q são
08. B
09. D
20. C
21. B
32. A
33. A
44. C
45. B
inteiros positivos, com q > 2 e b1 > 0. Sabe-se, também, que 10. D 22. E 34. C
a1 + b2 = 3, a4 + b3 = 26. O valor de b1 é:
11. B 23. C 35. E
a) 1
12. A 24. E 36. E
b) 2
EXERCÍCIOS DE COMBATE
c) 3
01. D 04. E 07. D 10. C
d) 4
02. D 05. B 08. C
e) 5
03. A 06. C 09. B
DESAFIO PRO
3 (IME) Sabendo-se que os números reais positivos a, b e c
5c  a 
formam uma progressão geométrica e log   , log  
01. D 03. E 05. D
 a  3b  02. A 04. A
a
e log   formam uma progressão aritmética, ambas nessa
 3b  ANOTAÇÕES
ordem, então se pode afirmar que a, b e c.
a) formam os lados de um triângulo obtusângulo.
b) formam os lados de um triângulo acutângulo não equilátero.
c) formam os lados de um triângulo equilátero.
d) formam os lados de um triângulo retângulo.
e) não podem formar os lados de um triângulo.

4 (IME) A soma dos termos de uma progressão aritmética


é 244. O primeiro termo, a razão e o número de termos
formam, nessa ordem, outra progressão aritmética de razão 1.
Determine a razão da primeira progressão aritmética.
a) 7
b) 8
c) 9
d) 10
e) 11

5 (IME) Os ângulos θ1, θ2, θ3, ..., θ100 são os termos de uma
π
progressão aritmética na qual θ11 + θ26 + θ75 + θ90 = . O
4

∑ 
100
valor de sen  θi  é:
 i=1 
a) –1

b) − 2
2
c) 0

d) 2
2
e) 1

223

PM_BOOK16 - MAT.indb 223 25/11/2022 19:14:13


PROGRESSÕES

ANOTAÇÕES

224

PM_BOOK16 - MAT.indb 224 25/11/2022 19:14:13


MATRIZES

DEFINIÇÕES a11  a1,n−2 a1,n−1 a1n 


As matrizes são frequentemente usadas para organizar dados, a  a a2,n−1 a2n 
como uma tabela indexada. Por exemplo, as notas dos alunos de uma  21 2,n−2
escola podem ser dispostas numa matriz cujas colunas correspondem
às matérias e as linhas representam cada um dos alunos.
M = a31  a3,n−2 a3,n−1 a3n 
 
Uma matriz de ordem m × n, M = (aij)m×n, é uma lista de números       
aij, onde 1 ≤ i ≤ m e 1 ≤ j ≤ n, dispostos em m linhas e n colunas, an1  an,n−2 an,n−1 ann 
na qual o elemento aij está localizado no cruzamento da i-ésima linha 
com a j-ésima coluna.
A soma dos elementos da diagonal principal de uma matriz
linha i
 a11 a12  a1n  quadrada é chamada traço da matriz.
a a22  a2n  n
aij M  (aij )mxn  
a
21
  tr(M)   a11  a22  a33    ann
    ii
  i1
 am1 am2  amn 
coluna j
IGUALDADE DE MATRIZES
Os elementos que possuem o mesmo 1º índice encontram-se na Duas matrizes são iguais se, e somente se, possuem a mesma
mesma linha e os que possuem o mesmo 2º índice encontram-se na ordem e todos os elementos correspondentes (elementos com índices
mesma coluna. iguais, i.é, que ocupam a mesma posição) são iguais.
A lista ordenada (ai1, ai2, ..., ain) chama-se i-ésima linha ou i-ésimo Sejam A = (aij)m×n e B = (bij)p×q, então
vetor-linha da matriz. A lista ordenada (a1j, a2j, ..., amj) chama-se
m  p e n  q
j-ésima coluna ou j-ésimo vetor-coluna da matriz. Assim, as linhas de 
uma matriz m × n são vetores do Rn e as colunas, vetores do Rm. A=B⇔  e
a  b
O número de elementos que constituem a matriz m × n é m · n.  ij ij , i, j, 1im, 1 jn

Assim, a matriz 2 × 3 abaixo possui 2 linhas e 3 colunas e um total


de 6 elementos.
ADIÇÃO E SUBTRAÇÃO DE MATRIZES
 1 2 3
M   Sejam as matrizes A e B de mesma ordem m × n, chama-se soma
 3 2 1 de A com B à matriz C = A + B, de ordem m × n, cujos elementos são
obtidos somando-se os elementos correspondentes da matrizes A e B.
Podemos identificar os elementos a11 = 1, a12 = 2, a13 = 3, a21 = −3, Sejam A = (aij)m×n e B = (bij)m×n, então C = A + B é tal que
a22 =−2 e a23 = −1.
C = (cij)m×n onde cij = aij + bij, ∀i, j, 1 ≤ i ≤ m e 1 ≤ j ≤ n
A matriz constituída pelo mesmo número de linhas e colunas é
chamada matriz quadrada. Assim, uma matriz constituída por n linhas  a11 a12  a1n   b11 b12  b1n 
e n colunas é uma matriz quadrada de ordem n × n ou simplesmente a   
uma matriz quadrada de ordem n.  21 a22  a2n    b21 b22  b2n  =
           
Em uma matriz quadrada de ordem n, o conjunto dos elementos    
aij tais que: a a
 m1 m2  amn  b b
 m1 m2  bmn 
a. i = j chama-se diagonal principal;
 a11  b11 a12  b12  a1n  b1n 
b. i + j = n + 1 chama-se diagonal secundária. a b 
 21 21 a22  b22  a2n  b2n 
a11 a12 a13  a1n  

    

a a22 a23  a2n  am1  bm1 am2  bm2  amn  bmn 
 21
M = a31 a32 a33  a3n  A adição de duas matrizes só é definida quando elas possuem a
  mesma ordem. Nesse caso, diz-se que elas são conformáveis para
       adição.
an1 an2 an3  ann  Exemplo:

 1 2 3  3 2 1   1 3 2  2 3  1  4 4 4 
 4 5 6   3 2 1   4  3 5  2 6  1   1 3 5 
       

225

PM_BOOK16 - MAT.indb 225 25/11/2022 19:14:18


MATRIZES

PROPRIEDADES DA ADIÇÃO Considerando as linhas da matriz A e as colunas da matriz B como


a) COMUTATIVA: A + B = B + A vetores no Rn, cada elemento cik é obtido pelo produto escalar do
i-ésimo vetor linha de A pelo k-ésimo vetor coluna de B.
b) ASSOCIATIVA: (A + B) + C = A + (B + C)
O produto de duas matrizes AB somente existe quando A possui
c) ELEMENTO NEUTRO: A + 0 = A onde 0 é a matriz nula da mesma tantas colunas quantas são as linhas de B. Nesse caso, diz-se que as
ordem de A e possui todos os seus elementos nulos. duas matrizes A e B são conformáveis para a multiplicação.
d) MATRIZ OPOSTA: A + (−A) = 0, onde −A é uma matriz da O produto AB é uma matriz que possui o número de linhas de A
mesma ordem de A e cujos elementos são opostos dos elementos e o número de colunas de B.
correspondentes de A.
Dadas A e B duas matrizes de mesma ordem, a diferença das
matrizes A − B é definida como A − B = A + (−B).
 a11 a12 a13  a1n 
Sendo A = (aij)m×n e B = (bij)m×n, então D = A − B é tal que   b11 b12  b1k  b1p   c11  c1k  c1p 
 a21 a2 a23  a2n  
b21 b2  b2k  b2p 
  
D = (dij)m×n, onde dij = aij – bij, ∀ i, j, 1 ≤ i ≤ m e 1 ≤ j ≤ n        ⋅  =       
  b31 b32  b3k  b3p   c i1  c ik  c ip
 ai1 ai2 ai3  ain     
 a11 a12  a1n   b11 b12  b1n                  
a         
  bn1 bn2  bnk  bnp  c 
 21 a22  a2n    b21 b22  b2n  =   m1  cm
k  cm
p 
am1 am2 am3  am
n 
           
   
a a  a b b  b
 m1 m2 mn   m1 m2 mn 
Amxn Cmxp
 a11  b11 a12  b12  a1n  b1n  Bnxp
a b 
 21 21 a22  b22  a2n  b2n 
      cik  ai1b1k  ai2b2k  ai3b3k    ainbnk
 
am1  bm1 am2  bm2  amn  bmn 
Exemplo:
Exemplo: 0 1 2
 1 2 3  3 2 1   1  3 22 3  1   2 0 2  1 2 3    1 0  2  2  3  1 1 1 2  1 3  2 1 2  2  0  3  3 
 4 5 6  2 1 0   4  0  5  2  6  1 4  1 5  1 6  2 4  2  5  0  6  3 
 4 5 6   3 2 1   4  ( 3) 5  ( 2) 6  ( 1)    7 7 7    1 2 3  
         
 7 9 11
 
MULTIPLICAÇÃO DE MATRIZ 16 21 26

POR ESCALAR
Seja um número k ∈  , e uma matriz A = (aij)m×n, o produto k · A
é a matriz B = (bij)m×n obtida multiplicando-se cada elemento de A por PROPRIEDADES
k, isto é, bij = k · aij para todo i e todo j. a. A multiplicação de matrizes não é comutativa,
isto é, para duas matrizes quaisquer A e B é falso que
 a11 a12  a1n   k  a11 k  a12  k  a1n  AB = BA necessariamente.
a a22  a2n   k  a21 k  a22  k  a2n 
k   21  1º CASO: AB existe e BA não existe
           
    m ≠ p ⇒ Am×n · Bn×p = ABm×p ⇒ Bnxp  Amxn  
 BA
am1 am2  amn  k  am1 k  am2  k  amn 
2º CASO: AB e BA existem, mas são de tipos diferentes
Exemplo:
Am×n · Bn×m = ABm×m
 1 2 3  2  1 2  2 2  3   2 4 6 
2     Bn×m · Am×n = BAn×n
 4 5 6 2  4 2  5 2  6 8 10 12
3º CASO: AB e BA existe e são do mesmo tipo (A e B são matrizes
PROPRIEDADES quadradas de mesma ordem), mesmo assim em geral temos AB ≠ BA.
Sejam A e B matrizes m × n e a, b ∈  . 4º CASO: Sejam A e B matrizes quadradas e de mesma ordem,
a. 1 ⋅ A = A quando ocorre AB = BA, diz-se que A e B comutam.
b. (−1) ⋅ A = −A b. Associatividade: (A ⋅ B) ⋅ C = A ⋅ (B ⋅ C)
c. a ⋅ 0m×n = 0m×n c. Distributividade em relação à adição:
d. 0 ⋅ A = 0m×n A ⋅ (B + C) = A ⋅ B + A ⋅ C ⇔ A ⋅ (B + C) = A ⋅ B + A ⋅ C
e. a · (A + B) = a ⋅ A + a ⋅ B d. (k ⋅ A) ⋅ B = A ⋅ (k ⋅ B) = k ⋅ (A ⋅ B)
f. (a + b) ⋅ A = a ⋅ A + b ⋅ A e. Elemento Neutro: Am×n · In = Im · Am×n = Am×n
g. a ⋅ (b ⋅ A) = (ab) ⋅ A
1 0  0
0 1  0
1, se i  j
MULTIPLICAÇÃO DE MATRIZES In  ( ij )nxn onde ij    In  
0, se i  j    
Sejam duas matrizes A = (aij)m×n e B = (bjk)n×p, o produto de A por  
0 0  1
B, A · B, é a matriz m × p, C = (cik)m×p, onde o elemento cik, localizado
na i-ésima linha e k-ésima coluna, é obtido multiplicando-se os
f. Multiplicação pela matriz nula:
elementos da i-ésima linha de A pelos correspondentes elementos da
k-ésima coluna de B e somando os produtos parciais assim obtidos. 0p×m · Am×n = 0p×n
n Am×n · 0n×p = 0m×p
cik  ai1b1k  ai2b2k  ai3b3k    ainbnk  a b
j1
ij jk

226

PM_BOOK16 - MAT.indb 226 25/11/2022 19:14:34


MATRIZES

Observação MATRIZ IDENTIDADE


1. Sendo A ⋅ B = 0 não se pode concluir que A = 0 ou B = 0. Veja É a matriz diagonal, na qual todos os elementos da diagonal
o seguinte exemplo onde A ≠ 0, B ≠ 0 e AB = 0. principal são iguais a 1.

 1 2 0  0 0 0  0 0 0  1 0  0
 1 1 0   0 0 0   0 0 0  0 1  0
      n  
 1 4 0  1 4 9 0 0 0    
 
0 0  1
2. Quando temos A ⋅ B = A ⋅ C ou (B ⋅ A = C ⋅ A) não se pode
concluir que B = C, mesmo que A ≠ 0. Veja o exemplo a seguir, A matriz identidade é o elemento neutro da multiplicação de
onde tem-se AB = AC e B ≠ C. matrizes, assim Am×n · In = Im · Am×n – Am×n.
 1 2 0  1 2 3   1 2 0 1 2 3 
 1 1 0   1 1 1     MATRIZ TRIANGULAR SUPERIOR
    =  1 1 0  1 1 1 =
 1 4 0 2 2 2   1 4 0 1 1 1  Chama-se matriz triangular superior a matriz quadrada que possui
todos os elementos abaixo da diagonal principal nulos.
3 4 1  A é triangular superior ⇒ aij = 0 , se i > j
2 3 2 
   a1n 
3 2 7 a11 a12 a13
0 a  a2n 
 22 a23
3. (A + B)2 = A 2 + AB + BA + B2 A0 0 a33  a3n 
 
      
0 0 0  ann 

TIPOS ESPECIAIS DE MATRIZES
MATRIZ QUADRADA MATRIZ TRIANGULAR INFERIOR
A matriz constituída pelo mesmo número de linhas e colunas é Chama-se matriz triangular inferior a matriz quadrada que possui
chamada matriz quadrada. todos os elementos abaixo da diagonal principal nulos.
Assim, uma matriz constituída por n linhas e n colunas é uma A é triangular inferior ⇒ aij = 0 , se i < j
matriz quadrada de ordem n × n ou simplesmente uma matriz
quadrada de ordem n.  a11 0 0  0
a a 0  0 
 21 22
 a11 a12  a1n  A  a31 a32 a33  0
a a  a2n   
M  (aij )nxn   21 22       
      a  ann 
   n1 an2 an3
a a
 n1 n2  ann 

MATRIZ TRANSPOSTA
MATRIZ NULA A matriz transposta de A, At, é a matriz obtida a partir de A,
É toda matriz que possui todos os seus elementos iguais a zero. trocando-se ordenadamente suas linhas por colunas.
Seja A = (aij)m×n, então At = (bij)m×n, onde bij = aij para todo i, j.
0 0  0   A matriz transposta de A possui tantas linhas quantas são as
0 0  0   colunas de A e tantas linhas quantas são as colunas de A.
0mxn    m linhas
      
 
00  0 
   a a’
a b c 
⇒ A t   b b ’
n colunas
A 
a’ b ’ c ’  
 c c ’
A matriz nula é o elemento neutro da adição de matrizes, assim
A + 0 = A e 0 + A = A. A transposta de uma matriz quadrada pode ser obtida invertendo
os elementos em relação à diagonal principal. Os elementos da
MATRIZ DIAGONAL diagonal principal não mudam de posição.
É toda matriz quadrada em que os elementos não pertencentes
à diagonal principal são iguais a zero, ou seja, aij = 0 sempre que i ≠ j. PROPRIEDADES
a. (At)t = A
a11 0  0 
0 a 0 
b. (A + B)t = At + Bt
22 
An   c. (A – B)t = At – Bt
     
  d. k ∈ R ⇒ (kA)t = k · At
0 0  ann 
e. (A · B)t = Bt · At
f. (A · B · C)t = Ct · Bt · At

227

PM_BOOK16 - MAT.indb 227 25/11/2022 19:14:41


MATRIZES

MATRIZ SIMÉTRICA Dessa forma, mesmo não havendo divisão entre matrizes,
podemos fazer um passo a passo que funciona de maneira similar,
Uma matriz quadrada diz-se simétrica quando aij = aji para todo por exemplo
1 ≤ i, j ≤ n, ou seja, quando é igual à sua matriz transposta.
A⋅X=B
Daí resulta que os elementos simétricos em relação à diagonal
principal são iguais. Queremos encontrar a matriz X, se estivéssemos trabalhando com
números reais, bastava simplesmente fazer
A é simétrica ⇒ A = A t
A
X=
B
Exemplo:
Mas, em se tratando de matrizes, vamos multiplicar ambos os lados
 1 4 5
da igualdade por A–1, mas aqui temos um momento de muita atenção.
A   4 2 6 é simétrica.
Em matrizes, temos que na multiplicação a ordem importa.
 5 6 3 Sendo assim, não necessariamente, A ⋅ B = B ⋅ A, então temos que
ter atenção se multiplicamos nos dois lados da igualdade A–1 pela
esquerda ou pela direita.
MATRIZ ANTISSIMÉTRICA
A⋅X=B
Uma matriz quadrada diz-se antissimétrica quando aij = –aji para
todo 1 ≤ i, j ≤ n, ou seja, quando é igual à oposta de sua matriz Vamos multiplicar os dois lados por A–1 a esquerda.
transposta. A −1 ⋅ A ⋅ X = A −1 ⋅ B

A é antissimétrica ⇒ A = −At Como sabemos que A −1 ⋅ A = In e que In é o elemento neutro da


multiplicação de matrizes, teremos
Daí resulta que os elementos simétricos em relação à diagonal −1
principal são opostos e os elementos pertencentes à diagonal principal A
 X A −1 ⋅ B
⋅ A ⋅=
In
são nulos.
−1
Exemplo: n ⋅ X = A ⋅B
I
X
 0 4 5
X A −1 ⋅ B
=
A   4 0 6 é antissimétrica.
Agora vamos ao processo que utilizaremos para encontrar uma
 5 6 0 
matriz inversa.
Neste módulo veremos apenas a maneira que utiliza a própria
definição, no módulo de determinantes, veremos uma outra maneira
MATRIZ INVERSA que utiliza o cálculo do determinante da matriz.
Em matrizes não há um algoritmo para divisão de matrizes.
Pela definição
Então para entender o funcionamento da matriz inversa vamos
primeiramente ver um exemplo com os números reais. A ⋅ A −1 =
In
Quando temos que finalizar uma equação e nos deparamos, por
2 1 
exemplo, com Então vejamos um exemplo para a matriz A =   . Logo
3x = 4  0 −1
 2 1  −1  1 0 
Nós dizemos de maneira mais coloquial que passamos o 3 para o  ⋅A =  
 0 −1  0 1
outro lado dividindo.
4 x y 
x= Vamos supor a matriz A −1 =  
3 z w
Na verdade, nós sabemos que multiplicamos ambos os lados da  2 1   x y   1 0
  ⋅ =  
equação pelo inverso de 3 que é
1
.  0 −1  z w   0 1 
3
3x = 4 Efetuaremos a multiplicação das matrizes
1 1
⋅ 3x =4 ⋅  2x + z 2y + w   1 0 
3 3  = 
 −z −w   0 1
4
x=
3 Agora da igualdade de matrizes teremos
Mas o que é o inverso de um número? Inverso de um número 1 2y + w =
2x + z = 0
é o número que quando multiplicado pelo número inicial resulta  e 
no elemento neutro da multiplicação. No caso dos números reais o  − z = 0  − w =
1
elemento neutro da multiplicação é o número 1. Do primeiro sistema
Assim 2x + z =
1 1
 ⇒z= 0 e 2x + z =1 ⇒ 2x =1 ⇒ x =
1  −z =0 2
a ⋅ x =1 ⇒ x =
a Do segundo sistema
1 1 2y + w = 0
Então o inverso do número a, com a ≠ 0, é pois a ⋅ =1. 1
a a  ⇒w=−1 e 2y + w = 0 ⇒ 2y − 1 = 0 ⇒ y =
 − w =1 2
Nas operações entre matrizes o elemento neutro da multiplicação Assim
é a matriz identidade In. Sendo assim se a matriz A–1 é a matriz inversa 1 1
da matriz A temos então que x y   
A −1 =
=  2 2 
A ⋅ A −1 =
In  z w   0 −1
 

228

PM_BOOK16 - MAT.indb 228 25/11/2022 19:14:44


MATRIZES

Como podemos perceber não é um processo curto e se fossemos  2 −1 1 1 


05. Seja A −1 =   a matriz inversa de A =   . Sabendo que
utilizá-lo para uma matriz 3 × 3, que possui 9 elementos, teríamos 3  −1 x  1 2 
sistemas 3 × 3. A ⋅ A-1 = I2, o valor de x é
a) 3. c) 1.
PROPRIEDADES b) 2. d) 0.
Se AB = I, necessariamente B = A–1 e então podemos garantir que
BA = I.  0, s e i = j
06. Seja a matriz A m= (aij)2×2 tal que aij =  .
I. (A ) = A
−1 −1

A soma dos elementos de A é


i + j, s e i ≠ j

II. (A ) = (A )
t −1 −1 t
a) 4. c) 6.
b) 5. d) 7.
( A1A 2  Ak ) = ( Ak )  ( A 2 ) ( A1 )
−1 −1 −1 −1
III.
07. A soma dos elementos da diagonal principal da matriz A = (aij)3×3,
(A )
k −1
= ( A −1 )
k
IV.
 i s e i ≠ j
2
tal que aij =  , é um número
i + j s e i =
j
MATRIZ ORTOGONAL
a) múltiplo de 3. c) divisor de 16.
Existe mais um tipo de matriz que é estudada majoritariamente
no nível superior, em álgebra linear, a matriz ortogonal. Dentre outras b) múltiplo de 5. d) divisor de 121.
propriedades, que não seriam assimiladas no momento caso aqui
fossem colocadas, temos que uma matriz é ortogonal (A–t) quando 5 −3
sua matriz inversa é igual a matriz transposta. Ou seja 08. Sendo a matriz A =   , então a transposta de A2 é
−1 6 −4 
=
A t A= A −t
6 7 36 25
Utilize o exemplo abaixo e verifique que encontrando a matriz a)  −2 −3 c) 16 9 
inversa de A teremos a mesma matriz que At.    

 1 2 2  25 36 7 6
b)  9 16  d)  −3 −2
 3 3  
A=   
2 2 −1 
 3 3 
09. (EEAR) O elemento X3,2 da matriz solução da equação matricial
 1 1 10 4 
3 ⋅ X + 2 4  = 2 16
 é
EXERCÍCIOS DE
6 8   0 8 

FIXAÇÃO a) 0
b) –2
c)
d) 1
3

10. (EEAR) O par (x,y), solução da equação matricial


2 −1 1 2   x −4   x 2   13 2x − 4 
01. Se B =   é a matriz inversa de A = 1 4  , então x – y é
x y     2 ⋅ =  3  é
 x y   y 1   x + y 2
8 
a) 2. c) –1.
 1   7 4
b) 1. d) 0. a) (6, ± 3 ) b) (± 5, −2 ) c)
 ± 2 , −5 
 
d) − , 
 3 5

 3 4 5 −2
02.= Sendo A =  e B 0 3  , a soma dos elementos da 2ª EXERCÍCIOS DE

TREINAMENTO
 2 1   
linha de (A – B)t é igual a
a) –4 c) 2
b) –2 d) 4

 1 a   b −1
01. (EEAR) Se   e   são matrizes opostas, os valores de
 2 −1  4 5 3  −1 2   x 2k 
03.= Sendo A =  e B  −1 0 3 , a soma dos elementos da
4 5    a, b, x e k são respectivamente
1ª linha “A ⋅ B” é a) 1,–1,1,1 c) 1,–1,1,–1
a) 22 c) 46 b) 1,1,–1,–1 d) –1,–1,–2,-2
b) 30 d) 58
 1 3 0 1
02. (EEAR) Dadas as matrizes A =   e B=  , o produto
2 1   x  6  A · B é a matriz 2 0   1 2
04. Se   .   = 0 , então o valor de x + y é
 1 −1  y   3 7  3 7
a) 2 2 c) 0 2
a) 4.
   
b) 5.
 4 7 4 4 
c) 6. b)  2 2 d) 0 2
   
d) 7.

229

PM_BOOK16 - MAT.indb 229 25/11/2022 19:14:45


MATRIZES

2 1  1 −1  1 0
03. (AFA) Uma montadora de automóveis prepara três modelos de 09. O valor de D para que a igualdade matricial   = 
carros, a saber: seja verdadeira é:  1 1  −1 a  0 1
MODELO 1 2 3 a) 1 c) 0 e) –1
CILINDRADA (em litro) 1.0 1.4 1.8 b) 2 d) –2

Essa montadora divulgou a matriz abaixo em que cada termo aij 10. (ESPCEX) A soma das raízes da equação a seguir onde 0 < x < 2π, é:
representa a distância percorrida, em km, pelo modelo i, com um litro
de combustível, à velocidade 10j km/h. cos x 1  2 0 0 1 
 3 ⋅ =
 −1  1 1 5 −1
6 7,6 7,2 8,9 8,2 11 10 12 11,8
5 7,5 7 8,5 8 10,5 9,5 11,5 11  a) 0 c) π e) 2π
 
3 2,7 5,9 5,5 8,1 7,4 9,8 9,4 13,1 b) π/2 d) 3π/2

11. (FGV) Uma matriz A de ordem 2 transmite uma palavra de 4 letras


Com base nisso, é correto dizer que em que cada elemento da matriz representa uma letra do alfabeto. A
a) para motoristas que somente trafegam a 30 km/h, o carro 1.4 é fim de dificultar a leitura da palavra, por se tratar de informação secreta,
o mais econômico.
 3 −1
b) se durante um mesmo período de tempo um carro 1.4 e um 1.8 a matriz A é multiplicada pela matriz B =   obtendo-se a matriz
trafegam a 50 km/h, o 1.4 será o mais econômico.  −5 2 
 −10 27 
c) para motoristas que somente trafegam a velocidade de 70 km/h, codificada B · A. Sabendo que a matriz B · A é igual a  ,
o carro 1.8 é o de maior consumo.  21 −39
d) para motoristas que somente trafegam a 80 km/h, o carro 1.0 é podemos afirmar que a soma dos elementos da matriz A é:
o mais econômico. a) 46 c) 49 e) 50
b) 48 d) 47
1 2 y
 
04. Se a matriz  x 4 5  for simétrica, então x + y + z é:  −2 x 
12. (AFA) O valor de x para que o produto das matrizes A =  
3 z 6
   1 −1  3 1
a) 7 d) 11 e B=  seja uma matriz simétrica é:
0 1 
b) 9 e) 12 a) –1 c) 1
c) 10 b) 0 d) 2

 2 x   4   −7  13. (ESPCEX) Os valores de x e y que satisfazem a igualdade


05. (EEAR) Sendo   .   =   , os valores de x e y na matriz
 y −3   −5   3  logx 3 1  1 0  1 1
acima são, respectivamente, log x 0 ⋅ log y 1 =  são, respectivamente:
 3   2  2 0 
a) 3 e –3 9 e –3 9
c) d) –3 e a) 3 e 1/2 c) 9 e 1/2 e) 9 e 2
b) –3 e 3 2 2
b) 3 e 2 d) 3 e 2
06. A, B e C são matrizes quadradas de ordem 3, e I é a matriz
identidade de mesma ordem. Assinale a alternativa correta:  Matriz   x   1
 dos  .  y  =  1
a) (A +B)2 = A2 + 2AB + B2 14. (ESPCEX) Na resolução do sistema       sabe-
 1 1 0  
coeficientes   z  2
b) B ⋅ C = C ⋅ B
se que a matriz  0 −1 2 é a inversa da matriz dos coeficientes.
c) (A + B)⋅(A − B) = A2− B2
 −1 0 1
d) C ⋅ I = C
Nessas condições, os valores de x, y e z são, respectivamente
a) 1, 2, 3 c) 2, 1, 3 e) 2, 3, 1
3 0   2 1
07. (EEAR) Dadas as matrizes A = 
1 −4  e B =  −1 0 , então b) 1, 3, 2 d) 3, 2, 1
A · B – B · A é igual a:    

0 0   −1 7  1 2
a) 0 0  c) 9 15. Assinale a proposição verdadeira: o produto da matriz 
   1 0 1

x y 
2 −3  −3 1 pela matriz   é comutativo se:
b) d) 2 0 1
5 0 
   7 a) x = 1 e y = 0

a b) x = 2 e y = 0
08. (FGV) A matriz b é a solução da equação matricial AX = M c) x = 1 e para todo y ∈ 
 
em que:  c  d) x = 5 e para todo y ∈ 
e) x = 10 e y = 0
1 2 5  28
A = 0 1 4  e M = 15  . Então a² + b² + c² vale: 16. (AFA) As matrizes A, B e C são do tipo m × 3, n × p e 4 × r,
0 0 3   9  respectivamente. Se a matriz transposta de (ABC) é do tipo 5 × 4, então
a) 67 c) 69 e) 71 a) m = p c) n+p=m+r
b) 68 d) 70 b) mp = nr d) r = n

230

PM_BOOK16 - MAT.indb 230 25/11/2022 19:14:47


MATRIZES

3
17. (FGV) Dada a matriz B =   e sabendo que a matriz 24. (AFA) Analise as sentenças abaixo:
2 −1  −4   2i 
A −1 =   é a matriz inversa da matriz A, podemos concluir que a   se i = j
5 3  I. Seja a matriz A = (aij)3×3 definida por  j  elemento
 i + 2j se i ≠ j
matriz X, que satisfaz a equação matricial AX = B, tem como soma de
( )
seus elementos o número da terceira linha e segunda coluna da matriz transposta de A é 8
a) 14 d) 12 II. Seja a matriz B = A – At (At é transposta de A), onde A é uma matriz
b) 13 e) 16 quadrada de ordem n. Então, a diagonal principal de B é nula.
c) 15  1 senθ  π
III. A matriz A =   é inversível se θ ≠ + kπ, k ∈  .
 sen θ 1  2
18.
(EPCAR 3° ANO) Sejam as matrizes inversíveis
 z 2x +2
log(2z − 4) 
 1 0 1 −1  
=A =  e B 1 1  . Marque a alternativa que corresponde à =
IV. A matriz M  4 x x (z + 1)!  é simétrica, então o
0 2    log y y! 
matriz solução da equação BAX = A  y 
produto dos elementos de sua diagonal principal é igual a 36.
 1   1   1  1 1
 2 1  2 1 − 2 1 2 4 É(são) FALSA(S) apenas
a)   b)   c)   d)  
− 1 1 − 1 1  1 1  −1 1 a) I e III. b) II e IV. c) IV. d) I e II.
 4 2   4 2   4 2   2 
25. (AFA) Assinale a alternativa INCORRETA.

 3 −1 6 −4 
19. (FGV) Sabendo que a inversa de uma matriz A é A −1 =  a) Se C =   , então C² é matriz nula.
, 9 −6 
 −5 2 
e que a matriz X é solução da equação matricial X · A = B, em que 1 1 1
1
B = [8 3], podemos afirmar que a soma dos elementos da matriz X é b) Se A = . 1 1 1 , então A² = A.
3  
a) 7 d) 10 1 1 1
b) 8 e) 11 c) A matriz M = (mij)3×3 tal que mij = [i(j + 1)], sendo i ∈ {1,2,3} e
c) 9 j ∈ {1,2,3}, é uma matriz simétrica.
d) Dada uma matriz quadrada T não-nula, a operação T – Tt, em
20. (AFA) Sejam m e n números reais tais que m ≠ n e as matrizes que Tt é a matriz transposta de T, tem como resultado uma matriz
2 1  −1 1 antissimétrica.
= A =  , B  0 1 . Para que a matriz mA + nB NÃO seja
 3 5   
inversível é necessário  9 1 1 1 
=
26. (EPCAR 3° ANO) Sejam as matrizes A =  , P 1 −4  e P a
-1

a) m e n sejam positivos.  4 6   
b) m e n sejam negativos. inversa de P. Se a matriz B é tal que B = P-1 · A · P, tem-se que
c) n + 7m = 0. a) det B = 0 c) B é a matriz diagonal
d) n² = 7m² b) det B = –50 d) B = A

−
 j , se i =
i
j 1 1 2   5 0 −3 
21. (FGV) Seja A = (aij)2x2 uma matriz tal que aij =  j . A 27. (EN) Sejam A =   e B=  e B’ a transposta
inversa da matriz A, denotada por A , é a matriz
-1
( −i) , se i ≠ j  4 −3 0  1 −2 6 
de B. O produto da matriz A pela matriz B’ é
 1  1 2  2 1
 −2 2  − 6 − 3  − 3 − 6  9 2 10   5 4
e)
 −1 10 
a)   c)   e)        
 1 − 1  1 − 2  1 − 1 a)  −8 6 0 c)  0 6  −2 1 
 6  21 −21 −6   −6 0 
 2  3   3 6     
 1  1 2  5 0 −6   −1 11
 −2 2 − 6 − 3  b)   d)  
b)   d)   4 6 0   20 10 
 −1 1  1 2 
 2   6 3   4 (16)y −1
28. (EN) Considere as seguintes matrizes R =  x ;
22. (AFA) Se A = (aij)2×3 e B = (bij)3×4, a expressão para encontrar o 9 a 0
elemento c23, onde AB = (cij), é igual a  1 (4)(2y −1) 2−1   b (2)(2y −1) − 10 c 
S= x  e T= .
a) a21b31 + a22b32 + a23b33 3 b 1 27 13 −6
b) a31b11 + a32b21 + a33b31 A soma dos quadrados das constantes reais x, y, a, b, c que satisfazem
c) a21b13 + a22b23 + a23b33 à equação matricial R – 6S = T é
d) a23b32 a) 23 c) 29 e) 40
b) 26 d) 32
23. Sejam A e B matrizes quadradas de ordem n tais que AB = A e
BA = B. Então [(A + B)t]² é igual a 29. (AFA) Considere A, B, C e X matrizes quadradas de ordem n e
a) (A + B)² d) At + Bt inversíveis. Assinale a alternativa FALSA.
b) 2(At · Bt) e) At · Bt a) (A-1)-1 = A c) A X C = B ⇒ A-1C-1B
c) 2(A + B )
t t b) (A · B · C) = C B A
-1 -1 -1 -1 det A
d) det (2 A B−1) = 2n
det B

231

PM_BOOK16 - MAT.indb 231 25/11/2022 19:14:48


MATRIZES

19941994 19941994   1 1
30. Considere as matrizes A=  e 02. Seja a matriz A    . A soma dos elementos da matriz A é
100

19941994 19941995   0 1
 1 −1
B=  a) 102 c) 150 e) 300
 −1 1  .
b) 118 d) 175
Seja A2 = A ⋅ A e B2 = B⋅B. Determine C = A2 − B2 − (A + B)(A − B).

03. Uma matriz real A é ortogonal se A · At = I, onde I indica a matriz


 a 2a + 1
31. (FUVEST) Considere a matriz A =   em que a é um 1 
a − 1 a + 1  x
identidade e At indica a transposta de A. Se A   2 é ortogonal,
número real. Sabendo que A admite inversa A-1 cuja primeira coluna é  
então x2 + y2 é igual a:  y z 
2a − 1
 −1  , a soma dos elementos da diagonal principal de A é igual a
-1
1 1 3
  a) c) e)
4 2 2
a) 5 b) 6 c) 7 d) 8 e) 9
3 3
b) d)
4 2
0 1
32. Com relação à matriz A =   , a opção correta é:  1 4 0 1 
 −1 −1 2 1 0   2 1 3 1 e C = AB, o
a) A24 = I2, sendo I2 a matriz identidade de ordem 2. 04. Seja A    e B  
 1 0 3  4 0 2 0 
b) A²² = I2, sendo I2 a matriz identidade de ordem 2.
resultado de c23 + c14 + c21 é:
c) A²¹ = A
a) um número natural menor que 2.
d) A²¹ = A²
b) um número cujo sua raiz quadrada resulta em um número
e) A²² = A² complexo conhecido como imaginário.
c) o mesmo resultado que a soma dos inversos das raízes da equação
33. (EFOMM) Determine uma matriz invertível P que satisfaça a
x² – 2x – 1 = 0.
5 0   1 −2
equação P −1 ⋅ A =
0 −2 , sendo A = 3 3  . d) o mesmo resultado que o conjunto verdade da equação
    exponencial 2x+2 + 2x-1 = 18.
 5 10  1 2 10  1  e) o mesmo resultado do produto dos 6 primeiros termos da PG (2-1,
3 9  c) P= 5 1 
a) P=  10 3 −3 e) P=  2-2, 2-3, ...).
2 − 2 3 − 3
 2 2
 3 9   −9 −   5 2  05. Os números das contas bancárias ou dos registros de identidade
3
d) P=  costumam ser seguidos por um ou dois dígitos, denominados dígitos
2 10   − 10 5 
b) P=  verificadores, que servem para conferir sua validade e prevenir erros
6 −15  9 3  de digitação. Em um grande banco, os números de todas as contas
são formados por algarismos de 0 a 9, na forma abcdef-xy, em que a
 π sequência (abcdef) representa, nessa ordem, os algarismos do número
34. Se α ∈ 0, 2  e satisfaz a identidade matricial da conta e x e y, nessa ordem, representam os dígitos verificadores.
Para obter os dígitos x e y, o sistema de processamento de dados do
 3 1  banco constrói as seguintes matrizes:
− −
 cos α − sen α   2 
5
2  , então, o valor correto de tgα é  1 2 1  x  a  b 
  =
 sen α cos α   1 3 A  0 1 0  B   y  C  c  d
 − 
 2 2  0 2 1  z   e  f 
igual a:
a) 0 3 e) 3 Os valores de x e y são obtidos pelo resultado da operação matricial
c)
3 2 A · B = C, desprezando-se o valor de z. Assim, os dígitos verificadores
b) correspondentes à conta corrente de número 356281 são
3 d) 1
a) 34
 0 2 10 b) 41
35. Considere a matriz A =  1

2
0
 . A matriz onde

 j=1
A j é: ∑ c) 49
d) 51
a) I c) I + A e) 7A e) 54
b) A d) 5(I + A)
06. (ESPCEX) As matrizes A, B e C são do tipo r × s, t × u e 2 × w,
respectivamente. Se a matriz (A − B)⋅C é do tipo 3 × 4, então
EXERCÍCIOS DE r + s + t + u + w é igual a

COMBATE a) 10
b) 11
c)
d) 13
12 e) 14

 1 tgx 
07. (ESPCEX) Considere as matrizes M1   e
01. (AFA 1994) Sejam as matrizes A = (aij)3×2 e B = (bij)2×4, com   cos †x cotgx 
aij = –2i + j e bij = 2i – j. O elemento c33 da matriz C = (cij)3x4 = AB é:  1  k
M2    para x  ,k   . A matriz resultante do produto
a) −1 c) 1 tgx  2
matricial M1.M2 é
b) 0 d) 2

232

PM_BOOK16 - MAT.indb 232 25/11/2022 19:14:58


MATRIZES

sec2 x 
a)  2 
cos x 
3 (ITA) Sejam A e B matrizes quadradas n × n tais que
A + B = A · B e In a matriz identidade n × n. Das afirmações:
I. In – B é inversível;
 tg2 x  II. In – A é inversível;
b)  2 
  cos x  III. A · B = B · A.
sec2 x  é (são) verdadeira(s)
c)  2 
sen x  a) Somente I. d) Somente I e II.
cossec2 x  b) Somente II. e) Todas.
d)  
  sen x 
2
c) Somente III.
cos2 x 

4
e)  2  (IME) Seja M uma matriz real 2 × 2. Defina uma função f
sen x  na qual cada elemento da matriz se desloca para a posição
c a
seguinte no sentido horário, ou seja, se f(M) =   . implica que
3 5  x y  4  d b
08. (ESPCEX) Considere as matrizes A    c a
 e B  y 3 
. Se f(M) =  .
1 x   d b
x e y são valores para os quais B é a transposta da Inversa da matriz A,
então o valor de x + y é Encontre todas as matrizes simétricas 2 × 2 reais na qual
a) –1 c) –3 e) –5 M² = f(M).
b) –2 d) –4

a a  b b
3 3
5 (ITA) Sejam x1, ..., x5 e y1, ..., y5 números reais arbitrários e
A = (aij) uma matriz 5 × 5 definida por aij = xi + yj, 1 ≤ i, j ≤ 5.
Se r é a característica da matriz A, então o maior valor possível
 
09. (ESPCEX) Considere a matriz M   a a3 0 . Se a e b são de r é
2
 5 3 a) 1. c) 3. e) 5.
números reais não nulos e det(M) = 0, então o valor de 14a² – 21b² b) 2. d) 4.
é igual a
a) 15 c) 35 e) 70
b) 28 d) 49 GABARITO
EXERCÍCIOS DE FIXAÇÃO
1 0 0 7 0 2
01. C 04. A 07. A 10. B
10. (ITA) Sejam D  0 2 0 e P  0 1 0 . Considere A = P-1DP.
    02. D 05. C 08. D
0 0 3 2 0 5 
03. A 06. C 09. A
O valor de det(A² + A) é
EXERCÍCIOS DE TREINAMENTO
a) 144 c) 240 e) 360
01. C 10. E 19. A 28. B
b) 180 d) 324
02. C 11. D 20. C 29. C
03. C 12. C 21. E  0 1
30.  
04. C 13. E 22. C  −1 0

DESAFIO PRO 05. A


06. D
14. E
15. C
23. C
24. C
31. A
32. A
07. C 16. A 25. C 33. E
08. A 17. B 26. C 34. B
 1 −1  2 1
1 (ITA) Se M =  e N=   , então M NT – M N é
-1
 09. B 18. B 27. D 35. D
igual a 2 0   −1 3
EXERCÍCIOS DE COMBATE
3 5 3 11 3 11 01. C 04. C 07. C 10. A
2 − 2 2 −2 2 −2 02. A 05. E 08. C
a)   c)   e)  
5 − 3  13 − 5  13 − 3  03. E 06. E 09. C
 2 2   2 2   2 2  DESAFIO PRO
3 1 3 5 01. C
 −   − 
b)  2 2 d)  2 2 x x − 1 1− x 
  02. a) B =   , com x e a números reais.
7 − 5 13 − 3  a 1+ a 1+ a 
 2 2   2 2 
1 0 0
b) B =  
 1 0 0 1 0
(ITA) Seja A a matriz de ordem 3 × 2, dada por A = 0 1 .
2
03. E
   0 1 
0   0 1   12 1  − 1
Determine todas as matrizes B tais que BA = I2.  1 1  2  ; 2 2 
04. S =   ; ; 
Existe uma matriz B com BA = I2 que satisfaça BBT = I2? Se sim, dê 0 0  1 0  1 1   1 − 1 
  2 2  2 2 
um exemplo de uma dessas matrizes. 05. B

233

PM_BOOK16 - MAT.indb 233 25/11/2022 19:15:02


MATRIZES

ANOTAÇÕES

234

PM_BOOK16 - MAT.indb 234 25/11/2022 19:15:02


DETERMINANTES

DETERMINANTES DE 1ª ORDEM Vamos aplicar essa definição a uma matriz quadrada de ordem 3
e obter a expressão do menor complementar de alguns elementos.
Seja A = (a11) uma matriz 1 × 1 então det A = |a11| = a11.
 a11 a12 a13 
 
DETERMINANTE DE 2ª ORDEM Considere a matriz A  a21 a22 a23 
a31 a32 a33 
 a11 a12  a11 a12
Seja A    então det A   a11a22  a12a21
 a21 a22  a21 a22 a22 a23
M11 = = a22a33 − a23a32
Exemplo: a32 a33
1 2
 ( 1).3  ( 2).4  5 a21 a23
4 3 M12 = = a21a33 − a23a31
a31 a33
DETERMINANTE DE 3ª ORDEM –
a11 a13
REGRA DE SARRUS M22 = = a11a33 − a13a31
a31 a33
a11 a12 a13
a21 a22 a23  a11a22a33  a12a23a31  a13a21a32  a13a22a31  a12a21a33  a11a23a32 a11 a12
M23 = = a11a32 − a12a31
a31 a32 a33 a31 a32

A Regra de Sarrus é um dispositivo mnemônico para calcular o COFATOR


determinante de 3ª ordem.
Seja uma matriz quadrada de ordem n ≥ 2 e aij um elemento
qualquer de A. O cofator do elemento aij é o número definido por
TERMOS POSITIVOS TERMOS NEGATIVOS Aij = (–1)i+j · Mij onde Mij é o menor complementar de aij.
diagonal principal e diagonal secundária e Calculando os cofatores a partir dos menores obtidos no exemplo
diagonais paralelas a ela diagonais paralelas a ela anterior:
A11 = (–1)1+1 M11 = M11 A12 = (–1)1+2 M12 = –M12
A 22 = (–1) M22 = M22 A 23 = (–1)2+3 M23 = –M23
2+2

Exemplo:
 a11 a12 
Dada M =   , os cofatores relativos a todos os elementos
a21 a22 
da matriz M são:
( 1)1+1 ⋅ a22 =−
A11 =− ( 1)2 ⋅ a22 =+a22 ;

MC11

Exemplo: A12 =−
( 1) 1+ 2
⋅ a
21 =−
( 1)3 ⋅ a21 =−a21 ;
3 −2 4 MC12

2 +1
1 2 −3 = 3⋅2⋅5+(-2)(-3)⋅4 +1⋅1⋅1 -4⋅2⋅4 -(-2)⋅1⋅5 -(-3)⋅1⋅3 = 45. A 21 =−
( 1) ⋅ a
12 =−
( 1)3 ⋅ a12 =−a12 ;
MC21
4 1 5
( 1)2 + 2 ⋅ a11 =−
A 22 =− ( 1)4 ⋅ a11 =+a11 .

MC22
TEOREMA DE LAPLACE
Assim, podemos também determinar a matriz dos cofatores (que
Este nos dá uma maneira geral de calcular o determinante,
será denotada por A) como sendo:
recursivamente, de qualquer matriz quadrada.
Vejamos alguns conceitos preliminares.  A11 A12   a22 −a21 
=A =  
 A 21 A 22   −a12 a11 
MENOR COMPLEMENTAR Exemplo:
Seja uma matriz quadrada de ordem n ≥ 2 e aij um elemento  a11 a12 a13 
qualquer de A. O menor complementar Mij do elemento aij é o  
Sendo M = a21 a22 a23  , vamos calcular os cofatores
determinante da matriz de ordem (n −1), obtida a partir de A A 22 , A 23 e A 31: a31 a32 a33 
eliminando-se a linha i e a coluna j.

235

PM_BOOK16 - MAT.indb 235 25/11/2022 19:15:11


DETERMINANTES

 a11 a13  2 3 −1
( −1)2 + 2 
A 22 = =( −1)4 ⋅ a11a33 − ( a13a31 )  =
( +1) ⋅ a11a33 − ( a13a31 ) 
a31 a33  D2 = 0 + 0 + 2( −1)2 + 3 3 −1 1 ⇒
1)4 a11a33 − ( a13a31 )  =
=⋅ ( +1) ⋅ a11a33 − ( a13a31 )  ; −1 0 3


D
 a11 a12  MC23
( −1)2 + 3 
A 23 = =( −1)5 ⋅ a11a32 − ( a12a31 )  =
( −1) ⋅ a11a32 − ( a12a31 ) 
a31 a32  Obs.: Então podemos rescrever D2 como: D2 = –2D (I)
1) a11a32 − ( a12a31 )  =
=⋅ 5
( −1) ⋅ a11a32 − ( a12a31 )  ; b) Agora precisamos calcular o valor de D para substituirmos em
(I) Para isso aplicamos Laplace na 3ª linha (mais conveniente, pois um
 a12 a13  dos elementos é nulo), e obtemos:
A 31 = ( −1)3 +1  =( −1)4 ⋅ a12a23 − ( a13a22 )  =
( +1) ⋅ a12a23 − ( a13a22 ) 
a22 a23  3 -1 2 3
D =−1( −1)3 +1 + 3( −1)3 + 3 ⇒
1)4 a12a23 − ( a13a22 )  =
=⋅ ( +1) ⋅ a12a23 − ( a13a22 )  . -1 1 3 -1

 
MC31 MC33

TEOREMA DE LAPLACE D =−1(3 − 1) + 3( −2 − 9) =−1(2) + 3( −11) =−2 − 33 ⇒ D = –35


Seja A uma matriz quadrada de ordem n ≥ 2, o determinante de Finalmente, substituindo esse valor em (I), obtemos:
A é a soma dos produtos dos elementos de uma fila (linha ou coluna)
qualquer pelos respectivos cofatores. D2 =−2D ⇒ D2 =
-2(-35) ⇒ D2 = 70

n n
det A   apj  Apj  a iq  Aiq PROPRIEDADE DOS DETERMINANTES
j1 i1 Propriedade 1: o determinante da matriz identidade vale 1.

Usando o teorema de Laplace na 3ª linha para o cálculo do 1 0 … 0


1 0 0
determinante abaixo: 1 0 0 1 … 0
1 =1; = 1; 0 1 0 = 1 ; =1
0 1    
1 2 −3 4 0 0 1
0 0 … 1
−4 2 1 3
= 3 ⋅ A31 − 3 ⋅ A34 = In = 1
3 0 0 −3
2 0 −2 3 Propriedade 2: para toda matriz quadrada A temos que
det(A) = det(At);
2 3 4 1 2 3
1 2 3 1 2 2
3  ( 1)31  2 1 3  3  ( 1)3 4  4 2 1 =
Det A = 2 1 2 = 9 e Det At = 2 1 4 = 9
0 2 3 2 2 2 3 2 3
2 4 3
= 3 ⋅ 20 −3⋅ (−4) = 48 Propriedade 3: seja B a matriz obtida a partir da matriz A pela
troca de duas filas (linhas ou colunas) paralelas. Desta forma, temos
Exemplo: Calcular com o auxílio do Teorema de Laplace, os det(B) = –det(A);
seguintes determinantes:
1 2 3
2 3 4 −1 2 1 −1 =−4
2 3 −4
0 0 2 0 3 2 1
a) D1 =
−2 1 2 b) D2 =
3 −1 1 1
0 5 6 Trocando as posições de L1 e L2, por exemplo, temos:
−1 0 2 3
2 1 −1
Resolução: 1 2 3 = +4
2 3 −4 3 2 1
D1 = −2 1 2
0 5 6 Propriedade 4: toda matriz que possui duas filas paralelas iguais
ou proporcionais tem determinante nulo;
Aplicando Laplace na coluna 1, temos:
2 5 3 5
1+1
1 2 3 −4 3 −4 1 4 2
=D1 2 (-1) + ( −2)(-1)2 +1 +0 3 +1
 (-1) 1 2 ⇒ 4 2 9 8
a11 5 6  5 6 = 0 , pois L1 = L3 e 2 1 4 = 0 , pois C3 = 2C1
 a21    a31
   2 1 3 5
A11(cofator11) CofatorA21 CofatorA31 3 2 6
9 7 4 3
1 2 3 −4
⇒=
D1 2 +2 +0⇒
5 6 5 6 Propriedade 5: toda matriz que possui uma fila com todos os
⇒ D1= 2(6-10) + 2(18 + 20)= 2(-4) + 2(38) ⇒ seus elementos nulos tem determinante nulo;
⇒ D1 =−8 + 76 =68 4 9 −8 7
3 0 15
0 0 0 0
a) Como três dos quatro elementos da 2ª linha são nulos, convém =0 2 0 −3 =0
aplicar Laplace nessa linha. 3 2 −1 3
−1 0 7
2 3 4 −1 18 12 9 3
0 0 2 0
D2 = Propriedade 6: seja B uma matriz obtida a partir de uma matriz A
3 −1 1 1 de modo que a i-ésima fila de B é igual a i-ésima fila de A multiplicada
−1 0 2 3 por uma constante k, então temos det(B) = k ⋅ det(A);

236

PM_BOOK16 - MAT.indb 236 25/11/2022 19:15:14


DETERMINANTES

1 2 3 REGRA DE CHIÓ
2 1 −1 =−4 Este algoritmo serve, assim como o teorema de Laplace, para
3 2 1 baixar a ordem do determinante. Importante saber é que só podemos
aplicar a regra de Chió se existir algum elemento igual a 1.
2 2 3
Multiplicando C1 por 2, temos: 4 1 −1 =2 ⋅ ( −4 ) =−8 ALGORITMO
6 2 1 1. Seja um determinante de ordem n onde aij = 1, suprimem-se
a i-ésima linha e a j-ésima coluna;
5 −10 0 2. De cada elemento restante apq do determinante subtraímos
3 7 4 = −145 apj · aiq;
2 0 −1 3. O novo determinante tem ordem n – 1 e quando multiplicado
por (–1)i+j torna-se igual ao determinante original.
1 −2 0
1
Multiplicando L1 por , temos: 3 7 4 = 1 ⋅ ( −145) =−29 Exemplo:
5 5
2 0 −1
1 2 3
2  2 1 4  3 1 0 1
Propriedade 7: seja A uma matriz de ordem n e k um número 1 2 4    0  7  3  1  3
3  20 7  30 3 7
real, então det(k · A) = kn · det(A); 0 3 7
Seja det A = 3 e que A é quadrada de ordem 3. Dessa forma
det (2A) = 23 ⋅ det A = 8 ⋅ 3 = 24
MATRIZ DE VANDERMONDE
Propriedade 8: Chamamos de matriz de Vandermonde a uma matriz da forma,
a11 … (b 1j + c1j ) … a1n  1 1 1 … 1 
 
a21 … (b2j + c2j ) … a2n
=  a1 a2 a3 … an 
     
V  a1 2
a22
a32 … an2  .
 
an1 … (bnj + cnj ) … ann       
 n1 n1 n1 
a11 … b1j … a1n a11 … c1j … a1n  a1 a2 a3 … ann1 
a21 … b2j … a2n a21 … c2j … a2n
+ O determinante de matrizes de Vandermonde é dado pelo
         
an1 … bnj … ann an1 … cnj … ann produto de todas as possíveis diferenças ai - aj onde i > j.

Propriedade 9: (Teorema de Jacobi) Adicionando-se a uma fila TEOREMA DE BINET


uma combinação linear de outras filas paralelas, o determinante não Sejam A e B matrizes quadradas de mesma ordem, então vale a
se altera; igualdade
1 2 3 det(AB) = det(A) · det(B)
2 1 2 =9
2 4 3
MATRIZ INVERSA
Substituindo a 1ª coluna pela soma dessa mesma coluna com o
dobro da 2ª, temos: Dizemos que uma matriz A quadrada de ordem n é inversível se
existe uma matriz B também de ordem n tal que AB = BA = I. Neste
C1 + 2C2

   caso, dizemos que B é a matriz inversa de A e denotamos B = A-1.
1+ 2 ⋅ 2 2 3 5 2 3
2 + 1⋅ 2= 1 2 4=1 2 9 PROPRIEDADES
2+ 4⋅2 4 3 10 4 3 Todas as matrizes aqui citadas serão quadradas de ordem n e inversíveis.
I. Se AB = I, necessariamente B = A-1 e então podemos garantir
Propriedade 10: para toda matriz triangular (superior ou inferior) que BA = I.
tem determinante igual ao produto dos elementos da diagonal. II. (A-1)-1 = A
a 0 0 x g h III. (At)-1 = (A-1)t
d b 0 = a⋅b⋅c 0 y i = x⋅y⋅z IV. (A1A2...Ak)-1 = (Ak)-1...(A2)-1(A1)-1
e f c 0 0 z V. (Ak)-1 = (A-1)k

Propriedade 11: quando, em uma matriz, os elementos acima ou  


VI. det A 1 
1 e assim A é inversível se, e somente se,
det A
abaixo da diagonal secundária são todos nulos, o determinante é igual
n⋅ (n −1)
seu determinante é não nulo.
ao produto dos elementos dessa diagonal, multiplicado por ( −1) 2 .

0 0 a CÁLCULO DA MATRIZ INVERSA


0 a
=−a ⋅ b 0 b x =−a ⋅ b ⋅ c Veremos agora o método do cálculo da matriz inversa através da
b x
c y z matriz chamada adjunta. No próximo módulo, de sistemas lineares,
veremos como calcular a matriz inversa resolvendo sistemas lineares.

237

PM_BOOK16 - MAT.indb 237 25/11/2022 19:15:17


DETERMINANTES

Dada uma matriz quadrada A, definimos a matriz adjunta de A 1 0 0 3


como adj(A) = (cofA)t, ou seja, a matriz adjunta é a transposta da 2 3 5 1 
matriz dos cofatores. 08. (EEAR) O determinante da matriz  é:
1 2 3 −1
Assim, temos o seguinte resultado:  
3 0 1 4
1
A 1  adj  A  a) 0 b) 8 c) 7 d) 6
det A
Este método é útil quando queremos encontrar um elemento 09. (EEAR) Seja uma matriz M do tipo 2 × 2. Se det M = 2, então
específico da matriz inversa ou quando queremos inverter uma matriz det (10M) é:
de ordem baixa (2 ou 3 em geral). a) 20 b) 80 c) 100 d) 200
ProBizu
10. (ESA) Sabendo-se que uma matriz quadrada é invertível se, e
a b  somente se, seu determinante é não-nulo e que, se A e B são duas
Dada uma matriz   inversível, sua inversa é dada por matrizes quadradas de mesma ordem, então det (A · B) = (det A) · (det B),
c d pode-se concluir que, sob essas condições:
1  d b a) se A é invertível, então A.B é invertível.
 .
ad  bc  c a  b) se B não é invertível, então A é invertível.
c) se A · B é invertível, então A é invertível e B não é invertível.
d) se A · B não é invertível, então A ou B não é invertível.
EXERCÍCIOS DE
e) se A · B é invertível, então B é invertível e A não é invertível.

FIXAÇÃO EXERCÍCIOS DE

A11 e A32.
 3 2 2

 1 1 4

01. Sendo a matriz A =  −3 0 1  , calcule o det A e os cofatores TREINAMENTO
 
1 −1 1
 
01. (EEAR) Para que o determinante da matriz 1 0 b  seja 3, o
 0 1 −1  1 1 −1 1 2
    valor de b deve ser igual a:
 1 
02. Sendo A =  3 2 1  e B =  5 0 −2  , calcule det (AB). a) 2 b) 0 c) –1 d) –2
0 1 5  3 2 1 
   
02. (EEAR) Sejam A = (aij) uma matriz real quadrada de ordem 2 e I2 a
1 0 2 matriz identidade também de ordem 2. Se “r1” e ” “r2” são as raízes
da equação det (A – r · I2 ) = n · r, onde n é um número inteiro positivo,
03. (EEAR) O valor do determinante −1 0 −2 é: podemos afirmar que:
2 3 4
a) r1 + r2 = a11 + a22 c) r1 · r2 = det A
a) – 2 b) 0 c) 1 d) 2 b) r1 + r2 = n(a11 + a22) d) r1 · r2 = –n · det A

x −1 x + 2 03. (EEAR) O elemento X3,2 da matriz solução da equação matricial


04. (EEAR) O número real x, tal que = 5 , é:
−3 x  1 1 10 4 
a) – 2 b) – 1 c) 0 d) 1 3 ⋅ X + 2 4  =  2 16 é:
 
6 8   0 8 
2 1 3
= 0 5 1 e B 2 3 a) 0 b) –2 c) 3 d) 1
05. (EEAR) Sejam as matrizes A =  0 9 . O valor de
 
3 2 1 04. (EEAR) Os valores de x que tornam verdadeira a igualdade
(det A) : (det B) é:
x 0 2
a) 4 c) –1 −1 −1 1 =−2 são tais que seu produto p é elemento do conjunto:
b) 3 d) – 2 3 1 x
a) {p ∈  / p > –3} c) {p ∈  / p < –6}
a b   −2a 2c 
06. (EEAR) Se as matrizes   e   tem determinantes b) {p ∈  / –3 < p ≤ 2} d) {p ∈  / –6 ≤ p < 2}
c d  −3b 3d
x
respectivamente iguais x e y, e ad ≠ bc, então o valor de é: 2 3 6
y
a) 2 c) – 6 05. (EEAR) Seja 4 x 0 = 64. O valor de x que torna verdadeira
b) 3 d) – 4 a igualdade é: − 2 0 − 2

a) 4 b) 5 c) –4 d) –5
1 1 1 
−1 −1 0 0
M  2 −3 x  . Se det M = ax² + bx + c,
07. (EEAR) Seja a matriz=
2 3 0 −1
então o valor de a é:  4 9 x²  06. (EEAR) Calculando o valor do determinante ,
−2 −1 0 0
a) 12 c) –5 obtém-se:
0 0 −1 1
b) 10 d) – 7
a) –3 b) –1 c) 1 d) 3

238

PM_BOOK16 - MAT.indb 238 25/11/2022 19:15:19


DETERMINANTES

07. (AFA) Sendo A = (aij) uma matriz de 2ª ordem, com aij = (–i)j+l – 3j2 e 15. Observe a matriz a seguir.
B a matriz dos cofatores dos elementos de A, o valor do determinante
sen x cos2 x 1
de A + B é:  
a) 680 b) 288 c) –288 d) –680 sen x cos x 0
sen x 1 1

1 2,4 9
Resolvendo seu determinante, será obtido o seguinte resultado:
08. (AFA) Se −0,5 0 k =10, então k é:
a) 1 b) sen x c) sen2 x d) sen3 x
−2 0,4 −1
a) menor que –4 83
c) igual a − 16. (AFA) Sejam A uma matriz quadrada de ordem 3, det A = d,
53 26 det(2A ⋅ At) = 4k, onde At é a matriz transposta de A, e d é a ordem
b) igual a −
22 5 da matriz quadrada B. Se det B = 2 e det 3B = 162, então o valor de
d) igual a −
2 k + d é:
a) 4 b) 8 c) 32 d) 36
09. (AFA) Sejam A, B e C matrizes reais 3 × 3 satisfazendo as seguintes
relações A · B = C–1, B = 2A. Se o determinante de C é 32, qual o valor
do módulo do determinante de A? 17. Seja A uma matriz quadrada de ordem 3, cujo determinante é
igual a 4, qual o valor de x na equação det (2 AAT) = 4x?
1 1 1 1
a) b) c) d) a) 4 b) 8 c) 16 d) 32 e) 64
16 8 4 2

 cos α − sen α 
  18. (AFA) Considere T ( α ) =  matriz quadrada definida
sen18° cos72°  sen α cos α 
10. (ESPCEX) Considere a matriz quadrada  .
  para todo α real. Sendo cof (T(α)) e det (T(α)), respectivamente, a
O valor do determinante de A é: sen36° cos54° 
matriz cofatora e o determinante da matriz T(α), é correto afirmar que:
a) -2 b) -1 c) 0 d) 1 e) 2
a) T(–α) = –T(α) c) T(–α) = (T(α))–1
b) cof T(α) = T(–α) d) det(T(2α)) = 4 det(T(α))
11. (ESPCEX) Para todo x e y reais, com x ≠ ±y, o quociente entre os
x+y x−y 0 0 3x 1
0 1 y 19. O número de raízes da equação 0 3x 2 = 0 é:
0 x x2 + y2 4 3x 3
determinantes é equivalente a:
x y a) 0 b) 1 c) 2 d) 3 e) 4
y x
 a b
x 2 − xy + y 2 x 2 − xy − y 2 x 2 − xy − y 2 20. (AFA) É dada a matriz A =   , onde a e b são números
a) c) e)  −b a
x−y x−y x+y
 0 1  a   5 
x 2 + xy + y 2 d)
x 2 + xy + y 2 reais. Se   .   =   , então o determinante de A vale:
b)  2 3   b   25 
x+y x−y
a) 2a² b) -2a² c) zero d) 2a + 2b
12. (ESPCEX) Sendo log2 3 1024 = a ; =b e
3 3 21. (AFA) O valor do determinante de uma matriz de ordem n é 21.
log70 log700 Se dividirmos a segunda linha desta matriz por 7 e multiplicarmos a
matriz por 3, o valor do novo determinante será:
log3(log5 125) = c, a ordem crescente desses números é:
a) 3n b) 3n+1 c) 3n d) 3n+3
a) a, b, c. c) c, b, a. e) c, a, b.
b) b, c, a. d) a, c, b. 12 18 9 32 60 14
22. (AFA) Sendo x = 21 17 15 e y = 63 51 45 , então:
13. (ESPCEX) Sendo {a,b} ∈ , a ≠ b e o determinante
32 60 14 12 18 9
a2 −4b b2 a) x = 3y
a 2 = a 128a − 128b , pode-se dizer que: b) x = –27y
b2 0 a2 c) y = –3x
a) a + b = 4 d) y = 27x
b) a + b = 8
c) a+b= 2 2 2 3 0 2 1 0
23. (AFA) Dados 0 2 1 = 9 , 0 1 1 = 5 e det A = –4, o valor de
d) a + b = 4 2
1 1 2 1 0 2
e) a + b = 2
x em A = é:
1 0 −1 0 1 x 0
14. (ESPCEX) O conjunto solução da inequação k 1 3 ≤ 0 é: 2 1 3 0
a) {k ∈  / -4 ≤ k ≤ 1} 1 k 3 0 1 2 1
b) {k ∈  / -1 ≤ k ≤ 4} 1 0 1 2
c) {k ∈  / k ≤ -1 ou k ≥ 4} 13 c) 1
a) −
d) {k ∈  / k ≤ -4 ou k ≥ 1} 5 d) 2
e) ∅ b) –1

239

PM_BOOK16 - MAT.indb 239 25/11/2022 19:15:21


DETERMINANTES

24. (AFA) Analise cada proposição a seguir classificando-a como 29. (AFA) Considere as seguintes simbologias em relação à matriz M:
VERDADEIRA ou FALSA. Mt é a matriz transposta de M
I. Sejam as matrizes A = (aij)3×n e B = (bjk)n×4 (n ≥ 1) então a matriz M-1 é a matriz inversa de M
4
det M é o determinante da matriz M
C = A·B é tal que o elemento c21 = ∑a .b
j =1
2j j1
Da equação (Xt)-1 = A·(B + C), em que A e (B + C) são matrizes
quadradas de ordem n e inversíveis, afirma-se que:

II. A e B são matrizes inversíveis de ordem n. Se A · Y · B = 2Bt, onde I. X= (A )


−1 t

−1 t
⋅ (B + C) 

Bt é a transposta de B, o determinante da inversa de A é igual a
1/4 e o determinante de B é igual a 1/2, então o determinante da 1
II. det X =
matriz Y é igual a 2n-2 det A ⋅ det (B + C)
III. X-1 = (Bt + Ct)·At
 então A =   , n ∈ *
n
III. Seja a matriz A = 
1 0  1 0 São corretas:
1 1 n 1
    a) apenas I e II
É correto afirmar que são verdadeiras: b) apenas II e III
a) todas as proposições. c) apenas I e III
b) apenas II e III. d) I, II e III
c) apenas I e II.
d) apenas I e III. 30. Se o determinante mostrado na figura é igual a zero, então 2x
pode ser:
1 1 1 1
25. (AFA) Sendo = 70 , o valor de :
2 3 4 a 4 3 2 a 1 2 + 2x 1 1
a) 280 0 0 2 0 2 0 0 0 1 1 3 − 2x 1
b) 0 3 −1 1 b 1 −1 3 b 1 1 1 1 − 2x
c) –70 −1 0 2 c 7 −1 0 b + 3c
a) 1/2 d) 4
d) –210 b) 1/4 e) 2
c) 1
26. Seja A = (aij) uma matriz quadrada de ordem 3 tal que,
p, se i = j 0, se i ≠ j
aij =  com p inteiro positivo. Em tais condições, é correto 
2p, se i ≠ j 31. Seja a matriz A = (aij)2 tal que a =  4 . O
i + j − j , se i =j
x 2 i j

afirmar que, necessariamente, det A é múltiplo de: determinante da inversa de A é: 


a) 2
1 1
b) 3 a) − d) −
4 2
c) 5
3 4
d) 7 b) e)
4 3
e) 11
3
c)
2
 1 2 3
27. Sejam A e B matrizes quadradas de ordem 3. Se=A 0 −1 1 a b c 
e B é tal que B-1 = 2A, o determinante de B será:  1 0 2 A =  
32. (EFOMM) Se o determinante da matriz d e f  é 5, então
a) 24 a a + b 3c g h i 
b) 6 d d + e 3f é igual a:
c) 3 g g + h 3i
d) 1/6 a) zero
e) 1/24 b) cinco
c) quinze
28. (AFA) Sejam a e b números positivos tais que o determinante d) trinta
1 0 0 −1
2 a e) quarenta e cinco
0 1 
da matriz  vale 24. Dessa forma o determinante da
 1 −1 b 1 2− x 1
   x
0 0 0 1 =
33. (EFOMM) Considere a matriz A  2 3x + 1 −1 , então o
 b 2  −4x + 1 2 0 
matriz   é igual a:
valor de f no ponto de abscissa 1, onde f(x) = det A é:
 3 a 
a) 18
a) 0
b) 21
b) 6
c) 36
c) –6
d) 81
d) 6
e) 270

240

PM_BOOK16 - MAT.indb 240 25/11/2022 19:15:22


DETERMINANTES

34. (EFOMM) Calcule o determinante da matriz A de ordem n: e  2 31/ 3 1


1 1 1 1 1 K 1   

1 3 1 1 1 K

1 
 2 3 4 5 6
 03. (EFOMM 2015) Sabendo-se que det  1 2 3 4 5   a,
1 1 5 1 1 K 1   
   0 1 3 5 12 
A=1 1 1 7 1 K 1   
1 1 1 1 9 K 1  3 1 2 0 4
   2e 2 8 241/ 3 2
M M M M M O 1   
  1 2 3 4 5
 1 1 1 1 1 K 2n − 1
calcule, em função de a, det  2 3 4 5 6 .
n −1
 

∏ 2n
a) 2a  0 1 3 5 12 
a) det(A) =  
b) –2a 3 0 5 5 16 
n =1
c) a
n

b) =
det(A) ∏ 2n − 1
n =1
d) –a
e) 3a
n −1

c) det(A) = ∏2
n =1
n 04. Seja A uma matriz quadrada inversível de ordem 4 tal que o
resultado da soma (A4 + 3A3) é uma matriz de elementos nulos. O
valor do determinante de A é:
n

d) det(A) = ∏2
n =1
n −1 a) −81
b) −27
c)
d) 27
−3 e) 81

e) det(A) = 1
05. (EFOMM 2011) Sejam A, B e C matrizes de ordem 3 × 3
 1 1
inversíveis tais que det A-1 = 3 e det   AB  I   4 . Sabendo-se
 2 
35. (EN) Uma função y = f(x) é definida pelo determinante da matriz
que I é a matriz identidade de ordem 3 tal que I = –3C-1 (2B-1 + A)T, o
 x 2 x − 1 x −2 
 3  determinante de C é igual a
x x x 1− x 
A= em cada x ∈  tal que A é invertível. É a) –8/3 c) –9 e) –288
1 0 0 0 
  b) –32/3 d) –54
 x 1 0 −1 
correto afirmar que o conjunto imagem de f é igual a: 06. (ESPCEX) As funções reais f e g são definidas pelos determinantes
a) (–∞, 4] d) (–∞, 4) que se seguem:
b) -{0, 4} e) [4, +∞) senx cos x senx 1
f( x )  e g( x ) 
c) (–∞, 4]–{0}  cos x senx 1 senx

2 5
Sendo h(x) = f(x) + g(x), então, o valor de h    h   é
 3   4 
EXERCÍCIOS DE

COMBATE
5 3+ 2
a) d)
4 2
1 3
b) e)
4 4
01. Quaisquer que sejam os números reais a, b e c, o determinante da 3− 2
c)
1 1 1 1  2
 
1 1  a 1 1 
matriz  vale: 07. (ESPCEX) Seja x um número real, I a matriz identidade de ordem
1 1 1 b 1 
  2 e A a matriz quadrada de ordem 2, cujos elementos são definidos
1 1 1 1 c  por aij = i – j.
a) ab + ac + bc d) abc + 1 Sobre a equação em x definida por det (A – xI) = x + detA é correto
b) abc e) 1 afirmar que
c) zero a) as raízes são 0 e 1/2.
b) todo x real satisfaz a equação.
1 2 1 0
c) apresenta apenas raízes inteiras.
0 2 2 4 
02. (EFOMM 2010) Sejam as matrizes A , d) uma raiz é nula e a outra negativa.
0 0 1 1
 1 2 3 7   e) apresenta apenas raízes negativas.
0 1 1 0 0 0 3
3
B e X = AB. O determinante da matriz 2X-1 é
08. (ESPCEX) Uma matriz quadrada A de ordem 3, é definida por
0 0 1 1
 
0 0 0 1 i  j, se i  j
aij   i j
. Então det(A-1) é igual a
igual a: ( 1) , se i  j
a) 1/6 c) 1 e) 6 a) 4 c) 0 e) 1/2
b) 1/3 d) 8/3 b) 1 d) 1/4

241

PM_BOOK16 - MAT.indb 241 25/11/2022 19:15:30


DETERMINANTES

09. (ESPCEX) O elemento da segunda linha e terceira coluna da matriz


 1 0 1
 
4 (ITA) Uma progressão aritmética (a1, a2, ..., an) satisfaz a
propriedade: para cada n ∈ , a soma da progressão é
igual a 2n² + 5n. Nessas condições, o determinante da matriz
inversa da matriz  2 1 0  é:
 0 1 1  a1 a2 a3 
   a
 4 a5 a6  é:
a) 2/3 c) 0 e) –1/3
a7 + 2 a8 a9 
b) 3/2 d) –2
a) –96 d) 99
10. (EN 2011) Sejam A e B matrizes quadradas de ordem n, cujos b) –85 e) 115
determinantes são diferentes de zero. Nas proposições abaixo,
c) 63
coloque (V) na coluna à esquerda quando a proposição for verdadeira
e (F) quando for falsa.
(
(
) det (–A) = (–1)n det A, onde –A é a matriz oposta de A.
) det A = –det At, onde At é a matriz transposta de A.
5 (IME) Sejam x1, x2, x3 e x4 os quatro primeiros termos de
uma P.A. com x1 = x e razão r, com x, r ∈ . O determinante
 x1 x1 x1 x1 
( ) det A-1 = (det A)-1, onde A-1 é a matriz inversa de A. x x x x 
de 
1 2 2 2
é:
( ) det (3A · B) = 3 · det A · det B.  x1 x 2 x 3 x 3 
 
( ) det (A + B) = det A + det B.  x1 x 2 x 3 x 4 
Lendo a coluna da esquerda, de cima para baixo, encontra-se: a) 0 d) x · r4
a) (V) (F) (V) (F) (F). b) x4 · r e) x · r³
b) (F) (F) (F) (V) (F). c) x4 · r3
c) (F) (V) (F) (V) (V).
d) (V) (V) (V) (F) (F).
e) (V) (F) (V) (F) (V). GABARITO
EXERCÍCIOS DE FIXAÇÃO
01. det A = 17, A11 = –1 e A32 = –9 06. C
02. det (AB) = 306 07. C

DESAFIO PRO 03. B


04. C
08. C
09. D
05. D 10. D
1 x x² x³  EXERCÍCIOS DE TREINAMENTO
1 2 3 4 
1 (ITA) Considere a matriz  ,x ∈ . Se o 01. B 10. C 19. A 28. D
 −1 3 4 5
  02. C 11. A 20. A 29. D
 −2 2 1 1
03. A 12. C 21. B 30. C
polinômio p(x) é dado por p(x) = det A, então o produto 04. D 13. A 22. C 31. A
das raízes de p(x) é:
05. B 14. D 23. C 32. C
1 1 1
a) . c) . e) . 06. B 15. D 24. B 33. B
2 5 11
1 1 07. A 16. D 25. D 34. A
b) . d) .
3 7 08. D 17. D 26. C 35. C
09. A 18. C 27. E 36. D

2 (ITA) Considere as seguintes afirmações a respeito de


matrizes A de ordem n × n inversíveis, tais que os seus
elementos e os de sua inversa sejam todos números inteiros:
EXERCÍCIOS DE COMBATE
01. B 04. E 07. C 10. A
02. D 05. E 08. D
I. |det(A)| = 1
03. B 06. A 09. A
II. AT = A-1
DESAFIO PRO
III. A + A-1 é uma matriz diagonal
01. D 03. E 05. E
É(são) sempre VERDADEIRA(S):
02. A 04. A
a) apenas I. d) apenas I e III.
b) apenas III. e) todas. ANOTAÇÕES
c) apenas I e II.

3 (IME) Calcule o valor do determinante:


4 2 1
log 81 log 900 log 300
(log 9)2 2 + 4 log 3 + 2 (log 3)2 (log 3 + 2)2

a) 1 c) 4 e) 16
b) 2 d) 8

242

PM_BOOK16 - MAT.indb 242 25/11/2022 19:15:32


SISTEMA LINEAR

SISTEMA LINEAR DETERMINADO UMA ÚNICA


Um sistema linear de m equações a n incógnitas é um conjunto (S.P.D.) SOLUÇÃO
SISTEMA POSSÍVEL
de m (m ≥ 1) equações lineares a n incógnitas e pode ser escrito como
OU COMPATÍVEL
segue: INDETERMINADO INFINITAS
(S.P.I.) SOLUÇÕES
a11x1  a12x 2  …  a1nxn  b1
a x  a x  …  a x  b SISTEMA IMPOSSÍVEL NENHUMA
 21 1 22 2 2n n 2
 OU INCOMPATÍVEL (S.I.) SOLUÇÃO

am1x1  am2x 2  …  amnxn  bm
Alguns exemplos simples são apresentados abaixo:
onde x1, x2, ..., xn são as incógnitas. x  y  3
O sistema acima pode ser escrito na forma matricial. . .D  
SP  S  {(2,1)}
x  y  1
 a11 a12  a1n   x1   b1 
x  y  3
a
 21 a22  a2n   x 2   b2  . .I  
SP  S  {( t, 3  t )t   }  infinitas
infinitas soluı es
soluções
  2x  2y  6
        
     
am1 am2  amn   xn  bm  x  y  3
SI..   S
2x  2y  7
A matriz dos coeficientes das equações é chamada matriz
incompleta do sistema.

 a11 a12  a1n  REGRA DE CRAMER


a a22  a2n  Seja um sistema linear de n equações e n incógnitas e A a
A   21
      sua matriz incompleta. Se det A ≠ 0, então o sistema é possível e
  determinado e a solução é tal que
am1 am2  amn 
det Ai
xi = para i = 1, 2, ..., n
A matriz das incógnitas é uma matriz coluna formada pelas det A
incógnitas do sistema.
onde Ai é a matriz obtida de A, substituindo-se a i-ésima coluna
 x1  pela coluna dos termos independentes. Nesse caso, diz-se que o
x  sistema é um Sistema de Cramer.
X   2 a11 a12  a1n

  a21 a22  a2n
 xn  det A = ≠ 0 ⇒ sistema de Cramer
   
A matriz dos termos independentes é uma matriz-coluna am1 am2  amn
formada pelas constantes do 2º membro.
Vamos aplicar o Teorema de Cramer na resolução do sistema
 b1  abaixo:
b 
C  2 x  y  z  6

1 1 1
  2x  y  z  1  det A  2 1 1  10  0
  3x  y  z  4
bm   3 1 1

⇒ sist. de Cramer ⇒ sistema possível e determinado


CLASSIFICAÇÃO DE UM
6 1 1 1 6 1
SISTEMA LINEAR det A x  1 1 1  10 det A y  2 1 1  20
A solução de um sistema linear de m equações e n incógnitas 4 1 1 3 4 1
é uma ênupla ordenada (α1, α2, ..., αn) que satisfaz cada uma das
m equações, onde na posição i aparece o valor a ser atribuído à 1 1 6
incógnita xi. det A z  2 1 1  30
De acordo com a quantidade de soluções, o sistema linear pode 3 1 4
ser classificado como segue:

243

PM_BOOK16 - MAT.indb 243 25/11/2022 19:15:41


SISTEMA LINEAR

det A x 10 det A y 20 det A z 30


x  1 y  2 z   3  S  {(1, 2, 3)}
det A 10 det A 10 det A 10
A Regra de Cramer permite identificar os sistemas possíveis e determinados e obter a sua solução.
Entretanto quando det A = 0, o sistema não é de Cramer, podendo ser possível e indeterminado ou impossível. Nos nossos concursos,
encontraremos no geral sistemas 3 × 3. Nestes casos, a abordagem será eliminar uma das variáveis, para reduzir a um sistema 2 × 2, onde a análise
será bem mais simples.

PROPRIEDADES DOS SISTEMAS EQUIVALENTES


P1: Trocando de posição as equações de um sistema, obtemos um outro sistema equivalente.
P2: Multiplicando uma ou mais equações de um sistema por um número k, k ∈ *, obtemos um sistema equivalente ao anterior.
P3: Adicionando a uma das equações de um sistema o produto de outra equação desse mesmo sistema por um número k, k ∈ *, obtemos
um sistema equivalente ao anterior.
Sistemas escalonados: a técnica de escalonar um sistema linear é muito mais utilizada, pois com essa técnica podemos encontrar soluções
para sistemas que não tenham o mesmo número de equações e incógnitas (o que não é permitido na Regra de Cramer). Além disso, quando
queremos resolver sistemas lineares cujo número de equações (e de incógnitas) excede três, não é conveniente utilizar a Regra de Cramer, por se
tornar muito trabalhosa. Por exemplo, um sistema com quatro equações e quatro incógnitas requer o cálculo de cinco determinantes de 4ª ordem.
Neste caso, usamos a técnica de escalonamento, que facilita a resolução e a discussão de um sistema.
Dado um sistema linear:
a11x1 + a12 x2 + a13 x3 +  + a1n x n =
b1
a x + a x + a x +  + a x = b2
 21 1 22 2 23 3 2n n
S=

am1x1 + am2 x2 + am3 x3 +  + amn x n = bm
Onde existe pelo menos um coeficiente não-nulo em cada equação, dizemos que S está escalonado se o número de coeficientes nulos antes
do primeiro coeficiente não nulo aumenta de equação para equação.

PROCEDIMENTOS PARA ESCALONAR UM SISTEMA


1) Fixamos como 1ª equação uma das que possuam o coeficiente da 1ª incógnita diferente de zero.
2) Utilizando as propriedades de sistemas equivalentes, anulamos todos os coeficientes da 1ª incógnita das demais equações.
3) Anulamos todos os coeficientes da 2ª incógnita a partir da 3ª equação.
4) Repetimos o processo com as demais incógnitas, até que o sistema se torne escalonado.
Exemplo 1:
2x − y + z =5
Vamos escalonar o sistema 
3x + 2y − 4z =0 . Para facilitar trocamos a 1ª e 3ª equação, pois esta possui coeficiente de x igual a 1.
x − 2y + z =2


=
 x − 2y + z 2 = x − 2y + z 2 = x − 2y + z 2  6 − 14 −8
   2ª Equação : −8y + 7(2) = 6 ⇒ y = = =1
3x + 2y − 4z =0 ⇒ L 2 =3.L1 − L 2  − 8y + 7z =6 ⇒ ⇒  − 8y + 7z =6 ⇒ −8 −8
2x − y + z = 5 L3 =2.L1 − L3  − 3y + z =−1 3  13z 26 3ª Equação : x= 2(1) − (2) + 2= 2
 L3 = .L2 − L3  = ⇒ z= 2
8  8 8
Portanto, o sistema é possível e determinado, admitindo uma única solução que é dada por: (x, y, z) = (2, 1, 2).

Exemplo 2:
x − 2y + z =3
Vamos escalonar o sistema 2x + y + z =
1 .
3x − y + 2z = 2

=
x − 2y + z 3 = x − 2y + z 3 =x − 2y + z 3
  
2x + y + z = 1 ⇒ L2 = 2.L1 − L2  − 5y + z = 5 ⇒  − 5y + z = 5
3x − y + 2z =2 L =3.L − L  − 5y + z =7 L =L − L  0z =−2
 3 1 3 3 2 3

Dessa forma fica escalonado. Como não existe valor real de z, tal que 0 . z = –2 , o sistema é impossível e, portanto, não tem solução.

Exemplo 3:
x + y + z =6
Vamos escalonar o sistema 2x + y − 2z =−1.
3x + 2y − z =
5

=
x + y + z 6 = x + y + z 6 = x + y + z 6
  
2x + y − 2z = −1 ⇒ L2 = 2.L1 − L2  y + 4z = 13 ⇒  y + 4z =13
3x + 2y − z = 5 
L3 = 3.L1 − L3  y + 4z = 13 L3 = L2 − L3  0z = 0

244

PM_BOOK16 - MAT.indb 244 25/11/2022 19:15:42


SISTEMA LINEAR

O sistema está escalonado. Entretanto, o número de equações (m) Exercício Resolvido


é menor que o número de incógnitas (n). Assim, o sistema é possível
e indeterminado, admitindo infinitas soluções. 03. Dois casais foram a um barzinho. O primeiro pagou R$ 5,40
Fazendo Z = α e substituindo esse valor na 2ª equação, obtemos: por 2 latas de refrigerantes e uma porção de batatas fritas. O
y + 4α = 13 ⇒ y = 13 – 4α segundo pagou R$ 9,60 por 3 latas de refrigerantes e 2 porções de
batatas fritas. A diferença, em reais, entre o preço de uma porção
Substituímos esses valores na 1ª equação x = 6 –13 + 4α – α ⇒
de fritas e de uma lata de refrigerante nesse bar é
x = – 7 + 3α:
a) R$ 1,50
Assim, a solução do sistema é dada por: S = {( –7 + 3α, 13 – 4α,
α)}, sendo α ∈ . b) R$ 1,60
Para cada valor que seja atribuído a α, encontraremos uma c) R$ 1,70
quádrupla que é solução para o sistema. d) R$ 1,80
Exemplo:
Resolução: D
α = 1 ⇒ S = {( – 7 + 3(1), 13 – 4(1), 1)} = {(–4, 9, 1)}
Formando o sistema relacionando “x” à lata de refrigerante e “y”
à porção de batatas fritas utilizamos as informações dos preços e
Exercício Resolvido
escalonamos:
01. Um jogador de basquete fez o seguinte acordo com o seu 2 x + y = 5,40 2 x + y = 5,40
clube: cada vez que ele convertesse um arremesso, receberia  ⇒ BL1 – 2L2 
3 x + 2 y =9,60  − y =− 3,00 ⇒ y =3,00
R$ 10,00 do clube e, caso errasse, pagaria R$ 5,00 ao clube. Ao final
de uma partida em que arremessou 20 vezes, recebeu a quantia de
Logo, 2x = 5,40 – 3,00 = 2,40. Então x = 1,20. A diferença entre
R$ 50,00. A quantidade de arremessos que ele acertou foi
a porção de fritas e a lata de refrigerante é: R$ 3,00 – R$1,20 =
a) 6 R$ 1,80.
b) 7
c) 8
d) 9 Exercício Resolvido
e) 10
04. Os valores de x, y, e z que satisfazem ao sistema:
Resolução: E  x y z
3 .3 .3 = 1
Será considerado que receber é um valor positivo para o jogador e  x
 2 são, respectivamente
pagar, negativo. Utilizando a variável “x” para o número de acertos  y z =4
e “y” para o número de erros, o sistema que resolve a questão é:  2 .2
 −x y z 1
x + y = 20 4 .16 .4 = 4

10 x − 5 y =50
a) (1, 1, –2)
=
 x + y 20 = x + y 20 b) (1, 1, 1)
 ⇒
10 x − 5 y= 50 15 y= 150 ⇒ y= 10 c) (–1, 1, –2)
d) (–2, 1, 1)
e) (–1, –1, –1)
Exercício Resolvido
Resolução: A
02. Uma loja de departamentos, para vender um televisor, um DVD Utilizando as propriedades das potências reescrevemos o sistema
e um aparelho de som, propôs a seguinte oferta: o televisor e o como potências de mesma base:
DVD juntos custam R$ 1.200,00; o DVD e o som juntos custam
R$ 1.100,00 e o televisor com o som custam juntos R$ 1.500,00.  x y z
3 .3 .3 = 1 3x + y + z = 30
Um cliente que comprar os três produtos pagará  x
 2  x − y −z
a) R$ 1.800,00  y z = 4 ⇒ 2 .2 .2 = 22
b) R$ 1.900,00  2 .2  −2 x 4 y 2 z −2
 −x y z 1 2 .2 .2 = 2
c) R$ 2.000,00 4 .16 .4 = 4
d) R$ 2.100,00
Igualando os expoentes e escalonando, vem:
e) R$ 2.200,00
=
x + y + z 0 =
L1 – L 2  x + y + z 0
 
Resolução: B x − y − z = 2 ⇒ 2 y + 2z =−2 ⇒
Formando o sistema relacionando “x” à TV, “y” à DVD e “z” ao −2 x + 4 y + 2z = −2 2L1 + L 3 6 y + 4 z =−2

Som utilizamos as informações dos preços e escalonamos:
x + y + z =0
= x + y 1200 =  x + y 1200 
  3L 2 − L 3 2 y + 2z =−2
 y +=z 1100 ⇒ L1 − L3  y +=z 1100 ⇒ 2z =−4 ⇒ z =−2
  
x + z =1500  y − z =−300
Substituindo “z” e calculando os outros valores, temos:
x + y =1200
  2
L2 − L3  y + z =
1100 2 y + 2( −2) =−2 ⇒ y = =1
2z = 1400 z =−2 ⇒  2

 x + 1+ ( −2) = 0 ⇒ x = 1

245

PM_BOOK16 - MAT.indb 245 25/11/2022 19:15:42


SISTEMA LINEAR

kx + 4ky =0
Exercício Resolvido 05. Relativas ao sistema  ,k ∈ , considere as afirmações
I, II e III abaixo. 3x + ky =8
 x + 3y =m I. Apresenta solução única para, exatamente, dois valores distintos de k.
05. O sistema linear abaixo, nas incógnitas x e y:  .
Será impossível quando: 2x − py =
2 II. Apresenta mais de 1 solução para um único valor de k.
a) nunca. III. É impossível para um único valor de k.
b) p ≠ –6 e m = 1. Dessa forma,
c) p ≠ –6 e m ≠ 1. a) somente I está correta.
d) p = –6 e m = 1. b) somente II e III estão corretas.
e) p = –6 e m ≠ 1. c) somente I e III estão corretas.
d) somente III está correta.
Resolução: E e) I, II e III estão corretas.
1 3
Se D = 0 ⇔ SPI ou SI, então = 0 ⇔ −p − 6 = 0 ⇔ p = −6.
2 −p 06. Uma geladeira é vendida em n parcelas iguais, sem juros. Caso se
 x + 3y = m queira adquirir o produto, pagando-se 3 ou 5 parcelas a menos, ainda
Fazendo p = –6, temos:  . Resolvendo temos sem juros, o valor de cada parcela deve ser acrescido de R$ 60,00
2 x + 6 y =
2
ou de R$ 125,00, respectivamente. Com base nessas informações,
0 = –2m + 2.
Logo, o sistema será SI quando – 2m + 2 for diferente de zero, ou conclui-se que o valor de n é igual a
seja, quando m ≠ 1. a) 13 c) 15 e) 17
b) 14 d) 16

07. Considere três números naturais a, b e c, nessa ordem. A soma


EXERCÍCIOS DE desses números é 888, a diferença entre o primeiro e o segundo é

FIXAÇÃO igual ao terceiro. O terceiro deles excede o segundo em 198.


O valor da diferença entre o primeiro e o terceiro é tal que excede
90 em
a) 23 b) 33 c) 43 d) 53
−x + 2 y − 2 z = −b
01. A terna (1, 2, 3) é solução do sistema 5x − y − az =−3b . Então,
2x − 2y − 2z − 2 = 0
o valor de a é −6x + y + z = b − a
 08. O sistema  2x + y + 3z = 6 é possível e determinado, quando
a) –4 d) 3  kx + y + 5z =
 9
b) –3 e) 4 o valor de k for:
c) –2 a) k ≠ 3. c) k = 3. e) k = 0.
b) k = 5. d) k ≠ 5.
 x + y +z= 1

02. Se x, y e z constitui a solução do sistema linear  x + 2y + 3z =
−2 5x + 3y + 4z =3
então o produto x ⋅ y ⋅ z é igual a x + 4y + 5z =−4 
 09. Considerando o sistema 15x + 9y + 8z =6 analise as
a) –4. c) –2. afirmativas abaixo e conclua. 20x + 12y + 16z =
 12
b) –8. d) –6. a) O sistema é impossível.
b) O sistema é possível e indeterminado.
 x + y + kz =1
c) O sistema é possível e determinado.

03. Seja o sistema linear nas incógnitas x, y e z  2x + k 2z =−1 .
x + y + 2z = d) O sistema admite como solução única x = 4, y = 8, z = –11.
Assinale a afirmativa correta: 0
 e) O sistema admite como solução, para qualquer valor de x a terna
a) para k = 1, possui mais de uma solução.
(x, x, 5x).
b) para k = 3, não possui solução.
c) para k = 2, possui infinitas soluções. 10. Uma empresa deve enlatar uma mistura de amendoim, castanha
d) para k = 2, não possui solução. de caju e castanha-do-pará. Sabe-se que o quilo de amendoim custa
R$ 5,00, o quilo de castanha de caju, R$ 20,00 e o quilo de castanha-
e) para k = 2, possui uma única solução.
do-pará, R$ 16,00. Cada lata deve conter meio quilo da mistura e o
custo total dos ingredientes de cada lata deve ser de R$5,75. Além
04. Um técnico em edificações realizou um trabalho em y dias, disso, a quantidade de castanha de caju em cada lata deve ser igual
fazendo x horas por dia. Se trabalhasse duas horas a mais por dia, a um terço da soma das outras duas. Nesse caso, as quantidades de
teria terminado o serviço dois dias antes, e se trabalhasse quatro horas cada ingrediente por lata são
a mais por dia, teria terminado o serviço três dias antes. Os valores de
x e y são tais que a) 270 g de amendoim, 125 g de castanha de caju e 105 g de
castanha-do-pará.
a) y é o dobro de x.
b) 270 g de amendoim, 172,5 g de castanha de caju e 57,5 g de
b) y é 1,5 de x. castanha-do-pará.
c) a sua diferença é 3. c) 250 g de amendoim, 125 g de castanha de caju e 125 g de
d) a sua soma é 9. castanha-do-pará.
e) x é a terça parte de y. d) 228 g de amendoim, 100 g de castanha de caju e 72 g de
castanha-do-pará.

246

PM_BOOK16 - MAT.indb 246 25/11/2022 19:15:43


SISTEMA LINEAR

 −x − 2y + 3z =0
EXERCÍCIOS DE 
06. (ESPCEX) O valor de m, para que o sistema  2x + y − 4z = 0

TREINAMENTO admita soluções além da solução trivial, é:

a) 1 c) 5

 4x +

e) 9
my − 10z = 0

b) 3 d) 7
01. (EFOMM) Na Escola de Marinha Mercante, há alunos de ambos os
sexos (130 mulheres e 370 homens), divididos entre os Cursos Básico, 07. (ESPCEX) A soma dos valores de x, y e z que tornam o sistema
de Máquinas e de Náutica. Sabe-se que do total de 130 alunos do  2x + y − z = 5
Curso de Máquinas, 20 são mulheres. O Curso de Náutica tem 270 
3x − 2y + z = −2 verdadeiro é:
alunos no total e o Curso Básico tem o mesmo número de homens e 
mulheres. Quantas mulheres há no Curso de Náutica?  x + z = 0
a) 50 c) 60 e) 70 a) 1 c) 2 e) 4
b) 55 d) 65 b) 3 d) 5

02. (AFA) O conjunto de soluções de uma única equação linear a1x + a²x + y =1
a2y + a3z = b é representado por um plano no sistema de coordenadas 08. (ESPCEX) Dado o sistema linear  , onde a é uma
constante real, pode-se afirmar que:  x + y =a
retangulares xyz (quando a1, a2, a3 não são todos iguais a zero). Analise
as figuras a seguir. a) o sistema é possível e determinado para a = –1.
(I) Três planos se (II) Três planos se (III) Três planos b) existe um único valor de a que torna o sistema possível e
cortando numa reta cortando num ponto sem interseção indeterminado.
c) o sistema é possível e determinado somente se a ≠ 1.
d) o sistema é possível e determinado ∀x ∈ .
e) o sistema é impossível ∀x ∈ .

 x−y+z= 8

09. (ESPCEX) A soma das soluções do sistema  2x + y + z =5 é:
x + 2y − z =−8
Assinale a opção verdadeira. 
a) A figura I representa um sistema de três equações com uma única a) 4 c) 6 e) 8
solução. b) 5 d) 7
b) A figura III representa um sistema de três equações cujo conjunto
solução é vazio. 2x − y + 3z =−1

c) A figura II representa um sistema de três equações com uma 10. O sistema de equações: x + 2y + 3z =1
infinidade de soluções. 4x − 7y + 3z = −5

d) As figuras I e III representam um sistema de três equações com a) Não possui solução.
soluções iguais. b) Possui uma infinidade de soluções.
c) Possui um número finito, maior que um soluções.
 x+y+z= 1
 d) Possui uma única solução, na qual o valor de z é positivo.
03. (ESPCEX) Sabendo que (x, y, z) é solução do sistema  x − y + 2z =3 ,
o valor de x² + y² + z² é: 2x + 3y − z = e) Possui uma única solução, na qual o valor de z é negativo.
 1
a) 5 c) 7 e) 10 11. (ESPCEX) Num curso de Matemática, cada bimestre teve três
b) 6 d) 9 provas. As questões valiam um ponto cada uma, mas os pesos das
provas eram diferentes. Alves, que acertou 6 questões na primeira
prova, 5 na segunda e 6 na terceira, obteve, no final, um total de 57
04. (AFA) Sr. Osvaldo possui certa quantia com a qual deseja adquirir
pontos. Tadeu acertou 3, 6 e 6 questões, respectivamente na 1ª, 2ª
um eletrodoméstico. Caso a loja ofereça um desconto de 40%,
e 3ª provas, totalizando 54 pontos. Por sua vez, João acertou 2, 7 e
ainda lhe faltarão 1000 reais. Se o Sr. Osvaldo aplicar sua quantia a
3 questões, respectivamente na 1ª, 2ª e 3ª provas, atingindo a soma
juros (simples) de 50% ao mês, ajunta, em três meses, o montante
de 40 pontos no final. Sabendo que Xavier fez 5 questões certas na
correspondente ao valor do eletrodoméstico sem o desconto.
primeira prova, 8 na segunda e 3 na terceira, o total de pontos de
Assim, o valor do eletrodoméstico e da quantia que o Sr. Osvaldo Xavier foi
possui somam, em reais,
a) 49 c) 51 e) 53
a) 4000 b) 5000 c) 7000 d) 8000
b) 50 d) 52

x − 3y + kz =0
 kx + 2y + 2z =5
05. (AFA) Considere o sistema linear homogêneo 3x + ky + z =0, 
kx + y = 12. (ESPCEX) Os valores de K para que o sistema linear  2x + ky + z =3
onde k é um número real. 0
 seja possível e tenha uma única solução são  2x + 3y + z =8

O único valor que torna o sistema, acima, possível e indeterminado, a) k =  – {–1,2}
pertence ao intervalo b) k =  – {–2,2}
a) (-4,-2] d) (2,4] c) k =  – {1,2}
b) (-2,1] e) (4,6] d) k =  – {3,4}
c) (1,2] e) k =  – {1,–2}

247

PM_BOOK16 - MAT.indb 247 25/11/2022 19:15:44


SISTEMA LINEAR

13. (ESPCEX) José e Maria, acompanhados de seu filho Pedro, Como o movimento da noite de segunda-feira estava fraco,
queriam se pesar. Para tanto, utilizaram uma balança defeituosa que o proprietário resolveu manter os preços individuais de cada
só indicava corretamente pesos superiores a 60 kg. Desta forma, eles componente da oferta para quaisquer combinações de pedidos dos
se pesaram, dois a dois, e obtiveram os seguintes resultados: produtos citados.
• José e Pedro: 87 kg Assim, as famílias A, B e C pagaram juntas 56 reais pelos produtos
• José e Maria: 123 kg consumidos, conforme o quadro abaixo:
• Maria e Pedro: 66 kg Quantidade Porção de Refrigerante
X-bacana
Diante desses resultados, pode-se concluir que Família fritas em lata
a) cada um deles pesa menos que 60 kg. A 5 4 4
b) dois deles pesam mais que 60 kg. B 3 0 2
c) José é mais pesado que Maria e Pedro juntos. C 1 2 2
d) Maria é a mais pesada dos três.
Sabendo-se que a família A gastou 3 reais a mais que o dobro do valor
e) o peso de Maria é a média aritmética dos pesos de José e Pedro.
gasto pela família B e que a família C gastou 3 reais a menos que a
família B, é INCORRETO afirmar que
2x + 3y − z = 3
 a) 6 refrigerantes em lata custam tanto quanto 10 porções de
14. O valor de a tal que no sistema x − y + az = 1 se tenha z = 3 é: batatas fritas.
z + y + z =5
 b) a família B gastou o equivalente a 30% das despesas das famílias
a) −2 c) 0 e) 2 A e C juntas.
b) −1 d) 1 c) o preço y da promoção sugerida não ultrapassa R$ 7,50.
d) a família B poderia ter optado por pedir duas promoções e sua
15. (EFOMM) Dado o sistema linear abaixo, analise as seguintes despesa seria a mesma.
afirmativas:
3 4 −6  x   −3
0 16 b  ⋅  y  =    x + y + az =1
    a 
19. (AFA) O sistema  x + 2y + z =2 é indeterminado para
 1 −4 2   z   3  2x + 5y − 3z =
 b
I. Se b ≠ –12, o sistema linear terá uma única solução. a) a ≠ 6 e b = 5 c) a = 6 e b ≠ 5
II. Se a = b = –12, o sistema linear terá infinitas soluções. b) a = 6 e b = 5 d) a ≠ 6 e b ≠ 5
III. Se b = –12, o sistema será impossível.
1 1 1   x1  k 
a) Todas as afirmativas são corretas.
20. (AFA) Sejam as matrizes A = 1 1 2  , X =  x 2  e B = 3 . Em
b) Todas as afirmativas são incorretas.  
1 1 −2  x 3  5
c) Somente as afirmativas I e III são corretas. relação à equação matricial AX = B, é correto afirmar que
d) Somente as afirmativas I e II são corretas. 7
e) Somente as afirmativas II e III são corretas. a) é impossível para k = .
2
7
b) admite solução única para k = .
x + 4z = −7 2
 c) toda solução satisfaz à condição x1 + x2 = 4.
16. Se x − 3y = −8 , então x + y +z é igual a:
y + z =1  1
 d) admite a terna ordenada  2, 1, −  como solução.
a) −2 c) 0 e) 2  2
b) −1 d) 1
21. (ESPCEX) Os números das contas bancárias ou dos registros
de identidade costumam ser seguidos por um ou dois dígitos,
17. (AFA) Três amigos Samuel, Vitória e Júlia, foram a uma lanchonete.
denominados dígitos verificadores, que servem para conferir sua
• Samuel tomou 1 guaraná, comeu 2 esfirras e pagou 5 reais. validade e prevenir erros de digitação.
• Vitória tomou 2 guaranás, comeu 1 esfirra e pagou 4 reais. Em um grande banco, os números de todas as contas são formados
• Júlia tomou 2 guaranás, comeu 2 esfirras e pagou k reais. por algarismos de 0 a 9, na forma abcdef – xy, em que a sequência
Considerando-se que cada um dos três pagou o valor exato do que (abcdef) representa, nessa ordem, os algarismos do número da conta
consumiu, é correto afirmar que e x e y, nessa ordem, representam os dígitos verificadores.
a) o guaraná custou o dobro da esfirra. Para obter os dígitos x e y, o sistema de processamento de dados do
banco constrói as seguintes matrizes:
b) os três amigos, juntos, consumiram 16 reais.
 1 −2 1  x  (a − b) 
c) cada esfirra custou 2 reais.
A = 0 1 0  B =  y  =C (c − d) 
d) Júlia pagou 8 reais pelo que consumiu.
0 2 −1  z   (e − f) 
18. (AFA) Um trailer de sanduíches anunciou para a segunda-feira, a Os valores de x e y são obtidos pelo resultado da operação matricial
seguinte promoção: A · B = C, desprezando-se o valor de z. Assim, os dígitos verificadores
correspondentes à conta corrente de número 356281 são
“Saboreie: 1 X-bacana, 1 porção de a) 34 d) 51
batatas fritas, 1 refrigerante em lata
b) 41 e) 54
e pague apenas y reais.”
c) 49

248

PM_BOOK16 - MAT.indb 248 25/11/2022 19:15:45


SISTEMA LINEAR

x − y x − y x − y x − y
 − + − + =
−1 28. (AFA) Analise as proposições, classificando-as em VERDADEIRA(S)
22. (AFA) A solução do sistema  2 6 18 54
é tal que x + y é igual a 3x − y =−2 ou FALSA(S).
 x+y= 0
a)
11
b)
10
c) −
7
d) −
8 I. O sistema linear  x + z =0 é indeterminado para m = –1 e uma
3 3 3 3 y + mz =
 0
de suas soluções é a terna ordenada (–1,1,1).
 1 1 − 1   x  5
23. A equação matricial  −1 1 1   y  =2 .
  II.
(m + 1)x + 7y =
Para que o sistema 
10
seja impossível deve-se ter
 1 3 − 1 z  k   4x + (m − 2)y =0
m = –5, somente.
a) é impossível para todos os valores de k.
b) admite solução qualquer que seja k. x − 1 y + 2   1 −1  3 0
III. Na equação matricial  ⋅ = a
 z x + y + z  0 1   −2 5 
c) admite solução somente se k = 4.
soma x + y + z é igual a 3.
d) admite solução somente se k = 8.
Tem-se a sequência correta:
e) admite solução somente se k = 12.
a) V, V, F b) F, V, F c) V, F, V d) F, F, V
24. (AFA) A condição que deve ser satisfeita pelos termos
independentes a, b e c (a, b e c ∈ ℝ*) para que seja compatível o 8x − y − 2z =0
29. (AFA) (x, y, z) são as soluções do sistema  . Se x, y
 x + 2y − z =a 7x + y − 3z =0

sistema  y + 2z = b é estabelecida por. e z formam, nesta ordem, uma progressão aritmética, então a razão
x + 3y + z = c dessa progressão aritmética é igual a

1 3 x+y+z
a) c – a + b = 0 a) b) c) x d)
3 2 3
b) a + b + c = 0
c) c+a–b=0 1 2 1
 a2 + b2 + c2 =9
d) a + b – c = 0 
2 1 1
30. (AFA) Considere o sistema abaixo  2 + 2 − 2 = 3 . Sabendo-
 a b c
x + 2y + 3z = 0
25. O conjunto de todas as soluções do sistema  . 3 1 2
 4x + 5y + 6z = 0  a2 − b2 − c 2 =
−4

a) é vazio. se que a, b e c são números reais não nulos, é INCORRETO afirmar que
b) consiste apenas no vetor nulo (0,0,0). a) | a | + | b | + | c |∈ (  − )
c) consiste apenas no vetor (1,–2,1). b) a² + b² + c² > 2.
d) consiste em todos os múltiplos {(a,-2a,a)} de (1,–2,1). a2 1 3
e) consiste em todos os múltiplos {(a,a,–2a)} de (1,1,–2).  2  1
c) O determinante da matriz  0 b 4  é igual a .
0 0 2  6
 c 
x + y + z =1 1 1 1
26. Assinale a afirmativa correta. O sistema  . d) + 2 + 2 é par.
 x + y − z =
1 a2
b c
a) não tem solução. x + 3y − 4z = 0
b) tem uma solução única x = 1, y = 0, z = 0. 
31. (AFA) Seja o sistema de equações  3x + y = a em que a e b
c) tem exatamente duas soluções.  4x + bz = 0

d) tem uma infinidade de soluções. são números reais. É correto afirmar que
e) tem uma solução com z = 1. a) se a = 0, existe b tal que S é impossível.
1 3 −4
27. (AFA) A quantidades dos produtos que Elaine, Pedro e Carla b) se b é tal que 0 3 1 0 ≠ 0, o sistema terá uma única solução,
comparam num mercado estão esquematizadas na tabela que segue.
4 0 b
Produto A Produto B Produto C qualquer que seja o valor de a.
Elaine 1 2 3 c) se b = 1 e a = 1, o sistema tem mais de uma solução.

Pedro 3 6 2 d) se a = 0, o sistema possui somente a solução trivial.

Carla 2 4 1 32. (AFA) Um suspeito de assaltar dois caixas de um supermercado foi


intimado a prestar depoimento e fez a seguinte declaração:
Sabendo-se que Pedro gastou R$ 21,00 e Carla R$ 13,00, pode-se
concluir, necessariamente, que. “No primeiro caixa foram roubados dois pacotes de notas de 20
reais, cinco pacotes de notas de 50 reais e um pacote de notas de 100
a) Elaine gastou R$ 10,00.
reais, totalizando 100 mil reais. No segundo caixa, foram roubados um
b) o preço do produto C é R$ 3,00. pacote de notas de 20 reais e três pacotes de notas de 100 reais, num
c) o preço do produto A é R$ 1,00. total de 50 mil reais. Os pacotes de notas de mesmo valor tinham a
mesma quantidade de notas. Cada pacote de notas de 100 reais tinha
d) o produto do produto B é R$ 3,00.
igual valor de cada pacote de notas de 50 reais.”

249

PM_BOOK16 - MAT.indb 249 25/11/2022 19:15:46


SISTEMA LINEAR

Diante do depoimento do suspeito, pode-se concluir que 37. (AFA) O sistema linear nas incógnitas x, y e z abaixo possui uma
a) ele pode ter falado a verdade. infinidade de soluções.
b) ele falou, necessariamente, a verdade. (sen a)x + y − z = 0

c) havia, necessariamente, 940 notas em cada pacote de notas de 20 reais.  x − (sen a)y + z =0
x + y = cos a
d) ele mentiu, necessariamente. 
Sobre o parâmetro a, a ∈ , pode-se afirmar que
33. (AFA) Irão participar do EPEMM, Encontro Pedagógico do Ensino
Médio Militar, um Congresso de Professores das Escolas Militares, 87 π
a) a = kπ, k ∈ . c) a= + 2kπ, k ∈ .
professores das disciplinas de Matemática, Física e Química. Sabe-se 2
que cada professor leciona apenas uma dessas três disciplinas e que o π
b) a = 2kπ, k ∈ . d) a= + kπ, k ∈ .
número de professores de Física é o triplo do número de professores 2
de Química.
(k + 2)x + y − z =0
Pode-se afirmar que 
38. O sistema linear x + ky + z = 0 é possível e determinado,
a) se o número de professores de Química for 16, os professores de −x + (k − 1)z =4

Matemática serão a metade dos de Física.
exceto para um número finito de valores de k. A soma de todos esses
b) o menor número possível de professores de Química é igual a 3. valores de k é:
c) o número de professores de Química será no máximo 21. a) −1 c) 0 e) 1
d) o número de professores de Química será maior do que o de b) −1/2 d) 1/2
Matemática, se o de Química for em quantidade maior ou igual
a 17.
39. Se S é o conjunto dos valores de a para os quais o sistema:

34. (AFA) Seja o sistema S de equações nas incógnitas x, y e z e  x+y+z=0
parâmetro real m. 
 x + 2y − z = 0  x + (log3a)2y + z = 0 é indeterminado, então:
 
S =  x − my − 3z = 0 2x + 2y +  log3 27  z = 0
x + 3y + mz = m   a 

a) S ⊂ [–3, 3] c) S ⊂ [2, 4] e) S ⊂ [0, 1]
Analise as proposições a seguir e assinale a alternativa INCORRETA
b) S é vazio d) S ⊂ [1, 3]
a) Se m = –3, então S é impossível
b) S é determinado se, e somente se, m ≠ 0. a11x1 + a12x 2 + ... + a1n xn =b1
c) Se S é homogêneo, então x + y + z é sempre um número múltiplo a x + a x + ... + a x = b2

de 3. 40. Considere o sistema linear:  21 1 22 2 2n n
onde
    
d) S admite solução para todo m ≠ 3. an1x1 + an2x 2 + ... + ann xn =
bn
aij ∈ , bi ∈ ; 1 ≤ i, j ≤ n. A afirmação correta está contida na
35. (AFA) Pedro e Maria com seus filhos Gabriel e João foram a uma
alternativa:
clínica médica para uma revisão de saúde. Fazia parte da avaliação
aferir o peso de cada um. A balança era muito antiga e tinha um a) A solução nula é a única solução do sistema.
defeito, só indicava pesos maiores que 60 kg. Para resolver a pesagem, b) O conjunto das soluções do sistema contém a solução nula.
procedeu-se da seguinte maneira: Pesou-se
c) Se (r1, r2, ..., rn) é a solução do sistema, então (Kr1, Kr2, ..., Krn)
• Pedro, Maria e Gabriel, totalizando 150 kg também é solução.
• Pedro, Gabriel e João, totalizando 117 kg d) Se aij ≠ 0, para 1 ≤ i ≤ n, então o sistema pode não ter solução.
• Maria, Gabriel e João, totalizando 97 kg e) n.r.a.
• Pedro, Maria, Gabriel e João, totalizando 172 kg
Com base nessas informações, é correto afirmar que
a) com essa balança é possível pesar Gabriel e João juntos. EXERCÍCIOS DE
b) a diferença entre os pesos de Pedro e Maria é o peso de João.
c) Pedro é mais pesado que Maria e João juntos. COMBATE
d) não é possível pesar Maria sozinha nessa balança.
01. Na figura a seguir, cada um dos sete quadros contém a medida de
36. (AFA) Alex possui apenas moedas de 25 centavos, de 50 centavos
um ângulo expressa em graus. Em quaisquer três quadros consecutivos
e de 1 real, totalizando 36 moedas.
temos os três ângulos internos de um triângulo.
Sabe-se que a soma do número de moedas de 25 centavos com o
dobro do número de moedas de 50 centavos é igual à diferença entre
Determine o valor do ângulo X.
82 e 5 vezes o número de moedas de 1 real. Nessas condições é
correto afirmar que a) 10°
a) esse problema possui no máximo 7 soluções. b) 15°
b) o número de moedas de 25 centavos nunca será igual ao número c) 20°
de moedas de 50 centavos. d) 25°
c) o número de moedas de 50 centavos poderá ser igual à soma do e) 30°
número de moedas de 25 centavos com as de 1 real.
d) o número de moedas de 1 real pode ser 3.

250

PM_BOOK16 - MAT.indb 250 25/11/2022 19:15:46


SISTEMA LINEAR

ax + y + z = 0
 08. (ESPCEX) Uma tropa realizou um exercício em que soldados,
02. (ESPCEX) A condição para que o sistema  x + 2y + z =0, a ∈ ,
tenha solução única é x + y + z = sargentos e oficiais executaram módulos padronizados de tiro,
 0 consumindo, individualmente, o número de munição estabelecido
a) a ≠ 1. d) a ≠ −2. conforme o seu nível hierárquico. No primeiro dia atiraram 16 soldados,
b) a ≠ −1. e) a ≠ 0. 8 sargentos e 4 oficiais, totalizando 96 munições; no segundo dia, 5
c) a ≠ 2. soldados, 4 sargentos e 3 oficiais, totalizando 38 munições; no terceiro
dia, 16 soldados, 4 sargentos e 1 oficial, totalizando 78 munições.
Quantas munições foram usadas no quarto dia, quando atiraram 14
3x  Ky  z  0 soldados, 8 sargentos e 2 oficiais?
03. (AFA) O sistema de equações lineares 
5x  4 y  5z  0 admite a) 78
mais de uma solução se: x  y  Kz  0
 b) 80
a) K = 7/6 c) 82
b) K= 7/5 ou K = 2 d) 84
c) K = 7/3 ou K = 2 e) 86
d) K = 7/2 ou K = 2
09. (ESPCEX) A figura abaixo é formada por um dispositivo de
3x  2y  z  7 forma triangular em que, nos vértices e nos pontos médios dos
04. Analisando o sistema 
x  y  z  0 concluímos que este é: lados, estão representados alguns valores, nem todos conhecidos.
Sabe-se que a soma dos valores correspondentes a cada lado do
2x  y  2z  1
 triângulo é sempre 24.

a) possível e determinado com xyz = 7. x


b) possível e determinado com xyz = −8.
c) possível e determinado com xyz = 6.
d) possível e indeterminado.
e) impossível. 5 10
x  z  w  0
 2
x  ky  k w  1
05. Considere o sistema: (P)  .
x  (k  1)z  w  1
x  z  kw  2 y z
 15
Podemos afirmar que (P) é possível e determinado quando:
a) k ≠ 0 Assim, o valor numérico da expressão x – y – z é
b) k ≠ 1 a) –2 d) 5
c) k ≠ −1 b) –1 e) 10
d) k ≠ 0 e k ≠ −1 c) 2
e) n.d.a.
 x + y + az = 1
06. (ESPCEX) No Brasil, três turistas trocaram por reais, no mesmo dia 
10. (ESPCEX) Para que o sistema linear  x + 2x + z = 2 , em que a
e pelas mesmas cotações, as quantias que possuíam em dólares, libras 2x + 5y − 3z =
 b
e euros, da seguinte forma:
e b são reais, seja possível e indeterminado, o valor de a + b é igual a
Turista A: 10 dólares, 20 libras e 15 euros por 122 reais; a) 10 d) 13
Turista B: 15 dólares, 10 libras e 20 euros por 114 reais; b) 11 e) 14
Turista C: 20 dólares, 10 libras e 10 euros por 108 reais. c) 12
O valor em reais recebido por uma libra foi
a) 2,60
b) 2,80
c) 3,00
d) 3,20
DESAFIO PRO
e) 3,40
 x + y + 4z =2
ax + y =−1 

07. (ESPCEX) No conjunto R, o sistema de equações  x + 2z =
 y−z =

0 é
2
1 (ITA) Se o sistema de equações  x + 2y + 7z =
3x + y + az =

3. É
b
impossível, então os valores de a e b são tais que
a) possível e determinado para todo a ≠ – 1/2.
a) a = 6 e b ≠ 4.
b) possível e indeterminado para a real qualquer.
b) a ≠ 6 e b ≠ 4.
c) impossível para a = – 1/2.
c) a ≠ 6 e b ≠ 4.
d) possível e indeterminado para a = 1/2.
d) a = 6 e b = 4.
e) impossível para a = 1/2.
e) a é arbitrário e b ≠ 4.

251

PM_BOOK16 - MAT.indb 251 25/11/2022 19:15:49


SISTEMA LINEAR

2 (ITA) Determine todos os valores reais de a para os quais o


seguinte sistema linear é impossível:
EXERCÍCIOS DE COMBATE
01. B 04. C 07. E 10. B
 x + ay + z = 2 02. A 05. D 08. D

 − x − 2y + 3z =−1. 03. B 06. D 09. D
3x + az = 5
  1 27 8 DESAFIO PRO
 x + y 2 + z3 =
3
 01. A 03. C 05. B
 4 81 40
3 (ITA) Considere o sistema de equações S  + 2 + 3 = 10 . 02. a = –6 04. B
Se (x,y,z) é uma solução real de S,  x y z
então |x| + |y| + |z| é igual a  2 54 24 ANOTAÇÕES
 + 2 + 3 = 7
x y z
a) 0 d) 9
b) 3 e) 12
c) 6
x + y + z = 0

4 (ITA) Se o sistema 2a2 y + (2a4 − a)z =
 3
0 admite infinitas
 x + ay + (a − 1)z =
0
soluções, então os possíveis valores do parâmetro a são

−1 − 3 −1 + 3
a) 0, − 1, , .
2 2
1− 3 1+ 3
b) 0, − 1, , .
2 2
−1 + 3 1 + 3
c) 0, − 1, , .
2 2
d) 0, − 1, − 1 − 3, − 1 + 3.
e) 0, − 1, 1 − 3, 1 + 3.

5 (ITA) Assinale a opção que identifica o lugar geométrico de


todos os pares ordenados (a,b) ∈ ² que tornam impossível
− x + 5y =10

o sistema linear S :  a2 2
.
 5 + 5b  x + 10aby =1
 
a) Uma elipse.
b) Uma reta.
c) Uma parábola.
d) Uma hipérbole.
e) Um único ponto.

GABARITO
EXERCÍCIOS DE FIXAÇÃO
01. A 04. D 07. C 10. B
02. B 05. D 08. C
03. C 06. D 09. A
EXERCÍCIOS DE TREINAMENTO
01. C 11. A 21. E 31. B
02. B 12. D 22. B 32. A
03. A 13. C 23. E 33. C
04. C 14. D 24. D 34. B
05. B 15. D 25. D 35. D
06. C 16. E 26. D 36. C
07. C 17. C 27. B 37. B
08. B 18. D 28. C 38. A
09. A 19. B 29. C 39. A
10. B 20. C 30. B 40. D

252

PM_BOOK16 - MAT.indb 252 25/11/2022 19:15:50


GEOMETRIA PLANA:
CONCEITOS FUNDAMENTAIS

CONCEITOS INICIAIS Um único plano fica determinado por:


a) Três pontos não colineares.
NOÇÕES AXIOMÁTICAS
Alguns elementos geométricos são tão primitivos que dispensam
qualquer definição, dentre eles estão o ponto

O PONTO
O ponto é representado por uma letra maiúscula do nosso
alfabeto.

A RETA
b) Uma reta e um ponto exterior.
A reta é a reunião de infinitos pontos alinhados, é um elemento
que não possui largura apenas comprimento. A reta é representada
por uma letra minúscula do nosso alfabeto ou ainda com símbolos
que veremos a frente.

O PLANO
É compreendido como uma superfície plana que não faz curva.
Planos são figuras geométricas bidimensionais formadas pela reunião de
infinitas retas, perpendiculares a uma reta dada, dispostas lado a lado

c) Duas retas concorrentes.

d) Duas retas paralelas distintas.

POSTULADOS PRINCIPAIS
• Dois pontos distintos determinam uma única reta que passa
por eles.
• Pontos colineares são pontos que pertencem a uma mesma
reta.
• Três pontos não colineares determinam um único plano que
passa por eles.
• Por um ponto não pertencente a uma reta, passa uma, e POSIÇÕES RELATIVAS
apenas uma, reta paralela à primeira (Euclides).
ENTRE PONTOS
DETERMINAÇÃO DO PLANO a) Coincidentes
Pontos coplanares são pontos que pertencem a um mesmo plano.

b) Distintos

253

PM_BOOK16 - MAT.indb 253 25/11/2022 19:15:50


GEOMETRIA PLANA: CONCEITOS FUNDAMENTAIS

ENTRE RETAS III. Segmento de Reta


a) Concorrentes: um ponto de interseção O segmento de reta possui início e fim e dessa forma é o único men-
surável, que se pode medir. É simbolizado pelos pontos dos seus ex-
( )
tremos AB .

CLASSIFICAÇÕES DOS SEGMENTOS


b) Paralelas Coincidentes: infinitos pontos de interseção I. Consecutivos
r≡s São os segmentos que onde termina o primeiro o seguinte
imediatamente continua.

c) Paralelas Paralelas: não há pontos de interseção

II. Colineares
São segmentos que estão alinhados, ou seja, sobre a mesma reta.

AB e CD são colineares e ambos estão sobre a reta r


Retas paralelas são retas coincidentes ou são retas coplanares
que não possuem ponto em comum. III. Consecutivos
• Se duas retas são paralelas a uma terceira, então elas são São os segmentos ao mesmo tempo consecutivos e colineares
paralelas entre si.

DEFINIÇÕES E REPRESENTAÇÕES
I. Reta
A reta é um elemento infinito e dessa forma sua representação é
mais complexa a partir somente de um desenho então pelo próprio
PERPENDICULARIDADE
postulado de determinação da reta, tendo conhecido 2 pontos A e B Retas perpendiculares ( ⊥ ) são retas concorrentes que formam

que pertencem a uma reta podemos representar uma reta por AB . ângulos adjacentes suplementares congruentes.

II. Semirreta
Podemos entender como um elemento que possui início mas não
possui fim, pode-se entender também como a reta com um de seus
lados tendo sido “cortado”, assim podemos ter a semirreta de início

em A e
na ( )
 direção de B AB ou a semirreta de início em B e na direção
( )
de A AB .

• Num plano, por um ponto dado de uma reta, passa uma


única reta perpendicular à reta dada.
• A projeção ortogonal de um ponto sobre uma reta é o ponto
de interseção da reta com a perpendicular a ela passando pelo
ponto dado.

254

PM_BOOK16 - MAT.indb 254 25/11/2022 19:15:51


GEOMETRIA PLANA: CONCEITOS FUNDAMENTAIS

Na figura acima, o ponto P’ é a projeção ortogonal do ponto P


sobre a reta r.
• A projeção ortogonal de um segmento de reta não ProBizu
perpendicular a uma reta sobre ela, é o segmento
A bissetriz de um ângulo é o lugar geométrico dos pontos que
determinado sobre a reta pelas projeções dos extremos do
equidistam dos lados de um ângulo.
segmento original.

ÂNGULOS OPOSTOS PELO VÉRTICE

ˆ = BOD
AOC ˆ

Na figura acima, o segmento A’B’ é a projeção ortogonal do


segmento de reta AB sobre a reta r.
Dois ângulos são opostos pelo vértice (o.p.v.) se, e somente se, os
lados de um deles são as respectivas semirretas opostas aos lados do outro.
DISTÂNCIAS Dois ângulos opostos pelo vértice são congruentes.
• A distância entre dois pontos A e B é a medida do
• Duas retas concorrentes determinam dois pares de ângulos
segmento de reta AB.
opostos pelo vértice.
• A distância entre um ponto e uma reta é distância do
• As bissetrizes de dois ângulos opostos pelo vértice são
ponto ao pé da perpendicular à reta conduzida pelo ponto.
semirretas opostas.
• A distância entre duas retas paralelas é distância entre um
ponto qualquer de uma das retas e a outra reta.

ÂNGULOS
Ângulo é a reunião de duas semirretas de mesma origem.

ÂNGULOS CONSECUTIVOS
Dois ângulos são consecutivos se, e somente se, possuem um lado
em comum.

ˆ ; OA OB; Ô; α.
Notações: AOB ˆ são
  ˆ e AOC
AOB
O ponto O é o vértice do ângulo e as semirretas OA e OB são os ângulos consecutivos
lados do ângulo.
Um ângulo determina dois setores angulares, um convexo e outro OA é o lado comum
côncavo, exceto no caso de semirretas opostas.

BISSETRIZ DE UM ÂNGULO
A bissetriz de um ângulo é uma semirreta que o divide em dois
ângulos congruentes.

255

PM_BOOK16 - MAT.indb 255 25/11/2022 19:15:52


GEOMETRIA PLANA: CONCEITOS FUNDAMENTAIS

ÂNGULO RETO
Ângulo reto é aquele que é igual a seu suplementar adjacente. A
medida de um ângulo reto é 90º.

ˆ e BOC
AOB ˆ são
ângulos consecutivos

OB é o lado comum

De fato, se θ é um ângulo reto, temos: θ = 180º - θ ⇔ θ = 90º.

ÂNGULOS ADJACENTES ÂNGULO AGUDO E ÂNGULO OBTUSO


Dois ângulos são adjacentes se, e somente se, são consecutivos e Ângulo agudo é aquele que é menor que um ângulo reto e ângulo
não possuem pontos internos comuns. obtuso é aquele que é maior que um ângulo reto.

ˆ é um ângulo agudo
AOB

α < 90º

Os ângulos AOB ˆ e BOC


ˆ , que possuem lado comum OB, são
ˆ é um ângulo
COD
ângulos adjacentes.
obtuso

ÂNGULO RASO
β > 90º
Ângulo raso é o ângulo determinado por duas semirretas opostas.
Um ângulo raso mede 180º.

UNIDADES DE MEDIDAS ANGULARES


A. SISTEMA SEXAGESIMAL − GRAU (°)
Um sistema sexagesimal é um sistema de numeração de base 60,
ÂNGULO SUPLEMENTAR ADJACENTE
ou seja, cada submúltiplo é 60 vezes menor que o anterior.
Dado o ângulo AOB ˆ , o ângulo suplementar adjacente de AOB ˆ é
  Para medidas angulares, é comum adotar um sistema sexagesimal
o ângulo determinado
 pelas semirretas OB e OC , semirreta oposta à
1
semirreta OA , ou seja, determinado por um dos lados do ângulo e com unidade de medida 1 grau (1º) que é de um ângulo reto.
pela semirreta oposta ao outro lado. 90
Um grau pode ser dividido em 60 minutos e cada minuto dividido
em 60 segundos.

1
1º = ⋅ ( ângulo reto ) ⇔ 1 ângulo reto =
90º
90
1 ângulo raso = 180
1 ângulo de uma volta = 360

Submúltiplos do grau:
1
Minuto: 1' = ⋅ 1º ⇔ 1º= 60'
60
A medida do ângulo suplementar adjacente de θ é 180º - θ. 1
Segundo: 1'' = ⋅ 1' ⇔ 1' = 60'' ⇔ 1 = 3600''
60

256

PM_BOOK16 - MAT.indb 256 25/11/2022 19:15:53


GEOMETRIA PLANA: CONCEITOS FUNDAMENTAIS

B. SISTEMA DECIMAL − GRADO (gr)


Um sistema decimal é um sistema numeração de base 10, ou seja,
cada submúltiplo é 10 vezes menor que o anterior.
Para medidas angulares, utiliza-se um sistema decimal com unidade
1
de medida 1 grado (1 gr), que equivale a de um ângulo reto.
100
α + β =90º

1 α e β são
1 gr = ⋅ ( ângulo reto ) ⇔ 1 ângulo reto =
100 gr
100 complementares
1 ângulo raso = 200 gr
1 ângulo de uma volta = 400 gr

C. SISTEMA CIRCULAR OU RADIOMÉTRICO − RADIANOS (rad)

O complemento de um ângulo é o ângulo que, junto ao


primeiro, forma um par de ângulos complementares. Portanto, o
complemento de x é 90º - x.
Complemento de x = 90º - x

ÂNGULOS SUPLEMENTARES
Ângulos suplementares são ângulos cujas medidas somam um
ângulo raso (180º).

O ângulo de 1 radiano (1 rad) é o ângulo central em uma


circunferência de raio R que determina um arco de comprimento R
sobre essa circunferência.
O sistema circular ou radiométrico adota como unidade de α + β =180º

medida 1 radiano (1 rad).
α e β são
Como o comprimento de uma circunferência de raio R é 2πR, suplementares
então um ângulo de uma volta mede 2π rad.

1
=
1 rad ⋅ ( ângulo de uma volta)

 O suplemento de um ângulo é o ângulo que, junto ao primeiro,
1 ângulo de uma volta= 2π rad forma um par de ângulos suplementares. Portanto, o suplemento de
x é 180º - x.
1 ângulo raso = π rad
Suplemento de x = 180º - x
π
1 ângulo reto = rad
2
As bissetrizes de dois ângulos adjacentes suplementares são
perpendiculares.
D. RELAÇÕES ENTRE AS UNIDADES
180º = 200 gr = π rad

E. NÚMERO COMPLEXO
Número complexo é um número que apresenta mais de uma
unidade de um mesmo sistema para exprimir uma grandeza.
Exemplos: 2h 30min 10seg e 15°20’32”.

Número incomplexo é um número que apresenta uma única


unidade de um sistema para exprimir uma grandeza.
Exemplos: 135,2 min e 65,32°.

ÂNGULOS COMPLEMENTARES
Ângulos complementares são ângulos cujas medidas somam um
ângulo reto (90º).

257

PM_BOOK16 - MAT.indb 257 25/11/2022 19:15:54


GEOMETRIA PLANA: CONCEITOS FUNDAMENTAIS

ÂNGULOS REPLEMENTARES
360º
Ângulos replementares são ângulos cujas medidas somam um O ângulo entre as marcações de horas é = 30º e o
12
ângulo de uma volta (360°).
360º
ângulo entre as marcações de minutos é = 6º .
60
A velocidade angular do ponteiro das horas é
30º
= 0,5º min e a velocidade angular do ponteiro dos
60 min
α + β =360º
⇓ minutos é 360º = 6º min .
α e β são 60 min
replementares Às H horas em ponto, o ângulo entre os ponteiros do relógio
ˆ= 30º ⋅ H.
é AOB
Entre H horas em ponto e H horas e M minutos, passaram-
se M minutos. Nesse período, o ponteiro das horas deslocou-
ˆ =
se BOD 0,5º min ⋅ M min = 0,5º ⋅ M e o ponteiro dos minutos
O replemento de um ângulo é o ângulo que, junto ao primeiro, deslocou-se AOC ˆ = 6 º min ⋅ M min = 6º⋅ M. Assim, há duas
forma um par de ângulos replementares. Portanto, o replemento de possibilidades para o ângulo entre os ponteiros das horas e dos
x é 360º - x. minutos:
1°. Se o ponteiro dos minutos não ultrapassou o ponteiro das
ProBizu
horas (Figura 1), temos:
Complemento de x = 90º - x ˆ = AOB ˆ + BOD
ˆ − AOC
ˆ = 30º ⋅ H + 0,5 ⋅ M − 6º ⋅ M
=θ COD = 30º ⋅ H − 5,5º ⋅ M
Suplemento de x = 180º - x
=
θ CODˆ = AOB
ˆ + BOD
ˆ − AOCˆ = 30º ⋅ H + 0,5 ⋅ M − 6º ⋅ M
= 30º ⋅ H − 5,5º ⋅ M.
Replemento de x = 360º - x
2°. Se o ponteiro dos minutos ultrapassou o ponteiro das
horas (Figura 2), temos:
Observações ˆ = AOC
θ = COD ˆ − AOB
ˆ − BOD
ˆ = 6º ⋅ M − 30º ⋅ H − 0,5 ⋅ M = 5,5º ⋅ M − 30º ⋅ H
Vamos analisar θo= problema
ˆ = AOC
COD ˆ de−identifi
ˆ −car
AOB BOD 6º ⋅ M − θ30º
ˆ o =ângulo entre
⋅ H − 0,5 ⋅ M = 5,5º ⋅ M − 30º ⋅ H.
os ponteiros das horas e dos minutos de um relógio às H horas
A expressão para o ângulo entre os ponteiros das horas
e M minutos.
e dos minutos de um relógio às H horas e M minutos pode ser
60º ⋅ H − 11º ⋅ M
representada de maneira única como θ = .
2

ProBizu
O ângulo entre os ponteiros de um relógio às H horas e M minutos é
60º ⋅ H − 11º ⋅ M
θ=
2

Exercício Resolvido

01. Sejam dois ângulos x e y tais que (2x) e (y + 10º) são ângulos
Figura 1
complementares e (5x) e (3y – 40º) são suplementares. O ângulo
x mede
a) 5º.
b) 10º.
c) 15º.
d) 20º.

Resolução: B
De acordo com as informações do problema, podemos escrever que
2x + y + 10° = 90° 2x + y = 80° −6x − 3y = −240°
 ⇒ ⇒
5x + 3y − 40° = 180° 5x + 3y = 220° 5x + 3y = 220°
Somando as equações, obtemos: x = 20º.

Figura 2

258

PM_BOOK16 - MAT.indb 258 25/11/2022 19:15:55


GEOMETRIA PLANA: CONCEITOS FUNDAMENTAIS

Exercício Resolvido a) um número primo maior que 23.


b) um número ímpar.
02. Os ângulos  e B̂ são congruentes. Sendo  = 2x + 15° e
B̂ = 5x − 9°. Assinale a alternativa que representa, corretamente, o c) um múltiplo de 4.
valor de x. d) um divisor de 60.
a) 2º b) 8º c) 12º d) 24º e) um múltiplo comum entre 5 e 7.

Resolução: B Resolução: D
 e B̂ são congruentes, podemos escrever que: Os ângulos (60º - α + 4α) = (60º + 3α) e 2α + 90º são alternos
2x + 15° = 5x − 9° ⇒ 24° = 3x ⇒ x = 8° internos. Portanto, 60º + 3α = 2α + 90º ⇔ α = 30º, que é um
divisor de 60.

Exercício Resolvido
EXERCÍCIOS DE
03. Calcule o valor de x, em graus, na figura:
FIXAÇÃO
01. (CFN) Qual é o menor ângulo formado entre os ponteiros de um
relógio quando são exatamente 7 horas?
a) 210° b) 180° c) 165° d) 150° e) 120°

02. (CFN) Na figura abaixo, sendo r//s, quais os valores de X, Y e Z,


a) 16. d) 58. respectivamente?
b) 10. e) 32.
c) 20.
80º
Resolução: A z
Os três ângulos juntos formam um ângulo reto, daí: r x
x + 3x + 10º + x = 90º ⇒ 5x = 80º ⇒ x = 16º. 150º
y
s
Exercício Resolvido

04. Duas retas paralelas “r” e “s”, cortadas por uma transversal
“t”, formam ângulos colaterais internos, dos quais um excede o a) 50°, 80° e 20° d) 80°, 150° e 100°
outro em 20º. O ângulo colateral interno agudo mede
b) 60°, 120° e 40° e) 100°, 80° e 30°
a) 20º b) 35º c) 55º d) 80º
c) 70°, 100° e 30°

Resolução: D 03. (CFN) Na figura abaixo, a medida do suplemento do menor


A soma dos ângulos colaterais de uma reta que atravessa retas ângulo é:
paralelas é 180º. Assim, se os ângulos forem x e y, pode-se deduzir:
2x + 70º
x + y =180
 ⇒ 2x= 200 → x= 100 → y= 80 3x - 40º
 x − y =20

Ângulos agudos são aqueles menores que 90º, portanto o ângulo a) 120° c) 132° e) 140°
colateral interno agudo mede 80º.
b) 130° d) 135°

Exercício Resolvido 04. (CFN) Na figura abaixo, a medida do complemento do menor


ângulo é
05. Na figura abaixo, a e b são retas paralelas.
2x + 20º
3x - 65º

a) 110° d) 20°
b) 70° e) 10°
c) 45°

A afirmação correta a respeito do número que expressa, em graus, 05. (CFN) Determine o valor da expressão 180° - 40°20’40’’.
a medida do ângulo α é a) 140° 20’ 40’’ d) 139° 40’ 20’’
b) 140° 39’ 20’’ e) 139° 39’ 20’’
c) 139° 39’ 40’’

259

PM_BOOK16 - MAT.indb 259 25/11/2022 19:15:56


GEOMETRIA PLANA: CONCEITOS FUNDAMENTAIS

06. (CFN) De acordo com a figura abaixo, determine o valor da EXERCÍCIOS DE


incógnita x.
TREINAMENTO
01. Sejam dois ângulos x e y tais que (2x) e (y + 10º) são ângulos
complementares e (5x) e (3y – 40º) são suplementares. O ângulo x
mede
2x + 20º
a) 5º. c) 15º.
3x - 65º
b) 10º. d) 20º.

a) 85° b) 45° c) 38° d) 27° e) 12° 02. Calcule o valor de x, em graus, na figura:

07. (CFN) Determine as medidas dos ângulos z, w, x e y.


r s

t
40º
r//s
t//u
z
x a) 16. c) 20. e) 32.
u
w b) 10. d) 58.
y
03. Duas retas paralelas “r” e “s”, cortadas por uma transversal “t”,
a) 40º, 180º, 40º e 10º. d) 140º, 40º, 40º e 140º. formam ângulos colaterais internos, dos quais um excede o outro em
20º. O ângulo colateral interno agudo mede
b) 40º, 140º, 40º e 140º. e) 180º, 90º, 30º e 60º.
a) 20º c) 55º
c) 140º, 60º, 140º e 60º.
b) 35º d) 80º
08. (EAM) Observe a figura a seguir.
04. (EAM) Se A = 10° 20’ 30” e B = 30° 50’ 10”, é correto afirmar que
30º r o valor de A + B é igual a:
a) 20° 30’ 20” d) 41° 10’ 40”
b) 40° 59’ 40” e) 51° 10’ 40”
75º
c) 41° 30’ 40”
x
05. (EAM) Duas retas paralelas r e s são cortadas por uma reta
s transversal formando, no mesmo plano, dois ângulos obtusos alternos
40º x   3x 
internos que medem  + 30°  e  + 15°  . Então o suplemento
2   5 
de um desses ângulos mede
Sabendo que, na figura acima, as retas r e s são paralelas, é correto
a) 75° b) 80° c) 82° d) 85° e) 88°
afirmar que o valor de x é igual a:
a) 90° b) 85° c) 80° d) 75° e) 70° 06. (EEAR) Sejam dois ângulos, α e β. Pode-se afirmar que β é
complemento de α se, e somente se,
09. (EAM) Qual é a medida do menor ângulo formado pelos ponteiros
a) suas medidas são iguais.
de um relógio às 15 horas e 20 minutos?
b) a soma de suas medidas é 90°.
a) 12° c) 20° e) 35°
c) a soma de suas medidas é 180°.
b) 15° d) 30°
d) seus lados são semiretas opostas.
10. (EAM) Observe a figura abaixo.
07. (EEAR) O complemento do suplemento do ângulo de 112° mede
A a
a) 18° b) 28° c) 12° d) 22°
140º
B x 08. (EEAR) Os ângulos  e B̂ e são congruentes. Sendo Aˆ = 2x + 15°
e B̂ = 5x − 9°. Assinale a alternativa que representa, corretamente, o
30º valor de x.
C b a) 2º b) 8º c) 12º d) 24º

Sabendo que a reta a é paralela à reta b, pode-se afirmar que, a partir 09. (EAM) Qual é a medida do menor ângulo formado pelos ponteiros
dos dados da figura acima, o valor do ângulo x é igual a: de um relógio às 15 horas e 20 minutos?
a) 10° c) 50° e) 100° a) 12° c) 20° e) 35°
b) 30° d) 70° b) 15° d) 30°

260

PM_BOOK16 - MAT.indb 260 25/11/2022 19:15:56


GEOMETRIA PLANA: CONCEITOS FUNDAMENTAIS

10. (EEAR) Somando-se o complemento e o suplemento do ângulo x, ˆ = 108º e ZOB


19. Na figura a seguir, AOC ˆ = 4º.
obtém-se a metade do replemento de x. Então, x é igual a _____.
X Z
a) 30° b) 40° c) 50° d) 60°
B
Y
11. Dois ângulos consecutivos são complementares. Então o ângulo
A
formado pelas bissetrizes desses ângulos é C
a) 20° d) 40°
b) 30° e) 45°
c) 35°
O
12. (CMF) A medida de um ângulo é igual a 60% da medida do seu ˆ , BOC
ˆ e XOY
ˆ ,
Sabendo-se que OX, OY e OZ são as bissetrizes de AOB
suplemento. O dobro da medida desse ângulo é igual a: ˆ .
respectivamente, determine a medida de AOB
a) 97º d) 128º
a) 54º c) 60º e) 72º
b) 108º e) 135º
b) 58º d) 62º
c) 117º
20. Na figura abaixo, calcular o valor do ângulo “x” em função dos
ˆ
13. (CMF) Na figura abaixo, as medidas (em graus) dos ângulos QPS ˆ .
ângulos “a” e “b”, sabendo que OJ é a bissetriz do ângulo AOC
ˆ são, respectivamente, x² - 70 e 3x, onde x é um número real
e RPT
ˆ é igual a:
positivo. A medida de RPS C

T Q
P
R
S O b J
a
a) 70º c) 110º e) 159º 2b B
b) 100º d) 150º
x

14. (CEFET-PE) A medida em graus de um ângulo, cuja terça parte da


medida de seu complemento mede 10º20’, é: A
a) 55º d) 58º M
b) 56º e) 59º
c) 57º a+b 2a + b a + 3b
a) x= c) x= e) x =
2 2 2
15. O resultado da média aritmética ponderada dos ângulos 55º15’37”
a + 2b 3a + b
e 20º42’30”, com pesos 2 e 3, respectivamente, é representado em b) x = d) x =
grados aproximadamente por: 2 2

a) 25 c) 32,4 e) 38,4
b) 28,2 d) 34,5
EXERCÍCIOS DE
16. Dois ângulos complementares  e B̂ , sendo Aˆ < Bˆ , têm medidas
na razão de 13 para 17. Consequentemente, a razão da medida do
suplemento do ângulo  para o suplemento do ângulo B̂ vale:
COMBATE
43 13 47
a) c) e)
17 17 43 ˆ , BOC
01. Sejam os ângulos adjacentes AOB ˆ e COD
ˆ tais que AOB
ˆ = 18
17 119 ˆ 
e COD = 24 . Calcule a medida do ângulo formado pelas bissetrizes
b) d)
13 48 ˆ e BOD
dos ângulos AOC ˆ .
a) 6° c) 21° e) 33°
17. Considere θ e α dois ângulos adjacentes e complementares. A
expressão que determina o valor do ângulo formado pelas bissetrizes b) 12° d) 25°
de θ e α é
02. (CN) Sabendo-se que um grado é a centésima parte de um ângulo
θ+α 90 − ( θ + α )
a) . c) . reto, quantos grados tem o ângulo de 45°36’54’’?
2 2
a) 50,48333... c) 50,68333... e) 50,88333...
θ+α 90 − ( θ + α )
b) . d) . b) 50,58333... d) 50,78333...
4 4
03. (CMRJ) A soma do triplo do suplemento do dobro da medida de
18. Determine o replemento do suplemento do complemento do um ângulo, com a quarta parte do complemento da medida desse
ângulo cujo número que o representa em graus somado ao número ângulo, tem como resultado 125°. Então, podemos afirmar que o
que o representa em grados vale 95. replemento da medida desse ângulo, em graus, é:
a) 45º c) 135º e) 275º a) 200. c) 240. e) 290.
b) 75º d) 225º b) 210. d) 260.

261

PM_BOOK16 - MAT.indb 261 25/11/2022 19:15:58


GEOMETRIA PLANA: CONCEITOS FUNDAMENTAIS

04. (AFA) Na figura, O é o centro da circunferência de raio r,


AD = DE = EB = r, e α é o menor ângulo formado pelos ponteiros de
ˆ é
um relógio às 9h 25min. O valor do ângulo β =CBE
DESAFIO PRO
 

1 Na figura ρ1 / / ρ2 , se o ângulo ABC é agudo, o valor máximo


inteiro de x é:

a) 120°. c) 126,25°.
b) 119,45°. d) 135,50°.

05. Quando, pela segunda vez, depois de 13 horas e 15 minutos,


o menor ângulo entre os ponteiros das horas e dos minutos de um
relógio mede 90°?
8 1
a) 13h 54min 32 seg d) 13h 21min 49 seg
11 11
5
b) 13h 49min 5 seg e) 14h 27min 16 4 seg a) 30°. d) 44º.
11 11
b) 46°. e) 60°.
c) 15h c) 45°.

06. (CN) Os ponteiros das horas, dos minutos e dos segundos de um


relógio indicam zero hora. Até às 9 horas do mesmo dia, os ponteiros
dos minutos e dos segundos terão se encontrado um número de vezes
2 Na figura, calcule x:

igual a:
a) 524. c) 540. e) 590.
b) 531. d) 573.

07. (EPCAR) De um ponto O, tomado sobre uma reta AB (O entre A e


B), traçam-se para um mesmo semiplano de AB, as semirretas ON, OP
e OQ. Os ângulos AON ˆ , POQ
ˆ , NOP ˆ medem, respectivamente,
ˆ e QOB
80° – 3x, 5x – 14°, x e 4x + 9°. O complemento do menor ângulo é
a) 68°. c) 78°.
b) 75°. d) 80°.
a) 30° d) 25°
08. (CN) Suponha que 1 (um) naval (símbolo n) seja a medida de um b) 24° e) 22°
ângulo convexo, menor que um ângulo reto, inscrito em um círculo de c) 20°
raio r, cujos lados determinam, nesse círculo, um arco de comprimento
r. Assim sendo, a soma das medidas dos ângulos internos de um
triângulo é igual a GABARITO
π c) πn . e) 4π n .
a) n. EXERCÍCIOS DE FIXAÇÃO
4
d) 2π n . 01. D 04. D 07. D 10. C
π
b) n.
2 02. C 05. E 08. B
03. B 06. D 09. C
09. (UFF) Sabendo que o replemento do dobro de um ângulo é igual EXERCÍCIOS DE TREINAMENTO
ao suplemento do complemento desse mesmo ângulo. Determine a
01. D 06. B 11. E 16. E
quarta parte deste ângulo.
02. A 07. D 12. E 17. A
a) 15° d) 60°
03. D 08. B 13. D 18. D
b) 22,5° e) 67,5°
04. D 09. C 14. E 19. D
c) 45°
05. A 10. D 15. E 20. E
10. (ITA) Entre duas superposições consecutivas dos ponteiros das EXERCÍCIOS DE COMBATE
horas e dos minutos de um relógio, o ponteiro dos minutos varre um 01. C 04. C 07. B 10. C
ângulo cuja medida, em radianos, é igual a
02. C 05. A 08. D
23 24 7
a) π. c) π. e) π. 03. E 06. B 09. B
11 11 3
DESAFIO PRO
13 25
b) π. d) π. 01. D 02. B
6 11

262

PM_BOOK16 - MAT.indb 262 25/11/2022 19:15:58


GEOMETRIA PLANA: TRIÂNGULOS,
CONGRUÊNCIA E PONTOS NOTÁVEIS

TRIÂNGULOS
DEFINIÇÃO E ELEMENTOS

Triângulo Equilátero

Os triângulos equiláteros possuem três lados de medidas iguais e


três ângulos internos iguais de medida 60°.
Se um triângulo possui dois lados de medidas iguais, ele é dito
isósceles.

Um triângulo é a figura geométrica formada por três segmentos


de reta consecutivos dois a dois unidos por suas extremidades.
Os segmentos de reta são os lados do triângulo e as extremidades
dos segmentos de reta, os vértices do triângulo.
Lados: AB , AC , BC
Vértices: A, B, C
Triângulo Isósceles
Os ângulos internos do triângulo são os ângulos formados pelos
O lado desigual de um triângulo isósceles, quando houver, é
lados do triângulo.
chamado de base e o ângulo interno oposto a ele, ângulo do vértice.
ˆ = Aˆ , ABC
Ângulos internos: BAC ˆ = Cˆ
ˆ = Bˆ , BCA
Os ângulos adjacentes à base de um triângulo isósceles são iguais.

Os ângulos externos são formados por um lado do triângulo e o Se um triângulo possui lados de medidas distintas duas a duas,
prolongamento do lado adjacente, e são o suplementar adjacente do ele é dito escaleno.
ângulo interno de mesmo vértice.
ˆ , B'BC
Ângulos externos: A'AB ˆ
ˆ , C'CA

CLASSIFICAÇÃO DOS TRIÂNGULOS

CLASSIFICAÇÃO DOS TRIÂNGULOS


QUANTO AOS LADOS
Se um triângulo possui três lados de medidas iguais, ele é dito
equilátero.
Triângulo Escaleno

Todo triângulo equilátero é também isósceles, mas a recíproca não


é verdadeira.

263

PM_BOOK16 - MAT.indb 263 25/11/2022 19:16:00


GEOMETRIA PLANA: TRIÂNGULOS, CONGRUÊNCIA E PONTOS NOTÁVEIS

CLASSIFICAÇÃO DOS TRIÂNGULOS Demonstração:


QUANTO AOS ÂNGULOS ˆ = ABC
Se DE  BC , então DAB ˆ e EAC ˆ .
ˆ = ACB
Se um triângulo possui os três ângulos agudos, ele é dito acutângulo. Logo,
ˆ + BAC
DAB ˆ + CAE ˆ =DAE
ˆ ⇔ ABC
ˆ + BAC ˆ =180° ⇔
ˆ + ACB
⇔ Aˆ + Bˆ + Cˆ = 180°

ÂNGULO EXTERNO DE UM TRIÂNGULO


Cada ângulo externo de um triângulo é igual à soma dos ângulos
internos não adjacentes.

α, β, γ < 90
Triângulo Acutângulo

Se um triângulo possui um ângulo reto, ele é dito retângulo.

Aˆ e= Bˆ + Cˆ
Triângulo Retângulo

Em um triângulo retângulo, o lado oposto ao ângulo reto é Demonstração:


chamado hipotenusa, e os lados adjacentes ao ângulo reto são Aˆ e + Cˆ = 180° ⇔ Aˆ e + Cˆ = Aˆ + Bˆ + Cˆ ⇔ Aˆ e = Bˆ + Cˆ
chamados catetos.
Se um triângulo possui um ângulo obtuso, ele é dito obtusângulo. Corolário: A soma das medidas dos três ângulos externos de um
triângulo é igual a 360°.

DESIGUALDADES NO TRIÂNGULO

RELAÇÃO ENTRE LADOS E ÂNGULOS


Teorema: Num triângulo qualquer, ao maior (menor) lado, opõe-
se o maior (menor) ângulo. Mais precisamente, em um ∆ABC , tem-se
Aˆ > Bˆ se, e somente se, BC > AC.
Demonstração:

θ > 90
Triângulo Obtusângulo

SOMA DOS ÂNGULOS INTERNOS


DE UM TRIÂNGULO
A soma dos ângulos internos de um triângulo é 180°.

(Volta) Seja BC > AC, então marquemos no segmento BC , um


ponto D tal que CD = AC . O triângulo ADC é isósceles, portanto
Aˆ > DAC
ˆ = ADC ˆ > Bˆ .

(Ida) Seja Aˆ > Bˆ . Não podemos ter BC = AC, pois se teria Aˆ = Bˆ .


Também não podemos ter BC < AC, pois a demonstração anterior
implicaria Aˆ < Bˆ . Portanto, a única possibilidade é BC > AC.

Aˆ + Bˆ + Cˆ =
180

264

PM_BOOK16 - MAT.indb 264 25/11/2022 19:16:01


GEOMETRIA PLANA: TRIÂNGULOS, CONGRUÊNCIA E PONTOS NOTÁVEIS

DESIGUALDADE TRIANGULAR ENVOLVENTE E ENVOLVIDA


Teorema: Cada lado de um triângulo é menor que a soma dos Teorema: Dadas duas curvas convexas que unem dois pontos
outros dois. distintos, sendo uma envolvente e a outra envolvida, a envolvente é
a < b + c maior que a envolvida.

Logo, em um triângulo ABC, de lados a, b e c, temos: b < a + c . Sejam 1 e 2 os comprimentos das duas curvas convexas indicadas
c < a + b na figura, então 2 > 1.

Demonstração:

CONGRUÊNCIA DE TRIÂNGULOS
Dois triângulos são ditos congruentes (símbolo ≡ ) se, e somente
Tomemos um ponto D na reta AB de forma que B está
se, é possível estabelecer uma correspondência entre seus vértices de
entre A e D e que BD = BC. Como o ∆DBC é isósceles, temos
modo que: seus lados são ordenadamente congruentes e seus ângulos

ADC ˆ
=
BDC ˆ < ACD
BCD ˆ .
são ordenadamente congruentes.
Aplicando a desigualdade (1.1) ao ∆ADC , tem-se AC < AD =
AB + BD = AB + BC, como queríamos demonstrar.

ProBizu
A condição pode ser garantida também, quando o maior lado
é conhecido, verificando-se apenas que o maior lado é menor que
a soma dos outros dois lados.
Alternativamente, quando o menor lado é conhecido, pode-
se verificar apenas que o menor lado é maior que o módulo da
diferença dos outros dois lados.

Corolário: A menor distância entre dois pontos é o segmento de


reta que os une.
Exemplo: Sejam os pontos A e B do mesmo lado de uma reta r. = AB A'B';
= =
AC A'C'; BC B'C'
∆ABC ≡ ∆A'B'C' ⇔ 
Identifique o ponto C ∈ r tal que AC + CB assume o valor mínimo. = ˆ ˆ=
A A'; ˆ ˆ=
B B'; ˆ ˆ
C C'

CRITÉRIOS DE CONGRUÊNCIA DE TRIÂNGULOS


Critério Lado – Ângulo – Lado (L.A.L.)
Se dois lados de um triângulo e o ângulo por eles formado
forem congruentes a dois lados de outro triângulo e ao ângulo por
eles formados, respectivamente, então esses dois triângulos são
congruentes (postulado).

Seja B’ a reflexão do ponto B em relação à reta r, então BC = B’C.


O menor caminho de A a B’ é o segmento de reta que une esses dois
pontos. Como AC + CB = AC + CB’, então o ponto C que faz AC + CB
assumir o valor mínimo é a interseção do segmento AB’ com a reta r. AB = A'B'   Bˆ = B'
ˆ
 (L.A.L.) 
= ˆ ˆ
A A'  ⇒ ∆ABC ≡ ∆A'B'C' ⇒ =
BC B'C'
  ˆ ˆ
AC = A'C'  C = C'

265

PM_BOOK16 - MAT.indb 265 25/11/2022 19:16:02


GEOMETRIA PLANA: TRIÂNGULOS, CONGRUÊNCIA E PONTOS NOTÁVEIS

Critério Ângulo – Lado – Ângulo (A.L.A.) CRITÉRIO ESPECIAL DE CONGRUÊNCIA DE


Se um dos lados de um triângulo e os ângulos adjacentes a esse TRIÂNGULOS RETÂNGULOS
lado forem congruentes a um dos lados de outro triângulo e aos Se dois triângulos retângulos têm um cateto e a hipotenusa,
ângulos adjacentes a esse lado, respectivamente, então esses dois respectivamente, congruentes, então esses dois triângulos são
triângulos são congruentes (teorema). congruentes.

ˆ A'
=
A ˆ= 90  AC = A'C'
 
=
AB A'B'  ⇒ ∆ABC ≡ ∆A'B'C' ⇒= Bˆ B'ˆ
  ˆ ˆ
BC = B'C'   C = C'

Bˆ = B'
ˆ  AB = A'B'
 ( A.L.A.) 
=
BC B'C' ⇒ ∆ABC ≡ ∆A'B'C' ⇒ = Aˆ A'ˆ LUGAR GEOMÉTRICO E CEVIANAS
ˆ 
ˆC = C' 
  AC = A'C' Um lugar geométrico (L.G.) é o conjunto de todos os pontos que
possuem uma determinada propriedade.
Critério Lado – Lado – Lado (L.L.L.) Assim, se o conjunto L é o lugar geométrico dos pontos que
possuem uma propriedade p, então:
Se os três lados de um triângulo são respectivamente congruentes
aos três lados de outro triângulo, então esses dois triângulos são 1°. Se o ponto A ∈ L, então A possui a propriedade p; e
congruentes (teorema). 2°. Se o ponto A possui a propriedade p, então A ∈ L.
Por exemplo, uma circunferência de centro em um ponto O e raio
r é o lugar geométrico dos pontos do plano que estão a uma distância
constante e igual a r do ponto O.
Um segmento que tem uma extremidade em um vértice
de um triângulo e a outro no lado oposto a esse vértice é
denominado ceviana.

MEDIATRIZES
A mediatriz de um segmento é a reta perpendicular ao segmento
pelo seu ponto médio.

AB = A'B'  Aˆ = A'ˆ


 (L.L.L.) 
=
BC B'C'  ⇒ ∆ABC ≡ ∆A'B'C' ⇒
= Bˆ B' ˆ
 ˆ ˆ
AC = A'C' C = C'

Critério Lado – Ângulo – Ângulo oposto (L.A.Ao.)


Se um lado, um ângulo adjacente e o ângulo oposto a esse lado,
de dois triângulos são ordenadamente congruentes, então esses dois
triângulos são congruentes.

A mediatriz de um segmento é o lugar geométrico dos


pontos equidistantes das extremidades do segmento.
BC = B'C'  Bˆ = B'
ˆ
 (L.A.Ao .)
 
=Cˆ C' ˆ  ⇒ ∆ABC ≡ ∆A'B'C' ⇒ =
AB A'B'
ˆ 
ˆA = A' 
AC = A'C'

266

PM_BOOK16 - MAT.indb 266 25/11/2022 19:16:04


GEOMETRIA PLANA: TRIÂNGULOS, CONGRUÊNCIA E PONTOS NOTÁVEIS

BISSETRIZES
A bissetriz de um ângulo é uma semirreta que o divide em dois
ângulos congruentes.

MEDIANAS
Uma mediana de um triângulo é um segmento com extremidades
em um dos vértices e no ponto médio do lado oposto.

A bissetriz de um ângulo é o lugar geométrico dos pontos


que equidistam dos lados de um ângulo.

ALTURAS
Uma altura de um triângulo é um segmento de reta perpendicular
à reta suporte de um lado do triângulo e com extremidades nesta reta
e no vértice oposto ao lado considerado.

BM ≡ MC ⇒ AM é a mediana relativa ao vértice A do ∆ABC .

PONTOS NOTÁVEIS DOS TRIÂNGULOS

BARICENTRO
As três medianas de um triângulo interceptam-se em um único
ponto denominado baricentro. O baricentro divide as medianas na
razão 2:1, onde a parte maior é a que contém o vértice.

267

PM_BOOK16 - MAT.indb 267 25/11/2022 19:16:05


GEOMETRIA PLANA: TRIÂNGULOS, CONGRUÊNCIA E PONTOS NOTÁVEIS

INCENTRO TRIÂNGULO ÓRTICO


As três bissetrizes internas de um triângulo interceptam-se em um O triângulo órtico é o triângulo formado pelos pés das alturas de
único ponto denominado incentro e que equidista dos lados do triângulo. um triângulo.

No triângulo retângulo o triângulo órtico não está definido.


Em qualquer triângulo acutângulo, o ortocentro é o incentro do
O incentro de um triângulo é o centro da circunferência inscrita triângulo órtico, e seus vértices são exincentros do triângulo órtico.
no triângulo. Em qualquer triângulo obtusângulo, o ortocentro é um dos
exincentros do triângulo órtico, o vértice do ângulo obtuso é o
incentro do triângulo órtico, e os outros dois vértices são os outros
dois exincentros do triângulo órtico.

CIRCUNCENTRO
As mediatrizes dos lados de um triângulo interceptam-se em um
único ponto denominado circuncentro e que equidista dos vértices
do triângulo.

ORTOCENTRO
As três alturas de um triângulo (ou seus prolongamentos)
concorrem em um único ponto denominado ortocentro.
Nas figuras seguintes, H é o ortocentro dos triângulos.

O circuncentro de um triângulo é o centro da circunferência


circunscrita ao triângulo.

268

PM_BOOK16 - MAT.indb 268 25/11/2022 19:16:06


GEOMETRIA PLANA: TRIÂNGULOS, CONGRUÊNCIA E PONTOS NOTÁVEIS

Exercício Resolvido Exercício Resolvido

01. A medida, em graus, do maior ângulo do triângulo abaixo é 03. Na figura abaixo, temos os segmentos PQ = PR
= QS e
( )  ?
 = 20° . Qual é a medida do ângulo RQS
m QPR

a) 90
a) 50°
b) 84
b) 60°
c) 72
c) 65°
d) 60
d) 70°
e) 54
e) 75°

Resolução: A
Resolução: B
2x = 30°, 4x = 60° e 6x = 90°
6x + 4x + 2x = 180° O triângulo PRQ é isósceles de base QR , logo podemos escrever:
12x = 180 ˆ
=
PQR Q=
R̂P x
x = 15° ˆ + QRP
ˆ + 20=
PQR ° 180° ⇒ x + x + 20=
° 180° ⇒ 2x
= 160° ⇒=
x 80°
Logo, 2x = 30°, 4x = 60° e 6x = 90°.
Sabemos que o triângulo PSQ é isósceles de base PS logo temos:
ˆ = QSˆ R
RPQ = 20°
Exercício Resolvido
Portanto:
02. Se S = a + b + c , considerando a figura abaixo, podemos ˆ = 180° − PRQ
O ângulo externo SRQ ˆ = 180° − 80=
° 100°
afirmar que:
Assim,
ˆ + QSR
ˆ + RQS
SRQ ˆ= 180° ⇒ 100° + RQS
ˆ + 20= ˆ= 60°
° 180° ⇒ RQS

Exercício Resolvido

04. Na figura, o triângulo ABC é retângulo em A, M é o ponto


médio do lado BC e AM = MC. Então a medida de α, em graus, é
A

a) S = 180°
b) S = 80°
c) S = 140°
40°
d) S = 360° α
B M C

Resolução: C
a) 80º
b) 90º
c) 100º
d) 110º
e) 120º

x= a + b

c + 40° +=
x 180°

a + b + 40° + c= 180°

a + b + c= 140°

269

PM_BOOK16 - MAT.indb 269 25/11/2022 19:16:07


GEOMETRIA PLANA: TRIÂNGULOS, CONGRUÊNCIA E PONTOS NOTÁVEIS

Resolução: A Resolução: B
Se M é o ponto médio do lado BC temos que BM = MC e Da soma dos ângulos internos do triângulo ABC teremos que
= 90°
consequentemente BM = MC = AM e o ângulo BÂC 90° + 40° + Cˆ= 180° ⇒ Cˆ= 50°
Sendo M o ponto médio de BC teremos que BM = MC e sendo N
ˆ = 50=
ˆ = NCB °
bissetriz do ângulo C teremos que ACN 25° .
2

Como o triângulo ACM é isósceles teremos


ˆ = ACˆ M
MAC = 40°

Como vimos no PROENEM EXPLICA, temos que AM = BM = CM.


ˆ = BAM
Assim o triângulo AMB é isósceles e MBA ˆ = 40° .

Assim α é o ângulo externo do triângulo ACM e pelo teorema do


ângulo externo.
α= 40° + 40°= 80°

Da soma dos ângulos internos do triângulo ACN


Exercício Resolvido ˆ = 190° ⇒ ANˆ C= 65°
90° + 25° + ANC
05. Na figura, M é o ponto médio do lado BC, CN é a bissetriz
interna e BM = AM. Então a medida α, em graus, é:
A

N α

40°
B M C

a) 80º
b) 75º
c) 70º Chamando a intersecção de AM com CN de P teremos o triângulo
d) 65º ANP, onde 40° + 65° + α= 180° ⇒ α= 75°
e) 60º

270

PM_BOOK16 - MAT.indb 270 25/11/2022 19:16:07


GEOMETRIA PLANA: TRIÂNGULOS, CONGRUÊNCIA E PONTOS NOTÁVEIS

EXERCÍCIOS DE 06. (EAM) As medidas dos ângulos de um triângulo são expressas,

FIXAÇÃO em grau, por X + 12º, 2X e X – 20º. Nessas condições, determine as


medidas dos três ângulos desse triângulo.
a) 60º, 85º e 35º
b) 59º, 94º e 27º
01. Qual dos pontos notáveis do triângulo pode ser um de seus
c) 74º, 92º e 14º
vértices?
d) 81º, 72º e 27º
a) baricentro
e) 92º, 56º e 32º
b) incentro
c) circuncentro 07. (EEAR) Se ABC é um triângulo, o valor de α é
d) ortocentro
e) ex-incentro.

02. O segmento da perpendicular traçada de um vértice de um


triângulo à reta do lado oposto é denominada altura. O ponto de
intersecção das três retas suportes das alturas do triângulo é chamado
a) Baricentro
b) Incentro
c) Circuncentro
d) Ortocentro
e) Mediana a) 10°
b) 15°
03. (CFN) Um dos ângulos da base de um triângulo isósceles mede
c) 20°
40°. Quanto mede o ângulo do vértice?
d) 25°
a) 108°
b) 100° 08. (EEAR) Se x, x + 20° e 2x são as medidas dos ângulos internos de
c) 99° um triângulo, então o maior desses ângulos mede ___.
d) 95° a) 50°
e) 90° b) 70°
c) 80°
04. (EAM) Considere o triângulo ABC, isósceles, de lados AB = AC.
d) 120°
Seja o ponto D, sobre o lado BC, de forma que o ângulo BAD é 30°.
Seja E o ponto sobre o lado AC, tal que o ângulo EDC vale x graus.
Tendo em vista que o segmento AD e AE têm as mesmas medidas, é 09. (EEAR) Num triângulo RST a medida do ângulo interno R é 68° e
correto afirmar que o valor da quarta parte de x é: do ângulo externo S é 105°. Então o ângulo interno T mede

a) 3º a) 52°.

b) 3º 20’ b) 45°.

c) 3º 30’ c) 37°.

d) 3º 35’ d) 30°.

e) 3º 45’
10. (EAM) Observe a figura abaixo

05. (CFN) Na figura abaixo, os pontos A, B e C estão alinhados. D C


Qual é a soma dos ângulos marcados em cinza? E

A B

Na figura apresentada, ABCD é um quadrado e ABE é um triângulo


A equilátero. Nestas condições, é correto afirmar que o triângulo AED é;
B C
a) retângulo em E
a) 120° b) escaleno e com ângulo AÊD = 60º
b) 180° c) isósceles e com ângulo AÊD = 75º
c) 270° d) acutângulo e com ângulo AÊD = 65º
d) 360° e) obtusângulo e com ângulo AÊD = 105º
e) 540°

271

PM_BOOK16 - MAT.indb 271 25/11/2022 19:16:08


GEOMETRIA PLANA: TRIÂNGULOS, CONGRUÊNCIA E PONTOS NOTÁVEIS

EXERCÍCIOS DE
06. (EEAR) Na figura, AH é altura do triângulo ABC. Assim, o valor

TREINAMENTO de x é

01. (EAM) Em um triângulo ABC, o ângulo interno em A é o dobro do


ângulo interno em B. Sabendo que o ângulo interno em C é o triplo
do ângulo interno em A, o menor ângulo interno deste triângulo é:
a) 30° d) 15°
b) 25° e) 10°
c) 20°
a) 20°.
02. (EEAR) Se 2x + 3, 5 e 3x − 5 são as três medidas, em cm, dos lados b) 15°.
de um triângulo, um valor que NÃO é possível para x é
c) 10°.
a) 3
d) 5°.
b) 4
c) 5 07. (EPCAR) Sabendo-se que os ângulos internos de um triângulo
d) 6 são diretamente proporcionais aos números 2, 3 e 4, tem-se que suas
medidas valem:
03. Dada a figura a) 40°, 60° e 80°
b) 30°, 50° e 100°
c) 20°, 40° e 120°
d) 50°, 60° e 70°

08. (EPCAR) Os valores de x para os quais é possível construir um


triângulo, cujos lados medem x, 5 e 9 unidades de medidas são
a) Todo x natural
b) Todo x natural menor que 14
c) x < 14
d) 4 < x < 14
Sobre as sentenças
I. O triângulo CDE é isósceles. 09. No triângulo ABC da figura, M é o ponto médio de BC. Se
II. O triângulo ABE é equilátero. ˆ = 15 e AMC
ABM ˆ é:
ˆ = 30 , a medida do ângulo BCA
III. AE é bissetriz do ângulo BÂD.
é verdade que
a) somente a I é falsa.
b) somente a II é falsa.
c) somente a III é falsa.
d) são todas falsas.
e) são todas verdadeiras.

04. A altura AH e a mediana AM traçadas do vértice A do ângulo reto a) 15°


de um triângulo retângulo formam um ângulo de 24°. Sendo assim, b) 30°
os ângulos agudos do triângulo são:
c) 45°
a) 33° e 57° d) 36° e 54°
d) 60°
b) 34° e 56° e) 37° e 53°
e) 75°
c) 35° e 55°
10. Num triângulo ABC, as bissetrizes dos ângulos externos do vértice
05. O triângulo ABC da figura é retângulo em A, AS é a bissetriz B e C formam um ângulo de medida 50°. Calcule o ângulo interno
interna, AM é mediana e BM = AM. Então, a medida de α, em graus, é do vértice A.
a) 110°
b) 90°
c) 80°
a) 10º d) 50°
b) 15º e) 20°
c) 20º
d) 25º
e) 30º

272

PM_BOOK16 - MAT.indb 272 25/11/2022 19:16:08


GEOMETRIA PLANA: TRIÂNGULOS, CONGRUÊNCIA E PONTOS NOTÁVEIS

11. (EPCAR) No triângulo ABC abaixo, a bissetriz do ângulo interno A 15. O triângulo ABC é isósceles, com AB = BC e o ângulo B vale 20°.
forma com o lado AB um ângulo de 55º. O ângulo β agudo formado Os triângulos ADC e DCE são também isósceles, com AD = AC e
pelas retas suporte das alturas relativas aos vértices B e C é ED = DC. O ângulo DCE mede:

a) 18°
b) 34°
c) 48°
d) 50°
a) Menor que 70°
e) 73°
b) O complemento de 20°
c) Igual ao dobro de 25° 16. No triângulo escaleno ABC, as bissetrizes AD e BE se intersectam
d) O suplemento de 120° em I. Sabendo que o ângulo BÎD = 70°, calcule o ângulo interno C
desse triângulo.
12. (EPCAR) Os lados de um triângulo são: 16 – x, 2x + 2, x + 12. a) 30°.
Sejam os conjuntos b) 40°.
A = {x ∈  | 10 < x < 15} c) 50°.
B = {x ∈  | 0 < x < 15} d) 60°.
e) 70°.
C = {x ∈  | 5 < x < 10}
17. (UFF) O triângulo MNP é tal que ângulo M = 80° e ângulo P = 60°.
 1  A medida do ângulo formado pela bissetriz do ângulo interno N com
D = x ∈  | ≤ x ≤ 15
 2  a bissetriz do ângulo externo P é:
Dizemos que x é solução, se para todo x real, o triângulo existe. Com a) 20°
base nisso, pode-se afirmar que b) 30°
a) Não existem soluções em A c) 40°
b) x é a solução somente se x ∈ B d) 50°
c) O triângulo existe para todo x ∈ C e) 60°
d) D é o conjunto de todas as soluções do problema
18. Sobre os triângulos ABC e DEF são feitas as seguintes afirmativas.
13. Considere os pontos notáveis de um triângulo, sendo: B I. Se AB = DE, BC = EF e  = D̂ , então, eles são congruentes.
– Baricentro; C – Circuncentro; I – Incentro e O – Ortocentro e as
afirmativas abaixo. II. Se AB = DE, BC = EF e B̂ = E , então, eles são congruentes.
( ) Ponto de encontro das medianas.  e BC > AB, então, eles são congruentes.
III. Se AB = DE, BC = EF, Â = D
( ) Ponto de encontro das mediatrizes dos lados de um triângulo. Qual(is) a(s) afirmação(ões) verdadeira(s)?
( ) Ponto de encontro das bissetrizes internas de um triângulo. a) Apenas II.
( ) Ponto de encontro das retas suportes das alturas. b) Apenas I e II.
( ) Ponto que divide cada mediana numa razão de 2 para 1. c) Apenas II e III.
( ) Centro da circunferência inscrita num triângulo. d) Apenas I e III.
( ) Centro da circunferência circunscrita a um triângulo.
( ) Ponto do plano de um triângulo e equidistante dos vértices desse 19. (UECE) No triângulo OYZ, o ângulo interno em O é igual a 90
triângulo. graus, o ponto H no lado YZ é o pé da altura traçada do vértice O e
M é o ponto médio do lado YZ. Se Yˆ − 2Zˆ = 10 graus (diferença entre
A quantidade de vezes que as afirmativas foram classificadas com C foi a medida do ângulo interno em Y e duas vezes a medida do ângulo
interno em Z igual a 10 graus), então, é correto afirmar que a medida
a) 1 c) 3 e) 5 ˆ
do ângulo HOM é igual a
b) 2 d) 4
170 110
a) graus. c) graus.
14. (FGV) A figura representa um triângulo ABC, com E e D sendo 3 3
pontos sobre AC. Sabe-se ainda que AB = AD, CB = CE e que EBDˆ
140 100
mede 39°. b) graus. d) graus.
3 3

Nas condições dadas, a 20. (ITA) Considere o triângulo ABC isósceles em que o ângulo distinto
medida de ABC é dos demais, BÂC, mede 40°. Sobre o lado AB , tome o ponto E tal
a) 102° que ACE = 15°. Sobre o lado AC , tome o ponto D tal que DBC = 35°.
Então, o ângulo EDB vale:
b) 108°
a) 35° d) 75°
c) 111°
b) 45° e) 85°
d) 115°
c) 55°
e) 117°

273

PM_BOOK16 - MAT.indb 273 25/11/2022 19:16:09


GEOMETRIA PLANA: TRIÂNGULOS, CONGRUÊNCIA E PONTOS NOTÁVEIS

21. (CMRJ) Os triângulos ABC e ABD da figura são isósceles com 25. Um triângulo ABC é equilátero, e, sobre o prolongamento de BC,
AB = AC = BD. Seja E o ponto de interseção de BD com AC. Se BD é = 20° , com B entre P e C. Sobre o
marca-se P, tal que o ângulo AP̂B
.
perpendicular a AC, então a soma dos ângulos Ĉe D segmento AP, toma-se Q, tal que o triângulo QPC é isósceles. Calcule
a medida do ângulo BQ̂C:
a) 20°.
b) 30°.
c) 40°.
d) 50°.
e) 60°.
a) 115°
26. As medidas dos ângulos do triângulo ABC são tais que
b) 120°  cortam
Aˆ < Bˆ < 90 < Cˆ . As bissetrizes externas dos ângulos  e C
c) 130°
os prolongamentos dos lados opostos BC e AB nos pontos P e Q,
d) 135° respectivamente. Sabendo que AP = CQ = AC , determine o ângulo
e) 140° de B̂.
a) 12°
22. (CMRJ) No triângulo ABC da figura abaixo, os pontos D e M
pertencem, respectivamente, aos lados AC e BC. Sabe-se que AB = b) 24°
BD, que o ângulo DBCˆ = 48 e que ABD
ˆ = MAC
ˆ = BCAˆ = α. c) 30°
d) 36°
e) 60°

27. Seja um triângulo ABC, onde as alturas AP, BQ e CR se interceptam


no ponto H interno ao triângulo. Sabendo-se que H é o ponto médio
de AP e que CH é o dobro de HR, pode-se afirmar que a medida do
ângulo ABCˆ é
ˆ
a) O triplo da medida de ACR.
Nestas condições, podemos afirmar que a medida do menor ângulo ˆ
b) O dobro da medida de CAP.
formado pelas retas AM e BD é igual a ˆ
c) Um terço da medida de AHC.
a) 60°
ˆ
d) Metade da medida de BAC.
b) 76°
ˆ
e) O dobro da medida de ABH.
c) 78°
d) 81°
28. Sabendo-se que dois ângulos internos do triângulo formado pelos pés
e) 86° das alturas do triângulo ABC acutângulo são 22° e 78°, pode-se afirmar
que a medida do maior ângulo externo do triângulo ABC pode ser
23. ABE é um triângulo equilátero construído exteriormente ao a) 130°
triângulo isósceles ABC, em que AB = AC. Calcule a medida do ângulo
BĈE : b) 128°

a) 20°. c) 170°

b) 30°. d) 139°

c) 45°. e) 141°

d) 60°.
29. No triângulo ABC, tem-se B̂= 2 ⋅ Cˆ e a bissetriz interna traçada a
24. (OBM) Na figura, AB = AC, AE = AD e o ângulo BÂD mede 30°. partir do vértice A intersecta BC em um ponto D tal que AB = CD . O
Então, o ângulo x mede: valor do ângulo  é
a) 30°
b) 36°
c) 48°
d) 60°
e) 72°

30. Em um triângulo ABC, tem-se AB = AC e BAC ˆ = α . Seja P ≠ B


um ponto sobre AB e Q um ponto sobre a altura traçada a partir do
ˆ em função de α é dado
vértice A tal que PQ = QC . O ângulo QPC
a) 10°. por
b) 20°. a) 2α α
d)
c) 15°. 3α 2
b)
d) 30°. 2 e) ˆ não depende de α
QPC
e) 5°. c) α

274

PM_BOOK16 - MAT.indb 274 25/11/2022 19:16:10


GEOMETRIA PLANA: TRIÂNGULOS, CONGRUÊNCIA E PONTOS NOTÁVEIS

EXERCÍCIOS DE 07. (CN) Em um triângulo acutângulo não equilátero, os três pontos

COMBATE notáveis (ortocentro, circuncentro e baricentro) estão alinhados. Dado


que a distância entre o ortocentro e o circuncentro é ‘k’, pode-se
concluir que a distância entre o circuncentro e o baricentro será
5k 4k e) k
a) c)
01. (EPCAR) Considere as retas r e s (r || s) e os ângulos ê , î e â da 2 5 3
figura abaixo
4k d) k
b)
3 2

ˆ = 90º , BC = 2 ⋅ BM =10 cm e ED= 2 ⋅ EN . Se


08. Na figura CAB
DP = 15 cm e AN = 7,5 cm , calcule MP .

Pode-se afirmar que:

a) eˆ + ˆi + aˆ =270 c) eˆ + ˆi = aˆ
b) eˆ + ˆi + aˆ =180 d) ˆe + ˆi + aˆ =90

02. Seja o triângulo isósceles ABC de vértice A.


Sabendo que os segmentos BC, CD, DE, EF e FA
são congruentes, o ângulo do vértice do triângulo
a) 4 cm c) 5 cm e) 6 cm
é igual a:
b) 4,5 cm d) 5,5 cm

a) 10º. 09. (CN) Sabe-se que o ortocentro H de um triângulo ABC é interior


b) 15º. ao triângulo, e seja Q o pé da altura relativa ao lado AC. Prolongando
BQ até o ponto P sobre a circunferência circunscrita ao triângulo,
c) 18º.
sabendo-se que BQ = 12 e HQ = 4, qual é o valor de QP?
d) 20º.
a) 8 d) 4,5
e) 22,5º.
b) 6 e) 4
c) 5,5

10. (CN) O ponto P interno ao triângulo ABC é equidistante de dois


03. (CN) Considere um quadrado ABCD e dois triângulos equiláteros de seus lados e dois de seus vértices. Certamente P é a interseção de:
ABP e BCQ, respectivamente, interno e externo ao quadrado. A soma
ˆ BQP ˆ é igual a:
ˆ e DPQ a) uma bissetriz interna e uma altura desse triângulo.
das medidas dos ângulos ADP,
b) uma bissetriz interna e uma mediatriz dos lados desse triângulo.
a) 270º. d) 360º.
c) uma mediatriz de um lado e uma mediana desse triângulo.
b) 300º. e) 390º.
d) uma altura e uma mediana desse triângulo.
c) 330º.
e) uma mediana e uma bissetriz interna desse triângulo.
04. (CN) Seja ABCD um quadrilátero qualquer onde os lados opostos
NÃO são paralelos. Se as medidas dos lados opostos AB e DC são,
respectivamente, iguais a 12 e 16, um valor possível para o segmento

DESAFIO PRO
de extremos M (ponto médio do lado AD) e N (ponto médio do lado
BC) é:
a) 12,5. d) 16.
b) 14. e) 17.
c) 14,5.

05. Em um triângulo ABC, tem-se AB = BC e B̂ = 20 . Sobre AB, 


1 Segundo a figura, BC = EG e AM = ME, calcule x:

ˆ = 60 e sobre BC, o ponto N tal que a) 70°


toma-se o ponto M tal que MCA
ˆ
NÂC = 50º. O ângulo NMC mede: b) 75°
a) 10º. c) 30º. e) 50º. c) 80°
b) 20º. d) 40º. d) 85°
e) 65°
06. (CN 1997) Quantos triângulos obtusângulos existem, cujos lados
são expressos por números inteiros consecutivos?
a) um c) três e) cinco
b) dois d) quatro

275

PM_BOOK16 - MAT.indb 275 25/11/2022 19:16:11


GEOMETRIA PLANA: TRIÂNGULOS, CONGRUÊNCIA E PONTOS NOTÁVEIS

2 No interior de um triângulo ABC se marca o ponto D tal


que BD = AC. Se ABC = BAD = BCD o valor de BAD será: ANOTAÇÕES
a) 30°. d) 20°.
b) 37°. e) 45°.
c) 53°.

3 Da figura DL = 4 e ADC = 90°, o valor de BC é:

a) 4.
b) 2.
c) 6.
d) 8.
e) 12.

4 Em um triângulo ABC, em AC se marcam os pontos


consecutivos M, Q e N, tal que AM = NC. Se Q é ponto
médio de MN e NBC = ABM = 20°. A medida do ângulo BQC é:
a) 40°.
b) 80°.
c) 70°.
d) 90°.
e) 100°.

5 Em um quadrilátero ABCD, em que ABC = ADC = 90°,


ACD = 2(ACB) e AB = 2, o valor inteiro de AD é:
a) 4.
b) 2.
c) 3.
d) 6.
e) 5.

GABARITO
EXERCÍCIOS DE FIXAÇÃO
01. D 04. E 07. B 10. C
02. D 05. D 08. C
03. B 06. B 09. C
EXERCÍCIOS DE TREINAMENTO
01. A 09. E 17. C 25. B
02. A 10. C 18. C 26. D
03. E 11. B 19. C 27. C
04. A 12. C 20. D 28. A
05. B 13. C 21. D 29. E
06. C 14. A 22. B 30. D
07. A 15. D 23. B
08. D 16. B 24. C
EXERCÍCIOS DE COMBATE
01. A 04. A 07. E 10. B
02. B 05. C 08. C
03. B 06. A 09. E
DESAFIO PRO
01. A 03. D 05. C
02. E 04. D

276

PM_BOOK16 - MAT.indb 276 25/11/2022 19:16:11


GEOMETRIA PLANA:
QUADRILÁTEROS E POLÍGONOS

QUADRILÁTEROS Os quadriláteros planos não entrecruzados são chamados


trapezoides.
Um quadrilátero é um polígono de quatro lados.
A D D

B Somente os quadriláteros convexos serão alvos do nosso estudo.


B
Chamamos de quadrilátero todo polígono que possui quatro lados.
C C
Quadrilátero convexo Quadrilátero côncavo

O quadrilátero é convexo quando as retas suportes de todos os


lados não cruzam o interior do quadrilátero. Quando o quadrilátero é
não convexo ou côncavo existe pelo menos uma reta suporte de um
dos lados que cruza a parte interna do quadrilátero.
A

Elementos:
Vértices: A, B, C e D.
Lados: AB, BC, CD e AD.
Diagonais: AC e BD, possuem sempre 2 diagonais
Ângulos internos: a soma dos ângulos internos vale sempre 360°.
C Ângulos externos: a soma dos ângulos externos vale sempre 360°.
B

Podemos perceber que a soma dos ângulos internos de qualquer


quadrilátero convexo é 360° quando “partimos” o quadrilátero em 2
D triângulos.

Quadrilátero côncavo

Assim temos por exemplo os triângulos ABD e BCD cada um com


180° de soma dos ângulos internos, fazendo com que o quadrilátero
tenha no total 360°.
D
Quadrilátero convexo

277

PM_BOOK16 - MAT.indb 277 25/11/2022 19:16:13


GEOMETRIA PLANA: QUADRILÁTEROS E POLÍGONOS

TRAPÉZIOS
Um quadrilátero plano convexo é um trapézio se, e somente se,
possui dois lados paralelos.

#ABCD é um trapézio

AB  CD ∨ AD  BC

AB  CD

Os lados paralelos são chamados bases do trapézio. Todo trapézio isósceles é inscritível em uma circunferência. Os
Os ângulos adjacentes a um mesmo lado não paralelo de lados paralelos são cordas paralelas dessa circunferência e os lados
um trapézio são suplementares (ângulos colaterais internos). não paralelos cordas que determinam arcos de mesma medida.

BASE MÉDIA DO TRIÂNGULO


Se um segmento tem extremidades nos pontos médios de dois
lados de um triângulo, então ele é paralelo ao terceiro lado e é igual à
AB  CD
metade do terceiro lado. Esse segmento é denominado base média
do triângulo, relativa ao terceiro lado.

A distância entre os lados paralelos é chamada altura do trapézio (h).


Se um trapézio possui dois ângulos retos adjacentes a um dos
lados não paralelos, ele é chamado trapézio retângulo.
D C

Se um segmento paralelo a um lado de um triângulo tem uma


extremidade no ponto médio de um lado e a outra extremidade no
terceiro lado, então esta extremidade é o ponto médio do terceiro lado.
A B
BASE MÉDIA DO TRAPÉZIO
Se os lados não paralelos de um trapézio não são congruentes, ele A base média de um trapézio é o segmento de reta que une os
é chamado trapézio escaleno. pontos médios dos lados não paralelos.
A B

C D

TRAPÉZIO ISÓSCELES AM ≡ MD e BN ≡ NC
Se os lados não paralelos de um trapézio são congruentes, ele é ⇓
chamado trapézio isósceles.
B+b
Os ângulos adjacentes às bases de um trapézio isósceles são MB  AB  CD e MN =
2
congruentes.
As diagonais de um trapézio isósceles são congruentes.

278

PM_BOOK16 - MAT.indb 278 25/11/2022 19:16:15


GEOMETRIA PLANA: QUADRILÁTEROS E POLÍGONOS

A base média de um trapézio é paralela às bases e igual à


semissoma das bases.
Se um segmento paralelo às bases de um trapézio tem uma
extremidade no ponto médio de um dos lados não paralelos e a outra
extremidade sobre o outro lado não paralelo, então esta extremidade
é o ponto médio deste lado.

MEDIANA DE EULER
A mediana de Euler de um trapézio é o segmento de reta que une Â ≡ Cˆ
os pontos médios das diagonais do trapézio. 
Bˆ ≡ Dˆ
A mediana de Euler está sobre a base média do trapézio e é AB  CD ∧ AD  BC ⇒ 
igual à semidiferença das bases. AB ≡ CD

AD ≡ BC

As diagonais de um paralelogramo intersectam-se ao meio.

AP ≡ PC e CQ ≡ QD

AB − CD B − b Todo quadrilátero convexo que possui dois lados paralelos e
PQ ⊂ MN, PQ  AB  CD e Me =
PQ = =congruentes é um paralelogramo.
2 2

RETÂNGULO
PARALELOGRAMO Um quadrilátero plano convexo é um retângulo se, e somente se,
Um quadrilátero plano convexo é um paralelogramo se, e somente possui os quatro ângulos congruentes (quadrilátero equiângulo).
se, possui lados opostos paralelos.
D C
# ABCD é um retângulo

ˆ ˆ ˆ
A= B= C= Dˆ= 90º

Os retângulos são paralelogramos, pois possuem ângulos


A B adjacentes congruentes. Assim, os retângulos possuem todas as
#ABCD é um paralelogramo propriedades dos paralelogramos.
 • As diagonais de um retângulo são congruentes.
AD  BC ∧ AB  CD • Todo paralelogramo que possui diagonais congruentes é um
retângulo.
• Em um paralelogramo, dois ângulos opostos quaisquer são • ABCD é um paralelogramo ⇒  + B = 180° ⇒ A
A  = B = 90° = C
 =D
⇒
congruentes.   °   °  
A + B = 180 ⇒ A = B = 90 = C = D ⇒ # ABCD é um retângulo
• Todo quadrilátero convexo que possui ângulos opostos • Todo retângulo é circunscritível e o ponto de concurso das
congruentes é um paralelogramo. diagonais é o seu circuncentro.
• Em um paralelogramo, dois lados opostos quaisquer são
congruentes.
• Todo quadrilátero convexo que possui lados opostos
congruentes é um paralelogramo.
• Como os ângulos opostos de ABCD são congruentes, então
ABCD é um paralelogramo.

279

PM_BOOK16 - MAT.indb 279 25/11/2022 19:16:16


GEOMETRIA PLANA: QUADRILÁTEROS E POLÍGONOS

• Os lados opostos do retângulo são cordas paralelas da • Todo losango é inscritível e o ponto de concurso das diagonais
circunferência circunscrita e suas diagonais são diâmetros é o seu incentro.
dessa circunferência.

LOSANGO
Um quadrilátero plano convexo é um losango se, e somente se,
possui os quatro lados congruentes (quadrilátero equilátero).

# ABCD é um losango

AB ≡ BC ≡ CD ≡ DA

QUADRADO
Um quadrilátero plano convexo é um quadrado se, e somente se,
possui quatro lados congruentes e quatro ângulos congruentes, ou
seja, o quadrado é o quadrilátero regular (equilátero e equiângulo).

Os losangos são paralelogramos, pois possuem lados opostos


congruentes. Assim, os losangos possuem todas as propriedades dos
paralelogramos.
• Todo losango possui diagonais perpendiculares.

AB ≡ BC ≡ CD ≡ DA

# ABCD é um quadrado ⇔  e
ˆ ˆ ˆ ˆ
 A= B= C= D= 90º

O quadrado é equilátero e equiângulo, portanto, possui


todas as propriedades dos losangos, dos retângulos e também dos
paralelogramos.
• Todo paralelogramo que tem diagonais perpendiculares é um • As diagonais de um quadrado cortam-se ao meio
losango. perpendicularmente, são iguais e são bissetrizes dos ângulos
internos.
• ∆AMB ≡ ∆AMD (L.A.L.) ⇒ AB ≡ AD ≡ CD ≡ BC ⇒ # ABCD é um losango
losango.
• As diagonais são bissetrizes dos ângulos internos do losango.

• Todo quadrado é inscritível e circunscritível, e o ponto de


concurso das diagonais é o seu centro.

280

PM_BOOK16 - MAT.indb 280 25/11/2022 19:16:16


GEOMETRIA PLANA: QUADRILÁTEROS E POLÍGONOS

n = 15 pentadecágono

n = 20 icoságono

n n-látero

Um polígono é simples se, e somente se, a interseção de


quaisquer dois lados não consecutivos é vazia.
Um polígono simples é convexo se, e somente se, a reta
determinada por quaisquer dois de seus vértices consecutivos deixa
todos os outros vértices no mesmo semiplano.
Um polígono que possui todos os lados congruentes é dito
equilátero. Um polígono que possui todos os ângulos congruentes
é dito equiângulo. Um polígono é dito regular se é equilátero e
equiângulo.
O desenvolvimento a seguir refere-se a polígonos simples que
O diâmetro do círculo inscrito no quadrado é igual ao lado do
possuem todos os vértices no mesmo plano.
quadrado e o diâmetro do círculo circunscrito é igual à sua diagonal.
A A D

DEFINIÇÃO B D
Seja uma sequência de pontos distintos A1,A2, A3, ..., AN, com
n ≥ 3, onde três pontos consecutivos não são colineares (AN–1, AN, A1 B C
C
e An, A1, A2 são considerados consecutivos). A reunião dos segmentos
Quadrilátero equilátero Quadrilátero equiângulo
A1A 2 , A 2A 3 , ..., An−1An , An A1 é o polígono A1A2...AN.
A D
Os pontos A1,A 2 ,A 3 , ,An são os vértices do polígono; os A
segmentos A1A 2 , A 2A 3 , ..., An−1An , An A1 são os seus lados; e os
ângulos Aˆ 1 = An Aˆ 1A 2, Aˆ 2 = A1Aˆ 2A 3, ..., Aˆ n = An−1Aˆ n A1 são os ângulos
internos do polígono.

B C B C
O triângulo regular é o O quadrilátero regular
triângulo equilátero é o quadrado

Um polígono de n vértices possui também n lados e n ângulos, e Hexágono Hexágono Hexágono regular
diz-se de gênero n. equilátero equiângulo

GÊNERO DENOMINAÇÃO

n=3 trilátero ou triângulo


DIAGONAIS
Diagonal de um polígono é um segmento cujas extremidades são
n=4 quadrilátero dois vértices não consecutivos do polígono.

n=5 pentágono

n=6 hexágono

n=7 heptágono

n=8 octógono

n=9 eneágono

n = 10 decágono

n = 11 undecágono

n = 12 dodecágono O número de diagonais que se pode traçar partindo-se de cada


vértice de um polígono de gênero n é: d = n – 3.

281

PM_BOOK16 - MAT.indb 281 25/11/2022 19:16:17


GEOMETRIA PLANA: QUADRILÁTEROS E POLÍGONOS

O número total de diagonais de um polígono de gênero n é: POLÍGONOS REGULARES


n ( n − 3) Um polígono regular é um polígono equilátero e equiângulo.
D= .
2 Os polígonos regulares são inscritíveis e circunscritíveis. O centro
das circunferências inscrita e circunscrita é chamado centro do
Demonstração: polígono.

Basta contar a quantidade de diagonais traçadas em todos os n


vértices, totalizando n (n – 3) e observar que cada diagonal é contada Cada ângulo interno de um polígono regular é igual a
duas vezes, uma em cada extremidade. Portanto, o número de Si 180 (n − 2) Se 360
Â=i = e cada ângulo externo é igual a Â= = .
n ( n − 3) n n e
n n
diagonais é D = .
2
ÂNGULO CENTRAL
ÂNGULOS INTERNOS Imagine o triângulo formado pelos raios da circunferência
A soma dos ângulos internos de um polígono de gênero n é: circunscrita ao polígono regular e um dos lados do polígono, como
na figura abaixo.
=Si 180 (n − 2).

Demonstração:

O ângulo central α é o ângulo do vértice do triângulo que coincide


com o centro do polígono.
360
O ângulo central de um polígono regular é igual a  C = .
n

Ligando-se um dos vértices de um polígono gênero n aos (n − 3) APÓTEMA DE UM POLÍGONO


vértices não adjacentes a ele, o polígono fica dividido em (n − 2) O apótema de um polígono regular é a distância do centro do
triângulos. A soma dos ângulos internos desses (n − 2) triângulos é polígono a qualquer um dos lados.
igual à soma dos ângulos internos do polígono. Assim, a soma dos
=
ângulos internos do polígono é Si 180 (n − 2).

ÂNGULOS EXTERNOS
Ângulo externo de um polígono é o ângulo suplementar adjacente
do ângulo interno do polígono.
A soma dos ângulos externos de um polígono de gênero n é:
Se = 360 .

Demonstração:
Há n pares de ângulos internos e externos, e cada par soma 180º.
Dessa forma, podemos perceber que sempre que traçamos o
Assim, a soma de todos os ângulos internos e externos do polígono é
 apótema temas o triângulo abaixo formado.
180 ⋅ n. Portanto,
Si + Se= 180 ⋅ n ⇔ 180 (n − 2) + Se= 180 n ⇔ Se= 360
Observe que a soma dos ângulos externos de um polígono é
constante e não depende de seu gênero.

282

PM_BOOK16 - MAT.indb 282 25/11/2022 19:16:18


GEOMETRIA PLANA: QUADRILÁTEROS E POLÍGONOS

Exercício Resolvido

03. Sabendo que x é a medida da base maior, y é a medida da


base menor, 5,5 cm é a medida da base média de um trapézio e
que x - y = 5 cm, determine as medidas de x e y.

Resolução:
x + y
 = 5,5 x + y =
11
 2 ⇒
 x − y =  x−y = 5
Dessa forma por termos sempre um triângulo isósceles, teremos que 5

Somando as equações
2x = 16 ⇒ x = 8
x + y = 11 ⇒ 8 + y = 11⇒ y = 3

Exercício Resolvido

04. Quantos lados têm um polígono cujo número total de


diagonais é igual ao quádruplo do seu número de vértices?
E assim sendo, o apótema a de qualquer polígono regular é a) 10
perpendicular ao lado do polígono dividindo-o ao meio.
b) 11
DIAGONAIS QUE PASSAM PELO CENTRO c) 13
n d) 9
Um polígono regular de gênero n par possui dc = diagonais que
( ) 2
n n−4 Resolução: B
passam pelo centro e dnc = diagonais que não passam pelo
centro. 2 nvértices = nlados
Um polígono regular de gênero ímpar não possui diagonais que n ⋅ (nlados − 3) 2
D = lados = 4nlados → ( nlados ) − 3nlados = 8nlados
passam pelo centro. 2

Exercício Resolvido (nlados )2 − 11nlados =


0
nlados = 11
01. Em um losango, um de seus ângulos internos é 55º. Quais são
as medidas dos demais ângulos internos?

Resolução: Exercício Resolvido


° ° °
55 + 55 + x + x =360 04. Quantos lados têm um polígono cuja soma dos ângulos
2x 360° − 110°
= internos e externos é 1980º?
2x = 250° a) 8
x = 125 ° b) 11
c) 13
d) 17
Exercício Resolvido
Resolução: B
02. No retângulo abaixo, determine as medidas de x e y indicadas:
Se = 360°
Si = (n − 2) ⋅ 180° → 1980° − 360° = (n − 2) ⋅ 180°
1980 − 360 = 180n − 360 → 180n = 1980 →= n 11

Resolução:
EXERCÍCIOS DE

FIXAÇÃO
12x + 2°+ 5x + 3=° 90°
17x + 5=° 90°
17x= 90°− 5°
= 85°
17x
01. (FAETEC) Ao traçarmos as duas diagonais de um losango sempre
85°
x= = 5° obteremos quatro triângulos classificados como
17
a) escalenos d) acutângulos
y = 5x + 3°
b) retângulos e) obtusângulos
y= 5 (5° ) + 3° c) equiláteros

283

PM_BOOK16 - MAT.indb 283 25/11/2022 19:16:19


GEOMETRIA PLANA: QUADRILÁTEROS E POLÍGONOS

02. (FAETEC) A soma das medidas dos ângulos internos de um EXERCÍCIOS DE

TREINAMENTO
polígono convexo é 1080º. Esse polígono denomina-se:
a) hexágono c) octógono e) decágono
b) heptágono d) eneágono

03. (FAETEC) Os ângulos internos de um polígono regular medem 01. (CFT) Considere um trapézio onde a base maior mede o dobro
170° cada um. Esse polígono possui o seguinte número de lados: da base menor. Se a base média desse trapézio tem 18 cm, então sua
base maior, em cm, mede
a) 36 b) 24 c) 18 d) 12 e) 10
a) 18. b) 20. c) 24. d) 38.
04. (CFN) Determine a medida do ângulo formado por dois lados
consecutivos de um hexágono regular. 02. (EEAR) A metade da medida do ângulo interno de um octógono
regular, em graus, é
a) 90º b) 120º c) 150º d) 155º e) 168º
a) 67,5 b) 78,6 c) 120 d) 85
05. (CFT) Considerando que ABCD é um paralelogramo, que M é o
ponto de encontro de suas diagonais, e que as medidas das distâncias 03. (EEAR) Um trapézio de bases x + 3 e 4x–3, tem base média
de seus vértices ao ponto M são dadas, tem-se que o valor de x + y é 2x + 2. A menor base mede
a) 7. b) 8. c) 9. d) 10.

04. (EEAR) No trapézio ACDF abaixo, considere AB = BC e DE = EF .


Assim, o valor de x² é

a) 5. b) 6. c) 7. d) 8.

06. (CFN) Qual o polígono cuja soma das medidas dos ângulos
internos é 900°?
a) 1 b) 4 c) 9 d) 16
a) Hexágono d) Pentadecágono
b) Heptágono e) Icoságono 05. (EAM) Observe a representação abaixo.
c) Octógono

07. (EAM) A partir de um dos vértices de um polígono convexo pode-


se traçar tantas diagonais quantas são o total de diagonais de um
pentágono. É correto afirmar que esse polígono é um:
a) Hexágono. d) Decágono.
b) Heptágono e) Dodecágono. No paralelogramo PQRS, PS = ST, e o ângulo PQR mede 56°, conforme
c) Octógono. mostra a figura. A medida do ângulo ST̂P , em graus, é:
a) 59 b) 60 c) 61 d) 62 e) 64
08. (UERJ) “Se um polígono tem todos os lados iguais, então todos os
seus ângulos internos são iguais.” Para mostrar que essa proposição é
falsa, pode-se usar como exemplo a figura denominada 06. (EEAR) Ao somar o número de diagonais e o número de lados de
um dodecágono obtém-se
a) losango. c) retângulo.
a) 66 b) 56 c) 44 d) 42
b) trapézio. d) quadrado.
07. (EEAR) Se A é o número de diagonais de um icoságono e B o
09. (EEAR) Seja ABCD o trapézio isósceles da figura. A soma das número de diagonais de um decágono, então A – B é igual a
medidas dos ângulos  e Ĉ é a) 85 b) 135 c) 165 d) 175

08. (CMRJ) A diferença entre as medidas do ângulo interno e do


ângulo externo de um polígono regular vale 144°. O número de lados
deste polígono é igual a:
a) 18 b) 20 c) 22 d) 24 e) 26
a) 90°. b) 120°. c) 150°. d) 180°.
09. (CMRJ) A diferença entre o número de lados de dois polígonos
é sete, e a soma de todos os ângulos internos dos dois polígonos é
10. (EEAR) Seja o paralelogramo ABCD. Sabendo que AP e DP são 4140°. O que tem menos vértices é um
bissetrizes dos ângulos internos  e D̂ , respectivamente, o valor de x é a) heptágono d) eneágono
b) icoságono e) octógono
c) decágono

10. (EEAR) O polígono regular cujo ângulo externo mede 24° tem
_____ lados.
a) 55° b) 45° c) 30° d) 15° a) 20 b) 15 c) 10 d) 5

284

PM_BOOK16 - MAT.indb 284 25/11/2022 19:16:19


GEOMETRIA PLANA: QUADRILÁTEROS E POLÍGONOS

11. (ESA) Os ângulos internos de um quadrilátero são inversamente 17. (ITA) Dadas as afirmações:
proporcionais aos números 2, 3, 4 e 5. O maior ângulo interno desse I. Quaisquer dois ângulos opostos de um quadrilátero são
quadrilátero mede, aproximadamente suplementares.
a) 140° d) 110° II. Quaisquer dois ângulos consecutivos de um paralelogramo são
b) 130° e) 100° suplementares.
c) 120° III. Se as diagonais de um paralelogramo são perpendiculares entre
si e se cruzam em seu ponto médio, então este paralelogramo é
12. Analise as afirmativas, marque V para as verdadeiras e F para as um losango.
falsas. Podemos garantir que:
( ) Em um paralelogramo, os ângulos opostos são congruentes. a) Todas são verdadeiras d) Apenas II é verdadeira
( ) Todo paralelogramo é um quadrado. b) Apenas I e II são verdadeiras e) Apenas III é verdadeira
( ) O losango é considerado um paralelogramo. c) Apenas II e III são verdadeiras
A sequência está correta em
a) F, F, V. b) V, V, F. c) V, F, V. d) V, V, V. 18. (ITA) O número de diagonais de um polígono regular de 2n lados,
que não passam pelo centro da circunferência circunscrita a este
13. Analise as afirmativas a seguir. polígono, é dado por:
I. Um ângulo agudo e um ângulo obtuso de um paralelogramo a) 2n(n - 2) c) 2n(n - 3) n (n − 5)
e)
sempre são complementares. n (n − 5) 2
b) 2n(n - 1) d)
II. Toda propriedade do losango vale para o quadrado. 2
III. Toda propriedade do losango vale para o paralelogramo.
19. (ITA) A soma das medidas dos ângulos internos de um polígono
IV. O quadrado tem as propriedades do paralelogramo, do retângulo regular é 2160°. Então o número de diagonais deste polígono, que
e do losango. não passam pelo centro da circunferência que o circunscreve, é:
Estão corretas apenas as afirmativas a) 50 b) 60 c) 70 d) 80 e) 90
a) I e II. b) I e IV. c) II e III. d) II e IV.
20. (ITA) Considere as afirmações sobre polígonos convexos:
14. Dadas as afirmativas a respeito de quadriláteros. I. Existe apenas um polígono cujo número de diagonais coincide
I. Um trapézio também é um paralelogramo. com o número de lados.
II. Um quadrado também é um retângulo. II. Não existe polígono cujo número de diagonais seja o quádruplo
III. Um retângulo também é um losango. do número de lados.
Verifica-se que está(ão) correta(s) III. Se a razão entre o número de diagonais e o de lados de um
polígono é um número natural, então o número de lados do
a) I, apenas. d) I e III, apenas. polígono é ímpar.
b) II, apenas. e) I, II e III.
a) Todas as afirmações são verdadeiras.
c) III, apenas.
b) Apenas (I) e (III) são verdadeiras.
15. (CBMRN) Analise as afirmativas a seguir. c) Apenas (I) é verdadeira.
I. Se um trapézio tem um ângulo externo reto, ele é trapézio d) Apenas (III) é verdadeira.
retângulo. e) Apenas (II) e (III) são verdadeiras.
II. Toda propriedade do paralelogramo vale para o losango.
III. Um ângulo agudo e um ângulo obtuso de um paralelogramo são
suplementares. EXERCÍCIOS DE
IV. O quadrado tem as propriedades do paralelogramo, do retângulo

V.
e do losango.
Dois ângulos consecutivos de um paralelogramo sempre são
COMBATE
congruentes.
VI. Se as diagonais de um paralelogramo são perpendiculares entre si 01. (EPCAR) Seja ABCD um paralelogramo cujos lados AB e BC
e se cruzam em seu ponto médio, então esse paralelogramo não medem, respectivamente, 5 e 10 . Prolongando o lado AB até o
é um losango. ˆ é congruente
ponto P, obtém-se o triângulo APD, cujo ângulo APD
ˆ , conforme a figura.
ao ângulo ACB
Estão INCORRETAS apenas as afirmativas
a) I e VI. c) I, II e III.
b) V e VI. d) II, IV e V.

16. (EN) Quando as diagonais de um paralelogramo são também


bissetrizes dos seus ângulos internos?
a) Só se dois ângulos internos e consecutivos forem complementares.
b) Só se o paralelogramo for um quadrado.
c) Só se o paralelogramo for um retângulo. Então, a medida AP é:
d) Só se o paralelogramo for um losango. 2 10 . 10 .
a) 0,2. b) 2. c) d)
e) Só se a soma dos ângulos internos for 360o. 5 5

285

PM_BOOK16 - MAT.indb 285 25/11/2022 19:16:20


GEOMETRIA PLANA: QUADRILÁTEROS E POLÍGONOS

02. (CN) As bases de um trapézio medem 3 cm e 9 cm. Os segmentos 08. (CN) Um estudante foi calculando o lado de um polígono regular
determinados pelas diagonais do trapézio sobre a base média são de 2n lados, inscrito em uma circunferência de raio 10 centímetros, para
proporcionais aos números: n sucessivamente igual a 6, 12, 24, 48, 96 etc. Após determinar cada
lado, calculou o perímetro p do respectivo polígono, e observou que p é
a) 1,1,1. c) 1,3,1. e) 2,3,4.
um número cada vez mais próximo, porém menor que
b) 1,2,1. d) 1,4,1.
a) 60. d) 63.
03. (CN) Considere as 4 afirmações abaixo. A seguir, coloque (V) ou b) 61. e) 64.
(F) nos parênteses, conforme sejam verdadeiras ou falsas, e assinale a c) 62.
alternativa correta.
1. ( ) Em qualquer trapézio circunscrito a uma circunferência, a 09. (CN) O número de diagonais de um polígono regular P inscrito em
medida da base média é a quarta parte do seu perímetro. um círculo K é 170. Logo,
2. ( ) As diagonais de um trapézio podem se intersectar no seu ponto a) o número de lados de P é ímpar.
médio. b) P não tem diagonais passando pelo centro de K.
3. ( ) Todo quadrilátero que tem as diagonais perpendiculares é um c) o ângulo externo de P mede 36°.
losango.
d) uma das diagonais de P é o lado do pentágono regular inscrito em K.
4. ( ) Existe quadrilátero plano cujos segmentos das diagonais não
se intersectam. e) o número de lados de P é múltiplo de 3.

a) Apenas 2 é verdadeira.
10. (CN) Um polígono convexo de n lados tem três dos seus ângulos
b) Apenas 3 é verdadeira. iguais a 83°, 137° e 142°. Qual é o menor valor de n para que nenhum
c) Apenas 3 e 4 são verdadeiras. dos outros ângulos desse polígono seja menor que 121°?
d) 2, 3 e 4 são verdadeiras. a) 6 d) 9
e) 1 e 4 são verdadeiras. b) 7 e) 10
c) 8
04. (CN) Um retângulo é obtido unindo-se os pontos médios de um
trapézio retângulo ABCD, de bases AB = 32 e CD = 8. A altura BC é
igual a:

DESAFIO PRO
a) 8. c) 12. e) 20.
b) 10. d) 16.

05. (CN) A, B, C e D são vértices consecutivos de um quadrado e PAB


é um triângulo equilátero, sendo P interno ao quadrado ABCD. Qual
é a medida do ângulo PCB? 1 Se na figura abaixo AB = BM = MC, o valor de x é

a) 30º c) 60º e) 90º a) 15°.


b) 45º d) 75º b) 30°.
c) 22° 30’.
06. (CN) Considere as afirmativas abaixo sobre um polígono regular
de n lados, em que o número de diagonais é múltiplo de n. d) 45°.
I. O polígono não pode ter diagonal que passe pelo seu centro. e) 36°.
II. n pode ser múltiplo de 17.
III. n pode ser um cubo perfeito.

2
IV. n pode ser primo. Seja um quadrilátero ABC tal que ADC = 90°, BAC = 30°,
Assinale a alternativa correta. BC = CD e ACD = 3(BCA). Calcule BAC.
a) Todas as afirmativas são falsas. a) 12°
b) Apenas a afirmativa II é verdadeira. b) 14°
c) Apenas as afirmativas II e III são verdadeiras. c) 15°
d) Apenas as afirmativas II, III e IV são verdadeiras. d) 16°
e) Todas as afirmativas são verdadeiras. e) 18°

07. (CN) Os pontos x, o e y são vértices de um polígono regular de n


ˆ mede 22°30’, considere as afirmativas:
lados. Se o ângulo XOY 3 Sejam E o ponto médio do lado CD de um quadrado
ABCD e M um ponto do interior do quadrado tal que
I. n pode ser igual a 8. ∠MAB = ∠MBC =∠BME = x . O valor de x é igual a:

II. n pode ser igual a 12. a) 15°.


III. n pode ser igual a 24. b) 30°.
Podemos afirmar que: c) 45°.
a) apenas I e II são verdadeiras. d) 60°.
b) apenas I e III são verdadeiras. e) 75°.
c) apenas II e III são verdadeiras.
d) apenas uma delas é verdadeira
e) I, II e III são verdadeiras.

286

PM_BOOK16 - MAT.indb 286 25/11/2022 19:16:20


GEOMETRIA PLANA: QUADRILÁTEROS E POLÍGONOS

4 (ITA) Considere as afirmações sobre polígonos convexos:

I. Existe apenas um polígono cujo número de diagonais


coincide com o número de lados.
II. Não existe polígono cujo número de diagonais seja o
quádruplo do número de lados.
III. Se a razão entre o número de diagonais e o de lados de um
polígono é um número natural, então o número de lados do
polígono é ímpar.
Então:
a) Todas as afirmações são verdadeiras.
b) Apenas (I) e (III) são verdadeiras.
c) Apenas (I) é verdadeira.
d) Apenas (III) é verdadeira.
e) Apenas (II) e (III) são verdadeiras.

5 O ângulo interior e o ângulo exterior de um polígono


regular medem θ e (k – 1)θ, respectivamente. Os valores
inteiros k para que o polígono exista são
a) 2 e 3.
b) 1, 2, 3 e 4.
c) 1, 2, 3, 4 e 5.
d) 1 e 2.
e) 1, 2 e 3.

GABARITO
EXERCÍCIOS DE FIXAÇÃO
01. B 04. B 07. C 10. B
02. C 05. B 08. A
03. A 06. B 09. D
EXERCÍCIOS DE TREINAMENTO
01. C 06. A 11. A 16. D
02. A 07. B 12. C 17. C
03. A 08. B 13. D 18. A
04. B 09. C 14. B 19. C
05. D 10. B 15. B 20. B
EXERCÍCIOS DE COMBATE
01. B 04. D 07. B 10. B
02. B 05. D 08. D
03. E 06. E 09. D
DESAFIO PRO
01. D 03. E 05. E
02. C 04. B

ANOTAÇÕES

287

PM_BOOK16 - MAT.indb 287 25/11/2022 19:16:20


GEOMETRIA PLANA: QUADRILÁTEROS E POLÍGONOS

ANOTAÇÕES

288

PM_BOOK16 - MAT.indb 288 25/11/2022 19:16:20


GEOMETRIA PLANA:
CIRCUNFERÊNCIA E CÍRCULO

ELEMENTOS E DEFINIÇÕES • Corda de uma circunferência é um segmento cujas


extremidades pertencem à circunferência.
• Circunferência é o lugar geométrico dos pontos do plano
cujas distâncias a um ponto fixo (centro) são iguais a uma • Raio de uma circunferência é um segmento que possui uma
constante (raio). extremidade no centro e outra sobre a circunferência e que
tem medida constante.

O ponto P pertence à • Diâmetro de uma circunferência é uma corda que passa pelo
circunferência de centro O e seu centro. O diâmetro é a maior corda da circunferência e
raio r sua medida é o dobro da do raio.
Dados dois pontos A e B sobre uma circunferência de centro O, o
  é a reunião dos pontos A e B com o conjunto
arco de circunferência AB
OP = r de todos os pontos sobre a circunferência interiores ao ângulo AOBˆ
. Na verdade, dois pontos sobre uma circunferência determinam dois
arcos, em geral denominados arco menor AB  e arco maior AB.

Seja a circunferência λ de centro O e raio r, então:
• Círculo (disco) é o lugar geométrico dos pontos do plano
cujas distâncias a um ponto fixo (centro) são menores ou
iguais a uma constante (raio).
OP = r é um raio
CD = 2r é um diâmetro
O ponto P pertence ao círculo
de centro O e raio r AB é uma corda
  menor é um arco de circunferência
AB
OP ≤ r

• Semicircunferência é um arco de circunferência determinado


por pontos diametralmente opostos.

Menor arco limitado por A e B


Corda
diâmetro
Flecha
raio
Setor Circular

Segmento Circular

Observação
a. Todo e qualquer raio perpendicular a uma corda AB divide
esta em duas partes de mesma medida.
Nota: Muitas vezes as expressões circunferência e círculo são
usadas indistintamente, ora para representar a borda da figura, ora
para representar a união da borda e do interior.

289

PM_BOOK16 - MAT.indb 289 25/11/2022 19:16:22


GEOMETRIA PLANA: CIRCUNFERÊNCIA E CÍRCULO

b. Duas cordas, de mesma medida (congruentes), possuem


mesma distância, em relação ao centro da circunferência.

POSIÇÕES RELATIVAS ENTRE PONTO POSIÇÕES RELATIVAS ENTRE RETA E


E CIRCUNFERÊNCIA CIRCUNFERÊNCIA

O ponto B pertence ao interior da circunferência de centro


O e raio r se, e somente se, a distância desse ponto ao centro da
circunferência é menor do que o raio: OB < r .
O ponto P pertence à circunferência de centro O e raio r se, e
somente se, a distância desse ponto ao centro da circunferência é
igual ao raio: OP = r.
A reta s é secante à circunferência de centro O e raio r se, e
O ponto A pertence ao exterior da circunferência de centro somente se, a distância do centro da circunferência à reta é menor do
O e raio r se, e somente se, a distância desse ponto ao centro da que o raio: d(O,s) < r.
circunferência é maior do que o raio: OA > r.
A reta t é tangente à circunferência de centro O e raio r se, e
somente se, a distância do centro da circunferência à reta é igual ao
Exemplo: raio: d(O,t) = r.
Seja uma circunferência de centro O e raio r = 3. Identifique a A reta u é exterior à circunferência de centro O e raio r se, e
posição relativa entre os pontos A, B, C e a circunferência, sabendo-se somente se, a distância do centro da circunferência à reta é maior do
que OA = 2, OB = 3 e OC = 4. que o raio: d(O,u) > r.

Resolução: Exemplo:
O ponto A é interior à circunferência, pois OA = 2 < 3 = r . Seja uma circunferência de centro O e raio R = 3. Identifique
a posição relativa entre as retas r, t, s e a circunferência, sabendo-
O ponto B pertence à circunferência, pois OB= 3= r. se que as distâncias entre as retas e o centro da circunferência são,
O ponto C é exterior à circunferência, pois OC = 4 > 3 = r. respectivamente, d(O,r) = 2, d(O,t) = 3 e d(O,s) = 4.
Veja a figura a seguir, na qual esses pontos estão representados. Resolução:
A reta r é secante à circunferência, pois d(O,r) = 2 < 3 = R.
A reta t é tangente à circunferência, pois d(O,t) = 3 = R.
A reta s é exterior à circunferência, pois d(O,s) = 4 > 3 = r.
Veja a figura a seguir, na qual essas retas estão representadas.

290

PM_BOOK16 - MAT.indb 290 25/11/2022 19:16:22


GEOMETRIA PLANA: CIRCUNFERÊNCIA E CÍRCULO

As circunferências são SECANTES se, e somente se, a distância


entre seus centros é maior do que o módulo da diferença entre seus
raios e menor do que a soma dos raios: |R – r| < d(O,O’) < R + r.

POSIÇÕES RELATIVAS ENTRE


CIRCUNFERÊNCIAS
Sejam duas circunferências de centros O e O’, e raios r e R,
respectivamente.
As circunferências são CONCÊNTRICAS se, e somente se, a
distâncias entre seus centros é nula: d(O,O’) = 0.

As circunferências são TANGENTES EXTERIORES se, e somente se, a


distância entre seus centros é igual à soma dos raios: d(O,O’) = R + r.

As circunferências são INTERIORES se, e somente se, a distância


entre seus centros é maior do que zero e menor do que o módulo da
diferença entre seus raios: 0 < d(O,O’) < |R – r|.
As circunferências são EXTERIORES se, e somente se, a distância
entre seus centros é maior do que a soma dos raios: d(O,O’) > R + r.

As circunferências são TANGENTES INTERIORES se, e somente


se, a distância entre seus centros é igual ao módulo da diferença entre
seus raios: d(O,O’) = |R – r|.

291

PM_BOOK16 - MAT.indb 291 25/11/2022 19:16:24


GEOMETRIA PLANA: CIRCUNFERÊNCIA E CÍRCULO

POSIÇÃO RELATIVA ENTRE AS CIRCUNFERÊNCIAS DISTÂNCIA ENTRE SEUS CENTROS

CONCÊNTRICAS d(O,O’) = 0

INTERIORES 0 < d(O,O’) < |R – r|

TANGENTES INTERIORES d(O,O’) = |R – r|

SECANTES |R – r| < d(O,O’) < R + r

TANGENTES EXTERIORES d(O,O’) = R + r

EXTERIORES d(O,O’) > R + r

Exemplo: PROPRIEDADE DA SECANTE


Sejam duas circunferências λ1 e λ 2 de raios R1 = 2 e R2 = 4 e Seja uma reta s secante a uma circunferência λ de centro O e raio
centros O1 e O2, respectivamente. Identifique a posição relativa entre r, que não passa por O e que intercepta a circunferência nos pontos A
as circunferências em cada um dos casos a seguir: e B distintos. O ponto M é o ponto médio da corda AB se, e somente
a) O1O2 = 1 se, OM ⊥ AB.

b) O1O2 = 2

c) O1O2 = 3

d) O1O2 = 6

e) O1O2 = 9

Resolução:
a) As circunferências são interiores, pois O1O2 =1 < 2 =R2 − R1.
b) As circunferências são tangentes interiores, pois O1O2 = 1= 2= R2 − R1
O1O2 = 1= 2= R2 − R1.
c) As circunferências são secantes, pois R2 − R1 = 2 < O1O2 = 3 < 6 = R1 + R2
R2 − R1 = 2 < O1O2 = 3 < 6 = R1 + R2 .
d) As circunferências são tangentes exteriores, pois O O = 6= R + R Demonstração:
1 2 1 2
O1O2= 6= R1 + R2. Supondo que o ponto M seja ponto médio de AB, então
∆OMA ≡ ∆OMB (L.L.L.), o que implica AMO
ˆ
= ˆ
=
BMO 90.
e) As circunferências são exteriores, pois O1O2 = 9 > 6 = R1 + R2.
Supondo que OM ⊥ AB, então ∆OMA ≡ ∆OMB (OM comum e
Veja a figura a seguir onde foi feita uma representação OA = OB , caso especial de congruência para triângulos retângulos),
esquemática das cinco situações. o que implica AM = MB.

Exemplo:
Calcule o comprimento de uma corda que dista 3 cm do centro de
uma circunferência de raio 5 cm.
Resolução:

292

PM_BOOK16 - MAT.indb 292 25/11/2022 19:16:25


GEOMETRIA PLANA: CIRCUNFERÊNCIA E CÍRCULO

Seja AB a corda em questão e OM ⊥ AB, então OM = 3 e M é SEGMENTOS TANGENTES


ponto médio de AB, ou seja, AM = MB. Os segmentos tangentes a uma circunferência, traçados por um
Aplicando o teorema de Pitágoras ao triângulo retângulo OMB, ponto exterior a ela, são congruentes.
temos:
2 2 2
= OM + BM
OB
2
⇔ 52 = 32 + BM
2
⇔ BM = 25 − 9 = 16
⇔ BM =
4

= BM
Logo, AM = 4 e AB = 8 cm.

PROPRIEDADE DA TANGENTE
Uma reta é tangente a uma circunferência se, e somente se, é
perpendicular ao raio no ponto de tangência.

Demonstração:
Aˆ = Bˆ= 90º 

OA = OB  ⇒ ∆OAP ≡ ∆OBP (caso especial de congruência de

OP comum
triângulos retângulos) ⇒ PA =
PB

ProBizu
O raio do círculo inscrito em um triângulo retângulo é igual ao
semiperímetro menos a hipotenusa.
Seja um triângulo retângulo ABC de hipotenusa BC = a e
semiperímetro p, então o raio do círculo inscrito é r = p – a.

Exemplo:
Calcule o comprimento do segmento tangente a uma
circunferência de raio 3 cm traçado a partir de um ponto que dista 5 r= p − a
cm do centro dessa circunferência.
Resolução:

Exemplo:
Calcule o perímetro de um triângulo retângulo de hipotenusa 5
cm e raio do círculo inscrito 1 cm.
Resolução:
Sabemos que o raio r do círculo inscrito em um triângulo retângulo
de semiperímetro p e hipotenusa a é dado por r = p - a.
Seja PT um segmento de reta tangente à circunferência, então
PT ⊥ OT . Substituindo os valores dados no enunciado, temos: 1 = p – 5 ⇔
p = 6.
Aplicando o teorema de Pitágoras no triângulo retângulo OPT,
temos: Logo, o perímetro do triângulo retângulo é 2p = 2 ⋅ 6 = 12 cm.
2 2 2
PT + OT =
OP
2
⇔ PT + 32 =
52
2
⇔ PT = 25 − 9 = 16
⇔ PT =
4

293

PM_BOOK16 - MAT.indb 293 25/11/2022 19:16:25


GEOMETRIA PLANA: CIRCUNFERÊNCIA E CÍRCULO

QUADRILÁTERO CIRCUNSCRITÍVEL COMPRIMENTO DO ARCO DE UMA


Teorema de Pitot: Um quadrilátero convexo é circunscritível se, CIRCUNFERÊNCIA
e somente se, as somas das medidas dos lados opostos são iguais. É fácil compreender que o comprimento de um arco de
circunferência e o ângulo central que o determina são grandezas,
diretamente, proporcionais.

#ABCD é circunscritível

AB + CD = AD + BC

Exemplo:
Determine o perímetro do quadrilátero circunscritível ABCD da
figura.

ÂNGULOS NA CIRCUNFERÊNCIA
ÂNGULO CENTRAL
Ângulo central é um ângulo cujo vértice é o centro da
circunferência e seus lados são raios. O ângulo central é igual ao arco
por ele determinado.
Resolução:
Como o quadrilátero ABCD é circunscritível, então as somas dos
lados opostos são iguais. Assim, temos:
AD + BC = AB + CD ⇔ 3x + 2x = ( 3x + 1) + ( x + 1) ⇔ x = 2 .
Portanto, o perímetro do quadrilátero é 2pABCD = ( 3x + 1) + 2x + ( x + 1) + 3x = 9x + 2 = 9 ⋅ 2 + 2 = 20
2pABCD = ( 3x + 1) + 2x + ( x + 1) + 3x = 9x + 2 = 9 ⋅ 2 + 2 = 20 .


COMPRIMENTO DA θ =AB

CIRCUNFERÊNCIA
A dificuldade da passagem de processos finitos a infinitos, ou
ainda, a não inclusão de itens como, por exemplo, limite de uma
função ou de uma sequência nos motiva a considerar o seguinte
experimento:
“Considere a construção de circunferências com comprimentos
distintos. Estabelecendo a razão entre o comprimento de uma
dessas circunferência e seu, respectivo, diâmetro encontramos, como ÂNGULO INSCRITO
resultado, uma constante irracional definida como π.” Ângulo inscrito é um ângulo com vértice sobre a circunferência e
cujos lados são secantes à circunferência. O ângulo inscrito é igual à
metade do arco por ele determinado.
Ou seja:
C
π= ∴ C = dπ ∴ C = 2πr
d
Sendo C, o comprimento da circunferência e r, a medida do seu raio.

294

PM_BOOK16 - MAT.indb 294 25/11/2022 19:16:26


GEOMETRIA PLANA: CIRCUNFERÊNCIA E CÍRCULO

 + CD
AB 

AB θ=
θ= 2
2

Todo ângulo reto é inscritível em uma semicircunferência e, ÂNGULO EXCÊNTRICO EXTERNO


reciprocamente, todo ângulo inscrito em uma semicircunferência e Ângulo excêntrico externo é o ângulo formado por duas secantes
com lados passando pelas extremidades da mesma, é reto. ou tangentes que se interceptam no exterior da circunferência.
O ângulo excêntrico externo é igual à semidiferença dos arcos por
ele determinados. No caso do ângulo formado por duas tangentes,
o ângulo excêntrico externo também pode ser calculado como o
suplemento do menor arco determinado.

 − CD
AB 
θ=
2

ÂNGULO DE SEGMENTO
Ângulo de segmento ou semi-inscrito é um ângulo com
vértice sobre a circunferência, um lado secante e outro tangente à
circunferência. O ângulo de segmento é igual à metade do arco por
ele determinado.

 − BT
AT 
θ=
2


AB
θ=
2

ÂNGULO EXCÊNTRICO INTERNO


Ângulo excêntrico interno é o ângulo formado por duas cordas
que se interceptam em um ponto interior da circunferência, distinto
do centro. O ângulo excêntrico interno é igual à semissoma dos arcos  maior − AB
AB  menor
=
θ  menor
= 180º −AB
por ele determinados. 2

295

PM_BOOK16 - MAT.indb 295 25/11/2022 19:16:27


GEOMETRIA PLANA: CIRCUNFERÊNCIA E CÍRCULO

Exemplo: Arcos capazes de ângulos suplementares, relativos a um segmento


= 30°.
= 60° e DE
Na figura, calcule α, β, γ, δ, ε, sabendo que AB AB, e em semiplanos opostos em relação à reta suporte do segmento
são partições de uma mesma circunferência.

PROPRIEDADES DA CIRCUNFERÊNCIA
Duas retas paralelas, secantes a uma circunferência, determinam
arcos de igual medida.
Resolução:
ˆ é um ângulo central, então α= AOB
AOB = 60° .
ˆ = AB

ˆ = AB= 60°= 30°.
ˆ é um ângulo inscrito, então β= ACB
ACB
2 2

ˆ é um ângulo de segmento, então γ= BAG
BAG ˆ = AB= 60°= 30°.
2 2
AFB ˆ é um ângulo excêntrico interno, então
 + DE
AB  60° + 30°
ˆ=
δ= AFB = = 45°.
2 2
DPEˆ é um ângulo excêntrico externo, então
 − DE
AB  60° − 30°
ˆ
ε= DPE= = = 15°.
2 2

=
r  s ⇔ AC 
ARCO CAPAZ BD

Um par de arcos capazes de θ sobre um segmento AB é o lugar


geométrico dos pontos do plano que são vértices de ângulos de
medida θ e extremidades em A e B.

 = CD
AB = CD ⇔ AB 

QUADRILÁTERO INSCRITÍVEL
Um quadrilátero está inscrito em uma circunferência se os seus
quatro vértices pertencem a essa circunferência.
Um quadrilátero convexo é inscritível em uma circunferência se, e
somente se, seus ângulos opostos são suplementares.

296

PM_BOOK16 - MAT.indb 296 25/11/2022 19:16:29


GEOMETRIA PLANA: CIRCUNFERÊNCIA E CÍRCULO

Exercício Resolvido

01. A ideia de usar rolos circulares para deslocar objetos pesados


provavelmente surgiu com os antigos egípcios ao construírem as
pirâmides.

# ABCD é inscritível

Aˆ + Cˆ = Bˆ + Dˆ = 180

Um quadrilátero é inscritível se, e somente se, as diagonais e dois


lados opostos determinam ângulos congruentes. Representando por R o raio da base dos rolos cilíndricos, em
metros, a expressão do deslocamento horizontal y do bloco de
pedra em função de R, após o rolo ter dado uma volta completa
sem deslizar, é

a) y = R. d) y = 2πR.
b) y = 2R. e) y = 4πR.
c) y = πR.
# ABCD é inscritível
 Resolução: E
ˆ ˆ
BAD = BDC Deslocamento do rolo em relação ao solo: 2π ⋅ R.
Deslocamento do bloco em relação ao rolo: 2π ⋅ R.
Deslocamento do bloco em relação ao solo: 4π ⋅ R.

Exercício Resolvido
Exemplo:
02. Um homem, determinado a melhorar sua saúde, resolveu
ˆ = θ.
Na figura abaixo, encontre o valor do ângulo BAD andar diariamente numa praça circular que há em frente à sua
casa. Todos os dias ele dá exatamente 15 voltas em torno da
praça, que tem 50 m de raio. Qual é a distância percorrida por esse
homem em sua caminhada diária?
Use 3 como aproximação para π.

a) 0,30 km d) 2,25 km
b) 0,75 km e) 4,50 km
c) 1,50 km

Resolução: E
A distância percorrida pelo homem em sua caminhada diária é
igual a 15 ⋅ 2 ⋅ π ⋅ 50 ≅ 4500 m = 4,5km.

Exercício Resolvido

Resolução: 03. O atletismo é um dos esportes que mais se identificam com o


ˆ + 30°= 80° ⇔ BCE ˆ = 50° (ângulo espírito olímpico. A figura ilustra uma pista de atletismo. A pista
No triângulo BCE, temos: BCE é composta por oito raias e tem largura de 9,76 m. As raias são
externo). numeradas do centro da pista para a extremidade e são construídas
Como BCA ˆ = BCE
ˆ = 50°= ADBˆ , então o quadrilátero ABCD é de segmentos de retas paralelas e arcos de circunferência. Os dois
inscritível, o que implica semicírculos da pista são iguais.
=θ BAD ˆ = 180° − BCDˆ = 180° − ( 36° + 50=
° ) 94° .

297

PM_BOOK16 - MAT.indb 297 25/11/2022 19:16:29


GEOMETRIA PLANA: CIRCUNFERÊNCIA E CÍRCULO

Exercício Resolvido

05. Considere a figura e os dados a seguir:

Se os atletas partissem do mesmo ponto, dando uma volta Dados:


completa, em qual das raias o corredor estaria sendo beneficiado?
• O é o circuncentro do triângulo ABC
a) 1 ˆ = 50°
• med(ACD)
b) 4
ˆ e BDC
• BEC ˆ são retos
c) 5
• FG é o diâmetro da circunferência de centro O
d) 7
ˆ em graus, é igual a
A medida do ângulo AFG,
e) 8
a) 40
Resolução: A b) 50
Na raia 1, o atleta percorreria a menor distância, pois seu c) 60
comprimento é menor. Os raios das semicircunferências são d) 70
menores.
Resolução: A
• Se o ângulo BDC é reto, então também é o ângulo CDA.
Exercício Resolvido • Se o ângulo CDA é reto e o ângulo ACD é igual a 50º,
então o ângulo DAC é igual a 40º (pois a soma dos ângulos
04. A cuia, vasilha utilizada pelos indígenas, pode ser usada para internos de um triângulo qualquer é sempre igual a 180º).
guardar coisas e também para comer e beber. A parte superior
de uma determinada cuia tem a forma de uma circunferência de • Se o ângulo BEC é reto, então também é o ângulo BEA.
50,24 cm de comprimento. • Se o ângulo BEA é reto e o ângulo DAC é igual a 40º,
então o ângulo ABF é igual a 50º.
• Se o ângulo ABF mede 50º, então a corda FA mede 100º.
• Se GF é o diâmetro da circunferência então a corda que vai
de F até G, passando pelo ponto A, mede 180º.
• Se a corda FA mede 100º e a corda que vai de F até G,
passando pelo ponto A, mede 180º, então a corda que
vai de A até G mede 80º. Assim, seu respectivo ângulo,
AFG, medirá 40º.
Qual a medida em centímetros do raio dessa circunferência?
Use π = 3,14.
a) 5
b) 6
c) 7
d) 8

Resolução: D
Considerando a fórmula para o comprimento de uma
circunferência, temos:
2 ⋅ π ⋅ r = 50,24
2 ⋅ 3,14 ⋅ R =50,24
6,28 ⋅ R =50,24
50,24
R=
6,28
R = 8 cm
Portanto, o raio da circunferência será 8 cm.

298

PM_BOOK16 - MAT.indb 298 25/11/2022 19:16:30


GEOMETRIA PLANA: CIRCUNFERÊNCIA E CÍRCULO

EXERCÍCIOS DE  , 3π rad.
 mede 36° e o arco BC
08. (EEAR) Na circunferência, o arco AB

FIXAÇÃO  , em radianos, é
A medida do arco ABC 5

01. (CFN) O diâmetro da roda de um caminhão é 1 metro. Para evitar


um acidente, trafegando a 60 km/h, sabe-se que o caminhão percorre
157 metros até parar. Quantas voltas completas a roda do caminhão
dará nessa situação? Considere π = 3,14.
a) 50 d) 100
b) 60 e) 150
c) 80 3π
a) .
4
02. (EAM) Sabendo que o diâmetro da roda de uma bicicleta de 29
polegadas (incluindo o pneu) é, aproximadamente, igual a 74 cm, b) 4π .
determine a distância, em metros, percorrida por essa roda, ao dar 4 5
voltas completas sem nenhum deslize.

Dado: número π = 3 c) .
4
a) 5,55m d) 328,55m

b) 6,66m e) 438,08m d) .
5
c) 8,88m
09. (EEAR) Na figura, as circunferências 1, 2, 3 e 4 são congruentes
03. (EAM) Supondo que um prato, de forma circular, possua um entre si e cada uma delas tangencia duas das outras. Se a circunferência
raio igual a 12 cm, qual é o comprimento, em centímetros, da 5 tem apenas um ponto em comum com cada uma das outras quatro,
circunferência desse prato? é correto afirmar que
Dados: π = 3,1
a) 37,20 d) 74,40
b) 44,64 e) 80,40
c) 64,40

04. (EEAR) Um carrinho de brinquedo que corre em uma pista circular


completa 8 voltas, percorrendo um total de 48 m. Desprezando a
largura da pista e considerando π = 3, o seu raio é, em metros, igual a
a) a circunferência 5 é secante às outras quatro circunferências.
b) a circunferência 5 é tangente exterior às outras quatro
circunferências.
c) todas as circunferências são tangentes interiores entre si.
d) todas as circunferências são tangentes exteriores entre si.

10. (EEAR) Um ângulo central α determina, em uma circunferência


a) 0,8 c) 1,2 2πr
de raio r, um arco de comprimento l = . A medida desse ângulo é
b) 1,0 d) 2,0 3
a) 150º. c) 100º.
05. (EEAR) Quanto uma pessoa percorrerá, em centímetros, se ela der
b) 120º. d) 80º.
6 voltas em torno de um canteiro circular de 1,5 m de raio?
Considere: π = 3,14
a) 2816 c) 4758 EXERCÍCIOS DE

b) 3127 d) 5652
TREINAMENTO
06. (EEAR) Com um fio de arame, deseja-se cercar dois jardins: um
circular, de raio 3 m, e o outro triangular, cujo perímetro é igual ao
comprimento da circunferência do primeiro. Considerando π = 3,14,  mede 70º, a medida do
01. (EEAR) Na figura, AB é diâmetro. Se AC
para cercar totalmente esses jardins, arredondando para inteiros, ângulo CÂB é
serão necessários ____ metros de arame.
a) 29 c) 35
b) 30 d) 38

07. (EEAR) Considere uma roda de 20 cm de raio que gira,


completamente e sem interrupção, 20 vezes no solo. Assim, a
distância que ela percorre é ____ π m.
a) 50°. c) 60°.
a) 100 c) 10
b) 55°. d) 65°.
b) 80 d) 8

299

PM_BOOK16 - MAT.indb 299 25/11/2022 19:16:31


GEOMETRIA PLANA: CIRCUNFERÊNCIA E CÍRCULO

02. Na figura, AB é o diâmetro da circunferência. O menor dos arcos 06. Um quadrilátero convexo está escrito em um círculo. A soma, em
(AC) mede radianos, dos ângulos α e β mostrados na figura é

π π π 3π e) 2π
a) b) c) d)
4 2 2
a) 100° d) 150°
b) 120° e) 160°
07. Em uma olimpíada de robótica, o robô BESOURO caminha de fora
c) 140° do círculo de manobras e, após se apresentar, retorna ao ponto inicial
conforme a figura a seguir.
03. O pentágono ABCDE abaixo está inscrito em um círculo de centro
O. O ângulo central CÔD mede 60°. Então x + y é igual a

Considerando que o caminho percorrido pelo robô está indicado pelas


setas, qual o ângulo x formado entre o caminho de saída e o caminho
de retorno do robô ao ponto inicial?
a) 180° d) 210° a) 28º b) 22º c) 21º d) 49º e) 56º
b) 185° e) 250°
08. Na figura, calcule x.
c) 190°

04. (EEAR) Na figura, AD é o diâmetro da circunferência, CÂD mede


35° e BD̂C mede 25°. A medida de AĈB é

a) 40° b) 45° c) 50° d) 55° e) 60°


a) 30°. b) 35°. c) 40°. d) 45°.
09. (CEFET) Sendo AB = x e CD = y, o valor de x + y é:
05. (EEAR) Considere o quadrilátero ABCO, de vértices A, B e C na
circunferência e vértice O no centro dela. Nessas condições x mede

a) 90° c) 140° e) 160°


a) 30° b) 45° c) 55° d) 60°
b) 120° d) 150°

300

PM_BOOK16 - MAT.indb 300 25/11/2022 19:16:31


GEOMETRIA PLANA: CIRCUNFERÊNCIA E CÍRCULO

10. (CEFET) Na figura, AB é o diâmetro da circunferência de centro O; 14. Os centros das três polias de um mecanismo estão sobre os
OX e OY são respectivamente bissetrizes de AOCˆ e BOD
ˆ . Dessa forma, vértices de um triângulo equilátero de lado I. O diâmetro de cada polia
ˆ
XOY mede: é muito próximo de I, como sugere a figura. O comprimento da correia
MNPQRSM que movimenta a polia é, aproximadamente

a) 76° d) 138°
b) 96° e) 181°
c) 109°

ˆ = 62°,
11. (EEAR) Na figura, O é o centro da circunferência, med (MON)
ˆ = 65°. O ângulo MAN
e med (PRQ) ˆ mede

a) ( π + 3) l ( π + 6)l
d)
2
b) ( 2π + 3) l e) 6πl
c) ( π + 6)l
a) 34°. c) 38°.
15. A, B, C, D, E e F são vértices de um hexágono regular inscrito na
b) 36°. d) 40°. circunferência de raio 5. Então, a soma dos comprimentos de todos
os arcos da figura é
12. Duas tangentes são traçadas a um círculo de um ponto exterior
A e tocam o círculo nos pontos B e C, respectivamente. Uma terceira
tangente intercepta o segmento AB em P e AC em R e toca o círculo em
Q. Se AB = 20 cm, então o perímetro do triângulo APR, em cm, é igual a

a) 30 d) 15π
b) 30π e) 6π
a) 39,5 d) 41 c) 15
b) 40 e) 41,5
c) 40,5 16. (CEFET) Na figura abaixo, temos dois arcos de duas circunferências
com centros O e P: o primeiro possui extremidades A e B e o segundo
13. (AFA) Conforme a figura abaixo, s e t são, respectivamente, possui extremidades A e C, respectivamente. Sabendo ainda que O
secante e tangente à circunferência de centro O. Se T é um ponto da é o ponto médio do segmento PA, B é um ponto do segmento PC e
circunferência comum às retas tangente e secante, então o ângulo α, que o primeiro arco mede 3,2 cm, então a medida, em centímetros,
formado por t e s, é do segundo arco é

a) 6,4 d) 3,0
b) 3,4 e) 1,6
a) 10° c) 30°
c) 3,2
b) 20° d) 40°

301

PM_BOOK16 - MAT.indb 301 25/11/2022 19:16:32


GEOMETRIA PLANA: CIRCUNFERÊNCIA E CÍRCULO

17. (FUVEST) Numa circunferência, c1 é o comprimento do arco de 21. (EFOMM) Tangenciando a reta r, encontramos três circunferências
π tangentes entre si. Determine a medida do raio da circunferência
radianos e c2 é o comprimento da secante determinada por este
6 menor, sabendo que as outras duas têm raios de medida igual a 5 cm.
c π
arco, como ilustrado na figura a seguir. Então, a razão 1 é igual a
multiplicado por: c2 6

a) 1,25 c) 1,75 e) 2
a) 2 c) (2 + 3) e) (3 + 3) b) 1,50 d) 1,85
b) (1 + 2 3) d) (2 + 2 3)
22. Na figura abaixo, AB = 21 e AC = 33. A distância entre os pontos
18. Qual é o valor do ângulo x na figura? de tangência P e Q é:

a) 6.
b) 8.
c) 9.
d) 10.
e) 12.

23. (CN) Sejam C1 e C2 dois círculos ortogonais de raios R1 e R2. A


distância entre os centros é π. A soma das áreas dos círculos é igual a:
3π2 5π2
a) . c) π2 . e) .
2 4
π2 d) π3.
b) .
4
a) 10° d) 20° 24. (CN) A distância entre os centros de dois círculos de raios iguais
b) 15° e) 25° a 5 e 4 é 41. Assinale a opção que apresenta a medida de um dos
c) 18° segmentos tangentes aos dois círculos.
a) 38,5 c) 39,5 e) 40,5
19. (AFA) Consideremos um triângulo retângulo que simultaneamente b) 39 d) 40
está circunscrito à circunferência C1 e inscrito na circunferência C2.
Sabendo-se que a soma dos comprimentos dos catetos do triângulo é 25. (CN) Se os lados de um triângulo medem, respectivamente, 3x,
K cm, então, a soma dos comprimentos dessas duas circunferências, 4x e 5x, em que x é um número inteiro positivo, então a distância
em cm, é entre os centros dos círculos inscritos e circunscritos a esse triângulo
4Kπ 2Kπ corresponde a
a) b) c) Kπ d) 2Kπ
3 3 5x 5x
a) c) x 2 e)
4 6
20. (CN) As quatro circunferências da figura abaixo têm raios (1+ 2 ) x x 5
b) d)
r = 0,5. O comprimento da linha que as envolve é, aproximadamente, 2 2
igual a
26. (CN) Considere a figura, em que x e y são medidas angulares
de arcos e z é a medida de ângulo assinalado. Pode-se afirmar que
x + y + z é igual a:

a) 255°.
b) 265°.
c) 275°.
d) 285°.
e) 295°.
a) 6,96 d) 9,96
b) 7,96 e) 10,96
c) 8,96

302

PM_BOOK16 - MAT.indb 302 25/11/2022 19:16:33


GEOMETRIA PLANA: CIRCUNFERÊNCIA E CÍRCULO

27. (CN) Na figura abaixo, o ponto P do menor arco AB dista 6 cm e 05. (AFA) Um triângulo retângulo está circunscrito a um círculo de raio
10 cm, respectivamente, das tangentes AQ e BQ. A distância, em cm, 15 m e inscrito em um círculo de raio 37,5 m. A área desse triângulo,
do ponto P à corda AB é igual a: em m², mede
a) 350. b) 750. c) 1050. d) 1350.
a) 30 .
b) 2 15. 06. (CN) Observe a figura a seguir.
c) 16.
d) 18.
e) 6 10.

28. (CN) Considere um triângulo retângulo e uma circunferência que


passa pelos pontos médios dos seus três lados. Se x, y e z, ( x < y < z )
são as medidas dos arcos dessa circunferência, em graus, exteriores
ao triângulo, então A figura acima mostra, num mesmo plano, duas ilhas representadas
a) =z 360° − y . c) x + y + z= 180°. e) = z 2x + y. pelos pontos ‘A’ e ‘B’ e os pontos ‘C’, ‘D’, ‘M’ e ‘P’ fixados no
continente por um observador. Sabe-se que ACB = ˆ ˆ
=
ADB =ˆ
APB 30,
b) z= x + y. d) x + y= 108° .
'M' é o ponto médio de cd = 100 e que PM = 10 m é perpendicular a
CD. Nessas condições, a distância entre as ilhas é de:
29. (CN) ABC é um triângulo retângulo de hipotenusa BC e altura AH.
Seja P um ponto do mesmo semiplano de A em relação à reta suporte a) 150 m. b) 130 m. c) 120 m. d) 80 m. e) 60 m.
de BC. Os ângulos HPC e ABC são iguais a 15°. Se o segmento PH é o
maior possível, pode-se afirmar que PH é igual a: 07. (IFRJ) Fernanda está de pé, penteando-se em frente ao seu espelho
a) AC. b) AB. c) BC 2. d) HC 2. e) AH. fixado em uma porta de armário que pode girar. Num dado momento,
um vento faz o espelho girar. Fernanda, que também é professora de
Matemática, percebeu que sua imagem se movimentou e imaginou o
seguinte problema para desafiar seus alunos:
EXERCÍCIOS DE “Eu me encontrava distante meio metro do espelho, antes de

COMBATE ele ter girado, com minha imagem centralizada. O espelho girou 15°,
afastando-se de mim. Minha imagem se deslocou, descrevendo um
caminho. Sabendo-se que o meu espelho é retangular, de dimensões
1 m × 1,7 m e que ocupa toda a porta do armário, determine a natureza
01. (CN) De um ponto P exterior a um círculo de raio 6, traçam- do caminho descrito pela imagem e o seu comprimento em metros.”
se secantes PXY, (PX < PY), X e Y pontos variantes pertencentes à A figura a seguir é um esquema que descreve a situação envolvida no
circunferência desse círculo. Os pontos médios das cordas XY desafio proposto.
descrevem um arco de circunferência de raio R. Assim sendo, qual
será o valor de R, sabendo-se que a tangente PT ao círculo mede 8?
a) 5 b) 6 c) 4 2 d) 4 3 e) 10

02. (CN) Duas tangentes a uma circunferência, de raio igual a dois


centímetros, partem de um mesmo ponto P e são perpendiculares
entre si. A área, em centímetros quadrados, da figura limitada pelo
conjunto de todos os pontos P do plano, que satisfazem às condições
dadas, é um número entre
(use π =3,14 )
a) vinte e um e vinte e dois. d) vinte e quatro e vinte e cinco.
b) vinte e dois e vinte e três. e) vinte e cinco e vinte e seis.
c) vinte e três e vinte e quatro.

03. (CN) Num quadrado ABCD de lado 6 cm, traça-se a circunferência K de


centro em A e raio 4 cm. Qual é a medida, em cm, do raio da circunferência
tangente exterior a K e tangente ao lado BC no ponto C? Assinale, dentre as opções abaixo, a resposta para o problema
a) 2,4 b) 2,5 c) 2,6 d) 2,7 e) 2,8 proposto por Fernanda.
1
a) Um segmento de reta de comprimento .
04. (CN) ABCD é um quadrado de lado L. Sejam K a semicircunferência, 6
traçada internamente ao quadrado, com diâmetro CD, e T a π 2
b) Um arco de circunferência de comprimento .
semicircunferência tangente ao lado AB com uma das extremidades 12
em A e tangente externamente a K. Nessas condições, o raio da π 2
c) Um arco de circunferência de comprimento .
semicircunferência T será 6
5L 4L 2L 3L L 3
a) b) c) d) e) d) Um segmento de reta de comprimento .
6 5 3 5 3 6
2
e) Um segmento de reta de comprimento .
12

303

PM_BOOK16 - MAT.indb 303 25/11/2022 19:16:34


GEOMETRIA PLANA: CIRCUNFERÊNCIA E CÍRCULO

08. (EPCAR) Na figura abaixo, os pontos A, B e C pertencem à


circunferência de centro O. Se β =150 e γ =50 , então α é:
DESAFIO PRO
1 Na figura, as circunferências são inscritas e circunscritas ao
triângulo ABC. Se AC = b e AB = c, o valor da medida do
segmento MN é

c + b.
a)
2
b) 3c – 2b.
c) 2c – b.
d) b – c.
e) c – b.

a) 15°.
b) 30°.
c) 35°.
2 Da figura, a diferença entre os perímetros dos triângulos
ABE e CDE é 12 cm, o perímetro do quadrilátero ABCD
circunscrito a circunferência é 18 cm. Sendo P, T e Q pontos de
d) 45°. tangência, o valor do segmento CD, em cm é
e) 10°.
a) 2.
09. (EPCAR) Nas figuras abaixo, é dado que AM = AP , BM = BQ e b) 3.
MP = MQ. Sendo assim, podemos afirmar que o valor de α + β é: c) 4.
d) 5.
e) 7.

a) 25°.
3 Em um triângulo ABC, traça-se a ceviana interior BP tal que
PC = 6(AP), CBP = 14° e BAC = 74°. O valor do ângulo BCA é
a) 2°. c) 10°. e) 14°.
b) 30º.
b) 4°. d) 8°.
c) 35°.
d) 40°.
e) 45°. 4 Se A, B, C, D e F são pontos de tangencia, o valor de x é

a) 100°.
10. Na figura, os segmentos PB e PD são secantes à circunferência,
b) 110°.
as cordas AD e BC são perpendiculares e AP = AD. A medida x do
ângulo BPD é c) 120°.
d) 130°.
e) 140°.

5
ˆ = CPQ
Da figura, ABC ˆ . Calcule x:
a) 30°.
b) 40°.
a) 50°
c) 50°.
b) 25°
d) 60°.
c) 70°
d) 80°
e) 60°

304

PM_BOOK16 - MAT.indb 304 25/11/2022 19:16:35


GEOMETRIA PLANA: CIRCUNFERÊNCIA E CÍRCULO

GABARITO
EXERCÍCIOS DE FIXAÇÃO
01. A 04. B 07. D 10. B
02. C 05. D 08. B
03. D 06. D 09. B
EXERCÍCIOS DE TREINAMENTO
01. B 09. C 17. C 25. D
02. A 10. E 18. C 26. C
03. D 11. A 19. C 27. B
04. A 12. B 20. B 28. B
05. D 13. A 21. A 29. A
06. C 14. A 22. E
07. C 15. B 23. D
08. A 16. C 24. D
EXERCÍCIOS DE COMBATE
01. A 04. E 07. B 10. A
02. E 05. D 08. C
03. E 06. B 09. C
DESAFIO PRO
01. D 03. A 05. A
02. B 04. A

ANOTAÇÕES

305

PM_BOOK16 - MAT.indb 305 25/11/2022 19:16:36


GEOMETRIA PLANA: CIRCUNFERÊNCIA E CÍRCULO

ANOTAÇÕES

306

PM_BOOK16 - MAT.indb 306 25/11/2022 19:16:36


GEOMETRIA PLANA:
SEGMENTOS PROPORCIONAIS

DIVISÃO DE SEGMENTOS Se 0 < k < 1 ⇒ dk < d ⇒ NA < NB , então N está à esquerda de A.


Se k > 1 ⇒ dk > d ⇒ NA > NB , então N está à direita de B.
DIVISÃO INTERNA Exemplo: Um ponto N divide o segmento AB , de 18 cm,
Um ponto M divide um segmento AB internamente na razão 4
externamente na razão . Calcule NA e NB .
AM 7
k > 0, quando M pertence ao segmento AB e =k . Resolução:
MB
Observe, inicialmente, que, como a razão de divisão externa
Os segmentos AM e MB são ditos segmentos aditivos, pois sua 4
soma é igual a AB. é < 1 , então NA < NB e o ponto N deve estar à esquerda do
7
Seja AB = d, então: segmento AB.

NA 4 NA NB NB − NA AB 18
AM AM MB AM + MB d dk d = ⇔ = = = = = 6 (segmentos subtrativos)
=⇔
k = = = ⇔ AM = ∧ MB = NB 7 4 7 7−4 3 3
MB k 1 k +1 k +1 k +1 k +1
⇒ NA = 4 ⋅ 6 = 24 e NB = 7 ⋅ 6 = 42
Se 0 < k < 1 ⇒ dk < d ⇒ AM < MB , então M está mais próximo
de A. DIVISÃO DE UM SEGMENTO EM MÉDIA E
Se k =
1 ⇒ AM =
MB , então M é ponto médio de AB. EXTREMA RAZÃO (DIVISÃO ÁUREA)
Se k > 1 ⇒ dk > d ⇒ AM > MB , então M está mais próximo de B. Um ponto P divide internamente um segmento de reta AB
Exemplo: Um ponto M divide o segmento AB , de 18 cm, segundo uma razão áurea ( ϕ ) quando a primeira parte está para a
2 segunda parte assim como o segmento todo está para a primeira
internamente na razão . Calcule MA e MB . parte, ou seja,
7
Resolução:
PA AB
= = ϕ
PB PA

O ponto P assim obtido é denominado ponto áureo de AB e o


segmento PA , segmento áureo de AB.
MA 2 MA MB MA = 2k
=⇔ = =k⇔ Note que o segmento áureo (PA) é a média geométrica entre o
MB 7 2 7 MB = 7k
segmento dado (AB) e o outro segmento aditivo (PB).
Segmentos aditivos: AB = MA + MB = 18 ⇒ 2k + 7k = 18 ⇔ k = 2 Sejam AP = a, PB = b, AB = a + b, onde o ponto P divide AB
⇒ MA = 2k = 2 ⋅ 2 = 4 e MB = 7k = 7 ⋅ 2 = 14 auricamente, temos:

DIVISÃO EXTERNA
Um ponto N divide um segmento AB externamente na razão
0 < k ≠ 1 , quando N pertence à reta suporte do segmento AB, mas
NA
não ao próprio segmento, e =k.
NB
Os segmentos NA e NB são ditos segmentos subtrativos, pois o
módulo da sua diferença é igual a AB.
Seja AB = d, então:
PA AB a a+b a b 1
= =ϕ⇔ = = ϕ ⇔ = 1+ = ϕ ⇔ ϕ = 1+ ⇔
PB PA b a b a ϕ
1 ± 5 ϕ> 0 1+ 5
⇔ ϕ2 − ϕ − 1= 0 ⇔ ϕ= ⇒ ϕ=
2 2
NA NA NB NA − NB d dk d
=k⇔ = = = ⇔ NA = ∧ NB =
NB k 1 k −1 k −1 k −1 k −1 1+ 5
=ϕ ≈ 1,618
2

307

PM_BOOK16 - MAT.indb 307 25/11/2022 19:16:38


GEOMETRIA PLANA: SEGMENTOS PROPORCIONAIS

TEOREMA DE TALES Exemplo: Em um triângulo ABC de lados AB = 12, AC = 8 e BC =


10, determine o maior segmento que a bissetriz interna do ângulo Â
Um feixe de retas paralelas determina sobre duas secantes determina sobre o lado BC.
quaisquer segmentos correspondentes proporcionais.
Resolução:

O maior segmento determinado pelo pé da bissetriz, D, sobre BC


é o correspondente ao maior dos lados adjacentes ao vértice A, ou
Sejam as retas r1  r2  r3    rn −1  rn , então seja, BD.
A1A 2 A 2A 3 An −1An Sendo BD = x, pelo teorema das bissetrizes internas, temos:
= = = .
B1B2 B2B3 Bn −1Bn 12 8
= ⇔ 120 − 12x = 8x ⇔ 20x = 120 ⇔ x = 6
x 10 − x
Exemplo: Determine o valor de x, sendo r, s e t retas paralelas.

TEOREMA DA BISSETRIZ EXTERNA


A bissetriz externa de um dos ângulos de um triângulo divide o
lado oposto externamente em segmentos proporcionais aos lados
adjacentes.
Seja AE a bissetriz interna do ângulo  de um triângulo ABC,
BE CE
então = .
AB AC

Resolução:
Como as retas r, s e t são paralelas, os segmentos de reta
determinados sobre as duas transversais são proporcionais. Assim,
x 6 4 ⋅6
temos: = ⇔x= = 3.
4 8 8

TEOREMA DAS BISSETRIZES Exemplo: Em um triângulo ABC de lados AB = 12, AC = 8 e BC =


10, determine a distância entre o pé da bissetriz externa do ângulo Â
TEOREMA DA BISSETRIZ INTERNA e o vértice mais próximo do lado BC.
A bissetriz interna de um dos ângulos de um triângulo divide o
lado oposto internamente em segmentos proporcionais aos lados
adjacentes.
Seja AD a bissetriz interna do ângulo  de um triângulo ABC,
BD DC
então = .
AB AC

Resolução:
O pé da bissetriz externa está do junto ao menor lado. Assim,
devemos calcular a medida de CE = x.
Pelo teorema das bissetrizes externas, temos:
12 8
= ⇔ 12x = 80 + 8x ⇔ 4x = 80 ⇔ x = 20
10 + x x

308

PM_BOOK16 - MAT.indb 308 25/11/2022 19:16:39


GEOMETRIA PLANA: SEGMENTOS PROPORCIONAIS

SEMELHANÇA DE TRIÂNGULOS
Se dois triângulos possuem lados respectivamente proporcionais,
então são semelhantes.

b c
= 
b' c'  ⇒ ∆ABC  ∆A'B'C'
ˆ 
Aˆ = A'

a b c
∆ABC  ∆A'B'C' ⇔ = = 3° caso: (Lp Lp Lp ) Se dois triângulos possuem os três lados
a' b' c'
respectivamente proporcionais, então são semelhantes.
Dois triângulos são semelhantes se, e somente se, seus ângulos
são respectivamente congruentes.
Dois triângulos de lados respectivamente paralelos são
semelhantes.
Se dois triângulos são semelhantes, então a razão entre duas
linhas homólogas é igual à razão de semelhança.

CASOS DE SEMELHANÇA DE TRIÂNGULOS


1° caso: (A.A.) Se dois triângulos possuem dois ângulos
respectivamente congruentes, então são semelhantes.

a b c
= = ⇒ ∆ABC  ∆A'B'C'
a' b' c'

Exemplo: Considere os quadrados da figura de lados a e b (a > b).


Então x é igual a

Bˆ = B'
ˆ 
 ⇒ ∆ABC  ∆A'B'C'
ˆC = C'
ˆ 

2° caso: (Lp ALp ) Se dois triângulos possuem dois lados


proporcionais adjacentes a ângulos congruentes, então são
semelhantes.

b2 ab
a) c)
a−b a+b
ab
a2 d)
b) a−b
a−b

309

PM_BOOK16 - MAT.indb 309 25/11/2022 19:16:40


GEOMETRIA PLANA: SEGMENTOS PROPORCIONAIS

Resolução:
Resolução:
Como as retas r, s e t são paralelas, os segmentos de reta
determinados sobre as duas transversais são proporcionais. Atente,
entretanto, para a correspondência entre os segmentos em cada
uma das transversais.
Em uma das transversais, são determinados segmentos de medida
3, 2 e y e, na outra transversal, segmentos de medida 5, x e 6,
respectivamente. Assim, temos:
3 2 y 2 ⋅ 5 10 3 ⋅ 6 18
= = ⇔x= = ∧ y= =
5 x 6 3 3 5 5
Uma outra maneira de identificar os segmentos proporcionais e
traçar uma paralela a uma das transversais, de forma que o ponto
de interseção dessas retas seja exterior às retas r e t.
Os triângulos retângulos “1” e “2” possuem lados paralelos, logo são
semelhantes.
a−b b b2
Assim, temos: = ⇔x= .
b x a−b

ProBizu

Exercício Resolvido

02. Considere o triângulo ABC, de lados AB = 15 , AC = 10 ,


BC = 12 e seu baricentro G. Traçam-se GE e GF paralelos a AB
e AC, respectivamente, conforme a figura abaixo. O perímetro
Quando utilizamos a semelhança de figuras planas aplicada
do triângulo GEF é um número que, escrito na forma de fração
a dimensões lineares (de comprimento) temos uma razão k de
irredutível, tem a soma do numerador com o denominador igual a
semelhança porém quando aplicamos a razão de semelhança em
relação as áreas das figuras a nossa razão passa a ser k².
Temos 2 triângulos semelhantes de razão de semelhança k.
ah
Nosso triângulo da esquerda possui área SI = enquanto o
2
ka ⋅ kh k ⋅ ah
2
=
nosso triângulo da direita possui área SII = = k 2 ⋅ SI
2 2 a) 43
Sendo assim fica claro que sempre que fizermos uma razão de b) 40
semelhança entre áreas a razão que deve ser utilizada é k².
c) 38
d) 35
Exercício Resolvido

01. Determine os valores de x e y, sendo r, s e t retas paralelas. Resolução: B


GM 1
Como G é o baricentro do triângulo ABC, então= .
AM 3
Como GE  AB e GF  AC , então ∆GEF  ∆ABC (A.A.) e a
GM 1
razão de semelhança é = , obtida a partir das medianas
homólogas. AM 3
Como a razão entre os perímetros dos triângulos também é igual à
razão de semelhança, temos:
2pGEF 2p 1 37
= GEF =⇔ 2pGEF =
2pABC 15 + 10 + 12 3 3

Como a fração obtida acima já se encontra em sua forma


irredutível, então a soma de seu numerador e seu denominador é
igual à 37 + 3 = 40.

310

PM_BOOK16 - MAT.indb 310 25/11/2022 19:16:41


GEOMETRIA PLANA: SEGMENTOS PROPORCIONAIS

Exercício Resolvido Resolução:


03. Em uma exposição artística um escultor apresentou sua obra
prima, intitulada “as torres vizinhas”. Repare que a mesma consta
de duas hastes paralelas de ferro fundidas perpendicularmente em
uma mesma base e escoradas por dois cabos de aço retilíneos,
como mostra a figura abaixo. As alturas das hastes medem,
respectivamente, 6 metros e 2 metros. Desprezando-se a espessura
dos cabos, determine a distância do ponto de interseção dos cabos
à base da escultura.

x=
2
92 + 122

x=
2
81 + 144

x 2= 225 → x= 225= 15
a) 2,25 m
=
y 20 + 15
2 2 2

b) 2,00 m
c) 1,75 m =
y 2 400 + 225
d) 1,50 m
y 2= 625 → y= 625= 25
e) 1,25 m
Logo, o maior lado mede 25.

Resolução: D
Exercício Resolvido

05. Considerando a figura abaixo, determine:

a) a medida x;
b) a medida y;
IJ CJ IJ CJ
∆CJI ~ ∆CAB ⇒ = ⇔ = c) a medida z;
AB CA 6 CA
d) o perímetro do trapézio PRST;
IJ AJ IJ AJ
∆AJI ~ ∆ACD ⇒ = ⇔ =
CD AC 2 AC Resolução:
IJ IJ AJ + CJ AC 3 a) Aplicando o Teorema de Pitágoras: ∆RSQ :
⇒ + = = =1 ⇔ IJ = =1,5 cm
6 2 AC AC 2 x 2 = 42 + 82 = 16 + 64 = 80

=x =
80 = 4 5
16.5
Exercício Resolvido b) ∆PTQ :
04. Marcelo fez um desenho como o ilustrado. As medidas de três y =22 + 42 =4 + 16 =20
2

lados estão indicadas. Quanto mede o maior lado?


=y =
20 2 5

311

PM_BOOK16 - MAT.indb 311 25/11/2022 19:16:42


GEOMETRIA PLANA: SEGMENTOS PROPORCIONAIS

Outra relação importante


2
c)=z2 (2 5 + (4 5)2 ah → (bc ) =
bc = ( ah) → b2c2 =a2h2
2 2

z = 20 + 80 = 100 = 100
2

1 a2 1 b2 + c2 1 b2 c2
z = 10 2
= 2 2 → 2= 2 2
→ 2= 2 2+ 2 2
h bc h bc h bc bc
d) o perímetro será: 2 + 4 + 4 + 8 + 10 =
28 Logo:
1 1 1
= +
h2 b2 c2
RELAÇÕES MÉTRICAS NO
TRIÂNGULO RETÂNGULO
Seja o triângulo ABC retângulo em A, conforme a figura a seguir: ALGUMAS APLICAÇÕES
IMPORTANTES
ALTURA DE UM TRIÂNGULO EQUILÁTERO
Seja o triângulo equilátero ABC de lado x, conforme a figura a seguir:

BC AC AB a b c a ⋅ h = b ⋅ c
∆ABC ~ ∆ABH ( A.A.A.) ⇒ = = ⇔ = = ⇒ 2
AB AH BH c h n c = a ⋅ n

BC AC AB a b c
∆ABC ~ ∆ACH ( A.A.A.) ⇒ = = ⇔ = = ⇒ b2 =a ⋅ m
AC CH AH b m h
AB AH BH c h n Seja M o ponto médio do lado BC , então AMBˆ = 90° . Aplicando
∆ABH ~ ∆ACH ( A.A.A.) ⇒ = = ⇔ = = ⇒ h2 = m ⋅ n o teorema de Pitágoras no triângulo retângulo AMB, temos:
AC CH AH b m h
2
As relações b² = a · m e c² = a · n mostram que cada cateto é a 2 2 2 2 x 2 x 2 3x 2 x 3
AM + BM = AB ⇔ AM +   = x 2 ⇔ AM = x 2 − = ⇔ AM =
média geométrica da hipotenusa e da sua projeção sobre a hipotenusa.  2 4 4 2
A relação h² = m · n mostra que a altura é a média geométrica das
projeções dos catetos sobre a hipotenusa. x 3
A altura de um triângulo equilátero de lado x é .
Temos as seguintes relações: 2

DIAGONAL DE UM QUADRADO
a⋅h = b ⋅c b2= a ⋅ m c 2= a ⋅ n
Seja o quadrado ABCD de lado x, conforme a figura a seguir:
1 1 1
h2= m ⋅ n = + a=
2
b2 + c2
h2 b2 c2

TEOREMA DE PITÁGORAS
Temos que:
b2 = a·m (I)

c2 = a·n (II)

Somando (I) e (II), temos:


b2 + c2 = a·m + a·n

b2 + c2 = a (m + n)  a = m + n

b2 + c2 =
a2 Aplicando o teorema de Pitágoras no triângulo retângulo ABC,
temos:
2 2 2 2
“O quadrado da medida hipotenusa é igual a soma AC = AB + BC ⇒ AC = x 2 + x 2 = 2x 2 ⇔ AC = x 2 .
dos quadrados das medidas dos catetos.”
A diagonal de um quadrado de lado x é x 2 .

312

PM_BOOK16 - MAT.indb 312 25/11/2022 19:16:43


GEOMETRIA PLANA: SEGMENTOS PROPORCIONAIS

TRAPÉZIO ISÓSCELES CIRCUNSCRITÍVEL QUADRILÁTEROS COM DIAGONAIS


PERPENDICULARES

Temos 2a = B + b (condição para que um quadrilátero seja


circunscritível)
B+b
a= Por Pitágoras
2
a=
2
s2 + p2
2x= B − b
B−b b=
2
q2 + r 2
x=
2 Somando as equações temos
Por Pitágoras, temos que a2 + b2 = p2 + s2 + q2 + r 2
x +h =
2
a2 2
Mas, também por Pitágoras
2 2
B+b B−b c=
2
s2 + q2
=  +h
2
 
 2   2 
d=
2
p2 + r 2
B + 2Bb + b
2
B − 2Bb + b
2 2 2
= + h2 Assim
4 4
a2 + b2 = p2 + s2 + q2 + r 2
4Bb = 4h2
a2 + b2 = q2
+ s2 + p
2
+
r2
 
h = Bb c2 d2

Então
TANGENTE COMUM A CÍRCULOS TANGENTES
a2 + b2 = c2 + d2

TERNOS PITAGÓRICOS
O termo (a, b, c) chama-se terno pitagórico se, e somente se, a, b
e c forem inteiros positivos tais que:
a2 + b2 = c2

Exemplo: 3, 4 e 5 é um terno pitagórico pois 32 + 42 = 52.


Seja m, um número ímpar, para formar-se um terno pitagórico
pode-se usar a relação:

 m2 − 1 m2 + 1
 m, , 
 2 2 

Alguns ternos pitagóricos conhecidos:


KA= R − r

AG= R= r a b c
3 4 5
Por Pitágoras
5 12 13
k 2 + (R − r ) = (R + r )
2 2

7 24 25
k 2 + R2 − 2Rr + r 2 = R2 + 2Rr + r 2 9 40 41
k = 4Rr
2

k = 2 Rr

313

PM_BOOK16 - MAT.indb 313 25/11/2022 19:16:44


GEOMETRIA PLANA: SEGMENTOS PROPORCIONAIS

EXERCÍCIOS DE 05. (EAM) Observe a figura abaixo.

FIXAÇÃO
01. (CFN) Um ciclista partindo do ponto A, percorre 15 Km para
norte; a seguir, fazendo um ângulo de 90°, percorre 20 Km para leste,
chegando ao ponto B. Qual a distância, em linha reta, do ponto B ao
ponto A?
O pé de uma escada de 10 m de comprimento está afastado 6 m de
a) 25 Km um muro. A que altura do chão, em metros, encontra-se o topo da
b) 17 Km escada?
c) 15 km a) 5 d) 8
d) 13 Km b) 6 e) 9
e) 10 km c) 7

06. (EAM) Sejam x, y, e z os lados de um triângulo retângulo. Sabendo


que y é a medida do maior lado, então
a) y=
2
x 2 + 2z2 d) y=
2
x 2 + z2

02. (EAM) Observe a figura abaixo. b) =


y 2 2x 2 + 2z2 e) =
y 2 2x 2 + z2

c) 2y=
2
x 2 + z2

07. (EEAR) Dois triângulos são semelhantes, e uma altura do primeiro


é igual aos 2/5 de sua homóloga no segundo. Se o perímetro do
primeiro triângulo é 140 cm, então o perímetro do segundo, em cm, é
a) 250. c) 300.
b) 280. d) 350.

08. (EEAR) Seja BDEF um losango de lado medindo 24 cm, inscrito no


triângulo ABC. Se BC = 60 cm, então AB = ______ cm.
Um prédio projeta no solo uma sombra de 30 m de extensão no
mesmo instante em que uma pessoa de 1,80m projeta uma sombra
de 2,0 m. Pode-se afirmar que a altura do prédio vale
a) 27 m c) 33 m e) 40 m
b) 30 m d) 36 m

03. (CFN) Um dos catetos de um triângulo retângulo mede 20 cm, e


o outro é igual a 3/4 do primeiro. Determine a medida da hipotenusa
a) 36 c) 42
desse triângulo.
b) 40 d) 48
a) 25. c) 23. e) 12.
b) 24. d) 15.
09. (ESA) Num triângulo retângulo cujos catetos medem 8 e 9 , a
hipotenusa mede
04. (EEAR) Sejam as relações métricas no triângulo ABC:
a) 10 d) 17
b) 11 e) 19
c) 13

10. (EEAR) Se os dados no triângulo ABC, retângulo em C, estão em


cm, então o triângulo BCD é

I. b2 = ax
II. a2 = b2 + c2 − 2b c ⋅ cos Â
III. h = xy

IV.
1 1 1
= +
h2 b2 c2

Se o triângulo ABC é retângulo em A, então o número de relações a) obtusângulo. c) isósceles.


verdadeiras acima é
b) retângulo. d) equilátero.
a) 1. b) 2. c) 3. d) 4.

314

PM_BOOK16 - MAT.indb 314 25/11/2022 19:16:44


GEOMETRIA PLANA: SEGMENTOS PROPORCIONAIS

EXERCÍCIOS DE 05. (EEAR) Num trapézio isósceles ABCD as bases AB e CD medem,

TREINAMENTO respectivamente, 16 cm e 4 cm. Traçando-se EF paralelo às bases,


sendo E ∈ AD e F ∈ BC, obtém-se os segmentos AE e DE , de modo
AE 1
que = . O comprimento de EF , em cm, é
DE 5
01. A medida, em metros, do segmento AD da figura abaixo é de
a) 8. b) 10. c) 12. d) 14.
a) 4
06. (EEAR) Conforme a figura, os triângulos ABC e CDE são
b) 5 retângulos. Se AB = 8 cm, BC = 15 cm e CD = 5 cm, então a medida
c) 6 de DE, em cm, é
d) 8
e) 10 a) 2/5
b) 3/2
c) 8/3
d) 1/4

07. (EEAR) Na figura, se BC = 60 cm, a medida de DE , em cm, é


02. No retângulo ABCD de lados AB = 4 e BC = 3, o segmento DM é
a) 20
perpendicular à diagonal AC . O segmento AM mede
b) 24
a) 3/2 c) 30
b) 12/5 d) 32
c) 5/2
d) 9/5
e) 2

08. (EAM) Observe a figura abaixo.

03. No polígono ABCD da figura precedente, os triângulos ABC e ACD


são semelhantes e retângulos — nos vértices B e C, respectivamente.
Além disso, AB = 16 cm, AC = 20 cm e CD é o lado menor do triângulo
ACD. Nessa situação, AD mede

a) 24 cm.
b) 25 cm.
c) 28 cm.
d) 32 cm.
e) 36 cm.

Considerando que os triângulos BDA e BCA apresentados acima são,


respectivamente, retângulos em D e C, calcule o valor de x em função
do lado c e assinale a opção correta.

a) c3 − 2 b) c2 − 1 c) c2 + 5 d) c−3 e) c2 − 3

09. (EEAR) Os pontos A, B, C e D estão alinhados entre si, assim


04. (EEAR) Na figura, o lado BC do triângulo ABC mede 12 cm, e a como os pontos A, E e F também estão. Considerando G o ponto de
altura relativa ao lado BC mede 8 cm. Se FG = 3EF, então o perímetro interseção de FC e ED , o valor de tg α é
do retângulo DEFG, em cm, é
a) 0,2
a) 20. b) 0,5
b) 28. c) 2
c) 85/3. d) 4
d) 64/3.

315

PM_BOOK16 - MAT.indb 315 25/11/2022 19:16:45


GEOMETRIA PLANA: SEGMENTOS PROPORCIONAIS

10. Na figura, AS e AP são, respectivamente, bissetrizes interna e externa 17. (FUVEST) O triângulo ABC tem altura h e base b (ver figura). Nele,
do triângulo ABC. Se BS = 8 m e SC = 6 m, então SP mede, em m, está inscrito o retângulo DEFG, cuja base é o dobro da altura. Nessas
condições, a altura do retângulo, em função de h e b, é dada pela fórmula:
a) 48.
b) 42.
c) 38.
d) 32.

11. (EEAR) Sabe-se que a hipotenusa de um triângulo retângulo tem


5 5 cm de comprimento e a soma dos catetos é igual a 15 cm. As
medidas, em cm, dos catetos são
a) 6 e 9 b) 2 e 13 c) 3 e 12 d) 5 e 10
(bh) d)
(bh)
a)
12. (EEAR) Se o triângulo ABC é retângulo em A, conforme a figura,
(h + b ) ( 2h + b )
e se O é o centro da circunferência circunscrita ao referido triângulo, ( 2bh) (bh)
então OH = ____ cm. b) e)
(h + b ) 2 (h + b ) 

c)
(bh)
(h + 2b )

18. (EPCAR) É dado o triângulo retângulo e isósceles ABC, onde


 = 90° e AB = m, como na figura abaixo

a) 1,5 b) 2,5 c) 2 d) 3

13. Seja um triângulo retângulo ABC, retângulo em A, tal que


AB = 30 cm e BC = 50 cm. Se um ponto D é marcado no lado AC , de
modo que BD = DC, então o segmento DC mede O lado do triângulo equilátero AQP mede
a) 31,25 cm c) 31,75 cm e) 32,25 cm
m 6 m 6
b) 31,50 cm d) 32 cm a) c)
3 2
m d) m
14. Em um triângulo retângulo ABC, as medidas das projeções dos b)
2
catetos AB e BC sobre a hipotenusa são, respectivamente, m e n.
1
Se a razão entre AB e BC, nesta ordem, é , então m : n é igual a 19. (EPCAR) Num triângulo retângulo isósceles cuja hipotenusa é
2 igual a h, a medida da área é numericamente igual ao triplo da medida
5 2 1 5 1 de um dos catetos. A soma das medidas dos catetos é
a) b) c) d) e)
2 2 2 4 4 a) 2h c) 2h
h h
15. Em um triângulo retângulo, a hipotenusa é a + 3 e um dos catetos b) d)
2 2
a – 3. Se o outro cateto vale 18, quanto vale a?
a) 20 b) 22 c) 24 d) 27 e) 30 20. Utilizando as três definições apresentadas a seguir, responda.
I. Um círculo de centro O e raio k é o lugar geométrico dos pontos
16. Na figura abaixo, ABCD é um retângulo. A medida do segmento do plano cuja distância ao centro O é menor ou igual a k.
EF é
II. Reta tangente a um círculo, de centro O, em um ponto P é a reta
a) 0,8 que intersecta o círculo no ponto P e é perpendicular ao raio OP.

b) 1,4 III. Círculos tangentes exteriores são círculos que se intersectam em


apenas um ponto, e a distância entre seus centros é igual a soma
c) 2,6 dos seus raios.
d) 3,2 Na figura abaixo, são apresentados três círculos de centros O1, O2 e O3
e) 3,8 e raios R, n e r respectivamente. Esses círculos são tangentes exteriores
e também tangentes a uma reta t. Assim o valor de n é

316

PM_BOOK16 - MAT.indb 316 25/11/2022 19:16:46


GEOMETRIA PLANA: SEGMENTOS PROPORCIONAIS

24. (CN) No triângulo ABC, tem-se BC = a e a altura AH = h. O lado do


triângulo equilátero DEF inscrito em ABC tal que DE é paralelo a BC,
é dado pela expressão:

Rr Rr
a) d)
2 R +2 r 2(R2 − r 2 )
Rr Rr
b) e)
R + 2 Rr + r 2( R − r )
Rr
c)
R − 2 Rr + r

21. (EPCAR 2013) Seja ABCD um paralelogramo cujos lados AB e


BC medem, respectivamente, 5 e 10. Prolongando o lado AB até o
ˆ é congruente ao
ponto P, obtém-se o triângulo APD, cujo ângulo APD
ˆ conforme a figura.
ângulo ACB,
2ah 2h 2ah
a) c) e)
a 3 + 2h h 3+a 2a 3 + h
ah 2a
b) d)
a 3 +h a 3 +h

25. (CN) Sobre os lados AB e AC de um triângulo ABC tomam-se os


pontos D e E, respectivamente, de modo que os triângulos ABC e ADE
sejam semelhantes.
Considere as 4 afirmações abaixo:
Então, a medida AP é
AD AE
a) 0,2 I. =
AB AC
b) 2
II. Bˆ = Dˆ e Ê = Cˆ
2 10
c)
5 III. AD = DE
AB BC
10
d) IV. Se a razão entre as áreas dos triângulos ABC e ADE é 16, então a
5 razão de semelhança é 4.

22. (CMRJ) Seja o triângulo isósceles ABC, com AC = BC = 7 cm e Pode-se concluir que o número de afirmações corretas é:
AB = 2 cm. Seja D um ponto situado na reta que contém o lado AB, a) 0 b) 1 c) 2 d) 3 e) 4
de tal modo que tenhamos o ponto B situado entre os pontos A e D, e
CD = 8 cm. Nestas condições, a medida de BD, em cm, vale 26. (CN) Num triângulo ABC traça-se a ceviana interna AD que o
a) 3 decompõe em dois triângulos semelhantes e não congruentes ABD e
b) 2 3 ACD. Conclui-se que tais condições:
c) 4 a) só são satisfeitas por triângulos acutângulos.
d) 5 b) só são satisfeitas por triângulos retângulos.
e) 4 2 c) só são satisfeitas por triângulos obtusângulos.
d) podem ser satisfeitas, tanto por triângulos acutângulos tanto
23. (CN) Num triângulo ABC de lado AC de medida 6 cm, traça-se quanto por triângulos retângulos.
a ceviana AD que divide internamente o lado BC nos segmentos BD e) podem ser satisfeitas, tanto por triângulos retângulos tanto
de medida 5 cm e DC de medida 4 cm. Se o ângulo B̂ mede 20º e o quanto por triângulos obtusângulos.
ˆ mede:
ângulo Ĉ mede 130º, então o ângulo BAD
a) 30º 27. (CN) Num triângulo ABC as medidas dos lados AB, AC e BC, são
b) 25º respectivamente iguais a 4, 6 e 8. Da extremidade D da bissetriz AD
traça-se o segmento DE, E pertencente ao lado AB, de tal forma que o
c) 20º
triângulo BDE é semelhante ao triângulo ABD. A medida do segmento
d) 15º BE é igual a:
e) 10º a) 2,56 c) 1,32 e) 1
b) 1,64 d) 1,28

317

PM_BOOK16 - MAT.indb 317 25/11/2022 19:16:47


GEOMETRIA PLANA: SEGMENTOS PROPORCIONAIS

28. (IME) Dado um quadrado ABCD, de lado a, marcam-se os pontos E 33. (CN) Em um triângulo retângulo ABC, BD é a bissetriz interna
sobre o lado AB, F sobre o lado BC, G sobre o lado CD e H sobre o lado relativa ao cateto maior AC e AH é a altura relativa à hipotenusa
AD, de modo que os segmentos formados AE, BF, CG, e DH tenham BC. Se o ponto I é a intersecção entre BD e AH, pode-se afirmar que
3a med(BH)
comprimento igual a . A área do novo quadrilátero formado pelas é igual a:
4 med(IH)
interseções dos segmentos AF, BG, CH, e DE mede:
med(BC) med(AD)
a) d)
a2 a2 2a2 med(AH) med(AI)
a) c) e)
25 16 9
med(BC) med(AD)
a2 a2 b) e)
b) med(AD) med(IH)
d)
18 9
med(BC)
c)
29. (CN) No triângulo ABC, os lados AB e AC têm a mesma medida x med(CD)
e a mediana BM tem a mesma medida y do lado BC. Sendo assim, é
x 34. (ITA) Seja ABC um triângulo retângulo cujos catetos AB e BC
correto afirmar que a razão é um valor compreendido entre:
y medem 8 cm e 6 cm, respectivamente. Se D e um ponto sobre AB
a) 0 e 1 c) 2e3 e) 4 e 5 e o triângulo ADC e isósceles, a medida do segmento AD, em cm, é
igual a
b) 1 e 2 d) 3 e 4
3 25
a) d)
30. (ITA) Considere o triângulo ABC, onde AD é a mediana relativa 4 4
ao lado BC. Por um ponto arbitrário M do segmento BD, tracemos o 15
b) 25
segmento MP paralelo a AD, onde P é o ponto de intersecção desta 6 e)
paralela com o prolongamento do lado AC (figura 1). Se N é o ponto 2
15
de intersecção de AB com MP, podemos afirmar que: c)
4

35. (ITA) Seja ABCD um trapézio isósceles com base maior AB


medindo 15, o lado AD medindo 9 e o ângulo ADB ˆ reto. A distância
entre o lado AB e o ponto E em que as diagonais se cortam é
21 37
a) . d) .
8 8
27 45
a) MN + MP = 2BM d) MN + MP = 2AD b) . e) .
8 8
b) MN + MP = 2CM e) MN + MP = 2AC 35
c) MN + MP = 2AB c) .
8

31. (CN) Analise as afirmativas abaixo:


I. Todo triângulo retângulo de lados inteiros e primos entre si possui
EXERCÍCIOS DE
um dos lados múltiplos de “5”.
II. Em um triângulo retângulo, o raio do círculo inscrito é igual ao
perímetro do triângulo menos a hipotenusa.
COMBATE
III. Há triângulos que não admitem triângulo órtico, ou seja, o
triângulo formado pelos pés das alturas.
01. (CN) O lado do hexágono equilátero inscrito numa
IV. O raio do círculo circunscrito a um triângulo retângulo é o dobro semicircunferência do círculo de raio r e centro O, em que uma de
da hipotenusa. suas bases está sobre o diâmetro, é:
Assinale a opção correta.
r
a) Apenas as afirmativas I e III são verdadeiras. a) .
2
b) Apenas as afirmativas I e IV são verdadeiras. r 2
c) Apenas as afirmativas I e II são verdadeiras. b) .
2
d) Apenas as afirmativas II e IV são verdadeiras. r 3
c) .
e) Apenas as afirmativas III e IV são verdadeiras. 2
d) r.
32. (CN) ABCD é um quadrado de lado L. Sejam K a semicircunferencia, 2r
e) .
traçada internamente ao quadrado, com diâmetro CD, e T a 3
semicircunferencia tangente ao lado AB em A e tangente à K. Nessas
condições, o raio da semicircunferencia T será
5L 3L 02. (CN) Considere o quadrilátero ABCD em que
a) d) med(AB) = 5 cm, med(BC) = 7,5 cm, med(CD) = 9 cm, med(AD) = 4 cm
6 5 ˆ deste quadrilátero é igual a:
e med(BD) = 6 cm. O ângulo ABC
4L L ˆ
b) e) ˆ + ADC. ˆ + ADC
d) 2 ⋅ BCD ˆ .
5 3 a) BCD
2 e) ADCˆ + 2 ⋅ BAC ˆ .
ˆ − BCD
2L ˆ + ADC
b) BAD ˆ
ˆ − BCD.
c)
3 ˆ .
ˆ + BCD
c) BAD

318

PM_BOOK16 - MAT.indb 318 25/11/2022 19:16:48


GEOMETRIA PLANA: SEGMENTOS PROPORCIONAIS

03. (CN 1998) Na figura abaixo, os segmentos AB e DA são tangentes 08. (ITA) Os catetos b e c de um triângulo retângulo de altura h (relativa
à circunferência determinada pelos pontos B, C e D. Sabendo-se que 1
os segmentos AB e CD são paralelos, pode-se afirmar que o lado BC é: à hipotenusa), são dados pelas seguintes expressões: = b k+ e
k
1
=
c k − onde k é um número real maior que 1.
k
Então o valor de h em função de k é:
k2 − 1 d) 2 (k 2 − 1)
a)
2k 2k
k2 − 1 k4 − 1
b) e)
k2 − 2 2k 3
1 + k2
c)
−1 − k 2

a) a média aritmética entre AB e CD. 09. (ITA) Considere o triângulo ABC retângulo em A. Sejam AE e AD
a altura e a mediana relativa à hipotenusa BC, respectivamente. Se
b) a média geométrica entre AB e CD.
c) a média harmônica entre AB e CD.
( )
a medida de BE é 2 − 1 cm e a medida de AD é 1 cm, então AC
mede, em cm,
d) o inverso da média aritmética de AB e CD.
e) o inverso da média harmônica entre AB e CD.
a) 4 2 − 5. d) 3 ( 2 −1 .)
b) 3 − 2.
e) 3 4 2 − 5.
04. (CN) Num triângulo acutângulo isósceles ABC, o segmento BP, P c) 6 − 2 2.
interno ao segmento AC, forma com o lado BA um ângulo de 15°.
Quanto mede o maior ângulo de PBC, sabendo que os triângulos ABP 10. Um triângulo retângulo possui perímetro 32 e área 20. A medida
e ABC são semelhantes? da sua hipotenusa é
a) 65,5° c) 97,5° e) 150° 57 63
a) d)
b) 82,5° d) 135º 4 4
b) 59 65
05. (CN) Num quadrilátero ABCD, tem-se: AB = 42, BC = 48, CD = 64, e)
4 4
DA = 49 e P é o ponto de interseção entre as diagonais AC e BD. Qual
é a razão entre os segmentos PA e PC, sabendo-se que a diagonal BD c) 61
é igual a 56? 4
7 8 7 6 49
a) b) c) d) e)
8 7 6 7 64

06. (UECE) No plano, as circunferências C1 e C2 cuja medida dos raios


são, respectivamente, 4 cm e 1 cm, tangenciam-se exteriormente
e são tangentes a uma reta r em pontos distintos. Uma terceira
DESAFIO PRO
circunferência C3, exterior a C1 e a C2, cuja medida do raio é menor
do que 1 cm tangencia a reta r e as circunferências C1 e C2: Nestas
condições a medida do raio da circunferência C3 é
1 (IME) Considere um triângulo ABC, em que BC = a, AB = c,
AC = b, c > b. O círculo inscrito a esse triângulo tangencia
BC em D, e DE é um diâmetro desse círculo. A reta que tangencia
1 4 o círculo e que passa por E intercepta AB em P e AC em Q. A reta
a) cm. c) cm. AE intercepta BC no ponto R. Determine os segmentos de reta
2 9
EQ e DR em função dos lados do triângulo: a, b e c.
1 5
b) cm. d) cm.
3 3

07. (CN) Analise as afirmativas abaixo, em relação ao triângulo ABC. 2 (IME) Dado um quadrado ABCD, de lado am, marcam-se os
pontos E sobre o lado AB, F sobre o lado BC, G sobre o lado
CD e sobre o lado AD, de modo que os segmentos formados AE,
I. Seja AB = c, AC = b e BC = a. Se o ângulo interno no vértice A é 3a
reto, então a2 = b2 + c2. BF, CG e DH tenham comprimento igual a .
4
II. Seja AB = c, AC = b e BC = a. Se a2 = b2 + c2, então o ângulo A área do novo quadrilátero, formado pelas interseções dos
interno no vértice A é reto. segmentos AF, BG, CH e DE, mede:
BC a2 a2
III. Se M é ponto médio de BC e AM = , ABC é retângulo.
2 a) d)
25 9
IV. Se ABC é retângulo, então o raio de seu círculo inscrito pode ser
igual a três quartos da hipotenusa. a2 2a2
b) e)
18 9
Assinale a opção correta.
a) Apenas as afirmativas I e II são verdadeiras. a2
c)
b) Apenas a afirmativa I é verdadeira. 16
c) Apenas as afirmativas II e IV são verdadeiras.
d) Apenas as afirmativas I, II e III são verdadeiras.
e) Apenas as afirmativas II, III e IV são verdadeiras.

319

PM_BOOK16 - MAT.indb 319 25/11/2022 19:16:49


GEOMETRIA PLANA: SEGMENTOS PROPORCIONAIS

3 (FUVEST) Na figura, o retângulo ABCDAB = 4BG tem lados


de comprimento AB = 4 e BC = 2. Sejam M o ponto médio do
lado BC e N o ponto médio do lado CD. Os segmentos AM e AC
ANOTAÇÕES

interceptam o segmento BN nos pontos E e F, respectivamente.

A área do triângulo AEF é igual a


24 61 23
a) c) e)
25 60 20
29 16
b) d)
30 15

4 (ESPCEX) Em um triângulo ABC, BC = 12 cm e a mediana


relativa a esse lado mede 6 cm. Sabendo-se que a mediana
relativa ao lado AB mede 9 cm, qual a área desse triângulo?
a) 35 cm2 . 2
c) 6 35 cm . e) 3 35 cm2 .
35
2
b) 2 35 cm . d) cm2 .
2

5 (ITA) Os triângulos equiláteros ABC e ABD têm lado comum


AB. Seja M o ponto médio de AB e N o ponto médio de
CD. Se MN = CN = 2 cm, então a altura relativa ao lado CD. do
triângulo ACD mede, em cm,

60 c) 40 2 6
a) . . e) .
3 3 3
50 30
b) . d) .
3 3

GABARITO
EXERCÍCIOS DE FIXAÇÃO
01. A 04. C 07. D 10. B
02. A 05. D 08. B
03. A 06. D 09. D
EXERCÍCIOS DE TREINAMENTO
01. C 10. A 19. C 28. A
02. D 11. D 20. B 29. B
03. B 12. D 21. B 30. D
04. D 13. A 22. A 31. A
05. D 14. E 23. C 32. E
06. C 15. D 24. A 33. C
07. B 16. B 25. D 34. D
08. E 17. B 26. B 35. E
09. B 18. A 27. A
EXERCÍCIOS DE COMBATE
01. B 04. C 07. D 10. B
02. C 05. E 08. E
03. B 06. C 09. C
DESAFIO PRO
01. BD = P – B e DR = DB – RB = P – b – (p – c) = c – b
02. A 03. D 04. C 05. A

320

PM_BOOK16 - MAT.indb 320 25/11/2022 19:16:50


GEOMETRIA PLANA: TRIGONOMETRIA NO
TRIÂNGULO RETÂNGULO E LEIS DOS SENOS E
COSSENOS

TRIGONOMETRIA NO TRIÂNGULO Seja o triângulo ABC retângulo em A, conforme a figura a seguir:

RETÂNGULO

Definem-se, para o ângulo agudo B̂:


cateto oposto b
Seno: senBˆ
= =
hipotenusa a
cateto adjacente c
Cosseno: cosBˆ
= =
hipotenusa a

ˆ cateto oposto b senBˆ


Tangente: tgB= = =
cateto adjacente c cosBˆ

c
Analogamente, são definidas para o ângulo agudo Ĉ: senCˆ = ;
a
b c b 1 1
cosCˆ = ; tgCˆ = , cotgCˆ = , sec Cˆ = e cossec Cˆ = .
a b c cosCˆ senCˆ
Considere os dois triângulos retângulos ABC e A’B’C’, então
AB AC BC
podemos escrever: = = .
A'B' A'C' B'C' ÂNGULOS NOTÁVEIS
Se conhecermos a medida dos lados do ∆ABC e de um dos lados
do ∆A’B’C’, então é possível calcular seus outros dois lados. ÂNGULOS DE 30° E 60°
Observe que todos esses dois triângulos possuem a mesma Seja o triângulo equilátero ABC de lado x, conforme a figura a seguir:
“forma”, diferindo apenas pelo “tamanho”.
Para qualquer triângulo retângulo semelhante ao ∆ABC, é sempre
possível, conhecendo-se um dos lados, calcular os outros dois.
A ideia da definição das linhas trigonométricas no triângulo
retângulo é identificar características do ∆ABC que permitam calcular
os lados dos triângulos retângulos semelhantes a ele sem precisar
de um “triângulo matriz”. Para isso vamos lançar mão de uma
característica que todos esses triângulos semelhantes têm em comum.
Eles possuem os mesmos ângulos.
Assim, para todos os triângulos retângulos semelhantes ao ∆ABC,
a razão entre o cateto oposto ao ângulo B̂ e a hipotenusa é a mesma.
A essa razão damos o nome de seno de B̂. Da mesma forma, a razão
entre o cateto adjacente ao ângulo B̂ e a hipotenusa é constante. A No triângulo retângulo AMB, temos:
essa razão damos o nome de cosseno de B̂. A razão entre o cateto
oposto e o cateto adjacente ao ângulo B̂ é chamada de tangente de B̂. x 3
AM 2 3 1
= =
sen60º = = cos60º
sen30º =
AB x 2 2

x
BM 2 1 3
=
cos60º = = =
cos =
30º sen60º
AB x 2 2

sen60º 32 1 1 3
=
tg60º = = 3 =
tg30º = =
cos60º 12 tg60º 3 3

321

PM_BOOK16 - MAT.indb 321 25/11/2022 19:16:51


GEOMETRIA PLANA: TRIGONOMETRIA NO TRIÂNGULO RETÂNGULO E LEIS DOS SENOS E COSSENOS

Exemplo: Exemplo:
Calcule x e y na figura. Calcule os catetos de um triângulo retângulo isósceles de
hipotenusa 4.
Resolução:

BC x 2
sen45°= = = ⇔ x= 2 2
Resolução: AC 4 2

BC y 3
sen60=
° = = ⇔=
y 3
AC 2 3 2 QUADRO RESUMO
Vamos resumir essas informações que serão necessárias em
AB x 1 quadro muito útil e que deve ser decorado, pois esses ângulos as
cos60°= = = ⇔ x= 3
AC 2 3 2 bancas de vestibulares não nos fornecerão os valores.

ÂNGULO DE 45° 30° 45° 60°

Seja o quadrado ABCD de lado x, conforme a figura a seguir:


1 2 3
SENO
2 2 2

3 2 1
COSSENO
2 2 2

3
TANGENTE 1 3
3

LEI DOS SENOS


Seja um triângulo ABC de lados BC = a, AC = b e AB = c e raio do
No triângulo retângulo ABC, temos:
círculo circunscrito R, então
=
sen 45 ˆ= BC
° senBAC =
x
=
2
AC x 2 2

=
cos 45 ˆ= AB
° cosBAC =
x
=
2
AC x 2 2

sen 45º 22
=
tg 45º = = 1
cos 45º 22

Note que sen45º = cos(90º - 45º) = cos45º.

a b c
= = = 2R
senAˆ senBˆ senCˆ

Demonstração:
Vamos inicialmente construir um triângulo A’BC, onde o lado A’B
será o diâmetro.

322

PM_BOOK16 - MAT.indb 322 25/11/2022 19:16:52


GEOMETRIA PLANA: TRIGONOMETRIA NO TRIÂNGULO RETÂNGULO E LEIS DOS SENOS E COSSENOS

LEI DOS COSSENOS


Seja um triângulo ABC de lados BC = a, AC = b e AB = c, então

a2 = b2 + c2 − 2bc ⋅ cos Aˆ

b2 = a2 + c2 − 2ac ⋅ cosBˆ

c2 = a2 + b2 − 2ab ⋅ cosCˆ

Demonstração:

Feito isso sabemos que o triângulo A’BC é retângulo em C pois


qualquer triângulo que possui como um de seus lados o diâmetro é
um triângulo retângulo o que equivale a dizer que qualquer triângulo
inscrito numa semicircunferência é retângulo.
Os ângulos inscritos na circunferência, A e A’, estão delimitados
 e dessa forma temos que A = A’.
pelo mesmo arco BC
Como o triângulo A’BC é retângulo podemos dessa forma, por
exemplo, calcular o seno do ângulo A’ e como A = A’ automaticamente
teremos que sen A' = sen A, sendo assim.
a
sen A' =
2R
Como A = A’ Aplicando o teorema de Pitágoras no triângulo retângulo ABD,
a temos: m² + h² = c².
=
sen =
A' sen A Aplicando o teorema de Pitágoras no triângulo retângulo BCD,
2R
temos: (b – m)² + h² = a².
Assim
(b − m)² + h2 = a2 ⇔ b2 − 2bm + m

2
+
h2 = a2 ⇔ b2 − 2bm + c2 = a2
a
2R = c2
sen A m
No triângulo retângulo ABD, temos: cos A = ⇔ m =c ⋅ cos A .
c
Adotando procedimento análogo para os outros vértices, temos
a b c ⇒ b2 − 2bm + c2 = a2 ⇔ a2 = b2 + c2 − 2bc cos A (C.Q.D.)
= = = 2R (C.Q.D.) .
sen Aˆ senBˆ senCˆ
Exemplo:
Exemplo:
Dois lados consecutivos de um paralelogramo medem 8 m e 12 m
=
Num triângulo ABC, são dados Aˆ 45º
= , Bˆ 30º e AC = 6 cm.
e formam entre si um ângulo de 60º. As medidas das diagonais desse
Então BC = _____ cm.
paralelogramo são tais que o número que expressa
a) 4 3 a) o seu produto é racional.
b) 6 2 b) a sua razão é maior que 2.
c) 3 /2 c) a sua soma é maior que 32.

d) 2 /2 d) a sua diferença é irracional.

Resolução: B Resolução: D

Lei dos cossenos no triângulo ABD: d2 = 82 + 122 − 2 ⋅ 8 ⋅ 12cos60º = 112 ⇔


d2 = 82 + 122 − 2 ⋅ 8 ⋅ 12cos60º = 112 ⇔ d = 112 = 4 7

Lei dos senos: Lei dos cossenos no triângulo ABC: D2 = 82 + 122 − 2 ⋅ 8 ⋅ 12cos120º = 304
D = 8 + 12 − 2 ⋅ 8 ⋅ 12cos120º = 304 ⇔ D = 304 = 4 19
2 2 2

BC AC BC 6 2 6
= ⇔ = ⇔ BC = ⋅ = 6 2 cm D −= d 4 19 − 4 7 ∉ 
sen Aˆ senBˆ sen 45º sen30º 2 12

323

PM_BOOK16 - MAT.indb 323 25/11/2022 19:16:53


GEOMETRIA PLANA: TRIGONOMETRIA NO TRIÂNGULO RETÂNGULO E LEIS DOS SENOS E COSSENOS

SÍNTESE DE CLAIRAUT Exercício Resolvido


Seja um triângulo ABC, onde a, b e c representam as medidas dos
02. Considere o triângulo ABC representado na figura.
lados e a é o maior lado, então

∆ABC é acutângulo ⇔ a² < b² + c²


∆ABC é retângulo ⇔ a² = b² + c²
∆ABC é obtusângulo ⇔ a² > b² + c²

Demonstração:
Se a é o maior lado do triângulo, então o ângulo  é o maior
ângulo.
Pela lei dos cossenos, temos: a2 = (b2 + c2 ) − a2
b2 + c2 − 2bc ⋅ cos Aˆ ⇔ 2bc ⋅ cos Aˆ =
=2 2 ˆ ˆ (b + c ) − a .
b + c − 2bc ⋅ cos A ⇔ 2bc ⋅ cos A = 2 2 2

a2 > b2 + c2 ⇔ cos Aˆ < 0 ⇔ Aˆ é obtuso e o ∆ABC é obtusângulo ˆ = 30° . Qual das expressões seguintes
Sabe-se que: AB= 8 e ACB
a2 =b2 + c2 ⇔ cos Aˆ = 0 ⇔ Aˆ = 90° e o ∆ABC é retângulo representa BC, em função de α?
a2 < b2 + c2 ⇔ cos Aˆ > 0 ⇔ Aˆ é agudo e o ∆ABC é acutângulo a) 16senα d) 16cosα
b) 8senα e) 4cosα
Exemplo: c) 4 3 sen α
Classifique os triângulos a seguir quanto aos ângulos.
a) triângulo de lados 8, 15 e 18. Gabarito: A
b) triângulo de lados 8, 15 e 17.
c) triângulo de lados 8, 15 e 16.
Resolução:
a) 18² = 324 > 289 = 15² + 8², então o triângulo é obtusângulo.
b) 17² = 289 = 15² + 8², então o triângulo é retângulo.
c) 16² = 256 < 289 = 15² + 8², então o triângulo é acutângulo.

Exercício Resolvido

01. Uma pessoa, com 1,7 m de altura, está em um plano horizontal


e caminha na direção perpendicular a um prédio cuja base está
situada neste mesmo plano. Em certo instante, essa pessoa h 1 h BC
No triângulo BCH, temos: sen30=
° ⇒= ⇒=
h
visualiza o ponto mais alto do prédio sob um ângulo de 30 graus. BC 2 BC 2
Ao caminhar mais 3 m, visualiza o ponto mais alto do prédio, agora
BC
sob um ângulo de 45 graus. Nestas condições, a medida da altura
h 2 ⇒ BC= 16 ⋅ senα
do prédio, em metros, é aproximadamente No triângulo BHA, temos: senα= ⇒ senα=
8 8
a) 5,6.
b) 6,6.
c) 7,6.
d) 8,6. Exercício Resolvido

03. A figura mostra um quadrado ABCD, com 6 cm de lado, e


Gabarito: A um triângulo retângulo ABF de hipotenusa AF , com o ponto F no
Seja h a altura do prédio. Tem-se que prolongamento do lado BC e o ponto E sendo a intersecção dos
segmentos DC e AF .
h − 1,7 3 h − 1,7 7,3
=
tg30 ° ⇔= ⇒ 1,7h + 2,2 ≅ 3h − 5,1 ⇒ h ≅ ⇒ h ≅ 5,6 m. F
h − 1,7 + 3 3 h + 1,3 1,3
7,3
⇒ 1,7h + 2,2 ≅ 3h − 5,1 ⇒ h ≅ ⇒ h ≅ 5,6 m.
1,3
E
D C

fora de escala

60º
A B

324

PM_BOOK16 - MAT.indb 324 25/11/2022 19:16:54


GEOMETRIA PLANA: TRIGONOMETRIA NO TRIÂNGULO RETÂNGULO E LEIS DOS SENOS E COSSENOS

ˆ mede 60º, a medida do segmento CE é


Sabendo que o ângulo FAB Gabarito: A

a) ( 3 + 3) cm.
b) (2 3 + 3) cm.
c) 2 ( 3 + 3) cm.

d) 2 3cm.

(
e) 2 3 − 3 cm. )
Gabarito: E
Aplicando Lei dos cossenos:
Veja que o menor ângulo interno é o oposto ao menor lado BC
22= 32 + 42 − 2 ⋅ 3 ⋅ 4 ⋅ cos α= 25 − 24cos α ⇒ 24cos α= 21

7
cos α =
8

Exercício Resolvido

15
05. Se sena = , com 0º < a < 90º, então a medida do lado BC
ˆ = 60º .
Já que AB || EC, então CEF 8
d do triângulo ABC é
Do triângulo ∆ABF tem-se que tg ( 60º ) = ⇒ d = 6 3cm
6
Mas,
=
BF BC + CF

⇒6 3 = 6 + CF

⇒ CF = 6 ( )
3 − 1 cm
a) 2 m d) 6 m
Do triângulo ∆ECF tem-se:
b) 3 m e) 7 m
6 ( 3 −1) =6−2
tg ( 60º ) =
CF
x
⇒x=
3
(
3 ⇒ x =2 3− 3 ) c) 5m

Gabarito: A

Exercício Resolvido

04. Num triângulo de lados 2 cm, 3 cm e 4 cm, o cosseno do


menor ângulo interno vale:
7
a)
8
3
b) 15
4 senaˆ =
8
1
c)
2 Calculemos o cosaˆ e aplicamos Lei dos cossenos:
2
2  15  7
d) sen2a + cos2a =
1 ⇒   + cos a =
2
1 ⇒ cosa =
2  8  8
3
e) (a é ângulo agudo ⇒ cosa > 0)
2
Logo:
7
x 2 =32 + 42 − 2 ⋅ 3 ⋅ 4 ⋅ cosa ⇒ x 2 =9 + 16 − 24 ⋅ =25 − 21 =4
8
x =2m

325

PM_BOOK16 - MAT.indb 325 25/11/2022 19:16:55


GEOMETRIA PLANA: TRIGONOMETRIA NO TRIÂNGULO RETÂNGULO E LEIS DOS SENOS E COSSENOS

EXERCÍCIOS DE 06. (EEAR) círculo da figura tem centro O e raio r. Sabendo-se que PQ

FIXAÇÃO equivale a
5r
12
e é tangente ao círculo no ponto P, o valor de senα é

01. (CFN) Em um triângulo retângulo, o seno de um de seus ângulos


agudos é
a) o inverso do cosseno desse ângulo.
b) o quadrado do cosseno desse ângulo.
a) 5/12. b) 5/13. c) 12/13. d) 0,48.
c) a razão entre as medidas dos catetos do triângulo.
d) a razão entre a medida da hipotenusa e a medida do lado 07. (EEAR) Pelo triângulo ABC, o valor de x² + 6x é
adjacente a esse ângulo.
e) a razão entre a medida do lado oposto a esse ângulo e a medida
da hipotenusa.

02. “Em qualquer triângulo, o quadrado de um lado é igual à soma


dos quadrados dos outros dois lados menos o duplo produto destes
lados pelo cosseno do ângulo entre eles.”. A regra que rege tal citação, a) 76 b) 88 c) 102 d) 144
bem como sua equação são, respectivamente:
08. (EEAR) No triângulo ABC, o valor de x é ___.

a) 3 b) 2 c) 2 d) 1
a) Lei dos cossenos; a2 = b2 + c2 − 2 ⋅ b ⋅ c ⋅ cos Aˆ
b) Lei dos senos; a2 = b2 + c2 − 2 ⋅ b ⋅ c ⋅ cos Aˆ 09. (ESA) Um terreno de forma triangular tem frentes de 20 metros
e 40 metros, em ruas que formam, entre si, um ângulo de 60º.
c) a b ⋅ c − 2 ⋅ b ⋅ c ⋅ cos Aˆ
Lei dos cossenos; = 2 2 2
Admitindo-se que 3 = 1,7, a medida do perímetro do terreno, em
d) Lei dos senos; a2 = b2 − c2 − 2 ⋅ b ⋅ c ⋅ cos Aˆ metros, é
a) 94 c) 92 e) 90
03. (EEAR) No triângulo ABC, se sen  = 0,3, então BC = _____ cm.
b) 93 d) 91

10. (EEAR) Em um triângulo retângulo, o quadrado da medida da


hipotenusa é igual ao dobro do produto das medidas dos catetos. Um
dos ângulos agudos desse triângulo mede
a) 6 c) 4 a) 15°. b) 30°. c) 45°. d) 60°.
b) 5 d) 3
EXERCÍCIOS DE
04. (EEAR) Se ABC é um triângulo, o valor de sen α é
TREINAMENTO
01. (EEAR) Considerando sen 40° = 0,6, o lado BC do triângulo ABC,
mede, em cm, aproximadamente

3 1
a) . c) .
2 2

2 d) 1.
b) . a) 6,11 b) 7,11 c) 8,33 d) 9,33
2

02. (EEAR) Na figura, o valor de x é


05. (EEAR) Em um triângulo retângulo, um dos catetos mede 4 cm, e
o ângulo que lhe é adjacente mede 60°. A hipotenusa desse triângulo,
em cm, mede
a) 6.
b) 7.
c) 8.
d) 9.
a) 20. b) 24. c) 30. d) 36.

326

PM_BOOK16 - MAT.indb 326 25/11/2022 19:16:56


GEOMETRIA PLANA: TRIGONOMETRIA NO TRIÂNGULO RETÂNGULO E LEIS DOS SENOS E COSSENOS

03. Considere os triângulos retângulos PQR e PQS da figura a seguir. 09. (EEAR) Considerando 37 = 6, o valor de x na figura é

a) 2,5. b) 3,5. c) 4,5. d) 5,5.

Se RS = 100, quanto vale PQ? 10. (ESPCEX) A água utilizada em uma fortificação é captada e
a) 100 3 d) 25 3 bombeada do rio para uma caixa d’água localizada a 50 m de distância
da bomba. A fortificação está a 80 m de distância da caixa d’água e o
b) 50
e)
(50 3 ) ângulo formado pelas direções bomba – caixa d’água e caixa d’água –
c) 50 3 3 fortificação é de 60º, conforme mostra a figura abaixo. Para bombear
água do mesmo ponto de captação, diretamente para a fortificação,
04. (EEAR) No triângulo AOB, OB = 5 cm; então AB, em cm, é igual a quantos metros de tubulação são necessários?

a) 6. c) 5 2.
b) 8. d) 6 3.

05. (EEAR) Na figura, são retângulos em E e em C, respectivamente, a) 54 metros. d) 70 metros.


os triângulos AEP e ACB. Se x = 30°, então a medida de PE , em cm, é b) 55 metros. e) 75 metros.
c) 65 metros.

EXERCÍCIOS DE

COMBATE
a) 10. c) 10 3 . 20 3
b) 3. d) .
3
01. (AFA) O acesso ao mezanino de uma construção deve ser feito
por uma rampa plana, com 2 m de comprimento. O ângulo α que
06. (EEAR) No triângulo, o menor valor que x pode assumir é
essa rampa faz com o piso inferior (conforme figura) para que nela
sejam construídos 8 degraus, cada um com 21,6 cm de altura, é,
aproximadamen-te, igual a

a) 4. b) 3. c) 2. d) 1.

07. (EEAR) Considere as medidas indicadas na figura e que


sen 70° = 0,9. Pela “Lei dos Senos”, obtém-se sen x = _____ .

a) 15º b) 30º c) 45º d) 60º

02. Na figura abaixo, tem-se o triângulo ABC inscrito em uma


circunferência de centro D. Se AB = 6 cm e AC = 9 cm, o perímetro do
triângulo ABC, em centímetros, é aproximadamente igual a

a) 0,4 b) 0,5 c) 0,6 d) 0,7

08. (EEAR) Na figura, o valor de x é

a) 3 2 . c) 3. a) 18,4 c) 20,6 e) 22,9


b) 2 3 . d) 4. b) 19,8 d) 21,4

327

PM_BOOK16 - MAT.indb 327 25/11/2022 19:16:56


GEOMETRIA PLANA: TRIGONOMETRIA NO TRIÂNGULO RETÂNGULO E LEIS DOS SENOS E COSSENOS

03. Investigações de um crime com arma de fogo indicam que sen ( α h) 1 − sen α
um atirador atingiu diretamente dois pontos, B e C, a partir de um a) R = d) R =
1 − sen α hsen α
único ponto A. São conhecidas as distâncias: AC = 3 m, AB = 2 m e
BC = 2,65 m. A medida do ângulo formado pelas duas direções nas 1 + sen α
hsen α e) R =
quais o atirador disparou os tiros é mais próxima de b) R =
1 − sen α hsen α
a) 30°. c) 60°. e) 90°.
b) 45°. d) 75°. hsen α
c) R =
sen α – 1
04. (EPCAR 3° ANO) Em um casarão abandonado, um portão medindo
3 m de altura por 2 m de largura se abre bruscamente girando 30°,
07. Calcular o perímetro do retângulo em que uma diagonal mede
conforme figura abaixo.
1
12 cm e forma ângulo α com um lado, onde sen α = .
3
(
a) 8 2 2 + 1 cm )
b) (
4 2 2 + 1 cm)
48 10
c) cm
5

96 10
d) cm
5

4
08. (AFA) Na figura a seguir, AD = 2 e CB = 5. Se tg α = , então
Esse fato faz com que uma aranha, seguindo sua teia, se desloque 5
cotgβ é
da posição A para a posição B’, sendo A’ e B’, respectivamente, as
extremidades superior e inferior do portão após o processo. É correto
afirmar que a distância que separa A e B’, em metros, é igual a

a) 17 − 4 3

b) 11 − 3

c) 11 + 3

d) 17 + 4 3
15
a)
05. (AFA) Na figura, o triângulo AEC é equilátero e ABCD é um 17
quadrado de lado 2 cm. A distância BE, em cm, vale. 13
b)
17
17
c)
20
19
d)
20

a) 2 3 c) 3+ 2 09. (EFOMM) Determine o perímetro do triângulo ABD, em cm,


b) 6 −1 d) 2 representado na figura abaixo:

06. (ESPCEX) Em uma das primeiras tentativas de determinar a


medida do raio da Terra, os matemáticos da antiguidade observavam,
do alto de uma torre ou montanha de altura conhecida, o ângulo sob
o qual se avistava o horizonte, tangente à Terra, considerada esférica,
conforme mostra a figura. Segundo esse raciocínio, o raio terrestre em
função do ângulo α é dado por:

a) 5 3 + 5
b) 5(2 + 2)( 3 + 1)
c) 20 + 4 5
d) 45
e) 50

328

PM_BOOK16 - MAT.indb 328 25/11/2022 19:16:57


GEOMETRIA PLANA: TRIGONOMETRIA NO TRIÂNGULO RETÂNGULO E LEIS DOS SENOS E COSSENOS
10. (EN) Um observador, de altura desprezível, situado a 25 m de um
prédio, observa-o sob um certo ângulo de elevação. Afastando- se

3
mais 50 m em linha reta, nota que o ângulo de visualização passa a (ITA) Num triângulo ABC, retângulo em Â, temos B̂ = 60º.
ser a metade do anterior. Podemos afirmar que a altura, em metros, As bissetrizes destes ângulos se encontram num ponto D.
do prédio é Se o segmento de reta BD mede 1 cm, então a hipotenusa mede:
a) 15 2 1+ 3
a) cm
b) 15 3 2
c) 15 5 b) 1 + 3 cm
d) 25 3
c) 2 + 3 cm
e) 25 5
d) 1 + 2 2 cm

e) 3 − 1 cm

DESAFIO PRO 4 (EFOMM) Considere um triângulo retângulo de catetos


9 cm e 12 cm. A bissetriz interna relativa à hipotenusa
desse triângulo mede, em cm,
36
1 (AFA) Na figura abaixo, a circunferência de centro O é
 tem medida α, 0 < α < π/2, e
trigonométrica, o arco AM
OMP é um triângulo retângulo em M. Esse triângulo tem por
a)
7
25
2

b) 2
perímetro 7
4
c) 2
15
7
d) 2
5
3
e) 2
5

1 + sen α + cos α
5 (EN) A figura abaixo mostra um paralelogramo ABCD.
Se d representa o comprimento da diagonal BD e α e β
são ângulos conhecidos (ver figura), podemos afirmar que o
a)
cos α comprimento x do lado AB é igual a
1 + sen α + cos α
b)
sen α
1 + 2 sen α + cos α
c)
cos α
1 + sen2a + cos a
d)
sena

2 As cidades A, B e C situam-se às margens de um rio e são


abastecidas por uma bomba situada em P, conforme figura
abaixo.
a) d cosβ
d senα
b)
sen ( α + β )
c) d senβ
d cos α
d)
sen ( α + β )

e) d cos (180º − ( α + β ) )

Sabe-se que o triângulo ABC é retângulo em B e a bissetriz do


ângulo reto corta AC no ponto P. Se BC = 6 3 km, então CP é,
em km, igual a
a) 6 + 3

(
b) 6 3 − 3 )
c) 9 3 − 2
d) 9 ( )
2 −1

329

PM_BOOK16 - MAT.indb 329 25/11/2022 19:16:58


GEOMETRIA PLANA: TRIGONOMETRIA NO TRIÂNGULO RETÂNGULO E LEIS DOS SENOS E COSSENOS

RESOLUÇÃO EM VÍDEO
Abra o ProApp, leia o QR Code, assista à resolução
de cada exercício e AVANCE NOS ESTUDOS!

GABARITO
EXERCÍCIOS DE FIXAÇÃO
01. E 04. A 07. D 10. C
02. A 05. C 08. D
03. D 06. B 09. A
EXERCÍCIOS DE TREINAMENTO
01. C 04. D 07. D 10. D
02. D 05. A 08. C
03. B 06. D 09. C
EXERCÍCIOS DE COMBATE
01. D 04. A 07. A 10. D
02. E 05. B 08. B
03. C 06. B 09. B
DESAFIO PRO
01. A 03. B 05. B
02. B 04. A

ANOTAÇÕES

330

PM_BOOK16 - MAT.indb 330 25/11/2022 19:16:58


GEOMETRIA PLANA: RELAÇÕES MÉTRICAS NO
TRIÂNGULO QUALQUER E NOS QUADRILÁTEROS

RELAÇÕES MÉTRICAS NOS Seja AD a paralela a LN passando por A. Pelo teorema de Tales,
temos:
TRIÂNGULOS
LB LA LA MB
LN  AD ⇒ = ⇔ ⋅ = 1
MB MD LB MD
TEOREMA DE MENELAUS
Se uma reta determina sobre os lados de um triângulo ABC os MD MC NC MD
LN  AD ⇒ = ⇔ ⋅ =1
pontos L, M e N, conforme a figura, então NA NC NA MC

LA MB NC Multiplicando as duas expressões, temos:


⋅ ⋅ =
1
LB MC NA LA MB NC MD LA MB NC
⋅ ⋅ ⋅ =⇔
1 ⋅ ⋅ =1
LB MD NA MC LB MC NA .

Exemplo:
OB
Na figura a seguir, calcule a razão .
OE

2
hA= p (p − a)(p − b )(p − c )
a
Resolução:
2 Aplicando o teorema de Menelaus no triângulo BCE com secante
hB= p (p − a)(p − b )(p − c )
b AOD, temos:
AE OB DC 2x OB 3y OB 5
⋅ ⋅ =⇔
1 ⋅ ⋅ =⇔
1 =
2 AC OE DB 5x OE y OE 6
hC= p (p − a)(p − b )(p − c )
c

TEOREMA RECÍPROCO DE MENELAUS


Demonstração: Se L, M e N são pontos sobre as retas suportes dos lados AB, BC
LA MB NC
e AC , respectivamente, e ⋅ ⋅ = 1 , então L, M e N estão
alinhados. LB MC NA

Demonstração:
Suponha que a reta NM corta o lado AB no ponto L’. Pelo
L'A MB NC
teorema de Menelaus, ⋅ ⋅ = 1 . Como é dado que
L'B MC NA
LA MB NC LA L'A
⋅ ⋅ = 1 , então = , ou seja, os pontos L e L’ dividem
LB MC NA LB L'B
o segmento AB na mesma razão e, portanto, são coincidentes. Logo,
L, M e N estão alinhados.

331

PM_BOOK16 - MAT.indb 331 25/11/2022 19:17:00


GEOMETRIA PLANA: RELAÇÕES MÉTRICAS NO TRIÂNGULO QUALQUER E NOS QUADRILÁTEROS

TEOREMA DE CEVA Demonstração:

Seja um triângulo ABC e três cevianas AD , BE e CF concorrentes,


então

Seja P a interseção de AD e BE , seja F’ a interseção de CP com


DB EC FA
o lado AB e suponha que ⋅ ⋅ = 1.
DC EA FB
DB EC F'A
Pelo teorema de Ceva, temos: ⋅ ⋅ =
1.
DC EA F'B
FA F'A
Comparando as duas igualdades, conclui-se que = .
FB F'B
DB EC FA
⋅ ⋅ =1 ⇔ EC ⋅ DB ⋅ FA =EA ⋅ DC ⋅ FB Logo, os pontos F e F’ dividem o lado AB internamente na
DC EA FB mesma razão, o que implica F ≡ F' e, consequentemente, a ceviana
CF passa pelo ponto P.
Demonstração:
Aplicando o teorema de Menelaus no ∆ABD com a secante CPF,
FA PD CB
RELAÇÃO DE STEWART
temos: ⋅ ⋅ =1
FB PA CD Seja um triângulo ABC de lados BC = a , AC = b e AB = c , e a
Aplicando o teorema de Menelaus no ∆ACD com a secante BPE, ceviana AP = x que divide o lado BC em dois segmentos BP = n e
BD PA EC CP = m , então
temos: ⋅ ⋅ =1
BC PD EA
Multiplicando as duas expressões, temos:
FA PD CB BD PA EC DB EC FA
⋅ ⋅ ⋅ ⋅ ⋅ =⇔
1 ⋅ ⋅ =1
FB PA CD BC PD EA DC EA FB

Exemplo:
Calcule x na figura a seguir.

c2m + b2n = x 2a + amn

Demonstração:
ˆ ˆ 180 − θ . Aplicando a lei dos cossenos
Seja APB = θ , então APC
=
aos triângulos APB e APC, temos:
x 2 + n2 − c2
Resolução: c2= n2 + x 2 − 2nx cos θ ⇔ cos θ=
2nx
Aplicando o teorema de Ceva, temos:
DB EC FA 12 3 9 27 b2 = m2 + x 2 − 2mx cos (180 − θ ) = m2 + x 2 + 2mx cos θ ⇔
⋅ ⋅ =1 ⇔ ⋅ ⋅ =1 ⇔ x = = 6,75
DC EA FB 8 x 6 4 b2 − m2 − x 2
⇔ cos θ =
2mx
TEOREMA RECÍPROCO DE CEVA x 2 + n2 − c2 b2 − m2 − x 2
= ⇔ mx 2 + mn2 − mc2 =nb2 − nm2 − nx 2 ⇔
Seja um triângulo ABC e três cevianas AD , BE e CF tais que 2nx 2mx
DB EC FA ⇔ nb2 − x 2 (m + n) + mc2 = mn (m + n) ⇔ nb2 − ax 2 + mc2 = amn ⇔
⋅ ⋅ = 1 , então as três cevianas concorrem em um único
DC EA FB b2 x 2 c2
ponto. ⇔ − + =⇔
1 c2m + b2n =x 2a + amn
am mn an

332

PM_BOOK16 - MAT.indb 332 25/11/2022 19:17:01


GEOMETRIA PLANA: RELAÇÕES MÉTRICAS NO TRIÂNGULO QUALQUER E NOS QUADRILÁTEROS

Exemplo: Demonstração:
Calcule x na figura abaixo Seja BHA = x , aplicando o teorema de Pitágoras no triângulo
ABHA , temos: x 2 + h2A = c2 ⇔ h2A = c2 − x 2 .
Aplicando a lei dos cossenos no ∆ABC , temos:

ˆ A = c2 + a2 − 2ax ⇔ x = c + a − b
2 2 2
b2 = c2 + a2 − 2ac cos ABH
2a
2
 c2 + a2 − b2   c2 + a2 − b2   c2 + a2 − b2 
h2A = c2 − x 2 = c2 −   = c + c − =
 2a   2a  2a 

( 2ac + c2 + a2 − b2 )( 2ac − c2 − a2 + b2 ) ( a + c ) − b  b − ( a − c ) 


2 2 2 2

= = =
4a2 4a 2

( a + b + c )( a + c − b )( a + b − c )(b + c − a) ( 2p )( 2p − 2b )( 2p − 2c )( 2p − 2a)
= = =
4a2 4a2
Resolução: 4 2
=2 ⋅ p (p − a)(p − b )(p − c ) ⇔ hA = p (p − a)(p − b )(p − c )
Pela relação de Stewart, temos: a a
42 ⋅ 6 + 62 ⋅ 2 = x 2 ⋅ 8 + 8 ⋅ 2 ⋅ 6 ⇔ 9x 2 = 72 ⇔ x 2 = 9 ⇔ x = 3 2
Analogamente, prova-se que hB= p (p − a)(p − b )(p − c ) e
2 b
hC= p (p − a)(p − b )(p − c ) .
ALTURA c

Seja um triângulo ABC de lados BC = a , AC = b e AB = c , MEDIANA


h h h
semiperímetro p, e alturas A , B e C relativas aos lados BC , AC e Seja um triângulo ABC de lados BC = a , AC = b e AB = c , e
AB , respectivamente, então medianas AM = mA , BN = mB e CP = mC relativas aos lados BC ,
AC e AB , respectivamente, então

(βiA )
2
= AC ⋅ AB − BD ⋅ CD

(βiB )
2
= BC ⋅ AB − CE ⋅ AE
a2
2 ( mA )
2
b +=
2
c 2
+
2
(βiC )
2
= AC ⋅ CB − AF ⋅ BF
b2
2 (mB )
2
a +=
2
c 2
+
2
Demonstração:
c2 Aplicando a relação de Stewart, temos:
b 2 (mC )
2
a +=
2 2
+
2 a a a a a2
c2. + b2. = x 2.a + a. . ⇔ c2 + b2 = 2x 2 + ⇔
2 2 2 2 2
a2
⇔ x = mA ⇔ b2 + c2 = 2 (mA ) +
2

b2
c2 2 (mB ) +
2
Analogamente, prova-se que a2 += e
2
c2
b 2 (mC ) + .
2
a +=
2 2

333

PM_BOOK16 - MAT.indb 333 25/11/2022 19:17:03


GEOMETRIA PLANA: RELAÇÕES MÉTRICAS NO TRIÂNGULO QUALQUER E NOS QUADRILÁTEROS

BISSETRIZES INTERNAS BISSETRIZES EXTERNAS


Seja um triângulo ABC de lados BC = a , AC = b e AB = c , Seja um triângulo ABC de lados BC = a , AC = b e AB = c ,
semiperímetro p, e bissetrizes internas AD = βiA , BE = βiB e CF = βiC semiperímetro p, e bissetrizes externas βeA , βeB e βeC relativas aos
relativas aos lados BC , AC e AB , respectivamente, então lados BC , AC e AB , respectivamente, então

Demonstração:
A demonstração é análoga à das bissetrizes internas. Deve-se
aplicar o teorema das bissetrizes externas para encontrar o ponto A’ e,
(βeA )
2
= CA' ⋅ CB − AC ⋅ AB posteriormente, a relação de Stewart para encontrar AA' .

(βeB )
2
= AB' ⋅ AC − BC ⋅ AB RELAÇÕES MÉTRICAS NOS
QUADRILÁTEROS
(βeC )
2
= AC' ⋅ AB − AC ⋅ BC
TEOREMA DE PTOLOMEU
Em um quadrilátero inscritível, o produto das diagonais é igual à
soma dos produtos dos lados opostos.
Demonstração:
Sejam BD = m e CD = n.
Aplicando o teorema das bissetrizes internas no ∆ABC, temos:
c b ac ab
a=m+ne = ⇒m= e n= .
m n b+c b+c
Aplicando a relação de Stewart, temos: c2n + b2m =x 2a + amn
Desenvolvendo as 2 equações teremos: ( βiA ) =bc − mn
2

(βiB )
2
Analogamente, prova-se que = BC ⋅ AB − CE ⋅ AE e
(βiC )
2
= AC ⋅ CB − AF ⋅ BF .

Exemplo:
No triângulo ABC, o ponto D do lado BC é tal que AD é bissetriz
do ângulo BAC. Se AB = 2, AD = 3 e AC = 6, o lado BC mede:
a) 3 c) 5 e) 12 ac + bd =
pq
b) 4 d) 10
Demonstração:
Resolução: B

Utilizando a fórmula β2 = bc − mn e o teorema da bissetriz


2 6
interna = ⇒ n = 3m temos:
m n
32 =2 ⋅ 6 − m ⋅ 3m ⇒ −3m2 =−3 ⇒ m =⇒
1 n =3m =3 ⇒ a =m + n =4

334

PM_BOOK16 - MAT.indb 334 25/11/2022 19:17:04


GEOMETRIA PLANA: RELAÇÕES MÉTRICAS NO TRIÂNGULO QUALQUER E NOS QUADRILÁTEROS

No quadrilátero ABCD, seja AJ isogonal de AC, então: Aplicando o teorema de Ptolomeu, temos:
DJ AD DJ a 8
∆AJD ~ ∆ABC ⇒ = ⇔ = p ⋅ q =1⋅ 2 + 2 ⋅ 3 ⇔ pq = 8 ⇔ q =
BC AC c p  ac + bd p
 ⇒ BJ +=
DJ ⇔ ac +=
bd pq
BJ AB BJ b p p 1⋅ 3 + 2 ⋅ 2 p 7
∆AJB ~ ∆ADC ⇒ = ⇔ = =
Aplicando o teorema de Hiparco, temos: = ⇔
CD AC d p  q 1⋅ 2 + 2 ⋅ 3 q 8
Exemplo: p 7 8 8 7
(p ⋅ q) ⋅ = 8 ⋅ = 7 ⇔ p2 = 7 ⇔ p = 7 ∧ q = =
Calcule o comprimento das diagonais do trapézio isósceles da q 8 7 7
figura.
Logo, a menor diagonal do quadrilátero é 7.

MEDIANA DE EULER
Sejam um quadrilátero ABCD de lados AB = a, BC = b, CD = c,
DA = d e diagonais AC = p e BD = q, cujos pontos médios são N e
M, respectivamente, então o segmento MN é chamado mediana de
Euler do quadrilátero e é dado pela expressão a seguir:

Observemos inicialmente que todo trapézio isósceles é inscritível.


Assim, aplicando o teorema de Ptolomeu, temos:
x ⋅ 2x + x ⋅ x = p ⋅ p ⇔ p2 = 3x 2 ⇔ p = x 3

TEOREMA DE HIPARCO
Em um quadrilátero inscritível, a razão entre as diagonais é igual
à razão entre as somas dos produtos dos lados que concorrem as
respectivas diagonais.

a2 + b2 + c2 + d2 = p2 + q2 + 4 ⋅ MN2

Demonstração:
Como N é ponto médio de AC, então BN e DN são medianas nos
triângulos ABC e ADC, respectivamente:
4 ⋅ BN2 = 2 ( a2 + b2 ) − p2 
 ⇒ a + b + c + d = p + 2 (BN + DN )
2 2 2 2 2 2 2

4 ⋅ DN2 = 2 ( c2 + d2 ) − p2 

Como M é ponto médio de BD, então NM é mediana no triângulo


BND:
p ab + cd
= 4 ⋅ MN2 = 2 (BN2 + DN2 ) − q2 ⇔ 2 (BN2 + DN2 ) = q2 + 4 ⋅ MN2
q ad + bc
Logo, a2 + b2 + c2 + d2 = p2 + q2 + 4 ⋅ MN2 .
Demonstração:
bcp adp abq cdq p ab + cd TEOREMA:
SBAC + SDAC= SABD + SCBD ⇔ + = + ⇔ =
4R 4R 4R 4R q ad + bc Em um trapézio, sejam a e c os lados não paralelos, B e b as bases,
B−b
p e q as diagonais, então a mediana de Euler é Me = , o que
2
implica p + q = a + c + 2Bb .
2 2 2 2
Exemplo:
Demonstração:
Calcule a menor diagonal do quadrilátero inscritível ABCD cujos
2
lados AB , BC , CD e DA medem, respectivamente, 1, 2, 2 e 3. B−b
a2 + b2 + c2 + B2 = p2 + q2 + 4 ⋅   ⇔
 2 
⇔ a2 + b2 + c2 + B2 = p2 + q2 + B2 − 2Bb + b2 ⇔
⇔ p2 + q2 = a2 + c2 + 2Bb

TEOREMA:
Num paralelogramo de lados a e b, e diagonais p e q, temos
2 ( a2 + b2 ) =p2 + q2 .
Demonstração:
Como as diagonais do paralelogramo cortam-se ao meio, sua
mediana de Euler tem medida zero, então:
a2 + b2 + a2 + b2 = p2 + q2 + 4 ⋅ 02 ⇔ 2 ( a2 + b2 ) = p2 + q2

335

PM_BOOK16 - MAT.indb 335 25/11/2022 19:17:05


GEOMETRIA PLANA: RELAÇÕES MÉTRICAS NO TRIÂNGULO QUALQUER E NOS QUADRILÁTEROS

EXERCÍCIOS DE 05. O valor de x no triângulo abaixo é

FIXAÇÃO
01. O valor da mediana AM no triângulo ABC da figura abaixo é

a) 6
b) 5
c) 4
d) 3
e) 3 6
a) 4
b) 4 2 06. No lado AB de um quadrado ABCD, constrói-se externamente
um triângulo retângulo ABF com hipotenusa AB. Calcule EF, sabendo
c) 2
que AF = 6, BF = 8 e que E é o ponto de encontro das diagonais do
d) 2 2 quadrado.
e) 2 a) 7 2
b) 2
02. O valor da bissetriz externa AP no triângulo abaixo é
c) 5 2
d) 2 7
e) 5

07. Um ponto P no lado AB de um triângulo retângulo ABC é tal


que BP = PA = 2. Sendo o ponto Q na hipotenusa AC tal que PQ é
perpendicular a AC, determine BQ, sabendo que CB =3
4
a) 7
5
b) 0,8
c) 5,04
a) 6
4
d) ⋅ 13
b) 2 6 5
c) 3 6 e) 13
d) 4 6
08. Na circunferência abaixo, temos que: AB = 4, BC = 2, AC é
e) 6 6 ˆ e CBD ˆ são iguais. Qual é o valor de BD?
diâmetro e os ângulos ABD

03. Em um triângulo ABC, em que AB = 6 e AC = 8, uma ceviana


interna AP mede 4. Sabendo-se que PC = 3·PB, temos que a medida A
de BC é:
a) 13.
b) 12.
c) 11.
d) 10.
e) 9.

04. Calcule a altura de um triângulo de lados AB = 13, BC = 14,


a) 2 3 +1
D
AC = 15, relativa ao lado BC:
a) 9. 9
b)
b) 10. 5 B
c) 11. c) 3 2
d) 12. d) 2+ 5 C
e) 13. e) 4

336

PM_BOOK16 - MAT.indb 336 25/11/2022 19:17:06


GEOMETRIA PLANA: RELAÇÕES MÉTRICAS NO TRIÂNGULO QUALQUER E NOS QUADRILÁTEROS

09. Seja um quadrilátero inscritível de lados a, b, c e d, nessa ordem, 03. As alturas de um triângulo de lados 6 cm, 10 cm e 12 cm são
então suas diagonais são:
8 14 16 14 4 14
a) cm; cm; cm
( ac + bd)( ab + cd) ( ac + bd)( ad + bc ) 3 3 3
a) e
ad + bc ab + cd
4 14 8 14 4 14
b) cm; cm; cm
( ac + bd)( ab + cd) ( ab + cd)( ad + bc ) 5 3 3
b) e
ad + bc ac + bd 8 14 8 14 4 14
c) cm; cm; cm
( ad + bc )( ab + cd) ( ac + bd)( ad + bc ) 3 5 3
c) e
ac + bd ab + cd 4 14 8 14 10 14
d) cm; cm; cm
ad + bc ab + cd 5 3 3
d) e
( ac + bd)( ab + cd) ( ac + bd)( ad + bc ) 4 14 12 14 10 14
e) cm; cm; cm
( ad + bc )2 ( ab + cd)2 5 5 3
e) e
( ac + bd)( ab + cd) ( ac + bd)( ad + bc )
04. Se AS é a bissetriz interna do triângulo abaixo o valor de x · y é
10. As diagonais de um quadrilátero ABCD inscritível, se encontram
em um ponto E. Sabendo que AE = 2, BE = 5, DE = 4 e BC = 7,5.
Determine quanto mede o raio da circunferência circunscrita ao
quadrilátero ABCD sabendo que a distância de DC ao centro O é de
2,5.
a) 7.
b) 10.
c) 15.
d) 4.
e) 3.

EXERCÍCIOS DE a) 60

TREINAMENTO b) 54
c) 50
d) 48
01. O valor de x no triângulo abaixo é e) 42

05. Se AP é a bissetriz interna do triângulo abaixo o valor de x · y é

a) 4
a) 162
b) 5
b) 150
c) 19
c) 140
d) 15
d) 500
e) 13 e) 120

02. A razão entre a soma dos quadrados das medianas de um


06. Em um círculo de 10 2 de diâmetro, temos duas cordas medindo 2
triângulo e a soma dos quadrados dos lados desse triângulo é
e 10. Achar a corda do arco diferença dos arcos das cordas anteriores.
a) 3/4
a) 4
b) 1/2
b) 2 2
c) 5/6
c) 3 2
d) 4/3
e) 2 d) 4 2
e) 6 2

337

PM_BOOK16 - MAT.indb 337 25/11/2022 19:17:07


GEOMETRIA PLANA: RELAÇÕES MÉTRICAS NO TRIÂNGULO QUALQUER E NOS QUADRILÁTEROS

07. (ITA) O comprimento da diagonal de um pentágono regular de 13. Num triângulo ABC, a bissetriz interna relativa ao ângulo A mede
lado medindo 1 unidade é igual à raiz positiva de: 6 cm, e determina sobre o lado BC segmentos adjacentes de 3 cm e
a) x² + x – 2 = 0 4 cm. Determine, em centímetros, a medida dos lados AB e AC desse
triângulo:
b) x² – x – 2 = 0
a) 8 e 10.
c) x² – 2x + 1 = 0
b) 6 e 8.
d) x² + x – 1 = 0
c) 9 e 12.
e) x² – x – 1 = 0
d) 4,5 e 6.
08. Em um hexágono regular ABCDEF inscrito em uma circunferência, e) 7,5 e 10.
 se marca o ponto P, tal que AP = 2 e PC = 4. Calcule PD.
sobre o arco AB
14. Os lados de um triângulo ABC são AB = 6, AC = 8 e BC = 10.
3 Sobre o lado BC, toma-se D tal que a ceviana AD = 5. Um possível
a) 5
3 valor de BD é:
10 a) 2,1.
b) 3
3 b) 2,2.
20 c) 2,3.
c) 3
3 d) 2,4.
15 e) 2,5.
d) 3
4
25 15. Num triângulo acutângulo ABC, BC = 4 cm, o ângulo C mede 30°
e) 3 e a projeção do lado AB sobre BC mede 2,5 cm. O comprimento da
2
mediana que sai do vértice A é igual a:
09. Exteriormente e relativo ao lado BC de um quadrado ABCD se a) 1 cm.
marca o ponto P, tal que DPC = 45°. Se PB + PD = 16, calcule PA.
b) 2 cm.
a) 8 2
c) 0,9 cm.
b) 12
d) 3 cm.
c) 9
e) 2 cm.
d) 9 2
e) 8 3 16. (CN) Uma expressão que dá o lado do eneágono regular, em
função das diagonais a , b e c , com a < b < c, é:
10. Em um quadrilátero ABCD circunscrito a uma circunferência c2 + b2
a)
AC = 6 e BD = 8, ( AB) + ( CD) =
2 2
58 e (AB)(CD) = (BC)(AD). Calcule a a
medida do segmento que une os pontos médios de AC e BD .
cb
a) 2 b)
a
b) 2
c2 − b2
c)
c) 3 a
d) 3 2
(c + b)
2

d)
e) 5 a

(c − b)
2
11. Num triângulo ABC, traçam-se as cevianas internas BX e CY, que e)
são concorrentes em P. A reta AP corta o lado BC no ponto Z. Sabendo a
que AX = 3·XC, e que BY = 2·YA, a razão BZ : ZC é 17. Uma expressão que dá o lado do heptágono regular em função
a) 1. das diagonais a e b, com a < b é:
b) 2. a2 + b2 − ab
a)
c) 3. a
d) 4.
a2 − b2 + ab
e) 6. b)
b

12. Em um triângulo ABC, tem-se que AB = 12 cm, AC = 8 cm e a2 + b2


c)
BC = 5 cm. Seja D o pé da bissetriz interna AD, e I o incentro do a
triângulo ABC.
a2 − ab
O valor da razão AI : ID é igual a: d)
a+b
a) 1.
b) 2. a2 + b2 − ab
e)
c) 3. ab
d) 4.
e) 6.

338

PM_BOOK16 - MAT.indb 338 25/11/2022 19:17:08


GEOMETRIA PLANA: RELAÇÕES MÉTRICAS NO TRIÂNGULO QUALQUER E NOS QUADRILÁTEROS

18. (IME) Em um círculo de 10 2 cm de diâmetro temos duas cordas QD


23. Na figura ABCD e BPDQ são paralelogramos, se = k . Calcule
de 2 cm e 10 cm. Achar a corda do arco soma dos arcos das cordas LP AR
anteriores. .
PS
a) 4 3 cm
b) 5 2 cm
c) 6 3 cm
d) 8 2 cm
e) 10 cm

19. (AFA) A figura a seguir é um pentágono regular de lado 2 cm.

a) k
b) k/2
c) k/3
d) k/4
A medida de AC, uma das diagonais do pentágono regular, em cm, e) 2k
é igual a
24. Em um triângulo retângulo ABC reto em B, se marcam os pontos
a) 1+ 5
P e Q sobre BC e AC respectivamente, tal que CP = CQ = 2,
b) −1 + 5 AP ∩ BQ = {R} , se traça a reta que contém R e C intersectando AB
5 em S, logo o prolongamento de QP intersecta o prolongamento de AB
c) 2+ em L. Se AB = 10 e AC = 8, calcule LS.
2
a) 12
d) 2 5 −1
b) 16
20. Dois círculos são concêntricos, de raios 2 e 5. Seja AB um diâmetro c) 20
do menor, e P um ponto sobre o maior. Calcule PA 2 + PB2 : d) 24
a) 36. e) 26
b) 42.
25. Sobre os lados AB e BC de um triângulo ABC, tomam-se os pontos
c) 48.
P e Q, de forma que as razões AP : PB e BQ : QC sejam iguais a 0,5.
d) 52. Seja X o ponto de interseção de PC com AQ. Quanto vale a soma das
e) 58. razões AX : XQ e PX : XC?
a) 7 : 8.
21. O lado AB de um quadrado é prolongado até P tal que BP = 2 AB. b) 8 : 9.
Com M, ponto médio de DC, BM é desenhado cortando AC em Q. PQ
corta BC em R. A razão CR/RB é c) 9 : 10.
a) 1/4 d) 10 : 11.
b) 3/4 e) 11 : 12.
c) 1/2
d) 3/2

22. No triângulo retângulo ABC, P e Q estão sobre BC e AC,


respectivamente, tais que CP = CQ = 2. Pelo ponto de interseção R de
AP e BQ, uma reta é desenhada passando também por C e cortando
AB em S. O prolongamento de PQ corta AB em T. Se a hipotenusa
AB = 10 e AC = 8, o valor de TS é
a) 1/4
b) 3/4
c) 1/2
d) 3/2

339

PM_BOOK16 - MAT.indb 339 25/11/2022 19:17:08


GEOMETRIA PLANA: RELAÇÕES MÉTRICAS NO TRIÂNGULO QUALQUER E NOS QUADRILÁTEROS

= MB
04. (EN) O triângulo da figura abaixo é equilátero, AM = 5 e
CD = 6. A área do triângulo MAE vale
EXERCÍCIOS DE

COMBATE
01. Os pontos E e D pertencem aos lados AB e BC de um triângulo
AE 1 CD 1
ABC e são tais que = e = . Sendo F o ponto de concurso
EB 3 DB 2
EF AF 200 3
de AD e CE , então + é igual a a)
FC FD 11
100 3
b)
11
100 2
c)
2
200 2
d)
11

e) 200 2
2

05. (CN) Considere que ABC é um triângulo retângulo em A, de lados


AC = b e BC = a. Seja H o pé da perpendicular traçada de A sobre BC,
e M o ponto médio de AB, se os segmentos AH e CM cortam-se em P,
AP
a razão será igual a:
PH
4 a2
a) a)
5 b2
5 a3
b) b)
4 b2
3
c) a2
2 c)
b3
d) 2
d) a3
5
e) b3
2
a
02. Sejam ABC um triângulo, E o ponto médio de AC e O o ponto e)
b
médio de BE. A reta AO intersecta o lado BC em D. Se AO = 12, então
OD é igual a
a) 2 06. Seja um triângulo ABC, onde AB = 27, AC = 26 e BC = 25. Seja
I o ponto de interseção das bissetrizes internas do triângulo ABC.
b) 2,5 A medida do segmento BI é
c) 3 a) 15
d) 4
b) 5 + 26 + 3 3
e) 6
c) 3 26
03. (AFA) Na figura abaixo o perímetro do triângulo equilátero ABC é 2
d) 546
72 cm, M é o ponto médio de AB e CE = 16 cm . Então, a medida do 3
segmento CN, em cm, é um sétimo de e) 9 3

07. ABCD é um quadrado sobre o qual se constrói externamente


o triângulo AFB, retângulo em F. Seja E o ponto de interseção das
diagonais de ABCD. Sabendo que FA = 6, FB = 8, calcule a medida
de EF:
a) 7 2
b) 2
c) 5 2
a) 51
d) 2 7
b) 50
e) 5
c) 49
d) 48

340

PM_BOOK16 - MAT.indb 340 25/11/2022 19:17:09


GEOMETRIA PLANA: RELAÇÕES MÉTRICAS NO TRIÂNGULO QUALQUER E NOS QUADRILÁTEROS

08. ABCD é um quadrilátero convexo inscrito numa semicircunferência


de diâmetro AD = 4. Sabe-se que AB = BC = 1. Quanto mede CD? 4 Seja um hexágono regular ABCDEF inscrito em uma
 se traça P, tal que AP = 2 e
circunferência. Se no arco AN
PC = 4. O valor de PD é:
a) 1,5.
b) 2. 3
a) 5
c) 2,5. 3
d) 3. b) 10 3
e) 3,5. 3
20
c) 3
09. Seja ABC um triângulo isósceles, com AB = AC = 10 e BC = 8, 3
inscrito em uma circunferência. Seja P um ponto sobre o arco BC desta
circunferência, que não contém A, tal que PB + PC = 12. A medida d) 25 3
de PA é 2
a) 12 25
e) 3
b) 13 2
c) 15
d) 16
e) 18
5 Exteriormente e relativo ao lado BC de um quadrado
ABCD se marca o ponto P, tal que DPC = 45°. Se PB + PD = 16
então a medida de PA é:

10. Seja ABCD um quadrilátero e seja O o ponto de intersecção das a) 8 2


diagonais AC e BD. Se BO = 4, OD = 6, OC = 3 e AB = 6, a medida b) 12
de AD é c) 9
a) 166 d) 9 2
b) 172 e) 8 3
c) 184
d) 14
GABARITO
EXERCÍCIOS DE FIXAÇÃO
01. B 04. D 07. D 10. A
02. E 05. D 08. C

DESAFIO PRO 03. A 06. A


EXERCÍCIOS DE TREINAMENTO
09. A

01. C 08. B 15. A 22. D

1 Num triângulo ABC, traçam-se as cevianas internas BX e


CY, que são concorrentes em P. A reta AP corta o lado BC
no ponto Z. Sabendo que AX = 3 · XC, e que BY = 2 · YA, a razão
02. A
03. C
09. A
10. A
16. C
17. B
23. B
24. D
BZ : ZC é: 04. D 11. E 18. D 25. A
a) 1 05. A 12. D 19. A
b) 2 06. E 13. B 20. E
c) 3 07. E 14. B 21. B
d) 4 EXERCÍCIOS DE COMBATE
e) 6 01. C 04. B 07. A 10. A
02. D 05. A 08. C

2 Em um triângulo ABC, tem-se que AB = 12 cm, AC = 8 cm e


BC = 5 cm. Seja D o pé da bissetriz interna AD, e I o incentro
do triângulo ABC. O valor da razão AI : ID é igual a:
03. D
DESAFIO PRO
06. A 09. C

01. E 03. A 05. A


a) 1
02. D 04. B
b) 2
c) 3
d) 4
e) 6

3 Sobre a hipotenusa AC de um triângulo retângulo ABC se


( )
marcam os pontos M e N M ∈ AN , tal que AM = MN = NC = 4.
O valor de (BM)² + (BN)² é:
a) 80
b) 64
c) 72
d) 84
e) 56

341

PM_BOOK16 - MAT.indb 341 25/11/2022 19:17:10


GEOMETRIA PLANA: RELAÇÕES MÉTRICAS NO TRIÂNGULO QUALQUER E NOS QUADRILÁTEROS

ANOTAÇÕES

342

PM_BOOK16 - MAT.indb 342 25/11/2022 19:17:10


GEOMETRIA PLANA: RELAÇÕES MÉTRICAS NOS
POLÍGONOS REGULARES E POTÊNCIA DE PONTO

RELAÇÕES MÉTRICAS NOS


R 2
POLÍGONOS REGULARES l4 = R 2 a4 =
2

TRIÂNGULO EQUILÁTERO INSCRITO


O lado de um triângulo equilátero (regular) inscrito em uma
R
circunferência de raio R é l3 = R 3 e o seu apótema é a3 = .
2

R
I3 = R 3 a3 =
2

Demonstração:
No triângulo retângulo AOM da figura temos:
l4
2 l4
sen 45º = 2 ⇔ = ⇔ l2 = R 2
R 2 2R
a4 2 a4 R 2
cos 45º = ⇔ = ⇔ a4 =
R 2 R 2
Demonstração: Poderíamos observar também que o triângulo da figura é um
No triângulo retângulo AOM da figura temos: l
triângulo retângulo isósceles e, portanto, a4 = 4 .
l3 2
3 l3
sen60º = 2 ⇔ = ⇔ l3 = R 3
R 2 2R
HEXÁGONO REGULAR CONVEXO INSCRITO
a3 1 a R
cos60º = ⇔ = 3 ⇔ a3 = O lado de um hexágono regular convexo inscrito em uma
R 2 R 2
a 3
circunferência de raio R é l6 = R e o seu apótema é a6 = .
2
QUADRADO INSCRITO
O lado de um quadrado (quadrilátero regular convexo) inscrito em a 3
l6 = R a6 =
R 2 2
uma circunferência de raio R é l4 = R 2 e o seu apótema é a4 = .
2

343

PM_BOOK16 - MAT.indb 343 25/11/2022 19:17:11


GEOMETRIA PLANA: RELAÇÕES MÉTRICAS NOS POLÍGONOS REGULARES E POTÊNCIA DE PONTO

Demonstração: HEXÁGONO REGULAR CIRCUNSCRITO


O hexágono regular pode ser dividido em seis triângulos
equiláteros. Dessa forma, o lado de cada triângulo equilátero é l6 = R e
R 3
o apótema é igual à altura desse triângulo a6 = .
2

TRIÂNGULO HEXÁGONO
QUADRADO
EQUILÁTERO REGULAR

LADO R 3 R 2 R

R R 2 R 3
APÓTEMA
2 2 2

TRIÂNGULO EQUILÁTERO CIRCUNSCRITO

Podemos ver que o triângulo OEF é equilátero de lado L6 e altura


L6 3 2R 3 2 3R
R, assim teremos que =
R ⇒ L=6 . = .
2 3 3 3
2 3R
= L6 = e A6 R
3

TRIÂNGULO HEXÁGONO
QUADRADO
EQUILÁTERO REGULAR
Pela razão de 2 : 1 do baricentro teremos que AO =2OR ⇒ AO =2R ⇒ AH =3R
AO =2OR ⇒ AO =2R ⇒ AH =3R, assim 2 3R
LADO 2 3R 2R
3
L 3 L 3 6R 3
=
AH ⇒ 3R
= ⇒L 3
= 6R ⇒ =
L . = 2 3R.
2 2 3 3 APÓTEMA R R R

=L3 2=
3R e A 3 R
OCTÓGONO REGULAR CONVEXO
QUADRADO CIRCUNSCRITO O lado de um octógono regular convexo inscrito em uma
circunferência de raio R é=
l8 R 2 − 2 e o seu apótema é
R
=
a8 2 + 2.
2

R
=
l8 R 2 − 2 =
a8 2+ 2
2

É fácil ver pela figura = =


que L 4 2R e A 4 R.

344

PM_BOOK16 - MAT.indb 344 25/11/2022 19:17:12


GEOMETRIA PLANA: RELAÇÕES MÉTRICAS NOS POLÍGONOS REGULARES E POTÊNCIA DE PONTO

Demonstração: l10 R −1 ± 5
∆OAB ~ ∆BAK ⇔ = ⇔ l10
2
+ R ⋅ l10 − R2 = 0 ⇔ l10 = R
Vamos utilizar a fórmula de duplicação de gênero desenvolvida R − l10 l10 2
anteriormente: l2n = 2R2 − R 4R2 − ln2 . ( 5 − 1) R
Como l10 > 0, então l10 = .
Assim, temos: 2
O apótema é dado por
l8 = 2R2 − R 4R2 − l24 = 2R2 − R 4R2 − (R 2 ) =R 2 − 2
2
2
l  R2 10 + 2 5 2 R
=R2 −  10  =R2 − ( 5 − 1) =
2
2
a10 R ⇔ a10 = 10 + 2 5
Utilizando agora a expressão para o cálculo do apótema 2 16 16 4
1
=an 4R2 − ln2 , temos:
2
PENTÁGONO REGULAR CONVEXO
( )
2
1 1 R R
=
a8 4R2 =
− l28 4R2 − R 2 − 2= 4 − (2 − =
2) 2+ 2 O lado de um pentágono regular convexo inscrito em uma
2 2 2 2
R
circunferência de raio R =
é l5 10 − 2 5 e o seu apótema é
R 2
DECÁGONO REGULAR CONVEXO =a5 ( )
5 +1 .
4
O lado de um decágono regular convexo inscrito em uma
R
circunferência de raio R é=
l10 ( 5 − 1) e o seu apótema é =l5
R
10 − 2 5 =
a5
R
( 5 + 1)
R 2 2 4
=
a10 10 + 2 5 .
4

R R
=
l10 ( 5 − 1) =
a10 10 + 2 5
2 4

Demonstração:

Demonstração:

Na figura da demonstração do lado do decágono regular, vamos


aplicar a lei dos cossenos ao ∆OAB. Assim, temos:
R2
( 5 − 1) = 2R2 (1 − cos 36º ) ⇔ 6 − 2 5 = 8 − 8cos 36º ⇔ co
2
2
l10 = R2 + R2 − 2R2 cos 36º ⇔
4
R2
( 5 − 1) = 2R2 (1 − cos 36º ) ⇔ 6 − 2 5 = 8 − 8cos 36º ⇔ cos 36º = 5 + 1
2
2
l10 = R2 + R2 − 2R2 cos 36º ⇔
4 4
Voltando à figura do pentágono, consideremos o triângulo
retângulo OAM. Assim, temos:
a5 5 +1 5 +1
cos 36º = = ⇔ a5 = R.
R 4 4
Aplicando o teorema de Pitágoras a esse triângulo, temos:
2
 l5  l2 R2 2 2
R2 − ( 5 + 1) =(10 − 2 5 ) ⋅ R ⇔ l52 =
(10 − 2 5 ) ⋅ R ⇔ l5 =R
2
  = R2 − a52 ⇔ 5 =
2 4 16 16 4 2

345

PM_BOOK16 - MAT.indb 345 25/11/2022 19:17:14


GEOMETRIA PLANA: RELAÇÕES MÉTRICAS NOS POLÍGONOS REGULARES E POTÊNCIA DE PONTO

R2 2
5 + 1) =
(10 − 2 5 ) ⋅ (10 − 2 5 ) ⋅ R ⇔ l5 =R
2
⇔ l52 = 10 − 2 5
16 4 2

DODECÁGONO REGULAR CONVEXO


O lado do dodecágono regular convexo inscrito em uma
l12 R 2 − 3 e o seu apótema é
circunferência de raio R é =
R
=
a12 2 + 3.
2

R
=
I12 R 2 − 3 =
a12 2+ 3
2

RETA DE EULER
Num triângulo ABC, o ortocentro (H), o baricentro (G) e o
circuncentro (O) são colineares.

Demonstração:
Vamos utilizar a fórmula de duplicação de gênero desenvolvida
anteriormente: l2n = 2R2 − R 4R2 − ln2 . Assim, temos:

l12 = 2R2 − R 4R2 − l62 = 2R2 − R 4R2 − R2 =R 2 − 3


Utilizando agora a expressão para o cálculo do apótema
1
=an 4R2 − ln2 , temos:
2
( )
2
1 1 R R
=
a12 4R2 −= 2
l12 4R2 − R 2 − 3= 4 − (2 − =
3) 2+ 3
2 2 2 2 Além disso uma consequência importante é que a razão AH : OF é
igual a 2 : 1 também. E além disso o baricentro (G) divide o segmento
)
2
R R
3= 4 − (2 − =
3) 2+ 3 HO na razão 2 : 1.
2 2

CÍRCULO DE NOVE PONTOS


OUTRAS PROPRIEDADES GEOMÉTRICAS
Num triângulo ABC, sejam M, N e P os pontos médios dos lados
IMPORTANTES BC, AC e AB. Sejam H1, H2 e H3 os pés das alturas de A, B e C, e
seja H ortocentro de ABC. Sejam ainda A’, B’ e C’ pontos médios dos
RETA DE SIMSON-WALLACE segmentos AH, BH e CH. Tem-se então que os pontos M, N, P, H1,
Se perpendiculares são traçadas a partir de um ponto sobre H2, H3, A’, B’ e C’ são concíclicos, ou seja, existe uma circunferência
o circuncírculo de um triângulo a seus lados, suas interseções com passando por todos esses pontos. O centro dessa circunferência é o
os lados do triângulo são colineares e pertencem à Reta de Simson- ponto médio do segmento OH, que liga o circuncentro O ao ortocentro
Wallace. (A recíproca também é verdadeira.) H do triângulo ABC.

346

PM_BOOK16 - MAT.indb 346 25/11/2022 19:17:14


GEOMETRIA PLANA: RELAÇÕES MÉTRICAS NOS POLÍGONOS REGULARES E POTÊNCIA DE PONTO

TEOREMA DE CARNOT

Exemplo:
Considerando o círculo da figura de centro O, calcule
Pot(O) A + Pot(O)B + Pot(O)C.

Sobre os lados BC, AC e AB de um triângulo ABC, tomam-se os


pontos D, E e F, respectivamente. Tem-se que as perpendiculares em
D, E, F aos lados aos quais eles pertencem são concorrentes se, e
somente se,
2 2 2 2 2 2
AF − FB + BE − EC + CD − DA =
0

POTÊNCIA DE PONTO
A potência de um ponto P em relação a um círculo de centro O e
Resolução:
raio R é dada por Pot (O)=
P d2 − R2, onde d é a distância de P ao centro
do círculo. Pot (O) A =
OA 2 − R2 =
32 − 52 =−
9 25 =
−16

P exterior ao círculo ⇒ d > R ⇒ Pot(O)P > 0 Pot (O)B = OB2 − R2 = 52 − 52 = 0


P pertence ao círculo ⇒ d = R ⇒ Pot(O)P = 0 Pot (O)C = OC2 − R2 = 72 − 52 = 49 − 25 = 24
P interior ao círculo ⇒ d < R ⇒ Pot(O)P < 0 Pot(O) A + Pot(O)B + Pot(O)C =−16 + 0 + 24 =8

Se um ponto está sobre uma circunferência, então a sua potência


em relação à essa circunferência é nula. POTÊNCIA DE PONTO EXTERIOR
Observe nas figuras a seguir que, pelo teorema de Pitágoras, Se por um ponto P exterior a uma circunferência são traçadas
temos PT 2 = d2 − R2 = Pot (O)P . duas secantes PAB e PCD a essa circunferência, então PA ⋅ PB = PC ⋅ PD.

347

PM_BOOK16 - MAT.indb 347 25/11/2022 19:17:16


GEOMETRIA PLANA: RELAÇÕES MÉTRICAS NOS POLÍGONOS REGULARES E POTÊNCIA DE PONTO

Resolução:
Sabe-se que PC2 = PA ⋅ PB ⇔ x 2 = 2 ⋅ 5 ⇔ x = 10

Se por um ponto P exterior a uma circunferência de raio R e


distante d unidades de seu centro (d > R) é traçada uma secante PAB
a essa circunferência, então PA ⋅ PB = d2 − R2.

PA ⋅ PB = PC ⋅ PD

Exemplo:
Calcule x na figura a seguir:

PA ⋅ PB = d2 − R2

Exemplo:
Seja P um ponto exterior a um círculo de centro O e raio R e tal
que OP = R 3 . Traça-se por P a secante PAB ao círculo. Se PA = R,
então calcule AB em função de R.
Resolução:

Resolução:
10 1
PA ⋅ PB = PC ⋅ PD ⇔ 2 ⋅ 5 = 3 ⋅ ( 3 + x ) ⇔ x = −3= .
3 3

Se por um ponto P exterior a uma circunferência são traçadas


uma secante PAB e uma tangente PT a essa circunferência, então
PT=
2
PA ⋅ PB.
Usando a proposição anterior, temos:
(R 3 )
2
PA ⋅ PB = OP2 − R2 ⇔ R ⋅ (R + x ) = − R2 ⇔ R (R + x ) = 2R2 ⇔ x = R
Observe que você poderia prolongar PO até encontrar a
circunferência, obtendo uma segunda secante, e encontraria a mesma
relação.

POTÊNCIA DE PONTO INTERIOR


Se por um ponto P interior a uma circunferência são traçadas duas
cordas APB e CPD nessa circunferência, então PA ⋅ PB = PC ⋅ PD.

2
PT= PA ⋅ PB

Exemplo:
Calcule x na figura a seguir.

PA ⋅ PB = PC ⋅ PD

348

PM_BOOK16 - MAT.indb 348 25/11/2022 19:17:17


GEOMETRIA PLANA: RELAÇÕES MÉTRICAS NOS POLÍGONOS REGULARES E POTÊNCIA DE PONTO

Exemplo: A seguir apresentamos a posição do eixo radical para as diversas


Calcule x na figura. posições relativas entre os círculos.
• CIRCUNFERÊNCIAS EXTERIORES

Resolução:
AP ⋅ BP = CP ⋅ DP ⇔ ( 4x − 2) ⋅ 2x = ( x + 1) ⋅ 4x ⇔ 4x 2 − 8x = 0 ⇔ 4x ( x − 2) = 0 ⇔ x = 2
+ 1) ⋅ 4x ⇔ 4x 2 − 8x = 0 ⇔ 4x ( x − 2) = 0 ⇔ x = 2
Note que como 2x e 4x são medidas de segmentos de reta, então
x ≠ 0. • CIRCUNFERÊNCIAS TANGENTES EXTERIORMENTE

Se por um ponto P interior a uma circunferência de raio R e


distante d unidades de seu centro (d < R) é traçada uma corda APB
nessa circunferência, então PA ⋅ PB = R2 − d2.

• CIRCUNFERÊNCIAS SECANTES

PA ⋅ PB = R2 − d2

Exemplo:
Calcule x na figura, onde O é o centro da circunferência.

• CIRCUNFERÊNCIAS TANGENTES INTERIORMENTE

Resolução:
PA ⋅ PB = R2 − d2 ⇔ 8 ⋅ 3 = x 2 − 42 ⇔ x 2 = 8 ⇔ x = 2 2

EIXO RADICAL
O lugar geométrico dos pontos cujas potências em relação a dois
círculos não concêntricos são iguais é uma reta perpendicular à reta que
une os centros dos dois círculos e é chamado eixo radical dos círculos.
Se (e.r.) é o eixo radical dos círculos de centro O1 e O2, então
P ∈ ( e.r.) ⇔ Pot (O1)P =
Pot (O2 )P.

349

PM_BOOK16 - MAT.indb 349 25/11/2022 19:17:18


GEOMETRIA PLANA: RELAÇÕES MÉTRICAS NOS POLÍGONOS REGULARES E POTÊNCIA DE PONTO

• CIRCUNFERÊNCIAS INTERIORES Exercício Resolvido

02. Na figura, PA é tangente à circunferência em A, e B é ponto


médio de PC. A medida de PC, em cm, é:

a) 12 2 c) 16
b) 14 2 d) 20

Resolução: C
Exercício Resolvido
Como B é o ponto médio de PC temos que PB = BC = x e assim
01. Uma circunferência tangencia o lado BC de um triângulo ABC podemos aplicar a potência do ponto exterior.
no ponto F e intersecta os lados AB e AC desse triângulo, nos
( )
2

pontos E e D respectivamente, conforme mostra a figura. PA 2 = PB ⋅ PC ⇒ 8 2 = x ⋅ 2x ⇒ 64 ⋅ 2 = 2x 2 ⇒ x 2 = 64 ⇒ x = 8

Como PC = 2x ⇒ PC = 2 ⋅ 8 = 16

Exercício Resolvido

03. Seja a circunferência e duas de suas cordas, AB e CD . A


medida de CD, em cm, é
C

a
A 16 cm 2 cm B

2a

Sabendo que essa circunferência passa pelo ponto A, a distância a) 10. b) 12. c) 14. d) 16.
entre os pontos D e E, em cm, é igual a
a) 10,5. Resolução: B
b) 10,9. Pela potência do ponto interior teremos
c) 11,3. a ⋅ 2a = 16 ⋅ 2 ⇒ 2a² = 16 ⋅ 2 ⇒ a² = 16 ⇒ a = 4
d) 11,7. Como CD = a + 2a = 3a = 3a = 3 ⋅ 4 = 12

Resolução: A
Calculando: Exercício Resolvido
CD ⋅⋅ CA
CA = = CF
CF2⇒ ⇒44⋅ (⋅4( 4+ +AD ) = ) 6= 26⇒
2 AD = 5
2
CD AD ⇒ AD = 5
2 4 ⋅ ( 4 + AE ) = 8 ⇒
04. Na figura, t é tangente à circunferência em B. Se AC = 8 cm e
CD⋅⋅ BA
⋅ CA= = BF 2⇒
4 4⋅ (⋅4( 4+ +AE ) = ) =826⇒
BE 2 2 2AE = 12
BE BA = BFCF⇒⇒ AD ⇒AEAD = 5
= 12 CD = 12 cm, então a medida de AB, em cm, é
141 47
BE22=⋅ BA
BC BA= 22BF
2 2
+ CA ⇒2−42⋅ (⋅ 4 CA) ⋅= cos
BA+⋅AE 82Â⇒⇒AE
142= = 12
216 + 2
2
92 − 22 ⋅ 16 ⋅ 9 ⋅ cos  ⇒ cos Â= = 141 47
= 5 BC = BA + CA − 2 ⋅ BA ⋅ CA ⋅ cos  ⇒ 14 = 16 + 9 − 2 ⋅ 16 ⋅ 9 ⋅ cos  ⇒ cos288 Â= 96 =
2 141 2 47
288 96
16 + 9 − 2 ⋅ 16 ⋅ 9 ⋅ cos  ED
2 2
⇒ Â=2 ++AD 2= − 2 ⋅ BA ⋅ CA ⋅ cos  ⇒ 47 9 ⋅ cos 441 cos Â= 21141= 47
== AE 12=2 +16
52 −+29⋅ 12− ⋅25⋅⋅16 ⋅ ⇒
47x = Â 2⇒ ⇒
2 2 2 2
BC2cos BA CA − 2 ⋅ AE ⋅ AD ⋅ cos  ⇒ x 2 = 14 2
x=
441 = 10,5
21 96
12
ED2 = AE2 +288 AD2 −96 2 ⋅ AE ⋅ AD ⋅ cos  ⇒ x 2 = 122 + 52 − 2 ⋅ 1296 ⋅5⋅ ⇒ x4= ⇒2288
x= = 10,5
47 441 21
141 47⋅ AD ⋅ cos  ⇒ x 2 = 122 + 52 − 2 ⋅ 12 ⋅ 5 ⋅ 96
47 4
441 2
21
2 22
+ 5 −
2 2
2 ⋅ 2 ⋅5⋅
12 ⇒ x =
2 2
ED
4 = 16 + 9 − 2 16 ⋅ 9 ⋅ cos  ⇒ cos Â= = AE⇒ 2
+
x =
AD 2
=
− 2
= ⋅
10,5
AE ⇒ x 2
= ⇒ x = = 10,5
96 4 2
288 96 96 4 2
47 441 21
= 122 + 52 − 2 ⋅ 12 ⋅ 5 ⋅ ⇒ x2 = ⇒ x= = 10,5
96 4 2
a) 4 10 . b) 2 5 . c) 10 . d) 5.

Resolução: A
Pela potência do ponto exterior teremos
AB² = AC ⋅ AD
Temos que AD = AC + CD = 8 + 12 = 20
AB² = AC ⋅ AD ⇒ AB² = 8 ⋅ 20 = 160 ⇒ AB = 160 ⇒ AB =
4 10

350

PM_BOOK16 - MAT.indb 350 25/11/2022 19:17:19


GEOMETRIA PLANA: RELAÇÕES MÉTRICAS NOS POLÍGONOS REGULARES E POTÊNCIA DE PONTO

Exercício Resolvido 05. Calcule a medida do apótema de um hexágono regular inscrito


em uma circunferência de área 36π cm²:
05. Se A, B, C e D são pontos da circunferência, o valor de x é a) 6 cm d) 3 3
múltiplo de
b) 3 cm e) 3 2 cm
c) 6 3

06. Na figura, as dimensões estão em centímetros. O valor de x é

a) 5 b) 6 c) 7 d) 8

Resolução: B
Pela potência do ponto exterior teremos
PA ⋅ PB = PC ⋅ PD ⇒ x ⋅ ( x + 8 ) = ( x − 2) ⋅ ( ( x − 2) + 14 ) ⇒ x 2 + 8x = ( x − 2)( x + 12)
) = ( x − 2) ⋅ ( ( x − 2) + 14 ) ⇒ x 2 + 8x = ( x − 2)( x + 12)
a) 16 cm c) 20 cm
x 2 + 8x = ( x − 2)( x + 12) ⇒ x 2 + 8x = x 2 + 12x − 2x − 24
b) 18 cm d) 22 cm
8x − 10x =−24 ⇒ −2x =−24 ⇒ x =
12
07. Calcule a área de um triângulo equilátero inscrito em uma
circunferência de diâmetro 12 cm:
a) 18 3 cm² d) 24 3 cm²
EXERCÍCIOS DE
b) 20 3 cm² e) 27 3 cm²

FIXAÇÃO c) 21 3 cm²

08. Na figura, as dimensões estão em centímetros. O valor de x é


01. Determine a medida do lado de um triângulo equilátero inscrito
em uma circunferência de raio 4 3 cm:
a) 10 cm b) 12 cm c) 14 cm d) 16 cm e) 18 cm

02. Na figura, as dimensões estão em centímetros. O valor de x é

a) 20 cm c) 22 cm
b) 21 cm d) 23 cm

09. Determine a área de um quadrado inscrito em uma circunferência


com raio igual a 5 cm.
a) 3 cm b) 4 cm c) 5 cm d) 6 cm
a) 50 cm² d) 25 cm²
03. Calcule a medida do lado de um quadrado inscrito em uma b) 40 cm² e) 20 cm²
circunferência de comprimento 16π cm: c) 30 cm²
a) 8 cm c) 8 2 cm e) 8 3 cm
b) 10 cm d) 10 2 cm 10. Na figura, as dimensões estão em centímetros. O valor de x é

04. Na figura, as dimensões estão em centímetros. O valor de x é

a) 3 cm c) 5 cm
a) 3 cm b) 4 cm c) 5 cm d) 6 cm b) 4 cm d) 6 cm

351

PM_BOOK16 - MAT.indb 351 25/11/2022 19:17:20


GEOMETRIA PLANA: RELAÇÕES MÉTRICAS NOS POLÍGONOS REGULARES E POTÊNCIA DE PONTO

EXERCÍCIOS DE 08. O valor de x na figura abaixo é igual a

TREINAMENTO
01. Determine a medida do lado de um triângulo equilátero
circunscrito em uma circunferência de raio 2 cm:
a) 1 3 cm c) 3 3 cm e) 5 3 cm
b) 2 3 cm d) 4 3 cm

02. Na circunferência representada na figura abaixo, o comprimento


da corda CD é a) 7 c) 9
b) 8 d) 10

09. Determine a área de um quadrado circunscrito em uma


circunferência com raio igual a 6 cm.
a) 196 cm² d) 121 cm²
b) 169 cm² e) 100 cm²
c) 144 cm²

10. O valor de x na figura abaixo é igual a

a) 2 b) 3 c) 4 d) 5

03. Calcule a medida do lado de um quadrado circunscrito em uma


circunferência de comprimento 16π cm:
a) 8 cm b) 10 cm c) 12 cm d) 14 cm e) 16 cm

04. O valor de x na figura abaixo é igual a

a) 7 c) 9
b) 8 d) 10

11. (EEAR) Em um triângulo equilátero de 12 3 m de perímetro, a


soma das medidas dos raios das circunferências inscrita e circunscrita
a esse triângulo, em m, é
a) 5. c) 7.
a) 15/2 b) 15/4 c) 7/2 d) 7/4
b) 6. d) 8.
05. Calcule a medida do lado de um hexágono regular circunscrito em
uma circunferência de área 9π cm²: 12. O valor de x na figura abaixo é igual a
a) 2 cm b) 3 cm c) 2 3 cm d) 3 3 cm e) 3 2 cm

06. O valor de x na figura abaixo é igual a

a) 6 c) 8
b) 7 d) 9

a) 15/2 b) 15/4 c) 7/2 d) 7/4 13. (EEAR) A razão entre as medidas dos apótemas do quadrado
inscrito e do quadrado circunscrito numa circunferência de raio R é
07. Calcule a área de um triângulo equilátero circunscrito em uma
circunferência de diâmetro 8 cm: 2
a) . c) 2.
2
a) 48 3 cm² c) 51 3 cm² e) 57 3 cm²
3 d) 2 3 .
b) 50 3 cm² d) 54 3 cm² b) .
2

352

PM_BOOK16 - MAT.indb 352 25/11/2022 19:17:21


GEOMETRIA PLANA: RELAÇÕES MÉTRICAS NOS POLÍGONOS REGULARES E POTÊNCIA DE PONTO

14. O valor de x na figura abaixo é igual a 21. (EPCAR) O lado de um quadrado inscrito em um disco de raio R é
igual a a – b, e o lado do triângulo equilátero inscrito no mesmo disco
é a + b. Então b/a vale:
a) 5 − 2 6. c) 5 + 2 6.
3
b) . d) 13.
7
22. (CMRJ) A medida, em cm, do lado de um pentágono regular cujas
diagonais medem 3 + 3 5 cm é:
a) 6. d) 9.
b) 7. e) 10.
c) 8.

a) 6 b) 7 c) 8 d) 9 23. (CMRJ) Um relógio circular foi construído de modo que os números


que indicam as horas estão nos vértices de um polígono regular.
15. Na figura, ABC é um triângulo no qual AB = 9 cm e AC = 8 Nesse relógio, o ponteiro das horas é 5 cm menor que o ponteiro dos
cm. Uma circunferência, passando por B e C, intercepta AB e AC, minutos, cujo comprimento é igual ao raio da circunferência onde o
respectivamente, em P e R. Calcule CR, sabendo que BP = 5 cm. polígono em questão, de lado medindo 15 2 – 3 cm, se encontra
inscrito. Os dois ponteiros estão presos no centro do círculo. Assim, o
comprimento do ponteiro menor do relógio, em centímetros, é igual a:
a) 30. d) 15.
b) 24. e) 10.
c) 20.

24. Qual é o perímetro de um quadrilátero convexo inscrito em


uma circunferência de raio unitário, sabendo-se que foi construído
utilizando-se, pelo menos uma vez e somente, os lados do triângulo
a) 15/2 b) 15/4 c) 7/2 d) 7/4 equilátero, quadrado e hexágono regular inscritos nessa circunferência?
a) 3+ 2+2 d) 3+2 2+2
16. (EEAR) Um hexágono regular ABCDEF, de 30 3 cm de perímetro,
b) 3 + 2 2 +1 e) 2 ( 3 + 2 + 1)
está inscrito em um círculo de raio R. A medida de sua diagonal
AC, em cm, é c) 2 3 + 2 +1
a) 5 3 . b) 5. c) 15 3 . d) 15.
25. (EPCAR) O apótema de um hexágono regular é igual à altura de
17. (EEAR) Seja um triângulo equilátero de apótema medindo um triângulo equilátero cujo lado mede 4 cm. A área do hexágono
2 3 cm. O lado desse triângulo mede ______ cm. mede, em cm²
a) 6 b) 8 c) 9 d) 12 a) 4 3 c) 18 3
b) 16 3 d) 24 3
18. (EEAR) Dado um hexágono regular de 6 cm de lado, considere o seu
apótema medindo α cm e o raio da circunferência a ele circunscrita 26. O perímetro do heptágono regular convexo, inscrito num círculo
medindo R cm. O valor de (R + a 3) é de raio 2,5, é um número x ∈ R tal que
a) 12 b) 15 c) 18 d) 25 a) 14 < x < 15 d) 17 < x < 18
b) 15 < x < 16 e) 18 < x < 19
19. (ESA) Um hexágono regular está inscrito em uma circunferência
de diâmetro 4 cm. O perímetro desse hexágono, em cm, é c) 16 < x < 17
a) 4π. d) 6.
27. (ITA) Um hexágono regular e um quadrado estão inscritos no
b) 8π. e) 12. mesmo círculo de raio R e o hexágono possui uma aresta paralela a
c) 24. uma aresta do quadrado. A distância entre estas arestas paralelas será
3− 2 2 −1
20. (EEAR) Sejam um hexágono regular e um triângulo equilátero, a) R d) R
2 2
ambos de lado l. A razão entre os apótemas do hexágono e do
triângulo é 2 +1 3 −1
b) R e) R
2 2
3 +1
c) R
2

28. (ITA) A razão entre as áreas de um triângulo equilátero inscrito


numa circunferência e de um hexágono regular, cujo apótema mede
10 cm, circunscrito a esta mesma circunferência é
1 1 3
a) 4. c) 2. a) b) 1 c) d) e) n.d.a.
2 3 8
b) 3. d) 1.

353

PM_BOOK16 - MAT.indb 353 25/11/2022 19:17:21


GEOMETRIA PLANA: RELAÇÕES MÉTRICAS NOS POLÍGONOS REGULARES E POTÊNCIA DE PONTO

29. (EPCAR) A figura abaixo representa um octógono regular tal que 34. Por um ponto distante 7 cm do centro de uma circunferência de
CH = 6 cm. 5 cm de raio traça-se uma secante de modo que sua parte externa
é 2/3 da secante total. Calcular, em cm, o comprimento da secante.
a) 4 2 c) 4 e) 6
b) 4 3 d) 5

35. (CMRJ) Na figura abaixo, ACDF é retângulo, B ∈ AC e E ∈ FD. Os


pontos B, C e E pertencem à circunferência de centro O. Sabe-se que
AB e AF são congruentes e, além disso, a medida de OA é 8 cm e a
medida de OC é 5 cm. Calcule a área do retângulo ACDF em cm².

A área desse polígono, em cm², é igual a


a) 56 ( 2 − 1) c) 72 ( 2 − 1)
b) 64 ( 2 − 1) d) 80 ( 2 − 1)

30. (EPCAR) A área do losango ABCO da figura abaixo mede 24 cm².


O lado do hexágono regular ABCDEF é, em cm, igual a
a) 24 b) 32 c) 36 d) 39 e) 48

36. (CN) Na figura abaixo, temos AB = 55 cm e AC = 5 cm.

a) 4 4 3 c) 4
b) 4 3 d) 16 3

31. (EPCAR) O perímetro de um quadrado inscrito numa circunferência


mede 20 2 m. O apótema do hexágono regular inscrito nessa mesma Calcule a razão entre a área do triângulo ABC e a área do triângulo
circunferência mede, em m, BDC.
6 5 11
a) 2 3 5 3 a) b) 1 c) d) e) 2
c) 5 6 6
2 3 2
b)
2 d) 5 3
37. (CN) Em um círculo as cordas AB e CD são perpendiculares e
se cortam no ponto I. Sabendo que AI = 6 cm, IB = 4 cm e CI = 2 cm,
32. (CN) Um hexágono regular ABCDEF está inscrito em uma
podemos dizer que a área do círculo é de:
circunferência de raio 6. Traçam-se as tangentes à circunferência nos
pontos A, B, D e F, obtendo-se, assim, um quadrilátero circunscrito a a) 144π cm² c) 120π cm² e) 50π cm²
essa circunferência. Usando-se 1,7 para raiz quadrada de 3, qual é o b) 100π cm² d) 60π cm²
perímetro desse quadrilátero?
a) 54,4 c) 40,8 e) 30,6 38. Considere o triângulo ABC retângulo, em que os catetos medem
b) 47,6 d) 34,0 AB = 4 e BC = 2. Traça-se a bissetriz do ângulo B, que intersecta a
circunferência circunscrita a ABC no ponto D. Quanto mede BD?
33. (CN) Um professor usa para medir comprimentos uma unidade a) 2 3 + 1 c) 3 2 e) 4
denominada “nix”, definida como 1 nix = 3 centímetros. Ele mediu 9 5
na unidade nix as diagonais de um hexágono regular de lado 1 cm e b) d) 2 + 5
5
encontrou para as menores x e para as maiores y. Pode-se concluir que
x e y são, respectivamente, 39. (CN) Se a distância do ponto P ao centro de um círculo aumentar
a) números racionais. 2
de de sua medida (x) a potência do ponto P em relação ao círculo
b) números irracionais. 5
aumentará de:
c) um número inteiro e um número irracional.
a) 20% de x² d) 86% de x²
d) um número irracional e um número inteiro.
b) 42% de x² e) 92% de x²
e) um número racional não inteiro e um número irracional.
c) 96% de x²

354

PM_BOOK16 - MAT.indb 354 25/11/2022 19:17:23


GEOMETRIA PLANA: RELAÇÕES MÉTRICAS NOS POLÍGONOS REGULARES E POTÊNCIA DE PONTO

___
40. Um ponto P está a 9 unidades do centro de uma circunferência 06. (CN) Na figura abaixo, PA é uma secante ao círculo, PT é uma
de raio 15. Quantas cordas distintas dessa circunferência contêm P e tangente ao círculo e BC é uma corda do círculo. Qual das relações
possuem comprimentos inteiros? abaixo sempre será válida?
a) 11. c) 13. e) 29.
b) 12. d) 14.

EXERCÍCIOS DE

COMBATE
01. (CN) Quando uma pessoa caminha em linha reta uma distância
x, ela gira para a esquerda de um ângulo de 60º; e quando caminha
=
em linha reta uma distância y x 2 − 2 , ela gira para a esquerda de
um ângulo de 45º. Caminhando x ou y a partir de um ponto P, pode-
se afirmar, para qualquer que seja o valor de x, é possível chegar ao ___ ____ ____ ___
ponto P descrevendo um: PD PT PT IG
a) ____
= ___
d) ___
= ___
I. Pentágono convexo PT PA CI PI
___ ____ ___ ___
II. Hexágono convexo PD PT PD CI
b) ____
= ___ e) ___
= ___
III. Heptágono convexo PT AD BI PA
___ ___
IV. Octógono convexo CI AI
c) ___
= ___
O número de assertivas verdadeiras é: BI DI
a) 0 c) 2 e) 4
b) 1 d) 3 07. (CN) Dois segmentos de uma reta, AB e CD, interceptam-se
interiormente no ponto O. Sabe-se que as medidas de AO e OB são
02. (CN) Um hexágono regular ABCDEF tem lado 3 cm. Considere os respectivamente, 3 cm e 4 cm, e que as medidas de CO e OD são,
pontos: M, pertencente a AB, tal que MB igual a 1 cm; N, pertencente respectivamente, 2 cm e 6 cm. Qual o número de pontos do plano,
a CD, tal que ND igual a 1 cm; e P, pertencente a EF, tal que PF igual a determinado por AB e CD, que equidistam dos pontos A, B, C e D?
1 cm. O perímetro, em centímetros, do triângulo MNP é igual a: a) zero. c) dois. e) infinito.
a) 3 15 c) 3 19 e) 3 23 b) um. d) três.
b) 3 17 d) 3 21
08. (CN) Define-se potência de um ponto P em relação a um círculo C, de
03. (CN) Sejam ABCDEFGHIJKL os vértices consecutivos de um centro O e raio r, como sendo o quadrado da distância de P a O, menos o
dodecágono regular num círculo de raio 6 . O perímetro do triângulo quadrado de r. Qual é a potência de um dos vértices do hexágono regular
de vértices AEH é igual a: circunscrito a um círculo de raio r, em relação a este círculo?
2r2
a) 3[3 + 2 + 3 ] . d) 3[2 + 2 + 3 3 ] . a) c) r2
e)
r2
3 3 6
b) 3[1 + 2 + 3 ] . e) 3[1 − 2 + 2 3 ] .
r2 r2
b) d)
c) 3[1 + 2 2+ 3] . 2 4

04. (CN) O quadrilátero ABCD está inscrito num círculo de raio unitário. 09. (CN) Considere um triângulo equilátero ABC, inscrito em um
Os lados AB, BC e CD são respectivamente, os lados do triângulo círculo de raio R. Os pontos M e N são, respectivamente, os pontos
equilátero, do quadrado e do pentágono regular inscrito no círculo. Se médios do arco menor AC e do segmento BC . Se a reta MN também
x é a medida do lado AD do quadrilátero, pode-se afirmar que: intercepta a circunferência desse círculo no ponto P, P ≠ M, então o
segmento NP mede
Observação: CD é aproximadamente igual a 1,2.
R 7 3R 7 R 5
a) 1,0 < x < 1,2 d) 1,6 < x < 1,8 a) c) e)
2 14 3
b) 1,2 < x < 1,4 e) 1,8 < x < 2,0
3R 3
c) 1,4 < x < 1,6 b) d) R 5
2 7
05. (CN) Sobre os lados de um hexágono regular de 4 cm de lado,
e exteriormente a ele, constroem-se quadrados, de modo que cada 10. (IME) Numa circunferência de centro O e diâmetro AB = 2R,
quadrado tenha um lado em comum com o hexágono. Calcular a área prolonga-se o diâmetro AB até um ponto M, tal que BM = R. Traça-
do dodecágono cujos vértices são os vértices dos quadrados que não se uma secante MNS tal que MN = NS, onde N e S são os pontos
são vértices do hexágono: de interseção da secante com a circunferência. Determine a área do
triângulo MOS.
a) 48 ( )
3 + 2 cm2 d) 192 cm²
5 2 5 2 3 2
a) R c) R e) R
b) 50 ( 3 + 2) cm2 e) 36 cm² 4 2 2

c) 24 ( )
3 + 4 cm2 b)
15 2
4
R d) 15 2
2
R

355

PM_BOOK16 - MAT.indb 355 25/11/2022 19:17:24


GEOMETRIA PLANA: RELAÇÕES MÉTRICAS NOS POLÍGONOS REGULARES E POTÊNCIA DE PONTO

AE EB GC
5 Da figura A, F e C são pontos de tangência. Se = =
3 5 2
,

DESAFIO PRO
AP
o valor de é:
PC

a) 8/3

1 Um grupo de 4 paraquedistas, desejando efetuar um salto


perfeito, desenhou um croqui de como eles ficariam no
momento da queda livre, conforme a figura abaixo.
b) 7/4
c) 9/5
d) 21/8
Como um dos saltadores era muito observador das coincidências
e) 17/9
matemáticas, ao ver o esquema, questionou-se de qual seria a
distância entre os paraquedistas A e C.
Ele verificou que os pontos M e N, no esquema a seguir, indicam
essa distância.
De acordo com o questionamento, a expressão que permite
calcular a distância entre os pontos M e N, em função da medida
a indicada na figura, é dada por:

(
a) a 2 2 − 2 ) GABARITO
b) a 2 ( 2 +1) EXERCÍCIOS DE FIXAÇÃO
01. B 04. A 07. E 10. A
(
c) a 2 5 + 2 2 ) 02. B 05. D 08. B
03. C 06. B 09. A
(
d) a 2 2 + 2 ) EXERCÍCIOS DE TREINAMENTO
01. D 11. D 21. A 31. C
02. C 12. D 22. A 32. A
03. E 13. A 23. E 33. C
04. A 14. D 24. B 34. E
05. C 15. C 25. D 35. D
06. B 16. D 26. B 36. C

2 O perímetro do heptágono regular convexo, inscrito num


círculo de raio 2,5 , é um número x ∈  tal que:
07. A
08. C
17. D
18. B
27. A
28. D
37. E
38. C
a) 14 < x < 15 c) 16 < x < 17 e) 18 < x < 19
09. C 19. E 29. C 39. C
b) 15 < x < 16 d) 17 < x < 18
10. B 20. B 30. A 40. B
EXERCÍCIOS DE COMBATE

3 (ITA 2007) Seja Pn um polígono regular de n lados, com


n > 2. Denote por an o apótema e por bn o comprimento de
um lado de Pn. O valor de npara o qual valem as desigualdades
01. D
02. D
04. B
05. A
07. B
08. C
10. B

e , pertence ao intervalo: 03. B 06. A 09. C


a) 3 < n < 7 c) 8 < n < 11 e) 12 < n < 15 DESAFIO PRO
b) 6 < n < 9 d) 10 < n < 13 01. D 03. B 05. C
02. B 04. A

4 Considere a circunferência de centro O e raio r e os dados


abaixo. ANOTAÇÕES

Sendo todos os comprimentos dos segmentos medidos em


centímetros, então, a medida de PC é, em centímetros, um
número do intervalo:
a) [8, 16 [ b) [16, 24 [ c) [24, 32 [ d) [32, 40 [

356

PM_BOOK16 - MAT.indb 356 25/11/2022 19:17:24


GEOMETRIA PLANA: ÁREAS

DEFINIÇÃO DE ÁREA Demonstração:


Cada figura plana está associada a um número positivo chamado
área que possui as seguintes propriedades:
P1: Figuras planas congruentes possuem a mesma área, ou seja,
são equivalentes.
P2: Se uma figura plana P for decomposta em duas outras P1 e P2
então a área de P é a soma das áreas de P1 e de P2.
P3: A área de um retângulo de base b e altura h é igual ao produto
b ⋅ h.
SADE =
SBCF ⇒ SABCD =
SDEFC =
b⋅h
RETÂNGULO
Exemplo: Calcule a área do paralelogramo da figura.
Como estabelecido na propriedade P1, a área de um retângulo de
base b e altura h é igual ao produto b ⋅ h.

S=b⋅h

SABCD = 5 ⋅ 3 = 15 u.a.
Exemplo: Calcule a área do retângulo da figura.

TRIÂNGULOS
Nessa seção serão apresentadas diversas fórmulas para o cálculo
da área de um triângulo.
A área de um triângulo é igual à metade do produto de um dos
lados pela altura relativa a ele.

Sret. = 4 ⋅ 2 = 8 u.a.

a ⋅ hA b ⋅ hB c ⋅ hC
PARALELOGRAMO =
SABC = =
2 2 2
A área de um paralelogramo de base b e altura h é igual ao produto
b ⋅ h, ou seja, é igual à área do retângulo de mesma base e altura.

Demonstração:

S=b⋅h

357

PM_BOOK16 - MAT.indb 357 25/11/2022 19:17:26


GEOMETRIA PLANA: ÁREAS

Sejam AD || BC e CD || AB, então o #ABCD é um paralelogramo e AC ⋅ BE AC b⋅c


∆ABC ≡ ∆CDA (L.L.L.). Logo, SABC = =⋅ ABsen Aˆ =sen Aˆ .
2 2 2
1 1 a ⋅ hA
Logo, SABC = SCDA, então SABC = SABCD = ⋅ a ⋅ hA = .
2 2 2 Exemplo: Calcule a área do triângulo da figura a seguir.

Note que, quando o triângulo é obtusângulo, o pé da altura


pode estar no prolongamento do lado, mas a fórmula funciona do
mesmo modo.

c ⋅ hC
SABC =
2

3⋅ 4 3
S= ⋅ sen60° = 6 ⋅ = 3 3 u.a.
Exemplo: Calcule a área dos triângulos das figuras a seguir. 2 2

A área de um triângulo é igual ao produto de seu semiperímetro


pelo raio do círculo inscrito nesse triângulo.

SABC= p ⋅ r

S = 3 ⋅ 3 = 9 u.a. S = 2 ⋅ 3 = 6 u.a.

A área de um triângulo é igual à metade do produto de dois lados Demonstração:


adjacentes multiplicado pelo seno do ângulo entre eles.
c ⋅r b ⋅r a⋅r  a + b + c 
SABC =
SABI + SACI + SBCI = + + =  ⋅ r =⋅
p r.
2 2 2  2 

Exemplo: Calcule a área do triângulo da figura a seguir, onde


7 3 − 39
r= .
6

b⋅c a⋅c a⋅b


=
SABC = senAˆ = senBˆ senCˆ
2 2 2

Demonstração:

3 + 4 + 13 7 + 13
=p =
2 2
7 + 13 7 3 − 39 49 3 − 7 39 + 7 39 − 13 3
S = p ⋅r = ⋅ = = 3 3 u.a.
2 6 12
7 + 13 7 3 − 39 49 3 − 7 39 + 7 39 − 13 3
S = p ⋅r = ⋅ = = 3 3 u.a.
2 6 12

AC ⋅ BE A área de um triângulo é igual ao produto da diferença entre o


Seja BE a altura relativa ao lado AC do ∆ABC, então SABC = . semiperímetro e um dos seus lados pelo raio do círculo ex-inscrito
2
relativo a esse lado.
BE
No triângulo retângulo ABE, temos sen Aˆ = ⇔ BE = AB ⋅ sen Aˆ .
AB

358

PM_BOOK16 - MAT.indb 358 25/11/2022 19:17:27


GEOMETRIA PLANA: ÁREAS

Exemplo: Calcule a área do triângulo da figura a seguir, sabendo


39
que R = .
3

SABC = (p − a) ⋅ ra = (p − b ) ⋅ rb = (p − c ) ⋅ rc

Demonstração: abc 3 ⋅ 4 ⋅ 13
=S = = 3 3 u.a.
Na figura AD = p – a e AT = p. 4R 39
4⋅
ID AD r p−a 3
∆ADI ~ ∆ATIA ⇒ = ⇒ = ⇔ p ⋅ r = (p − a) ⋅ ra ⇒ SABC = (p − a) ⋅ ra
IA T AT ra p
p ⋅ r = (p − a) ⋅ ra ⇒ SABC = (p − a) ⋅ ra A área de um triângulo é igual ao dobro do quadrado do raio do
círculo circunscrito multiplicado pelo produto dos senos de seus ângulos.

A área de um triângulo é igual ao produto dos três lados dividido


pelo quádruplo do raio do círculo circunscrito a esse triângulo.

SABC =2R2 ⋅ senAˆ ⋅ senBˆ ⋅ senCˆ


a⋅b ⋅c
SABC =
4R

Demonstração:
Aplicando a lei dos senos ao ∆ABC, temos:
Demonstração: a b c
= = = 2R .
sen Aˆ senBˆ senCˆ
a ⋅ b ⋅ c 2R sen Aˆ ⋅ 2R senBˆ ⋅ 2R senCˆ
⇒ SABC = = =2R2 ⋅ sen Aˆ ⋅ senBˆ ⋅ senCˆ
4R 4R

FÓRMULA DE HERON
A área de um triângulo é igual à raiz quadrada do produto do
semiperímetro pela diferença entre o semiperímetro e cada um dos
lados do triângulo.

SABC = p ⋅ (p − a ) ⋅ (p − b ) ⋅ (p − c )

ˆ = 90°.
AOF é um diâmetro do círculo circunscrito, então ACF

ABC ˆ =AC ⇒ BAD
ˆ =AFC ˆ =CAF
ˆ
2 a+b+c
onde p = é o semiperímetro do triângulo.
AB AD c hA b⋅c 2
∆BDA ~ ∆FCA ⇒ = ⇒ = ⇔ hA =
AF AC 2R b 2R
a⋅h a bc abc
SABC = A = ⋅ =
2 2 2R 4R

359

PM_BOOK16 - MAT.indb 359 25/11/2022 19:17:29


GEOMETRIA PLANA: ÁREAS

Demonstração: A área de um triângulo é igual à raiz quadrada do produto do raio


do círculo inscrito e dos três raios dos círculos ex-inscritos ao triângulo.

Aplicando o teorema de Pitágoras nos ∆ABD e ∆ACD, temos:


h2A + x 2 =
c2
h2A + ( a − x ) =
2
b2
a2 − b2 + c2
x 2 − ( a − x ) = c2 − b2 ⇔ a ⋅ ( 2x − a) = c2 − b2 ⇔ x =
2

2a
 a2 − b2 + c2   a2 − b2 + c2 
⇒ h2A = c2 − x 2 = ( c + x )( c − x ) =  c + c − =
 2a  2a 
21 1
 a −=
2
b + c 2 ( 2ac
2
  2 2 2

a − b+ c+ c− b )( 2ac − a − c a+ −=  2
 
b b) + c 2 c( a−+ac )− −
2 2 2 2 2
b b+c[b2 − ( a − c=
2
2)
]
⇒ hA2 =c c−2 −+ xa2 =
2 2 2 2 2 2 2
x )( c − x ) =  c + ( c + x)(=c − x ) =  c +  =
 2a 4a  2a   2a 4a   2a  SABC = r ⋅ ra ⋅ rb ⋅ rc
2 1 2 1
2+)=c  1 122 (((2ac
 2 2 2 b2=
) = − ca +−ac −+b= aaa+2+c−+c+b)a22b
2+ )( [
− )(2ac aa)−]2 c−)(c 2ba2++−= ) c1 ⋅)−( 2p [ )( )( ] )
2 2
−)(bc2 −)(x2ac  − + bcac222+

 cbb2−2−b )(( a b−+c−=2 c2b−)2 a+=
b  (2a⋅ +2p
ac b
− b−+2b
2 2 2 2 2 2
c b  2p
− ( a 2 )2p − 2a=
− −2cc=
⇒4a h Ac22 −= c − x = ( c +=x )( c − x ) =  c +
4a 2
4ac − =
 2a 4a 2a   2a 4a   2a  Demonstração:
41 1 1
2c)( b )(b− +a2a−−cc2)(+b=
−2ac c −=a=1=
b+2 )= ) 21(2⋅ap((+a2⋅ (⋅+p2p
)c2−+⋅ 2)2p
a(b ⋅ (p
)( −
a[ −
2+
2b
2cb)()−(2⋅2p
b(p−)(−−cb=
)( 2c+
2c))])(
a −
2p 2 c−)(2a b )
=
+ c2− ) a ) 1 (2 ⋅ 2p ⋅)2( 2p −2 2b
= [ )(
2 2p( − 2c )(
]
2 )2p − 2a= )
− −  a + c − b  b − a − c= SABC = p ⋅ r = (p − a) ⋅ ra = (p − b ) ⋅ rb = (p − c ) ⋅ rc
2

a 4a c b  b
2ac + a + c − b 2ac − a − c + = a 2
b
4a24a4a2 4a24a
4 S4 =p ⋅ r ⋅ (p − a) ⋅ ra ⋅ (p − b ) ⋅ rb ⋅ (p − c ) ⋅ rc =p ⋅ (p − a) ⋅ (p − b ) ⋅ (p − c ) ⋅ r ⋅ ra ⋅ rb ⋅ rc =S2ABC ⋅ r ⋅ r
bb))(
⋅ (bp +− ac )− c )(b + c − a= ) = 12 2⋅ p⋅( a2p
a 2 +⋅c( 2p
⋅ (p −+ab)−)( ⋅ (2bp
a+ )(cb2p)−⋅−b(p)(
− 2c−b)(c+)2pa −− c2a )(=)b + c − a=) 12 ⋅ 2p ⋅ ( 2p − 2b )( 2p − 2c )( ⇒ 2p −ABC 2a=)
4a 4a ⇒ 4 p −=
( ar) ⋅⋅r(ap⋅ r−b b⋅ r)c ⋅⇒ c ) ⋅=r ⋅ rr ⋅⋅ rra ⋅⋅ rrb ⋅=
2
⇒ S4ABC =p ⋅ r ⋅ (p − a) ⋅ ra ⋅ (p − b ) ⋅ rb ⋅ (p − c ) ⋅ rc = ⇒p SS⋅ ABC (p S−ABC
ABC =p r ⋅ ( p − a ) ⋅ ra ⋅ ( p − ba) bb (cp − cABC
rc S2) ⋅ r⋅ r=
⋅ rpa ⋅ r(bp⋅ r−c a) ⋅ (p − b ) ⋅ (p − c ) ⋅ r ⋅ ra ⋅ rb ⋅ rc =S2ABC ⋅ r ⋅ r
⋅ (p − c ) 4 c
= 2 ⋅ p ⋅ (p − a) ⋅ (p − b ) ⋅ (p − c ) ⇒ S2 = r ⋅ r ⋅ r ⋅ r ⇒ S = r ⋅ r ⋅ r ⋅ r
a ABC a b c ABC a b c ⇒ SABC = r ⋅ ra ⋅ rb ⋅ rc ⇒ SABC = r ⋅ ra ⋅ rb ⋅ rc
2

2 a ⋅ hA
⇒ hA = ⋅ p ⋅ (p − a) ⋅ (p − b ) ⋅ (p − c ) ⇒ SABC = = p ⋅ (p − a) ⋅ (p − b ) ⋅ (p − c )
a 2
A área de um triângulo é igual à raiz quadrada da metade do
a ⋅ hA produto do raio do círculo circunscrito por cada uma das três alturas
⋅ (p − c ) ⇒ SABC = = p ⋅ (p − a) ⋅ (p − b ) ⋅ (p − c )
2 do triângulo.

Exemplo: Calcule a área do triângulo da figura.

R ⋅ hA ⋅ hB ⋅ hC
SABC =
2

Demonstração:
3 + 4 + 13 7 + 13
=p = abc ahA bhB chC
2 2 = = = S=
ABC
4R 2 2 2
7 + 13  7 + 13   7 + 13   7 + 13  7 + 13  13 + 1  13 − 1  7 − 13 
S= ⋅ − 3 ⋅  − 4 ⋅  − 13  = ⋅ ⋅  2SABC  ⋅2S
2S ABC  ABC =
2  2   2   2  2  2   ⋅ 2 ⋅  2 
hA hB hC RhAhBhC RhAhBhC
1 SABC = ⇔ 4RhAhBhCSABC = 8S3ABC ⇔ S2ABC = ⇔ SABC =
  7 + 13  ++ 13
77= 13 (49
7+ + 1) ⋅ (13
− 13
13 13   −13
= 113
17) −+12S  36
7 −2S 13 732S
⋅ 12
= + ABC
313  7 + 13  13 + 1  13 − 1 4R 7 − 13  2 2
− 4 ⋅  − 13S == 4 ⋅⋅ −⋅ 3
⋅  ⋅  −⋅ 4  ⋅  ⋅ =
4 ABC ABC − 13  = ⋅ ⋅ ⋅ =
  2  22  2    2 2hA 2h   h2  2  2   RhA2hBhC  2 RhAhBhC
SABC = B C
⇔ 4RhAhBhCSABC = 8S3ABC ⇔ S2ABC = ⇔ SABC =
1 ( 1
3 3 = 49 − 13) ⋅ (13 −= 1) 36 ⋅ 12= 34R3 2 2
4 4

360

PM_BOOK16 - MAT.indb 360 25/11/2022 19:17:31


GEOMETRIA PLANA: ÁREAS

TEOREMA DE BURLET Demonstração:


Em um triângulo retângulo, a área é igual ao produto dos
segmentos determinados pelo círculo inscrito sobre a hipotenusa.

= BT ⋅ TC
SABC
a⋅h
Demonstração: a h SABC a h
∆ABC ~ ∆A'B'C' ⇒ = =k ⇒ = 2 = . =k 2
a' h' S a'⋅ h' a' h'
BT =− BT p =− b;p CTb; =−p =−
CT c;p r c;=− pr =− a;p SABCa; S=⋅ p =⋅
ABC rp r A 'B'C'
2
= p−⋅ (ap) ⋅−(ap)−⋅ (bp) −
⋅ (pbp)−⋅a;
(cp) S−⇒c )S2ABC
⇒ S= 2
=p ⋅ (p − a) ⋅ (p − b ) ⋅ (p − c ) =p ⋅ r ⋅ BT ⋅ CT =SABC ⋅⋅CT
BT ⋅ CT
pABCpb;⋅ (p rp ⋅ (p − a) ⋅ (p − b ) ⋅ (p − c ) =p ⋅ r ⋅ BT ⋅ CT =SABC ⋅ BT
SABC=−S=
BT CT =− p c; r =− ABC =⋅ p ABC
⇔ S = BT ⋅ CT Note que essa propriedade vale para quaisquer figuras
S⇔
⋅ (p − a) ⋅ (p − b ) ⋅ (p − c ) = ⋅ BT=
p S⋅ rABC CT⋅ CT
⋅BT
ABC = p ⋅ ( p − a )ABC
=S ⋅ (p⋅ BT
− b⋅)CT ⋅ (p − c ) ⇒ S2ABC =p ⋅ (p − a) ⋅ (p − b ) ⋅ (p − c ) =p ⋅ r ⋅ BTsemelhantes,
ABC
⋅ CT =SABC ⋅ BT
não⋅ CT
só para triângulos.
⇔ SABC =BT ⋅ CT
Exemplo: Sejam M e N pontos médios dos lados AB e AC,
respectivamente, de um triângulo ABC. Calcule a razão entre as áreas
FIGURAS EQUIVALENTES E RAZÃO dos triângulos AMN e ABC.

ENTRE ÁREAS
Figuras equivalentes são aquelas que possuem a mesma área.
Se dois triângulos possuem bases e alturas congruentes, então
eles são equivalentes.

SABC = SA 'BC
BC
MN é base média do triângulo ABC, então MN || BC e MN = .
2
2 2
S  MN   1  1
MN  BC ⇒ ∆AMN  ∆ABC ⇒ AMN
=  =  = 
SABC  BC   2  4
Se dois triângulos são semelhantes, então a razão entre suas áreas
é o quadrado da razão de semelhança. Se dois triângulos possuem bases sobre a mesma reta e vértice
comum, então a razão entre suas áreas é igual à razão entre suas bases.

SABC a
=
SAB'C ' a'

SABC
∆ABC ~ ∆A'B'C' ⇒ k2
=
SA 'B'C '
Demonstração:
onde k é a razão de semelhança. Seja h a distância do ponto A à reta r, então h é altura do ∆ABC e
a⋅h
do ∆AB’C’. Assim, S= ABC
=2 a.
SAB'C' a'⋅ h a'
2

361

PM_BOOK16 - MAT.indb 361 25/11/2022 19:17:32


GEOMETRIA PLANA: ÁREAS

Uma consequência imediata da proposição anterior é que a razão Uma mediana divide o triângulo em duas regiões equivalentes.
entre as áreas em que uma ceviana divide um triângulo é igual à razão Seja AM a mediana relativa ao lado BC do ∆ABC.
entre as medidas dos segmentos em que essa ceviana divide o lado,
S BD SABD BD SACD CD
ou seja, ABD = ; = ; = .
SACD CD SABC BC SABC BC

SABC
S=
ABM S=
ACM
2

SABD SACD SABC


= =
BD CD BC

As três medianas de um triângulo dividem esse triângulo em seis


triângulos equivalentes.
Sejam AM, BN e CP as medianas do ∆ABC, então
Exemplo: Seja um triângulo ABC de lados AB = 4 e AC = 5, e
o ponto D sobre BC é o pé da bissetriz do ângulo Â. Calcule a razão
entre as áreas dos triângulos ABD e ACD.

BD CD BD CD SABC
Pelo teorema da bissetriz interna, temos: = ⇔ = . S=
AGN S=
AGP = S=
SBGM BGP = S=
SCGM CGN
AB AC 4 5 6
S BD 4
= =
A razão entre as áreas dos triângulos ABD e ACD é ABD . Demonstração:
SACD CD 5

Se dois triângulos possuem base comum e o vértice de um deles


pertence a uma ceviana do outro partindo do vértice oposto à base
comum, então a razão entre a área do maior e do menor deles é igual
à razão entre a medida da ceviana e a medida da parte entre o vértice
do menor e a base comum.

SABC AE
=
SBCD DE

Como M, N e P são pontos médios, então os ∆BGC, ∆AGC e ∆AGB


são divididos em duas áreas equivalentes S1, S2 e S3, respectivamente.
AM é mediana ⇒ SABM = SACM ⇒ S1 + 2 ⋅ S3 = S1 + 2 ⋅ S2 ⇔ S2 = S3
BN é mediana ⇒ SBCN = SBAN ⇒ S2 + 2 ⋅ S1 = S2 + 2 ⋅ S3 ⇒ S1 = S3
Demonstração: SABC
⇒ S1 = S2 = S3 =
6
SABE AE S AE S + SACE AE S AE
= ∧ ACE = ⇒ ABE = ⇒ ABC =
SBDE DE SCDE DE SBDE + SCDE DE SBCD DE
As três bases médias de um triângulo dividem o triângulo em
quatro regiões equivalentes.
Sejam M, N e P os pontos médios dos lados do ∆ABC, então

362

PM_BOOK16 - MAT.indb 362 25/11/2022 19:17:33


GEOMETRIA PLANA: ÁREAS

A bissetriz de um dos ângulos de um triângulo divide-o em dois


triângulos cujas áreas estão na mesma razão que os lados adjacentes
ao ângulo.
Seja AD a bissetriz do ângulo  do ∆ABC, então

SABD AB c
= =
SACD AC b

SABC
SMNP= SANP= SBMP= SCMN= S=
4

Demonstração:
Demonstração:
Basta observar que ∆MNP ≡ ∆ANP ≡ ∆BMP ≡ ∆CMN.
BD AB
As três medianas e as três bases médias de um triângulo dividem Pelo teorema das bissetrizes, temos: = . Logo,
SABD BD AB c CD AC
1 = = = .
o triângulo em 12 triângulos, 6 deles equivalentes a da área do SACD CD AC b
1 8
triângulo e 6 deles equivalentes a da área do triângulo. Se dois triângulos possuem um ângulo comum, então a razão
24
entre suas áreas é igual à razão entre os produtos dos lados adjacentes
Sejam M, N e P os pontos médios dos lados do ∆ABC, então a esse ângulo.

SADE AD ⋅ AE
=
SABC AB ⋅ AC

Demonstração:
SABC AD ⋅ AE
SAPA= SANA= SBMB= SBPB= SCMC= SCNC= =
3s sen α
SADE 2 AD ⋅ AE
1 1 1 1 1 1
8 = =
SABC AB ⋅ AC AB ⋅ AC
SABC sen α
SA1GP= SA1GN= SB1GM= SB1GP= SC1GM= SC1GN= s= 2
24

Demonstração: QUADRILÁTEROS
Os pontos A1, B1 e C1 são pontos médios de PN, MN e MP,
respectivamente. QUADRADO
AM
2 1  2 1 AM AA1 A área do quadrado
SA1AN éAigual
A ao seu lado elevado ao quadrado.
AG =AM ∧ AA1 =AM ⇒ A1G = AG − AA1 = −  AM = ⇒ =2 =⇒ 3 =1 = 3
3 2  3 2 6 A1G AM SA1GN A1G
AM 6
 2 1 AM AA1 2 SA1AN A1A
=
AG − AA1 =
 −  AM = ⇒ = =⇒ 3 = = 3
 3 2 6 A1G AM SA1GN A1G
6
Analogamente, temos
SA1AN SA1AP SB1BM SB1BP SC1CM SC1CN
= = = = = = 3
SA1GN SA1GP SB1GM SB1GP SC1GM SC1GN
SABC SABC
SAPN =
SAPA1 + SANA1 =
2 ⋅ SANA1 = ⇒ SANA1 =
4 8
SA1GN 1 1 S S
=⇒ SA1GN =⋅ ABC =ABC Seja  o lado do quadrado ABCD, então sua área é SABCD = 2.
SANA1 3 3 8 24

Analogamente, prova-se para os outros triângulos. Demonstração:


Basta utilizar a expressão da área do retângulo onde a base e a
altura são ambas iguais a . Assim, SABCD =  ⋅  = 2.

363

PM_BOOK16 - MAT.indb 363 25/11/2022 19:17:35


GEOMETRIA PLANA: ÁREAS

LOSANGO ÁREA DO QUADRILÁTERO CONVEXO


A área do losango é igual à metade do produto de suas diagonais. A área do quadrilátero convexo é igual à metade do produto das
diagonais vezes o seno do ângulo entre as diagonais.

Seja um quadrilátero convexo cujo ângulo entre as diagonais


pq
AC = p e BD = q é igual a θ, então a área do quadrilátero é S = ⋅ sen θ.
2

Seja o losango ABCD de diagonais AC = p e BD = q, então sua Demonstração:


p⋅q S = SSAEB + SBEC++S+S SCED+ S+CEDS + = S=DEA =
S
área é ABCD = . S ABCDS=
ABCD ABCD AEB= +SS
AEB
BEC CED +
BEC DEADEA
2 AE⋅⋅BE BE ⋅ CE
BE⋅ CE CE ⋅ DE DE ⋅ AE
⋅ ⋅ (sen  ) )CE ⋅ DE CE ⋅ DEDE ⋅ AE DE( ⋅ AE(
== = 2 sen
AE AE BE
senθθsen BE
++ θ + sen
BE CE
sen(180 180 −(180
θ− +θ −+ θ ) + sen sen
θ + θsen+ θ +sen 180 =
sen sen
= 180 =− θ) − θ )
− θ )(180

Demonstração: 2 2 22 2 2 2 2 2 2 2
AE⋅⋅BE
AE BE
AEsen ⋅ CE
⋅ BEθθ++BEBE⋅ CE ⋅ CE
BE sen CECE⋅ DE⋅ DE ⋅ DE DE ⋅DEAE⋅ AE ⋅ AEθ
Inicialmente, cabe observar que o losango possui diagonais
== = 22 sen sen θ2+2 sen + θ + CE
θ +θsen θ +θ + =
sensen =DEθsen
sen
perpendiculares. Assim, 2 2 2 2 2 sen2θ +2 2= sen θ
AE ⋅⋅((BE ++(DE )) CE ⋅ (BE
⋅ ( + DE
+ ) ) (BE(+BE
DE+ (
) DEAE) (
+ CE ) CE )
=
AC ⋅ BM AC ⋅ MD AC ⋅ (BM + MD)=
AE BE DE
AE ⋅ BE +sen DEθ)θ+ + = CE BE DE
CE ⋅ (BEsen θ )θ
+ sen
DE (=
BE += DE ) (+AE
AE CE )θ
sen+θsen
SABCD = SABC + SACD = + =
AC ⋅ BD p ⋅ q
= = = 22
sen sen θ += 2 2= sen θ 2 2 = sen θ
2 2 2 2 2 AC ⋅ BD 2 2 2
p ⋅ q
MD AC ⋅ (BM + MD) AC ⋅ BD p ⋅ q = AC ⋅ AC
=
BD
= ⋅ BD
sen θθ p ⋅ q senp ⋅θq
= = = = 22 = sen = sen2θ2 sen θsen θ
2 2 2 2 2

TRAPÉZIO QUADRILÁTERO CIRCUNSCRITÍVEL


A área do trapézio é igual ao produto da semissoma de suas bases A área do quadrilátero circunscritível é igual ao produto do seu
pela sua altura. semiperímetro pelo raio do círculo inscrito.

Seja o trapézio ABCD de bases AB = B e CD = b, e altura h, então


B+b Seja o quadrilátero circunscritível ABCD, cujo semiperímetro é
sua área=éS ⋅ h.
2 a+b+c +d
p= , então sua área é SABCD= p ⋅ r.
2
Demonstração:
Demonstração:

AB ⋅ h CD ⋅ h ( AB + CD) (B + b )
SABCD= SABC + SACD= + = ⋅ h= ⋅h
2 2 2 2

364

PM_BOOK16 - MAT.indb 364 25/11/2022 19:17:36


GEOMETRIA PLANA: ÁREAS

AB ⋅ OH BC ⋅ OE CD ⋅ OF DA ⋅ OG 2
SABCD = SAOB + SBOC + SCOD + SDOA =
+ + + = ⇒ sen
= Aˆ (p − a)(p − b )(p − c )(p − d)
2 2 2 2 ad + bc
a⋅r b⋅r c ⋅r d⋅r a+b + c + d ad + bc
= + + + =
r⋅ =p ⋅r ad bc
2 2 2 2  2  SABCD = SABD + SBCD = senAˆ + senCˆ = senAˆ =
2 2 2
Note que essa propriedade S é válida ad
para qualquer polígono ad +ˆ bc
ˆ + bc senCˆ = ad + bc=senA 2
ABCD = SABD + SBCD = senA = ⋅ (p − a)(p − b )(p − c )(p − d) = (p − a)(p − b )(p − c )(p − d)
circunscritível. 2 2 2 2 ad + bc
ad + bc 2
= ⋅ (p − a)(p − b )(p − c )(p − d) = (p − a)(p − b )(p − c )(p − d)
QUADRILÁTERO INSCRITÍVEL 2 ad + bc

QUADRILÁTERO INSCRITÍVEL E
CIRCUNSCRITÍVEL
A área do quadrilátero inscritível e circunscritível é igual à raiz
quadrada do produto dos seus lados.

Seja o quadrilátero inscritível ABCD de lados AB = a, BC = b, Seja o quadrilátero inscritível e circunscritível ABCD de lados
a+b+c +d
CD = c e DA = d, e semiperímetro p = , então sua área é AB = a, BC = b, CD = c e DA = d, então sua área é S = abcd .
2
SABCD = (p − a)(p − b )(p − c )(p − d) . Demonstração:
Como o #ABCD é circunscritível, então, pelo teorema de Pitot,
Demonstração: temos: a + c = b + d = p. Assim, a = p - c, c = p – a, b = p – d e
d = p – b.
Aplicando a lei dos cossenos no ∆ABD, temos:
Utilizando a fórmula para o cálculo da área do quadrilátero
BD2 = a2 + d2 − 2ad ⋅ cos Aˆ (*) inscritível, temos:
Aplicando a lei dos cossenos no ∆BCD, temos:
SABCD = (p − a)(p − b )(p − c )(p − d) = abcd .
BD2 = b2 + c2 − 2bc ⋅ cosCˆ (**)
Como o #ABCD é inscritível, então
Aˆ + Cˆ =180 ⇒ cosCˆ = − cos Aˆ ∧ senCˆ =senAˆ .
POLÍGONO REGULAR
A área de um polígono regular é igual ao produto do semiperímetro
Igualando (*) e (**), temos:
pelo apótema.
a2 + d2 − 2ad ⋅ cos Aˆ = BD2 = b2 + c2 − 2bc ⋅ ( − cos Aˆ )
2 ( ad + bc ) cos Aˆ = a2 − b2 − c2 + d2
2
 a2 − b2 − c2 + d2 
sen2 Aˆ =
1 − cos2 Aˆ =
1−   =
 2 ( ad + bc ) 

4 ( ad + bc ) − ( a2 − b2 − c2 + d2 )
2 2

=
4 ( ad + bc )
2

( 2ad + 2bc + a2 − b2 − c2 + d2 )( 2ad + 2bc − a2 + b2 + c2 − d2 )


=
4 ( ad + bc )
2

( a + d)2 − (b − c )2  (b + c )2 − ( a − d)2 


=
4 ( ad + bc )
2

( a + d + b − c )( a + d − b + c )(b + c + a − d)(b + c − a + d)
=
4 ( ad + bc )
2
Seja um polígono regular de semiperímetro p e apótema a, então
sua área é S= p ⋅ a .
( 2p − 2c )( 2p − 2b )( 2p − 2d)( 2p − 2a)
=
4 ( ad + bc )
2
Demonstração:
4 (p − a)(p − b )(p − c )(p − d) Seja um polígono regular de gênero n, lado x e apótema a.
=
( ad + bc )2 Ele pode ser dividido em n triângulos isósceles de vértice no centro
do círculo circunscrito ao polígono, cuja base é o lado e a altura é

365

PM_BOOK16 - MAT.indb 365 25/11/2022 19:17:37


GEOMETRIA PLANA: ÁREAS

o apótema do polígono. A área de cada um desses triângulos Demonstração:


x ⋅a A área do setor circular é proporcional ao ângulo central. Assim,
isósceles é S∆ = . Portanto, a área do polígono é dada por
2 α
x ⋅ a nx n⋅x
um setor circular de α radianos representa da área total do círculo,
S = n ⋅ S∆ = n ⋅ = ⋅ a = p ⋅ a, onde utilizamos p = . 2π
2 2 2 α α ⋅R 2
ou seja, ⋅ πR =
2
.
2π 2

REGIÕES CIRCULARES SEGMENTO CIRCULAR


Um segmento circular é uma região da circunferência delimitada
CÍRCULO por uma corda e um arco e também é caracterizado pelo ângulo
A área do círculo é o produto do quadrado do seu raio pelo central associado à corda.
número irracional π.

Seja um círculo de raio R, então sua área é S = π ⋅ R2 . Seja um segmento circular de α em radianos e de raio R, então
sua área é
Demonstração:
R2 (
Observe que a área do círculo pode ser calculada considerando-o Ssegmento =
α Ssetor α − Striângulo= ⋅ α − sen α )
2
um polígono regular cujo número de lado tende ao infinito. Assim, sua
2πR
área é p produto do seu semiperímetro = πR pelo seu apótema R,
2 Exercício Resolvido
ou seja, S = πR ⋅ R = π ⋅ R2.
01. Aumentando os lados de um quadrado de 10% a sua área
SETOR CIRCULAR aumenta de
Um setor circular é a região da circunferência delimitada por a) 10%
dois raios e um arco e é caracterizado pelo ângulo central por ele b) 20%
determinado. c) 21%
A área do setor circular é igual à metade do produto do quadrado d) 22%
do raio pelo ângulo central em radianos.
e) 25%

Resolução: C

A1 = x 2
A 2 = (1,1x)2
A 2 = 1,21x 2

Seja um setor circular de ângulo central α em radianos e de raio R, A 2 1,21x 2


= = 1,21 ⇒ acréscimo de 21%
α ⋅ R2 A1 1x 2
então sua área é S = .
2
Observe que se o ângulo central estiver expresso em graus, a
πR2 ⋅ α
expressão resultante é S = .
360

366

PM_BOOK16 - MAT.indb 366 25/11/2022 19:17:38


GEOMETRIA PLANA: ÁREAS

Exercício Resolvido Exercício Resolvido

02. Catarina desenha um quadrado com lado de 10 cm. Ela liga 04. Na figura a seguir, há 4 circunferências concêntricas cujos raios
os pontos médios dos lados do quadrado para obter um quadrado medem 1,0 cm; 0,9 cm; 0,8 cm; 0,7 cm.
menor. Qual é a área do quadrado menor?

a) 10 cm²
b) 20 cm²
c) 25 cm²
d) 40 cm²
e) 50 cm²

Resolução: E
A área da região sombreada, em cm², é
Ligando os pontos médios de dois lados adjacentes do quadrado
(use 3 como aproximação para π)
de lado 10 cm, obtemos um triângulo retângulo cujos catetos
medem ambos 5 cm e cuja hipotenusa é o lado do quadrado
menor, de medida x. Então, pelo Teorema de Pitágoras, a) 1,02. c) 1,92.
x² = 5² + 5² ⇔ x² = 50. Mas a área desse quadrado é x², logo vale b) 1,59. d) 2,25.
50 cm². Podemos resolver o problema quadriculando o quadrado
maior e somando as áreas que compõem o quadrado interno.
Resolução: A
Calculando:
Shachurada1 = π ⋅ (1,0 ) − π ⋅ ( 0,9 ) = 0,19π
2 2

⇒ 0,19π + 0,15
=π 0,34 π ≈ 1,02 cm2
Shachurada1 = π ⋅ (1,0 ) − π S⋅ ( 0,9 ) = =0,19
2 2
π ⋅ (π0,8 )⇒−0,19
π ⋅ (π )0,15
2 2
hachurada2 0,7
+ ==0,15
π π π =≈ 1,02 cm2
0,34
Shachurada2 = π ⋅ ( 0,8 ) − π ⋅ ( 0,7) = 0,15π
2 2

Exercício Resolvido

05. Considere o alvo mostrado na figura a seguir, construído com


Exercício Resolvido
= AC
três circunferências tangentes duas a duas, com DA = 10 e os
03. Pedro quer encadernar seus cadernos da escola, ele sabe que a pontos D, A e C colineares.
capa de cada caderno tem a forma de um retângulo de dimensões
30 cm × 20 cm e são 7 cadernos, qual é a área mínima de cartolina
que Pedro precisará para encadernar os cadernos?
a) 4.200 cm².
b) 5.000 cm².
c) 3.500 cm².
d) 7.000 cm².

Resolução: A
Área de cada capa de caderno = 30 ⋅ 20 = 600 cm²
Área de 7 cadernos = 7 ⋅ 600 = 4200 cm²
Serão necessários 4200 cm² de cartolina para encadernar os
cadernos.

Um dardo é lançado e atinge o alvo. A probabilidade de o dardo


atingir a região sombreada é de

1 1
a) . d) .
5 2
1 2
b) . e) .
4 3
1
c) .
3

367

PM_BOOK16 - MAT.indb 367 25/11/2022 19:17:38


GEOMETRIA PLANA: ÁREAS

04. (CFN) Uma sala retangular, medindo 3,52 m de largura e


Resolução: D 4,16 m de comprimento, terá seu piso totalmente revestido com
Calculando: ladrilhos inteiros, quadrados e de mesma dimensão, sem que haja
ScentroA = π ⋅ 202 = 400π espaço entre os ladrilhos vizinhos. Os ladrilhos serão escolhidos de
modo que possuam o maior tamanho possível. Nessas condições, qual
ScentroD = ScentroC = π ⋅ 102 = 100π o tamanho máximo do lado do ladrilho?
= 400π − 100π − 100=
Ssombreada π 200π a) Maior de 10 cm e menor de 15 cm
200π 1 b) Maior de 15 cm e menor de 20 cm
=
P(X) =
400π 2
c) Maior de 20 cm e menor de 25 cm
d) Maior de 25 cm e menor de 30 cm
e) Maior de 30 cm e menor de 35 cm

EXERCÍCIOS DE 05. (CFN) Com base na figura abaixo, determine a área da figura

FIXAÇÃO hachurada.

28 mm
38 mm 38 mm
01. (CFN) Nas figuras abaixo, as medidas são dadas na mesma
unidade de medida.
58 mm

a) 1900 mm² c) 1320 mm² e) 1140 mm²


b) 1520 mm² d) 1240 mm²

06. O retângulo e o quadrado abaixo são equivalentes (têm a mesma


área). Observe atentamente as figuras e determine qual a medida do
lado do quadrado e o seu perímetro, respectivamente.

Pode-se afirmar que:


a) a área do quadrado é igual à área do triângulo.
a) 8 e 32 c) 8 e 16 e) 8 e 12,8
b) a área do quadrado é igual à área do retângulo.
b) 12,8 e 35,6 d) 12,8 e 20,8
c) a área do retângulo é metade da área do quadrado.
d) a área do quadrado é o triplo da área do retângulo. 07. (CFN) Uma região retangular teve suas dimensões descritas em
e) a área do triângulo é igual à área do retângulo. metros conforme a figura abaixo. O valor de X que faz com que a área
dessa figura seja igual a 30 m² é
02. (CFN) Para envelopar os dois lados de uma porta de vidro
retangular de 2,10 m de altura, foram usados 3,78 m² de um adesivo
próprio para envelopamento. Determine a largura dessa porta.
a) 70 cm d) 100 cm
b) 80 cm e) 110 cm
c) 90 cm a) 5 b) 4 c) 3 d) -8 e) 8

03. (CFN) Com base na figura abaixo, determine a área da figura 08. (CFN) Sendo E um ponto qualquer do lado CD do retângulo
hachurada. ABCD, a área do triângulo hachurado será

a) 6 cm² c) 12 cm² e) 16 cm²


b) 8 cm² d) 14 cm²
a) 1805 cm²
b) 1225 cm² 09. (CFN) Um círculo tem área igual a 16π cm². Se aumentarmos seu
raio em 50%, consequentemente, sua área irá aumentar em
c) 1075 cm²
a) 50% c) 100% e) 150%
d) 1205 cm²
b) 75% d) 125%
e) 1005 cm²

368

PM_BOOK16 - MAT.indb 368 25/11/2022 19:17:39


GEOMETRIA PLANA: ÁREAS

10. (CFN) Determine a área da região hachurada na figura abaixo, 05. (EEAR) Um trapézio isósceles tem bases medindo 12 cm e 20 cm.
onde AM = MB. Se a medida de um de seus lados oblíquos é 5 cm, então sua área,
em cm², é
a) 25. b) 39. c) 48. d) 54.

06. (EEAR) O piso de uma sala foi revestido completamente com


300 placas quadradas justapostas, de 20 cm de lado. Considerando
que todas as placas utilizadas não foram cortadas e que não há espaço
entre elas, a área da sala, em metros quadrados, é
a) 120 b) 80 c) 12 d) 8
a) 200,86 cm² c) 100,48 cm² e) 25,12 cm²
b) 198,00 cm² d) 50,24 cm² 07. (EEAR) O lado de um triângulo equilátero mede 12 cm. Se a área
desse triângulo é igual à área de um hexágono regular de lado x cm,
então o valor de x é
EXERCÍCIOS DE a) 2 b) 6 c) 2 6 d) 3 6

TREINAMENTO 08. (EEAR) As diagonais de um paralelogramo medem 10 m e 20 m


e formam entre si um ângulo de 60°. A área desse paralelogramo,
em m², é
01. (EAM) Deseja-se azulejar, até o teto, as 4 paredes de uma cozinha. a) 200. b) 100. c) 50 3. d) 25 3.
Sabe-se que a cozinha possui 2 portas medindo 210 cm de altura e
80 cm de largura cada uma, e uma janela com 150 cm de altura e 09. (EEAR) S6 e S3 são, respectivamente, as áreas do hexágono regular
110 cm de comprimento. O comprimento, a largura e a altura da e do triângulo equilátero, ambos inscritos na mesma circunferência.
cozinha são iguais a 5,0 m, 4,0 m e 3,0 m, respectivamente. Determine Nessas condições, a relação verdadeira é
o número mínimo de metros quadrados inteiros de azulejos que
a) S6 = S3. c) S6 = 2S3.
devem ser comprados e assinale a opção correta.
a) 42 b) 43 c) 49 d) 55 e) 58 b) S6 = 3S3. d) S3 = 2S6 .

02. (EAM) Analise a figura a seguir. 10. (EEAR) Os lados de um triângulo medem 7 cm, 8 cm e 9 cm. A
área desse triângulo, em cm², é
a) 12 3. b) 12 5. c) 8 2. d) 8 3.

11. (EEAR) A área de um hexágono regular inscrito em um círculo de


6 cm de raio é _____ 3 cm².
a) 6 b) 9 c) 12 d) 15

12. (EEAR) A figura representa a parte móvel de um catavento (4


hélices triangulares planas). Se o material utilizado para a confecção
dessas hélices custa R$ 300,00 o m² e considerando 2 = 1,4 , o custo
dessas peças, em R$, foi de

a) 22 3 b) 22 c) 13 + 4 3 d) 11 e) 11 3

03. (EAM) Analise a figura a seguir.

a) 280 b) 340 c) 420 d) 560

13. (EEAR) A figura mostra um paralelogramo sombreado formado


Sabendo que EP é o raio da semicircunferência de centro em E, como
pela superposição de dois retângulos, e apresenta uma dimensão de
mostra a figura acima, determine o valor da área mais escura e assinale
cada retângulo. Se um dos lados do paralelogramo mede 3,5 cm,
a opção correta,
então a sua área é _____ cm².
Dado: número π = 3
a) 10 cm² b) 12 cm² c) 18 cm² d) 22 cm² e) 24 cm²

04. (EEAR) Um quadrado e um losango têm o mesmo perímetro. Se


as diagonais do losango estão entre si como 3 para 5, então a razão
entre a área do quadrado e a do losango é
17 13 17 11
a) b) c) d)
15 15 13 13 a) 12 b) 18 c) 21 d) 23

369

PM_BOOK16 - MAT.indb 369 25/11/2022 19:17:40


GEOMETRIA PLANA: ÁREAS

14. (EEAR) A malha da figura abaixo é formada por losangos cujas 20. (AFA) Considere, no triângulo ABC abaixo, os pontos P ∈ AB,
diagonais medem 0,50 cm e 2,00 cm. A área hachurada é de _____cm². Q ∈ BC, R ∈ AC e os segmentos PQ e QR paralelos, respectivamente,
a AC e AB.

a) 20 b) 22 c) 23 d) 25

15. (EEAR) Dois círculos concêntricos têm 4 m e 6 m de raio. A área da


coroa circular por eles determinada, em m², é
a) 2π. b) 10π. c) 20π. d) 52π.
Sabendo que BQ = 3 cm, QC = 1cm e que a área do triângulo ABC é
16. (EEAR) Da figura, sabe-se que OB = r é raio do semicírculo de 8 cm², então a área do paralelogramo hachurado, em cm², é igual a
centro O e de diâmetro AC. Se AB = BC, a área hachurada da figura, a) 2 c) 4
em unidades quadradas, é
b) 3 d) 5

21. (CN) O triângulo de lados 0,333... cm, 0,5 cm e 0,666... cm


é equivalente ao triângulo isósceles de base 0,333... cm e lados
congruentes medindo x centímetros cada um. Com base nos dados
apresentados, é correto afirmar que x é igual a
3 257
r2 ⋅ π π  1 a) d)
a) −1 b) r 2  −1 c) r 2 ( π −2) d) r 2 ⋅ π − 2 48
2 2  2
151 15 + 4 6
b)
24 e)
17. (EEAR) A figura mostra um quadro que possui quatro círculos de 36
1
raio R e um de raio r, ambos medidos em cm. Considerando que os c)
R 3
círculos não são secantes entre si, que r = e 4R + 2r = 30 cm, a área
2
que os círculos ocupam é _____ π cm². 22. (CN) Considere o triangulo escaleno ABC e os pontos P e Q
pertencentes ao plano de ABC e exteriores a esse triângulo.
Se: as medidas dos ângulos PAC e QBC são iguais; as medidas dos
ângulos PCA e QCB são iguais; M é o ponto médio de AC; N é o ponto
médio de BC; S1 é a área do triangulo PAM; S2 é a área do triangulo
QBN; S2 é a área do triangulo PMC; e S4 é a área do triangulo QNC,
analise as afirmativas:
I. S1 está para S4, assim como S3 está para S2.
II. S1 está para S2, assim como (PM)² está para (QN)².
a) 120 b) 138 c) 150 d) 153 III. S1 está para S3, assim como S2 está para S4.

18. (EEAR) Na figura, os arcos que limitam a região sombreada são Logo pode-se concluir, corretamente, que:
arcos de circunferências de raio R e centrados nos vértices do quadrado a) apenas a afirmativa I é verdadeira.
ABCD. Se o lado do quadrado mede 2R e considerando π = 3, então b) apenas as afirmativas I e II são verdadeiras.
a razão entre a área sombreada e a área branca é
c) apenas as afirmativas I e III são verdadeiras.
d) apenas as afirmativas II e III são verdadeiras.
e) as afirmativas I, II e III são verdadeiras.

23. (CN) Considere o quadrado ABCD e o triângulo equilátero ABP,


sendo P interior ao quadrado. Nestas condições o triângulo cobre
cerca de quantos porcento da área do quadrado?
a) 40 d) 50
b) 43 e) 53
a) 1 1
b) c) 2 d) 3 c) 45
2 3
24. (CN) Num triângulo ABC, retângulo em A, os lados AB e AC
19. A área da superfície hachurada na figura mede, em cm² valem, respectivamente c e b. Seja o ponto G o baricentro do triângulo
ABC. A área do triângulo AGC é:
bc bc
a) d)
2 6
bc bc
b) e)
3 9
bc
c)
4
a) 3 + 2π. b) 6 + 4π. c) 28 – 6π. d) 22 – 4π.

370

PM_BOOK16 - MAT.indb 370 25/11/2022 19:17:41


GEOMETRIA PLANA: ÁREAS

25. (CN) O triângulo ADE da figura é equivalente ao quadrilátero 29. (CN) Considere a figura abaixo.
2
BCDE. Se AE = AB, então AD é qual fração de AC?
3

2 4
a) d)
3 5
3 5
b) e) S (MPQ )
4 8 A razão , entre as áreas dos triângulos MPQ e ABC, é
S ( ABC)
1 7 8
c) a) d)
2 12 15
5 7
26. (CN) Num triângulo retângulo ABC de catetos AB = 8 e AC = 6, a b) e)
12 8
mediana AM intercepta a bissetriz BD no ponto E. A área do triângulo
BME é expressa pelo número real x, tal que: 7
c)
a) 3,5 ≤ x ≤ 4,0 d) 5,0 < x ≤ 5,5 15

b) 4,0 < x ≤ 4,5 e) 5,5 < x ≤ 6,5


30. (EPCAR) Na figura, O é o centro do círculo de raio r, AT é tangente
c) 4,5 < x ≤ 5,0 ao círculo e MT é perpendicular a AT. Então, a área hachurada é

27. (CMRJ) No retângulo ABCD, os pontos F e G pertencem ao lado


AB e são tais que AF = FG = GB. O ponto médio do lado CD é o ponto
E. A diagonal AC intercepta os segmentos EF e EG, respectivamente,
nos pontos H e J. A área do retângulo ABCD mede 70 cm². A área do
triângulo EHJ, então, é igual a

r2
a)
24
(
9 3 − 4π )
r2
b)
24
(
15 3 − 4 π )
5 7 r2
a)
2
cm2 d)
2
cm2 c)
24
(
6 3 − 4π )
35 35 r2
b)
12
cm2 e)
8
cm2 d)
24
(
4 3 − 4π )
2
c) 3 cm
31. Na figura abaixo, P é o ponto médio do lado DE do hexágono
regular ABCDEF.
AB
28. (CN) Num paralelogramo ABCD de altura CP = 3, a razão = 2.
BC
Seja ‘M’ o ponto médio de AB e ‘P’ o pé da altura de ABCD baixada
sobre o prolongamento de AB, a partir de C. Sabe-se que a razão
( )
entre as áreas dos triângulos MPC e ADM é S MPC = 2 + 3 . A área
do triângulo BPC é igual a S ( ADM ) 2

15 3 3 3
a) d)
2 2

9 3 3
b) e) A razão entre as áreas do quadrilátero BCDP e do hexágono é:
2 2
a) 3/7. c) 1/4. e) 1/2.
5 3 b) 2/5. d) 1/3.
c)
2

371

PM_BOOK16 - MAT.indb 371 25/11/2022 19:17:42


GEOMETRIA PLANA: ÁREAS

32. (CN) As diagonais AC, BD, CE, DF, EA e FB de um hexágono regular AD, AB e BC são arcos de circunferência com centro em O2, O e O1,
ABCDEF interceptam-se formando outro hexágono A’B’C’D’E’F’ respectivamente, cujos raios medem 2r.
conforme a figura abaixo. Qual a razão entre as áreas do maior e a do Das figuras abaixo, a única em que a área sombreada NÃO é igual a S, é
menor hexágono?
a)

Circunferência de diâmetro
AB e semicircunferências de
diâmetros OA e OB .

3 b)
a) 2 b) 3 c) d) 2 e) 3
2

33. (CN) Uma massa fermentada, ao ser colocada para descansar,


ocupou uma área circular S de raio r. Após um certo tempo t, ela
passou a ocupar uma área 21% maior que S. Qual o maior valor Circunferência de centro O.
possível de r, em centímetros, para que a massa não transborde,
quando colocada para descansar durante o tempo t, em um tabuleiro
circular de raio 22 centímetros?
a) 17,38 2 c) 20 d) 20,38 e) 21
b) 18
11 c)
 
34. (CN) Na figura abaixo, tem-se: QB e QA são tangentes ao círculo
de raio 2; a medida do segmento PA é 2 3 e a potência do ponto P
em relação ao círculo é igual a 24. A área sombreada da figura é igual a
Circunferência de centro O.

d)

Circunferência de centro O inscrita


num quadrado.
Dois setores circulares de raio r.

a)
4
3
(2 3−π ) c)
4
3
( 3−π ) e)
4
3
(
6 3−π )
b)
4
3
(3 3−π ) d)
4
3
(4 3−π )
36. A borda da sombra na Lua é sempre um arco de círculo. Em
um certo dia, a Lua é vista com a sombra passando por pontos
35. (EPCAR) Considere a área S da parte sombreada no triângulo
diametralmente opostos. Se o centro do arco de círculo que forma a
retângulo isósceles OO1O2.
sombra está sobre a circunferência da Lua, determine a proporção da
Lua que não está na sombra.

1 1 1 1 π −1
a) b) c) d) e)
4 3 π π +1 π

372

PM_BOOK16 - MAT.indb 372 25/11/2022 19:17:43


GEOMETRIA PLANA: ÁREAS

37. (AFA) Considere num mesmo plano os pontos da figura abaixo, 42. (CMRJ) Um aluno do CMRJ traçou uma circunferência de raio R
de tal forma que: cm e dividiu o círculo correspondente em duas regiões, usando uma
I. AW ≡ CW ≡ EW ≡ GW ≡ IW ≡ LW ≡ NW ≡ PW corda AB de comprimento R 3 cm, conforme mostra a figura abaixo.
Sabendo que a área da região sombreada é ( 4 π − 3 3 ) cm2, então, a
II. BW ≡ DW ≡ FW ≡ HW ≡ JW ≡ MW ≡ OW ≡ QW medida de R é:
 ≡ BWC
III. AWB  ≡ CWD
 ≡  ≡ PWQ
 ≡ QWA

IV. PC ≡ AE ≡ CG ≡ EI ≡ GL ≡ IN ≡ NA ≡ LP ≡ a
A área da região sombreada da figura, em função de a, é:
a) 12a2 − 8a2 2 d) 6a2 − 4a2 2
b) 6a2 + 4a2 2 e) 6a2
c) 12a + 8a
2 2
2 a) 2 3 c) 2π 3 e) (5π − 3)
3 d) 3π
b)
38. (EPCAR) Num semicírculo está inscrito um trapézio isósceles. A 2
base maior é o diâmetro, e a menor é o lado do triângulo equilátero,
inscrito no círculo de mesmo raio, cujo apótema vale 6. A área do 43. (CN 2013) Observe a figura a seguir.
trapézio é igual a:
a) 12(1 + 3 ). c) 36(2 + 3 ).
b) 12. d) 36.

39. (EPCAR) Em um círculo cujo comprimento da circunferência é


igual a 6π são traçadas duas cordas AB e CD que medem 2 3 e 4 2 ,
respectivamente, e cujas retas suporte não se intersectam. Calculando
a área do quadrilátero ABCD inscrito no círculo, tem-se o número
a) 5( 2 + 3) c) 5 2 + 3
b) 5 3 + 2 d) 5 3 − 2

40. (EN) Considere o trapézio MNPQ de bases MN = m e PQ = 4, com


m > 4 e altura igual a 6, conforme figura abaixo. Sendo A e B os
pontos médios dos lados MP e NQ , respectivamente, e sabendo que A figura acima apresenta um quadrado ABCD de lado 2. Sabe-se que
AB = 10, então a área do trapézio MCDN vale: E e F são os pontos médios dos lados DC e CB, respectivamente. Além
P Q
disso, EFGH também forma um quadrado e I está sobre o lado GH, de
GH
modo que GI = . Qual é a área do triângulo BCI?
4
7 4
A B a) d)
8 5
C D
6 3
b) e)
7 4
M N 5
c)
6
a) 28 b) 33 c) 37 d) 42 e) 45
44. (CN) O triângulo ABC da figura abaixo tem área S. Sabendo que
41. (CMRJ) Na figura abaixo, temos o semicírculo de diâmetro = BC
AB = 2AC, BH é altura e AD é bissetriz do ângulo  , a área da
AB = 4 cm e centro O. Sejam M o ponto médio de AO e N o ponto região hachurada, em função de S é igual a:
médio de OB. Com centros em M, O e N, traçam-se 3 semicírculos de
raios iguais a 1 cm e contidos no interior do semicírculo de diâmetro
4 cm e centro O. A área da região sombreada, em cm², a qual está
situada no interior do semicírculo maior e exterior aos três semicírculos
menores, vale

2S S S
a) c) e)
π+ 2 7π 3 15 18 21
a) π − 3 c) e) −
2 6 2 S 7S
b) d)
π+ 3 10 30
b) π − 2 d)
2

373

PM_BOOK16 - MAT.indb 373 25/11/2022 19:17:45


GEOMETRIA PLANA: ÁREAS

45. (EFOMM) João construiu um círculo de papel com centro O e raio 03. (CN) No triângulo ABC, retângulo em A, da figura, AB = c,
4cm (Figura 1). Traçou dois diâmetros AC e BD perpendiculares e, em AC = b, AM = 2 e AH é a altura relativa ao lado BC. Qual é a área do
seguida, dobrou o papel fazendo coincidir A, O e C, conforme sugere triângulo AHM?
a Figura 2.

bc bc2 bc
a) c) e)
b2 + c2 b + c2
2
b + c2
2

b2c2 b2c2
b) d)
b2 + c2 b2 + c2
A área da parte do círculo não encoberta pelas dobras, sombreada na
figura 2, é igual a
04. (CN) Em um triângulo de vértices A, B e C, retângulo em A, os
1
a) ( 96 − 16π ) cm2 catetos AB e AC medem respectivamente 6 3 cm e 6 cm. Traça-
3
se o segmento AM , com M pertencente e interno ao segmento BC
1
b) (16π − 48 ) cm2 ˆ mede 15°, a razão entre as áreas dos
. Sabendo-se que ângulo MAC
3
triângulos AMC e ABC é:
c)
1
3
(
16π − 12 3 cm2 ) a)
3 +1
d)
2− 3
2 2
d)
1
3
(
16π + 12 3 cm2 ) 3 −1 e) impossível de se determinar
b) com apenas esses dados.
2
e)
1
3
(
48 3 − 16π cm2 )
2+ 3
c)
2

EXERCÍCIOS DE 05. (CN) Os lados de um triângulo medem AB = 40, AC = 50 e BC = 60 .

COMBATE Sendo D a interseção da bissetriz interna do ângulo B com o lado AC,


a área do triângulo ABD é:

a) 225 7 c) 150 7 e) 75 7
01. (CN) Dado um trapézio qualquer, de bases 6 e 8, traça-se 375 d) 125 7
b) 7
paralelamente às bases um segmento de medida x que o divide em 2
outros dois trapézios equivalentes. Podemos afirmar que:
06. (EN) As circunferências da figura abaixo possuem centro nos pontos
a) x = 6,5 d) x = 5 2 T e Q, têm raios 3cm e 2cm, respectivamente, são tangentes entre si e
b) x = 4 3 tangenciam os lados do quadrado ABCD nos pontos P, R, S e U.
e) x = 7,5
c) x =7

02. (CN) Dadas as afirmativas abaixo, coloque (V) verdadeiro ou (F)


falso:
( ) Se a altura AH de um triângulo ABC o divide em dois triângulos
ABH e ACH semelhantes e não congruentes, então o triângulo
ABC é retângulo.
( ) A mediana AM de um triângulo ABC o divide em dois triângulos
AMB e AMC equivalentes.
( ) A bissetriz interna AD de um triângulo ABC o divide em dois
triângulos ABD e ACD cujas áreas são, respectivamente,
proporcionais aos lados AB e AC. Qual o valor da área da figura plana de vértices P, T, Q, R e D em cm²?
Assinale a alternativa correta.
7 2 + 18 30 2 + 25
a) (V), (V), (V) a) d)
2 2 4
b) (V), (V), (F)
50 2 + 23 50 2 + 49
c) (V), (F), (V) b) e)
8 4
d) (F), (V), (F)
e) (V), (F), (F) 15 2 + 2
c)
4

374

PM_BOOK16 - MAT.indb 374 25/11/2022 19:17:46


GEOMETRIA PLANA: ÁREAS

07. (IME) Dado um círculo de raio R e centro O, constroem-se três


círculos iguais de raios r, tangentes dois a dois, nos pontos E, F, G e
tangentes interiores ao círculo dado. Determine, em função de R, a
área da superfície EFG, compreendida entre os três círculos e limitada
pelos arcos EG, GF e FE.
DESAFIO PRO
(2 3 − π )( 2 3 − 3) 2
1
2
(CN) Considere a figura a seguir.
a) R
2

(2 3 − π )( 2 3 − 3) 2
2

b) R
4

(3 3 − π )( 3 3 − 3) 2
2

c) R
2

(3 3 − π )( 3 3 − 3) 2
2

d) R
4

(π − 3 )( 3 − 3 ) 2
2

e) R
2

08. (CN) Observe a figura abaixo, onde os seis lados do


hexágono regular ABCDEF foram prolongados de segmentos
= BB'
AA' = CC' = DD'
= EE'= FF' , de modo que a medida do S (MPQ )
segmento AA’ corresponde a P% da medida do lado AB, (P > 0). A razão , entre as áreas dos triângulos MPQ e ABC, é:
S ( ABC )
Se o percentual de aumento que a área do hexágono A’B’C’D’E’F’
7 5 7 8
apresenta em relação à área do hexágono original é 75%, então o a) b) c) d)
valor de P é: 12 12 15 15

D’ C’

D
2 (CN) Num paralelogramo ABCD de altura CP = 3, a razão
AB
= 2. Seja ‘M’ o ponto médio de AB e ‘P’ o pé da altura
BC
C de ABCD baixada sobre o prolongamento de AB, a partir de C.
E B’
E’ Sabe-se que a razão entre as áreas dos triângulos MPC e ADM é
B S (MPC ) 2 + 3
= . A área do triângulo BPC é igual a:
F S ( ADM) 2
A
15 3 5 3 3
a) c) e)
F’ A’ 2 2 2

a) 25 d) 50 9 3 3 3
b) d)
2 2
b) 30 e) 75
c) 45

09. (CN) Com a “ponta seca” de um compasso, colocada no centro


3 No retângulo ABCD, os pontos F e G pertencem ao lado
AB e são tais que AF = FG = GB. O ponto médio do lado CD
é o ponto E. A diagonal AC intercepta os segmentos EF e EG,
de um quadrado de lado 2, traça-se uma circunferência de raio r.
respectivamente, nos pontos H e J. A área do retângulo ABCD
Observa-se que cada arco da circunferência, externo ao quadrado,
mede 70 cm². A área do triângulo EHJ, então, é igual a
tem o dobro do comprimento de cada arco interno. Usando-se raiz
quadrada de 3 igual a 1,7 e π = 3, qual a área da região intersecção
do quadrado e do círculo, assim determinado?
a) 2,8 d) 3,4
b) 3,0 e) 3,6
c) 3,2

10. (CN) Um círculo α de centro num ponto A e raio 2 3 é tangente


interior, num ponto B, a um círculo β de centro num ponto O e raio
6 3 . Se o raio OC é tangente a α num ponto D, a medida da área
limitada pelo segmento DC e os menores arcos BC de β e BD de α é 5 7
a) cm2 d) cm2
igual a 2 2

a) 4π − 3 3 d) 5π − 6 3 b) 35 cm2 e)
35
cm2
12 8
b) 5π − 4 3 e) 5π − 5 3
c) 3 cm²
c) 4π − 6 3

375

PM_BOOK16 - MAT.indb 375 25/11/2022 19:17:47


GEOMETRIA PLANA: ÁREAS

4 Considere a figura abaixo:


GABARITO
EXERCÍCIOS DE FIXAÇÃO
01. E 04. E 07. A 10. D
02. C 05. E 08. E
03. C 06. A 09. D
EXERCÍCIOS DE TREINAMENTO
01. C 13. C 25. B 37. D
02. B 14. C 26. C 38. C
03. B 15. C 27. C 39. A
04. A 16. B 28. B 40. B
05. C 17. D 29. C 41. E
06. C 18. D 30. A 42. A
Nela ABCD é um quadrado de lado a, onde AB = BC
= CD
= AD 07. C 19. D 31. D 43. E
são arcos de circunferência cujo raio mede a. A expressão
08. C 20. B 32. E 44. D
algébrica que permite calcular a área hachurada em função da
medida a do quadrado é: 09. C 21. B 33. C 45. E
10. B 22. E 34. B
 2π 
a) a²  − 3
 3  11. B 23. B 35. D
12. C 24. D 36. E
a²  2π 3
b)  −  EXERCÍCIOS DE COMBATE
2  3 2 
01. D 04. D 07. A 10. D
π  02. A 05. C 08. D
c) a²  − 1
2  03. C 06. E 09. E
d) a²(π-2) DESAFIO PRO
01. B 03. C 05. B

5 Na figura abaixo temos um heptágono regular de lado a e

e de raios a.
     eF
arcos de circunferência AB,BC,CD,DE,EF G congruentes
02. B 04. A

ANOTAÇÕES

O valor da área hachurada da figura, em função de a, é:


a2
a)
4
(
4π − 7 3 )
2

(
b) 7a 3π − 2 3
12
)
7a2
c)
6
(
3π − 2 3 )
a2
d)
12
(
3π − 2 3 )

376

PM_BOOK16 - MAT.indb 376 25/11/2022 19:17:48


CÍRCULO TRIGONOMÉTRICO

LINHAS TRIGONOMÉTRICAS O sistema circular ou radiométrico adota como unidade de


medida 1 radiano (1 rad).
Como o comprimento de uma circunferência de raio R é 2πR ,
UNIDADES DE MEDIDAS ANGULARES
então um ângulo de uma volta mede 2π rad .

SISTEMA SEXAGESIMAL – GRAU (°) 1


=
1rad ⋅ ( ângulo de uma volta)
Um sistema sexagesimal é um sistema de numeração de base 60, 2π
ou seja, cada submúltiplo é 60 vezes menor que o anterior. 
Para medidas angulares, é comum adotar um sistema sexagesimal 1ângulo de uma volta= 2π rad
1 1ângulo raso = π rad
com unidade de medida 1 grau (1º) que é de um ângulo reto.
90
π
1ângulo reto = rad
Um grau pode ser dividido em 60 minutos e cada minuto dividido 2
em 60 segundos.

1
1 = ⋅ ( ângulo reto ) ⇔ 1ângulo reto =
90 COMPRIMENTO DO ARCO DE CIRCUNFERÊNCIA
90
1ângulo raso = 180
1ângulo de uma volta = 360

Submúltiplos do grau:
1 
Minuto: 1' = ⋅ 1 ⇔ 1 = 60'
60
1
Segundo: 1'' = ⋅ 1' ⇔ 1' = 60'' ⇔ 1 = 3600''
60

Exemplo:
180°
Calcule .
48
Resolução: 2parco = α ⋅ R , onde α em radianos
180°
= 3,75° = 3° + 0,75 ⋅ 60' = 3°45'
48
θ πRθ
2parco = 2πR ⋅ = , onde θ em graus
SISTEMA CIRCULAR OU RADIOMÉTRICO − 360 180
RADIANOS (RAD)
O ângulo de 1 radiano (1 rad) é o ângulo central em uma
Exemplo:
circunferência de raio R que determina um arco de comprimento R
sobre essa circunferência.
π
Calcule o comprimento de um arco de circunferência de rad
em uma circunferência de raio 3 cm. 3
Resolução:
π
O comprimento do arco é  =α ⋅ R = ⋅ 3 =π cm .
3

RELAÇÕES ENTRE AS UNIDADES

180° = π rad

377

PM_BOOK16 - MAT.indb 377 25/11/2022 19:17:50


CÍRCULO TRIGONOMÉTRICO

Exemplo: A figura a seguir traz a indicação dos quadrantes no ciclo


π trigonométrico.
Converta o ângulo de 15° para radianos e o ângulo de rad
para graus. 8
Resolução:
π rad π
15°= 15° ⋅ = rad
180° 12
π π 180°
rad= rad ⋅ = 22,5°= 22°30'
8 8 π rad

O CICLO TRIGONOMÉTRICO
O ciclo trigonométrico é uma circunferência orientada de raio unitário
e centrada na origem. A origem do sistema de medidas é no ponto de
coordenadas A(1, 0) e o sentido positivo é o sentido anti-horário.

Exemplo:
Identifique a que quadrante pertence cada um dos arcos a seguir:
3π 9π 2π
a) c) e)
4 7 15
7π d) 265°
b)
4

Resolução:
3π π 3π
A medida dos ângulos é feita por meio de uma função dos a) ∈ QΙΙ , pois < <π
números reais sobre a circunferência, que associa a cada número real 4 2 4
θ um único ponto P sobre a circunferência. O ponto P associado ao 7π 3π 7π
número real θ é o ponto final de um percurso de comprimento θ sobre b) ∈ QΙV , pois < < 2π
4 2 4
a circunferência, a partir de A, no sentido anti-horário para θ > 0 ou
no sentido horário para θ < 0. O Ponto P associado ao número real θ 9π 5π 3π
c) ∈ QΙΙΙ , pois π < <
é chamado imagem de θ no ciclo trigonométrico. 7 4 2
Isso é como se você enrolasse a reta real na circunferência com
d) 265° ∈ QΙΙΙ , pois 180° < 265° < 270°
o zero sobre o ponto A e o sentido positivo no sentido anti-horário.
Observe que o ciclo trigonométrico tem comprimento 2π ⋅ 1 = 2π. 2π 2π π
e) ∈ QΙ , pois 0 < <
Assim, os arcos pertencentes ao intervalo [0,2π[ estão na primeira 15 15 2
volta; os arcos pertencentes ao intervalo [2π,4 π[ estão na segunda
volta; os arcos pertencentes ao intervalo [ −2π,0[ estão na volta –1; ARCOS CÔNGRUOS
e assim por diante. Como o ciclo trigonométrico tem comprimento 2π, a função que
π define o ponto P do ciclo trigonométrico associado a um número
O ciclo trigonométrico é divido em 4 quadrantes de radianos
2 real θ é periódica de período 2π, ou seja, números que diferem por
enumerados no sentido anti-horário. Assim, arcos cujas extremidades múltiplos de 2π possuem a mesma imagem no ciclo trigonométrico.
estão sobre o arco AB  estão no primeiro quadrante ( Q ) ; sobre
Ι
Dois arcos são ditos côngruos quando possuem a mesma

 , no segundo quadrante ( QΙΙ ) ; sobre o arco A'B'
o arco BA' , no extremidade no ciclo trigonométrico.
terceiro quadrante ( Q ΙΙΙ ) e sobre o arco 
B'A , no quarto quadrante
Assim, dois arcos α e β, expressos em radianos, são côngruos
( Q ΙV ) .
( α ≡ β ) se, e somente se, α − β = 2π ⋅ k para algum k ∈  .
Para arcos no intervalo [0,2π] (primeira volta), os arcos do Da mesma forma, dois arcos α e β, expressos em graus, são
 π côngruos ( α ≡ β ) se, e somente se, α −=
β 360° ⋅ k para algum
primeiro quadrante pertencem ao intervalo  0,  , os arcos do
 2 k∈ .
π 
segundo quadrante pertencem ao intervalo  , π  , os arcos do
2  α ≡ β ⇔ α − β= 2k ⋅ π, k ∈  (radianos )
 3π 
terceiro quadrante pertencem ao intervalo  π,  e os arcos do
 2
 3π  β 360° ⋅ π, k ∈  ( graus )
α ≡ β ⇔ α −=
quarto quadrante pertencem ao intervalo  ,2π  .
 2 

378

PM_BOOK16 - MAT.indb 378 25/11/2022 19:17:51


CÍRCULO TRIGONOMÉTRICO

Exemplo: Exemplo:
Marque no ciclo trigonométrico a imagem de cada um dos Encontre a primeira determinação positiva dos seguintes arcos.
números a seguir e identifique os arcos côngruos. 35π
a)
7π 4
a)
3
b) 25π
19π 6
b)
4 40π
c) −
5π 3
c) −
4 d) 2880°
11π e) –1200°
d) −
4
5π Resolução:
e) −
3 35π 3π
a) A primeira determinação positiva de é , pois
14 π 4 4
f) 35π 32π + 3π 3π
3 = = + 2π ⋅ 4 .
4 4 4
Resolução:
25π π
7π π 6π π b) A primeira determinação positiva de é , pois
a) = + = + 2π 25π π + 24 π π 6 6
3 3 3 3 = = + 2π ⋅ 2 .
6 6 6
19π 3π + 16π 3π
b) = = + 2π ⋅ 2 40π 2π
4 4 4 c) A primeira determinação positiva de − é , pois
40π −42π + 2π 2π 3 3
5π 3π − 8π 3π − = = + 2π ⋅ ( −7) .
c) − = = + 2π ⋅ ( −1) 3 3 3
4 4 4
11π 5π − 16π 5π d) A primeira determinação positiva de 2730° é 210°, pois
d) − = = + 2π ⋅ ( −2)
4 4 4 =
2730 ° 360° ⋅ 7 + 210° .
5π π − 6π π e) A primeira determinação positiva de –1200° é 240°,pois
e) − = = + 2π ⋅ ( −1)
3 3 3 −1200
= ° 360° ⋅ ( −4 ) + 240° .
14 π 2π + 12π 2π
f) = = + 2π ⋅ 2
3 3 3 LINHAS TRIGONOMÉTRICAS DE UM ÂNGULO
QUALQUER

SENO E COSSENO
Seja P a imagem de um ângulo θ no ciclo trigonométrico. Define-
se o seno do ângulo θ como a ordenada de P e o cosseno de θ como
a abscissa de P.
Assim, para obter o seno de θ, devemos projetar P sobre o eixo
vertical Oy, denominado eixo dos senos, e, para obter o cosseno de
θ, devemos projetar P sobre o eixo horizontal Ox, denominado eixo
dos cossenos.
Na figura a seguir, temos:

Os arcos das opções A e E são côngruos, assim como os das


opções B e C.

A primeira determinação positiva de um arco


β é o arco α ∈ [0,2π[ côngruo a β .

Para se identificar a imagem de um arco no ciclo trigonométrico


e os valores de suas linhas trigonométricas é sempre útil encontrar a
primeira determinação positiva.

sen θ =OPy cos θ =OPx

379

PM_BOOK16 - MAT.indb 379 25/11/2022 19:17:52


CÍRCULO TRIGONOMÉTRICO

Observe que, OPy e OPx são segmentos orientados. TANGENTE E COTANGENTE


Dessa forma, o seno de um ângulo assume valores positivos No ciclo trigonométrico, o eixo paralelo ao eixo Oy, com a mesma
quando sua imagem no ciclo trigonométrico possui ordenada positiva, orientação que este e passando pelo ponto A é denominado eixo das
ou seja, quando essa imagem está acima do eixo Ox, e valores tangentes, e o eixo paralelo ao eixo Ox, com a mesma orientação que
negativos, caso contrário. este e passando pelo ponto B é denominado eixo das cotangentes.
Do mesmo modo, o cosseno de um ângulo assume valores π
Seja θ um ângulo tal que θ ≠ + k ⋅ π , k ∈  , e cuja imagem
positivos quando sua imagem no ciclo trigonométrico possui abscissa 2
positiva, ou seja, quando essa imagem está à direita do eixo OY, e no ciclo trigonométrico é P. A tangente de θ é a medida algébrica do
valores negativos, caso contrário.
segmento AP1 , onde P1 é a interseção da reta OP com o eixo das
tangentes.
Observe que a definição acima é coerente com a que foi π
estabelecida nos triângulos retângulos para ângulos agudos. Basta Note que, se θ= + k ⋅ π , k ∈  , a reta OP não intersecta o eixo
2
observar que o triângulo retângulo OPxP possui hipotenusa 1 (raio das tangentes e, portanto, a tangente de θ não está definida.
do ciclo trigonométrico) e catetos OPx e OPy . Assim, temos:
PPx OPy OPx OPx Seja θ um ângulo tal que θ ≠ k ⋅ π , k ∈  , e cuja imagem no
sen=
θ = = OPy e cos=
θ = = OPx .
OP 1 OP 1 ciclo trigonométrico é P. A cotangente de θ é a medida algébrica do
segmento BP2 , onde P2 é a interseção da reta OP com o eixo das
Denomina-se função seno, a função de  em  definida por cotangentes.
f(x) = sen x.
Note que, se θ = k ⋅ π , k ∈  , a reta OP não intersecta o eixo das
O domínio da função seno é Dsen =  e a imagem Imsen = [ −1,1] .
cotangentes e, portanto, a cotangente de θ não está definida.

Denomina-se função cosseno, a função de  em  definida por


Na figura a seguir, temos:
f(x) = cos x.
O domínio da função cosseno é Dcos =  e a imagem
Imcos = [ −1,1] .

sen θ ∈ [ −1,1] , ∀θ ∈ 

cos θ ∈ [ −1,1] , ∀θ ∈ 

Seno e cosseno são funções periódicas de período 2π.

Os sinais das funções seno e cosseno em cada um dos quadrantes


estão representados nos diagramas a seguir:

tg θ =AP1

cotg θ =BP2

Observe que a definição acima é coerente com a que foi


estabelecida nos triângulos retângulos para ângulos agudos. Basta
notar que, no triângulo retângulo OAP1 , temos:
AP1 AP1
tg θ= = ⇔ AP1= tg θ
OA 1
BP2 BP2
cotg=
θ = ⇔ BP
=2 cotg θ
OB 1

Note ainda que, se Px e Py são as projeções de P sobre os eixos


dos cossenos e dos senos, respectivamente, então temos:
PPx AP1 sen θ tg θ sen θ
∆OPPx  ∆OP1A ⇒ = ⇔ = ⇔ tg=
θ
OPx OA cos θ 1 cos θ

PPy BP2 cos θ cotg θ cos θ


∆OPPy  ∆OP2B ⇒ = ⇔ = ⇔ cotg=
θ
OPy OB sen θ 1 sen θ

380

PM_BOOK16 - MAT.indb 380 25/11/2022 19:17:54


CÍRCULO TRIGONOMÉTRICO

Essa demonstração foi feita para um ângulo no primeiro Na figura a seguir, temos:
quadrante, para verificar que essa relação vale para qualquer ângulo,
basta verificar que os sinais em cada quadrante satisfazem essa
condição.
Portanto, para um ângulo θ qualquer, no qual as linhas
trigonométricas estejam definidas, valem as relações:
sen θ cos θ 1
=tg θ = cotg θ = cotg θ
cos θ sen θ tg θ

Denomina-se função tangente, a função de Dtg em  definida


por f(x) = tg x.

O domínio da função tangente é Dtg =


e a imagem Imtg =  .
{x∈ | x ≠
π
2
+ kπ, k ∈  }
Dcotg
Denomina-se função cotangente, a função de em 
definida por f(x) = cotg x.

O domínio da função cotangente é


Dcotg = {x ∈  | x ≠ kπ, k ∈ }
Imcotg = 
e a imagem .

Tangente e cotangente são funções periódicas de período π.

De acordo com a definição acima, os ângulos do 1º e 3º


quadrantes possuem tangente e cotangente positivas e os ângulos
do 2º e 4º quadrantes possuem tangente e cotangente negativas, sec θ =OP'
conforme representado no diagrama a seguir.
cossec θ =OP"

Observe que a definição acima é coerente com a que foi


estabelecida nos triângulos retângulos para ângulos agudos. Basta
notar que:
OP 1 1
∆OPP' : cos=
θ = ⇔ OP'
= ⇔ sec=
θ OP'
OP' OP' cos θ
OP 1 1
∆OPP" : sen
= θ = ⇔ OP''
= ⇔ cossec
= θ OP''
OP'' OP'' sen θ

Denomina-se função secante, a função de Dsec em  definida


por f(x) = sec x.

O domínio da função secante é Dsec = {x∈ | x ≠


π
2 }
+ kπ, k ∈  e
a imagem Imsec = ]−∞, −1] ∪ [1, +∞[ =  − ]−1,1[ .

Denomina-se função cossecante, a função de Dcossec em 


SECANTE E COSSECANTE definida por f(x) = cossec x.

π O domínio da função cossecante é Dcossec = {x ∈  | x ≠ kπ, k ∈ }


Seja θ um ângulo tal que θ ≠ + k ⋅ π , k ∈  , e cuja imagem e a imagem Imcossec = ]−∞, −1] ∪ [1, +∞[ =  − ]−1,1[ .
2
no ciclo trigonométrico é P. A secante de θ é a medida algébrica
do segmento OP' , onde P’ é a interseção da reta tangente ao ciclo
trigonométrico em P com o eixo dos cossenos. Secante e cossecante são funções periódicas de período 2π.
π
Note que, se θ= + k ⋅ π , k ∈  , a reta tangente não intersecta
Os sinais da secante e da cossecante acompanham os sinais do
o eixo cos cossenos e, 2portanto, a secante de θ não está definida.
cosseno e do seno, respectivamente.

Seja θ um ângulo tal que θ ≠ k ⋅ π , k ∈  , e cuja imagem no


ciclo trigonométrico é P. A cossecante de θ é a medida algébrica do
segmento OP" , onde P’’ é a interseção da reta tangente ao ciclo
trigonométrico em P com o eixo dos senos.
Note que, se θ = k ⋅ π , k ∈  , a reta tangente não intersecta o
eixo dos senos e, portanto, a cossecante de θ não está definida.

381

PM_BOOK16 - MAT.indb 381 25/11/2022 19:17:55


CÍRCULO TRIGONOMÉTRICO

RELAÇÕES FUNDAMENTAIS Exemplo:


 de medida x no círculo trigonométrico. 1
Imaginemos um arco O'P Sabendo que o seno de um ângulo agudo é igual a , calcule o
seu cosseno. 3

Resolução:
2
 1 1 8
sen2 θ + cos2 θ = 1 ⇒   + cos2 θ = 1 ⇔ cos2 θ = 1 − =
 3 9 9
Como θ é um ângulo agudo, então 0 < cos θ < 1 . Assim, temos:
2 2
cos θ = .
3

Exemplo:
Sabendo que a tangente de um ângulo agudo é igual a 2, calcule
o seu cosseno.

Resolução:
1 1
1 + tg=
2
θ sec2 θ ⇒ 1 + =
22 sec2 θ ⇔ sec=
2
θ = 5 ⇔ cos=
2
θ
cos2 θ 5
Como θ é um ângulo agudo, então 0 < cos θ < 1 . Assim, temos:
1 5
cos=
θ = .
5 5

Vemos a formação do triângulo retângulo de catetos sen x e Exemplo:


cos x e hipotenusa 1.
 3π  1
Sabendo que θ ∈  π,  e que sen θ = , calcule cos θ.
 2 3

Resolução:
2
 1 1 8
sen2 θ + cos2 θ = 1 ⇒   + cos2 θ = 1 ⇔ cos2 θ = 1 − =
 3 9 9

 3π  2 2
Como θ ∈  π,  , então cos θ < 0. Assim, temos: cos θ = − .
 2 3

REDUÇÕES AO 1º QUADRANTE
REDUÇÃO DO 2º AO PARA O 1º QUADRANTE
Pelo teorema de Pitágoras, sabemos que π
Dado o número α tal que < α < π , seja P´ o ponto de α no ciclo.
2
sen x + cos
2
=
2
x 1 ⇔ sen x + cos
2 2
= 2
x 1
Seja P o ponto do círculo, simétrico de P´ em relação ao eixo dos senos.
Neste quadrante temos os valores de seno e cosseno
Assim, para um ângulo agudo θ, podemos escrever:
respectivamente positivos e negativos.
Temos, então:
sen2 θ + cos2 θ =1
(relação fundamental da Trigonometria)

Dividindo a relação fundamental por cos2 θ , temos

sen2 θ 1
+
= 1 ⇔ 1 + tg=
2
θ sec2 θ
cos2 θ cos2 θ

Dividindo a relação fundamental por sen2 θ , temos

cos2 θ 1
1+ = ⇔ 1 + cotg
= 2
θ cossec2 θ
sen2 θ sen2 θ

1 + tg=
2
θ sec2 θ

1 + cotg
= 2
θ cossec2 θ  + AP'
=  + AP'
 = 180°
AP π rad ou AP

382

PM_BOOK16 - MAT.indb 382 25/11/2022 19:17:56


CÍRCULO TRIGONOMÉTRICO

 e AP'
Ou seja AP  são suplementares. Vemos que P’ e P são opostos pelo diâmetro então se voltarmos,
a partir de P’, π rad ou 180° chegaremos em P.
 = AP
P'C 

 = π − AP'
AP 

Sendo assim:
sen α = − sen( α − π)

Sendo assim cos α =− cos( α − π)


sen=
α sen ( π − α ) tg α= tg( α − π)
cos α =− cos( π − α )

tg α =− tg( π − α ) Outras relações importantes:


cot g ( α ) = cot g ( α − π)

Outras relações importantes: sec ( α ) =− sec ( α − π)


cot g( α ) = − cot g ( π − α ) cos sec ( α ) =− cos sec ( α − π)
sec( α ) = − sec( π − α )

=
cos sec( α ) cos sec( π − α ) REDUÇÃO DO 4º AO PARA O 1º QUADRANTE

Dado o número α tal que < α < 2π , seja P´ o ponto de α no
2
REDUÇÃO DO 3º AO PARA O 1º QUADRANTE ciclo. Seja P o ponto do círculo, simétrico de P´ em relação ao eixo dos
3π cossenos.
Dado o número α tal que π < α < , seja P´ o ponto de α no
2 Neste quadrante temos os valores de seno e cosseno
ciclo. Seja P o ponto do círculo, simétrico de P´ em relação ao centro. respectivamente negativos e positivos.
Neste quadrante temos os valores de seno e cosseno negativos. Temos, então:
Temos, então:

383

PM_BOOK16 - MAT.indb 383 25/11/2022 19:17:57


CÍRCULO TRIGONOMÉTRICO

 pelo sentido positivo e AP'  pelo sentido 2º QUADRANTE


Temos o arco AP
  
negativo e AP = −AP' então a relação existente ente os arcos AP 2π  2π  π 3
sen = sen  π − =  sen=
 
 é que AP + AP′ =  + AP
2π ou AP  
′ = 360° e AP′ = 360° − AP . 3  3  3 2
e AP'

2π  2π  π 1
cos = −cos  π −  = −cos = −
3  3  3 2

3π  3π  π 2
sen = sen  π − =  sen=
4  4  4 2

3π  3π  π 2
cos = −cos  π −  = −cos = −
4  4  4 2

5π  5π  π 1
sen = sen  π − =  sen=
6  6  6 2

5π  5π  π 3
cos = −cos  π −  = −cos = −
6  6  6 2

3º QUADRANTE

7π  7π  π 1
sen = −sen  − π  = −sen = −
6  6  6 2

Sendo assim: 7π  7π  π 3
cos = −cos  − π  = −cos = −
sen α =− sen (2π − α ) 6  6  6 2
=
cos α cos (2π − α )
5π  5π  π 2
sen = −sen  − π  − sen = −
tg α =− tg (2π − α ) 4  4  4 2

5π  5π  π 2
Outras relações importantes: cos = −cos  − π  = −cos = −
4  4  4 2
cot g ( α ) =− cot g (2π − α )

sec ( α ) =sec (2π − α ) 4π  4π  π 3


sen = −sen  − π  − sen = −
3  3  3 2
cos sec ( α ) =− cos sec (2π − α )

4π  4π  π 1
cos = −cos  − π  = −cos = −
Em resumo: 3  3  3 2
2º QUADRANTE

sen ( π − θ=
) sen θ sen (180 − θ=
) sen θ
4º QUADRANTE
cos ( π − θ ) = − cos θ cos (180 − θ ) = − cos θ
5π  5π  π 3
tg ( π − θ ) = − tg θ tg (180 − θ ) = − tg θ sen = −sen  2π −  = −sen = −
3  3  3 2
3º QUADRANTE

5π  5π  π 1
sen ( θ − π) = −sen θ sen ( θ − 180°) = −sen θ cos= cos  2π − = =
 cos
3  3  3 2
cos ( θ − π) = − cos θ cos ( θ − 180°) = − cos θ
7π  7π  π 2
tg ( θ − π)= tg θ tg ( θ − 180°)= tg θ sen = −sen  2π −  − sen = −
4  4  4 2

7π 7π  π
4º QUADRANTE

 2
sen ( 2π − θ ) = − sen θ sen ( 360 − θ ) = − sen θ cos= cos  2π − = =
 cos
4  4  4 2
cos ( 2π − θ ) = − cos θ cos ( 360 − θ ) = − cos θ
11π  11π  π 1
sen = −sen  2π −  − sen = −
tg ( 2π − θ ) = − tg θ tg ( 360 − θ ) = − tg θ 6  6  6 2

11π  11π  π 3
Vamos ver como exemplos todos as reduções nos 3 quadrantes cos= cos  2π − = =
 cos
6  6  6 2
que resultam em ângulos notáveis.

384

PM_BOOK16 - MAT.indb 384 25/11/2022 19:17:58


CÍRCULO TRIGONOMÉTRICO

2º QUADRANTE

) sen (180 − ( 90 − θ=


sen ( 90 + θ= ) ) sen ( 90 − θ=
) cos θ

cos ( 90 + θ ) = cos (180 − ( 90 − θ ) ) = − cos ( 90 − θ ) = − sen θ

tg ( 90 + θ ) = tg (180 − ( 90 − θ ) ) = − tg ( 90 − θ ) = − cotg θ

3º QUADRANTE

sen ( 270 − θ ) = sen (180 + ( 90 − θ ) ) = − sen ( 90 − θ ) = − cos θ

cos ( 270 − θ ) = cos (180 + ( 90 − θ ) ) = − cos ( 90 − θ ) = − sen θ

θ ) tg (180 + ( 90 − θ=
tg ( 270 −= ) ) tg ( 90 −=
θ ) cotg θ

4º QUADRANTE

sen ( 270 + θ ) = sen ( 360 − ( 90 − θ ) ) = − sen ( 90 − θ ) = − cos θ

) cos ( 360 − ( 90 − θ=


cos ( 270 + θ= ) ) cos ( 90 − θ=
) sen θ
ProBizu
Vamos primeiramente relembrar uma relação importante.
tg ( 270 + θ ) = tg ( 360 − ( 90 − θ ) ) = − tg ( 90 − θ ) = − cotg θ
b
cos( α ) = = sen(β)
a
c Observe que nessas relações o resultado é sempre a colinha
sen( α ) = = cos(β)
a trigonométrica com o sinal da linha trigonométrica original naquele
c quadrante. Por exemplo, no 4º quadrante, o seno é negativo, o
tg( α ) = = cot g(β)
b cosseno positivo e a tangente negativa.

π π
Dados dois números α e β tais que 0 < α < e 0<â < e π 
π 2 2 sen  + θ=
 cos θ
2º QUADRANTE

α + β = , teremos que: 2 
2 sen ( 90° + θ=
) cos θ
π 
cos  + θ  = − sen θ cos ( 90° + θ ) = − sen θ
sen ( α ) = cos ( β ) sen ( β ) = cos ( α ) tg ( α ) = cot g ( β ) 2 
tg ( 90° + θ ) = − cotg θ
π 
tg ( β ) = cot g ( α ) sec ( α ) = cos sec ( β ) sec ( β ) = cos sec ( α ) tg  + θ  = − cotg θ
2 

Vimos que a linha trigonométrica de um ângulo é igual à


colinha trigonométrica do seu complemento.  3π 
sen  − θ  = − cos θ
3º QUADRANTE

 2 
sen ( 270° − θ ) = − cos θ
 3π 
π  cos  − θ  = − sen θ cos ( 270° − θ ) = − sen θ
=
sen θ cos  − θ   2 
2 
sen θ cos ( 90° − θ )
= tg ( 270° −=
θ ) cotg θ
 3π 
π  cos θ sen ( 90° − θ )
=
tg  −=
θ  cotg θ
=
cos θ sen  − θ   2 
2 
tg θ cotg ( 90° − θ )
=
π 
=
tg θ cotg  − θ   3π 
2  sen  + θ  = − cos θ
4º QUADRANTE

 2 
sen ( 270° + θ ) = − cos θ
 3π 
Assim, usando as relações de redução ao 1º quadrante e depois cos  + θ=
 sen θ cos ( 270° + θ=
) sen θ
as relações acima, é possível relacionar as linhas trigonométricas dos  2 
ângulos da forma 90° + θ e 270° ± θ com as linhas de θ. Entretanto, tg ( 270° + θ ) = − cotg θ
 3π 
podemos desenvolver também relações para fazermos essa associação tg  + θ  = − cotg θ
 2 
em um passo único, conforme a seguir:

385

PM_BOOK16 - MAT.indb 385 25/11/2022 19:18:00


CÍRCULO TRIGONOMÉTRICO

PASSO A PASSO PARA A REDUÇÃO AO 1º QUADRANTE OU sec y


06. (EEAR) Se sen y = m e cos y = n, o valor de é
SIMPLIFICAÇÃO DE EXPRESSÕES TRIGONOMÉTRICAS cos sec y
a) m.
1º) Calcule a primeira determinação positiva do ângulo; b) n².
2º) Identifique o quadrante do ângulo; c) mn.
3º) Identifique o sinal da linha trigonométrica original nesse m
quadrante; d) .
n
4º) Identifique se o ângulo é da forma “180° ± θ ou 360° – θ”
(referenciado a A ou A’) ou da forma “90° + θ ou 270 ± θ” 07. (EEAR) Ao simplificar a expressão (1 + cos x) (1 – cos x), tem-se
(referenciado a B ou B’); a) 1
5º) No primeiro grupo (referenciado a A ou A’) mantenha a linha b) sen²x
trigonométrica. No segundo grupo (referenciado a B ou B’) utilize
c) cos²x
a colinha trigonométrica; e
d) 2 + cos²x
6º) Atribua à linha trigonométrica obtida no 5º passo o sinal
identificado no 3º passo.
08. (EEAR) Para x ⋅ y ≠ 0 , a expressão a seguir equivale a
y² cos180° − x y sen 270° + y² sen 90°
EXERCÍCIOS DE x² cos 0°

FIXAÇÃO
y
a) .
x
1
b) .
x
3π y
01. (EEAR) Gabriel verificou que a medida de um ângulo é rad.
10 c) .
Essa medida é igual a x²
a) 48° y²
d) .
b) 54° x²
c) 66°
09. (EEAR) Sendo tg x = 1/t e sen x = u, uma maneira de expressar o
d) 72° valor de cos x é
a) t.

02. (EEAR) O valor de rad em graus é b) u/t.
30
a) 36. c) u · t.
b) 38. d) u + t.
c) 42.
d) 46. 10. (EEAR) O valor de sen 1270° é igual a
a) – cos 10°
5π b) – sen 30°
03. (EEAR) Um arco de circunferência de rad pode ser dividido em
_____ arcos de 30°. 6 c) – sen 10°
a) 6 d) – cos 30°
b) 5
c) 4 EXERCÍCIOS DE

TREINAMENTO
d) 3


04. (EEAR) Ao somar as medidas angulares 120° e rad, obtém-se
2
a medida de um arco pertencente ao ___ quadrante
sen x ⋅ sec x
a) 1º 01. (EEAR) Seja A = . com tg x ≠ 0 . Nessas condições, o
valor de A é tg x
b) 2º
c) 3º 2
a)
2
d) 4º
b) 2
05. (EEAR) Dados os ângulos de 30° e 150°, pode-se afirmar que c) 2
a) sen 30° = cos 150°. d) 1
b) sen 30° = sen 150°.
c) cos 30° = cos 150°. 02. (EEAR) Se A = tg 120° e B = tg 240°, então
d) cos 30° = – sen 150°. a) B = A.
b) B = – A.
c) B = 2A.
d) B = – 2A.

386

PM_BOOK16 - MAT.indb 386 25/11/2022 19:18:00


CÍRCULO TRIGONOMÉTRICO

tg x + cot g x 10. A figura abaixo representa uma circunferência trigonométrica em


03. Simplificando-se a expressão , obtém-se 5π
cos sec x que MN é diâmetro e o ângulo α mede radianos.
a) cossec x. 6
b) cos x.
c) sec x.
d) tg x.

04. (ESA) A soma dos valores de m que satisfazem a ambas as


m +1 m+2
igualdades sen x = e cos x = é
m m
a) 5
b) 6
c) –4
d) 4
e) –6

05. (EEAR) Considere x um arco do 3º quadrante e cotangente de x A razão entre as medidas dos segmentos AB e AC é
2 2 a) 26 3.
igual a ctg x. Se sen x = − , então o valor de =
A tg x + é
2 ctg² x
a) 3 b) 3.

b) 2 3
c) .
c) 2 2
d) 3 3
d) .
3
06. O seno de um arco de medida 2340° é igual a
a) –1 11. O valor de y = cos 150º + sen 300º – tg 225° – cos 90° é
b) –1/2 − 3−3
a) −
c) 0 2
d) 1/2 b) − 3 +1
c) − 3 −1
( 3 cos180° − 4 sen210° + 2 tg135° )
07. O número N = pertence ao d) 3 −1
intervalo (6 sen245° )
a) ] − 4 , −3[ 1
12. Se θ for um ângulo tal que 0° < θ < 90° e cos θ < , é CORRETO
b) [−3 , −2[ afirmar que: 5
c) [−2 , −1] a) 0° < θ < 30°.
d) ] − 1 , 0] b) 30° < θ < 45°.
c) 45° < θ < 60°.
23π
08. Considerando-se o arco trigonométrico α = rad, assinale a d) 60° < θ < 75°.
alternativa falsa. 3
e) 75° < θ < 90°.
a) =α 1.380°.
b) α dá três voltas e para no 4° quadrante. 13. Analise as afirmativas abaixo:
c) sen α = −sen 60°. I. cos 225º < cos 215º

d) =
cos α cos 60°.  5π   5π 
II. tg   > sen  
e) α dá três voltas e para no 1° quadrante.  12   12 
III. sen 160° > sen 172°
09. O valor de cos (2.280º) é Das afirmações acima:
1 a) todas são verdadeiras.
a) − .
2 b) todas são falsas.
1 c) somente II e III são verdadeiras.
b) .
2 d) somente II é verdadeira.
e) somente I e II são verdadeiras.
2
c) − .
2

3
d) − .
2

3
e) .
2

387

PM_BOOK16 - MAT.indb 387 25/11/2022 19:18:01


CÍRCULO TRIGONOMÉTRICO

sen 30° + tg 225° 32kπ


rad , onde k ∈  , é tal que
*
14. O valor da expressão é 19. (ESPCEX) O ângulo α =
π 3
a) 1. cos − sen ( −60°) a) senα ⋅ cos α > 0 , se k = 1
2
1 b) senα ⋅ cos α < 0 , se k = 2
.
b) 2 c) senα ⋅ cos α > 0 , se k = 3
− 3. d) senα não varia se k = 1 ou k = 2
c)
e) cos α não varia para k = 1 ou k = 2
d) 3.
1 5
− . 20. (ESPCEX) Sabendo que cos sec x = e que x pertence ao primeiro
2 4
e) quadrante, o valor da expressão 25 sen² x – 9tg² x é
a) 2
15. Assinale a alternativa que corresponde ao valor da expressão:
b) 3
 13π  2  11π   7π  2  31π  c) 0
6cos2   − 4 cos   + sen  −  + tg  
 6   4   6   3 
d) 4
e) 1
a) 6
b) 5
21. (ESPCEX) Sendo k ∈  o número de valores distintos assumidos
c) 9 kπ
por sen é igual a
2 9
d) 3 a) 5
23 b) 8
e)
4 c) 9
16. Considere as afirmativas abaixo. d) 10
I. tan 92° = – tan 88° e) 18
II. tan 178° = tan 88°
22. (FUVEST) Na figura a seguir, a reta r passa pelo ponto T = (0, 1) e é
III. tan 268° = tan 88° paralela ao eixo Ox. A semirreta Ot forma um ângulo α com o semieixo
IV. tan 272° = – tan 88° Ox (0° < α < 90°) e intercepta a circunferência trigonométrica e a reta
Quais estão corretas? r nos pontos A e B, respectivamente.
a) Apenas I e III.
b) Apenas III e IV.
c) Apenas I, II e IV.
d) Apenas I, III e IV.
e) Apenas II, III e IV.

π
17. Se 0 < x < , é válido afirmar-se que:
4
π 
a) sen  − x  = sen x
2 
b) cos( π − x) = cos x A área do ∆TAB, como função de α, é dada por:
c) sen ( π + x ) =sen x 1 − sen α
a) ⋅ cos α
2
π 
d) sen  − x  =
cos x
2  1 − cos α
b) ⋅ sen α
cos ( π + x ) =sen x 2
e)
1 − sen α
c) ⋅ tg α
53π 2
18. (ESPCEX) O valor de sen é igual a
6
a) cos 225° 1 − sen α
d) ⋅ cotg α
b) cos 150° 2
c) cos 60° 1 − sen α
e) ⋅ sen α
d) sen 210° 2
e) sen 120°
23. (EEAR) Comparando-se tg 20°, tg 110° e tg 200°, obtém-se
a) tg20° = tg 200° > tg110°.
b) tg 20° = tg 110° < tg 200°.
c) tg 20° < tg 110° < tg 200°.
d) tg 200° < tg 20° < tg 110°.

388

PM_BOOK16 - MAT.indb 388 25/11/2022 19:18:02


CÍRCULO TRIGONOMÉTRICO

24. (EEAR) Se x é um arco do 1º quadrante, com sen x = a e cos x = b, 30. (AFA) Na figura abaixo, a circunferência de centro O é
senx ⋅ cos x  tem medida α, 0 < α < π , e OMP é um
trigonométrica, o arco AM
então y = é
tgx ⋅ cos ( π + x ) 2
a) a triângulo retângulo em M. Esse triângulo tem por perímetro
b) b
c) –a
d) –b

25. (ESPCEX) Se y é a medida de um ângulo 0° < y < 30°, o maior


dentre os números sen y, cos y, sen² y, cos² y e sen y · cos y é
a) sen y
b) cos y
c) sen² y
d) cos² y
e) sen y · cos y
1 + sen α + cos α 1 + 2sen α + cos α
a) c)
sen x − cos x
3 3 cos α cos α
26. (ESPCEX) A expressão é equivalente a
senx − cos x 1 + sen α + cos α 1 + sen2α + cos α
a) 1 b) d)
sen α sen α
b) 2
c) sen x + cos x  π 2nπ   π 2nπ 
31. (ESPCEX) Considere a expressão Vn = cos  +  + sen  + 
d) 1 + sen x · cos x 3 3  3 3 
onde n ∈ N. O valor de V0 + V2 é igual a
2
e) a) 2+ 3 d) 2− 3
senx
b) 3 e) 0
1 3 − x2 c) 1
27. (ESPCEX) Se=
cos sec θ = e sec θ , então um valor
x −1 3 − x2
de x que verifica essas igualdades é
32. (ESPCEX) Um triângulo equilátero ABC é inscrito num círculo
1 trigonométrico de raio unitário, conforme a figura abaixo.
a)
2
1
b)
3
1
c)
4
3
d)
4
3
e)
2
1
28. (AFA) Se x é um arco do terceiro quadrante e tg x = , então sen
x + sec x vale 3

10
a)
3

13 10
b)
30

10 Os vértices do triângulo estão nos pontos:


c)
5  3 1  3 −1
a) A  ,  , B ( −1, 0 ) e C  , 
d) 3 10  2 2  2 2
5
 3 1 1 − 3
b) A  ,  ,B ( −1,0 ) e C  , 
( cos sec x ) − 2
2

29. (AFA) Simplificando a expressão , para cossec x  2 2 2 2 


( cos sec x )
2
0, obtemos
1 2 1 − 2
a) cos x c) A  ,  , B ( −1,0 ) e C  , 
b) cos x 2 2 2  2 2 

c) sen2 x  2 2  2 − 2
d) A  ,  , B ( −1,0 ) e C  , 
d) cos 2x  2 2   2 2 

1 3 1 − 3
e) A  ,  , B ( −1,0 ) e C  , 
2 2  2 2 

389

PM_BOOK16 - MAT.indb 389 25/11/2022 19:18:03


CÍRCULO TRIGONOMÉTRICO

33. (ESPCEX) O número de arcos existentes 0° e 1560° cujo seno vale 36. Considere o produto abaixo, cujos fatores são os cossenos de
2 todos os arcos trigonométricos cujas medidas, em graus, são números
é inteiros pertencentes ao intervalo [91, 269].
7
a) 6 d) 9 =P cos 91° ⋅ cos 92° ⋅ cos 93° ⋅ ... ⋅ cos 268° ⋅ cos 269°
b) 7 e) 10
c) 8 Nessas condições, é correto afirmar que
1
a) −1 < P < − .
34. Na figura, em que está representada a circunferência 4
trigonométrica, P é a extremidade de um arco trigonométrico da 1ª 1
volta cuja medida, em radianos, é igual a α. Observe que P é um ponto b) − < P < 0.
4
do 2º quadrante localizado no interior do retângulo ABCD.
c) P = 0.
1
d) 0<P < .
4
1
e) < P < 1.
4

37. No círculo trigonométrico de raio unitário indicado na figura, o


 mede α. Assim, PM é igual a
arco AB

As coordenadas dos vértices do retângulo são dadas por:


 2 3  2 3  2 3  2 3
A =  ;  , B=  − ;  , C =
 − ,− =eD  ;−  .
 2 2   2 2   2 2   2 2 

Assim, é necessariamente verdadeira a desigualdade


π 2π 5π
a) <α< d) <α<π
2 3 6
2π 3π 7π a) −1 − tg α
b) <α< e) π<α<
3 4 6 b) 1 − cos α
3π 5π c) 1 + cos α
c) <α<
4 6 d) 1 + sen α
e) −1 + cotg α
35. Na figura abaixo, em que o quadrado PQRS está inscrito na
 e AQ
circunferência trigonométrica, os arcos AP  têm medidas iguais 38. (ESPCEX) O valor numérico da expressão a seguir é:
a α e β, respectivamente, com 0 < α < β < π. sec 1320°  53π 
 + ( tg 2220° )
2
− 2 ⋅ cos 
2  3 
a) –1
b) 0
1
c)
2
d) 1
3
e) −
2

 3π 
cos  − a  ⋅ csc( − a)
39. Simplifique y =  2  .
π 
tg(π + a) ⋅ sec  + a 
2 
a) –cos a
Sabendo que cos α = 0,8, pode-se concluir que o valor de cos β é b) sen a
a) −0, 8. d) 0, 6. c) tg a
b) 0, 8. e) −0, 2. d) –sec a
c) −0, 6. e) cossec a

390

PM_BOOK16 - MAT.indb 390 25/11/2022 19:18:04


CÍRCULO TRIGONOMÉTRICO

7cos(5π − x) − 3cos(3π + x) π EXERCÍCIOS DE

COMBATE
40. Sendo A = , com x ≠ + kπ , k ∈ ,
π  2
então: 8sen  − x 
2 
a) A = –1
b) 2A = 1
c) 2A + 1 = 0  15   
sen   x   cot g  x  
d) 4A + 5 = 0 01. O valor da expressão y   2   2  , com x
e) 5A – 4 = 0 cos 180  x   sec(  x )
pertencente ao 1º quadrante, é:
a) y   cos x d) y   sec x
41. Determine o valor da expressão E = cos 15° + cos 30° + cos 45° +
... + cos 150° + cos 165° é: b) y  sen x e) y = cos sec x
a) –2 c) y = tg x
b) 4
02. (AFA 1998) Seja P o produto dos fatores  sen n  cos n  , onde
c) 1
n = 45, 46, 47, ,149,150 . Pode-se afirmar que:
d) 0
a) P = 0 c) 1≤P<8
e) –1
b) P = 2 90
d) 8 ≤ P < 290

42. Sabendo que tgy = 3 e π < y < , calcule o valor numérico da
2 03. (ESPCEX 2012) O valor numérico da expressão
sec y − tg ( π + y)
2 2
expressão: . sec 1320  53 
   tg 2220  é:
 2
2sen( π − y) ⋅ cot gy  2  cos 
a) 1 2  3 
1 3
b) −1 a) –1 c) e) −
2 2
c) 3
b) 0 d) 1
2
d) −
2
04. (EFOMM 2010) O valor numérico da expressão
e) 3 44   33 
3 cos  sec 2400  tg   
3  4  é igual a:
π 3π 5π 7π 9π cossec2  780 
43. Simplificando tg ⋅ tg ⋅ tg ⋅ tg ⋅ tg , obtém-se
20 20 20 20 20 4 3
a) 1 a) 1 c) e)
3 8
b) –1
3 1
3 b) − d)
c) 4 2
3
d) 3 
05. Seja o ângulo θ tal que     . Sobre a expressão
e) 0 2
 29   35   23 
sen       cos       sec    
π 3  2   2   2 
44. (EFOMM) Sabendo que < θ < π e que senθ = , o valor de y
2 5 sen  11     cos  13     sec  16   
π 
cos  + θ  − sen ( π − 2θ ) é igual a: pode-se afirmar que:
2 
9 a) y>0 d) y  1
a)
25 b) y<0 e) y =1
39
b) − c) y  cotg 
25
c) 2− 2 06. (ESPCEX) O valor de (cos 165º + sen 155º + cos 145º – sen 25º +
4+ 5 cos 35º + cos 15º ) é:
d)
25 a) √2 d) 1
3− 2 b) -1 e) 1/2
e)
9 c) 0

45. A expressão a seguir onde θ é um ângulo do 3° quadrante vale: 07. Seja o ângulo θ tal que 90    180. Sobre a expressão
sen  2610     cos  3150     sec  2070   
 19π   21π   23π  y
sen (17π − 2θ ) ⋅ sen  − θ  + cos  + θ  ⋅ sen  + 2θ  sen  1980     cos  2340     sec  2880   
 2   2   2 
pode-se afirmar que:
a) sen θ
a) y>0 d) y  1
b) – sen θ
b) y<0 e) y =1
c) cos θ
c) y  cotg 
d) sen 2θ
e) –cos 2θ

391

PM_BOOK16 - MAT.indb 391 25/11/2022 19:18:21


CÍRCULO TRIGONOMÉTRICO

08. (ESPCEX) O produto cotg x ⋅ cos x é positivo, portanto x pertence ao: EXERCÍCIOS DE COMBATE
a) 1º ou 2º quadrantes. d) 2º ou 4º quadrantes. 01. B 04. E 07. A 10. A
b) 1º ou 4º quadrantes. e) 3º ou 4º quadrantes. 02. A 05. A 08. A
c) 2º ou 3º quadrantes. 03. D 06. C 09. C

09. (ESPCEX) No círculo trigonométrico (raio = 1), representado na ANOTAÇÕES


figura, a medida de β é 150° e AB representa um diâmetro. O valor do
produto das medidas dos segmentos OC e OD é:

A D
β
C
O x

1 1 3 3
a) a) d) d)
4 4 2 2
1 1 2 2
b) b) e) e)
2 2 2 2
3 3
c) c)
4 4

  11   x
2 sen  x     cotg2 x   tg
10. (ITA 2009) A expressão   2   2 é
equivalente a: x
1 tg2
2
a) cos x  sen2 x   cotg x
b) sen x  cos x   tg x
c) cos2 x  sen x   cotg2 x
d) 1 cotg2 x   sen x

e) 1 cotg2 x   sen x  cos x 

GABARITO
EXERCÍCIOS DE FIXAÇÃO
01. B 04. A 07. B 10. C
02. C 05. B 08. A
03. B 06. D 09. C
EXERCÍCIOS DE TREINAMENTO
01. D 13. C 25. B 37. C
02. B 14. D 26. D 38. D
03. C 15. A 27. E 39. A
04. E 16. D 28. B 40. C
05. D 17. D 29. D 41. D
06. C 18. C 30. A 42. B
07. C 19. E 31. C 43. A
08. E 20. C 32. E 44. C
09. A 21. C 33. C 45. A
10. B 22. D 34. B
11. C 23. C 35. C
12. E 24. A 36. B

392

PM_BOOK16 - MAT.indb 392 25/11/2022 19:18:25


FÓRMULAS DE ADIÇÃO

FÓRMULAS DE ARCO SOMA E  2  2 cos       2  2 cos  cos   2 sen  sen   cos       cos  cos  
DIFERENÇA  2  2 cos       2  2 cos  cos   2 sen  sen   cos       cos  cos   sen  sen
As fórmulas a seguir permitem calcular o seno e o cosseno da
soma e da diferença de arcos. cos       cos         cos  cos     sen  sen     cos  cos   sen 
cos       cos         cos  cos     sen  sen     cos  cos   sen     sen    cos  cos   sen
sen  
 
cos  sen 
cos 
 cos

 sen
 cos
 coscos     sen  sen     cos  cos   sen     sen    cos  cos   sen  sen 
sen       sen   cos   sen   cos 
        
cos       cos   cos   sen   sen  sen       cos          cos          cos     cos   sen  
2   2   2  2
cos       cos   cos   sen   sen 
         
sen       cos          cos          cos     cos   sen     sen   sen  cos   sen  cos 
2   2   2  2 
 Demonstração:
        
sen       cos          cos          cos     cos   sen     sen   sen  cos   sen  cos 
2   2   2  2 
sen       sen         sen  cos     sen    cos   sen  cos   sen  c
sen       sen         sen  cos     sen    cos   sen  cos   sen  cos 
sen       sen         sen  cos     sen    cos   sen  cos   sen  cos 

As fórmulas a seguir permitem o cálculo da tangente da soma e



da diferença de arcos, com , ,   ,      k, k   .
2
tg   tg 
tg      
1 tg   tg 
tg   tg 
tg      
1 tg   tg 

Demonstração:
sen      sen  cos   sen  cos 
Sejam Q, R e S a imagem no ciclo trigonométrico de arcos com tg        
primeira determinação positiva α, (α + β) e (–β), respectivamente. cos      cos  cos   sen  sen 
 = QS
Logo, AR  , o que implica AR = QS.  sen  cos   sen  cos    cos  cos 
 
As coordenadas desses pontos são dadas por:  cos  cos   sen  sen    cos  coss 
Q   cos ,sen   , R  cos      ,sen       e sen  sen 

S   cos    ,sen       cos ,  sen   . cos  cos  tg   tg 
 
sen  sen  1 tg   tg 
Aplicando a fórmula da distância entre pontos, temos: 1 
cos  cos 
AR  QS   cos       1   sen       0    cos   cos     sen     sen   
2 2 2 2
tg   tg   
t       tg         
   
 1
2
 sen     
 0
2
  cos   cos   
2
 sen     sen  
2
1  tg   tg   
tg   tg  tg   tg 
 cos2       2 cos       1 sen2       cos2   2 cos  cos   cos
2
1 sen 2
tg   tg

  1sen
2 sen 
 tg  sen 2
tg  
s       1 sen2       cos2   2 cos  cos   cos2   sen2   2 sen  sen   sen2 

s  cos   cos2   sen2   2 sen  sen   sen2 

393

PM_BOOK16 - MAT.indb 393 25/11/2022 19:18:39


FÓRMULAS DE ADIÇÃO

Exercício Resolvido cos 2  cos       cos  cos   sen  sen  

01. Calcule o seno, o cosseno e a tangente de 15°.  cos2   sen2   cos 2  cos2   sen2  
 cos2   1 cos2    2 cos2   1  cos 2 
Resolução:
 cos2   sen2   1 sen2    sen2   1 2 sen2 
sen15  sen  45  30  
tg   tg  2 tg 
 sen 45 cos 30  sen 30 cos 45  tg 2  tg       
1 tg   tg  1 tg2 
2 3 1 2 6 2
    
2 2 2 2 4 
Note que a fórmula de tg 2α só é válida se 2   k, k  .
cos 15  cos  45  30   2
 cos 45 cos 30  sen 30 sen 45  As fórmulas a seguir permitem calcular o seno, o cosseno e a
tangente do triplo de um arco.
2 3 1 2 6 2
    
2 2 2 2 4
tg 3 
tg 45  tg 30 sen 3  cos 3 
tg15  tg  45  30   3 tg   tg3 
1 tg 45  tg 30  3 sen   4 sen3   4 cos3   3 cos  
1 3 tg2 
3
1
3  3 3  3 
2

 3    Demonstração:
3 3 3 93
1 1 sen 3  sen  2     sen 2 cos   sen  cos 2 
3
12  6 3  2 sen  cos   cos   sen   1 2 sen2   
  2 3
6  2 sen  1 sen2    sen   2 sen3  
 3 sen   4 sen3 

Observação cos 3  cos  2     cos 2 cos   sen 2 sen  


A expressão y = a sen x + b cos x, onde a² + b² ≠ 0, tem valor   2 cos2   1  cos   2 sen  cos   sen  
 2 cos3   cos   2 cos  1 cos2   
2 2
mínimo  a2  b2 e valor máximo a + b .
 4 cos3   3 cos 
Demonstração:
y  a sen x  b cos x  tg 2  tg 
tg 3  tg  2     
1 tg 2  tg 
 a b
 a2  b2  sen x  cos x  2 tg 
2
 a b
2
a2  b2   tg 
1 tg2  2 tg   tg   tg3 
  
2 tg  1 tg2   2 tg2 
 a  
2
b 
2 1  tg 
Como  2 2    2 2   1 (relação fundamental 1 tg2 
 a  b   a  b  3 tg   tg3 
a 
da trigonometria), então podemos fazer  cos  e 1 3 tg2 
b a2  b2
 sen  . Assim, temos:
a  b2
2
FÓRMULAS DE ARCO METADE
y  a sen x  b cos x 
As seguintes fórmulas permitem calcular o seno, o cosseno e a
 a2  b2  cos  sen x  sen  cos x   tangente da metade de um arco, a menos do sinal.

 a2  b2 sen  x   
 1 cos 
sen 
  a2  b2  y  a2  b2 2 2

 1 cos 
cos 
FÓRMULAS DE ARCO DOBRO E TRIPLO 2 2
As fórmulas a seguir permitem calcular o seno, o cosseno e a  1 cos 
tangente do dobro de um arco. tg 
2 1 cos 
cos 2 
sen 2   cos2   sen2   2 tg  Demonstração:
tg 2 
 2 sen   cos  2
 2 cos   1  1 tg2    
cos   cos  2    2 cos2  1 
 1 2 sen2   2 2
 cos   1  cos   1
Demonstração:  cos2   cos  
2 2 2 2
sen 2  sen      
 sen  cos   sen  cos   2 sen  cos 

394

PM_BOOK16 - MAT.indb 394 25/11/2022 19:18:56


FÓRMULAS DE ADIÇÃO

   Exercício Resolvido
cos   cos  2    1 2 sen2 
 2 2
03. (UNIFESO 2006) Se x é a medida de um arco do primeiro
 1 cos   1 cos  quadrante e se sen x = 3 cos x, então sen (2x) é igual a
 sen2   sen  
2 2 2 2
5 1+ 5 3
 1 cos  a) c) e)
sen  5 5 2
 2 2 1 cos 
tg    3 4
2 cos  1 cos  1 cos  b) d)
 5 5
2 2
Resolução: B
Exercício Resolvido
sen x  3 cos x  tg x  3 
π 2 tg x 23 3
02. Calcule o seno, o cosseno e a tangente de .  sen 2x   
8 1 tg2 x 1 32 5

Resolução:
A fórmula seguinte permite calcular a tangente da metade de um
 ângulo conhecendo-se o seno e o cosseno do ângulo.
Como  Q , então todas as suas linhas trigonométricas são
8
positivas. x sen x
tg 
 2 2 1 cos x
1 cos 1
 4 4 2 2 2
sen  sen   
8 2 2 2 2 Demonstração:
 2 x x x x x
1 cos 1 2 sen cos 2 sen cos sen
 4 4 2 2 2 sen x 2 2  2 2 2  tg x
cos  cos    
8 2 2 2 2 1 cos x  x  x x 2
1  2 cos2  1 2 cos2 cos
 2  2 2
 2
1 cos 1
 4 4  2  Essa relação pode ser facilmente identificada no ciclo trigonométrico
tg  tg 
8 2 1 cos
 2 para ângulos agudos. Basta fazer AOP ˆ = x, o que implica AAˆ ’P = x
1 2
4 2 x
(ângulo inscrito) e podemos calcular a tg dividindo o cateto oposto
2 2 2 2 2 2 sen x pelo cateto adjacente 1 + cos x. 2
    2 1
2 2 2 2 42
FÓRMULAS DE PROSTAFÉRESE
OU DE WERNER
FÓRMULAS DE DUPLICAÇÃO As fórmulas de Prostaférese ou de Werner permitem transformar
USANDO TANGENTE somas ou diferenças de senos, cossenos e tangentes em produtos ou
vice-versa.
As seguintes fórmulas permitem calcular o seno e o cosseno de
um arco conhecendo-se a tangente do seu arco metade. As fórmulas a seguir permitem transformar somas e diferenças
em produtos.
x x
2 tg 1 tg2
sen x  2 cos x  2 pq pq
x x sen p  sen q  2 sen cos
1 tg2 1 tg2 2 2
2 2
pq pq
sen p  sen q  2 sen cos
Demonstração: 2 2
x
sen
x x 2  cos2 x 
sen x  2 sen cos  2 
2 2 x 2
cos pq pq
2 cos p  cos q  2 cos cos
x x 2 2
2 tg 2 tg
 2  2 pq pq
x x cos p  cos q  2 sen sen
sec2 1 tg2 2 2
2 2
 2 x 
x
2 2 x
 sen 2  2 x
cos x  cos  sen  1   cos 
2 2  cos2 x  2 sen p  q
 2
tg p  tg q 
cos p  cos q
2 x 2 x
1 tg 1 tg
 2 2 sen p  q
2 x 2 x tg p  tg q 
sec 1 tg cos p  cos q
2 2

395

PM_BOOK16 - MAT.indb 395 25/11/2022 19:19:12


FÓRMULAS DE ADIÇÃO

Demonstração: cos p  q  cos p  q 


pq pq
sen p  sen    p  q   p  q  p  q   p  q 
 2 2   2 cos cos 
2 2
pq pq pq pq  2 cos p cos q
 sen   cos    sen   cos  
 2   2   2   2 
1
pq pq  cos p  cos q  cos p  q  cos p  q 
sen q  sen    2
 2 2 
sen p  q  sen p  q 
pq pq pq pq
 sen   cos    sen   cos   p  q   p  q  p  q   p  q 
 2   2   2   2   2 sen cos 
2 2
 sen p  sen q 
 2 sen p cos q
pq pq
 2 sen   cos    sen p  sen q  1
 2   2   sen p  cos q  sen p  q  sen p  q 
2
pq pq
 2 sen   cos  
 2   2 
Exercício Resolvido
pq pq
cos p  cos    04. (IME 2012) O valor de y = sen 70° cos 50° + sen 260° cos
 2 2 
280° é:
pq pq pq pq
 cos   cos    sen   sen  
 2   2   2   2  a) 3 3 3
c) e)
pq pq 3 5
cos q  cos    3
 2 2  b) 3
d)
pq pq pq pq 2 4
 cos   cos    sen   sen  
 2   2   2   2 
Resolução: D
 cos p  cos q 
1
pq pq Como sen a  cos b  sen  a  b   sen  a  b  , temos:
 2 cos   cos    cos p  cos q  2
 2   2 
y  sen 70 cos 50  sen 260 cos 280 
pq pq
 2 sen   sen   1
 2   2   sen  70  50   sen  70  50   
2
sen p sen q
tg p  tg q    1
cosp cos q  sen  260  280   sen  260  280   
2
sen p cos q  sen q cos p sen p  q 1
  sen120  sen 20    sen 540  sen  20   
  1
cos p cos q cos p cos q 2 2
  sen 60  sen 20    sen  3  180   sen 20  
sen p sen q 1 1
tg p  tg q   
cosp cos q 2 2
sen p cos q  sen q cos p sen p  q 1 3  3
     sen 20  0  sen 20  
cos p cos q cos p cos q 2 2  4

As fórmulas a seguir permitem transformar produtos em soma. REFERÊNCIA: Gelca, R e Andreescu, T. – Putnam and Beyond – pg. 233.

1
sen p  sen q  cos p  q  cos p  q  EXERCÍCIOS DE
2

cos p  cos q 
1
2
cos p  q  cos p  q  FIXAÇÃO
1
sen p  cos q  sen p  q  sen p  q  01. (EEAR) O valor de cos 735º é:
2

1 3 2+ 6 2+ 6
a) b) c) d)
4 4 4 8
Demonstração:
cos p  q  cos p  q 
p  q   p  q  p  q   p  q  1
 2 sen sen  02. (CFT) Considerando tg 25° =
tg25º , o valor de tg 20º será:
2 2 2
 2 sen p sen  q  2 sen p sen q a) 1/6. b) 1/5. c) 1/4. d) 1/3.

1
 sen p  sen q  cos p  q  cos p  q  03. (EEAR) O valor correspondente ao cos 15º é:
2
2+ 6 2+ 3 3
a) b) c) d) 1
4 2 4

396

PM_BOOK16 - MAT.indb 396 25/11/2022 19:19:27


FÓRMULAS DE ADIÇÃO

4 36 π
04. (EEAR) Se senα ⋅ cos β = e senβ ⋅ cos α = , então sen ( α + β ) 05. (PUCRJ) Sendo x um arco e satisfazendo < x < π e sen(x) =
24
,
é igual a: 13 65 2 25
 x
a) 56/65 b) 40/65 c) 13/36 d) 13/56 o valor de cos   é:
 2
1 1 3
05. (EEAR) Dados sena = x, cosa = y, senb = z e cosb = w, então a) c) e)
sen(a + b) é igual a: 25 5 5
a) xw + yz b) xz + yw c) xy – wz d) xw – yz 1 3
b) − d) −
5 5
π x x
06. (EEAR) Se 0 < x < , então a expressão tg + cotg é
2 2 2 06. (FGV) Se 1 + cos α + cos2 α + cos3 α + cos 4 α + ... =5, com
equivalente a: π
0 < α < , então, sen2α é igual a:
a) 2senx b) 2secx c) 2cosx d) 2cossecx 2
a) 0,84 d) 0,94
07. (EEAR) Sendo a – b = 30º, calculando y = (sen a + cos b)² + b) 0,90 e) 0,96
(sen b – cos a)², obtemos: c) 0,92
2 3
a) 1 b) c) 3 d) 2+ 07. (MACKENZIE) A expressão
3 2
cos(a2 − 2b2 ) ⋅ cos(b2 ) − sen(a2 − 2b2 ) ⋅ sen(b2 ) é igual a:
π π π π π 4 π 16π d) sen[(a + b) ⋅ (a − b)]
08. (ESPCEX) Sendo X = + + + ... e Y = + + + + ..., a) cos(a2 + b2 )
3 6 12 4 5 25 125
o valor de sen (x + y) é: b) sen(b2 ) e) cos[(a + b) ⋅ (a − b)]

− 3+ 2 − 6− 2 3− 2 c) cos(a2 )
a) c) e)
2 4 2

− 6+ 2 6− 2 08. (ESPCEX) O cosseno do menor ângulo formado pelos ponteiros de


b) d) um relógio às 14 horas e 30 minutos vale:
4 4

09. (ESPCEX) Para todo x real, podemos afirmar que: ( 3 +1) c)


(1+ 2 ) e)
( 2+ 3 )
a) − 4 4
2
a) cos x = − cos ( π + x ) d) − cos
= x cos ( 2π − x )
( 2 +1) d) −
( 6− 2 )
b) x cos ( π − x )
cos= e) =
cos x sen ( 2π + x ) b) − 4
2
π 
c) cos x =
−sen  − x 15
2  09. (FGV) Se sen x + sen y =e cos x + cos y = 1, então,
3
10. (ESPCEX) O valor de 3sen 10º ⋅ (tg 5º + cotg 5º) é igual a: sec(x - y) é igual a:
a) 3/2 b) 2 c) 3 d) 5 e) 6 a) 1 c) 2 e) 4
3
b) 1 d) 3
EXERCÍCIOS DE
2

TREINAMENTO 10. (FUVEST) O valor de (tg10º +cotg10º)sen20º é:


a) 1/2 d) 5/2
b) 1 e) 4
01. (EEAR) Simplificando a expressão sen(2π-x) + sen(3π+x), obtém-se:
c) 2
a) senx b) –senx c) 2senx d) -2senx
3π 1
11. (PUCRJ) Sabendo que π < x < e sen (x) = − , é correto afirmar
02. (AFA)O valor da expressão cos15º + sen105º é: que sen(2x) é: 2 3

6+ 2 6− 2 6+ 2 6− 2 2 3 4 2
a) b) c) d) a) − c) e)
4 4 2 2 3 8 9
03. (AFA) Seja ABC um triângulo retângulo em A, circunscrito por uma 1 1
 = x . A razão entre a área do triângulo b) − d)
circunferência de raio r, e ABC 6 27
e o quadrado da metade do valor da hipotenusa é:
d) cos 2x
2 2
a) sen 2x b) sen x c) cos x 4  π
2 2 2 12. (PUCRJ) Sabemos que cos x = e x ∈ 0,  . Quanto vale tg 2x?
5  2
04. (ESPCEX) Se cosx⋅cosy ≠ 0, então a soma tgx + tgy é equivalente 3 24 1
ao produto: a) c) e)
4 7 24
a) (senx + seny )(cos x ⋅ cos y ) d) senx ⋅ sec y ⋅ sen ( x + y )
7 1
b) (senx + seny )(sec x ⋅ sec y ) e) senx ⋅ seny ⋅ cos ( x + y ) b)
24
d)
25
c) sen ( x + y )( sec x + sec y )

397

PM_BOOK16 - MAT.indb 397 25/11/2022 19:19:31


FÓRMULAS DE ADIÇÃO

4
13. (MACKENZIE) Se sen x = e tgx < 0, então tg2x vale:
5

24 8
a) . d) .
7 3

24 4
b) − . e) − .
7 3

8
c) − .
3

14. (FUVEST) O número real x, com 0 < x < π , satisfaz a equação


log3 (1 − cos x) + log3 (1 + cos x) =
−2 . Então, cos2x + senx vale:
a) 1,15 b) 1,25 c) 1,35 d) 1,75
1 7 10
a) c) e)
3 9 9 19. (FGV) Uma esfera de raio r está apoiada sobre o chão plano em um
dia iluminado pelo sol. Em determinado horário, a sombra projetada
2 8 à direita do ponto onde a esfera toca o chão tinha comprimento de
b) d)
3 9 10 m, como indica a figura.

15. (ESPCEX) Para todo k ∈ , n ∈ * e x ∈ , a expressão


[(sen x + cos x)2– sen 2x]n é equivalente a:
n
  π 
a) [sen(2kπ )]n d) sen  2kπ + 2  
 

b) [cos(2kπ + π )]n e) [sen(nkπ )]

c) [cos(nkπ )] Nesse mesmo horário, a sombra projetada por uma vareta reta de
1m, fincada perpendicularmente ao chão, tinha 2m de comprimento.
Assumindo o paralelismo dos raios solares, o raio da esfera, em
16. (FGV) O valor de y no sistema de equações metros, é igual a:

 1 a) 5 5 − 10. c) 5 5 − 5. e) 10 5 − 10.
sen10°x − cos10°y =− sen50°
 é: b) 10 5 − 20. d) 5 5 − 2.
sen50°x + cos50°y = 1
 sen10°
20. (AFA) O valor da expressão cos 35° (sen 25° + cos 55°) +
4 3 tg31º + tg14º
a) + sen 35° (cos 25° - sen 55°) + é:
3 3
d) 1 − tg31º ⋅tg14º
3
b) 3 2−3 3+ 2 2+ 3 2− 3
a) b) c) d)
3 2 2 2 3
e)
c) 3 3 4
21. (AFA) No triângulo retângulo ABC, os catetos AB e AC medem,
 
respectivamente, 2 + 2 e 2. Seja D um ponto de AB, tal que AD = AC .
cos17° 0 sen17° 
 e ABC  , então
17. (MACKENZIE) Para a matriz quadrada M =  1 1 1  Se α e β são, respectivamente, as medidasde ADC
o valor do determinante de M é:
10
sen28° 0 cos 28° tg(α+β) é:

a) 2 -1 b) 2+2 c) 2 2 -1 d) 2 2+1
1 1
a) d)
16 128 22. (AFA) Sendo ABC um triângulo qualquer, o valor da expressão:
1 1  a b  a c  b c
b) e) Y =  +  ⋅ cosC +  +  ⋅ cosB +  +  ⋅ cos A, é:
32 256  b a  c a  c b

1 a) 1 b) 2 c) 3 d) 1
c)
64 2

tg x
18. (AFA) Ao saltar do avião que sobrevoa o ponto A (veja na figura), 23. Se sen ( x + y ) =
m e sen ( x − y ) =
n , então é igual a:
tg y
um paraquedista cai e toca o solo no ponto V. Um observador que está m+n m
a) 2mn c) e)
em R contacta a equipe de resgate localizada em O. A distância, em m−n n
km, entre o ponto em que o paraquedista tocou o solo e a equipe de
m−n d) m+n
resgate é igual a: b)
2 (m + n) 2 (m − n)

398

PM_BOOK16 - MAT.indb 398 25/11/2022 19:19:34


FÓRMULAS DE ADIÇÃO

24. (AFA) Simplificando a expressão 30. (FUVEST) A figura representa um quadrado ABCD de lado 1. O
 3π   3π  5
sen  − x + cos(4 π − x) + tg  − x , obtém-se uma nova ponto F está em BC, BF mede , o ponto E está em CD e AF
 2   2  4
expressão E. O conjunto domínio, o conjunto imagem e o período da  Nessas condições, o segmento DE mede:
função f(x) = E são, respectivamente: é bissetriz do ângulo BAE.

a) { x ∈  | x ≠ kπ,k ∈ } , , π
b) ,[ −1,1] ,2π

c) { x ∈ | x ≠
π
2 }
+ kπ,k ∈  , , π

d) { x ∈  | x ≠ kπ,k ∈ } ,[ −1,1] ,2π


25. (ESPCEX) Considere o triângulo com ângulos internos x, 45º e
120º. O valor de tg²(x) é igual a:

a) 3 − 2. d) 2 − 3.
3 5 9 5 13 5
a) c) e)
b) 4 3 − 7. e) 2 − 4 3. 40 40 40

c) 7 − 4 3. 7 5 11 5
b) d)
40 40
3  π
26. (AFA) Dados senx + cos x + , tem-se que cos  x −  vale:
3  4 π 3
31. (EFOMM) Sabendo que < θ < π e que senθ = , o valor de
−1 + 3 6 2 5
a) c)
2 3 π 
cos  + θ − sen ( π − 2θ ) é igual a:
2 
2 6
b) − d)
3 6 9 c) 2− 2 3− 2
a) e)
25 9
cos 2θ 39 d) 4+ 5
27. (FUVEST 2001) Se tg θ =2 , então o valor de é: b) −
1 + sen2θ 25 25
d) 1
a) –3 −
3
b) 2 32. (EN) Sabendo-se que tgx = a e tgy = b ; pode-se reescrever
3 e) 3
4 sen2x + sen2y
1 Z= como:
c) sen2x − sen2y
3
 1 − ab   a − b   1 + ab   −a + b 
a)   ⋅  d)   ⋅ 
1 + ab   a + b  1 − ab   a − b 

cos x senx 1  1 + ab   a − b   1 + ab   a + b 
28. (EFOMM) Se det = − , então o valor de b)   ⋅  e)   ⋅ 
seny cos y 3 1 − ab   a + b  1 − ab   a − b 
π
3sen ( x + y ) + tg ( x + y ) − sec ( x + y ) , para ≤ x + y ≤ π, é igual a:  1 − ab   a + b 
2 c)   ⋅ 
1 + ab   a − b 
a) 0 d) 3

b) 1 e) 1
33. (AFA) Um passageiro em um avião voando a 10,5 km de altura,
3 2 avista duas cidades a esquerda da aeronave. Os ângulos de depressão
c) 2 em relação as cidades são 30° e 75° conforme a figura abaixo. A
distância, em km, entre os prédios A e B situados nessas cidades é
igual a:
29. (FUVEST) Sejam x e y números reais positivos tais que x + y =π .
2
Sabendo-se que sen ( y − x ) =1 , o valor de tg2y − tg2x é igual a:
3

3 1
a) d)
2 4
1
5 e)
b) 8
4
1
c)
2

399

PM_BOOK16 - MAT.indb 399 25/11/2022 19:19:38


FÓRMULAS DE ADIÇÃO

a) (
21 3 − 1 ) c) ( 3)
21 02. (EFOMM 1997) Sabendo-se que   6730’ , o valor de
 
2 sen4    cos 4   é:
b)
21
2
( 3 −1 ) d) ( 3 − 1)  3  3
4
a) 5 2 d)
3
34. (AFA) Um balão sobrevoa certa cidade a uma altura de 750 m 3
b) 2
em relação ao solo, na horizontal. Deste balão avistam-se pontos 4 e) 3
luminosos A, B e C, conforme a figura abaixo. O valor da tgα é igual a: 4
2
c) 2
3

03. (EN 2015) O valor do produto cos 40 ⋅ cos 80 ⋅ cos 160 é:
1
a) −
8
1
b) −
4
c) –1
3
d) −
2
2
e) −
2

a) 7/3 b) 9/8 c) 3/2 d) 1/3  5   


04. (EN 1998) Sendo y  sen   cos   , o valor numérico de y é:
 12   12 
35. (EPCAR 3° ANO) Um observador, no ponto O da figura abaixo, vê um.
prédio segundo um ângulo de 75°. Se esse observador está situado a 1 3
a) +
uma distância de 12 m do prédio e a 12 m de altura do plano horizontal 2 4
que passa pelo pé do prédio, então a altura do prédio, em metros, é:
b) 3
2
c) 1
2
d) 3+2
e) 2  3 1
senx  seny 1
05. (EN 1994) Se  2 e tgx = , então tgy é igual a:
cos x  cos y 3
a) 3
1
d) −
b) 1 6
6
c) 0 e) −3

a) (
4 3+ 3 ) 06. (ESPCEX) Os pontos P e Q representados no círculo trigonométrico
b) 6( 2 + 2) abaixo correspondem às extremidades de dois arcos, ambos com
origem em (1, 0), denominados respectivamente α e β, medidos no
c) 3 +1 sentido positivo. O valor de tg (α + β) é:
d) 2+ 3
a) 3+ 3
3

EXERCÍCIOS DE
b) 3− 3

COMBATE
3
c) 2 + √3
d) 2 - √3
e) -1 + √3
5
01. (AFA 2000) Se a  b  , então 1 tg a 1 tg b  é:
4
a) 0 b) 1 c) 2 d) 3

400

PM_BOOK16 - MAT.indb 400 25/11/2022 19:20:03


FÓRMULAS DE ADIÇÃO

07. (ESPCEX) O cosseno do menor ângulo formado pelos ponteiros de


um relógio às 14 horas e 30 minutos vale: 2 (IME) Os lados a, b e c de um triângulo estão em PA nesta
 B e C,
ordem, sendo opostos aos ângulos internos A, 
a) − ( 3 + 1) c) (1 + 2) e) ( 2 − 3)
 −C
A 
2 4 4 cos
respectivamente. Determine o valor da expressão: 2
b) ( 2 + 1)  +C
A 
− d) ( 6 − 2)
2 − cos
4 a) 2 2
b) 2
13π 11π c) 2 2
08. (ESPCEX) O valor numérico da expressão a seguir sen ⋅ cos
12 12 d) 3
é:
a) 1/2 e) 4

b) 1/3
c) 1/4
d) 1/6
3 (ITA) Considere um retângulo ABCD em que o comprimento
do lado AB é o dobro do comprimento do lado BC. Sejam
M o ponto médio de BC e N o ponto médio de CM. A tangente
e) 1/8 do ângulo MÂN é igual a:

sen 32  sen 38  sen 70 1


09. O valor da expressão é igual a: a) .
cos 16.cos 19.cos 55 35
a) 0 2
b) .
b) 1 35
c) 2
4
d) 3 c) .
35
e) 4
8
d) .
2 35
1 tg x 
10. (ITA 1974) A expressão   é equivalente a:
 1 tg x  16
e) .
35
1 2 sen 2x
a)
1 sen 2x
1 2 sen 2x  3π 
b)
1 sen 2x 4 (ITA) Se tgx = 7 e x ∈  π ,  , então sen3x é igual a:
 2

c) 1 sen 2x
14
1 sen 2x a) − .
8
1 sen 2x
d) 14
1 sen 2x b) .
8
e) tg2x
14
c) .
4

14

DESAFIO PRO
d) − .
4

14
e) .
6
1 (IME) O valor de y = sen70º cos50º + sen260º cos280º é:

a) 3
5 (IME) Assinale a alternativa que apresenta o mesmo valor
da expressão  4cos2 ( 9°) – 3  4cos2 ( 27°) – 3 :
3 a) sen (9°)
b)
2
b) tg (9°)
3 c) cos (9°)
c)
3 d) sec (9°)

3 e) cossec (9°)
d)
4

3
e)
5

401

PM_BOOK16 - MAT.indb 401 25/11/2022 19:20:09


FÓRMULAS DE ADIÇÃO

GABARITO
EXERCÍCIOS DE FIXAÇÃO
01. C 04. A 07. C 10. E
02. D 05. A 08. B
03. A 06. D 09. A
EXERCÍCIOS DE TREINAMENTO
01. D 10. C 19. B 28. D
02. C 11. E 20. C 29. A
03. A 12. C 21. D 30. D
04. D 13. A 22. C 31. A
05. E 14. E 23. C 32. E
06. E 15. D 24. A 33. A
07. D 16. A 25. C 34. B
08. D 17. B 26. D 35. A
09. E 18. B 27. D
EXERCÍCIOS DE COMBATE
01. C 04. A 07. D 10. D
02. B 05. E 08. C
03. A 06. D 09. E
DESAFIO PRO
01. D 03. C 05. B
02. B 04. B

ANOTAÇÕES

402

PM_BOOK16 - MAT.indb 402 25/11/2022 19:20:10


FUNÇÕES TRIGONOMÉTRICAS E
FUNÇÕES TRIGONOMÉTRICAS INVERSAS

FUNÇÕES TRIGONOMÉTRICAS A análise do seno no ciclo trigonométrico permitiu identificar os


intervalos de crescimento, decrescimento e os pontos de máximo e
mínimo.
FUNÇÃO SENO
Vamos estudar a segunda derivada da função para identificar a
Seja P a imagem de um ângulo θ no ciclo trigonométrico. Já vimos sua concavidade.
que o seno do ângulo θ é definido como a ordenada de P, ou seja,
f(x) = sen x ⇒ f’(x) = cos x ⇒ f’’(x) = –sen x
sen   OPy . Assim, para obter o seno de θ,devemos projetar P sobre o
eixo vertical Oy, denominado eixo dos senos. Assim, no 1º e no 2º quadrantes, onde a função seno é positiva,
a segunda derivada será negativa e a concavidade da função estará
voltada para baixo. Já no 3º e no 4º quadrantes, onde a função
seno é negativa, a segunda derivada será positiva e a concavidade
da função estará voltada para cima. Nos arcos de imagem A e A’
ocorrem mudanças de concavidade, ou seja, esses pontos são pontos
de inflexão da função seno.
A partir dessa análise, vamos construir um esboço do gráfico da
função seno.
sen   OPy

FUNÇÃO COSSENO
Seja P a imagem de um ângulo θ no ciclo trigonométrico. Já vimos
A função seno é a função de  em  definida por f(x) = sen x. que o cosseno do ângulo θ é definido como a abscissa de P, ou seja,
O domínio da função seno é Dsen =  e a imagem Imsen = [–1,1]. cos   OPx . Assim, para obter o cosseno de θ, devemos projetar P
sobre o eixo horizontal Ox, denominado eixo dos cossenos.
A função seno é periódica de período 2π.

Vamos analisar o gráfico da função seno, estudando os valores do


seno de um ângulo de 0 a 2π. Assim, observe o que acontece com o
segmento orientado OPy conforme o ponto P dá uma volta no ciclo
trigonométrico.

1º) De A até B, ou seja, de θ = 0 até   , o seno cresce de cos θ = OP x
  2
f(0) = sen0 = 0 até f    sen  1 .
 2 2

2º) De B até A’, ou seja, de   até θ = π, o seno decresce de
2
 
f    sen  1 até f(π) = sen π = 0.
 2 2
3
3º) De A’ até B’, ou seja, de θ = π até   , o seno decresce de
2 A função cosseno é a função de  em  definida por f(x) = cos x.
3 3
f(π) = senπ = 0 até f    sen  1 . O domínio da função cosseno é Dcos =  e a imagem Imcos = [–1,1].
 2  2
A função cosseno é periódica de período 2π.
3
4º) De B’ até A, ou seja, de   até θ = 2π, o seno cresce de
  2
 
3 3
até f(2π) = sen2π = 0. Vamos analisar o gráfico da função cosseno, estudando os valores
f    sen  1
 2  2 do cosseno de um ângulo de 0 a 2π. Assim, observe o que acontece
com o segmento orientado OPx conforme o ponto P dá uma volta no
ciclo trigonométrico.

403

PM_BOOK16 - MAT.indb 403 25/11/2022 19:20:19


FUNÇÕES TRIGONOMÉTRICAS E FUNÇÕES TRIGONOMÉTRICAS INVERSAS


1º) De A até B, ou seja, de θ = 0 até   , o cosseno decresce de A função tangente é a função de Dtg em  definida por
2
  f(x) = tg x.
f(0) = cos 0 = 1 até f    cos  0 .
 2 2

2º) De B até A’, ou seja, de  



até θ = π, o cosseno decresce e a imagem Im = .
tg
 
O domínio da função tangente é Dtg  x   | x   k, k  
2 
2 A função tangente é periódica de período π.
 
de f    cos  0 até f(π) = cos π = –1.
 2 2
3
3º) De A’ até B’, ou seja, de θ = π até   , o cosseno cresce de Vamos analisar o gráfico da função tangente, estudando os
2
 3  3 valores da tangente de um ângulo de 0 a 2π. Assim, observe o que
f(π) = cos π = –1 até f    cos  0.
 2  2 acontece com o segmento orientado AP1 conforme o ponto P dá uma
3 volta no ciclo trigonométrico.
4º) De B’ até A, ou seja, de   até θ = 2π, o cosseno cresce
2 
 3  3 1º) De A até B, ou seja, de θ = 0 até   (exclusive), a tangente
de f    cos  0 até f(2π) = cos 2π = 1. cresce de f(0) = tg0 = 0 até +∞. 2
 2  2
A análise do cosseno no ciclo trigonométrico permitiu identificar 
2º) De B até A’, ou seja, de   (exclusive) até θ = π, a tangente
os intervalos de crescimento, decrescimento e os pontos de máximo cresce de –∞ até f(π) = tgπ = 0. 2
e mínimo.
3
Vamos estudar a segunda derivada da função para identificar a 3º) De A’ até B’, ou seja, de θ = π até   (exclusive), a
sua concavidade. tangente cresce de f(π) = tgπ = 0 até +∞. 2
f(x) = cos x ⇒ f’(x) = –sen x ⇒ f’’(x) = –cos x 3
4º) De B’ até A, ou seja, de   (exclusive) até θ = 2π, a
Assim, no 1º e no 4º quadrantes, onde a função cosseno é 2
tangente cresce de –∞ até f(2π) = tg2π = 0.
positiva, a segunda derivada será negativa e a concavidade da função
estará voltada para baixo. Já no 2º e no 3º quadrantes, onde a função A análise da tangente no ciclo trigonométrico permitiu identificar
cosseno é negativa, a segunda derivada será positiva e a concavidade os intervalos de crescimento e os pontos de descontinuidade.
da função estará voltada para cima. Nos arcos de imagem A e A’ Vamos estudar a segunda derivada da função para identificar a
ocorrem mudanças de concavidade, ou seja, esses pontos são pontos sua concavidade.
de inflexão da função cosseno. f(x) = tgx ⇒ f’(x) = sec² x ⇒ f’’(x) = 2tgx · sec² x
A partir dessa análise, vamos construir um esboço do gráfico da
função cosseno. Assim, no 1º e no 3º quadrantes, onde a função tangente é
positiva, a segunda derivada será positiva e a concavidade da função
estará voltada para cima. Já no 2º e no 4º quadrantes, onde a função
tangente é negativa, a segunda derivada será negativa e a concavidade
da função estará voltada para baixo. Nos arcos de imagem A e A’
ocorrem mudanças de concavidade, ou seja, esses pontos são pontos
de inflexão da função tangente. Nos arcos de imagem B e B’ também
há mudança de concavidade antes e depois, mas eles são pontos de
descontinuidade.
A partir dessa análise, vamos construir um esboço do gráfico da
função tangente.
FUNÇÃO TANGENTE
Já vimos que, no ciclo trigonométrico, o eixo paralelo ao eixo
Oy com a mesma orientação que este, e passando pelo ponto A, é
denominado eixo das tangentes.

Vimos também que, se um ângulo θ tal que    k   , k ∈ ,
2
tem imagem no ciclo trigonométrico P, então a tangente de θ é a
medida algébrica do segmento AP1, onde P1 é a interseção da reta OP
com o eixo das tangentes.

FUNÇÃO COTANGENTE
Já vimos que, no ciclo trigonométrico, o eixo paralelo ao eixo
Ox com a mesma orientação que este e passando pelo ponto B é
denominado eixo das cotangentes.
tg   AP1 Vimos também que, se um ângulo θ tal que θ ≠ k · π, k ∈ ,tem
imagem no ciclo trigonométrico P, então a cotangente de θ é a medida
algébrica do segmento BP2, onde P2 é a interseção da reta OP com o
eixo das cotangentes.

404

PM_BOOK16 - MAT.indb 404 25/11/2022 19:20:29


FUNÇÕES TRIGONOMÉTRICAS E FUNÇÕES TRIGONOMÉTRICAS INVERSAS

cotg   BP2
FUNÇÃO SECANTE

Seja θ um ângulo tal que    k   , k ∈ , e cuja imagem
2
no ciclo trigonométrico é P. A secante de θ é a medida algébrica
do segmento OP’, onde P’ é a interseção da reta tangente ao ciclo
trigonométrico em P com o eixo dos cossenos.
A função cotangente é a função de Dcotg em  definida por
f(x) = cotg x.
O domínio da função cotangente é Dcotg = {x ∈  | x ≠ kπ, k ∈ }
e a imagem Imcotg = .
A função cotangente é periódica de período π.

Vamos analisar o gráfico da função cotangente, estudando os


valores da cotangente de um ângulo de 0 a 2π. Assim, observe o que sec θ = OP'
acontece com o segmento orientado BP2 conforme o ponto P dá uma
volta no ciclo trigonométrico.

1º) De A até B, ou seja, de θ = 0 (exclusive) até   , a cotangente
2
 
decresce de +∞ até f    cotg  0 .
 2 2

2º) De B até A’, ou seja, de   até θ = π (exclusive), a
2

  
cotangente decresce de f    cotg  0 até –∞.
 2 2
3
3º) De A’ até B’, ou seja, de θ = π (exclusive) até   , a
2 A função secante é a função de Dsec em  definida por
 3  3
cotangente decresce de +∞ até f    cotg  0. f(x) = sec x.
 2  2

4º) De B’ até A, ou seja, de  


3
2
até θ = 2π (exclusive), a  
O domínio da função secante é Dsec  x   | x   k, k  
2 
 3  3 e a imagem Imsec = ]–∞,–1] ∪ [1,+∞[ =  – ]–1,1[ .
cotangente decresce de f    cotg  0 até –∞.
 2  2 A função secante é periódica de período 2π.

A análise da cotangente no ciclo trigonométrico permitiu identificar


os intervalos de decrescimento e os pontos de descontinuidade. Vamos analisar o gráfico da função secante, estudando os valores
Vamos estudar a segunda derivada da função para identificar a da secante de um ângulo de 0 a 2π. Assim, observe o que acontece
sua concavidade. com o segmento orientado OP’ conforme o ponto P dá uma volta no
ciclo trigonométrico.
f(x) = cotg x ⇒ f’(x) = –cossec² x ⇒ f’’(x) = 2 cotg x · cossec² x

Assim, no 1º e no 3º quadrantes, onde a função cotangente 1º) De A até B, ou seja, de θ = 0 até   (exclusive), a secante
cresce de f(0) = sec 0 = 1 até +∞. 2
é positiva, a segunda derivada será positiva e a concavidade da
função estará voltada para cima. Já no 2º e no 4º quadrantes, onde 
a função cotangente é negativa, a segunda derivada será negativa 2º) De B até A’, ou seja, de   (exclusive) até θ = π, a secante
2
e a concavidade da função estará voltada para baixo. Nos arcos de cresce de –∞ até f(π) = secπ = –1.
imagem B e B’ ocorrem mudanças de concavidade, ou seja, esses 3
pontos são pontos de inflexão da função cotangente. Nos arcos de 3º) De A’ até B’, ou seja, de θ = π até   (exclusive), a secante
decresce de f(π) = sec π = –1 até –∞. 2
imagem A e A’ também há mudança de concavidade antes e depois,
mas eles são pontos de descontinuidade. 3
4º) De B’ até A, ou seja, de   (exclusive) até θ = 2π, a
A partir dessa análise, vamos construir um esboço do gráfico da 2
função cotangente. secante decresce de +∞ até f(2π) = sec 2π = 1.
A análise da secante no ciclo trigonométrico permitiu identificar
os intervalos de crescimento, decrescimento, os pontos de máximo e
mínimo locais e os pontos de descontinuidade.

405

PM_BOOK16 - MAT.indb 405 25/11/2022 19:20:38


FUNÇÕES TRIGONOMÉTRICAS E FUNÇÕES TRIGONOMÉTRICAS INVERSAS

Vamos estudar a segunda derivada da função para identificar a 


2º) De B até A’, ou seja, de   até θ = π (exclusive), a cossecante
sua concavidade. 2
 
cresce de f    cossec  1 até +∞.
f(x) = sec x ⇒ f’(x) = sec x · tg x ⇒ f’’(x) = sec x · (tg² x + sec² x)  2 2
3
Assim, no 1º e no 4º quadrantes, onde a função secante é 3º) De A’ até B’, ou seja, de θ = π (exclusive) até   , a
2
positiva, a segunda derivada será positiva e a concavidade da função  3  3
cossecante cresce de –∞ até f    cossec  1 .
estará voltada para cima. Já no 2º e no 3º quadrantes, onde a função  2  2
secante é negativa, a segunda derivada será negativa e a concavidade 3
da função estará voltada para baixo. 4º) De B’ até A, ou seja, de   até θ = 2π (exclusive), a
2
A partir dessa análise, vamos construir um esboço do gráfico da  3  3
função cotangente. cossecante decresce de f    cossec  1 até –∞.
 2  2
A análise da cossecante no ciclo trigonométrico permitiu identificar
os intervalos de crescimento, decrescimento, os pontos de máximo e
mínimo locais e os pontos de descontinuidade.
Vamos estudar a segunda derivada da função para identificar a
sua concavidade.
f(x) = cossec x ⇒ f’(x) = –cossec x · cotg x ⇒
f’’(x) = cossec x · (cotg² x + cossec² x)
Assim, no 1º e no 2º quadrantes, onde a função cossecante é
positiva, a segunda derivada será positiva e a concavidade da função
estará voltada para cima. Já no 3º e no 4º quadrantes, onde a
função cossecante é negativa, a segunda derivada será negativa e a
concavidade da função estará voltada para baixo.
A partir dessa análise, vamos construir um esboço do gráfico da
FUNÇÃO COSSECANTE função cotangente.
Seja θ um ângulo tal que θ ≠ k · π, k ∈ , e cuja imagem no
ciclo trigonométrico é P. A cossecante de θ é a medida algébrica
do segmento OP’’, onde P’’ é a interseção da reta tangente ao ciclo
trigonométrico em P com o eixo dos senos.

ESTUDO DOS GRÁFICOS


Vamos estudar os gráficos de funções trigonométricas da forma
f(x) = A sen (Bx + C) + D. Para isso vamos analisar a influência de cada
um dos coeficientes separadamente. Observe que o desenvolvimento
cossec   OP" feito para a função cosseno se aplica de maneira similar às outras
funções trigonométricas.

REFLEXÃO EM RELAÇÃO AO EIXO OX


A função f(x) = –sen x possui gráfico simétrico ao gráfico de
g(x) = sen x em relação ao eixo Ox.
A função cossecante é a função de Dcossec em  definida por
f(x) = cossec x.
O domínio da função secante é Dcossec = {x ∈  | x ≠ kπ, k ∈ } e
a imagem Imcossec = ]–∞,–1] ∪ [1,+∞[ =  – ]–1,1[ .
A função cossecante é periódica de período 2π.

Vamos analisar o gráfico da função cossecante, estudando os


valores da cossecante de um ângulo de 0 a 2π. Assim, observe o que
acontece com o segmento orientado OP’’ conforme o ponto P dá uma
volta no ciclo trigonométrico.

1º) De A até B, ou seja, de θ = 0 (exclusive) até   , a cossecante
2

  
decresce de +∞ até f    cossec  1 .
 2 2

406

PM_BOOK16 - MAT.indb 406 25/11/2022 19:20:44


FUNÇÕES TRIGONOMÉTRICAS E FUNÇÕES TRIGONOMÉTRICAS INVERSAS

AMPLITUDE DESLOCAMENTO HORIZONTAL


A função g(x) = sen x tem amplitude 1 e imagem Img = [–1,1]. O gráfico da função f(x) = sen(Bx + C) é igual ao gráfico igual ao
A função f(x) = A sen x, com A > 0, tem amplitude A e imagem C  C
Imf [–A,A]. gráfico de g(x) = senBx deslocado na horizontal de − . Se     0,
B  B
 C
Exemplo 1: o gráfico se desloca para a direita e, se     0 , o gráfico se desloca
para a esquerda.  B
O gráfico da função f(x) = 2 sen x tem amplitude A = 2 e imagem
Im = [–2,2].
Observação
Para encontrar o deslocamento na horizontal da função f(x) = sen
C
(Bx + C), devemos fazer Bx  C  0  x   . Se o resultado
B
for positivo, o deslocamento é para a direita e, se for negativo, o
deslocamento é para a esquerda.

Exemplo 1:
 
A função f  x   sen  x   tem gráfico igual ao de f(x) = sen x
 4
π
deslocado de unidades para a esquerda.
4

Exemplo 2:
1 1
O gráfico da função f  x   sen x tem amplitude A = e
2 2
 1 1
imagem Im    ,  .
 2 2

Exemplo 2:
 
A função f  x   sen  x   tem gráfico igual ao de f(x) = sen x
π  4
deslocado de unidades para a direita.
4

PERÍODO
2
A função f(x) = sen Bx possui período T  .
B
Exemplo 1:
2 Exemplo 3:
A função f(x) = sen 2x possui período T   .
2  
A função f  x   sen  2x   tem gráfico igual ao de f(x) = sen x
 4
π
deslocado de unidades para a direita.
8

Exemplo 2:
x 2
A função f  x   sen possui período T   4 . DESLOCAMENTO VERTICAL
2 12
O gráfico da função f(x) = sen x + D é igual ao gráfico de f(x) = sen x
deslocado na vertical de D unidades. Se D > 0, o gráfico se desloca
para cima e, se D < 0, o gráfico se desloca para baixo.

407

PM_BOOK16 - MAT.indb 407 25/11/2022 19:20:55


FUNÇÕES TRIGONOMÉTRICAS E FUNÇÕES TRIGONOMÉTRICAS INVERSAS

Exemplo 1:
A função f(x) = sen x + 1 tem gráfico igual ao de g(x) = sen x
deslocado de 1 unidade para cima.

CÁLCULO DO PERÍODO
Exemplo 2: Seja f(x) uma função periódica de período P, então o período da
A função f(x) = sen x – 1 tem gráfico igual ao de g(x) = sen x P
função g(x) = A · f(Bx + C) + D é T = .
deslocado de 1 unidade para baixo. B
Note que as funções seno, cosseno, secante e cossecante são
periódicas de período 2π e as funções tangente e cotangente são
periódicas de período π.

Exercício Resolvido

01. Calcule o período das seguintes funções.


a) y = sen 2x  
f) y  cossec  2x  
x  6
b) y = cos
2 x 
c) y = tg 3x g) y  2 tg  3
 6 
x  2 
d) y = cotg cos  3x 
3 
h) y  3  1
ProBizu   2
e) y  sec  x  
O gráfico de f(x) = Asen (Bx + C) + D é tal que:  3

• |A| é a amplitude; Resolução: 2


2 e) T  2
• T é o período; 2 1
B a) T  
2 2
 C
f) T 
2 2
•    é o número de fase, ou seja, o deslocamento na b) T   4
 B 12 
horizontal (para direita, se positivo, ou para a esquerda, se g) T  6
 16
negativo); e c) T 
3 2
• D indica o deslocamento vertical (para cima, se positivo, ou h) T
para baixo, se negativo).  3
d) T   3
13
Exemplo:
 
Construa o gráfico de f  x   2 sen  2x    1 . Sejam f1(x) e f2(x) duas funções periódicas de período P1 e P2,
 4 P1 n1
respectivamente, com P1 ≠ P2. Se = , onde n1 e n2 são inteiros
1º) Constrói-se f1(x) = sen x. P2 n2
2º) Constrói-se f2(x) = –sen 2x, a partir de f1, com período positivos e primos entre si, então as funções (f1 + f2)(x) = f1(x) + f2(x) e
2 (f1 · f2)(x) = f1(x) · f2(x) são periódicas de período P = n2P1 = n1P2.
T  .
2
 
3º) Constrói-se f3  x   sen  2x   , a partir de f2, deslocando-se Exercício Resolvido
π  4
na horizontal para a direita.
8 02. Calcule o período das seguintes funções.

4º) Constrói-se f4  x   2 sen  2x   , a partir de f3, com a) y = tg 3x + cos 4x
amplitude 2.  4 
x
  b) y  sen  cos 3x
5º) Constrói-sef  x   2 sen  2x    1 , a partir de f4, 2
 4
deslocando-se 1 na vertical para cima. c) y = sec x – sen x

408

PM_BOOK16 - MAT.indb 408 25/11/2022 19:21:11


FUNÇÕES TRIGONOMÉTRICAS E FUNÇÕES TRIGONOMÉTRICAS INVERSAS

Resolução:
 2 
a) tg 3x : P1 
; cos 4 x : P2   ;
3 4 2
P1  3 2  
   T  3  2  
P2  2 3 3 2
x 2 2
b) sen : P1   4  ; cos 3x : P2  ;
2 12 3
P1 4 6 2
   T  1 4   6   4
P2 2 3 1 3
c) y = sec x – sen x: P1 = P2 = 2π 1 1
 sen 2x
1  2 sen x cos x 1
1 1 sen x cos x 2 2
y  sec x  sen x   sen x   
cos x cos x cos x cos x
1 1
1  2 sen x cos x 1  sen 2x
1 1 sen x cos x 2 2
  sen x   
cos x cos x cos x cos x
1 2
1 sen 2x : P1    ; cos x: P2 – 2π;
2 2
P1  1
   T  2    1 2  2
P2 2 2
FUNÇÃO ARCO COSSENO
Seja f: [0,π] → [–1,1] tal que f(x) = cos x uma função bijetora,
então a sua inversa é f-1: [–1,1] → [0,π] tal que f-1(x) = arccos x. Assim,
FUNÇÕES TRIGONOMÉTRICAS temos:
INVERSAS y = f(x) = cos x ⇔ x = f-1(y) = arccos y
Vamos agora estudar as funções trigonométricas inversas. Todas
as funções trigonométricas que nós estudamos não são bijetoras. Para Propriedade fundamental:
podermos definir suas funções inversas, vamos restringir o domínio
das funções de maneira conveniente a fim de obter uma função   arc cotg k, k    cotg   k    0, 
bijetora. O gráfico das funções trigonométricas inversas pode ser
obtido refletindo-se o gráfico da função trigonométrica em relação à
reta y = x (bissetriz dos quadrantes ímpares). Propriedades:
arccos   x     arccos x, x   11
, 
FUNÇÃO ARCO SENO
cos  arccos x   x; x   11
, 
  
Seja f :   ,    11
,  tal que f(x) = sen x uma função bijetora,
 2 2 arccos  cos y   y; y  0, 
  
então a sua inversa é f :  11
1
,     ,  tal que f-1 (x) = arcsen x. A figura seguinte mostra o gráfico da função arco cosseno.
Assim, temos:  2 2

y = f(x) = sen x ⇔ x = f-1(y) = arcsen y

Propriedade fundamental:
  
  arctg k, k    tg   k      , 
 2 2

Propriedades:
arcsen   x    arcsen x, x   11
, 
sen  arcsen x   x; x   11
, 
  
arcsen  sen y   y; y    , 
 2 2
A figura seguinte mostra o gráfico da função arco seno.

409

PM_BOOK16 - MAT.indb 409 25/11/2022 19:21:24


FUNÇÕES TRIGONOMÉTRICAS E FUNÇÕES TRIGONOMÉTRICAS INVERSAS

FUNÇÃO ARCO TANGENTE


  
Seja f :   ,    tal que f(x) = tg x uma função bijetora,
 2 2   
então a sua inversa é f 1 :     ,  tal que f-1(x) = arctg x. Assim,
temos:  2 2
y = f(x) = tg x ⇔ x = f-1(y) = arctg y

Propriedade fundamental:
  
  arctg k, k    tg   k      , 
 2 2

Propriedades:
arctg   x    arctg x, x  
FUNÇÃO ARCO SECANTE
tg  arctg x   x; x  
   
   Seja f : 0,    ,    , 1  1,  tal que
arctg  tg y   y; y    ,   2 2 
 2 2 f(x) = sec x uma função bijetora, então a sua inversa é
A figura seguinte mostra o gráfico da função arco tangente.    
f 1 : , 1  1,   0,    ,   tal que f-1(x) = arcsec x.
 2 2 
Assim, temos:
y = f(x) = sec x ⇔ x = f-1(y) = arcsec y

Propriedade fundamental:
   
  arcsec k, k    sec   k    0,    ,  
 2 2 

Propriedades:
arcsec   x     arcsec x, x  
sec  arcsec x   x; x  , 1  1, 
   
arcsec  sec y   y; y  0,    ,  
 2 2 
FUNÇÃO ARCO COTANGENTE A figura seguinte mostra o gráfico da função arco secante.
Seja f: ]0,π[ →  tal que f(x) = cotg x uma função bijetora, então
a sua inversa é f-1:  → ]0,π[ tal que f-1(x) = arccotg x. Assim, temos:
y = f(x) = cotg x ⇔ x = f-1(y) = arccotg y

Propriedade fundamental:
  arc cotg k, k    cotg   k    0, 

Propriedades:
arc cotg   x     arc cotg x, x  
cotg  arc cotg x   x; x  
arc cotg  cotg y   y; y  0, 
A figura seguinte mostra o gráfico da função arco cotangente.

FUNÇÃO ARCO COSSECANTE


    
Seja f :   , 0    0,   , 1  1,  tal que
 2   2
f(x) = cossec c uma função bijetora, então a sua inversa é
    
f 1 : , 1  1,     , 0    0,  tal que f-1(x) = arccossec x.
 2   2
Assim, temos:
y = f(x) = cossec x ⇔ x = f-1(y) = arccossec y

410

PM_BOOK16 - MAT.indb 410 25/11/2022 19:21:36


FUNÇÕES TRIGONOMÉTRICAS E FUNÇÕES TRIGONOMÉTRICAS INVERSAS

y = arccotg x x∈ y ∈ ]0,π[


Propriedade fundamental:


    
  arccossec k, k    cossec   k      , 0    0,  
 2   2 y = arcsec x x ∈ ]–∞,–1[ ∪ [1,+∞[ y  0,  
2

Propriedades:   
y = arccossec x x ∈ ]–∞,–1] ∪ [1,+∞[ y    ,   0
 2 2
arccossec  x    arccossec x, x  
cossec  arccossec x   x; x  , 1  1, 
Exercício Resolvido
    
arccossec  cossec y   y; y    , 0    0, 
 2   2 03. Calcule o valor das expressões a seguir:
A figura seguinte mostra o gráfico da função arco cossecante. 1
a) arcsen
2
2
b) arccos
2
c) arctg 3
d) arc cotg 1
e) arc sec 2
2 3
f) arccossec
3
 1
g) arcsen   
 2
 3
h) arccos   
 2 
i) arctg(–1)
OUTRAS PROPRIEDADES
j) arc cotg   3 
cos  arc sen x   1 x 2 , x   11
, 

sen  arccos x   1 x 2 , x   11


,  Resolução:
 1 
arcsen x  arccos x  , x   11
,  a) arcsen 
2 2 6

arctg x  arc cotg x  , x   2 
2 b) arccos 
2 4

arcsec x  arccossec x  , x  , 1  1,  
2 c) arctg 3 
3
 1 
arcsen x  arccossec   ; x   11
,   0 d) arc cotg1 
x 4
 1 
arccos x  arcsec   ; x   11
,   0 e) arcsec 2 
x 3
 1 2 3 
arctg x  arc cotg   ; x  * f) arccossec 
x 3 3
 1 
 1 g) arcsen     
arctg x  arc cotg    ; x  *  2 6
x
 3  5
QUADRO RESUMO h) arccos   
 2  6
FUNÇÃO 
DOMÍNIO IMAGEM i) arctg  1  
INVERSA 4
5
   j) arc cotg   3  
y = arcen x x ∈ [–1,1] y   ,  6
 2 2

y = arccos x x ∈ [–1,1] y ∈ [0,π]

  
y = arctg x x∈ y   , 
 2 2

411

PM_BOOK16 - MAT.indb 411 25/11/2022 19:21:56


FUNÇÕES TRIGONOMÉTRICAS E FUNÇÕES TRIGONOMÉTRICAS INVERSAS

x
EQUAÇÕES TRIGONOMÉTRICAS 02. Seja f : ( −π, π) →  definida por f(x) = cos   , então, é
 2
INVERSAS verdade que
a) A função é crescente no intervalo (–π,0], decrescente no intervalo
Devemos lembrar que as funções inversas resultam em ângulos.
[0,π) e não possui raízes reais.
Fazendo uma analogia a máquina de funções temos que as funções
trigonométricas inversas tem em sua entrada número e na sua saída b) A função é crescente no intervalo (–π,0], decrescente no intervalo
ângulo. Veja por exemplo a equação abaixo. [0,π) e possui duas raízes reais.
c) A função é decrescente no intervalo (–π,0], crescente no intervalo
π
arccosx + arccos2x = [0,π) e possui duas raízes reais.
3
d) A função é decrescente no intervalo (–π,π) e não possui raízes reais.
Sabemos que arccosx e arccos2xsão ângulos, assim e) A função é crescente no intervalo [0,π) e possui uma raiz real.
π
arccosx = α e arccos2x = β , assim temos que α + β = . Sabemos
3 03. Em 2014 foi inaugurada a maior roda-gigante do mundo, a High
que cos ( arccosx ) = x e cos ( arccos2x ) = 2x . Roller, situada em Las Vegas. A figura representa um esboço dessa
Assim vamos aplicar a função cosseno em ambos os lados da roda-gigante, no qual o ponto A representa uma de suas cadeiras:
igualdade.

π
cos ( α + β ) = cos  
3

1
cosα.cosβ − senα.senβ =
2

Como vimos cos ( arccosx ) = x e cos ( arccos2x ) = 2x porém


sen ( arccosx ) e sen ( arccos2x ) ? Vamos utilizar a relação fundamental.

Sendo cosα =x termos que sen2α = 1 − x 2 ⇒ senα = 1 − x 2 e da


mesma forma se cosβ =2x teremos sen β = 1 − 4x ⇒ senβ = 1 − 4x 2 .
2 2

1 1
cosα.cosβ − senα.senβ= ⇒ x.2x − 1 − x 2 . 1 − 4x 2 = ⇒ Disponível em: http:en.wikipedia.org. Acesso em: 22 abr. 2014 (adaptado).
2 2

1 1 A partir da posição indicada, em que o segmento OA se encontra


2x 2 − = 1 − x 2 − 4x 2 + 4x 4 ⇒ 4x 4 − 2x 2 + =1 − x 2 − 4x 2 + 4x 4 ⇒ paralelo ao plano do solo, rotaciona-se a High Roller no sentido
2 4
anti-horário, em torno do ponto O. Sejam t o ângulo determinado
3 1 1 pelo segmento OA em relação à sua posição inicial, e f a função que
3x 2 = ⇒ x2 = ⇒ x = ± descreve a altura do ponto A, em relação ao solo, em função de t.
4 4 2
Após duas voltas completas, f tem o seguinte gráfico:
Porém como resolvemos uma equação irracional devemos verificar
se alguma, ou mais de uma, das raízes que encontramos é ou são
“raízes estanhas”.
1
Testando a raiz x = − teremos
2
 1  1  1
arccos  −  + arccos2  − = arccos  −  + arccos ( −1) =
 2  2  2
π 4 π 2π
= += π ≠ .
3 3 3
1
Logo a única raiz da nossa equação é x = .
2 A expressão da função altura é dada por
Para resoluções de equações trigonométricas inversas é de suma =
importância lembrarmos o domínio e a imagem das funções, ou seja, a) f(t) 80 sen(t) + 88
saber bem o quadro resumo do tópico QUADRO RESUMO. =
b) f(t) 80 cos(t) + 88
=
c) f(t) 88 cos(t) + 168
EXERCÍCIOS DE =
d) f(t) 168 sen(t) + 88 cos(t)
FIXAÇÃO =
e) f(t) 88 sen(t) + 168 cos(t)

3
01. Considere a função real de variável real f(x) =
3 − 5 sen (2x + 4). Os 04. Seja f :  →  definida por f(x) = . Se M e m são
2 + sen x
valores de máximo, mínimo e o período de f(x) são, respectivamente,
respectivamente os valores máximo e mínimo que a função f assume,
a) −2, 8, π. d) π, 8, − 2. o valor do produto M · m é
b) 8, − 2, π. e) 8, π, − 2. a) 2,0. c) 3,0.
c) π. − 2, 8. b) 3,5. d) 1,5.

412

PM_BOOK16 - MAT.indb 412 25/11/2022 19:21:59


FUNÇÕES TRIGONOMÉTRICAS E FUNÇÕES TRIGONOMÉTRICAS INVERSAS

05. A atração gravitacional que existe entre a Terra e a Lua provoca, 10. O gráfico abaixo representa uma função real de variável real.
entre outros fenômenos, o da chamada maré astronômica, que se
caracteriza pelo periódico aumento e diminuição do nível do mar.
Medindo e tabulando essas variações, os estudiosos do assunto
podem descrever matematicamente o comportamento do nível do
mar em determinado local por meio de uma função.A fórmula a
seguir corresponde a medições feitas na cidade de Boston, no dia 10
de fevereiro de 1990.
π 
h(t) = 1,5 + 1,4 ⋅ cos  ⋅ t 
6 
Nessa função, h(t) (em metros) corresponde à altura do nível do mar, e t,
ao tempo transcorrido desde a meia-noite (em horas). Com base nessas
informações, quantas horas se passaram desde o início da medição até Assinale a alternativa em que consta a função representada pelo gráfico.
que o nível do mar tenha atingido 2,2 metros pela primeira vez?
a) f(x) = −2cos x d) f(x) = 2 sen 2x
a) 2 horas
x x
b) 3 horas b) f(x) = 2 cos e) f(x) = sen
2 2
c) 4 horas
c) f(x) = 2 sen x
d) 5 horas
e) 6 horas
EXERCÍCIOS DE
06. Os gráficos das funções reais f(x) = cos(x) e g(x) = sen (x) não
coincidem. Entretanto, a partir de uma transformação, é possível fazer
o gráfico de g(x) coincidir com o gráfico de f(x). Essa transformação
TREINAMENTO
é a função
π  π 01. (AFA) A função real f(x) = sen2x + cos x tem valor máximo em
a) h(x) = sen x. c) =
h(x) sen  x +  .
2  2 a) x = (2k + 1)π, k ∈  c) x = (2k + 1/2)π, k ∈ 
π   π b) x = (k ± 1/6)π, k ∈  d) x = (2k ±1/3)π, k ∈ 
b) h(x) = sen  x  . d) =
h(x) sen  x −  .
2   2
02. (AFA) Os valores de α, 0 ≤ α < 2π, que satisfazem a desigualdade
07. A pressão arterial é a pressão que o sangue exerce sobre as paredes –x2 + 1/2 < sen α, para todo x real, pertencem ao intervalo
das artérias. Ela atinge o valor máximo (pressão sistólica) quando os
ventrículos se contraem, e o valor mínimo (pressão diastólica) quando π 5π
a) 0<α< c) <α<π
eles estão em repouso. Suponhamos que a variação da pressão arterial 2 6
(em mmHg) de um cidadão portoalegrense em função do tempo (em
8π  π π 5
segundos) é dada por P(t) = 100 − 20 ⋅ cos  ⋅ t  . Diante disso, b) 0<α< d) <α< π
 3  6 6 6
os valores da pressão diastólica e sistólica, em mmHg, são iguais,
respectivamente, a 03. (AFA) O gráfico que melhor representa a função y = |sen x + cos x|,
y
a) 60 e 100 com 02≤ x < 2π, é
y
b) 60 e 120 a) y2
y2
c) 80 e 120 12

d) 80 e 130 1
1
1
e) 90 e 120 0 2
y0 2
20 2
1  π y0 2
08. Os valores de x (x ∈ ), para os quais a função
= f(x) tg  3x −  b)
y2
y
não é definida, são 3  4 2
12
a) π + kπ, k ∈  π 1
d) + kπ, k ∈  1
1
π 4 0  2 x
b) + kπ, k ∈ 
2 π kπ 0  2 x
e) + ,k∈ y0
0


2
2
x
x
3π 4 3 2
c) + kπ, k ∈  c) y
y
4 y2
2
2
1
09. Se f é uma função real dada por f(x) = 2 – cos(2x), então é correto 1
11
afirmar que
0
a) 1 ≤ f(x) ≤ 3 para todo x real. 0
0
0
b) O gráfico de f intercepta o eixo x. y
d)
c) f(x) ≤ 2 para todo x real. y2
yy
2
d) f(0) = 2. 22
1
e) f(x) ≥ 3 para todo x real.
1
11
0  2 x
0  2 x
00 
 2
2 xx

413

PM_BOOK16 - MAT.indb 413 25/11/2022 19:22:02


FUNÇÕES TRIGONOMÉTRICAS E FUNÇÕES TRIGONOMÉTRICAS INVERSAS

2 2 08. (AFA) Analise as alternativas seguintes e classifique-as como


04. (AFA) O valor de cotg (arc sen )é
3
verdadeiras (V) ou falsas (F).
2 2 I. O período e o conjunto-imagem da função f:  →  definida por
a) c)
2 4
1  1 1
3 2 f(x) = .sen x.cos x são, respectivamente, 2π e  − 4 , 4  .
b) 2 2 d) 4
4
II. A função y = 2 arc cos 4x tem por domínio o conjunto de todos os
05. (EN) Considerando a função f(x) = cos x , 0 ≤ x ≤ π , é inversível,
 1
 2 valores de x pertencentes a 0,  .
o valor de tg  arccos  é  4
 5
 π π
III. Para todo x ∈  − ,  , o valor de (tg2x + 1)(sen2 x –1) é –1.
21 21  2 2
a) − d)
5 25 A opção que corresponde à classificação acima é
4 21 a) F–V–F c) F–F–V
b) − e)
25 2 b) V–V–F d) V–F–V

21
c) − 09. (AFA) Considere a função real  →  , definida por
2
cos 2x
06. (AFA) Analise e classifique as sentenças como V (verdadeiras) ou y= e as seguintes afirmações:
1 + sen2x
F (falsas).
I. A função é decrescente em todo o seu domínio
( ) f:  →  definida por f(x) = cos x é par.
π
( ) f:  →  definida por f(x) = sen x é sobrejetora. II. O gráfico da função apresenta assíntotas nos arccos  + kπ  , k ∈ 
2 
π
( ) f: [0, π] → [–1, 1] definida por y = cos x é inversível. III. A função é positiva em 0, 
π π  4
( ) f:  − ,  → B definida por y = sen x é inversível, se, e somente
π
 2 2 IV. A função admite inversa em 0, 
se, B = [0, 1].  2 

A alternativa que corresponde à sequência correta é São verdadeiras somente as afirmativas contidas nos itens
a) V F V F a) I e II c) III e IV
b) F F V F b) II e III d) I e IV
c) VFVV
10. (AFA) Na figura abaixo tem-se a representação gráfica da função
d) F V F V x
real f(x) = 2sen para x ∈ [a,g].
2
07. (AFA) Em uma apresentação da esquadrilha da fumaça, dois
pilotos fizeram manobras em momentos diferentes deixando rastros
de fumaça, conforme mostra a figura abaixo.

altura

f
5
4
3
2 f1
1 É correto afirmar que o baricentro do triângulo DEF é o ponto
0 distância
a)  π 1
 ,  b)  π 2
 ,  c)  1
 π,  d)  2
 π, 
 2 3  2 3  3  3

As funções f1 e f2 que correspondem às manobras executadas pelos 11. (AFA) Analise os itens abaixo classificando-os como VERDADEIRO(S)
pilotos são ou FALSO(S).

4  4  1
a) f1(x)= 2 − sen  x  e f2 (x)= 4 − sen  x  I. Se senx + cos x = , então sen2x = −0,666...
3  3  3

4  II. Se f(x) =x 2 + 2x + senα , α ∈ [0, 2π], é positiva ∀x ∈ , então


f1(x)= 2 + sen  x  e f2 (x)= 4 − sen  x 
4
b)
3  3  π 5π
<α< .
6 6
π 2  π 4  III. O gráfico de f(x) = sen(arcsenx) é uma reta.
c) 4 + sen  − x  e f2 (x) =
f1(x) = 2 − sen  + x 
2 3  2 3 
A sequência correta é:
2  π 4  a) V, V, F c) F, V, V
d) f1(x) = 1 + sen  x  e f2 (x) =
1 − 3sen  − x 
3  2 3  b) F, V, F d) V, F, V

414

PM_BOOK16 - MAT.indb 414 25/11/2022 19:22:05


FUNÇÕES TRIGONOMÉTRICAS E FUNÇÕES TRIGONOMÉTRICAS INVERSAS

cos x 1 + senx
=
12. (AFA) Seja f: D → , definida por f(x) + . O gráfico 13. (AFA) Identifique as alternativas FALSAS, assinalando, a seguir, a
1 + senx cos x alternativa que corresponde à soma dos números a elas associados.
que MELHOR representa um período completo da função f é:
sen ( 3ex ) cos ( 3ex )
a) =
(01) A função f(x) − , para qualquer que seja x
sen ( e x
) cos ( ex )
pertencente ao seu domínio, tem imagem 2
(02) sen x + cos x ≥ 1 para todo x ∈ [0,π]
5  π 5
(04) Se sec x = , então cos sec  x −  =

4  2 4
 x − 1
(08) O domínio da função f(x) = arcsen   é o intervalo ] –7, 7]
 6 
(16) O período da função f(x) = |senx · cosx| é π

a) 26 b) 23 c) 15 d) 07

14. (AFA) Classifique em (V) verdadeira ou (F) falsa cada afirmativa abaixo.
b) 1
I. O domínio da função real f definida por f(x) = arccos é o
x −1
conjunto { x ∈  / x ≤ 0 ou x ≥ 2}.
II. No intervalo [0, 2π] o gráfico da função real y = –2sen³x corta o
eixo x um número ímpar de vezes.
III. A função real f: A → [0,1] tal que f(x) = sen²(2x) admite inversa,
π π
se A =  ,  .
4 2
Conclui-se que são verdadeiras
a) I, II e III
b) apenas I e III
c) apenas II e III
d) apenas I e II
c)
15. (AFA) Analise as proposições seguintes e classifique-as em (V)
verdadeiras ou (F) falsas.
( ) Se o ponteiro dos minutos de um relógio mede 10 cm, então
a distância que sua extremidade percorre em 30 minutos é de
aproximadamente 31,4 cm.
( ) O domínio da função real f definida por f(x) = sec x + cos sec x é
 kπ 
o conjunto x ∈  / x ≠ , com k ∈  .
 2 
( ) A equação cos x · tg x – cos x = 0 possui 4 raízesno intervalo
[0, 2π].
( ) O período e a imagem da função trigonométrica f definida por
f(x) = 2 cos² x – 2 sen² x, são respectivamente iguais a 2π e [–2, 2].
A sequência correta é
a) V–V–F–F
d) b) F–F–V–V
c) F–V–F–V
d) V–V–V–F

16. (AFA) Considerando as definições e propriedades das funções


trigonométricas, marque a alternativa correta.
a) A função f definida por f(x) = |sen2x + cos2x| possui período e
imagem, respectivamente, iguais a π e [0, 2].
b) Se f e g são funções tais que f(x) = tgx e g(x) = |x|, sabendo-se que
existe a função j definida por j(x) = (fog)(x), então j é periódica.
π π
c) No intervalo de  ,  a função h definida por h(x) = |cos2x|
é decrescente.  4 2

3x − 1
d) O domínio da função g : D →  definida por g(x) = 3arcsen
2
é D =  ,1 .
1
3 

415

PM_BOOK16 - MAT.indb 415 25/11/2022 19:22:06


FUNÇÕES TRIGONOMÉTRICAS E FUNÇÕES TRIGONOMÉTRICAS INVERSAS

17. (AFA) Considere as situações a seguir. a)


I. Suponha que a passagem de um pinguim, da água para a
superfície de uma geleira, possa ser representada comono
esquema da Figura 1.

b)

II. Suponha também que uma sequência de saltos uniformes de uma


lebre, possa ser representada como no esquema da Figura 2.

c)

Transportando as situações acima para um plano cartesiano, considere


• o eixo das abscissas coincidindo com o nível da água gelada para d)
o pinguim;
• o eixo das abscissas coincidindo com o solo para a lebre;
• a altura do salto do pinguim e da lebre indicada no eixo das
ordenadas.
Tendo por base as situações apresentadas, nas figuras 1 e 2 e ainda
a teoria dos gráficos das funções trigonométricas, pode-se relacionar
aos saltos um tipo de gráfico dessas funções. Assim sendo, as funções 21. (EPCAR 3° ANO) No sistema cartesiano abaixo, estão sobrepostos
P e L estabelecem os saltos do Pinguim e da Lebre, respectivamente. os gráficos detrêsfunções:y1=k1.cotgx,y2=k2.cotgxey3=k3.cotgx.
A opção que contém funções que podem representar a situação
descrita, sabendo-se que a função P está restrita a um único período, é

 π
a) P(x) =
−tg  x −  e L(x) =
2 senx
 2
 π
b) P(x) =cotg  x −  e L(x) =2sen | x |
 2
=
c) =
P(x) tgx e L(x) 2 sen2x

d) P(x) =
−2tgx e L(x) =
sen2x

18. (AFA) Considere a função real f: A → [1,3] definida por


 x π
1sen  −  Tem-se, necessariamente, que
f(x) =  2 6  . Sabendo-se que a função f é inversível, é
−2 1 a) k1< k2<k3 c) k3< k2<k1
b) k1= k2=k3 d) k2< k3<k1
correto afirmar que um possível intervalo para o conjunto A é

a)  π 7π  b)  4 π 7π  c)  4 π 10π   7π 10π  22. (EPCAR 3° ANO) Se a curva da figura abaixo representa o gráfico
 3 , 3   3 , 3   3 , 3  d)  3 , 3 
de um período da função y = a + b ⋅ cos (cx), (a, b, c ∈ ), então, o
valor da área hachurada é
19. (EPCAR 3° ANO) Seja a função f: [–1,0[ →  definida por
f(x) = tg(arccosx). O conjunto imagem de f é dado por

π 
a)  2 , π  c) ]−∞,0]
b) [0,1[ d) ]−∞,0[
π 
sen( −x).cos  + x 
 2 
20. (EPCAR 3° ANO) Simplificando a expressão
tg ( 2π − x ) .cos ( π − x )

obtém-se uma nova expressão E. Seja f :  →  definida por f(x) = E,


o gráfico que melhor representa f é

416

PM_BOOK16 - MAT.indb 416 25/11/2022 19:22:08


FUNÇÕES TRIGONOMÉTRICAS E FUNÇÕES TRIGONOMÉTRICAS INVERSAS

a)
1
(
π 1+ 3 ) c) (
3π 1 + 3 ) 27. (AFA) Sejam as funções reais f, g e h definidas por
sen x cos x
3 f (x)
= + , g(x) = |sec x| e h(x) = |cossec x|, nos seus
d) (
π 1+ 3 ) cossec x sec x
b)
1
2
(
π 1+ 3 ) domínios mais amplos contidos no intervalo [0,2π]. A(s) quantidade(s)
de interseção(ões) dos gráficos de f e g; f e h; g e h é(são),
respectivamente
23. (AFA) Em relação à função real f definida por a) 0, 0 e 4
f(x) =
1 − 8sen2 (2x)cos2 (2x) − 2 é INCORRETO afirmar que b) 3, 1 e 4
a) Im(f) = [–2, –1] c) 2, 3 e 4
d) 0, 2 e 3
π π
b) tem seu valor mínimo como imagem de algum x ∈  , 
8 4
π 28. (AFA) Considere A o conjunto mais amplo possível na função
c) seu período é igual a sen x cos x
8 real f : A →  , dada por
= f (x) + . Sobre a função f
 π 3π  cossec x sec x
d) é estritamente crescente em  , 
16 16  é correto afirmar que

24. (AFA) Considere as funções reais f e g definidas por


a) {
A =x ∈  | x ≠

2
, k∈ . }
 2 cos ( 2x )  b) é periódica com período igual a π.

{ }
1   1 ( )
f ( x )= ⋅ 1 , g ( x )= −f x e marque a π
2 2sen ( 2x ) 2 c) é decrescente se x ∈ x ∈  | + 2kπ < x < π + 2kπ, k ∈  .
 2  2
alternativa INCORRETA. d) é ímpar.
a) O conjunto imagem da função f é o intervalo [0,1].
sen3x + senx
b) A função g é ímpar. 29. (AFA) O período da função real f definida por f(x) =
é igual a cos 3x + cos x
1
c) A função real h definida por h ( x ) =− + g ( x ) possui duas raízes a) 2π
2
π
no intervalo 0,  b) π
 2
π π
d) O período da função real j definida por j ( x ) =− 1 + g ( x ) é . c)
2 2 4
π
d)
sen2x 2
25. (AFA) Sejam f e g funções reais dadas por f ( x ) = e
cos x
g(x) = 2, cada uma definida no seu domínio mais amplo possível. 30. (AFA) Sobre a função real f definida por f ( x ) = – 1– | 6 ( senx )( cos x ) | ,
Analise as afirmações abaixo. é INCORRETO afirmar que:
I. O conjunto solução da equação f(x) = g(x) contém infinitos a) Im(f) = [–1,2].
elementos.  π 3π 
b) é decrescente para todo x ∈  ,  .
 3π 5π  4 4 
II. No intervalo  ,  , a função f é crescente.
4 4 c) possui 8 raízes no intervalo [0,2π].
III. O período da função f é p = π. d) tem período igual ao período da função real g dada por
g(x) = 2f(x).
Sobre as afirmações é correto afirmar que
a) apenas III é verdadeira.
 1
b) apenas I e II são verdadeiras. 31. (ESPCEX) = =
Sendo M arctg(X), N arctg   e P = tg(M – N), o
X
c) todas são falsas. valor de 30P para X = 15 é
d) apenas II e III são verdadeiras. 224 c) 45.
a) .
30 d) 224.
26. (AFA) Uma piscina com ondas artificiais foi programada de modo 45 e) 225.
b) .
que a altura da onda varie com o tempo de acordo com o modelo 6
 π πx   πx   πx    sen ( x )  π   π
=f ( x ) 3sen  +  sen   sen   em que y = f(x) é a altura da 32. (EFOMM) O gráfico da = função f ( x ) arc tg   −  ⋅ −x − 
2 4   4   2    cos ( x )  5   7
onda, em metros, e x o tempo, em minutos. Dentre   sen (que
x)  π  π
= f ( x )as alternativas
arc tg   −  ⋅ −x −  intercepta o eixo x nos pontos de coordenadas:
seguem, assinale a única cuja conclusão NÃO condiz com o modelo cos ( x )  5   7
proposto.
π d)  0, − π  e  0, π 
a) A altura de uma onda nunca atinge 2 metros. a)  − ,0  e  π ,0 
 7  5   7  5
b) Entre o momento de detecção de uma crista (altura máxima de
uma onda) e o de outra seguinte, passam-se 2 minutos. π
b)  − ,0  e  − π ,0  e)  0, π  e  0, − π 
c) De zero a 4 minutos, podem ser observadas mais de duas cristas.  7   5   7  5
d) As alturas das ondas observadas com 30, 90, 150, ... segundos
c) π  e  π 
são sempre iguais.  ,0   − ,0 
7   5 

417

PM_BOOK16 - MAT.indb 417 25/11/2022 19:22:10


FUNÇÕES TRIGONOMÉTRICAS E FUNÇÕES TRIGONOMÉTRICAS INVERSAS

33. (ESPCEX) Dentre as alternativas a seguir, aquela que apresenta uma EXERCÍCIOS DE
função trigonométrica de período 2π, cujo gráfico está representado
na figura abaixo é COMBATE
01. (EPCAR 2008) Resguardado seu respectivo domínio, o gráfico que
representa um período da função f definida por
 
sen   x   cos    x 
f x  2  é
 3 
2 cos   x   cot g    x 
 2 
a) f(x) = 1 − sen ( π − x). d) f(x) = 2 − sen ( π + x).
f(x) = 1 + cos ( π − x). a)
b) e) f(x) = 1 − cos ( π − x).
c) f(x) = 2 − cos ( π + x).

34. (AFA) Considere a função real sobrejetora f: A → B definida por


sen3x cos 3x
=
f(x) − . Sobre f é FALSO afirmar que
sen x cos x

 kπ 
a) O conjunto A é x ∈  | x ≠ ,k ∈ 
 2 
b) f é par.
c) f é injetora.
d) B = {2}
b)
35. (EN) Sejam A a matriz quadrada de ordem 2 definida por
  π 
2cos  2x −  cos(x + π) 
A=  2  e f a função real de variável real tal
 
 cos x 1 
que f(x) = |A + AT|, onde AT representa a matriz transposta de A. O
gráfico que melhor representa a função y = f(x) no intervalo – π ≤ x
≤πé
a)

b)
c)

c)

d)
d)

e)

418

PM_BOOK16 - MAT.indb 418 25/11/2022 19:22:12


FUNÇÕES TRIGONOMÉTRICAS E FUNÇÕES TRIGONOMÉTRICAS INVERSAS

02. Assinale a alternativa cujo gráfico pode representar a função a) o período chuvoso corresponde a seis meses do ano.
  b) a população atinge seu máximo em t = 6.
f( x )  2 cos  2x    1
 4 c) o período de seca corresponde a 4 meses do ano.
a) d) d) a população média anual é de 6.000 animais.
e) a população atinge seu mínimo em t = 4 com 6.000 animais.

07. (ESPCEX) A função real f(x) está representada no gráfico abaixo.

b) e)
-|senx|,sex<0
a) f(x)= 
 |cosx|,sex³0
|cosx|,sex<0
b) f(x)= 
 |senx|,sex³0
-|cosx|,sex<0
c) f(x)= 
A expressão algébrica de f(x) é  |senx|,sex³0
-|senx|,sex<0 |senx|,sex<0
a) f(x)=  d) f(x)= 
 |cosx|,sex³0  |cosx|,sex³0
c) |cosx|,sex<0
b) f(x)=  -senx,sex<0
e) f(x)= 
 |senx|,sex³0  cosx,sex³0
-|cosx|,sex<0
c) f(x)= 
 |senx|,sex³0
|senx|,sex<0
d) f(x)=
08. (ESPCEX)
 As funções y = sen x e y = cos x estão representadas
 |cosx|,sex³0
no gráfico abaixo. Então, a medida da área do triângulo retângulo
-senx,sex<0
definidopelos segmentos retilíneos AB, BC e AC é:
e) f(x)= 
 cosx,sex³0
03. (ITA 2000) Considere f:  →  definida por
 x 
f  x   2 sen 3x  cos   . Sobre f podemos afirmar que:
 2 
a) é uma função par.
b) é uma função ímpar e periódica de período fundamental 4π.
c) é uma função ímpar e periódica de período fundamental 4 π 3 .
d) é uma função periódica de período fundamental 2π. π
e) não é par, não é ímpar e não é periódica.
a)
8
(2− 2 )
π
b)
04. (EN 2014) Considerando que a função f(x) = cos x, 0 < x < π, é 8
π
inversível, o valor de tg  arccos  é
2 c)
16
(
2− 2 )
 5 π
(
a) 2 − 2
8
) d)
π 2
21 21 21 16
a) − c) − e) π
π
4
5 2 2 b)
8 e)
16
(
1− 2 )
21 π
b) −
25
d)
25
c)
16
(
2− 2 )
π 2
d)
05. Qual o valor de arcsen(sen 2000°)? 09.16 (EPCAR 3º ANO) Os valores reais de x que satisfazem a equação
 π  1+ x   1+ −x  π
a) 2000°
c)
17π e) 
9
e) (
16  2 
)
arctg 1− 2  + arctg 
 2  4
= são números
100π 9
b) π
9 d) a) simétricos.
9
b) cuja soma é 1.
06. (ESPCEX 2015) A população de peixes em uma lagoa varia c) primos.
conforme o regime de chuvas da região. Ela cresce no período
chuvoso e decresce no período de estiagem. Esta população é descrita d) recíprocos.
  t 2  
pela expressão P  t   103  cos       5  em que o tempo t é
  6   
medido em meses. É correto afirmar que

419

PM_BOOK16 - MAT.indb 419 25/11/2022 19:22:26


FUNÇÕES TRIGONOMÉTRICAS E FUNÇÕES TRIGONOMÉTRICAS INVERSAS

π 
sen(− x).cos + x 
10. Simplificando a expressão 2  obtêm-se uma a) admite infinitas soluções, todas positivas.
tg(2π − x).cos(π − x) b) admite uma única solução, e esta é positiva.
nova expressão E. Seja f:  →  definida por f(x) = E, o gráfico que  5 3
melhor representa f é c) admite três soluções que se encontram no intervalo  − ,  .
 2 2
d) admite apenas soluções negativas.
a) y e) não admite solução.

1
3 S=

(ITA) Seja
 e− x − e x   ex − e− x  π 
 x ∈  | arc sen   + arccos  = .
  3 2 x   2   2  2 
2 2
Então
–1
a) S = ∅ d) S = +
b) S = {0} e) S = 
b) y c) S = + \ {0}

1  π π
4 (ITA) Seja f : [−1, 1] →  − , 
 2 2
a função definida por
4

2
 3
2
2 x f(x) = arcsen (x). Então, a soma ∑ f  cos 23π  é igual a:
n=0
n

–1
253 d) − 82 π.
a) π.
162 81
245 79
c) y b) π. e) − π.
162 162
1
152
c) − π.
 81
 3 2
5
2
x (IME) A menor raiz real positiva da equação
2
–1    3  2π
arctg  x ⋅ tg  arcsen     = encontra-se no intervalo:
   5    x +2

a) (0,1] d) (3,4]
b) (1,2] e) (4,5]
y
d) c) (2,3]
1
GABARITO
  3
2 EXERCÍCIOS DE FIXAÇÃO
2 2 x
01. B 04. C 07. C 10. D
–1
02. A 05. A 08. E
03. A 06. C 09. A
EXERCÍCIOS DE TREINAMENTO
01. D 10. D 19. C 28. A

DESAFIO PRO 02. D


03. A
11. A
12. C
20. A
21. A
29. D
30. B
04. C 13. A 22. D 31. D
1 3
1 (ITA) O maior valor de tgx, com x = arcsen   e 05. E 14. A 23. D 32. A
2 5
 π 06. A 15. A 24. C 33. E
x ∈ 0,  , é
 2 07. B 16. D 25. A 34. C
1 d) 2.
a) . 08. C 17. A 26. C 35. D
4
e) 3. 09. C 18. B 27. A
1
b) . EXERCÍCIOS DE COMBATE
3
01. D 04. E 07. A 10. A
1
c) . 02. D 05. E 08. C
2
03. B 06. A 09. A

2 (ITA) A equação em x, arctg (ex + 2) – arccotg


 ex  π
= ,x ∈R\{0}
DESAFIO PRO
01. B 03. B 05. D
 e2x − 1 4 02. B 04. B

420

PM_BOOK16 - MAT.indb 420 25/11/2022 19:22:28


EQUAÇÕES E INEQUAÇÕES
TRIGONOMÉTRICAS

EQUAÇÕES TRIGONOMÉTRICAS A identificação das soluções da equação no ciclo trigonométrico


encontra-se na figura seguinte.
BÁSICAS
Vamos mostrar como resolver equações trigonométricas básicas,
onde temos uma linha trigonométrica aplicada sobre uma função e
igual a um determinado valor. Um exemplo desse tipo de equação é
1
sen2x = . Normalmente, mesmo as equações trigonométricas mais
2
complexas, terminam com a resolução de uma equação dessa forma.

EQUAÇÃO EM SENO
Seja a ∈  tal que |a| ≤ 1, então
sen   a    arc sen a  2k, k     
   arc sen a  2k, k  
k
Notação resumida:   k   1  arc sen a, k  
Observe que representamos a solução da equação utilizando Exemplo:
a função arco seno por se tratar de um caso geral. Na maioria 1  
dos problemas são apresentados ângulos cujos valores das linhas cos 2x   2x  2k  , k    x  k  , k  
2 3 6
trigonométricas são conhecidos.
Ao resolver uma equação trigonométrica, é sempre útil identificar Se, na equação cos α = a, o valor de a for tal que |a| > 1, então o
as soluções no ciclo trigonométrico, como na figura seguinte. conjunto solução da equação é vazio.
Exemplo:
cos x = 2 ⇔ S = ∅

EQUAÇÃO EM TANGENTE
Seja a ∈ , então
tg   a    arc tg a  k, k  
Observe que a equação em tangente possui solução para qualquer
valor de a real.
A identificação das soluções da equação no ciclo trigonométrico
encontra-se na figura seguinte.

Exemplo:
1 k 
sen 2x   2x  k   1  , k   
2 6
k  k 
x  1  , k  
2 12
Se, na equação sen α = a, o valor de a for tal que |a| > 1, então o
conjunto solução da equação é vazio.
Exemplo:
sen x = 2 ⇔ S = ∅

EQUAÇÃO EM COSSENO Exemplo:


Seja a ∈  tal que |a| ≤ 1, então
  k
cos   a    2k  arccos a, k   tg2x  1  2x   k, k    x   , k  
4 8 2

421

PM_BOOK16 - MAT.indb 421 25/11/2022 19:22:34


EQUAÇÕES E INEQUAÇÕES TRIGONOMÉTRICAS

EQUAÇÕES COM IGUALDADE DE Exemplo:

LINHAS TRIGONOMÉTRICAS  
3x   2x    2k, k   
 3
 2k
EQUAÇÃO COM IGUALDADE DE SENOS x  ,k
  15 5
sen   sen   cos 3x  cos  2x   
 3  
     2k, k          2k, k   3x   2x    2k, k   
 3
Observe que dois arcos que possuem o mesmo seno ou são 
côngruos      2k, k    ou suas imagens são simétricas em  x   2k, k  
3
relação ao eixo Oy.
No segundo caso, os ângulos seriam dados, sem perda de
generalidade, por     2k1, k1   e         2k 2, k 2  , o Exercício Resolvido
que implica       2 k1  k 2      2k, k  . 01. (ITA 1993) O conjunto das soluções da equação sen 5x = cos 3x
Outra maneira de resolver essa equação é usando as fórmulas contém o seguinte conjunto:

 
de Werner:
 
sen   sen   sen   sen   0  a) k ,k
16 5
  
 2 sen 
 2 

 cos 
 2 
0 b)  
16

k ,k
3 
 
     
 sen    0  cos  0
 2   2  c) k ,k
4 3
  
sen 
 2 
0
 
2
 k, k        2k, k   d)  
4

k ,k
2 

cos 
 2 
0
 
2
  k, k   
2
e)  
4
 2k, k   
       2k, k  
Resolução: E
k
Notação resumida:   k   1  , k    
sen 5x  cos 3x  cos   5x   cos 3x 
2 
Exemplo:
    k
     5x   3x  2k  x   ,
2x   x    2k, k    2  16 4
 4 
 k    x   k, k  
 x    2k, k   4
 
 
4
sen 2x  sen  x     k 
 4   S  x|x   , k    x   k, k   
2x   x      2k, k    16 4 4
 4
x
5 2k
12

3
,k
  
4
 2k, k   
EQUAÇÃO COM IGUALDADE DE COSSENOS
EQUAÇÃO COM IGUALDADE DE TANGENTES
cos   cos       2k, k  
tg   tg       k, k  
Observe que dois arcos que possuem o mesmo cosseno ou são
Observe que dois arcos que possuem o mesmo cosseno ou são
côngruos      2k, k    ou suas imagens são simétricas em
côngruos      2k, k    ou suas imagens são simétricas em
relação ao eixo Ox.
relação à origem dos eixos ordenados.
No segundo caso, os ângulos seriam dados, sem perda de
No segundo caso, os ângulos seriam dados, sem perda de
generalidade, por     2k1, k1   e     2k 2, k 2   , o que
generalidade, por         2k1, k1   e     2k 2, k 2   , o
implica     2 k1  k 2    2k, k   .
que implica       2 k1  k 2      2k, .k  
A notação     2k, k   representa a união das soluções de
ambos os casos. A notação     k, k   representa a união das soluções de
ambos os casos.
Outra maneira de resolver essa equação é usando as fórmulas
de Werner: Outra maneira de resolver essa equação é usando as fórmulas
cos   cos   cos   cos   0  de Werner:
sen     
    tg  tg  tg  tg  0  0
 2 sen    sen  0 cos   cos 
 2   2 
     sen       0      k, k  
 sen    0  sen  0
 2   2  Exemplo:
     
sen  0  k, k        2k, k   tg  4 x    tg2x   4 x    2x  k, k   
 2  2  4  4
      k
 x   ,k
sen  0  k, k        2k, k   8 2
 2  2

422

PM_BOOK16 - MAT.indb 422 25/11/2022 19:22:56


EQUAÇÕES E INEQUAÇÕES TRIGONOMÉTRICAS

OUTRAS EQUAÇÕES
TRIGONOMÉTRICAS
EQUAÇÃO DO TIPO a senx + b cosx = c
Seja a equação a senx + b cos x = c, onde a · b ≠ 0, temos:
a b c
a sen x  b cos x  c  sen x  cos x 
a2  b2 a2  b2 a2  b2
a b
Seja o ângulo θ tal que sen   e cos   ,
a2  b2 a2  b2
c c
então sen  sen x  cos  cos x   cos  x     .
2 2
a b a  b2
2

Os valores que satisfazem a inequação são aqueles cujo seno


c  c 
Se 1   1, então x    arccos   2k, k  . 1     5 
2 2 2 2  é menor ou igual a , ou seja, S  0,    , 2  .
a b  a b  2  6  6 

EQUAÇÃO DO TIPO a sen²x + b senx cosx +


c cos²x = d EXERCÍCIOS DE

FIXAÇÃO
Seja a equação a sen²x + b senx cosx + c cos²x = d, onde abc ≠
0, temos:
a sen2 x  b sen x cos x  c cos2 x  d   cos2 x  
 a tg2 x  b tg x  c  d sec2 x
3
 a tg2 x  b tg x  c  d 1 tg2 x   01. Se senx = e 0 ≤ x ≤ 2π , então a soma dos valores possíveis
para x é 2
  a  g tg2 x  b tg x   c  d  0
Observe que ao dividir a equação original por cos²x, obtivemos π 3π π
a) c) e)
uma nova equação em tgx. Se a equação em tgx possuir raízes reais, 2 2 3
então a expressão reduz-se a duas equações da forma tgx = p.
b) π d) 2π

EQUAÇÃO DO TIPO a(senx + cosx) + b senx 02. (EEAR) No ciclo trigonométrico, a igualdade sen(πx) = 0 é
cosx = c verdadeira se e somente se x é um número
Vamos efetuar uma substituição de variável para resolver essa a) real qualquer. c) imaginário.
equação. b) inteiro. d) irracional.
2
sen x  cos x  z   sen x  cos x   z2 
03. Se sen x + cos 2x = 1, então um dos valores de sen x é
z2  1
 1 2 sen x cos x  z2  sen x cos x 
2 a) 1 2 1
c) e) −
Substituindo esses valores na equação original, temos: 2 2
1 − 3
a  sen x  cos x   b sen x cos x  c  b) d)
2
2 3
z 1
 az  b   c  bz2  2az  b  2c   0  π
2 04. Os valores de x que satisfazem a equação cos  3x −  =
0 , são
 5
Basta agora resolver a equação em z e retornar a substituição,
observando que  2  sen x  cos x  2 (vide a solução da equação 7π π
a) x= + k ; k = 0, ± 1, ± 2,...
do item 3.1). 30 3

7π π
INEQUAÇÕES TRIGONOMÉTRICAS b) x=
15
+ k ; k = 0, ± 1, ± 2,...
3
Para simplificar uma inequação trigonométrica são utilizadas as
mesmas técnicas utilizadas nas equações trigonométricas, obtendo- 7π π
c) x= + k ; k = 0, ± 1, ± 2,...
se ao final uma inequação básica. Para resolver essa inequação, 2 4
basta marcar as soluções da “equação” no ciclo trigonométrico e,
7π π
posteriormente, identificar os intervalos que satisfazem a inequação. d) x= + k ; k = 0, ± 1, ± 2,...
5 2
Exercício Resolvido
7π π
e) x= + k ; k = 0, ± 1, ± 2,...
1 4 6
01. Resolva a inequação sen x ≤ em [0,2π].
2 05. O número de raízes da equação cos x + sen x = 0 no intervalo
Resolução: [ π,3π ] é:
Inicialmente, vamos marcar no ciclo trigonométrico as raízes de a) 2 d) 4
1 π 5π b) 1 e) 0
sen x = que são e .
2 6 6
c) 3

423

PM_BOOK16 - MAT.indb 423 25/11/2022 19:23:08


EQUAÇÕES E INEQUAÇÕES TRIGONOMÉTRICAS

π 1
06. Se θ é um ângulo tal que 0 < θ < e o dobro do seu seno é igual 04. (ESPCEX) O domínio e a imagem da função f(x) = são,
2 respectivamente, 5 − senx
ao triplo do quadrado da sua tangente, então o valor do seu cosseno é
a)  − {5} e [−1,1]  1 1
3 3 e) 1 d) * e  , 
a) c)  1 1 6 3
3 2 b)  e − , 
 5 4  1
2 d) 2 e)  − {5} e  −1, 
b)  3
3  1 1
2 c)  e , 
6 4 
07. No intervalo [0,π], a soma das raízes da equação 3cos2x –
7sen2x + 2 = 0 é igual a 05. (ESPCEX) O conjunto de todos os valores de x em [0,2π], em que
1
a) 4π c) 2π a função f(x) = está definida, é
tgx − 1
b) 3π d) π
 π   3π   π π   5π 
a)  0, 2    π, 2  d)  4 , 2    π, 4 
08. A soma das raízes da equação 1 – 4xos2x = 0, compreendias entre
0 e π é:
 π   3π   π   5π 
π 3π 7π
b) 0, 2    π, 2  e)  0, 4    π, 4 
a) c) e)
3 4 6
 π   5π 
b) π d)
5π c) 0, 4    π, 4 
6
06. (ESPCEX) Na figura abaixo estão representados os gráficos das
09. A solução da inequação sen2x < 2senx, no intervalo fechado [0,2π] é: funções reais f(x) = cos x e g(x) = log x.

a) 0 < x < 2π c) 0<x<π e) π

3π π
b) π<x< d) 0<x<
2 2
 19π 
10. (ESPCEX) O número de arcos no intervalo 0, cujo valor do
 6 
1
cosseno é igual a é
2
a) 1 d) 4
b) 2 e) 5
c) 3

EXERCÍCIOS DE

TREINAMENTO
O valor de x que satisfaz a equação log x = cos x está entre
a) 0e1 c) 1,6 e 2,4 e) 3,2 e 4,0
01. (EEAR) Uma das raízes da equação x – (2tg a)x – 1 = 0 é, sendo
2

π b) 1 e 1,6 d) 2,4 e 3,2


a ≠ − + kπ, k ∈  ,
2
a) tg a + cossec a c) tg a + sen a. 07. (AFA) A soma das soluções da equação tg2x + sen2 x = 3 cos2x,
b) tg a – cos a. d) tg a – sec a. pertencentes ao intervalo fechado [0,2π] é:
a) π b) 2π c) 4π d) 6π
2
625cos x
02. (ESPCEX) A soma das soluções da equação = 1 , para
π 25cosx 08. (AFA) A solução geral da equação sen(3x + 40o) = cos(x – 20o) é,
0≤x≤ é: para k ∈ :
2
a) x = k ⋅ 360º – 70º
π π 5π b) x = k ⋅ 180º + 35º
a) c) e)
6 2 6
c) x = k ⋅ 180º + 15º ou x = k ⋅ 90º + 17º 30’
π 2π d) x = k ⋅ 360º – 70º ou x = k ⋅ 180º + 35º
b) d)
3 3
09. (AFA) As soluções de sen 3x + sen 5x = cos 2x – cos 6x, são para
03. (ESPCEX) O número de soluções da equação sen4 x + cos 4 x =
1, k ∈ , da forma:
satisfazendo a condição 0 ≤ x < 2π , é kπ k π kπ 5π
a) x= ou x = kπ + ( −1) ⋅ c) x= ou x= 2kπ +
a) infinito d) 1 4 6 2 6
b) 4 e) 0 π π π
b) x = kπ + ou x = 2kπ + d) x= 2kπ +
c) 2 2 6 6

424

PM_BOOK16 - MAT.indb 424 25/11/2022 19:23:10


EQUAÇÕES E INEQUAÇÕES TRIGONOMÉTRICAS

2x −=1 3senθ


10. (ESPCEX) Pode-se afirmar que o sistema  , 17. (EPCAR 3° ANO 2003) Se 0 ≤ x ≤ 5π e senx + 3 cos x = α,
x ∈  e 0 ≤ θ < 2π ,  x − 2= cos θ α ∈  então a SOMA dos valores de x para que α seja máximo é
a) possui apenas um par ordenado (x,θ) como solução. π 13π 13π 25π
a) b) c) d)
b) possui dois pares ordenados (x,θ) como solução. 6 2 6 6
c) possui dois pares ordenados (x,θ) como solução.
1 + tg x
d) possui infinitas soluções. 18. A solução da equação = 1 + sen2x , onde k ∈ , é:
1 − tg x
e) não possui solução.
π
a) x = kπ d) x = kπ −
4
x =1 − sen 2θ
11. (MACKENZIE) O valor de θ que satisfaz o sistema  ,
2x =2 + cos θ kπ
para x e θ reais, com 0 ≤ θ ≤ π é b) x= 2kπ e) x=
2
a) 0 c) π π
e)
π π 3 π
b) d) c) x = kπ − ou x = kπ
2 4 4
1 2
12. (AFA) A solução da inequação ≤ cos x.senx ≤ , onde x ∈ [ 0, π ] 19. (AFA) Sendo x ∈ [ 0, 2π ] , a interpretação gráfica no
4 2
ciclo trigonométrico para o conjunto solução da inequação
 π π  5π 5π  − 8sen4 x + 10sen2x − 3 < 0 é dada por
a) S=  ,  c) S=  , 
12 6   12 6 
a)
 π 5π   π 5π 
b) S= ,  d) S=  , 
 12 3  12 12 

2sen2x + sen2x π
13. A solução da inequação 0 < < 1 para x ∈ 0, 
1 + tgx  2
é o conjunto
 π   π e) π π 
a)  0, 4  . c)  0, 2  .  4 , 2  .

 π d)  π . b)
b)  0, 4  .  0, 2 

1 log103
14. (ESPCEX) Seja β= ⋅ . O conjunto solução da
2 log103 − log107
β
 3
desigualdade 3cos ( x ) ≤   no intervalo [ 0,2π ) , é igual a
 7

 π π   3π 
a) 0, 3  . c)  3 ,2π  . e)  2 ,2π . c)

 π 5π  π 
b)  3 , 3  . d)  3 ,2π .

15. (ESPCEX) A soma de todas as soluções da equação

2cos (x) − cos2 (x) − 2cos(x) + 1 =0, que estão contidas no intervalo
3

[0,2π], é igual a
a) 2π. d) 5π.
b) 3π. e) 6π. d)
c) 4π.

16. (AFA) Seja a equação trigonométrica tg3x − 2 tg2x − tgx + 2 =0,



com x ∈  [0, 2π[− ,
 { }
π 3π 
2 2 
. Sobre a quantidade de elementos
distintos do conjunto solução dessa equação, é correto afirmar que
são, exatamente,
a) três. c) cinco.
b) quatro. d) seis.

425

PM_BOOK16 - MAT.indb 425 25/11/2022 19:23:14


EQUAÇÕES E INEQUAÇÕES TRIGONOMÉTRICAS

20. A área do polígono, cujos vértices são a representação EXERCÍCIOS DE


no ciclo trigonométrico das soluções
( senx + sen2x + sen3x )2 + ( cos x + cos 2x + cos 3x )2 =
da
1 , é:
equação
COMBATE
1 3
a) d)
2 2
01. (ESPCEX 2015) 18) A soma de todas as soluções da equação
b) 1 3 3
e) 2 cos3  x   cos2  x   2 cos  x   1  0 , que estão contidas no intervalo
c) 2 4 [0,2π], é igual a
 1 a) 2π d) 5π
21. (EN 2003) O número de soluções reais da equação sen  = x − 2
 x
é igual a n; assim, pode-se concluir que: b) 3π e) 6π
a) n = 0 d) n = 3 c) 4π
b) n = 1 e) n > 3
c) n=2 5
02. (EN 2006) No intervalo [0,π] a equação sen4 x  cos 4 x 
8
possui soma dos inversos das raízes igual a:
22. (EN 2004) O número de soluções da equação
15 15 117
(1 − cos x )2 (1 − cos x )3 a) c) e)
(1 − cos x ) + + + =
2 para x real, quando 2π π 5π
2 4 d) 2π
117
0 ≤ x ≤ 4π é b)
10π
a) 1 d) 4
b) 2 e) 6 03. (EN 2011) Sejam A e B conjuntos de números reais tais que seus
elementos constituem, respectivamente, o domínio da função
c) 3
1 2 sen x
f x  no universo [0,2π] e o conjunto solução da
23. (EN 2005) Os pontos A = (x1, y1) e B = (x 2 , y 2 ) são soluções do 1 2 sen x
1 1 
sen (x + y) + sen (x − y) =
2 inequação   0 para 0 < x < π, com x  . Pode-se
sistema de equações  onde x ∈[0, 2π]. cossec x sec x 2
sen x + cos y = 2 afirmar que B – A é igual a
A distância desde A até B é:      5 11       7 11 
π c) π e) 3π
a)  6 , 4    4 , 6  d)  6 , 4    6 , 6 
a)
2
d) 2π  5 7   5 
b)
3
π
b)  6 , 6  e)  6 ,  
2
c) ∅
24. (EN 2007) O conjunto de todos os valores de θ ∈ [ 0, π ] que
 2 3
04. (EFOMM 2010) Considere a equação de incógnita real x:
x + x + tgθ > 4
satisfazem ao sistema  , ∀x ∈ , é 2 cos 4 x  2 cos2 x  1  cos 4 x . Se x 0   0;   é uma de suas
 1 + 1 >1 soluções e x0 centímetros é a medida da diagonal de um cubo, então
 ln θ 1 − ln θ
a área da superfície total desse cubo, em cm², é igual a:
a) ]1, π[
3 2
π π a) π
b) , 8
 4 2 
1 2
b) π
c)
 π 2
1,
 2 
c) 6
d) π  27 2
 2 ,e  d) π
8
e) ]e, π[ e) 6π2


25. (EN 2009) O termo de mais alto grau da equação biquadrada  2
B(x) = 0 tem coeficiente igual a 1. Sabe-se que duas das raízes dessa 05. (AFA 2000) A inequação 2
senx
   , com x  0, 2 e
 3
equação são, respectivamente, o termo central do desenvolvimento log 2
6  , tem como solução os valores de x pertencentes a
de 
1

1
e a quantidade de soluções da equação log 2  log 3
 2 
5
sen2x − 6senx cos x + 8cos2 x =
0 no intervalo [0,2π]. Pode-se afirmar
a) 0,  3  2 3 , 2
que a soma dos coeficientes de B(x) vale b) 0,  2  3 2 , 2
a) –9 d) 7 c) 0,  6  5 6 , 2
b) –6 e) 12
d) 0, 4  3  5 6 , 2
c) 3

426

PM_BOOK16 - MAT.indb 426 25/11/2022 19:23:31


EQUAÇÕES E INEQUAÇÕES TRIGONOMÉTRICAS

06. (ESPCEX) O valor de cos x + sen x, sabendo que 3 ⋅ sen x + 4.cos 10. (EN 2008) Sejam f e g funções reais definidas por
x = 5, é f  x   2sen2x  6 cos x e g  x   k  cos 2x , k ∈ . Se
a) 3/5   7  19
f    g   , então a soma das soluções da equação
b) 4/5  3  4  2
c) 1/4  21 16 
f(x) = g(x) no intervalo  , é
d) 1/6  11 5 
13π
e) 1/8 a)
6
07. (ESPCEX) A soma de todas as soluções da equação 13π
2 cos³(x) – cos²(x) – 2 cos(x) + 1 = 0, que estão contidas no intervalo b)
3
[0,2], é igual a

a) 2π c)
3
b) 3π
25π
c) 4π d)
6
d) 5π
e) 6π
16π
e)
3
08. (AFA 2000) Os valores de m ∈  para os quais a equação
2  sen x  cos x   m2  2 admite soluções, são
a) –1 ≤ m ≤ 1
b) –2 ≤ m ≤ 2 DESAFIO PRO
c) 0 ≤ m ≤ √2

1
d) –√2 ≤ m ≤ √2 (IME) Seja arcsen x + arcsen y + arcsenz = , onde x, y
2
e z são números reais pertencentes ao intervalo [–1,1].
09. (AFA 2010) Seja a função real f definida por
9
  Determine o valor de x100 + y100 + z100 − 101 .
f  x   cos  4 x   sen   6x  . Marque a alternativa que possui x + y101 + z101
2  a) –2
a melhor representação, no ciclo trigonométrico, de todas as raízes b) –1
da função f. c) 0
a) c) d) 1
e) 2
sen sen
2
2 (ITA) Seja x ∈ [ 0,2π ] tal que sen(x) cos(x) =
5
. Então, o
produto e a soma de todos os possíveis valores de tg(x)
são, respectivamente.
0 cos 0 cos a) 1 e 0.
5
b) 1 e .
2
c) – 1 e 0.
d) 1 e 5.
5
b) d) e) −1 e - .
2

3
sen sen (ITA) Determine os valores reais de x de modo que
sen(2x) − 3 cos(2x) seja máximo.

0 cos 0 cos 4 Resolva a equação tg2 (2x) + 2tg(2x)tg(3x) − 1 =0

5 Sejam a, b e c números reais, todos diferentes de –1 e 1, tais


que a + b + c = abc. Prove que:
a b c 4abc
+ + =2
1 − a2 1 − b2 1 − c2 ( )( )(
1 − a 1 − b2 1 − c 2 )

427

PM_BOOK16 - MAT.indb 427 25/11/2022 19:23:38


EQUAÇÕES E INEQUAÇÕES TRIGONOMÉTRICAS

GABARITO
EXERCÍCIOS DE FIXAÇÃO
01. B 04. A 07. D 10. C
02. B 05. A 08. B
03. B 06. C 09. D
EXERCÍCIOS DE TREINAMENTO
01. D 08. C 15. D 22. D
02. E 09. A 16. D 23. D
03. B 10. B 17. B 24. C
04. C 11. B 18. C 25. A
05. D 12. D 19. B
06. B 13. A 20. B
07. C 14. B 21. B
EXERCÍCIOS DE COMBATE
01. D 04. B 07. B 10. B
02. B 05. D 08. B
03. E 06. E 09. A
DESAFIO PRO
01. C
02. B
1 3   π
03. sen(2x) − 3 cos(2x)
=  sen(2x) − =
cos(2x) ⋅ 2 2.sen  2x − 3 
2 2 
2kπ π 2kπ π
04. x = ± ou x = ±
7 14 3 6

05. a = tgx, b = tgy e c = tgz onde x,y,y ≠ , para k inteiro.
4
(a + b + c) = abc se transforma em tg (x + y + z) = 0.
2tg(x + y + z)
tg(2x + 2y + 2z) = =0 ⇒
1 − tg2 (x + y + z)
⇒ tg(2x) + tg(2y) + tg(2z) = tg(2x).tg(2y).tg(2z)

2x 2y 2z 2x 2y 2z
Daí + + =. .
1 − tg2x 1 − tg2y 1 − tg2z 1 − tg2x 1 − tg2y 1 − tg2z

ANOTAÇÕES

428

PM_BOOK16 - MAT.indb 428 25/11/2022 19:23:39


FUNDAMENTOS DE
GEOMETRIA ESPACIAL

FUNDAMENTOS D. Duas retas paralelas distintas.


Conceitos primitivos: ponto, reta e plano.

POSTULADOS PRINCIPAIS
• Dois pontos distintos determinam uma única reta que passa
por eles.
• Três pontos não colineares determinam um único plano que
passa por eles.
• Se dois pontos distintos de uma reta pertencem a um plano,
então essa reta está contida nesse plano.
• Se dois planos possuem um ponto comum, então possuem pelo três pontos não colineares
menos algum outro ponto comum. Isso indica que a interseção
de dois planos distintos que se interceptam é uma reta. uma reta e um ponto fora dela
Determinação de um plano
• Por um ponto não pertencente a uma reta, passa uma, e duas retas concorrentes
apenas uma, reta paralela à primeira. (Euclides)
duas retas paralelas distintas
DETERMINAÇÃO DO PLANO
Um único plano fica determinado por:
POSIÇÕES RELATIVAS ENTRE RETAS
A. Três pontos não colineares.
RETAS COPLANARES
B A. Concorrentes: um ponto de interseção.

A C

B. Uma reta e um ponto exterior.

B. Paralelas Coincidentes: infinitos pontos de interseção.


r
A r≡s

C. Duas retas concorrentes.

C. Paralelas Distintas: não há pontos de interseção.

429

PM_BOOK16 - MAT.indb 429 25/11/2022 19:23:40


FUNDAMENTOS DE GEOMETRIA ESPACIAL

RETAS NÃO COPLANARES


POSIÇÕES RELATIVAS ENTRE UMA RETA E UM PLANO
RETAS REVERSAS: não há pontos de interseção.
todos os pontos da reta pertencem ao
contida
plano

paralela não têm ponto em comum

concorrente (ou
têm somente um ponto em comum
secante)
α ≡β

POSIÇÕES RELATIVAS ENTRE PLANOS

POSIÇÕES RELATIVAS ENTRE DOIS PLANOS

POSIÇÕES RELATIVAS ENTRE


INTERSEÇÃO
DOIS PLANOS
POSIÇÕES RELATIVAS ENTRE DUAS RETAS INTERSEÇÃO
coincidentes todo o plano
concorrentes 1 ponto paralelos
distintos vazia
coplanares coincidentes toda a reta
paralelas secantes (ou concorrentes) uma única reta (traço)
distintas vazia

não coplanares reversas vazia A. Planos paralelos coincidentes: a interseção é todo o plano.
α≡β

POSIÇÕES RELATIVAS ENTRE RETA E PLANO


A. Reta e plano contidos: infinitos pontos de interseção.

B. Planos paralelos distintos: não há pontos de interseção.


αβ
B. Reta e plano paralelos: não há pontos de interseção.

C. Reta e plano secantes: um único ponto de interseção.

430

PM_BOOK16 - MAT.indb 430 25/11/2022 19:23:41


FUNDAMENTOS DE GEOMETRIA ESPACIAL

C. Planos secantes (ou concorrentes): a interseção é uma


reta.

    r

r, s   

r e s concorrentes     
res 


• Se dois planos são secantes e uma reta de um deles é paralela


ao outro, então essa reta é paralela à interseção.

PARALELISMO
• Se duas retas são paralelas a uma terceira, então elas são
paralelas entre si.
• Uma reta é paralela a um plano se, e somente se, eles não têm
ponto em comum.
TEOREMA: a condição necessária e suficiente para que uma reta
não contida em um plano seja paralela a esse plano é que ela seja
paralela a uma reta do plano.

 e  secantes 

s    sr
s 


• Se duas retas distintas são paralelas entre si e um plano


paralelo à primeira contém um ponto da segunda, então esse
plano contém a segunda.
Dois planos são paralelos se, e somente se, não têm ponto em • Se uma reta é paralela a dois planos secantes, então ela é
comum ou são coincidentes. paralela à interseção desses planos.
• Por um ponto fora de um plano passa um único plano paralelo
a esse plano.
TEOREMA: a condição necessária e suficiente para que dois planos
sejam paralelos é que um deles contenha duas retas concorrentes,
paralelas ao outro.

431

PM_BOOK16 - MAT.indb 431 25/11/2022 19:23:43


FUNDAMENTOS DE GEOMETRIA ESPACIAL

PERPENDICULARIDADE
RETAS ORTOGONAIS
Ângulo entre retas reversas é o ângulo formado por duas retas
concorrentes paralelas às retas dadas.

Retas ortogonais são retas reversas que formam ângulo reto.

 e  secantes 
  sr
s e s 

• Se dois planos são paralelos a um terceiro, então eles são


paralelos entre si.
• Se dois planos paralelos entre si são interceptados por um
terceiro, então as interseções são paralelas entre si.

RETA E PLANO PERPENDICULARES


Uma reta e um plano são perpendiculares se, e somente se, a reta
é perpendicular ou ortogonal a todas as retas do plano.

 

    r  r  s
    s 

Se uma reta e um plano são perpendiculares, o traço (P) da reta


no plano é o pé da perpendicular.
• Se uma reta é perpendicular a um plano, então ela é
perpendicular ou ortogonal a qualquer reta do plano.

432

PM_BOOK16 - MAT.indb 432 25/11/2022 19:23:45


FUNDAMENTOS DE GEOMETRIA ESPACIAL

• Uma reta é perpendicular a um plano se, e somente se, é • Dois planos secantes são perpendiculares se, e somente
perpendicular (ou ortogonal) a duas retas concorrentes se, toda reta de um deles, perpendicular à interseção, é
desse plano. perpendicular ao outro.
• Se uma reta é perpendicular a um plano, qualquer outro
PLANOS PERPENDICULARES plano que a contenha é perpendicular ao primeiro.
• Um plano α é perpendicular a um plano β se, e somente se, α • Se um plano é perpendicular a dois planos secantes, então ele
contém uma reta perpendicular a β. é perpendicular à interseção desses planos.

  
    r   
  

r  

r  

• Por uma reta r não perpendicular a um plano α, existe um


único plano β perpendicular a α. DISTÂNCIAS
• Se dois planos são perpendiculares entre si e uma reta de um
deles é perpendicular à interseção dos planos, então essa reta Distância entre dois
é perpendicular ao outro plano. segmento de reta AB
pontos A e B

Distância entre um distância do ponto ao pé da perpendicular


ponto e uma reta à reta conduzida pelo ponto

Distância entre distância entre um ponto qualquer de


duas retas paralelas uma das retas e a outra reta

distância entre o ponto e o pé da


Distância entre
perpendicular ao plano conduzida
ponto e plano
pelo ponto

Distância entre reta distância entre um ponto qualquer da


e planos paralelos reta e o plano

Distância entre distância entre um ponto qualquer de


planos paralelos um deles e o outro plano

distância entre um ponto qualquer de


Distância entre
uma delas e o plano que passa pela
duas retas reversas
outra e é paralelo à primeira

PERPENDICULAR COMUM: dadas duas retas reversas, existe


uma única reta que é perpendicular comum a essas retas. De todos os
segmentos que têm extremidades em cada uma das retas reversas, o
  menor é o da perpendicular comum.
r 
r  s    

433

PM_BOOK16 - MAT.indb 433 25/11/2022 19:23:47


FUNDAMENTOS DE GEOMETRIA ESPACIAL

EXERCÍCIOS DE

FIXAÇÃO
01. Uma pessoa pede informação na recepção de um prédio comercial
de como chegar a uma sala, e recebe as seguintes instruções: suba a
escada em forma de U à frente, ao final dela vire à esquerda, siga um
pouco à frente e em seguida vire à direita e siga pelo corredor. Ao
final do corredor, vire à direita. Uma possível projeção vertical dessa
trajetória no plano da base do prédio é:

a)

• Todo plano que passa pelo ponto médio de um segmento é


equidistante das extremidades do segmento.
b)

PROJEÇÃO ORTOGONAL
• Projeção ortogonal de um ponto sobre um plano: é o pé da
perpendicular ao plano conduzida pelo ponto.
• Projeção ortogonal de uma figura sobre um plano: é o conjunto
das projeções ortogonais dos pontos da figura sobre o plano.
• Projeção ortogonal de uma reta sobre um plano:
1°) reta perpendicular ao plano: traço da reta sobre o plano;
2°) reta não perpendicular ao plano: a projeção ortogonal da reta
r sobre o plano α é o traço em α do plano β, perpendicular a α, c)
conduzido por r.
α→ plano de projeção
β→ plano projetante
• Projeção ortogonal de um segmento de reta AB sobre um
plano α, contido numa reta não perpendicular a esse plano é
o segmento A’B’ de α onde A’ e B’ são as projeções ortogonais
de A e B, respectivamente, sobre o plano α.

ÂNGULO DE UMA RETA COM UM PLANO d)


• O ângulo entre uma reta e um plano oblíquos é o ângulo que
a reta forma com sua projeção ortogonal sobre o plano.
• O ângulo entre uma reta e um plano perpendiculares é reto.
• Se a reta é paralela ou está contida no plano, o ângulo entre
a reta e o plano é nulo.
• Se uma reta é oblíqua a um plano α e o intercepta em A,
então o ângulo agudo de r com sua projeção ortogonal r’
sobre α é menor que o ângulo agudo de r com qualquer outra
reta de α que passe por A.
• Se dois planos α e β são oblíquos, r é sua intersecção, e por um e)
ponto P de α, não pertencente a r, são conduzidas duas retas
concorrentes a e b, sendo a perpendicular a r, então o ângulo
entre a reta a e β é maior que o ângulo entre a reta b e β.
• Reta de maior declive de um plano em relação a outro: se
dois planos α e β são oblíquos, toda reta de α perpendicular
à interseção dos planos é chamada reta de maior declive de
α em relação a β. A reta de maior declive é a reta de α que
forma ângulo máximo com β.

434

PM_BOOK16 - MAT.indb 434 25/11/2022 19:23:48


FUNDAMENTOS DE GEOMETRIA ESPACIAL

02. Uma lagartixa está no interior de um quarto e começa a se


deslocar. Esse quarto, apresentando o formato de um paralelepípedo
retangular, é representado pela figura.

d) e)

04. Um grupo de escoteiros mirins, numa atividade no parque da


cidade onde moram, montou uma barraca conforme a foto da Figura
1. A Figura 2 mostra o esquema da estrutura dessa barraca, em forma
de um prisma reto, em que foram usadas hastes metálicas.

A lagartixa parte do ponto B e vai até o ponto A. A seguir, de A ela


se desloca, pela parede, até o ponto M, que é o ponto médio do
segmento EF. Finalmente, pelo teto, ela vai do ponto M até o ponto H.
Considere que todos esses deslocamentos foram feitos pelo caminho
de menor distância entre os respectivos pontos envolvidos. A projeção
ortogonal desses deslocamentos no plano que contém o chão do
quarto é dado por:
a)

d)
b) Após a armação das hastes, um dos escoteiros observou um inseto
deslocar-se sobre elas, partindo do vértice A em direção ao vértice B,
deste em direção ao vértice E e, finalmente, fez o trajeto do vértice E ao
C. Considere que todos esses deslocamentos foram feitos pelo caminho
de menor distância entre os pontos. A projeção do deslocamento do
e) inseto no plano que contém a base ABCD é dada por:

c)

a) d)
03. Os alunos de uma escola utilizaram cadeiras iguais às da figura
para uma aula ao ar livre. A professora, ao final da aula, solicitou que
os alunos fechassem as cadeiras para guardá-las. Depois de guardadas,
os alunos fizeram um esboço da vista lateral da cadeira fechada.
b) e)

c)

05. A figura a seguir representa uma cadeira onde o assento é um


paralelogramo perpendicular ao encosto.

Qual é o esboço obtido pelos alunos?

a) b) c)

435

PM_BOOK16 - MAT.indb 435 25/11/2022 19:23:49


FUNDAMENTOS DE GEOMETRIA ESPACIAL

A partir dos pontos dados, é correto afirmar que os segmentos de


retas:
a) CD e EF são paralelos.
b) BD e FJ são concorrentes.
c) AC e CD são coincidentes.
d) AB e EI são perpendiculares.

06. O galpão da figura a seguir está no prumo e a cumeeira está


“bem no meio” da parede.

Quantos pares de retas reversas é possível formar com as retas suportes


das arestas do sólido, sendo r uma das retas do par?
a) 12
b) 10
c) 8
d) 7
e) 6

09. Na figura a seguir tem-se: o plano α definido pelas retas c e d,


perpendiculares entre si; a reta b, perpendicular a α em A, com A ∈ c;
Das retas assinaladas podemos afirmar que: o ponto B, intersecção de c e d. Se X é um ponto de b, X ∉ α, então
a reta s, definida por X e B,
a) t e u são reversas
b) s e u são reversas
c) t e u são concorrentes
d) s e r são concorrentes
e) t e u são perpendiculares

07. Na figura abaixo, está representado um cubo.

a) é paralela à reta c
b) é paralela à reta b
c) está contida no plano α
d) é perpendicular à reta d
e) é perpendicular à reta b

10. No contexto da Geometria Espacial, afirma-se:


I. Se uma reta é paralela a um plano, então ela está contida nesse
plano.
II. Duas retas sem ponto comum são paralelas ou reversas.
III. Se dois planos são paralelos, então toda reta de um deles é
A seção produzida no cubo pelo plano CDE tem a forma de:
paralela ao outro.
a) triângulo
IV. Duas retas distintas paralelas a um plano são paralelas entre si.
b) trapézio
São corretas apenas as afirmativas:
c) retângulo
a) I e II
d) pentágono
b) I e III
e) hexágono
c) II e III
d) II e IV
08. Considere o sólido geométrico exibido na figura, constituído de
um paralelepípedo encimado por uma pirâmide. Seja r a reta suporte e) III e IV
de uma das arestas do sólido, conforme mostrado.

436

PM_BOOK16 - MAT.indb 436 25/11/2022 19:23:49


FUNDAMENTOS DE GEOMETRIA ESPACIAL

EXERCÍCIOS DE a) são perpendiculares

TREINAMENTO b) são reversas


c) são paralelas
d) podem ser paralelas ou reversas
01. Marque a opção que indica quantos pares de retas reversas são
08. A reta r é a intersecção dos planos α e β, perpendiculares entre si.
formados pelas retas suportes das arestas de um tetraedro.
A reta s, contida em α, intercepta r no ponto P. A reta t, perpendicular
a) Um par. a β, intercepta-o no ponto Q, não pertencente a r. Nessas condições,
b) Dois pares. é verdade que as retas:
c) Três pares. a) r e s são perpendiculares entre si.
d) Quatro pares. b) s e t são paralelas entre si.
e) Cinco pares. c) r e t são concorrentes.
d) s e t são reversas.
02. Seja A um ponto pertencente à reta r, contida no plano α. É e) r e t são ortogonais.
verdade que:
a) existe uma única reta que é perpendicular à reta r no ponto A. 09. Sejam r uma reta e β1 e β2 dois planos no espaço, considere as
b) existe uma única reta, não contida no plano α, que é paralela seguintes afirmações:
à reta r. I. Se r ∩ β1 = {P1} e r ∩ β2 = {P2}, com P1 e P2 pontos distintos, então
c) existem infinitos planos distintos entre si, paralelos ao plano α, β1 é paralelo a β2.
que contêm a reta r. II. r ∩ β1 = ∅ e r ∩ β2 = ∅, então β1 é paralelo a β2 ou β1 é coincidente
d) existem infinitos planos distintos entre si, perpendiculares ao de β2.
plano α e que contêm a reta r. III. Se existem dois pontos distintos em r ∩ β1, então r ∩ β1 = r.
e) existem infinitas retas distintas entre si, contidas no plano α e que
É CORRETO afirmar que:
são paralelas à reta r.
a) Apenas I é verdadeira.
03. (AFA) Se a reta r é paralela ao plano α, r ⊄ α, então: b) Apenas II é verdadeira.
a) todas as retas de α são paralelas a r. c) Apenas III é verdadeira.
b) existem em α retas paralelas e perpendiculares a r. d) Apenas I e II são verdadeiras.
c) a reta r não pode ser coplanar com nenhuma reta de α. e) Apenas II e III são verdadeiras.
d) existem em α retas paralelas a r e retas reversas a r.
10. Observe e classifique as afirmações abaixo como sendo
04. Seja uma reta r e os planos secantes α e β, de modo que verdadeiras ou falsas:
α ∩ β = r. Seja s uma reta paralela à reta r, de modo que s ∩ β = ∅. I. Se um plano intercepta dois outros planos paralelos, então as
Seja t uma reta secante ao plano β no ponto P, de modo que P ∈ r. De interseções são retas paralelas.
acordo com essas informações, necessariamente: II. Se dois planos são paralelos, qualquer reta de um deles é paralela
a) s ∩ α = s a qualquer reta do outro.
b) t ∩ β = ∅ III. Se uma reta é paralela a dois planos, então esses planos são paralelos.
c) P∉α IV. Se dois planos são paralelos, uma reta de um deles pode ser
d) r ∩ t ≠ ∅ reversa a uma reta do outro.
Marque a alternativa CORRETA:
05. (AFA) A intersecção de 3 superfícies esféricas distintas pode ser, a) Apenas as afirmações I e II são verdadeiras.
somente, ou:
b) Apenas as afirmações I e III são verdadeiras.
a) 1 ponto, ou vazia, ou 1 circunferência.
c) Apenas as afirmações I e IV são verdadeiras.
b) 1 ponto, ou vazia, ou 2 circunferências.
d) Apenas as afirmações II e IV são verdadeiras.
c) 1 segmento de reta, ou vazia, ou 1 circunferência.
e) Apenas as afirmações III e IV são verdadeiras.
d) 2 pontos, ou 1 ponto, ou vazia, ou 1 circunferência.
11. As afirmações seguintes podem ser verdadeiras ou falsas.
06. (AFA) Qual das afirmações abaixo é verdadeira?
I. A projeção ortogonal de uma reta num plano é uma reta.
a) Por uma reta dada pode-se conduzir um plano paralelo a um
plano dado. II. Distância entre duas retas reversas é a perpendicular comum a
essas retas.
b) Se uma reta é paralela a dois planos, então esses planos são paralelos.
III. A distância entre dois planos só é definida se esses planos são paralelos.
c) Por um ponto qualquer é possível traçar uma reta que intercepta
duas retas reversas dadas. É correto afirmar que SOMENTE:
d) Se duas retas concorrentes de um plano são, respectivamente, a) II é verdadeira.
paralelas a duas retas de outro plano, então estes planos são b) III é verdadeira.
paralelos.
c) I e II são verdadeiras.
07. (AFA) A reta r é paralela ao plano α; o plano β contém r e intercepta d) I e III são verdadeiras.
o plano α segundo a reta s. O que se pode afirmar sobre as retas r e s? e) II e III são verdadeiras.

437

PM_BOOK16 - MAT.indb 437 25/11/2022 19:23:49


FUNDAMENTOS DE GEOMETRIA ESPACIAL

12. Considere uma reta s, contida em um plano α, e uma reta r c) Se duas retas r e s são reversas, então não existem dois planos α e
perpendicular a s. Então, necessariamente: β, perpendiculares entre si, tais que r ∈ α e s ∈ β.
a) r é perpendicular a α. d) Duas retas do espaço, paralelas a uma terceira, são paralelas entre si.
b) r e s são coplanares.
17. Analise as afirmativas a seguir, relativas à geometria espacial e
c) r é paralela a α.
coloque V nas Verdadeiras e F nas Falsas.
d) r está contida em α.
( ) Se uma reta está contida em um plano, então toda reta
e) Todas as retas paralelas a r interceptam s. perpendicular a ela será perpendicular ao plano.
( ) Se dois planos distintos são paralelos, então toda reta perpendicular
13. Considere um plano α e um ponto P qualquer do espaço. Se por a um deles é paralela ao outro.
P traçarmos a reta perpendicular a α, a intersecção dessa reta com
α é um ponto chamado projeção ortogonal do ponto P sobre α. No ( ) Se dois planos distintos são paralelos a uma reta fora deles, então
caso de uma figura F do espaço, a projeção ortogonal de F sobre α é eles são paralelos entre si.
definida pelo conjunto das projeções ortogonais de seus pontos. Com ( ) Se dois planos distintos são paralelos, qualquer reta de um deles é
relação a um plano á qualquer fixado, pode-se dizer que: paralela a qualquer reta do outro.
a) a projeção ortogonal de um segmento de reta pode resultar Assinale a alternativa que apresenta a sequência CORRETA.
numa semi-reta.
a) F – F – V – V
b) a projeção ortogonal de uma reta sempre resulta numa reta.
b) F – V – V – F
c) a projeção ortogonal de uma parábola pode resultar num
c) F–F–F–F
segmento de reta.
d) V – F – F – V
d) a projeção ortogonal de um triângulo pode resultar num quadrilátero.
e) V – V – F – F
e) a projeção ortogonal de uma circunferência pode resultar num
segmento de reta.
18. (ESPCEX) Considere as seguintes proposições:
14. Sobre os conhecimentos de geometria tridimensional, considere I. Toda reta paralela a um plano é paralela a qualquer reta desse plano.
as afirmativas: II. Uma reta e um ponto determinam sempre um único plano.
I. Se duas retas distintas não são paralelas, então elas são III. Se uma reta é perpendicular a duas retas concorrentes de um
concorrentes. plano, então ela é perpendicular a esse plano.
II. Três pontos distintos entre si determinam um único plano. Pode-se afirmar que:
III. Duas retas paralelas distintas determinam um plano. a) Só I é verdadeira.
IV. Se duas retas r e s são reversas, então existe um único plano á que b) Só III é verdadeira.
contém r e é paralelo a s.
c) Só I e III são verdadeiras.
A alternativa que contém todas as afirmativas corretas é:
d) Só III é falsa.
a) I e II
e) Só I e III são falsas.
b) I e IV
c) III e IV 19. O lugar geométrico dos pontos do espaço que equidistam de três
d) I, II e III pontos não colineares distintos é:
e) II, III e IV a) uma reta
b) um plano
15. (AFA) Considere as proposições a seguir: c) uma esfera
I. Se dois planos são paralelos, então toda reta que é paralela a um d) um ponto
deles é paralela ou está contida no outro.
e) vazio
II. Se uma reta é paralela a um plano, então é paralela a todas as
retas do plano. 20. (ESPCEX) Considere as afirmações abaixo:
III. Se uma reta possui dois pontos distintos num plano, então ela I. Se um plano encontra outros dois planos paralelos, então as
está contida no plano. intersecções são retas paralelas.
IV. Se dois planos são secantes, toda reta de um, sempre intercepta II. Uma reta perpendicular a uma reta de um plano e ortogonal a
o outro plano. outra reta desse plano é perpendicular ao plano.
Pode-se afirmar que as proposições verdadeiras são: III. Se a intersecção de uma reta r com um plano é o ponto P, reta essa
a) I e IV não perpendicular ao plano, então existe uma única reta s contida
b) II e III nesse plano que é perpendicular à reta r passando por P.
c) I e III Pode-se afirmar:
d) II e IV a) que todas são verdadeiras.
b) apenas I e II são verdadeiras.
16. Assinale a única alternativa FALSA. c) apenas I e III são verdadeiras.
a) Se um plano α é perpendicular a um plano β, então existem d) apenas II e III são verdadeiras.
infinitas retas contidas em α e perpendiculares a β.
e) todas são falsas.
b) Se α e β são planos perpendiculares entre si e γ é um plano
perpendicular à reta comum a α e β, então pode-se afirmar que 21. (ESPCEX) Considere dois planos α e β perpendiculares e três retas
as retas r, r = α ∩ γ e s, s = β ∩ γ, são perpendiculares entre si. distintas r, s e t tais que r ⊂ α, s ⊂ β e t = α ∩ β. Sobre essas retas e os
planos é correto afirmar que:

438

PM_BOOK16 - MAT.indb 438 25/11/2022 19:23:50


FUNDAMENTOS DE GEOMETRIA ESPACIAL

a) as retas r e s somente definirão um plano se forem concorrentes O número de afirmativas verdadeiras é:


com t em um único ponto. a) 1
b) as retas r e s podem definir um plano paralelo à reta t. b) 2
c) as retas r e s são necessariamente concorrentes. c) 3
d) se r e s forem paralelas, então elas definem um plano d) 4
perpendicular a α e β.
e) o plano definido por r e t é necessariamente paralelo a s. 26. (EPCAR 3° ANO) Analise as afirmativas abaixo, classificando-as em
(V) verdadeira ou (F) falsa.
22. (ESPCEX) Considere as seguintes afirmações: I. Se um plano é perpendicular a outro, então ele é perpendicular a
I. Se uma reta r é perpendicular a um plano α, então todas as retas qualquer reta desse outro.
de α são perpendiculares ou ortogonais a r; II. Duas retas reversas podem ser ambas perpendiculares a uma
II. Se a medida da projeção ortogonal de um segmento AB sobre um mesma reta t.
plano α é a metade da medida do segmento AB, então a reta AB III. Se dois planos distintos são paralelos entre si, então toda reta de
faz com α um ângulo de 60°; um é paralela a toda reta do outro plano.
III. Dados dois planos paralelos α e β, se um terceiro plano γ intercepta IV. Se uma reta é perpendicular a um plano, então ela é perpendicular
α e β, as interseções entre esses planos serão retas reversas; a qualquer reta do plano.
IV. Se α e β são dois planos secantes, todas as retas de α também
A sequência correta é:
interceptam β.
a) V – F – F – V
Estão corretas as afirmações:
b) F – V – F – F
a) apenas I e II
c) F–V–F–V
b) apenas II e III
d) V – F – V – F
c) I, II e III
d) I, II e IV
27. Sobre o estudo da Geometria Espacial, é INCORRETO afirmar que:
e) II, III e IV
a) dados três pontos não-colineares do espaço, existe um, e somente
um, plano que os contém.
23. Considere as seguintes afirmativas sobre pontos, retas e planos
b) por um ponto passa uma única reta perpendicular a um plano dado.
no espaço:
c) se dois planos são perpendiculares, toda reta de um deles que for
I. Três pontos distintos determinam um único plano.
perpendicular à interseção dos dois planos será perpendicular
II. Um ponto e uma reta determinam um único plano. ao outro.
III. Duas retas concorrentes determinam um único plano. d) se dois planos distintos são paralelos, qualquer reta de um deles é
IV. Duas retas paralelas determinam um único plano. paralela a qualquer reta do outro.
Pode-se afirmar que:
28. Qual das afirmações abaixo é verdadeira?
a) nenhuma das afirmativas é verdadeira.
a) Três pontos, distintos dois a dois, determinam um plano.
b) apenas uma das afirmativas é verdadeira.
b) Um ponto e uma reta determinam um plano.
c) apenas duas das afirmativas são verdadeiras.
c) Se dois planos distintos têm um ponto em comum, tal ponto é único.
d) apenas três das afirmativas são verdadeiras.
d) Se uma reta é paralela a um plano e não está contida neste plano,
e) Todas as afirmativas são verdadeiras. então ela é paralela a qualquer reta desse plano.
e) Se α é o plano determinado por duas retas concorrentes r e
24. Assinale a opção que apresenta a afirmativa incorreta. s, então toda reta m desse plano, que é paralela à r, não será
a) Se uma reta é paralela a dois planos, então esses planos são paralelos. paralela à reta s.
b) Duas retas que não possuem pontos em comum não são
necessariamente paralelas. 29. (EN) Nas proposições abaixo, coloque V na coluna à esquerda
c) A reta interseção de dois planos perpendiculares a um terceiro é quando a proposição for verdadeira e F quando for falsa.
perpendicular a este. ( ) Se uma reta é perpendicular a duas retas distintas de um plano,
d) Dados uma reta e um ponto, existe apenas um plano perpendicular então ela é perpendicular ao plano.
à reta que contém o ponto. ( ) Se uma rota é perpendicular a uma reta perpendicular a um plano,
e) Por uma reta não paralela e não perpendicular a um plano α passa então ela é paralela a uma reta do plano.
um único plano perpendicular a α. ( ) Duas retas perpendiculares a um plano são paralelas.
( ) Se dois planos são perpendiculares, todo plano paralelo a um
25. (AFA) Considere as afirmativas abaixo: deles é perpendicular ao outro.
I. Se α e β são planos interceptando-se na reta r e a reta s é paralela ( ) Se três planos são dois a dois perpendiculares , eles têm um único
a α e a β, então s também é paralela a r. ponto em comum.
II. Se uma reta intercepta um plano α, existe um plano β paralelo a Lendo-se a coluna da esquerda, de cima para baixo, encontra-se:
α que não é interceptado pela reta.
a) F – F – V – F – V
III. Se dois planos são paralelos, toda reta contida em um deles é
paralela ao outro plano. b) V – F – V – V – F
IV. Dois planos perpendiculares a um terceiro plano são sempre c) V–V–F–V–V
paralelos entre si. d) F – V – V – V – V
V. Se três retas têm um ponto comum, elas são coplanares. e) V – V – V – V – V

439

PM_BOOK16 - MAT.indb 439 25/11/2022 19:23:50


FUNDAMENTOS DE GEOMETRIA ESPACIAL

30. (OBMEP) Os quatro dados da figura são idênticos, e há três pares a) Apenas uma é verdadeira.
de faces em contato. Qual é o valor da soma dessas faces? b) Apenas duas são verdadeiras.
c) Apenas três são verdadeiras.
d) Todas são verdadeiras.

04. Entre todas as retas suportes das arestas de um certo cubo,


considere duas, r e s, reversas. Seja t a perpendicular comum a r e a
s. Então:
a) t é a reta suporte de uma das diagonais de uma das faces do cubo.
b) t é a reta suporte de uma das diagonais do cubo.
c) t é a reta suporte de uma das arestas do cubo.
a) 73 c) 75 e) 77
d) t é a reta que passa pelos pontos médios das arestas contidas em r e s.
b) 74 d) 76
e) t é a reta perpendicular a duas faces do cubo, por seus pontos médios.

05. (AFA 1996) Os planos α e β são paralelos. A reta r é perpendicular


EXERCÍCIOS DE a α e a reta s é perpendicular a β. Pode-se concluir que r e s são:

COMBATE a) coplanares.
b) reversas.
c) ortogonais.
d) perpendiculares.

06. (ESPCEX) A ilustração a seguir representa um paralelepípedo


01. (EN 2004) Analise as afirmativas abaixo. retângulo ABCDEFGH e um prisma reto triangular de base EHJ
I. Se uma reta e um plano são concorrentes, então a reta é seccionado por um plano, gerando o triângulo isósceles ADI, cuja
concorrente com qualquer reta do plano. medida AI é igual à medida DI. Diante das informações acima,
podemos afirmar que:
II. Se duas retas distintas são paralelas a um plano, então elas são
paralelas entre si. B A
III. Duas retas ou são coplanares ou são reversas.
IV. Se uma reta é paralela a dois planos, então esses planos são paralelos.
Assinale a alternativa correta.
C D
a) Apenas a afirmativa III é verdadeira.
b) Apenas as afirmativas III e IV são verdadeiras.
c) Todas as afirmativas são falsas.
d) Apenas a afirmativa I é verdadeira.
e) Apenas a afirmativa II é falsa. I

02. Sejam as afirmações:


I. Por um ponto passa uma única reta.
II. Um ponto e uma reta determinam um único plano. F E
III. Se dois pontos de uma reta pertencem a um plano, então a reta
está contida nesse plano.
J
IV. Por um ponto situado fora de uma reta, existe uma reta paralela
à reta dada. G
H
Podemos garantir que:
a) apenas (III) é verdadeira. a) a reta JH é ortogonal à reta DC.
b) (I) e (II) são falsas. b) as retas EJ e FG são reversas.
c) apenas (I) é falsa. c) a reta IJ é ortogonal à reta EF.
d) apenas (II) e (III) são verdadeiras. d) a reta AI é concorrente à reta BC.
e) a reta AI é paralela à reta EJ.
e) apenas (III) e (IV) são verdadeiras.

07. (ESPCEX) Considere um plano α e os pontos A, B, C e D tais que:


03. Coloque V ou F nas assertivas abaixo.
• o segmento AB tem 6 cm de comprimento e está contido em α;
I. Se dois planos distintos têm um ponto em comum, então eles têm
uma reta comum que passa pelo ponto. • o segmento BC tem 24 cm de comprimento, está contido em α
e é perpendicular a AB;
II. Uma reta e um plano paralelos não têm ponto em comum.
• o segmento AD tem 8 cm de comprimento e é perpendicular a α.
III. Se dois planos são paralelos então uma reta de um deles pode ser
paralela a uma reta do outro. Nessas condições, a medida do segmento CD é:
IV. Se um plano contém duas retas distintas paralelas a um outro a) 26 cm d) 32 cm
plano então os planos são paralelos. b) 28 cm e) 34 cm
c) 30 cm

440

PM_BOOK16 - MAT.indb 440 25/11/2022 19:23:50


FUNDAMENTOS DE GEOMETRIA ESPACIAL

08. (ESPCEX) Considere as seguintes afirmações:

DESAFIO PRO
I. Se dois planos α e β são paralelos distintos, então as retas r1 ⊂ α
e r2 ⊂ β são sempre paralelas.
II. Se α e β são planos não paralelos distintos, existem as retas tal
que r1 e r2 são paralelas.
III. Se uma reta r é perpendicular a um plano α no ponto P, então
qualquer reta de α que passa por P é perpendicular a r.
Dentre as afirmações acima, é (são) verdadeira(s):
1 (IME) Sejam dois quadrados de lado a situados em
planos distintos que são paralelos entre si e situados a
uma distância d, um do outro. A reta que liga os centros dos
a) somente II. quadrados é perpendicular a esses planos. Cada diagonal
de um quadrado é paralela a dois lados do outro quadrado.
b) I e II.
Liga-se cada vértice de cada quadrado aos dois vértices mais
c) I e III. próximos do outro quadrado. Obtêm-se, assim, triângulos que,
d) II e III. conjuntamente com os quadrados, formam um sólido S. Qual a
e) I, II e III. distância entre estes planos distintos em função de a, de modo
que os triângulos descritos acima sejam equiláteros?
09. (ESPCEX) O sólido geométrico abaixo é formado pela justaposição a a4 2
a) d)
de um bloco retangular e um prisma reto, com uma face em comum. 2 8
Na figura estão indicados os vértices, tanto do bloco quanto do
prisma. Considere os seguintes pares de retas definidas por pontos b) a 3 a(4 − 3 2
2 e)
dessa figura: as retas LB e GE; as retas AG e HI e as retas AD e GK. As 2
posições relativas desses pares de retas são, respectivamente, a 10
c)
8
K

J
2 (ITA) Das afirmações:

I. Duas retas coplanares são concorrentes;


H
L II. Duas retas que não têm ponto em comum são reversas;
I III. Dadas duas retas reversas, existem dois, e apenas dois,
D planos paralelos, cada um contendo uma das retas;
G IV. Os pontos médios dos lados de um quadrilátero reverso
definem um paralelogramo,
C É(são) verdadeira(s) apenas:
E
a) III.
b) I e III.
B c) II e III.
d) III e IV.
F
e) I e II e IV.

A GABARITO

a) concorrentes; reversas; reversas. EXERCÍCIOS DE FIXAÇÃO


b) reversas; reversas; paralelas. 01. B 04. E 07. B 10. C
c) concorrentes, reversas; paralelas. 02. B 05. A 08. C
d) reversas; concorrentes; reversas. 03. C 06. A 09. D
e) concorrentes; concorrentes; reversas. EXERCÍCIOS DE TREINAMENTO
01. C 09. C 17. C 25. B
10. (ESPCEX) Considere as seguintes afirmações: 02. E 10. C 18. B 26. B
I. Se uma reta r é perpendicular a um plano α, então todas as retas 03. D 11. B 19. A 27. D
de α são perpendiculares ou ortogonais a r. 04. D 12. B 20. C 28. E
II. Se a medida da projeção ortogonal de um segmento AB sobre um 05. D 13. E 21. B 29. D
plano α é a metade da medida do segmento AB, então a reta AB
06. D 14. C 22. A 30. D
faz com α um ângulo de 60º.
07. C 15. C 23. C
III. Dados dois planos paralelos α e β, se um terceiro plano γ intercepta
α e β, as interseções entre esses planos serão retas reversas; 08. E 16. C 24. A
IV. Se α e β são dois planos secantes, todas as retas de α também EXERCÍCIOS DE COMBATE
interceptam β. 01. A 04. C 07. A 10. A
Estão corretas as afirmações: 02. E 05. A 08. D
a) apenas I e II d) I, II e IV 03. C 06. C 09. E
b) apenas II e III e) II, III e IV DESAFIO PRO
c) I, II e III 01. D 02. D

441

PM_BOOK16 - MAT.indb 441 25/11/2022 19:23:51


FUNDAMENTOS DE GEOMETRIA ESPACIAL

ANOTAÇÕES

442

PM_BOOK16 - MAT.indb 442 25/11/2022 19:23:51


GEOMETRIA ESPACIAL: DIEDROS,
TRIEDROS E POLIEDROS

DIEDROS
Ângulo diedro ou diedro ou ângulo diédrico é a reunião de dois
semiplanos α e β de mesma origem não contidos num mesmo plano
e é denotado por αβ.

A origem comum V é chamada vértice do triedro e cada uma das


semirretas a, b e c, aresta.
Um triângulo que possui um vértice em cada aresta do triedro é
uma seção do triedro, como o triângulo ABC.
Triedro tri-retângulo: faces são ângulos retos e os diedros são
A reta r chama-se aresta do diedro e os semiplanos α e β, faces. diedros retos.
Seção reta ou seção normal de um diedro é uma seção cujo
plano é perpendicular à aresta do diedro.

Triedro
tri-retângulo

CONDIÇÃO DE EXISTÊNCIA
1. A medida de cada face está compreendida entre 0° e 180°.
2. Em todo triedro, qualquer face é menor que a soma das
outras duas.
Na figura r ⊥ γ e o ângulo a^b é a seção reta do diedro. 3. A soma das medidas das faces de um triedro qualquer é
A medida de um diedro é a medida de sua seção reta (a menor que 360°.
medida de um diedro é igual ao ângulo entre as faces).
Diedro reto é aquele cuja seção normal é um ângulo reto. ÂNGULOS POLIÉDRICOS CONVEXOS
O conceito de ângulos poliédricos convexos é uma
TRIEDROS extensão do conceito de triedros. Dado um número finito
n ≥ 3 de semirretas Va1, Va2, ..., Van, de mesma origem V, tais que o
Sejam três semirretas de mesma origem e não coplanares. Três plano de duas consecutivas deixa as demais num mesmo semiespaço,
planos podem ser formados, um a partir de cada par de retas. Cada consideremos n semiespaços E1, E2, ..., En, cada um deles com origem
plano determina um semiespaço que contém a terceira semirreta. no plano de duas semirretas consecutivas e contendo as restantes.
Assim, o triedro V(a,b,c) ou V(A,B,C) formado pelas três semirretas é a Então o ângulo poliédrico convexo determinado por Va1, Va2, ..., Van
interseção desses três semiespaços. é a interseção dos semiespaços E1, E2, ..., En.

443

PM_BOOK16 - MAT.indb 443 25/11/2022 19:23:52


GEOMETRIA ESPACIAL: DIEDROS, TRIEDROS E POLIEDROS

CONDIÇÃO DE EXISTÊNCIA RELAÇÃO DE EULER


1. A medida de cada face está compreendida entre 0° e 180°. Seja um poliedro convexo com V vértices, A arestas e F faces, então
2. Em todo ângulo poliédrico convexo, qualquer face é menor V F  A 2
que a soma das demais.
3. A soma das medidas das faces de um ângulo poliédrico Os poliedros que satisfazem a relação de Euler são chamados
convexo qualquer é menor que 360°. poliedros eulerianos.
Todo poliedro convexo é euleriano, mas nem todo poliedro
POLIEDROS CONVEXOS E euleriano é convexo.
A soma dos ângulos de todas as faces de um poliedro convexo
RELAÇÃO DE EULER de V vértices é

DEFINIÇÕES S  360   V  2
Poliedro convexo é uma reunião de um número finito de Seja fn o número de faces de gênero n, então:
polígonos planos convexos chamados faces onde:
1º) dois polígonos não estão no mesmo plano; 3f3  4 f4    2A
2º) cada lado de polígono é comum a dois e somente dois Seja vp o número de vértices onde concorrem p arestas, então:
polígonos; e
3º) o plano de cada polígono deixa os demais polígonos no 3v 3  4 v 4    2A
mesmo semiespaço.
Seja ∑df o total de diagonais das faces, então o número de
diagonais do poliedro D é:

V ( V  1)
D
2
A  df
POLIEDROS DE PLATÃO
Um poliedro é chamado poliedro de Platão se, e somente se:
Os vértices das faces são também os vértices do poliedro e os 1º) todas as faces têm o mesmo número de arestas;
lados das faces são chamadas arestas do poliedro. 2) todos os ângulos poliédricos têm o mesmo número de arestas; e
Cada vértice do poliedro corresponde a um ângulo poliédrico, no 3º) vale a relação de Euler (V − A + F = 2).
qual está contido todo o poliedro.
Existem exatamente cinco poliedros de Platão.
Diagonal do poliedro é qualquer segmento com extremidades
em dois vértices do poliedro e não contido em uma face. Seja n o mesmo número de arestas de cada face e m o número de
arestas dos ângulos poliédricos, temos:

Poliedros de Platão
M N A V F NOME

3 3 6 4 4 tetraedro

3 4 12 8 6 hexaedro

4 3 12 6 8 octaedro

3 5 30 20 12 dodecaedro

5 3 30 12 20 icosaedro

No poliedro acima, por exemplo, BH é uma diagonal. tetraedro hexaedro octaedro


Os poliedros são classificados de acordo com o seu número de faces.
O poliedro com menor número de faces é o tetraedro e possui 4 faces.

dodecaedro icosaedro

444

PM_BOOK16 - MAT.indb 444 25/11/2022 19:23:56


GEOMETRIA ESPACIAL: DIEDROS, TRIEDROS E POLIEDROS

POLIEDROS REGULARES
Um poliedro convexo é regular quando suas faces são polígonos
regulares e congruentes e seus ângulos poliédricos são congruentes.
Todo poliedro regular convexo é um poliedro de Platão, mas nem
c)
todo poliedro de Platão é um poliedro regular.
Os poliedros convexos regulares são cinco: o tetraedro regular,
o hexaedro regular, o octaedro regular, o dodecaedro regular e o
icosaedro regular.

tetraedro regular hexaedro regular octaedro regular

d)

dodecaedro regular icosaedro regular

e)

02. O hábito cristalino é um termo utilizado por mineralogistas para


EXERCÍCIOS DE descrever a aparência típica de um cristal em termos de tamanho e

FIXAÇÃO
forma. A granada é um mineral cujo hábito cristalino é um poliedro
com 30 arestas e 20 vértices. Um mineralogista construiu um modelo
ilustrativo de um cristal de granada pela junção dos polígonos
correspondentes às faces.
01. Observe, abaixo, uma imagem desse vírus que tem a forma de um Supondo que o poliedro ilustrativo de um cristal de granada é
sólido geométrico. convexo, então a quantidade de faces utilizadas na montagem do
modelo ilustrativo desse cristal é igual a:
a) 10
b) 12
c) 25
d) 42
e) 50

03. Dado um tetraedro regular de aresta 6 m, assinale os pontos que


dividem cada aresta em três partes iguais. Corte o tetraedro pelos planos
que passam pelos três pontos de divisão mais próximos de cada vértice e
remova os pequenos tetraedros regulares que ficaram formados.
A soma dos comprimentos de todas as arestas do sólido resultante,
Qual é a planificação do sólido representado por esse vírus? em centímetros, é:
a) 56
b) 32
c) 30
d) 36
a)
e) 48

04. Um poliedro convexo com 32 vértices possui apenas faces


triangulares. O número de arestas deste poliedro é:
a) 100
b) 120
c) 90
d) 80
b)

445

PM_BOOK16 - MAT.indb 445 25/11/2022 19:23:57


GEOMETRIA ESPACIAL: DIEDROS, TRIEDROS E POLIEDROS

05. Os sólidos de Platão são poliedros convexos cujas faces são todas Atualmente, além dos dados em forma de cubo (hexaedro),
congruentes a um único polígono regular, todos os vértices têm o encontram-se dados em vários formatos, inclusive esféricos, como
mesmo número de arestas incidentes e cada aresta é compartilhada mostram as figuras a seguir.
por apenas duas faces. Eles são importantes, por exemplo, na
classificação das formas dos cristais minerais e no desenvolvimento de
diversos objetos. Como todo poliedro convexo, os sólidos de Platão
respeitam a relação de Euler V – A + F = 2, em que V, A e F são os
números de vértices, arestas e faces do poliedro, respectivamente.
Em um cristal, cuja forma é a de um poliedro de Platão de faces
triangulares, qual é a relação entre o número de vértices e o número
de faces?
a) 2V – 4F = 4 d) 2V + F = 4
Apesar do formato esférico, ao ser lançado, o dado mostra pontos de um
b) 2V – 2F = 4 e) 2V + 5F = 4 a seis, como se fosse um dado cúbico. Isso acontece porque no interior da
c) 2V – F = 4 esfera existe uma cavidade em forma de octaedro, na qual existe um peso
(um chumbinho) que se aloja em um dos vértices do octaedro.
06. Um lapidador recebeu de um joalheiro a encomenda para
trabalhar em uma pedra preciosa cujo formato é o de uma pirâmide,
conforme ilustra a Figura 1. Para tanto, o lapidador fará quatro
cortes de formatos iguais nos cantos da base. Os cantos retirados
correspondem a pequenas pirâmides, nos vértices P, Q, R e S, ao longo
dos segmentos tracejados, ilustrados na Figura 2.

Assinale a alternativa que apresenta, corretamente, a propriedade dos


poliedros regulares que justifica o fato de a cavidade no interior da
esfera ser octaédrica.
a) O número de vértices do octaedro é igual ao número de faces
do hexaedro.
b) O número de vértices do octaedro é diferente do número de faces
do hexaedro.
c) O número de arestas do octaedro é igual ao número de arestas
do hexaedro.
Depois de efetuados os cortes, o lapidador obteve, a partir da pedra
maior, uma joia poliédrica cujos números de faces, arestas e vértices d) O número de faces do octaedro é igual ao número de vértices do
são, respectivamente, iguais a: hexaedro.
a) 9, 20 e 13 e) O número de faces do octaedro é diferente do número de vértices
do hexaedro.
b) 3, 24 e 13
c) 7, 15 e 12
09. A bola de futebol evoluiu ao longo do tempo e, atualmente, é um
d) 10, 16 e 5 icosaedro truncado, formado por 32 peças, denominadas de gomos e,
e) 11, 16 e 5 geometricamente, de faces. Nessa bola, 12 faces são pentágonos regulares,
e as outras, hexágonos, também regulares. Os lados dos pentágonos e dos
07. O poliedro representado na figura (octaedro truncado) é hexágonos são iguais e costurados. Ao unirem-se os dois lados costurados
construído a partir de um octaedro regular, cortando-se, para tal, em das faces, formam-se as arestas. O encontro das arestas formam os vértices.
cada vértice, uma pirâmide regular de base quadrangular. A soma dos Quando cheio, o poliedro é similar a uma esfera.
ângulos internos de todas as faces do octaedro truncado é:

a) 2160º
b) 5760º
c) 7920º
d) 10080º
e) 13680º

08. Leia o texto a seguir.


O número de arestas e o número de vértices existentes nessa bola de
Originalmente os dados eram feitos de osso, marfim ou argila. Há futebol são, respectivamente,
evidências da existência deles no Paquistão, Afeganistão e noroeste da
Índia, datando de 3500 a.C. Os dados cúbicos de argila continham de Pode ser utilizado o Teorema de Descartes-Euler, A + 2 = V + F
1 a 6 pontos, dispostos de tal maneira que a soma dos pontos de cada a) 80 e 60 d) 90 e 60
par de faces opostas é sete. b) 80 e 50 e) 90 e 50
Adaptado de: Museu Arqueológico do Red Fort. Delhi, India.
c) 70 e 40

446

PM_BOOK16 - MAT.indb 446 25/11/2022 19:23:58


GEOMETRIA ESPACIAL: DIEDROS, TRIEDROS E POLIEDROS

10. Um poliedro convexo tem 9 faces e 16 arestas. Podemos afirmar 08. (PUC-SP) Um poliedro convexo tem 3 faces pentagonais e
que o total de vértices desse poliedro é algumas faces triangulares. Qual o número de faces desse poliedro,
a) 12 c) 15 e) 10 sabendo-se que o número de arestas é o quádruplo do número de
faces triangulares?
b) 9 d) 11
a) 4 d) 6
b) 3 e) 8
EXERCÍCIOS DE c) 5

TREINAMENTO 09. Determine o número de faces de um poliedro convexo e fechado


que tem 5 ângulos tetraédricos e 6 ângulos triédricos.

01. Um poliedro convexo tem 32 faces, sendo 20 hexágonos e 12 10. Um poliedro apresenta faces triangulares e quadrangulares. A
pentágonos. O número de vértices deste polígono: soma dos ângulos das faces é igual a 2160°. Determine o número de
a) 90 c) 60 faces de cada espécie desse poliedro, sabendo que ele tem 15 arestas.
b) 72 d) 56
11. Um poliedro convexo é formado por faces quadrangulares e 4
faces triangulares. A soma dos ângulos de todas as faces é igual a 12
02. De cada vértice de um prisma hexagonal regular foi retirado
retos. Qual o número de arestas desse poliedro?
um tetraedro, como exemplificado para um dos vértices do prisma
desenhado a seguir.
12. Um poliedro convexo apresenta faces quadrangulares e
triangulares. Calcule o número de faces desse poliedro, sabendo que
o número de arestas é o quádruplo do número de faces triangulares e
o número de faces quadrangulares é cinco.

13. (AFA) Um poliedro convexo tem 16 faces. De um dos seus vértices


partem 5 arestas; de cinco outros vértices partem 4 arestas e, de cada
um dos vértices restantes, partem 3 arestas. Qual o número total de
arestas desse poliedro?

14. (ITA-SP) Um poliedro convexo tem 13 faces. De um dos seus


vértices partem 6 arestas; de 6 outros vértices partem, de cada um,
4 arestas, e finalmente, de cada um dos vértices restantes partem 3
arestas. O número de arestas desse poliedro é:
O plano que definiu cada corte feito para retirar os tetraedros passa
pelos pontos médios das três arestas que concorrem num mesmo a) 13 d) 24
vértice do prisma. O número de faces do poliedro obtido depois de b) 17 e) 27
terem sido retirados todos os tetraedros é:
c) 21
a) 24 c) 18 e) 12
b) 20 d) 16 15. Seja um triedro tal que as duas maiores faces medem 70° e
120°, qual das alternativas abaixo apresenta um valor que pode ser a
03. Um poliedro convexo possui 8 (oito) faces, todas triangulares. medida da terceira face?
Nestas condições, assumindo que tal poliedro exista, o número a) 40° d) 70°
esperado de vértices para este será:
b) 50° e) 90°
a) 10 c) 8 e) 6
c) 60°
b) 9 d) 7
16. Dado um quadrado ABCD, toma-se AJ perpendicular ao plano do
04. O número de faces de um poliedro convexo com 20 vértices e com quadrado tal que AJ = AB. O ângulo entre os planos AJC e BJC vale:
todas as faces triangulares é igual a:
a) 90° c) 45°
a) 28 c) 32 e) 36
b) 60° d) 30°
b) 30 d) 34
17. Se a partir de um ponto P interior a um diedro forem traçadas
05. Um poliedro convexo possui 10 faces triangulares, 10 faces semi-retas perpendiculares às faces, o ângulo formado por elas é
quadrangulares e 1 face decagonal. Determine o número de vértices 105º. O ângulo entre os dois planos que formam o diedro é:
deste poliedro.
a) 15º d) 75º
b) 45º e) 105º
06. Um poliedro convexo é formado por 4 faces triangulares, 2 faces
quadrangulares e 1 face hexagonal. Determine o número de vértices c) 60º
desse poliedro.
18. Um ponto A interior a um diedro dista 4 m de sua aresta, 2 m
07. (CEFET - PR) Um poliedro convexo possui duas faces triangulares, de uma face e 2 2 m da outra face. O valor do ângulo retilíneo do
duas quadrangulares e quatro pentagonais. Logo, a soma dos ângulos diedro é:
internos de todas as faces será: a) 30º d) 75º
a) 3240º d) 4000º b) 45º e) 90º
b) 3640º e) 4060º c) 60º
c) 3840º

447

PM_BOOK16 - MAT.indb 447 25/11/2022 19:23:58


GEOMETRIA ESPACIAL: DIEDROS, TRIEDROS E POLIEDROS

19. Considere as afirmações: EXERCÍCIOS DE


I.
II.
Existe um triedro cujas 3 faces têm a mesma medida a = 120°.
Existe um ângulo poliédrico convexo cujas faces medem, COMBATE
respectivamente, 30°, 45°, 50°, 50° e 170°.
III. Um poliedro convexo que tem 3 faces triangulares, 1 face
quadrangular, 1 face pentagonal e 2 faces hexagonais tem 9 vértices. 5
01. O ângulo θ formado por dois planos α e β é tal que tg  .
IV. A soma das medidas de todas as faces de um poliedro convexo 5
com 10 vértices é 2880°. O ponto P pertence a α e a distância de P a β vale 1. Então, a distância
Destas, é(são) correta(s) apenas: de P à reta intersecção de α e β é igual a:
a) II d) I, II e IV a) 3 d) 7
b) IV e) II, III e IV 5
b) e) 8
c) II e IV
c) 6
20. (EN) Um poliedro convexo tem 6 faces retangulares e 12 faces
triangulares. O número de diagonais desse poliedro é: 02. Um poliedro convexo de nove vértices possui quatro ângulos
triédricos e cinco ângulos tetraédricos. Então o número de faces deste
a) 49 d) 61 poliedro é:
b) 52 e) 91 a) 12 d) 9
c) 60 b) 11 e) 8
c) 10
21. (EN) Os átomos de uma molécula de determinada substância
química se dispõem sobre os vértices de um poliedro convexo tal que
todas as faces possuem o mesmo número de arestas e cuja soma 03. Um poliedro convexo de 10 vértices apresenta faces triangulares
dos ângulos de todas as faces vale 2,088×104 graus. Sabendo que e quadrangulares. O número de faces quadrangulares, o número de
o poliedro tem 90 arestas, o menor inteiro que se deve somar ao faces triangulares e o número total de faces formam, nesta ordem,
número de faces para obter um quadrado perfeito é: uma progressão aritmética. O número de arestas é:

a) 1 d) 8 a) 10 d) 22

b) 4 e) 17 b) 17 e) 23

c) 7 c) 20

22. Um poliedro convexo possui 13 faces. Sabe-se que de um de seus 04. (AFA) Um poliedro platônico, cujas faces são triangulares, tem 30
vértices partem 4 arestas, de 5 outros vértices partem 3 arestas e de arestas. Determine o número de arestas que concorrem em cada vértice.
cada vértice restante partem 5 arestas. O número de arestas desse a) 3 c) 4
poliedro é: b) 5 d) 6
a) 32 d) 22
b) 12 e) 27 05. A soma de todos os ângulos das faces de todos exceto um dos
vértices de um poliedro simples é 5160º. Encontre a soma de todos os
c) 17
ângulos das faces do poliedro.

23. Um poliedro convexo possui 11 faces. Sabemos que, de um de a) 5340º d) 5760º


seus vértices partem 5 arestas, de outros 5 vértices partem 4 arestas b) 5400º e) 6120º
e de cada vértice restante partem 3 arestas. O número de arestas do c) 5520º
poliedro é:
a) 20 d) 37 06. (UERJ) Dois dados, com doze faces pentagonais cada um, têm a
b) 25 e) 41 forma de dodecaedros regulares. Se os dodecaedros estão justapostos
por uma de suas faces, que coincidem perfeitamente, formam um
c) 30
poliedro côncavo, conforme ilustra a figura.

24. Um poliedro convexo possui apenas faces triangulares e


quadrangulares. Sabendo que os números de faces triangulares e
quadrangulares são diretamente proporcionais aos números 2 e 3 e
que o número de arestas é o dobro do número de vértices, calcule o
número total de faces desse poliedro.
a) 10 c) 15
b) 12 d) 24

25. (ITA) Um poliedro convexo de 10 vértices apresenta faces


Considere o número de vértices V, de faces F e de arestas A desse
triangulares e quadrangulares. O número de faces quadrangulares, o
poliedro côncavo. A soma V + F + A é igual a:
número de faces triangulares e o número total de faces formam, nesta
ordem, uma progressão aritmética. O número de arestas é: a) 102
a) 10 d) 22 b) 106
b) 17 e) 23 c) 110
c) 20 d) 112

448

PM_BOOK16 - MAT.indb 448 25/11/2022 19:24:01


GEOMETRIA ESPACIAL: DIEDROS, TRIEDROS E POLIEDROS

07. (FGV) Dado um tetraedro regular de aresta 6 cm, assinale os


pontos que dividem cada aresta em três partes iguais. Corte o ANOTAÇÕES
tetraedro pelos planos que passam pelos três pontos de divisão mais
próximos de cada vértice e remova os pequenos tetraedros regulares
que ficaram formados.
A soma dos comprimentos de todas as arestas do sólido resultante,
em centímetros, é:
a) 56 b) 32 c) 30 d) 36 e) 48

08. (ITA) Considere as afirmações:


I. Existe um triedro cujas 3 faces tem a mesma medida α = 120°.
II. Existe um ângulo poliédrico convexo cujas faces medem,
respectivamente, 30°, 45°, 50°, 50° e 170°.
III. Um poliedro convexo que tem 3 faces triangulares, 1 face
quadrangular, 1 face pentagonal e 2 faces hexagonais tem 9
vértices.
IV. A soma das medidas de todas as faces de um poliedro convexo
com 10 vértices e 2880°.
Destas, é(são) correta(s) apenas
a) II. b) IV. c) II e IV. d) I, II e IV. e) II, III e IV.

09. Um poliedro convexo possui somente faces triangulares e


quadrangulares. Sendo o número de faces triangulares o dobro do
número de faces quadrangulares e o seu total de vértices igual a 30
pode-se afirmar que
a) o número de faces triangulares é igual a 6.
b) o número de faces quadrangulares é igual a 6.
c) o número de faces triangulares é igual a 8.
d) o número de faces quadrangulares é igual a 8.

10. (ITA) Um poliedro convexo de 16 arestas é formado por faces


triangulares e quadrangulares. Seccionando-o por um plano
convenientemente escolhido, dele se destaca um novo poliedro
convexo, que possui apenas faces quadrangulares. Este novo
poliedro possui um vértice a menos que o original e uma face a
mais que o número de faces quadrangulares do original. Sendo m
e n, respectivamente, o número de faces e o número de vértices do
poliedro original, então:
a) m = 9, n = 7 d) m = 10, n = 8
b) m = n = 9 e) m = 7, n = 9
c) m = 8, n = 10

GABARITO
EXERCÍCIOS DE FIXAÇÃO
01. A 04. C 07. C 10. B
02. B 05. C 08. A
03. D 06. A 09. D
EXERCÍCIOS DE TREINAMENTO
01. C 08. D 15. C 22. D
02. B 09. F = 10 16. B 23. A
03. E 10. 6FT e 3FQ 17. D 24. D
04. E 11. A = 8 18. D 25. C
05. V = 21 12. F = 9 19. C
06. V = 8 13. A = 35 20. A
07. A 14. A = 24 21. B
EXERCÍCIOS DE COMBATE
01. C 04. B 07. D 10. B
02. D 05. B 08. C
03. C 06. D 09. C

449

PM_BOOK16 - MAT.indb 449 25/11/2022 19:24:01


GEOMETRIA ESPACIAL: DIEDROS, TRIEDROS E POLIEDROS

ANOTAÇÕES

450

PM_BOOK16 - MAT.indb 450 25/11/2022 19:24:01


GEOMETRIA ESPACIAL:
PRISMAS E PIRÂMIDES

DEFINIÇÕES (PRISMAS) Prisma regular: prisma reto cujas bases são polígonos regulares.
Chama-se prisma todo poliedro convexo composto por duas
faces (bases) que são polígonos congruentes contidos em planos
paralelos e as demais faces (faces laterais) que são paralelogramos
determinados por pares de lados correspondentes nas duas faces.

Natureza de um prisma: o prisma será triangular, quadrangular,


pentagonal, etc, conforme a base seja um triângulo, quadrilátero,
pentágono, etc.

As arestas das bases são duas a duas congruentes e as arestas


laterais são todas congruentes entre si.
A altura do prisma é a distância entre os planos das duas bases.
Seção de um prisma é a interseção do prisma com um plano que
intercepte todas as arestas laterais.
A seção reta ou seção normal é a seção cujo plano é
perpendicular às arestas laterais.
Prisma reto: possui arestas laterais perpendiculares aos planos Paralelepípedo: prisma cujas bases são paralelogramos.
das bases e as faces laterais são retângulos.

Paralelepípedo reto: prisma reto cujas bases são paralelogramos.


Prisma oblíquo: as arestas laterais são oblíquas aos planos das
bases.

451

PM_BOOK16 - MAT.indb 451 25/11/2022 19:24:03


GEOMETRIA ESPACIAL: PRISMAS E PIRÂMIDES

Paralelepípedo reto-retângulo ou paralelepípedo retângulo Seja um prisma onde:


ou ortoedro: prisma reto cujas bases são retângulos. a → aresta lateral
2p → perímetro da seção reta
h → altura
SB → área da base

SL  2p  a

ST  2p  a  2  SB

V  SB  h

Hexaedro regular ou cubo: paralelepípedo retângulo cujas


arestas são todas congruentes.

No prisma reto, a aresta lateral é igual a altura (a = h) e a seção


reta é a própria base.

SL  2p  h

ST  2p  h  2  SB

V  SB  h

PARALELEPÍPEDO RETÂNGULO
Seja um paralelepípedo retângulo de dimensões a, b e c, temos: Seja um polígono contido em um plano e um ponto V localizado
fora desse plano. Uma Pirâmide é a reunião de todos os segmentos
D'
C' que têm uma extremidade em P e a outra num ponto qualquer do
polígono.
B' c
A'
d
D C
f1 b
A B
a

Base
D C
Seção diagonal
D' B'
b f1
c d

A a B D f1 B

Vértice: ponto V
Diagonal: 2
d  a b c 2 2 Base: polígono A1A2...An
Área total: ST = 2 · (ab + ac + bc) Altura: h (distância do vértice V ao plano da base).
Arestas da base: lados do polígono A1A2...An
Volume: V=a·b·c
Arestas laterais: VA1, VA2, ..., VAn
Faces laterais: triângulos formados pelo vértice e as arestas da base.
No caso particular do cubo de aresta a, a diagonal é d = a 3, a
Área lateral (Aℓ): área das faces laterais da pirâmide.
área total é ST = 6a² e o volume é V = a³.
Área total (At): área lateral mais a área da base.

ÁREA LATERAL, ÁREA TOTAL E At = Aℓ + Ab


VOLUME DO PRISMA
Área lateral (SL): área de todas as faces laterais. Natureza de uma pirâmide: está associada ao número de lados do
Área total (ST): área lateral mais a área das bases. polígono da base.

452

PM_BOOK16 - MAT.indb 452 25/11/2022 19:24:08


GEOMETRIA ESPACIAL: PRISMAS E PIRÂMIDES

Triangular (tetraedro) Quadrangular VOLUME


1
V  Ab  h
3

SEÇÃO TRANSVERSAL
Seção transversal de uma pirâmide é a interseção da pirâmide
com um plano paralelo à base da mesma.
outra pirâmide

seção transversal

base: triângulo base: quadrilátero

tronco de pirâmide
Pentagonal Hexagonal

Uma seção transversal divide a pirâmide em duas partes: a que


contém o vértice é uma pirâmide semelhante à original e a outra parte
é chamada tronco de pirâmide de bases paralelas.
Se a pirâmide possui altura h e a seção está a uma distância d do
vértice, a semelhança das duas pirâmides implica as seguintes relações:
Aresta pirâmide d
=
Aresta pirâmide maior h
base: pentágono base: hexágono Aresta pirâmide d
=
Aresta pirâmide maior h
2
Área da face lateral da pirâmide menor  d 
PIRÂMIDE REGULAR  
Área da face lateral da pirâmide maior  h 
Pirâmide regular é uma pirâmide cuja base é um polígono regular 2
e a projeção ortogonal do vértice sobre o plano da base é o centro Área da base da pirâmide menor  d 
 
da base. or  h 
Área da base da pirâmide maio
3
As arestas laterais são congruentes e as faces laterais são pirâmide d
= 
triângulos isósceles congruentes. pirâmide h
A obtenção das características da pirâmide menor possibilita
encontrar as características do tronco de pirâmide.

VOLUME DO TRONCO DE PIRÂMIDE


Sejam:
B → área da base maior.
b → área da base menor.
h → altura do tronco.

h
ab = r → ápotema da base (raio do círculo inscrito à base). V  B  B  b  b 
R → raio da circunferência circunscrita a base (OD na figura). 3

ap → apótema da pirâmide (altura relativa à base de uma face


lateral).
aℓ → aresta lateral.
TETRAEDRO REGULAR
Pirâmide triangular com todas as 6 arestas congruentes.
al2  ap2 R 2
a 6
Sendo a a aresta do tetraedro regular, sua altura é igual a e
3
ap2  ab2  h2 a3 2
seu volume é igual a .
12

453

PM_BOOK16 - MAT.indb 453 25/11/2022 19:24:14


GEOMETRIA ESPACIAL: PRISMAS E PIRÂMIDES

EXERCÍCIOS DE 10. Uma pirâmide com exatamente seis arestas congruentes é

FIXAÇÃO denominada tetraedro regular. Admita que a aresta do tetraedro


regular ilustrado a seguir, de vértices ABCD, mede 6 cm e que o ponto
médio da aresta BC é M.

01. Um cubo tem 216 cm² de área total. A medida, em cm, de sua
diagonal é
a) 6 2 b) 6 3 c) 2 6 d) 2 2 O cosseno do ângulo
AMDˆ equivale a:
02. A diagonal de um cubo mede 3 cm. O volume desse cubo, em 1
cm³, é a)
2
a) 9 b) 6 c) 3 3 d) 2 6 1
b)
3
03. Um cubo tem 3 cm de altura, e um paralelepípedo retângulo tem
2
dimensões 1 cm, 2 cm e 3 cm. A razão entre os volumes do cubo e c)
do paralelepípedo é 3
2
a) 3/2 b) 4/3 c) 9/2 d) 8/3 d)
5
04. Quatro cubos idênticos são dispostos como na figura a seguir,
formando um único sólido. Considerando que a diagonal de cada
EXERCÍCIOS DE
cubo mede 10 3 , a diagonal desse sólido é, em cm, igual a

a) 30 3 TREINAMENTO
b) 40 3
c) 20 01. A altura de um paralelepípedo retângulo mede 60 cm e sua base é
d) 30 um quadrado. A diagonal do paralelepípedo forma um ângulo de 60°
com o plano da base. O volume do paralelepípedo retângulo é em cm3
a) 12000 c) 24000
b) 18000 d) 36000

02. (ESPCEX) Aumentando-se em 10% as arestas da base e a altura


05. A pirâmide de Quéops, em Gizé, no Egito, tem aproximadamente
de uma pirâmide regular, seu volume será aumentado de:
90 2 metros de altura, possui uma base quadrada e suas faces laterais
são triângulos equiláteros. Nessas condições, pode-se afirmar que, em a) 10% d) 30%
metros, cada uma de suas arestas mede: b) 20% e) 33,1%
a) 90 c) 160 e) 200 c) 21%
b) 120 d) 180
03. (ESPCEX) A área da base de uma pirâmide quadrangular regular
06. Em uma pirâmide reta de base quadrada, de 4 m de altura, uma é 36 m². Se a altura da pirâmide mede 4 m, sua área total, em m², é
aresta da base mede 6 m. A área total dessa pirâmide, em m² é igual a:
a) 144 c) 48 e) 96 a) 48 d) 120
b) 84 d) 72 b) 54 e) 144
c) 96
07. Sejam duas pirâmides quadrangulares regulares de bases
congruentes, cujas alturas são 4 cm e 3 cm, e cujo apótema da base 04. (ESPCEX) Uma pirâmide quadrangular regular tem a por aresta
mede 4 cm. Unindo-se essas pirâmides pelas bases, de forma que suas da base e 2a por aresta lateral. A altura e o volume dessa pirâmide
arestas coincidam, obtém-se um octaedro cuja área total, em cm2, é medem, respectivamente:
igual a
a 15 a3 15 a 12 a3 12
a) (
8 5+ 2 ) c) (
16 5 + 2 2 ) a)
2
e
3
d)
2
e
3
b) (
8 5+4 2 ) d) (
16 5 + 4 2 ) b)
a 3 a3 3
e e)
a 10 a3 10
e
2 6 2 3
08. (EEAR) Um pedaço de queijo, em forma de prisma triangular a 14 a3 14
regular, tem 6 cm de altura e possui como base um triângulo de c) e
2 6
10 cm de lado. O volume desse pedaço de queijo é ____ 3 cm3.
a) 150 b) 165 c) 185 d) 200 05. (ESPCEX) Uma piscina em forma de paralelepípedo retângulo tem
largura de 6 metros, diagonal do fundo com 10 metros e diagonal
09. (EFOMM) Duas caixas cúbicas e retangulares perfeitas, têm seis da face que contém o comprimento igual a 4 5 metros. Para enchê-
faces de quadrados perfeitos. As faces da primeira caixa têm 3 m² de la com água será utilizado um caminhão tanque com capacidade de
área, e casa face da segunda caixa tem 9 m² de área. A razão entre o 6000 litros. O número de cargas completas, desse mesmo caminhão,
volume da primeira caixa e o volume da segunda é: necessárias para que a piscina fique completamente cheia é:
a) 31/2 c) 3-3/2 e) 3-2/3 a) 24 c) 32 e) 80
b) 3-1/2 d) 33/2 b) 28 d) 54

454

PM_BOOK16 - MAT.indb 454 25/11/2022 19:24:16


GEOMETRIA ESPACIAL: PRISMAS E PIRÂMIDES

06. (ESPCEX) Uma pirâmide hexagonal regular tem área da base igual 13. Um paralelepípedo retângulo de volume V tem dimensões
a 18 3 m2. Sabendo-se que sua altura é igual ao triplo do apótema da inversamente proporcionais a A, B e C. A área total do paralelepípedo é:
base, então seu volume é: 2VABC
a) d) 3 V(AB + AC + BC)
a) 36 m3 d) 54 3 m³ A +B+C
V2
b) 27 3 m³ V(A + B + C) e) 2(A + B + C) 3
e) 81 6 m³ b) ABC
ABC
c) 36 3 m³
c) 3
2V 2 (A + B + C)
07. Um prisma hexagonal regular ABCDEF – A’B’C’D’E’F’ tem todas as
arestas de mesmo comprimento (AA’ é uma aresta lateral). O cosseno 14. (AFA) A figura seguinte representa uma pirâmide regular de base
do ângulo CAˆ ′D é: quadrada, onde M é o ponto médio de DE e CM pertence ao plano da
2 3 5 base. Se DE = 100 m, AB = 10 m, AC = 12 m e AM = 28 m, então, o
a) c) e) 1
3 5 9 volume (em m3) de uma esfera cujo raio é da altura dessa pirâmide
é igual a 5
3 5
b) d)
4 8
08. (ESPCEX) Uma fábrica produz monitores para computador que
têm a forma de um bloco retangular associado a um tronco de
pirâmide, conforme o desenho e dimensões abaixo. Os monitores
são acondicionados para venda em caixas cúbicas, com aresta 40 cm,
medidos internamente. Os espaços vazios da caixa são preenchidos
com isopor, para proteger o aparelho. Sabendo que a produção diária
da fábrica é de 300 aparelhos, podemos dizer que o consumo diário
de isopor em metros cúbicos é de

a) 2
b) 3
a) 4500π
c) 4
b) 3375π
d) 5
c) 2200π
e) 6
d) 1125π

15. (AFA) Seja P uma pirâmide cujo vértice é o centro de uma das
faces de um cubo de aresta a e cuja base é a face oposta. Então, a área
lateral dessa pirâmide é igual a.
a2 5
a) a2 5 b) 2a2 3 c) a2 3 d)
4

16. A área total de uma pirâmide triangular regular é 36 3 cm2 e o


raio do círculo inscrito na base mede 2 cm. A altura da pirâmide é,
em cm:
a) 3 12 c) 4 3 e) 2 3
09. (AFA) Seja uma pirâmide de base quadrada com arestas de mesma
b) 2 15 d) 4
medida. O arco cosseno do ângulo entre as faces laterais que se
interceptam numa aresta é
17. (AFA) Uma pirâmide regular de 6 faces laterais tem sua base
a) –2/3 c) 1/3
inscrita num círculo de raio R. Sabendo-se que suas arestas laterais
b) –1/3 d) 2/3 têm comprimento L, então o volume dessa pirâmide é

10. (AFA) A área total da pirâmide regular de apótema A2, onde A1 e a) R2 3 (L2 − R2 )
2p são, respectivamente, apótema e perímetro de sua base, é
R2
a) p(A1 + A2) c) 2p(A1 + A2) b)
2
(L2
− R2 )
p  A 
b) ( A1 + A 2 ) d) p  A1 + 2 
2 R2
 2  c) 2 (L2 − R2 )
3
11. (AFA) Uma pirâmide regular, de base quadrada, tem altura igual
ao apótema da base. Então, o número de vezes que sua área lateral R2
d) 3 (L2 − R2 )
é a área da base é: 2
a) 2 2 c) 2
18. (AFA) O produto da maior diagonal pela menor diagonal de um
b) 2 d) √22 prisma hexagonal regular de área lateral igual a 144 cm2 e volume
igual a 3 144 3 cm é
12. (AFA) O volume, em cm3, do octaedro regular inscrito numa esfera
com volume 36π cm3 é a) 10 7 c) 10 21
2
a) 18 c) 54 2 3  m − 1
b) 20 7 d) πR  
b) 36 d) 72 3  m 

455

PM_BOOK16 - MAT.indb 455 25/11/2022 19:24:18


GEOMETRIA ESPACIAL: PRISMAS E PIRÂMIDES

19. (AFA) Um cubo tem quatro vértices nos pontos médios das arestas a) 7 <h< 8 c) 2 3 <h<3 3 e) 2 2 <h<3 2
laterais de uma pirâmide quadrangular regular, e os outros quatro na
base da pirâmide, como mostra a figura abaixo. b) 6 <h< 7 d) 1< h < 2

A razão entre os volumes do cubo 27. (ESPCEX) Considere um prisma hexagonal regular tal que a razão
e da pirâmide é 3
entre a aresta da base a e a aresta lateral  é . Sabendo-se que a
a) 3/4 3
b) 1/2 aresta da base for aumentada de 2 cm, o volume V do prisma ficará
aumentado de 108 cm3 considerando que aresta lateral permanece a
c) 3/8 mesma, podemos afirmar que o volume do prisma é:
d) 1/8 a) 10 cm3 c) 3/2 cm3 e) 27/2 cm3
b) 12 cm 3
d) 36 cm 3

28. (EN) Um depósito de óleo diesel existente em uma das


Organizações Militares da MB tem a forma de um prisma hexagonal
regular com altura de 2 metros. Sabendo-se que o comprimento da
2 30
diagonal maior do depósito vale do comprimento da diagonal
9
20. (ESPCEX) Determine o volume (em cm³) de uma pirâmide menor da base, pode-se dizer que o valor da função f, definida por
1
retangular de altura “a” e lados da base “b” e “c” (a, b e c em −

centímetros), sabendo que a + b + c = 36 e “a”, “b” e “c” são, f(x) = 2x 3 no número V representante do volume do depósito vale:
respectivamente, números diretamente proporcionais a 6, 4 e 2. 6
3 6
243 6
243
a) 2 c) 2 e) 2
a) 16 c) 108 e) 648 9 9 3
b) 36 d) 432 3 6
243
b) 2 d) 2
9 5
21. (AFA) Se uma pirâmide hexagonal regular está inscrita num cone
29. Em uma pirâmide regular, de base hexagonal, o apótema da base
equilátero cujo volume é igual a 10 3 π cm3 , então o volume dessa mede 1 cm. Se a altura da pirâmide mede o dobro da medida da
pirâmide, em cm³, é igual a 7 diagonal de um cubo de 8 cm3 de volume, então a razão entre a área
45 135 lateral da pirâmide e a área total do cubo vale:
a) b) 15 3 c) 30 3 d)
7 7 7 7 3 3 5 3 e) 2 3
a) c)
16 6
22. (ESPCEX) As medidas das arestas de um paralelepípedo retângulo 13 3
7 3 d)
são diretamente proporcionais a 3, 4 e 5 e a soma dessas medidas é b)
12 12
igual a 48 cm. Então a medida da sua área total, em cm², é
a) 752 c) 1.024 e) 1.504 30. Seja V o vértice de uma pirâmide com base triangular ABC. O
b) 820 d) 1.302 segmento AV, de comprimento unitário, é perpendicular à base. Os
ângulos das faces laterais, no vértice V, são todos de 45°. Deste modo,
23. (EN) Em um polígono regular, cujos vértices A, B e C são o volume da pirâmide será igual a:
consecutivos, a diagonal AC forma com o lado BC um ângulo de 30°. 1 1
a) 2 2−2 c) 2− 2
Se o lado do polígono mede  unidades de comprimento, o volume 6 3
da pirâmide, cuja base é esse polígono e cuja altura vale o triplo da 1 1
medida do lado, é igual a b) 2− 2 d) 2 2 −1
6 6
3 3 3
3 3 c)  3 e) 3 3
a)
2 2 3

b) 32 3 d) 3 3 EXERCÍCIOS DE
2 4

24. (AFA) Considere uma pirâmide regular ABCDV de base ABCD.


COMBATE
Sendo 2 2 cm a medida da aresta da base e 2 3 cm a medida da
altura dessa pirâmide, a distância, em cm, de A à aresta lateral VC é
01. Em cm3, qual o volume de um paralelepípedo retângulo de área
a) 2 2 b) 2 3 c) 4 d) 3 total 180 cm3, de diagonal da base 10 cm e com a soma das arestas
que concorrem no mesmo vértice igual a 17 cm?
25. (EN) Num prisma hexagonal regular a área lateral é 75% da área a) 99 b) 120 c) 135 d) 144
total. A razão entre a aresta lateral e a aresta da base é

a) 2 5 c) 5 3 e) 5 2 02. O produto da maior diagonal pela menor diagonal de um prisma


3 2 3 hexagonal regular de área lateral igual a 144 cm² e volume igual a
3 3 2 3 144 3 cm3 é:
b) d)
2 5 a) 10 7 b) 20 7 c) 10 21 d) 20 21
26. Um tronco de pirâmide regular tem como bases triângulos
equiláteros, cujos lados medem, respectivamente, 2 cm e 4 cm. Se a 03. A área da base de uma pirâmide quadrangular regular é 36 m².
aresta lateral do tronco mede 3 cm, então o valor de sua altura h, em Se a altura da pirâmide mede 4 m, sua área total, em m², é igual a:
cm, é tal que: a) 48 b) 54 c) 96 d) 120

456

PM_BOOK16 - MAT.indb 456 25/11/2022 19:24:23


GEOMETRIA ESPACIAL: PRISMAS E PIRÂMIDES

04. Uma pirâmide quadrangular regular tem as oito arestas iguais a 10. (EN) Uma pirâmide triangular tem como base um triângulo de
2 m. O volume dessa pirâmide vale: lados 13 cm, 14 cm e 15 cm; as outras arestas medem I. Sabendo
2 3 4 3 que o volume da pirâmide é de 105 22 cm3, o valor de I, em cm,
a) 1 m³ b) 2 m³ c) m d) m é igual a:
3 3
155 275 95
05. O apótema de uma pirâmide regular, com base hexagonal, é 9 3 a) c) e)
8 9 8
cm. Se a sua área lateral é o triplo da área de sua base, então, o seu
volume, em cm3, é 335 205
b) d)
3 323 81 35 11 8
a) b) c) 81 3 d) 324 2
4 4

06. (ESPCEX) Um prisma reto com 5 cm de altura e base retangular


com dimensões de 4 cm e 6 cm contém água até uma altura de 3
cm. Um cubo maciço de aresta igual a 2 cm é colocado dentro deste
prisma, ficando totalmente submerso. A partir de então, a altura do
nível da água, em cm, passa a ser de:
DESAFIO PRO
1
a) 13/4 c) 15/4 e) 14/4 (EN) Uma pirâmide triangular tem como base um triângulo
b) 10/3 d) 13/3 de lados 13 cm, 14 cm e 15 cm; as outras arestas medem ℓ.

Sabendo que o volume da pirâmide é de 105 22 cm3 , o valor


07. (ESPCEX) Em um cubo de aresta medindo 4 cm, forma-se um
de ℓ, em cm, é igual a:
triângulo VEF, conforme figura abaixo, em que V é o centro do
quadrado ABCD. A área, em cm², do triângulo VEF é igual a a) 155 c)
275
e)
95
8 9 8
a) 4√5 335 205
b) d)
b) 4√6 11 8

2
c) 5√5 (IME) Em um prisma oblíquo ABCDEFA’B’C’D’E’F’, cuja
d) 5√6 base ABCDEF é um hexágono regular de lado a, a face
e) 6√6 lateral EFF’E’ está inclinada 45° em relação à base, e a projeção
ortogonal da aresta F’E’ sobre a base ABCDEF coincide com a
aresta BC. O volume do prisma é:

08. (ESPCEX) Para obter o sólido geométrico representado abaixo, 3 3 3 5 3 3 5 3


a) a c) a e) a
partiu-se de um cubo de aresta L e retirou-se de cada um dos vértices 2 3 2
desse cubo uma pirâmide de base triangular com as arestas laterais 9 3 9 3
medindo L/4, conforme a figura. Denominando-se V o volume do b) a d) a
4 2
cubo a partir do qual foi obtido o sólido, pode-se concluir que o
volume desse sólido é
3 (ITA) Considere a classificação: dois vértices de um
paralelepípedo são não adjacentes quando não pertencem
à mesma aresta. Um tetraedro é formado por vértices não
a) 23 adjacentes de um paralelepípedo de arestas 3 cm, 4 cm e 5 cm.
V
24 Se o tetraedro tem suas arestas opostas de mesmo comprimento,
47 então o volume do tetraedro é, em cm³:
b) V
48 a) 10. c) 15. e) 30.
71 b) 12. d) 20.
c) V
72

d) 95
96
V 4 (IME) Um tronco de pirâmide regular possui 12 vértices. A
soma dos perímetros das bases é 36 cm, a soma das áreas
das bases é 30 3 cm2 e sua altura mede 3 cm. Calcule o volume
e) 143 do tronco de pirâmide.
V
144 a) 50 cm³

09. (ESPCEX) Considere um prisma regular reto de base hexagonal 3


b) 42 cm3
3
tal que a razão entre a aresta da base e a aresta lateral é 3 .
3 3
c) 43 cm3
Aumentando-se a aresta da base em 2 cm e mantendo-se a aresta 2
lateral, o volume do prisma ficará aumentado de 108 cm³. O volume
d) 43 2 cm3
do prisma original é
a) 18 cm³ e) 42 3 cm3
b) 36 cm³
c) 18√3 cm³
d) 36√3 cm³
5 (ITA) Na construção de um tetraedro, dobra-se uma folha
retangular de papel, com lados de 3 cm e 4 cm, ao longo de
uma de suas diagonais, de modo que essas duas partes da folha
e) 40 cm³ formem um ângulo reto e constituam duas faces do tetraedro.
Numa segunda etapa, de maneira adequada, completa-se com
outro papel as faces restantes para formar o tetraedro. Obtenha
as medidas das arestas do tetraedro.

457

PM_BOOK16 - MAT.indb 457 25/11/2022 19:24:29


GEOMETRIA ESPACIAL: PRISMAS E PIRÂMIDES

GABARITO
EXERCÍCIOS DE FIXAÇÃO
01. B 04. D 07. D 10. B
02. C 05. D 08. A
03. C 06. E 09. C
EXERCÍCIOS DE TREINAMENTO
01. E 09. B 17. D 25. B
02. E 10. A 18. D 26. A
03. C 11. C 19. C 27. D
04. C 12. B 20. D 28. C
05. C 13. E 21. A 29. B
06. D 14. A 22. E 30. A
07. D 15. A 23. A
08. E 16. E 24. B
EXERCÍCIOS DE COMBATE
01. D 04. C 07. A 10. B
02. D 05. D 08. B
03. C 06. B 09. B
DESAFIO PRO
01. A 03. D 05. 337
cm.
5
02. D 04. E

ANOTAÇÕES

458

PM_BOOK16 - MAT.indb 458 25/11/2022 19:24:29


GEOMETRIA ESPACIAL:
CONE E CILINDRO

CILINDRO CIRCULAR
Considere dois planos paralelos, α e β, seja R um círculo no plano
α, seja s uma reta secante aos dois planos que não intersecta R, o
conjunto de todos os segmentos de retas paralelos a s que contém
um ponto de R e um ponto no plano paralelo β formam um cilindro.

 cilindro reto cilindro oblíquo


B
d
e
Observação
s
C No cilindro circular reto a geratriz é igual à altura. No cilindro
a circular oblíquo a geratriz é maior que a altura.
A
R

VOLUME
O volume do cilindro é calculado pelo produto da área da sua base
Todo cilindro possui três superfícies, as duas bases e a superfície pela sua altura.
lateral. O eixo do cilindro é a reta que passa pelos centros das suas
V = AB · h = πr2h
bases, na figura acima a reta que passa pelos pontos A e B. Se o eixo
for perpendicular às bases o cilindro será circular reto, caso contrário,
será um cilindro oblíquo.

DEFINIÇÕES
Superfície cilíndrica de revolução é a superfície gerada pela
rotação de uma reta g (geratriz) ao redor de uma reta fixa paralela OO’
(eixo). A distância constante entre g e OO’ é o raio. A circunferência
descrita por qualquer ponto de D é um paralelo. Qualquer plano que
passe por OO’ é chamado plano meridiano.
Seção reta é a seção da superfície cilíndrica por qualquer plano
perpendicular às geratrizes.

ÁREA LATERAL E ÁREA TOTAL DO


CILINDRO CIRCULAR RETO
Planificando o cilindro circular reto obtemos:

h
h
2r

Um cilindro é reto ou oblíquo conforme suas bases sejam seções


retas ou oblíquas da superfície cilíndrica. r

459

PM_BOOK16 - MAT.indb 459 25/11/2022 19:24:31


GEOMETRIA ESPACIAL: CONE E CILINDRO

Portanto sua área lateral do cilindro circular reto é igual a área do CONE CIRCULAR
retângulo de dimensões 2 · π · r e h. Considere um círculo R contido num plano α, e B um ponto que
O comprimento do retângulo 2 · π · r é igual ao perímetro da não está em α. O conjunto de todos os segmentos de reta que unem
os pontos de R ao ponto B formam um cone circular.
circunferência da base do cilindro.
Portanto a área lateral é dada por AI = 2 · π · r · h.
E a sua área total é dada pela soma da área lateral com as áreas
das duas bases.
Área total = At = AI + 2Ab - 2 · π · r · h + 2 · π · r2 = 2 · π · r ·(r + h). B

SEÇÃO MERIDIANA DE UM CILINDRO h

CIRCULAR (QUADRILÁTERO ABCD) A


É a interseção do cilindro com um plano que contém a reta que R
liga os centros das bases (OO’). 

A reta que passa pelos pontos A, centro da base, e o ponto B, é


o eixo do cone.
No caso o segmento AB é perpendicular à base, o cone é circular
reto, caso contrário será oblíquo.

DEFINIÇÕES

B
No cilindro oblíquo a seção meridiana é um paralelogramo e no
cilindro reto é um retângulo.

CILINDRO EQUILÁTERO
O cilindro que possui as seções meridianas quadradas é chamado
de cilindro equilátero. No cilindro equilátero a altura é igual ao h g
diâmetro da base.

A D
C r R

No cone circular reto o segmento que une os pontos A e B é


chamado de altura e o segmento que une os pontos B e C é chamado
de geratriz, B é o vértice do cone e o círculo R é a base do cone, a
distância de A até C é o raio da base.
h = 2r No cone circular reto vale a seguinte relação g² = r² + h²

Como consequências imediatas, temos para o cilindro equilátero


Área lateral – SL

SL = 2πrh =2πr ⋅ 2r =4 πr 2

Área total – ST

ST = 2SB + SL = 2πr 2 + 4 πr 2 = 6πr 2

Volume – V

V=
πr2h =
πr2 ⋅ 2r =
2πr3

460

PM_BOOK16 - MAT.indb 460 25/11/2022 19:24:32


GEOMETRIA ESPACIAL: CONE E CILINDRO

CONE OBLÍQUO

Assim
g = 2r

CONE CIRCULAR RETO OU CONE DE REVOLUÇÃO


O cone circular reto é criado pela rotação de um triângulo
retângulo em torno de um dos seus catetos. Este é o motivo pelo qual
a superfície de um cone recebe o nome de superfície de revolução.

Dessa forma, sendo nossa seção meridiana um triângulo


equilátero, teremos que:

g = 2r

g 3 2r 3
=h = = r 3
2 2

Assim teremos também:

SL =πrg =πr ⋅ 2r =2πr 2


SEÇÃO MERIDIANA E CONE EQUILÁTERO
Seção meridiana de um cone é a intersecção dele com um plano 1 1 2 1 2 πr 3 3
que contém o eixo. A seção meridiana de um cone reto é um triângulo V= ⋅ SL ⋅ h= ⋅ πr ⋅ h= ⋅ πr ⋅ r 3 =
3 3 3 3
equilátero.

ÁREA LATERAL (cone reto):

Alateral = πrg

ÁREA TOTAL (cone reto): Atotal = Alateral + Abase = πr(g + r)


Cone equilátero é um cone cuja seção meridiana é um triângulo 1 1
equilátero. VOLUME (reto ou oblíquo): V  Abase  h  r2h
3 3

461

PM_BOOK16 - MAT.indb 461 25/11/2022 19:24:34


GEOMETRIA ESPACIAL: CONE E CILINDRO

SÓLIDOS DE REVOLUÇÃO Ao rotacionarmos o retângulo ABCD acima em torno de um de


Sólidos de revolução são sólidos circulares formados pela rotação seus lados, por exemplo o lado AD , teremos que a área do retângulo
de 360° de uma superfície plana (polígono qualquer) de área S em ABCD irá “varrer” uma região no espaço.
volta de um eixo de rotação previamente definido.
Vamos ilustrar aqui diferentes sólidos de revolução.
Para efeito do nosso estudo vale ressaltar que para polígonos que
possuem simetria em relação a um eixo, como triângulos equiláteros,
quadrados, retângulos, será indiferente rotacioná-los integralmente
ou apenas uma das “metades” em relação ao eixo de simetria.
Por exemplo, rotacionar todo o triângulo isósceles ABC abaixo em
relação a altura AH é o mesmo que rotacionar o triângulo ABH em
relação ao lado AH .

Assim o resultado será o cilindro reto de raio das bases CD


(segmento que ficou na horizontal) e altura AD (segmento que ficou
na vertical).

Os triângulos ABH e ACH são congruentes, daí rotacionar ABH ou


ABH ou até mesmo ACH em torno da reta suporte de AH resultará
no mesmo sólido de revolução que será um cone reto.

CILINDRO RETO
O cilindro reto é resultado da rotação de um retângulo em torno
de algum de seus lados.

( )
2 2
O volume desse sólido resultante será V = π AB ⋅ AD = πAB ⋅ AD

CILINDRO EQUILÁTERO
O cilindro equilátero é o resultado da rotação de um quadrado
em relação ao seu eixo de simetria (reta que liga os pontos médios
dos lados).

462

PM_BOOK16 - MAT.indb 462 25/11/2022 19:24:34


GEOMETRIA ESPACIAL: CONE E CILINDRO

( )
2 2
O seu volume é dado por V = π AB ⋅ AD = πAB ⋅ AD , como
AD = 2AB teremos
2 3
V=
πAB ⋅ 2AB =
2πAB

CONE RETO
O cone reto é resultado da rotação de um triângulo retângulo em
torno de um de seus catetos.

Ou do retângulo que possua uma dimensão igual ao dobro da


outra, sendo rotacionado sobre a reta suporte do maior dos lados.

Assim o resultado será o cone reto de raio da base BC (segmento


que ficou na horizontal) e altura AC (segmento que ficou na vertical).

2
1 BC ⋅ AC ⋅ π
( )
2
O seu volume é dado por V = ⋅ π BC ⋅ AC = .
3 3
Um detalhe interessante é que o lado AB é o responsável por
formar a geratriz do cone.

463

PM_BOOK16 - MAT.indb 463 25/11/2022 19:24:35


GEOMETRIA ESPACIAL: CONE E CILINDRO

CONE EQUILÁTERO
O cone equilátero é o resultado da rotação de um triângulo
equilátero em relação a sua altura.

2
1 BC ⋅ AC ⋅ π
( )
2
O seu volume é dado por V = ⋅ π BC ⋅ AC = , como
3 3
AC = 3.BC
2 3
BC ⋅ 3 ⋅ BC ⋅ π BC ⋅ 3π
V= =
3 3

DOIS CONES RETOS


Até aqui vimos apenas sólidos de rotação resultantes de rotações
de polígonos em relação a um eixo de simetria perpendicular a um
de seus lados. Vamos pensar por exemplo se fizermos a rotação do
triângulo ABC abaixo em relação a reta suporte do lado BC , o que
vamos obter?

Ou do triângulo retângulo que possua um cateto medindo 3


vezes a medida do outro cateto, sendo rotacionado sobre o maior
dos catetos.

Vamos traçar a altura relativa ao lado BC e olhar novamente.

464

PM_BOOK16 - MAT.indb 464 25/11/2022 19:24:36


GEOMETRIA ESPACIAL: CONE E CILINDRO

Cada um dos triângulos, ABH e ACH, será responsável por Para isso utilizaremos as relações métricas no triângulo retângulo.
“varrer” uma região que delimitará um cone reto cada.
12
AC ⋅ BH = AB ⋅ BC ⇒ 5 ⋅ h = 4 ⋅ 3 ⇒ h = cm
5

Assim ao rotacionarmos o triângulo ABC teremos como resultado


os 2 cones retos abaixo.

O cone gerado pelo triângulo ABH (hachurado de azul) possuirá


raio da base AH e altura BH enquanto o cone gerado pelo triângulo
ACH (hachurado de vermelho) possuirá raio da base AH e altura CH.
Assim teremos 2 cones formados.

Perceba que não precisamos encontrar as medidas de AH e CH,


mesmo sendo possível também utilizando relações métricas no
triângulo retângulo, pois para o cálculo volume só será necessário a
medida de todo o lado AC .
2
1  12  1 144 48π
V=
3  5 
(
⋅   ⋅ AH + CH = ⋅)
3 25
⋅ 5 ⋅ π=
5
cm3

Vamos ver agora alguns exemplos de sólidos de revolução


“vazados”, aqueles que temos de imaginar um sólido maior sendo
O volume será dado pela soma dos volumes dos 2 cones, assim
diminuído de um ou mais sólidos menores.
2
1 1 π ⋅ BH Exemplo. Calcule o volume do sólido de revolução gerado pela
( ) ( ) ( )
2 2
V= V1 + V2= ⋅ π ⋅ BH ⋅ AH + ⋅ π ⋅=
BH ⋅ CH AH + CH
3 3 3 rotação de 360° do triângulo ABC em relação ao lado AB sabendo
que AB = 4 cm , BC = 6 cm e AB̂C
= 150° .
= AH + CH teremos que
porém, como AC
2
π ⋅ BH ⋅ AC
V=
3

Exemplo 1. Calcular o volume do sólido obtido pela rotação de


360° do triângulo retângulo de lados medindo 3 cm, 4 cm e 5 cm em
torno da reta suporte da hipotenusa.
Primeiramente deveremos encontrar a medida da altura relativa a
hipotenusa BH .

465

PM_BOOK16 - MAT.indb 465 25/11/2022 19:24:37


GEOMETRIA ESPACIAL: CONE E CILINDRO

Primeiramente teremos que encontrar a altura relativa ao lado AB, Como AH − BH =


AB , teremos
ou a projeção do vértice C sobre o eixo de rotação. 1 π 2
⋅ π ⋅ ( CH) ⋅ AB=
2
V= ⋅ 3 ⋅ 4= 12π cm3
3 3

Geralmente teremos sólidos vazados quando a rotação se der


em torno de uma reta paralela a um dos lados do polígono. Vejamos
alguns exemplos.
I. Sólido de revolução gerado pela rotação de um retângulo ABCD,
em torno de uma reta r, paralela a AB e distando d de AB.

Aplicando razões trigonométricas no triângulo BCH podemos


encontrar o valor de CH .
CH 1 CH
sen30=
° ⇒= ⇒ CH
= 3
BC 2 6

Assim quando fizermos a rotação do triângulo ABC veremos que


o sólido gerado é resultado de um cone reto “maior”, gerado pelo
triângulo ACH diminuído de um cone reto “menor” gerado pela
rotação do triângulo BCH. Onde dizemos que o volume gerado pela
rotação de BCH é um volume não útil.

Assim o volume procurado é o volume do cone ACC’ subtraído


do cone BCC’.
1 1
⋅ π ⋅ ( CH) ⋅ AH − ⋅ π ⋅ ( CH) ⋅ BH
2 2
V= VACC′ − VBCC′=
3 3

1
⋅ π ⋅ ( CH) ( AH − BH)
2
V=
3

466

PM_BOOK16 - MAT.indb 466 25/11/2022 19:24:37


GEOMETRIA ESPACIAL: CONE E CILINDRO

O sólido gerado é um cilindro “furado”, ou um anel cilíndrico. 04. Um reservatório de água tem formato de um cilindro circular
O volume final será o volume do cilindro maior BCC’B’ subtraído do reto, com altura de 6 m e raio de 1,5 m (medidas internas). Deseja-se
cilindro menor ADD’A’. ampliar a capacidade do reservatório em dois terços, sem modificar
seu diâmetro. Pode-se afirmar que a altura do reservatório, para que
V = Vmaior − Vmenor = π ( d + AB) BC − π ( d) BC
2 2
essa ampliação seja realizada, deve ser:
a) triplicada.
V = π ⋅ BC ⋅ ( d + AB) − d2  = π ⋅ BC ⋅ d2 − 2d ⋅ AB + ( AB) − d2 
2 2
    b) dobrada.
1
c) aumentada em.
V = π ⋅ BC ⋅ ( AB) − 2d ⋅ AB
2
  3
5
d) aumentada em .
Ou pensando apenas que o cilindro maior terá raio R e o menor 3
r e ambos altura H 2
e) aumentada em .
V= πH R2 − r2 ( ) 3
05. O volume de uma lata cilíndrica é de 540 mililitros. Se a altura da
lata é de 20 centímetros, qual é o diâmetro de sua base? (dados: π = 3)

EXERCÍCIOS DE
a) 3 cm

FIXAÇÃO
b) 6 cm
c) 9 cm
d) 12 cm

01. Um cilindro possui base de raio 4 cm e altura 12 cm. Seu volume, 06. Um cilindro reto apresenta diâmetro de 8 cm e altura de 25 cm
em cm³, é: possui um volume de:
Dado: considere π = 3 a) 16π cm3 d) 280π cm3
a) 48.
b) 124 π cm3 e) 120π cm3
b) 335.
c) 400π cm3
c) 478.
d) 576.
07. Certo suco é vendido em latinhas de alumínio, no formato de
cilindro. Cada latinha contém 270 mL de suco, o que corresponde a
02. O cone é um sólido geométrico obtido pela rotação de um 9
triângulo. Com base na figura a seguir, é correto deduzir que a do volume total da latinha, se utilizado π = 3. Se o diâmetro da
10
fórmula do seu volume (V) é dada por latinha é de 6 cm, e cada cm³ corresponde a 1 mL, então a altura de
cada latinha é de, aproximadamente,
a) 8 cm. d) 11 cm.
b) 9 cm. e) 12 cm.
c) 10 cm.
g
08. Considerando-se que certo cilindro possui volume igual 785 cm³
h
e altura igual a 10 cm, assinalar a alternativa que apresenta o valor do
raio da base desse cilindro: (considerar π = 3,14)
a) 25 cm
b) 7 cm
c) 6 cm
r
d) 5 cm
e) 14 cm
a) V = πr 2h
1 2 09. Qual será o volume de um cone de diâmetro de base igual a 6 cm
b) V= πr h e geratriz de 5 cm?
3
1 2 a) 12π cm
c) V= πr g
3 b) 4 π cm2
d) V = πr g
2
c) 12π cm3

d) 4 π cm3
03. Se um cone tiver o raio da sua base aumentado em 20%, seu
volume aumentará em
10. Um cilindro tem raio da base medindo 4 cm e altura 6 cm.
a) 20%. A metade do seu volume, em cm3, é:
b) 36,4%. a) 12π d) 36π
c) 44%. b) 18π e) 48π
d) 72,8%. c) 24π

467

PM_BOOK16 - MAT.indb 467 25/11/2022 19:24:38


GEOMETRIA ESPACIAL: CONE E CILINDRO

EXERCÍCIOS DE 06. (EEAR) Um plano determina dois semicilindros quando secciona

TREINAMENTO
um cilindro reto de 2,5 cm de altura e 4 cm de diâmetro da base,
passando pelos centros de suas bases. A área total de cada um desses
semicilindros, em cm2, é aproximadamente igual a
a) 28. c) 38.
01. Qual será o volume total (V) de água, em litros, que poderá ser b) 30. d) 40
armazenado nesta cisterna?
Considere π = 3,14 2
07. (EEAR) Um cilindro circular reto, de altura igual a do raio da
3
r=1m base e de 12π cm² de área lateral, possui raio da base igual a _____
cm.
a) 5 c) 3
h=3m b) 4 d) 2

08. (EEAR) Um cilindro de 18 cm de altura e raio da base igual a 5 cm


contém água até a metade de sua altura. Por algum motivo, houve
necessidade de despejar essa água em outro cilindro com 40 cm de
a) V = 0,942 litros. altura, cujo raio da base mede 4 cm.
b) V = 942 litros.
c) V = 9,42 litros.
d) V = 9420 litros.

02. Maria adquiriu uma piscina no formato de um cilindro reto


com 1,20 metros de profundidade e 6 metros de diâmetro. Sabe-se
que para encher 1 m³ são necessários 1.000 litros de água, sendo
assim, quantos litros de água serão necessários para encher 75% da
capacidade total dessa piscina? (Considere π = 3)
a) 8.100 litros.
b) 12.600 litros.
Considerando π = 3, o valor que mais se aproxima da altura atingida
c) 16.200 litros. pela água no segundo cilindro é
d) 24.300 litros. a) 14 cm
e) 32.400 litros. b) 16 cm
c) 20 cm
03. Qual o volume de um cilindro circular reto, em cm³, cujo diâmetro
é de 12 cm e altura de 15 cm? (utilizar π = 3,14) d) 24 cm
a) 565,2.
09. (ESA) Dobrando o raio da base de um cone e reduzindo sua altura
b) 1130,4. à metade, seu volume
c) 1695,6. a) dobra.
d) 2260,8. b) quadruplica.
e) 2826. c) não se altera.
d) reduz-se à metade do volume original.
04. Os volumes de dois cilindros estão, entre si, na razão 27. Calcule
o raio do maior cilindro, sabendo que o raio do menor mede 3 cm. e) reduz-se a um quarto do volume original.
Assinale a alternativa CORRETA.
a) R = 4 cm 10. (EEAR) A base de um cone circular reto está inscrita num triângulo
equilátero de área 9 3 cm² . Se as alturas do cone e do triângulo são
b) R = 6 cm congruentes, então o volume do cone, em cm³, é
c) R = 9 cm
a) 3π 6
d) R = 12 cm
b) 3π 3
e) R = 15 cm
c) 6π 3
05. Dobrando-se a altura de um cilindro circular reto e triplicando o d) 6π 6
raio de sua base, pode-se afirmar que seu volume fica multiplicado por
a) 6 11. (EEAR) Um chapéu de festa, feito de cartolina, tem a forma de um
b) 9 cone de 1 dm de raio e 5 dm de geratriz. Para fazer 20 chapéus, são
necessários, no mínimo, ________ dm² de cartolina.
c) 12
Considere π = 3,14.
d) 18
a) 157
e) 36
b) 225
c) 314
d) 426

468

PM_BOOK16 - MAT.indb 468 25/11/2022 19:24:38


GEOMETRIA ESPACIAL: CONE E CILINDRO

12. (EEAR) Um cone tem 3 cm de altura 8π cm³ de volume. O raio da 16. Em um cilindro reto, a área lateral é 64π cm². A medida da altura
base desse cone, em cm, é (h) é igual ao dobro da medida dos raios (r) das bases. A altura (h) e o
a) 4 raio (r) são, respectivamente
b) 6 a) 8 e 4
c) 3 2 b) 4 e 2
d) 2 2 c) 4e8
d) 3 e 6
13. Um copo pequeno foi preenchido completamente com água. Em e) 2 e 4
seguida, todo o líquido foi despejado em um copo grande, inicialmente
vazio. As medias dos copos estão indicadas na figura abaixo: 17. (CFN) Em um cilindro circular reto qualquer, a altura é duas vezes
o tamanho do diâmetro da base. Determine o volume deste sólido
sabendo-se que sua base está inscrita em um quadrado de lado igual
a 6 cm.
a) 72π cm³
b) 108π cm³
c) 144π cm³
d) 216π cm³
e) 432π cm³

18. (EEAR) A figura é composta de um cone e um cilindro, ambos


retos e de mesma base, que estão justapostos. Considerando as
dimensões dadas, a área total da superfície da figura é πcm².
a) 144
DADO: π = 3,14
b) 96
Após a água ser transferida para o copo grande, qual foi a altura h
aproximada atingida pela água? c) 84
a) 5,4 cm. d) 68
b) 6,2 cm.
19. (EEAR) Uma caixa cúbica, de aresta 10 cm, está totalmente cheia
c) 7,7 cm. de água. Ao despejar toda a água num tubo cilíndrico de 5 cm de
d) 8,5 cm. raio, essa água atingirá a altura de ____/ π cm no tubo. (Considere as
dimensões como sendo internas aos recipientes e que o tubo tem a
14. Um reservatório em formato cilíndrico encontrava-se altura necessária para o evento.)
completamente cheio de água. Essa água foi distribuída em baldes, a) 50
1
também cilíndricos, cujo raio da base é do raio da base do b) 40
4
reservatório. Sabendo-se que foram necessários 128 baldes para c) 35
esgotar toda a água desse reservatório a altura de cada balde, em d) 25
relação à altura do reservatório, é
1 20. (EEAR) Considere um cilindro reto, cuja altura é igual ao diâmetro
a) da base e a área da seção perpendicular às bases, contendo os centros
10
destas bases, mede 32 cm².
1
b) Baseando-se nessas informações, calcule a área da base desse cilindro
12
e assinale a opção correta.
1
c) a) 8π cm².
9
b) 16π cm².
1
d) c) 24π cm².
15
1 d) 32π cm².
e)
8
21. (PMES) O volume do sólido gerado pela rotação de um triângulo
15. Uma piscina cilíndrica (cilindro circular reto), com raio igual a 25 isósceles de lados congruentes medindo 5 cm e base medindo 6 cm,
m e profundidade igual a 8% do raio, foi destruída para que outra em torno da base é igual a
pudesse ser construída em seu lugar, com o intuito de se realizar a) 8π cm³
uma competição de natação. A nova piscina tem o formato de um
b) 16π cm³
paralelepípedo reto-retangular e possui o mesmo volume da piscina
anterior. A profundidade da piscina é de π m e seu comprimento é c) 24π cm³
igual ao valor de sua largura acrescido de 25 m. Com base nessa d) 32π cm³
situação hipotética, é correto afirmar que a nova piscina tem e) 64π cm³
a) 25 m de largura.
b) 30 m de largura.
c) 35 m de largura.
d) 50 m de largura.
e) 60 m de largura.

469

PM_BOOK16 - MAT.indb 469 25/11/2022 19:24:39


GEOMETRIA ESPACIAL: CONE E CILINDRO

22. (BARRO BRANCO) Uma garrafa de vidro tem a forma de dois 25. (ESPCEX) O volume de um cilindro equilátero de 1 metro de raio
cilindros sobrepostos, ambos com 8 cm de altura e bases com raios R é, aproximadamente, igual a:
e r, conforme mostra a figura. a) 3,1 m3
b) 6,3 m3
c) 9,4 m3
d) 12,6 m3
e) 15,7 m3

26. (ESPCEX) O sólido geométrico abaixo é formado por dois cones


circulares retos de mesma base. Sabendo-se que a seção que contém
os pontos A e B é paralela à base comum dos cones e divide todo o
sólido em duas partes de igual volume, então o valor de x³ + y³ é:

B
O volume da água, quando seu nível atinge 6 cm de altura, é igual a A x
96 π cm³. Quando totalmente cheio, o volume da água é igual a 178π
cm³. Desse modo, é correto afirmar que R e r medem, em centímetros, 6
respectivamente,
Y
a) 4,0 e 2,0.
b) 4,0 e 2,5.
c) 5,0 e 3,0.
a) 96
d) 6,25 e 4,0.
b) 128
e) 6,25 e 4,5.
c) 144
23. A geratriz (g) do cone circular reto mostrado na figura abaixo d) 162
mede 4 3 cm. O volume desse cone, em cm³, é igual a e) 248

27. Considere a sequência infinita de cilindros circulares retos


{c1,c2 ,...,cn ,...} com
base em um círculo de raio 1, e a altura de ci
igual a metade da altura de ci − 1 , para todo i (altura de c2 é igual a
metade da altura de c1, altura de c3 é igual a metade da altura de c2,
e assim por diante). Se a altura de c1 é 1, então a soma dos volumes
dos cilindros é
a) π2
a) 24π b) 5π
b) 18 3π c) π
c) 36π d) 1
d) 12π e) 2π
e) 6 3π
28. Pedro recorta em uma folha de papel um setor circular OAB de
24. (ESPCEX) O volume de uma lata cilíndrica é 4π cm3. O custo de raio 12 cm e ângulo de 120°. Juntando e colando os raios OA e OB ele
fabricação das bases é R$ 0,04 por cm2 e o custo de fabricação da faz um cone como mostra a figura abaixo.
superfície lateral é de R$ 0,02 por cm2. O custo de fabricação da lata O
(em R$) em função do raio R (em cm) das bases é: B

 1
a) 0,04 π  R2 + 
 R

 1 120º
b) 0,06π  R2 +  A=B
 R O 12 A

 2
c) 0,06π  R2 +  A altura desse cone é, aproximadamente:
 R
a) 9,6 cm
 2
d) 0,08π  R2 +  b) 10,4 cm
 R
c) 10,8 cm
 1 d) 11,3 cm
e) 0,08π  R2 + 
 R
e) 11,7 cm

470

PM_BOOK16 - MAT.indb 470 25/11/2022 19:24:40


GEOMETRIA ESPACIAL: CONE E CILINDRO

29. O comprimento do círculo da base de um cilindro circular reto é 32. (ESPCEX) Num recipiente em forma de cilindro circular reto,
igual a 6 e sua altura é igual a 4. Um ponto P da base superior está com raio da base 2 cm e altura 6 3 cm (dimensões internas), há um
diametralmente oposto a um ponto Q da base inferior. A menor volume de água de 16 3π cm3. O maior ângulo α que o plano da base
distância do ponto P ao ponto Q sobre a superfície do cilindro é igual a: do cilindro pode fazer com a horizontal é de, aproximadamente
a) 52
6
b) 4+
π
c) 5
d) 7
36
e) 16 +
π2 α

30. (ESPCEX) A razão entre a altura de um cilindro circular reto e


a altura de um cone circular reto, de mesmo volume, é igual a 1/3. Dados (aproximados):
Sendo “R” o raio do cilindro e “r” o raio do cone, pode-se afirmar que tg30º = 0,58
r tg40º = 0,84
a) R=
9
tg50º = 1,19
r
b) R= tg60º = 1,73
3
tg70º = 2,75
c) R = 3r
a) 30°
d) R = r
b) 40°
e) R = 2r
c) 50°
31. (ESPCEX) Deseja-se estimar a quantidade de combustível existente d) 60°
em um tanque cilíndrico disposto horizontalmente, medindo-se a e) 70°
parte molhada de uma régua, conforme a figura abaixo. Sabendo
que o tanque tem 2 m de raio e 12 m de comprimento, e que a 33. (AFA) A razão entre os volumes de dois cones equiláteros de
parte molhada da régua tem 3 m de comprimento, pode-se concluir alturas h e 2h é
que o volume de combustível, em litros, existente no tanque está
1 1
compreendido entre a) c)
2 6
Dados: utilizar π = 3,1 e 3 = 1,7
1 1
b) d)
4 8

34. (AFA) A área total do sólido gerado pela rotação do polígono


ABCDE em torno do eixo y, que contém o lado AE, é, em m2, igual a

Dados:
AE = 2m
AB = 6m
BC = 6m
CD = 3m

a) 144π
b) 150π
c) 168π
d) 170π
a) 145000 e 155000
b) 135000 e 145000
c) 125000 e 135000
d) 115000 e 125000
e) 105000 e 115000

471

PM_BOOK16 - MAT.indb 471 25/11/2022 19:24:40


GEOMETRIA ESPACIAL: CONE E CILINDRO

35. (ESPCEX) O valor da altura de um cilindro reto de raio R, cujo EXERCÍCIOS DE

COMBATE
volume é a soma dos volumes dos sólidos 1 e 2 é

01. Um galão de vinho de formato cilíndrico tem raio da base igual


a 2 m e altura 3 m. Se 40% do seu volume está ocupado por vinho,
é CORRETO afirmar que a quantidade de vinho existente no galão é:
Dados: π = 3,14
a) 3.768 litros. c) 18.840 litros. e) 15.072 litros.
b) 37.680 litros. d) 1.507 litros.

02. (ITA 1987) A área lateral de um cilindro de revolução, de x metros


de altura, é igual à área de sua base. O volume deste cilindro é:
a) 2πx3 m3 d) π 3x33 m
m33
b) 4πx3 m3 e) 6πx3 m3

c) π 2x33 m
m33
13 5 17
a) a. c) a. e) a.
12 4 12 03. (EFOMM 2004) As medidas de raio e altura de um cilindro
7 4 equilátero foram duplicadas. A relação entre o novo volume e o
b) a. d) a. anterior é:
6 3
a) 2 c) 8 e) 32
36. (ESPCEX) Corta-se de uma circunferência de raio 4 cm, um setor b) 4 d) 16
π
circular de ângulo rad (ver desenho ilustrativo), onde o ponto C é
2 04. (AFA) Um reservatório de forma cilíndrica (cilindro circular reto)
o centro da circunferência. Um cone circular reto é construído a partir
de altura 30 cm e raio da base 10 cm está cheio de água. São feitos,
desse setor circular ao se juntar os raios CA e CB.
simultaneamente, dois furos no reservatório: um no fundo e outro a
10 cm de altura do fundo. Cada um desses furos permite uma vazão
de 1 litro por minuto. A quantidade de água restante no reservatório

após minutos é, em litros,
3
a) π

b)
4
c) 2π
3
π
d)
4
05. Numa experiência em sala de aula, são utilizados dois cilindros
graduados com capacidade de um litro. Sabe-se que cada cilindro tem
a altura igual ao dobro do diâmetro de sua base. Um dos cilindros está
vazio e se encontra sobre a mesa, enquanto o outro, que está cheio
de um líquido, será inclinado suavemente de modo que o líquido seja
derramado dentro do primeiro. Veja ilustração na figura abaixo.
O volume desse cone, em cm³, é igual a
3 15 5
a) π c) π e) π
3 3 5
3 15
b) π d) π
5 5

37. (EN) Um copo cilíndrico tem 6 cm de altura e tem uma circunferência


da base medindo 16 cm. Um inseto está do lado de fora do copo, a 1
cm do topo, enquanto, do lado de dentro, a 5 cm do topo, está uma
gota de mel. A gota e o inseto encontram-se em geratrizes do cilindro
que são simétricas em relação ao eixo do cilindro. A menor distância
que o inseto deve andar para atingir a gota de mel é: Se o líquido que foi derramado dentro do cilindro que está sobre a
mesa marca 250 ml em sua graduação, podemos concluir que a maior
a) 10 cm
inclinação α ocorrida no outro cilindro é de
b) 14 cm
a) 60°
c) ( 65 + 5) cm b) 30°
d) ( 89 + 1) cm c) 35°
e) 4 5 cm d) 45°

472

PM_BOOK16 - MAT.indb 472 25/11/2022 19:24:43


GEOMETRIA ESPACIAL: CONE E CILINDRO

06. (ESPCEX) Uma barraca de campanha militar possui o formato 09. (AFA) Num cone reto, a medida do raio da base, da altura, e da
apresentado no desenho abaixo. geratriz estão, nessa ordem, em progressão aritmética de razão igual a
1. Sabendo-se que a soma destas medidas é 12 dm e que a área total
da superfície deste cone é igual à área da superfície de uma esfera, a
medida do raio da esfera, em dm, é
a) 2 c) 5
15 d) 6
b)
2
10. Nesta figura, está representada a região T, do plano cartesiano,
limitada pelo eixo y e pelas retas y = x + 1 e y = 3x:

A curva ABC é um arco de 90º de uma circunferência com 10 metros


de raio. O segmento mede 20 metros. Admitindo π = 3,14, podemos
concluir que o volume do interior da barraca é de aproximadamente:
a) 480 m³
b) 570 m³
c) 618 m³
d) 1140 m³
e) 2880 m³

07. (ESPCEX) Uma esfera de 2 cm de raio é colocada no interior de


um vaso cônico, conforme a figura a seguir. O vaso tem 12 cm de
altura e sua abertura é uma circunferência com 5 cm de raio. Nessas
condições, a menor distância (d) entre a esfera e o vértice do cone é:
Seja S o sólido obtido pela rotação da região T em torno do eixo y.
Então, é CORRETO afirmar que o volume de S é
π π π
a) c) e)
24 8 2
π π
b) d)
12 4

a) 3,0 cm
b) 3,2 cm
c) 3,4 cm DESAFIO PRO
d) 3,6 cm
e) 3,8 cm
1 (ITA) Um triângulo retângulo com hipotenusa= c 2(1 + 6 )
está circunscrito a um círculo de raio unitário. Determine a
área total da superfície do cone obtido ao girar o triângulo em
08. (ESPCEX) A figura abaixo representa dois tanques cilíndricos,
torno do seu maior cateto.
T1 e T2, ambos com altura h, e cujos raios das bases medem R e
R√2, respectivamente. Esses tanques são usados para armazenar
combustível e a quantidade de combustível existente em cada um
deles é tal que seu nível corresponde a 2/3 da altura. O tanque T1 2 (IME) Em um cone equilátero são inscritas duas esferas de
raios
3 −1
R e R, conforme a figura abaixo. Um plano
contém gasolina pura e o tanque T2 contém uma mistura etanol- 3 +1
gasolina, com 25% de etanol. Deseja-se transferir gasolina pura do secante ao cone é traçado de forma que este seja tangente
tanque T1 para T2 até que o teor de etanol na mistura em T2 caia às duas esferas. Determine em termos de R o maior segmento
para 20%. Nessas condições, ao final da operação, a diferença entre a possível que une dois pontos da curva formada pela interseção
altura dos níveis de T1 e T2 será: do referido plano com o cone.

1
a) h
2
1
b) h
3
1
c) h
4
1
d) h
5
1
e) h
6

473

PM_BOOK16 - MAT.indb 473 25/11/2022 19:24:43


GEOMETRIA ESPACIAL: CONE E CILINDRO

3 (ITA) Em um cone circular reto de altura 1 e raio da base


1 inscreve-se um tetraedro regular com uma de suas faces
paralela à base do cone, e o vértice oposto coincidindo com o
ANOTAÇÕES

centro da base do cone. Determine o volume do tetraedro.

4 (ITA) Uma esfera S1, de raio R > 0, está inscrita num cone
circular reto K. Outra esfera, S2, de raio r, com 0 < r < R, está
contida no interior de K e é simultaneamente tangente à esfera
S1 e à superfície lateral de K. O volume de K é igual a:
πR5
a) .
3r(R − r)
2πR5
b) .
3r(R − r)
πR5
c) .
r(R − r)
4 πR5
d) .
3r(R − r)
5πR5
e) .
3r(R − r)

5 (ITA) Um cilindro reto de altura h = 1 cm tem sua base no


plano xy definida por x² + y² – 2x – 4y + 4 ≤ 0. Um plano,
contendo a reta y – x = 0 e paralelo ao eixo do cilindro, o
secciona em dois sólidos. Calcule a área total da superfície do
menor sólido.

GABARITO
EXERCÍCIOS DE FIXAÇÃO
01. D 04. E 07. D 10. E
02. B 05. B 08. D
03. C 06. C 09. C
EXERCÍCIOS DE TREINAMENTO
01. D 14. E 27. E
02. D 15. A 28. D
03. C 16. A 29. C
04. C 17. B 30. D
05. D 18. C 31. D
06. C 19. B 32. D
07. C 20. A 33. D
08. A 21. D 34. B
09. A 22. B 35. E
10. B 23. A 36. C
11. C 24. D 37. A
12. D 25. B
13. C 26. D
EXERCÍCIOS DE COMBATE
MV
01. E=3 tg 60 = ° 04. C→ MV = R 307. B 10. B
R
02. B
Cone ⇒ S3total
05. C
(
08. A
− 1= Slateral + Sbase R ⋅ 3 − 3 )
03. CNV = R⋅ ⋅ B3 → NV = 09. D
06.
3 +1 3 +1
( ) ( )
2
Stotal =PRO
DESAFIO πRg + πR2 ⇒ Stotal = π ⋅ 6 ⋅ 2 + 2 6 + π ⋅ 6
R⋅ 3− 3( ) (
R⋅ 3+ 3 −3+ 3 2R 3 )
Stotal=
01. MN =R2π3 − (
9 + 6 u.a. =
3 +1 ) 3 +1
=
3 +1
( )
02. MN = R 3 − 3 → maior segmento possível

(
6 ⋅ 5 2 −7 ).
03. V =
 π4 1  π
04. BA t =2 ⋅  4 − 2  + 2 + 2
05. A t = ( π + 2 − 1) unid2

474

PM_BOOK16 - MAT.indb 474 25/11/2022 19:24:44


ESFERAS E TRONCOS

DEFINIÇÃO
Chama-se esfera a um sólido limitado por uma superfície em que
todos os pontos de um ponto interior chamado de centro.
A superfície que delimita a esfera, deomina-se: superfície
esféricae qualquer segmento OP que une o centro O com um ponto P
da superfície esférica denomina-se raio da esfera.

A área do fuso esférico e o volume da cunha esférica são


proporcionais ao ângulo do fuso (α).
Área do fuso esférico de ângulo α radianos: Sfuso = 2αR2.
Volume da cunha esférica de ângulo α radianos: Vcunha= 2 αR3.
3

SEGMENTO E CALOTA ESFÉRICA


ÁREA DA SUPERFÍCIE ESFÉRICA DE RAIO R Quando seccionamos uma esfera por um plano e um outro plano
A área da superfície esférica é igual ao quádruplo da área do tangencia a esfera, o plano secante divide a esfera em dois segmentos
círculo máximo da esfera que tem área igual a πR². Portanto a área da esféricos, e a superfície esférica em duas calotas esféricas.
superfície esférica é S = 4πR².

VOLUME DA ESFERA
4
O Volume da esfera de raio R é igual ao produto de do raio
3
pela área do círculo de área máxima logo seu volume é dado pela
4 3
fórmula V= πR .
3

FUSO E CUNHA ESFÉRICA VOLUME DO SEGMENTO ESFÉRICO


De modo análogo ao volume da esfera, podemos calcular o
Denomina-se fuso esférico a superfície gerada pela rotação de
volume do segmento esférico de altura h subtraindo um tronco de
uma circunferência que gira um ângulo α menor que 2π radianos ao
cone de altura h, de um cilindro de raio R e também altura h.
redor do seu diâmetro.
Chama-se cunha esférica ao sólido gerado pela rotação, de um
ângulo α, menor que 2π radianos, de um semicírculo, ao redor do
seu diâmetro.

475

PM_BOOK16 - MAT.indb 475 25/11/2022 19:24:45


ESFERA E TRONCOS

Volume do segmento esférico de altura h em uma esfera de raio r


Logo a área do círculo delimitado pelo plano α é
πh2 S = πs² = π(r² – h²)
=
Vsegmento ( 3r − h)
3

Também podemos calcular como II. Área da seção determinada na anticlepsidra.


A área determinada pelo plano α na anticlepsidra é a área da
πh 2
V=
segmento
6
( 3a + h2 ) coroa circular assinalada em verde.
S = πr² – πh² = π(r² – h²)

Área da calota esférica de altura h em uma esfera de raio R


Logo vemos que as áreas delimitadas por qualquer plano
Vcalota = 2πRh paralelo ao plano horizontal onde estão situadas a esfera e a
anticlepsidra são iguais e assim a esfera terá o mesmo volume da
Scalota = 2πrh anticlepsidra.
Assim o volume da esfera será
Observação 1  2
Vesf =Vcil − 2Vcone =
πr 2 ⋅ 2r − 2  πr 2 ⋅ r  =π
2 r 3 − πr 3
O VOLUME DA ESFERA PELO PRINCÍPIO DE CAVALIERI 3  3

Vamos inicialmente definir a anticlepsidra. 4 3


Vesf= πr
A anticlepsidra é sólido geométrico formado a partir de um 3
cilindro equilátero, do qual subtraímos dois cones opostos pelos
vértices (como uma ampulheta) cujas bases coincidam com as bases
do cilindro e cuja altura, obviamente, seja igual ao raio da base.
ESFERA DE REVOLUÇÃO
A esfera é obtida pela rotação de um semicírculo ou um círculo
completo em torno de seu diâmetro.

Agora tomemos uma esfera e uma anticlepsidra, de mesmo


raio r, assentadas num mesmo plano horizontal.

Pelo princípio de Cavalieri se as seções dos sólidos, obtidas


a uma mesma altura, têm a mesma área, então os sólidos têm o
mesmo volume.
De acordo com a figura acima tomamos uma seção transversal
por um plano α a partir de uma altura h tomado a partir dos
centros da esfera e da anticlepsidra.
Vamos calcular as áreas das 2 seções que estão assinaladas em
verde na figura.
Vamos ilustrar um sólido de revolução “diferente” já cobrado em
I. Área da seção determinada na esfera.
concurso militar. Vamos ver o resultado da rotação de 360° de um
Do Teorema de Pitágoras no triângulo retângulo de lados r, s setor circular.
e h temos
r² = h² + s² ⇒ s² = r² – h²

476

PM_BOOK16 - MAT.indb 476 25/11/2022 19:24:46


ESFERA E TRONCOS

Vamos rotacionar o setor circular OAB em torno do eixo


especificado.

TEOREMAS DE PAPO-GULDINO
Teremos como volume final o volume da semiesfera AO’A’ de
centro O subtraído do volume do cone reto OBB’ e também subtraído
do volume da calota esférica BO’B’. PRIMEIRO TEOREMA
O primeiro teorema define que a área de uma superfície de
revolução é igual ao produto do comprimento da curva geratriz pelo
comprimento do caminho percorrido pelo centroide (centro de massa)
dessa mesma curva ao longo do ângulo que gera a superfície.
Ou seja, o comprimento do caminho percorrido pelo centroide
é o comprimento de circunferência, assim 2π multiplicado pelo raio,
que nesse caso é a distância do centro de massa ao eixo de rotação.
Assim S = 2πdL, sendo L o comprimento da curva.

477

PM_BOOK16 - MAT.indb 477 25/11/2022 19:24:46


ESFERA E TRONCOS

SEGUNDO TEOREMA Assim para calcularmos a área e o volume desse sólido teríamos.
O segundo teorema define que o volume de um sólido de
revolução é igual ao produto da área da superfície geratriz pelo
comprimento do caminho percorrido pelo centroide dessa mesma
superfície ao longo do ângulo que gera o volume.
Sendo A a área que será rotacionada, teremos de maneira análoga
ao primeiro teorema V = 2πdA.
Veja o exemplo
Vamos imaginar a figura abaixo sendo rotacionada em relação ao
eixo de rotação destacado.

Assim sua área e volume serão


S =2πdL =2πd ( AB + BC + CD + AD)

 V = 2πd ⋅ SABCD
Vamos supor que o lado AD dista 2 cm do eixo de rotação e que
AD = 10 cm, BC = 6 cm, e CD = 2 cm.

Assim d =
2+
(CD) =
3.
Ao rotacionarmos teremos a formação do seguinte sólido. 2
Para calcularmos AB utilizaremos o teorema de Pitágoras.

AB2 = CD2 + ( AD − BC) = 22 + 42 = 20 ⇒ AB = 2 5


2

Assim

( ) (
S = 2πd ( AB + BC + CD + AD) = 2π ⋅ 3 ⋅ 2 5 + 10 + 2 + 6 = 12π 9 + 5 cm² )
V = 2πd ⋅ SABCD = 2π ⋅ 3 ⋅
(10 + 6 ) ⋅ 2 = 96π cm³
2

SÓLIDOS HOMOGÊNEOS
Sabemos, do dia a dia inclusive, que quando temos sólidos
homogêneos, cubos, paralelepípedos, prismas retos ou cilindros
retos, que quando queremos tomar uma porção de seu volume
basta tomarmos a mesma fração da altura que queremos de
volume. Veja abaixo.

478

PM_BOOK16 - MAT.indb 478 25/11/2022 19:24:47


ESFERA E TRONCOS

Todos os círculos, de centros O1, O2 e O3 possuem as mesmas


áreas πR², fazendo com que seus volumes dependam somente das
suas alturas. Assim a fração da altura que se efetua o corte define a
fração do volume. Assim no nosso exemplo, cada sólido resultante do
corte terá metade do volume do sólido inicial.
Não é à toa que se uma receita pede que seja utilizado metade
do volume de uma lata de leite condensado nós utilizamos o leite
condensado até a metade da lata. Se enchermos um cubo até um
terço de sua altura teremos um terço do volume total e assim para
qualquer sólido homogêneo.

Observação
Nos sólidos homogêneos os cortes em diagonal também
geram metade do volume.

Temos um cilindro circular reto e vamos fazer um corte utilizando


um plano paralelo as bases exatamente na metade da sua altura.

Cada sólido resultante tem metade do volume do sólido


original.

PIRÂMIDES E CONES
H
Com isso geramos 2 novos cilindros retos, mas com alturas O estudo para as pirâmides retas e cones retos se dá da mesma
2
maneira quando seccionados por planos paralelos a suas bases.
porém, pelo fato de ser um sólido homogêneo, qualquer corte por
um plano paralelo as bases irão gerar o mesmo polígono das bases e Utilizando a mesma ideia dos sólidos homogêneos fica fácil
de mesma área S. perceber que após um corte por um plano paralelo a base os sólidos
resultantes não terão seus volumes ligados diretamente a proporção
de suas alturas. Dessa forma temos de recorrer a semelhança entre os
sólidos. Tomemos o cone reto abaixo.

479

PM_BOOK16 - MAT.indb 479 25/11/2022 19:24:47


ESFERA E TRONCOS

Vamos fazer uma secção por um plano paralelo a base de centro Tronco é o sólido resultante da retirada do sólido superior ao plano
O do cone. de corte (sólido que é semelhante ao sólido original). Vale lembrar que
os troncos eles não são sólidos semelhantes com os sólidos originais
ou os retirados, utilizá-los na semelhança é um erro muito comum.

A figura acima ilustra um tronco de cone reto, resultado da


remoção do cone menor do cone maior.
Chegaremos ao seu volume utilizando semelhança entre os
sólidos encontrando os volumes do sólido maior e do menor e no final
fazendo a diferença entre seus volumes.

Vemos que o novo cone gerado, VCD de centro O’, também tem
como base um círculo, porém de raio diferente do cone VAB de centro
O. Dessa forma seu volume não fica dependendo somente de sua
altura h como também passa a depender desse seu novo raio da base Vamos “ver” de frente os sólidos e ver os triângulos que temos.
r, que é definido através de uma semelhança. Quando fazemos a semelhança de medidas lineares, medidas de
comprimento, apenas uma dimensão, dizemos que essa razão de
semelhança é k. Assim para os triângulos da figura teríamos
r h VD
k= = = =…
R H VA

VD g
*Para o cone =
VA G
Sabemos que quando fazemos razões entre áreas de figuras
planas nossa razão passa a ser k². Agora quando fizermos razões entre
volumes teremos a razão k³.
Dessa forma podemos fazer razões entre os volumes dos sólidos
VCD e VAB.
VVCD
= k3
VVAB

Assim para encontrarmos k basta utilizarmos a razão que nos for


r h
mais conveniente, seja ou ou qualquer outra razão linear.
R H

480

PM_BOOK16 - MAT.indb 480 25/11/2022 19:24:48


ESFERA E TRONCOS

Depois de encontrado o volume do sólido “menor” para Assim fazendo a semelhança entre os volumes dos 2 cones
encontrarmos o volume do tronco basta efetuar a subtração entre os
3
volumes dos cones “maior” e “menor”. Vmenor V  1 Vmenor 1
= k 3 ⇒ menor =
  ⇒ =⇒ Vmenor =
8
Vmaior 64  2 64 8
Exemplo:
Assim o volume do cone “menor” é 8 cm3 e consequentemente
Um cone reto de volume igual a 64 cm³ é seccionado na metade
o volume do tronco gerado será Vtronco = 64 − 8 = 56 cm³.
de sua altura por um plano paralelo a sua base. Calcular o volume do
tronco resultante.
Existem 2 fórmulas que calculam direto os volumes do tronco de
cone e de pirâmide quando são conhecidas as medidas das 2 bases e
da altura do tronco.

I. Tronco de cone

πH 2
=
Vtronco
3
(R + Rr + r2 )

R → Raio da base maior


Ao utilizarmos a subtração teremos r → Raio da base menor
H → Altura do tronco

II. Tronco de pirâmide

* A fórmula serve para troncos de pirâmides retas de qualquer


base, a figura utilizou a quadrangular só para exemplificar.

k=
h r
=
H R
Vtronco=
H
3
(
S+ S⋅s + s )
H S → Área da base maior
H 1 s → Área da base menor
k 2=
Onde é mais conveniente utilizarmos h = e assim = .
2 H 2 H → Altura do tronco
* A semelhança também nos permite concluir que
H
2 = r ⇒ 1= r ⇒r = R
H R 2 R 2

481

PM_BOOK16 - MAT.indb 481 25/11/2022 19:24:48


ESFERA E TRONCOS

TRONCO DE CONE RETO DE


REVOLUÇÃO
O tronco de cone reto é obtido pela rotação de um trapézio
retângulo em torno da reta suporte do lado reto (altura do trapézio).

Para calcular o volume do tronco de cone podemos utilizar a


conhecida fórmula
π ⋅H 2
=
Vtronco
3
(R + Rr + r2 )
=
Vtronco
π ⋅ AD
3
( 2
CD + CD ⋅ AB + AB
2
)
Vamos ver exemplos de troncos de revolução vazados.

I. Sólido de revolução gerado pela rotação de um triângulo


retângulo ABC, retângulo em A, em torno de uma reta r,
paralela a AB e distando d de AB.

Assim teremos o tronco de cone reto com o raio da base maior


sendo CD, da base menor sendo AB e altura AD .

482

PM_BOOK16 - MAT.indb 482 25/11/2022 19:24:49


ESFERA E TRONCOS

Teremos o tronco de cone BCC’B’ vazado pelo cilindro ABB’A’.

II. Sólido de revolução gerado pela rotação de um trapézio


retângulo ABCD, em torno de uma reta r, paralela a AB e
distando d de AB.

Teremos o tronco de cone CDD’C’ vazado pelo cilindro ABB’A’.

483

PM_BOOK16 - MAT.indb 483 25/11/2022 19:24:49


ESFERA E TRONCOS

Exercício Resolvido Resolução: E

Área da Esfera : 4 πR
2
01. Duas esferas de aço de raio 4 cm e 3 61 cm fundem-se para A 2
 2
⇒ 4 πR2 = 6A 2 ⇒  = π
formar uma esfera maior. Considerando que não houve perda de Área do Cubo : 6A R 3
material das esferas durante o processo de fundição, a medida do
3
raio da nova esfera é de: A3
3
3  2π  3 2π 2π 1 2π π
Logo: Vcubo = 3 A
=  =  =⋅ = =
a) 5 cm Vesfera 4 πR3 4 π  R  4 π  3  4π 3 3 2 3 6
b) 5,5 cm 3
3 3
c) 4,5 cm Vcubo A3 3 A 3  2π  3 2π 2π 1 2π π
= =  =  =⋅ = =
d) 6 cm Vesfera 4 πR3 4 π  R  4 π  3  4π 3 3 2 3 6
e) 7 cm 3

Resolução: A
Exercício Resolvido
Seja: V(n) = o volume da nova esfera
n = o raio da nova esfera 04. Um cone e uma esfera têm o mesmo volume. Se o raio da
V( 4 ) = o volume da esfera de raio 4 cm base do cone é o quíntuplo do raio da esfera, então a razão entre
a altura do cone e o raio da esfera é igual a:
V3 61 = o volume da esfera de raio 3
61 cm
4 1 4
V(r ) = π(r)3 = o volume da esfera de raio r a) d)
3 5 25
Como não houve perda de material durante o processo de 4 4
b) e)
fundição, o volume da nova esfera, de raio n é a soma dos volumes 5 15
das iniciais: 1
4 4 4 4 c)
V(n) + V( 4 ) + V 3 61 = π(4 cm)3 + π( 3 61 cm)3 = π64 cm3 + π61 cm3 25
( ) 3 3 3 3
4 Resolução: D
→ V(n) = π125 cm3
3
Logo, n³ = 125 cm³ → n = 5 cm

Exercício Resolvido

02. (UFRGS) Se um jarro com capacidade para 2 litros está


completamente cheio de água, a menor medida inteira, em cm,
que o raio de uma bacia com a forma semiesférica deve ter para
comportar toda a água do jarro é
a) 8.
b) 10. Como possuem o mesmo volume:
c) 12.
4 3 1 4R
πR = π (5R ) h ⇒ 4R3 = 25R2h ⇒
2
d) 14. =h
3 3 25
e) 16.
h h 4
Logo: ⇒ =
Resolução: B R R 25
1 4 ⋅ π ⋅ r3 2 ⋅ π ⋅ r3
Vsemiesfera = ⋅ = = 2000 cm3. Logo, r = 10 cm.
2 4 3

Exercício Resolvido
Exercício Resolvido 05. O interior de um reservatório de água tem forma de uma
semiesfera com 12 metros de diâmetro. A capacidade desse
03. Seja uma esfera de raio R e um cubo de aresta A, ambos com reservatório é de:
a mesma área de superfície. A razão entre o volume do cubo e o
volume da esfera é igual a: a) 56π m³ d) 288π m³
b) 88π m³ e) 312π m³
1 2π π
a) c) e) c) 144π m³
π 3 6
π π Resolução: C
b) d)
12 3 =
D 12 m ⇒ =
R 6m

1 4 3 2
V ( semi − esfera) = ⋅ πR = π ( 6 ) ⇒ V = 144 π m3
3

2 3 3

484

PM_BOOK16 - MAT.indb 484 25/11/2022 19:24:50


ESFERA E TRONCOS

EXERCÍCIOS DE 09. Uma garrafinha de suco tem a parte interna no formato esférico,

FIXAÇÃO com diâmetro de 12 cm. Arredondando-se π para 3, tem-se o


volume correto de suco que é colocado nessa garrafinha, para a
comercialização. Se cada cm³ corresponde a 1 mL, no rótulo dessa
garrafinha consta que o volume de suco nela contida é de
01. Seja uma esfera de raio r. É correto afirmar que seu volume é a) 144 mL. c) 864 mL. e) 2592 mL.
dado por b) 576 mL. d) 1256 mL.
4 3
a) V = r 3 c) V= πr
3 10. Uma esfera possui seu volume igual ao dobro da sua área de
superfície. Se o raio da esfera é dado em cm, o volume desta esfera,
b) V = 2πr d) V= 4 πr 2
em cm³, é
a) 6π. b) 36π. c) 144π. d) 288π.
02. Para uma esfera de raio R, qual é a opção que mostra a razão
entre o volume e a área superficial?
1 1 EXERCÍCIOS DE

TREINAMENTO
a) R c) R² e) π
3 4
b) R d) π

03. Considere uma esfera de raio de 3 cm. A razão entre o volume e


a área da superfície dessa esfera é 01. A figura seguinte é um recipiente cilíndrico reto, e contém água
até a metade de sua altura. Uma esfera maciça, colocada no seu
a) 1 cm c) 2π cm e) π
interior, fica totalmente submersa, elevando a altura da água em
b) 1 cm³ d) π cm² 2 cm. Qual o volume dessa esfera?

04. Assinale a alternativa que apresenta o raio de uma esfera cujo


volume é igual a 972π cm³.
a) 6 cm c) 9 cm e) 13 cm
b) 7,5 cm d) 11,5 cm

05. Uma esfera possui raio de 5 cm. O volume desta esfera, em cm³, é
(Dado: π = 3)
a) 15 c) 100 e) 1500
b) 75 d) 500

06. O diâmetro de uma esfera é 18 cm. Seu volume, em m³, é,


aproximadamente:
a) 16π cm³ c) 24π cm³ e) 32π cm³
Dado: utilize π = 3.
b) 20π cm³ d) 28π cm³
a) 0,9 ⋅ 10−3 c) 2,9 ⋅ 10−3
b) 1,3 ⋅ 10 −3
d) 3,6.10−3 02. Um laboratório possui dois recipientes de vidro. O primeiro
recipiente tem a forma de uma esfera cujo raio mede R metros, e
07. Um artesão vai fazer uma bola de couro com 20 cm de diâmetro. o segundo tem a forma de um cone, cujo raio da base e cuja altura
Qual será a área (A) da superfície desta bola? (Faça π = 3,14) medem R metros. A razão entre a medida do volume do recipiente
esférico e a medida do volume do recipiente cônico, ambas dadas em
a) A = 1230 cm². c) A = 1000,5 cm². metros cúbicos, é
b) A = 950,7 cm². d) A = 1256 cm². a) 2 c) 4 e) 8
b) 3 d) 6
08. Acerca da geometria espacial, se o raio da esfera a seguir for igual a
20 cm, é correto afirmar que o seu volume será de, aproximadamente:
03. Num cone circular reto de raio de 4 cm é inscrito uma esfera de
Dado: considere π = 3,14. diâmetro de 4 cm. O valor da altura do cone para que o volume do
cone seja o triplo do volume da esfera é
a) 8 cm c) 8π cm e) 6 cm
b) 6π cm d) 7 cm

04. (EEAR) Uma esfera tem 36π m³ de volume. A medida de sua


superfície, em m², é
a) 72π b) 56π c) 48π d) 36π

05. (EEAR) Um reservatório, com volume igual a 144π m³, tem a


forma de uma semiesfera. Para aumentar seu volume em 342π m³, é
preciso aumentar o raio do reservatório em
a) 12m c) 6m
a) 28.495,35 cm³ c) 32.282,33 cm³
b) 9m d) 3m
b) 31.159,21 cm³ d) 33.493,33 cm³

485

PM_BOOK16 - MAT.indb 485 25/11/2022 19:24:51


ESFERA E TRONCOS

06. (EEAR) Um escultor irá pintar completamente a superfície de 14. Um silo foi construído em forma de tronco de pirâmide, conforme
uma esfera de 6 m de diâmetro, utilizando uma tinta que, para essa a figura abaixo. Sua altura é de 9 metros e suas bases são quadrados de
superfície, rende 3 m² por litro. Para essa tarefa, o escultor gastará, no lados iguais a 4 metros e 3 metros, respectivamente. Com base nessas
mínimo, _____ litros de tinta. (Considere π ≅ 3) informações, pode-se concluir que a capacidade total desse silo é
a) 18 c) 36
b) 24 d) 48

07. (EEAR) Em um recipiente cúbico vazio, foram colocadas 1000


esferas idênticas, sem que elas ultrapassassem as bordas desse
recipiente. Em seguida, verificou-se que o volume do cubo não
ocupado pelas esferas era de 4 dm³. Se internamente as arestas do
recipiente medem 20 cm, o volume de cada esfera é cm³
a) 4 c) 2
b) 3 d) 1 a) 48 m³. d) 111 m³.
b) 64 m³. e) 144 m³.
08. (EEAR) No tronco de cone reto, as bases são paralelas. Se o raio c) 108 m³.
da base maior mede 5 cm e a distância entre as duas bases, 4 3 cm,
então o volume desse tronco de cone, em cm³, é 15. Um cubo cuja medida de cada aresta é 3 dm está inscrito em uma
esfera de raio R A medida de um diâmetro (2R) da esfera é
a) 2 3 dm. c) 3 3 dm.
b) 3 2 dm. d) 4 3 dm.

16. Considere uma esfera de raio 2 cm com área total A e volume V.


Suponha que os valores y, A, V formem uma progressão geométrica
nessa ordem. Em centímetros, quanto vale y?
124 π 3 96π 3 3π
a) . c) . c) 8π e) 96π
3 3 a)
2
b) 125π 3 . d) 124 π 3. 8π
b) d) 24π
3
09. (EEAR) Uma esfera está inscrita num cilindro equilátero cuja área
lateral mede 16π cm². O volume da esfera inscrita é 17. (EFOMM) Seja uma esfera de raio R e um cubo de aresta A, ambos
32 com a mesma área de superfície. A razão entre o volume do cubo e o
a) 8π c) π volume da esfera é igual a
3
256 1 2π π
b) 16π π a) . c) . e) .
d) π 3 6
3
π π
b) . d) .
10. (ESA) Ao triplicarmos o raio e tomarmos a terça parte de uma 12 3
esfera, ela possuirá, em relação à esfera original, um volume
a) 2 vezes maior d) 12 vezes maior 18. Um reservatório tem a forma de uma esfera. Se aumentarmos o
b) 3 vezes maior e) 20 vezes maior raio da esfera em 20%, o volume do novo reservatório, em relação ao
volume inicial, aumentará
c) 9 vezes maior
a) 60% d) 69,6%
11. (ESA) Duas esferas de aço de raio 4 cm e 3 61 cm fundem-se b) 63,2% e) 72,8%
para formar uma esfera maior. Considerando que não houve perda de c) 66,4%
material das esferas durante o processo de fundição, a medida do raio
da nova esfera é de: 19. As alturas de um cone circular reto de volume P e de um cilindro
a) 5 cm c) 4,5 cm e) 7 cm reto de volume Q são iguais ao diâmetro de uma esfera de volume R.
b) 5,5 cm d) 6 cm Se os raios das bases do cone e do cilindro são iguais ao raio da esfera,
então, P – Q + R é igual a
12. O volume de um cilindro de 8 cm de altura equivale a 75% a) 0. c) π. e) 2π.
do volume de uma esfera com 8 cm de diâmetro. A área lateral do 2π 4π
cilindro, em cm², é b) . d) .
3 3
a) 42 2π c) 32 2π
20. (ESA) Um tanque subterrâneo tem a forma de um cone invertido.
b) 36 3π d) 24 3π Esse tanque está completamente cheio com 8 dm³ de água e 56 dm³
de petróleo. Petróleo e água não se misturam, ficando o petróleo na
13. O volume de um tronco de pirâmide de 4 dm de altura e cujas parte superior do tanque e a água na parte inferior. Sabendo que o
áreas das bases são iguais a 36 dm² e 144 dm² vale tanque tem 12 m de profundidade, a altura da camada de petróleo é

a) 330 m² d) 720 dm³ a) 10m. d) 7m.

b) 330 cm² e) 336 dm³ b) 9m. e) 6m.

c) 360 dm³ c) 8m.

486

PM_BOOK16 - MAT.indb 486 25/11/2022 19:24:52


ESFERA E TRONCOS

21. Um cone circular reto tem o diâmetro da base medindo 12 cm e 27. Uma esfera confeccionada em aço é usada em um rolamento de
altura medindo 9 cm. Este cone é interceptado por um plano β que é motor de um navio da Marinha do Brasil. Se o raio da esfera mede
paralelo à base e está distante 6 cm do vértice. O volume do tronco de
cone assim formado é: 3 5 3 5 3  cm, então seu volume vale
a) 76π cm³ c) 238,64π cm³ a) 45 ⋅ 10−3 π dm3 d) 0,15 ⋅ 103 π dm3
b) 108π cm³ d) 304π cm³ −3
e) 60 ⋅ 103 π dm3
b) 0,45 ⋅ 10 π dm 3

22. Um fabricante de cristais produz três tipos de taças para servir c) 60 ⋅ 10−3 π dm3
vinho. Uma delas tem o bojo no formato de uma semiesfera de raio
r; a outra, no formato de um cone reto de base circular de raio 2r e 28. Uma calota esférica é uma parte da esfera que foi cortada por um
altura h; e a última, no formato de um cilindro reto de base circular plano. A calota esférica da figura a seguir possui altura (h) igual a 1 e a
de raio x e altura h. Sabendo-se que as taças dos três tipos, quando área da sua base (região hachurada) vale 9π. Com base nessa situação
completamente cheias, comportam a mesma quantidade de vinho, é hipotética, o volume da esfera é igual a
correto afirmar que a razão x/h é igual a:

a) ( 3) c) (2 3 ) e) (4 3)
6 3 3

b)
( ) 3
d) 3
3

23. Três esferas, cujas medidas dos raios são respectivamente 1 cm,
2 cm e 3 cm, repousam sobre um plano horizontal e tangenciam-se
mutuamente, isto é, cada esfera tangencia as outras duas. Os pontos
de contato dessas esferas com o plano são vértices de um triângulo.
O produto das medidas dos lados desse triângulo, em cm³, é igual a 500 π
a) π. c) 12π . e) .
a) 48. c) 36. 3 3
b) 12. d) 24. 256 4
b) π. d) π.
3 3
24. O volume de um tronco de pirâmide quadrangular regular de
apótema da base igual a 6 m e de apótema da pirâmide igual a 10 m, 29. O diâmetro de uma das bases de um tronco de cone é 100 mm. Se
sabendo que o tronco tem um quarto da altura da pirâmide, é: o diâmetro é aumentado de 21%, mantendo-se a altura e o tamanho
a) 18 m³. c) 288 m³. e) 384 m³. da outra base inalterados, o volume do tronco de cone também
b) 222 m³. d) 388 m³. aumentará 21%. Qual o diâmetro da outra base?
a) 90 mm c) 105 mm e) 121 mm
25. (ESPCEX) Um trapézio isósceles, cujas bases medem 2 cm e 4 cm b) 100 mm d) 110 mm
e cuja altura é 1 cm, sofre uma rotação de 180° em torno do eixo que
passa pelos pontos médios das bases. O volume, em cm³, do sólido 30. (ESPCEX) Considere as proposições abaixo:
gerado por essa rotação é:
I. O volume V de um cilindro equilátero de raio r é V = 4πr³.
4π 8π
a) c) 2π e) II. O volume de um cubo de área total 600 cm² é 1000 cm³.
3 3

III. Quando o raio de uma esfera aumenta 100%, o volume da esfera
5π d) aumenta 700%.
b) 3
3
IV. Uma reta r e um plano a são perpendiculares a uma outra reta t,
em pontos distintos, então r e a são paralelos.
26. Os três recipientes a seguir possuem formatos de sólidos
geométricos distintos, porém com a mesma altura (h) e mesmo Dentre as proposições acima somente é/são falsa(s) a(s):
2 a) I c) I e III e) III e IV
diâmetro de abertura (D). Neles se colocou água até da altura. Seja
3 b) II d) I e IV
V1, V2 e V3 os volumes dos recipientes R1, R2 e R3, respectivamente. A
correta relação entre os volumes é 31. (AFA) Uma esfera de 10 cm de raio e um cone reto de 10 cm
de raio da base e altura 20 cm, estão situados sobre um plano α. A
distância x, de um plano β paralelo ao plano α, tal que as áreas das
secções obtidas pela intercessão do plano β com os sólidos, esfera e
cone, sejam iguais, é, em cm, igual a
a) 1 b) 2 c) 6 d) 4

32. Seja uma esfera de raio R e um cubo de aresta A, ambos com a


mesma área de superfície. A razão entre o volume do cubo e o volume
da esfera é igual a
1 2π π
a) . c) . e) .
π 3 6
a) V1 > V2 > V3 d) V1 > V2 =
V3
π π
b) V1 > V3 > V2 e) =
V3 V1 > V2 b) . d) .
12 3
c) V3 > V1 > V2

487

PM_BOOK16 - MAT.indb 487 25/11/2022 19:24:53


ESFERA E TRONCOS

33. (AFA) Assinale a alternativa que preenche corretamente a lacuna 37. (AFA) Um objeto de decoração foi elaborado a partir de sólidos
abaixo. O volume do sólido gerado pela rotação de 360° da região utilizados na rotina de estudos de um estudante de matemática.
hachurada da figura em torno do eixo é de _________ π cm³. Inicialmente, partiu-se de um cubo sólido de volume igual a 19683 cm³.
Do interior desse cubo, retirou-se, sem perda de material, um sólido
formado por dois troncos de pirâmide idênticos e um prisma reto,
como mostra o esquema da figura a seguir.

a) 230 c) 374
224 608
b) d)
3 3
Sabe-se que:
34. (AFA) Considere o sólido geométrico obtido pela rotação de 360º
do triângulo ABC em torno da reta que passa por C e é paralela ao • as bases maiores dos troncos estão contidas em faces opostas
lado AB. do cubo;
• as bases dos troncos são quadradas;
Sabe-se que este triângulo é isósceles, com AC ≡ BC = R 2 m,
AB = 2R m (sendo R uma constante real não nula), e que o volume do • a diagonal da base maior de cada tronco está contida na
diagonal da face do cubo que a contém e mede a sua terça
sólido obtido é V= 4 π 3 m3.
parte;
A medida de R, em metros, é igual a
• a diagonal da base menor de cada tronco mede a terça parte
a) 6
3 c) 3
9 da diagonal da base maior do tronco; e

b) 3
3 d) 3 • os troncos e o prisma têm alturas iguais.
Assim, o volume do objeto de decoração obtido da diferença entre o
35. (AFA) Um sistema de irrigação para plantas é composto por volume do cubo e o volume do sólido esquematizado na figura acima,
uma caixa d’água, em formato de cone circular reto, interligada a em cm³, é um número do intervalo
30 esferas, idênticas. O conteúdo da caixa d’água chega até as esferas a) [17200, 17800]
por encanamentos cuja capacidade de armazenamento é desprezível. b) ]17800, 18400]
O desenho a seguir ilustra a ligação entre a caixa d’água e uma das
1 c) ]18400, 19000]

30 esferas, cujo raio interno mede r = π 3
dm . d) ]19000, 19600]

38. (AFA) Considere um tronco de pirâmide obtido de uma pirâmide


quadrangular regular. Por esse tronco, passa-se um plano a paralelo às
bases gerando um quadrilátero de área x cm², tal que
• a razão entre a distância da base menor do tronco ao plano
3
a e a distância do plano a à base maior do tronco é igual a ;
2
• a área da base maior do tronco mede 441 cm²; e
• a área da base menor do tronco mede 64 cm².

A área x do quadrilátero, em cm², é igual a


Se a caixa d’água está cheia e as esferas, bem como os encanamentos, 8441 6241
estão vazios, então, no momento em que todas as 30 esferas ficarem a) c)
64 25
cheias, restará, no cone, apenas a metade de sua capacidade
total. Assim, a área lateral de um cone equilátero cujo raio da base é 12661 4772
congruente ao da caixa d’água, em dm², é igual a b) d)
81 16
a) 80 c) 20
b) 40 d) 10 39. (AFA) Considere a região E do plano cartesiano dada por
y x
36. (EN) Um prisma quadrangular regular tem área lateral 36 6 3 + 3 ≤ 1
unidades de área. Sabendo que suas diagonais formam um ângulo de 
60º com suas bases, então a razão do volume de uma esfera de raio E = y + x ≥ 1 . O volume do sólido gerado, se E efetuar uma rotação
x ≥ 0
241 6 unidades de comprimento para o volume do prisma é 
y ≥ 0
8 8π
a) d) 
81π 27 de 270º em torno do eixo Ox em unidades de volume, é igual a
81π 81 26π 13π
b) e) a) c)
8 8π 3 2
8π 13π
c) b) 26π d)
81 3

488

PM_BOOK16 - MAT.indb 488 25/11/2022 19:24:54


ESFERA E TRONCOS

40. (EFOMM) Ao rotacionar o triângulo equilátero AOC em torno do 05. (ESPCEX) Um recipiente em forma de cone circular reto, com raio
eixo y, conforme ilustra a figura a seguir, obteremos um sólido. Assinale da base R e altura h, está completamente cheio com água e óleo.
a altemativa que representa o volume desse sólido em unidades de Sabe-se que a superfície de contato entre os líquidos está inicialmente
volume, sabendo que o vértice O do triângulo AOC sobrepõe-se à na metade da altura do cone. O recipiente dispõe de uma torneira
origem dos eixos. que permite escoar os líquidos de seu interior, conforme indicado na
figura. Se essa torneira for aberta, exatamente até o instante em que
toda água e nenhum óleo escoar, a altura do nível do óleo, medida a
partir do vértice será

πx 3 3 πx 3 3 7πx 3 3
a) c) e)
6 2 24

πx 3 3 d) πx 3 3
b)
4 24

3 3
EXERCÍCIOS DE 7 23
a) h d) h

COMBATE
2 2
3
3
7 23
b) h e) h
3 3
3
01. (ESPCEX) A angioplastia é um procedimento médico caracterizado 12
c) h
pela inserção de um cateter em uma veia ou artéria com o enchimento 2
de um pequeno balão esférico localizado na ponta desse cateter.
Considerando que, num procedimento de angioplastia, o raio
06. O sólido geométrico abaixo é formado por dois cones circulares
inicial do balão seja desprezível e aumente a uma taxa constante de
retos de mesma base. Sabendo-se que a seção que contém os pontos
0,5 mm/s até que o volume seja igual a 500 mm³, então o tempo, em
A e B é paralela à base comum dos cones e divide todo o sólido em
segundos, que o balão leva para atingir esse volume é
duas partes de igual volume, então o valor de x³ + y³ é:
2 3
a) 10. c) 10 3 . e) 10 3 .
π π
5
b) 10 3 . d) 10 3 π.
π

02. (ESPCEX) Considere que uma laranja tem a forma de uma esfera
de raio 4 cm, composta de 12 gomos exatamente iguais. A superfície
total de cada gomo mede:
43 π 42 π e) 43 π cm2
a) cm2 c) cm2
3 3

43 π 42 π
b) cm2 d) cm2
9 9

03. (ESPCEX) Um recipiente cilíndrico, cujo raio da base tem


medida R, contém água até uma certa altura. Uma esfera de aço é
mergulhada nesse recipiente ficando totalmente submersa, sem haver
9
transbordamento de água. Se a altura da água subiu R, então o a) 96 d) 162
raio da esfera mede 16
b) 128 e) 248
2 4 9
a) R c) R e) R c) 144
3 9 16
3 1 07. Um tronco de pirâmide regular tem como bases triângulos
b) R d) R
4 3 equiláteros, cujos lados medem, respectivamente, 2 cm e 4 cm. Se a
aresta lateral do tronco mede 3 cm, então o valor de sua altura h, em
cm, é tal que:
04. (EFOMM) Um recipiente na forma de um cilindro circular reto
contém um líquido até um certo nível. Colocando-se nesse recipiente a) 7 <h< 8
uma esfera, o nível do líquido aumenta 2 cm. Sabendo-se que o raio b) 6 <h< 7
do cilindro mede 3 2 cm, conclui-se que o raio da esfera, em cm,
c) 2 3 < h < 3 3
mede:
a) 2 c) 4 e) 6 d) 1 < h < 2
b) 3 d) 5 e) 2 2 <h<3 2

489

PM_BOOK16 - MAT.indb 489 25/11/2022 19:24:55


ESFERA E TRONCOS

08. (ITA) Uma taça em forma de cone circular reto contém um certo
volume de um líquido cuja superfície dista h do vértice do cone.
Adicionando-se um volume idêntico de líquido na taça, a superfície
4 (IME) Um tronco de pirâmide regular possui 12 vértices. A
soma dos perímetros das bases é 36 cm, a soma das áreas
das bases é 30 3 cm2 e sua altura mede 3 cm. Calcule o volume
do líquido, em relação à original, subirá de do tronco de pirâmide.
a) 3
2 − h. c) ( 3 2 − 1)h. h a) 50 cm³ 3 e) 42 3 cm3
e) . c) 43 cm3
d) h. 2 2
b) 3
2 − 1. 3
b) 42 cm3
3 d) 43 2 cm3
09. (EN) Um astronauta, em sua nave espacial, consegue observar, em
1

5
certo momento, exatamente da superfície da Terra. Que distância (ITA) Os volumes de um tronco de cone, de uma esfera de
10
ele está do nosso planeta? Considere o raio da Terra igual a 6400 km raio 5 cm e de um cilindro de altura 11 cm formam nessa
ordem uma progressão aritmética. O tronco de cone é obtido
a) 1200 km c) 1600 km e) 4200 km por rotação de um trapézio retângulo, de altura 4 cm e bases
b) 1280 km d) 3200 km medindo 5 cm e 9 cm, em torno de uma reta passando pelo lado
de menor medida. Então, o raio da base do cilindro é, em cm,
10. (EN) Considere dois cones circulares retos, de altura H e raio da igual a
base 1 cm, de modo que o vértice de cada um deles é o centro da base a) 2 2. c) 4. e) 2 6.
do outro. O volume comum aos dois cones coincide com o volume
do sólido obtido pela rotação do setor circular, sombreado na figura b) 2 3. d) 2 5.
abaixo, em torno do eixo l. O valor de H é, em cm,
GABARITO
EXERCÍCIOS DE FIXAÇÃO
01. C 04. C 07. D 10. D
02. A 05. D 08. D
03. A 06. C 09. C
EXERCÍCIOS DE TREINAMENTO
01. E 11. A 21. A 31. C
a) (2 + 3 )r 3
c)
4 3
3
r
e) 4r³ 02. C 12. C 22. E 32. E
03. E 13. E 23. A 33. B
b) 2 3r 3
d) 2r³ 04. D 14. D 24. C 34. D
05. D 15. C 25. D 35. A
06. C 16. D 26. B 36. C
07. A 17. E 27. C 37. C

DESAFIO PRO 08. D


09. C
18. E
19. A
28. A
29. D
38. C
39. C
10. C 20. E 30. A 40. B

1 (ITA) Seis esferas de mesmo raio R são colocadas sobre


uma superfície horizontal de tal forma que seus centros
definam os vértices de um hexágono regular de aresta 2R.
EXERCÍCIOS DE COMBATE
01. E 04. B 07. A 10. E
Sobre estas esferas é colocada uma sétima esfera de raio 2R que 02. A 05. A 08. C
tangencia todas as demais. Determine a distância do centro da 03. A 06. D 09. C
sétima esfera à superfície horizontal. DESAFIO PRO
01. d =
R +R 5 =
R(1 + 5)
2 (ITA) As superfícies de duas esferas se interceptam
ortogonalmente (isto é, em cada ponto da intersecção os
respectivos planos tangentes são perpendiculares). Sabendo que
02. a) d2 = 22 + (3/2)2 ⇔ d2 =
25
4
⇔d=
5
2
3
os raios destas esferas medem 2 cm e cm, respectivamente, A
calcule 2
2 3/2
a) a distância entre os centros das duas esferas.
b) a área da superfície do sólido obtido pela intersecção das O
1
d O
2
duas esferas.

3 (ITA) Em um plano estão situados uma circunferência ω de


raio 2 cm e um ponto P que dista 2 2 cm do centro de ω.
b) Logo a área total será
8π 2
cm +
9π 2 17π 2
cm = cm
Considere os segmentos PA e PB tangentes a ω nos pontos A e 5 5 5
B, respectivamente. Ao girar a região fechada delimitada pelos
segmentos PA e PB e pelo arco menor AB em torno de um eixo 03. a) A = 4 π + 8π + 8π = 20 π cm2
passando pelo centro de ω e perpendicular ao segmento PA, 16π 8π 3
b) V = 8π − = cm
obtém-se um sólido de revolução. Determine: 3 3
a) A área total da superfície do sólido. 04. E
b) O volume do sólido. 05. B

490

PM_BOOK16 - MAT.indb 490 25/11/2022 19:24:56


GEOMETRIA PLANA: TRIÂNGULOS,
CONGRUÊNCIA E PONTOS NOTÁVEIS

REVISÃO – PRINCIPAIS POLÍGONOS PIRÂMIDE REGULAR INSCRITA


REGULARES INSCRITOS EM UM PRISMA REGULAR
DE MESMA BASE

Figura 1

Podemos perceber que ambos os sólidos, pirâmide e prisma,


possuem a mesma altura (H) e a mesma área da base. Vamos chamar
de Sprisma a área da base do prisma e de Spir a área da base da
pirâmide.
Sendo assim o volume do prisma será Vprisma = Sprisma .H enquanto
1
o volume da pirâmide será Vpir = .Spir .H .
3
Fazendo a razão entre os volumes do prisma e da pirâmide
teremos:
Figura 2
Vprisma Sprisma .H S
= = 3 prisma
Vpir 1 Spir
.S .H
3 pir

Como o prisma e a pirâmide possuem como bases os mesmos


polígonos regulares, teremos que Sprisma = Spir e assim

Vprisma Sprisma
= 3= 3
Vpir Spir

A mesma razão é válida para um cone inscrito num cilindro.

Vcilindro
=3
Vcone

Figura 3

491

PM_BOOK16 - MAT.indb 491 25/11/2022 19:24:57


GEOMETRIA PLANA: TRIÂNGULOS, CONGRUÊNCIA E PONTOS NOTÁVEIS

PIRÂMIDE REGULAR INSCRITA Teremos a área do triângulo equilátero

EM UM PRISMA REGULAR  
( )
2

 l 3 3  3l2 3
DE BASE DIFERENTE =S ACE =
4

4
 
 

Assim vemos que a área do hexágono regular é o dobro da


área do triângulo equilátero. Sendo o hexágono regular a base
do prisma regular e o triângulo equilátero a base da pirâmide
regular teremos que
Vprisma Sprisma
=3 como Sprisma = 2Spir então
Vpir Spir

Vprisma S
= 3. ABCDE
= 3.2
= 6
Vpir S ACE

II. Pirâmide quadrangular regular inscrita num prisma


triangular regular

Podemos perceber que ambos os sólidos, pirâmide e prisma,


possuem a mesma altura (H). Vamos chamar de Sprisma a área da base
do prisma e de Spir a área da base da pirâmide.
Sendo assim o volume do prisma será Vprisma = Sprisma .H enquanto
1
o volume da pirâmide será Vpir = .Spir .H .
3

Fazendo a razão entre os volumes do prisma e da pirâmide


teremos:
Vprisma Sprisma .H S
= = 3 prisma
Vpir 1 Spir
.S .H
3 pir
Ou seja, a razão entre os volumes será o triplo da razão entre as
áreas das bases.
Vamos mostrar alguns casos
I. Pirâmide triangular regular inscrita num prisma
hexagonal regular

Chamando o lado do hexágono regular ABCDEF de l teremos


que o lado do triângulo equilátero ACE será l 3 . Fazendo a
área do hexágono teremos

 l2 3  3l2 3
S ABCDEF 6.=
=  
 4  2
 

492

PM_BOOK16 - MAT.indb 492 25/11/2022 19:24:58


GEOMETRIA PLANA: TRIÂNGULOS, CONGRUÊNCIA E PONTOS NOTÁVEIS

Sendo o lado do triângulo equilátero ABC igual a l e o lado A razão entre os volumes do cilindro reto e do prisma regular
do quadrado MNPQ igual a x montaremos as razões entre os inscrito em sua base dar-se-á por
triângulos semelhantes AMN e ABC.
Vcilindro Scilindro .H Scilindro
AH BC = =
= Vprisma Sprisma .H Sprisma
AO PQ
Teremos então que a razão entre os volumes será dada pela razão
l 3 entre as áreas das bases, que com a ajuda da revisão dos principais
l x 3 l 3 x 3 l 3  3  l 3
2 = ⇒ = −x ⇒ +x = ⇒ x + 1 = polígonos regulares inscritos poderemos definir para os principais
l 3 x 2 2 2 2  2  2
−x   prismas regulares retos.
2
I. Prisma triangular regular inscrito no cilindro reto

=x
l 3
=2 l 3 2− 3
. =
l 3 2− 3( ) ( ) Scilindro = πR2
3 +2 2+ 3 2− 3 4 −3
( ) A relação entre o lado do triângulo inscrito e o raio do círculo
2
circunscrito é dado por (figura 1)
x= 2l 3 − 3l
= l 2 3 −3 ( ) l=R 3
Assim a área do quadrado MNPQ será
(R 3 ) =3
2

l2 3 3 3R2
(l(2 3 − 3)) = l (4.3 − 12 3 + 9)
2
SMNPQ = 2 Assim Sprisma
= = , logo
4 4 4

SMNPQ =l ( 21 − 12 3 ) =l ( 3 ( 7 − 4 3 ) ) =3l ( 7 − 4 3 )
2 2 2

Vcilindro Scilindro πR2 4π 4 3π


= = = =
Enquanto a área do triângulo equilátero será dada por Vprisma Sprisma 3 3R2 3 3 9
l2 3 4
S ABC =
4
II. Prisma quadrangular regular inscrito no cilindro reto
Assim a razão entre os volumes do prisma e da pirâmide será
l2 3 Scilindro = πR2
l2 3 como Sprisma
= S= ABC
2 e
Sprisma
= S= ABC
4
2 4
A relação entre o lado do quadrado inscrito e o raio do círculo
Spir S=
= MNPQ (
3l2 7 − 4 3 ) circunscrito é dado por (figura 2)

Então l=R 2

( )
2
l 3
( )
2
Assim Sprisma= l2= R 2 = 2R2 , logo
Vprisma S ABC 3 7+4 3 7 3 + 4.3 12 + 7 3
= 3.= 3. =4 . = =
Vpir SMNPQ 3l 7 − 4 3
2
4 7−4 3 7+4 3( 4 ( 49 −) (
16.3 ) )( )4
Vcilindro Scilindro πR2 π
= = =
Vprisma Sprisma 2R2 2
S ABC
3.= 3.
l2 3
4
=
3 7+4 3
. =
(
7 3 + 4.3
=
)
12 + 7 3
SMNPQ
(
3l 7 − 4 3
2
) (
4 7−4 3 7+4 3 4 )(
( 49 − 16.3 ) )4 III. Prisma triangular regular inscrito no cilindro reto
Scilindro = πR2

A relação entre o lado do hexágono inscrito e o raio do círculo


PRISMA REGULAR INSCRITO EM UM circunscrito é dado por (figura 3)

CILINDRO RETO l=R


l 3 2  R2 3  3 3R2
Sprisma 6=
Assim=   6 =  , logo
 4   4  2
   

Vcilindro Scilindro πR2 2π 2 3π


= = = =
Vprisma Sprisma 3 3R2 3 3 9
2

493

PM_BOOK16 - MAT.indb 493 25/11/2022 19:24:59


GEOMETRIA PLANA: TRIÂNGULOS, CONGRUÊNCIA E PONTOS NOTÁVEIS

PIRÂMIDE REGULAR INSCRITA EM 1


Sendo assim o volume do cone será Vcone = .Scone .H enquanto o
UM CILINDRO RETO 3
1
volume da pirâmide será Vpir = .Spir .H .
3
Fazendo a razão entre os volumes do cone e da pirâmide:

1
Vcone 3 .Scone .H Scone
= =
Vpir 1 Spir
.S .H
3 pir

A área da base do cone é dada por Scone = π.R2 e utilizando os


polígonos regulares das principais pirâmides regulares teremos os
mesmos resultados que os encontrados nos prismas regulares inscritos
nos cilindros retos.
I. Pirâmide triangular regular inscrita no cone reto

Vcone 4 3π
=
Vpir 9

De maneira análoga ao item anterior teremos II. Pirâmide quadrangular regular inscrita no cone reto

Vcilindro Scilindro .H S Vcone π


= = 3. cilindro =
Vpir 1 Sprisma Vpir 2
.S .H
3 prisma
III. Prisma triangular regular inscrita no cone reto
Então vemos que temos o triplo das razões anteriores, assim
Vcone 2 3π
I. Pirâmide triangular regular inscrita no cilindro reto
Vpir 9
Vcilindro Vcilindro 4 3π 4 3π
= 3.= 3.=
Vpir Vprisma 9 3
CONE RETO INSCRITO EM UMA
II. Pirâmide quadrangular regular inscrita no cilindro reto
PIRÂMIDE REGULAR
Vcilindro V π 3π
= 3. cilindro
= 3.=
Vpir Vprisma 2 2

III. Pirâmide triangular regular inscrita no cilindro reto

Vcilindro Vcilindro 2 3π 2 3π
= 3.= 3.=
Vpir Vprisma 9 3

PIRÂMIDE REGULAR INSCRITA EM


UM CONE RETO

Podemos perceber que ambos os sólidos, pirâmide e cone,


possuem a mesma altura (H). Vamos chamar de Scone a área da base
do cone e de Spir a área da base da pirâmide.
1
Sendo assim o volume do cone será Vcone = .Scone .H enquanto o
3
1
Podemos perceber que ambos os sólidos, pirâmide e cone, volume da pirâmide será Vpir = .Spir .H .
3
possuem a mesma altura (H). Vamos chamar de Scone a área da base
do cone e de Spir a área da base da pirâmide.

494

PM_BOOK16 - MAT.indb 494 25/11/2022 19:25:00


GEOMETRIA PLANA: TRIÂNGULOS, CONGRUÊNCIA E PONTOS NOTÁVEIS

Fazendo a razão entre os volumes da pirâmide e do cone teremos: a


É fácil ver que h = , assim
1 2
Vpir .S .H Spir
= 3 pir
= 2

Vcone 1 Scone 2  a 2  a 2 a2 .2 a a3
.S .H Voctaedro
= . =  . =. .
3 cone 3  2  2 3 4 2 6

A área da base do cone é dada por Scone = π·R² enquanto, que para
1
a área da pirâmide podemos utilizar a fórmula da área de um polígono Isso quer dizer que o volume do octaedro regular é do volume
regular Spir = p · m, onde m é o apótema da base e p é o semiperímetro do cubo. 6
do polígono. Como neste caso o círculo da base do cone está inscrito
no polígono da base da pirâmide teremos que o apótema do polígono
será igual ao próprio raio do círculo, assim m = R. Assim CONE RETO INSCRITO NA ESFERA
Vpir Spir p.m p.R p
= = = =
Vcone Scone π.R2 π.R2 πR

O que nos faz concluir que a razão entre os volumes é a razão


entre os perímetros das bases.

OCTAEDRO FORMADO PELOS


CENTROS DAS FACES DE UM CUBO

Podemos fazer a assimilação do triângulo inscrito no círculo.

Vamos ligar 2 centros de faces laterais consecutivas do cubo.

Utilizaremos a fórmula da área do triângulo em função do raio


abc
do círculo circunscrito S ∆ = , onde poderemos calcular a área do
4R
2r ·H
triângulo como S ∆ = . No caso analisado os lados do triângulo
2
VAB serão g,g e 2r. Assim
2r ·g·g 2r · H 2rg2 g2
S∆ = ⇒ = ⇒ 2RH = g2 ⇒ R =
Agora temos o triângulo retângulo indicado na figura e dessa 4R 2 4R 2H
forma podemos calcular a aresta do octaedro regular em função da
aresta a do cubo. Vamos chamar a aresta do octaedro regular de x. ProBizu
2 2 No caso do cone equilátero podemos imaginar o caso do
a a a2 a2 a2 a2 a a 2
x2 =   +   = + = ⇒x= = ⇒x= triângulo equilátero inscrito no círculo de raio R, onde
2
   2 4 4 2 2 2 2
2 2  2r 3  2r 3
R= H=   ⇒ R=
Dessa forma o volume do octaedro regular será o dobro do volume 3 3  2  3
de uma das pirâmides quadrangulares regulares que o formam.

1 
Voctaedro = 2  .Squadrado .h 
3 

495

PM_BOOK16 - MAT.indb 495 25/11/2022 19:25:01


GEOMETRIA PLANA: TRIÂNGULOS, CONGRUÊNCIA E PONTOS NOTÁVEIS

CILINDRO RETO INSCRITO NA ESFERA INSCRITA NO CONE RETO


ESFERA

Neste caso podemos fazer a vista frontal e utilizarmos a ideia de


um retângulo inscrito num círculo de raio R.

Podemos fazer a assimilação do triângulo circunscrito ao círculo.

Assim podemos relacionar nossos elementos aplicando o Teorema


de Pitágoras no triângulo retângulo BCD.

(2R ) (2r )
2 2
= + H2

4R
=2
4r 2 + H2
Utilizaremos a fórmula da área do triângulo em função do raio
H= 4R2 − 4r 2 = (
4 R2 − r 2 ) do círculo inscrito S ∆ = p·R , onde poderemos calcular a área do
2r·H
triângulo como S ∆ = . No caso analisado os lados do triângulo
=H 2 R2 − r 2 ( ) 2
VAB serão g,g e 2r. Assim

ProBizu ( g + g + 2r ) .R ⇒ 2r.H =
No caso do cilindro equilátero podemos imaginar o caso do
S∆ =
2 2
(r + g) .R ⇒ R = rrH
+g
quadrado inscrito no círculo de raio R, onde teremos:

H =2r ⇒ 4R2 =4r 2 + ( 2r ) ⇒ 4R2 =4r 2 + 4r 2 ⇒ 4R2 =8r 2


2
ProBizu
No caso do cone equilátero podemos imaginar o caso do
R2 = 2r 2 ⇒ R = r 2
triângulo equilátero circunscrito ao círculo de raio R, onde g = 2r e
2r 3
=H = r 3 , assim
2

=R =
( )
rH r. r 3
= =
r2 3 r 3
r+g r + 2r 3r 3

496

PM_BOOK16 - MAT.indb 496 25/11/2022 19:25:02


GEOMETRIA PLANA: TRIÂNGULOS, CONGRUÊNCIA E PONTOS NOTÁVEIS

ProBizu O diâmetro da esfera é igual a aresta do cubo, assim


a.
A esfera só pode ser inscrita no cilindro equilátero. 2R =a ⇒ R =
2

Observação
Uma ideia muito interessante que podemos desenvolver é a da
distância no espaço e consequentemente a decomposição de um
sólido em sólidos menores que quando tem seus volumes somados
resultam no volume do sólido original. Vamos ver um exemplo de
como podemos utilizar essa ideia na esfera inscrita no tetraedro
regular.

ESFERA INSCRITA NO TETRAEDRO


REGULAR
Seja uma esfera de centro O inscrita em um tetraedro regular
VABC como na figura.
H
No cilindro equilátero H = 2r, assim R
= = r
2

CUBO INSCRITO NA ESFERA

Ao traçarmos as distâncias do centro O da esfera as 4 faces VAB,


VAC, VBC e ABC do tetraedro regular teremos todas essas 4 distâncias
iguais ao raio da esfera (r).

A diagonal do cubo é o diâmetro da esfera, assim

a 3
2R= a 3 ⇒ R=
2

ESFERA INSCRITA NO CUBO

Ao ligarmos o centro O da esfera aos 4 vértices do tetraedro


formaremos 4 tetraedros OVAB, OVBC, OVCA e OABC todos de
alturas relativas as bases VAB, VBC, VCA e ABC iguais a r.

497

PM_BOOK16 - MAT.indb 497 25/11/2022 19:25:02


GEOMETRIA PLANA: TRIÂNGULOS, CONGRUÊNCIA E PONTOS NOTÁVEIS

Exercício Resolvido

01. (UNITAU) Um cone circular reto está perfeitamente inscrito em


um cubo, conforme ilustrado na figura abaixo.

Sabendo-se que a aresta do cubo mede 6 cm, o volume do cone é


a) 18π cm3
18
b) π cm3
4
As bases VAB, VBC, VCA e ABC são todas de mesma área,
pois são os mesmos triângulos equiláteros que formam o tetraedro
c) 16π cm3
regular VABC. 14
d) π cm3
S= S= S= S ABC 4
VAB VBC VCA
e) 16 cm³
Assim podemos concluir que o volume do tetraedro VABC pode ser
decomposto no volume dos tetraedros OVAB, OVBC, OVCA e OABC. Resolução: A

VVABC = VOABC + VOVAB + VOVAC + VOVBC

1 1 1 1 1 r
⋅ S ⋅H =
3 ABC
⋅ S ⋅ r + ⋅ S VAB ⋅ r + ⋅ S VAC ⋅ r + ⋅ S VBC ⋅ r =
3 ABC 3 3 3
( S + SVAB + SVAC + SVBC )
3 ABC

Como S=
VAB
S=
VBC
S=
VCA
S ABC teremos que¨
1 r H
· S ABC ·H = ( S ABC + S VAB + S VAC + S VBC ) ⇒ H = 4r ⇒ r =
3 3 4

H
r=
4

Perceba que se tivéssemos um ponto P qualquer interior ao


tetraedro teríamos as distâncias em relação as 4 faces: d1, d2, d3 e d4.
Assim teríamos
1 1 1 1 1
⋅ S ⋅ H = ⋅ S ABC ⋅ d1 + ⋅ S VAB ⋅ d2 + ⋅ S VAC ⋅ d3 + ⋅ S VBC ⋅ d4
3 ABC 3 3 3 3 A altura do cone é a aresta do cubo ⇒ h = a = 6 cm
Procedendo de maneira análoga teremos que Já no raio da base do cone vale: 2R = a = 6 ⇒ R = 3 cm
1 1 1
Logo: V = ⋅ Sb ⋅ h = ⋅ πR2 ⋅ h = π ⋅ ( 3) ⋅ 6 = 18 π cm3
2
H = d1 + d2 + d3 + d4
3 3 3

498

PM_BOOK16 - MAT.indb 498 25/11/2022 19:25:03


GEOMETRIA PLANA: TRIÂNGULOS, CONGRUÊNCIA E PONTOS NOTÁVEIS

Exercício Resolvido EXERCÍCIOS DE

02. (USP) A menor esfera na qual um paralelepípedo reto‐retângulo


de medidas 7 cm x 4 cm x 4 cm está inscrito tem diâmetro de
FIXAÇÃO
a) 9 cm
b) 10 cm 01. O raio da esfera inscrita em um cilindro equilátero de volume 64 π
cm³, em cm, é
c) 11 cm
a) 4
d) 12 cm
e) 15 cm b) 23 4
c) 23 8
Resolução: A d) 2
Todo paralelepípedo reto retângulo pode ser inscrito numa esfera e) 2 3
e sua diagonal será o diâmetro da esfera.
02. O volume do cubo circunscrito a uma esfera de raio 2 cm é igual a
a) 8 cm3 c) 64 cm3 e) 144 cm3
b) 36 cm 3
d) 125 cm 3

03. O volume do cone equilátero que circunscreve a esfera de raio 1


cm é igual a
a) 3π cm³ c) 2π cm³ e) 4 π cm³
b) 3 π cm³ d) 2 π cm³

04. Uma esfera de raio de 3cm é colocada dentro de um cubo, de


forma que a esfera fique tangente a cada uma das seis faces do
cubo. O volume, em centímetros cúbicos, da região interna ao cubo
e externa a esfera é: (Se necessário, considere π = 3.)
a) 96
b) 108
Observe que o raio da esfera independe da ordem que vamos dar
c) 132
às 3 dimensões, pois D = a2 + b2 + c2 , e dessa maneira podemos
construir o seguinte sistema com as informações apresentadas: d) 148

x= 4 + 4
2 2 2
05. Determine o volume de um cubo que está inscrito numa esfera
 2
D= x + 7
2 2
de raio igual a 4,5 cm.
a) 27 cm³
D2 = 42 + 42 + 42
b) 27 3 cm3
D= 16 + 16 + 49= 81 ⇒ D= 9 cm c) 36 2 cm3
d) 81 cm³
e) 81 3 cm3

06. Uma esfera com raio de medida 3 m está inscrita em um cone


reto, cuja base é um círculo de área 12 πm2. O volume do cone é:
a) 288 πm³.

b) 16π 2 m3 .
c) 96 πm³.

d) 128π 2 m3 .

07. (EEAR) Se um cilindro reto está circunscrito a uma esfera de raio


“R”, então a razão entre a área da superfície esférica e a área total
do cilindro é
a) 1 2
c)
3
1 4
b) d)
2 5

499

PM_BOOK16 - MAT.indb 499 25/11/2022 19:25:05


GEOMETRIA PLANA: TRIÂNGULOS, CONGRUÊNCIA E PONTOS NOTÁVEIS

08. A soma de todas as arestas de um cubo é 36 cm. Uma estera está EXERCÍCIOS DE
inscrita nesse cubo (figura abaixo). A área da superfície dessa esfera é
TREINAMENTO
01. Um cone invertido de altura 8 cm com geratriz 14 cm estava
completamente cheio de um determinado líquido. Quando foi inserido
uma esfera de raio r = 2 cm, conforme figura, ao inserir a esfera uma
quantidade do líquido transbordou. Determine a quantidade do
líquido que restou no cone.

a) 3 π cm² c) 9 π cm²
b) 6 π cm² d) 16 π cm²

09. Numa esfera de raia R circunscrevemos um cone reto cuja geratriz


é igual ao diâmetro da base. O raio da base do cone é r. O volume do
cone em função do raio R da esfera é


a)
3
( 132 − 4) cm3

π
b)
3
( 132 − 4) cm3


c)
3
( 132 − 8) cm3


d)
3
( 132 − 8) cm3


a) πR3
e)
3
(
4 132 − 1) cm3

b) 2πR3 02. Uma esfera está mergulhada em um tanque com formato


c) 3πR 3 cilíndrico. O volume restante no tanque está completamente
preenchido com água, como ilustra a figura.
d) 3πR3

10. Na figura, a esfera está inscrita em um cilindro reto. Sejam V1 o


volume da estera e V2 o volume do cilindro. Marque a afirmativa correta

O raio, tanto da esfera como do cilindro, é r, e a altura do cilindro é


de 2r. Se a esfera for retirada do tanque, como indicado na figura,
a) V1 = V2 a altura h, do nível da água, corresponderá a que fração da altura
do cilindro?
b) V2
V1 = 1 2 5
3 a) . c) . e) .
5 3 4
2V2
c) V1 = 1 4
3 b) . d) .
3 3
3V2
d) V1 =
5

500

PM_BOOK16 - MAT.indb 500 25/11/2022 19:25:05


GEOMETRIA PLANA: TRIÂNGULOS, CONGRUÊNCIA E PONTOS NOTÁVEIS

09. A área da superfície lateral de um cone equilátero inscrito numa


03. Calcule os raios das bases de um tronco de cone reto, no qual esfera de raio R é:
foi inscrita uma esfera com raio de 4 cm, de modo que o volume do π R2 3
tronco seja três vezes o volume da esfera. a)
2
a) 6 + 2 13 e 6 − 13 π R2 3
b)
b) 6 − 2 13 e 6 + 2 13 3
3 π R2
c) 6 + 2 13 e 2 13 − 6 c)
4
d) 6 − 2 13 e 3 + 2 13 3 π R2
d)
2
04. (PUCRS) A circunferência de uma bola de voleibol é 66 cm. Para
colocá-la em uma caixa cúbica, essa caixa deve ter, no mínimo, uma e) 3 π R²
aresta interna, em centímetros, de
10. Os vértices de um tetraedro regular são também vértices de um
a) 33 66
d) cubo de aresta 2. A área de uma face desse tetraedro é
π
33
b) a) 2 3
π π
e) b) 4
66
c) 66 c) 3 2

05. Se um cilindro reto está circunscrito a uma esfera de raio “R”, d) 3 3


então a razão entre a área da superfície esférica e a área total do e) 6
cilindro é
a) 1 11. (FUVEST) A menor esfera na qual um paralelepípedo reto‐retângulo
de medidas 7 cm x 4 cm x 4 cm está inscrito tem diâmetro de
1
b) a) 9 cm. c) 11 cm. e) 15 cm.
2
2 b) 10 cm. d) 12 cm.
c)
3
d) 2 12. Uma esfera está inscrita num cilindro equilátero cuja área lateral
mede 16 π cm². O volume da esfera inscrita é
06. Um cubo cuja medida de cada aresta é 3 dm está inscrito em uma a) 8 π
esfera de raio R. A medida de um diâmetro (2R) da esfera é b) 16 π
a) 2 3 dm. 32
c) π
b) 3 2 dm. 3

c) 256
3 3 dm. d) π
3
d) 4 3 dm.
13. Uma esfera de raio 10 cm está inscrita em um cone equilátero. O
07. A razão entre a área lateral do cilindro equilátero e da superfície volume desse cone, em cm³, é igual a
esférica, da esfera nele inscrita, é: a) 1000 π c) 2000 π e) 3000 π
a) 1 b) 1500 π d) 2500 π
1
b) .
3 14. (UECE) Como mostra a figura, o cilindro reto está inscrito na
1 esfera de raio 4 cm.
c) .
3
1
d) .
4
2
e) .
3

08. A razão entre os volumes das esferas circunscrita e inscrita a um


mesmo cubo é:

a) 3

b) 2 3
c) 3 3
(3)
d) 4 Sabe-se que o diâmetro da base e a altura do cilindro possuem a
3
mesma medida. O volume do cilindro é
(3)
e) 3 a) 18 π 2 cm3 c) 32 π 2 cm3
2
b) 24 π 2 cm3 d) 36 π 2 cm3

501

PM_BOOK16 - MAT.indb 501 25/11/2022 19:25:07


GEOMETRIA PLANA: TRIÂNGULOS, CONGRUÊNCIA E PONTOS NOTÁVEIS

15. (ESPCEX) O volume de uma esfera inscrita em um cubo com 20. (UEMG) Observe as figuras.
volume 216 cm³ é igual a
a) 38 π cm³.
b) 36 π cm³
c) 34 π cm³
d) 32 π cm³
e) 30 π cm³

16. (AFA) O volume, em cm³, do octaedro regular inscrito numa esfera


com volume 36π cm³ é
a) 18
b) 36
c) 54
d) 72 Nas figuras acima, tem-se um cilindro circular equilátero (S1),
circunscrevendo um cone (S2), e um cilindro circular oblíquo
17. (UERJ) Uma esfera de centro A e raio igual a 3 dm é tangente ao (S3). A razão determinada pelo volume de S3 com a superfície
plano α de uma mesa em um ponto T. Uma fonte de luz encontra-se em total de S2 é
um ponto F de modo que F, A e T são colineares. Observe a ilustração:
5 −1 5 + 16
a) cm. c) cm.
4 4
b) 5 − 1cm. d) 5 + 16 cm.

21. (ESPCEX) Calculando-se o volume de uma esfera circunscrita a um


cone equilátero cujo raio da base mede 3 cm , obtém-se
8π 64 π
a) cm3 . d) cm3 .
3 3
4π 32π
b) cm3 . e) cm3 .
3 3
Considere o cone de vértice F cuja base é o círculo de centro T definido
pela sombra da esfera projetada sobre a mesa. 16π
c) cm3 .
Se esse círculo tem área igual à da superfície esférica, então a distância 3
FT , em decímetros, corresponde a:
22. Nesta figura, estão representados o cubo ABCDEFGH e o sólido
a) 10 OPQRST:
b) 9
c) 8
d) 7

18. (AFA) Seja P uma pirâmide cujo vértice é o centro de uma das
faces de um cubo de aresta α e cuja base é a face oposta. Então, a
área lateral dessa pirâmide é igual a.
a) a2 5
b) 2a2 3
c) a2 3
a2 5
d)
4

19. (ESPCEX) Uma esfera de raio 10 cm está inscrita em um cone


equilátero. O volume desse cone, em cm³, é igual a Cada aresta do cubo mede 4 cm e os vértices do sólido OPQRST são
a) 1000 π os pontos centrais das faces do cubo.
b) 1500 π Então, é correto afirmar que a área lateral total do sólido OPQRST mede
c) 2000 π a) 8 2 cm².
d) 2500 π b) 8 3 cm².
e) 3000 π c) 16 2 cm².
d) 16 3 cm².

502

PM_BOOK16 - MAT.indb 502 25/11/2022 19:25:08


GEOMETRIA PLANA: TRIÂNGULOS, CONGRUÊNCIA E PONTOS NOTÁVEIS

23. (UECE) Considerando-se um cubo cuja medida de cada aresta é 26. A esfera ε, de centro O e raio r > 0, é tangente ao plano α. O plano
igual a 1 m pode-se afirmar corretamente que a medida do volume do β é paralelo a α e contém O. Nessas condições, o volume da pirâmide
poliedro convexo cujos vértices são os centros das faces desse cubo é que tem como base um hexágono regular inscrito na intersecção de ε
2 3 com β e, como vértice, um ponto em α, é igual a
a) m.
3 3r 3
a)
2 3 4
b) m.
7 5 3r 3
b)
1 3 16
c) m.
6 3 3r 3
c)
4 3 8
d) m.
7
7 3r 3
d)
16
24. (UFRGS) Considere um cubo de aresta a. Os pontos I, J, K, L, M e
N são os centros das faces ABCD, BCFG, DCGH, ADHE, ABFE e EFGH, 3r 3
e)
respectivamente, conforme representado na figura abaixo. 2

27. Considerando-se um cubo cuja medida de cada aresta é igual a 1


m pode-se afirmar corretamente que a medida do volume do poliedro
convexo cujos vértices são os centros das faces desse cubo é
2 3
a) m.
3
2 3
b) m.
7
1 3
c) m.
6
4 3
d) m.
7

28. Uma esfera está circunscrita a um cubo cuja medida da aresta é


2 m. A medida do volume da região exterior ao cubo e interior à
esfera é
O octaedro regular, cujos vértices são os pontos I, J, K, L, M e N, tem
aresta medindo a) (
4 π 3 − 2 m³ )
a) a 3. b) 3( π 3 + 2) m³
b) a 2.
a 3
c) (
4 π 3 + 2 m³)
c) .
2
d) (
3 π 3 − 2 m³)
a 5
d) .
2 29. (AFA) Na figura seguinte, tem-se uma esfera de maior raio contida
num cone reto e tangente ao plano da base do mesmo. Sabe-se que
a 2
e) . o raio da base e a altura desse cone são, respectivamente, iguais a
2 6 cm e 8 cm. A metade do volume da região do cone exterior à esfera
é, em cm³, igual a.
25. Um cubo de aresta m está inscrito em uma semiesfera de raio R
de tal modo que os vértices de uma das faces pertencem ao plano
equatorial da semiesfera e os demais vértices pertencem à superfície
da semiesfera. Então, m é igual a
2
a) R
3
2
b) R
2
3
c) R
3
d) R a) 66 π
3 b) 48 π
e) R
2 c) 30 π
d) 18 π

503

PM_BOOK16 - MAT.indb 503 25/11/2022 19:25:09


GEOMETRIA PLANA: TRIÂNGULOS, CONGRUÊNCIA E PONTOS NOTÁVEIS

30. A Figura a seguir mostra um cilindro reto, um cone reto e uma 02. A figura abaixo possui um cone interno ao prisma hexagonal
esfera que tangencia a base do cilindro e as geratrizes do cilindro e do inscrito no cilindro de raio r. A razão entre a área da base do cone
cone. O cone e o cilindro têm como base um círculo de raio 7 cm e a A  1
mesma altura que mede 24 cm. A2 e a área da base do cilindro A1 é  2  = . Nessas condições,
 A1  3
calcule a diferença entre o volume de ar contido no prisma hexagonal
(externo ao cone) e o volume de ar contido no cilindro (externo ao
prisma hexagonal) sabendo que a altura dos sólidos é 4r. E, assinale a
alternativa CORRETA

Qual o volume, em centímetros cúbicos, da região interior ao cilindro


e exterior à esfera e ao cone?
a) 800 π c) 748 π e) 648 π
40 3
b) 784 π d) 684 π a) πr
9

32 3
b) πr
EXERCÍCIOS DE 9

COMBATE c) (
r 3 12 3 − 5π )
4r 3
d)
9
(
27 3 − 10π )
01. (AFA) Um cubo tem quatro vértices nos pontos médios das arestas
laterais de uma pirâmide quadrangular regular, e os outros quatro na 4r 3
base da pirâmide, como mostra a figura abaixo. e)
9
(
24 3 − 10π )
03. (AFA) Se uma pirâmide hexagonal regular está inscrita num cone
10 3
equilátero cujo volume é igual a π cm3 , então o volume dessa
pirâmide, em cm³, é igual a 7
45
a)
7
15 3
b)
7
30 3
c)
7
135
d)
7

04. (EN) Um fabricante de bolas de tênis (bolas em formatos esféricos)


deseja vender as bolas em embalagens cilíndricas (cilindros circulares
retos) de raio R e altura H, cada uma. Em cada embalagem há n bolas
de tênis de raio R, cada bola. O fabricante deseja que área total das
superfícies das bolas seja igual à área lateral da embalagem (cilindro).
A razão entre os volumes do cubo e da pirâmide é Dessa forma, é correto afirmar que:
3 H 2H
a) a) R= . d) R= .
4 n 3n
1 H 3H
b) R= e) R= .
2 b) .
2n 4n
3
c) H
8 c) R= .
3n
1
d)
8

504

PM_BOOK16 - MAT.indb 504 25/11/2022 19:25:12


GEOMETRIA PLANA: TRIÂNGULOS, CONGRUÊNCIA E PONTOS NOTÁVEIS

05. (AFA) Um cone equilátero tem, em seu interior, duas esferas 07. (AFA) Nela está representada a inscrição de uma esfera num cubo
tangentes entre si e tangentes ao cone, conforme figura a seguir. que, por sua vez, está inscrito num cone equilátero, de tal forma que
uma de suas faces está apoiada na base do cone e os vértices da face
oposta estão na lateral do cone. A projeção ortogonal do vértice do
cone à sua base contém dois pontos de tangência da esfera com o
cubo. Se R e r são, respectivamente, as medidas do raio da base do
cone e do raio da esfera, em cm, então

A distância do vértice do cone ao ponto de tangência entre o cone e a


esfera de menor raio é igual a π 3 cm . O volume desse cone, em cm³,
é igual a
a) 81π4 c) 243π4
b) 81π 3
d) 243π3

06. (EFOMM) Seja a pirâmide quadrangular regular ABCDE com aresta


da base 4 2 e aresta lateral 8. Considere o prisma quadrangular
regular interior à pirâmide. O prisma possui base inferior sobre a R 3+2 3
base da pirâmide e os vértices da base superior estão sobre as arestas a) =
r 3
laterais da pirâmide, como sugere a figura abaixo.
r 3 2−2 3
b) =
R 2
R 2 6 +3 2
c) =
r 3
r 2 6 −3 2
d) =
R 2

08. (EFOMM) Seja a esfera de raio R inscrita na pirâmide quadrangular


regular de aresta base 2 cm e aresta lateral 38 cm . Sabendo-se que
a esfera tangencia todas as faces da pirâmide, o valor de R, em cm, é
1
a) 37 +
6
1
b) 39 −
38
12
c) 6 38 +
17
1
d) 37 −
6
12
O volume máximo do prisma é igual a e) 6 38 −
17
512 3
a)
27 09. (IME CG) Um cubo está inscrito em um cone reto. O raio da base
do cone é igual a r e a sua área lateral é igual ao dobro de sua área da
128 3 base. Determine a aresta do cubo em função de r.
b)
9
a) 2( 3 − 2 )r

c)
256 3
b) 2( 3 + 2 )r
9
c) (3 3 − 1) r
64 3
d) d) (2 2 − 1) r
3
e) (3 2 − 2 3 )r
256 3
e)
27

505

PM_BOOK16 - MAT.indb 505 25/11/2022 19:25:14


GEOMETRIA PLANA: TRIÂNGULOS, CONGRUÊNCIA E PONTOS NOTÁVEIS

6
10. (ITA) Um cilindro reto de altura
3
cm está inscrito num tetraedro
regular e tem sua base em uma das faces do tetraedro. Se as arestas
4 (ITA) P uma pirâmide regular cujo vértice V é um dos
vértices de um cubo de lado l e cuja base é o hexágono
formado pelos pontos médios das seis arestas do cubo que
do tetraedro medem 3 cm, o volume do cilindro, em cm³, é igual a não contém V nem o vértice oposto a V. O raio da esfera que
π 3 π 6 circunscreve P é
a) d)
4 9 l 2
a) .
π 3 π 12
b) e)
6 3 l 3
b) .
π 6 12
c)
6 5l 2
c) .
12

5l 3

DESAFIO PRO
d) .
12

l 3
e) .
6

1 (IME) Considere uma pirâmide regular de base hexagonal e


altura h. Uma esfera de raio R está inscrita nesta pirâmide.
O volume desta pirâmide é
5 (IME) Seja um cubo regular, onde os centros de suas faces
são vértices de um octaedro. Por sua vez, os centros das
faces deste octaedro formado são vértices de outro cubo.
2h 3 R2h h 3 R2h Obtendo consecutivamente octaedros e cubos infinitamente,
a) d) determine a razão da soma do volume de todos os poliedros
3 h − 2R 3 h − 2R
inscritos pelo volume do cubo inicial.
h 3 R2h 2h 3 R2h
b) e)
3 h + 2R 3 h−R GABARITO
2h 3 R2h
c) EXERCÍCIOS DE FIXAÇÃO
3 h + 2R
01. B 04. B 07. C 10. C

2 (IME) Seja um tetraedro regular ABCD de aresta a e 02. C 05. E 08. C


um octaedro inscrito no tetraedro, com seus vértices 03. B 06. C 09. D
posicionados nos pontos médios das arestas do tetraedro. EXERCÍCIOS DE TREINAMENTO
Obtenha a área da seção do octaedro formada pelo plano
horizontal paralelo à base do tetraedro BCD, distando desta 01. A 09. D 17. C 25. A
base de um quarto da altura do tetraedro. 02. B 10. A 18. A 26. E
3 2 3 3 2 03. C 11. A 19. E 27. C
a) a d) a
192 64 04. D 12. C 20. B 28. A
05. C 13. E 21. E 29. C
3 2 e) 9 3 2
b) a a 06. C 14. C 22. D 30. C
96 64
07. A 15. B 23. C
3 3 2 08. C 16. B 24. E
c) a
32
EXERCÍCIOS DE COMBATE
01. C 04. B 07. B 10. D
3 (ITA) Uma esfera S1, de raio R > 0 está inscrita num cone
circular reto K. Outra esfera, S2, de raio r, com 0< r < R, está
contida no interior de K e é simultaneamente tangente à esfera
02. D
03. A
05. A
06. A
08. D
09. E
S1 e à superficie lateral de K. O volume de K é igual a DESAFIO PRO
πR5 01. A 02. C 03. B 04. D
a) .
3r (R − r ) 05.

2πR5 a) 20πcm2
b) .
3r (R − r ) 8π
cm3
b) V =
3
πR5
c) .
r (R − r )
ANOTAÇÕES
5
4 πR
d) .
3r (R − r )

5πR5
e) .
3r (R − r )

506

PM_BOOK16 - MAT.indb 506 25/11/2022 19:25:15


GEOMETRIA ANALÍTICA:
COORDENADAS NO PLANO E DISTÂNCIA

INTRODUÇÃO
Algumas das utilidades são: atribuir um significado geométrico a
fatos de natureza numérica, como o comportamento de uma função
real e resolver problemas de Geometria Plana e Espacial.
Os problemas de Geometria Analítica são resolvidos através de
coordenadas, equações e processos algébricos.

O PONTO NO PLANO
COORDENADAS CARTESIANAS
Sejam os eixos Ox e Oy, perpendiculares em O. Eles determinam Verificamos facilmente que existe uma correspondência biunívoca
um plano (π). Consideremos um ponto qualquer P, P ∈ (π) e tracemos entre o conjunto dos pontos P do plano e o conjunto dos pares
por ele as retas (x’) paralela a Ox e (y’) paralela a Oy. Chamemos P1 ordenados (xp,yp).
e P2, respectivamente, as intersecções de (y’) com o eixo Ox e de (x’)
com o eixo Ox. Assim, o ponto A tem sua posição definida no plano cartesiano
(π) pelo par ordenado (3,4) e indicamos por A(3,4) e lemos ponto A de
coordenadas cartesianas 3 e 4.
Da mesma forma os pontos B, C e D.
B(–4,1), C(–2,–5) e D(5,–3)

O e P1 determinam o segmento orientado OP1 cuja medida


algébrica é a abscissa do ponto P.
OP1 = xP Um ponto pertencente ao eixo das abscissas tem ordenada nula.
O e P2 determinam o segmento orientado OP2 cuja medida Se pertencente ao eixo das ordenadas tem abscissa nula, e na
algébrica é a ordenada do ponto P. origem ambas as coordenadas são nulas, x = y = 0.
Um ponto pertencente à bissetriz do 1º e 3º quadrantes tem
OP2 = yP coordenadas iguais e quando pertencente à bissetriz dos quadrantes
Os números reais xp e yp constituem um par ordenado que pares tem coordenadas simétricas.
determina a posição do ponto P no plano (π). São as coordenadas b1,3 = {P(x, x) | x ∈ }
do ponto P.
b2,4 = {P(x, –x) | x ∈ }
O plano (π) é denominado plano cartesiano e os eixos Ox e Oy
que o determinam são os eixos cartesianos, sendo o eixo Ox o eixo das
abscissas e Ou o eixo das ordenadas.
xOy indica o sistema de eixos cartesianos ortogonais (ou
ortogonais, ou retangulares).
O ponto O é a origem do sistema.

QUADRANTES
Os eixos cartesianos determinam 4 regiões distintas no plano
cartesiano, os quadrantes.

507

PM_BOOK16 - MAT.indb 507 25/11/2022 19:25:18


GEOMETRIA ANALÍTICA: COORDENADAS NO PLANO E DISTÂNCIA

DISTÂNCIA DE DOIS PONTOS PONTO MÉDIO DO SEGMENTO


Sejam os pontos A(xA,yA) e B(xB,yB) referidos num sistema de eixos
cartesianos ortogonais.
Procuremos a distância d entre dois pontos.

Do triângulo ABC tiramos d  ( xB  x A )2  ( yB  y A )2

Sendo M(xM,yM) o ponto médio do segmento cujas extremidades


são A(xA,yA) e B(xB,yB), tem-se:
x A  xB y  yB
xM  e yM  A
2 2

BARICENTRO
O baricentro de um triângulo é o ponto de concurso de suas
medianas. Ele divide cada mediana na razão 2 : 1.
RAZÃO DE SECÇÃO
RAZÃO DE SECÇÃO DE UM SEGMENTO POR
UM PONTO
Sejam os pontosA ≠ B ≠ C colineares. Chamamos razão de
secção do segmento AB pelo pontoCao número
 real r, razão entre as
medidas algébricas dos segmentos AC e CB .
AC
=r (=
ABC)
CB
Tomemos A(xA,yA), B(xB,yB) e C(xC,yC).

x A  xB  x C y  yB  y C
xG  , yG  A
3 3

PONTO NO ESPAÇO
Dado um ponto P do espaço, sua posição fica determinada
plenamente em relação ao sistema através de suas distâncias PF, PV
e PH aos 3 panos coordenados ou pelas projeções destas distâncias
sobre os eixos coordenados, respectivamente, AO, OB e OC.
O feixe de paralelas A1A, C1C e B1B determina, sobre as
retas AB e OY e o feixe de retas paralelas A2A, C2C e B2B,
determina sobre as retas AB e OX segmentos proporcionais, então
AC A1C1 x C  x A y C  y A
r    .
CB C1B1 xB  x C yB  y A
Portanto as coordenadas (x,y) do ponto que divide o segmento
compreendido por P1(x1,y1) e P2(x2,y2) segundo a razão r:
PP x  rx 2 y  ry 2
1
= r são dadas pelas fórmulas: x  1 , y 1 .
PP2 1 r 1 r

508

PM_BOOK16 - MAT.indb 508 25/11/2022 19:25:25


GEOMETRIA ANALÍTICA: COORDENADAS NO PLANO E DISTÂNCIA

AO = BH = CV=PF = x (abscissa) Exercício Resolvido


OB = AH = CF = PV = y (ordenada)
01. Calcule a área do triângulo ABC, dados A(1,–1) , B(7,5) e
OC = BF = AV = PH = z (cota)
C(–2,6).
As fórmulas vistas para o ponto no plano podem ser utilizadas no
espaço, acrescentando mais uma coordenada.
Resolução:
DISTÂNCIA ENTRE DOIS PONTOS 1 1
1 7 5 1 1
d  ( xB  x A )2  ( yB  y A )2  ( zB  z A )2 Área (ABC) =   (5  42  2  7  10  6)  . 60  30 u.a.
2 2 6 2 2
1 1
BARICENTRO
x A  xB  x C y  yB  y C z z z
xG  , yG  A , zG  A B C
3 3 3
ÁREA DE POLÍGONOS
ÁREA DE UM TRIÂNGULO Dado um polígono P qualquer, é possível uma divisão de P em
triângulos.
Dado um triângulo ABC, de vértices A(x1,y1), B(x2,y2), C(x3,y3),
desejamos expressar sua área em função das coordenadas de A, B e C. Exercício Resolvido
Seja r a reta suporte do seguimento BC. A equação de r é dada por:
02. Calcule a área do pentágono ABCDE de vértices: A(3,0),
B(1,2), C(–2,2), D(–8,–7) e E(6,–1).

Resolução:
y
3 A 3 0
B 1 2
1C 1 2 2 1 98
y
A Área
Área  ABCDE= 
(ABCDE)    65  33   49u.aa.
1 2D 2 8 7 2 2
E 6 1
A 3 0
y B
2

x1 x2 x3 CONDIÇÃO DE ALINHAMENTO DE 3
r PONTOS
x y 1 Sejam os pontos P1(x1,y1) ≠ P2(x2,y2) e P3(x3,y3).
x2 y2 1 = 0 (1) Sabemos que os pontos P1 e P2 determinam a reta (r) da equação
x3 y3 1 x  x1 y  y1

| ax1  by1  c |
x 2  x1 y 2  y1
A distância de A à reta r é: d( A, r ) 
a2  b2 Para P3 pertencer à reta (r) é necessário e suficiente que suas
onde, de acordo com (1), coordenadas satisfaçam sua equação ⇒
a = y2 – y3; b = x3 – x2 e c = x2y3 – x3y2. Assim, x1 y1 1
x1 y1 1 x − x1 y 3 − y1 ou
⇒ 3 = x2 y2 1 =0
x 2 − x1 y 2 − y1
x2 y2 1 x3 y3 1
x3 y3 1 x1 x 2 x3 x1
d( A, r )   ou =0
( x 3  x 2 )2  ( y 2  y 3 )2 y1 y 2 y3 y1
1
a área do triângulo ABC é igual a ( ABC) = d( A, r ) . d(B, C) =
2
EXERCÍCIOS DE

FIXAÇÃO
1  x2 y2 x1 y1 x1 y1 
desenvolvendo obtemos     
2  x3 y3 x3 y3 x2 y2 
x1 y1
1 x2 y2
( ABC) = 01. Sejam A e B os pontos (1,1) e (5,7) no plano. O ponto médio do
2 x3 y3
segmento AB é:
x1 y1
a) (3,4) d) (1,7)
logo a área do triângulo ABC é igual à metade do valor absoluto b) (4,6) e) (2,3)
do “determinante” acima e pode ser calculado da seguinte maneira:
c) (–4,–6)
= x1y2 + x2y3 + x3y1 – x2y1 – x3y2 – x1y3

509

PM_BOOK16 - MAT.indb 509 25/11/2022 19:25:32


GEOMETRIA ANALÍTICA: COORDENADAS NO PLANO E DISTÂNCIA

02. (EEAR) Sejam A(–3,3), B(3,1), C(5,–3) e D(–1,–2) vértices de um 03. Os pontos M (–3,1) e P (1,–1) são equidistantes do ponto S (2,b).
quadrilátero convexo. A medida de uma de suas diagonais é: Desta forma, pode-se afirmar que b é um número:
a) 15 b) 13 c) 12 d) 10 a) primo. c) divisor de 10. e) maior que 7.
b) múltiplo de 3. d) irracional.
03. (EEAR) O triângulo ABC formado pelos pontos A(7,3), B(–4,3) e
C(–4,–2) é: 04. Um quadrado ABCD está contido completamente no 1º
a) escaleno c) equiângulo quadrante do sistema cartesiano. Os pontos A(5, 1) e B(8, 3) são
b) isósceles d) obtusângulo vértices consecutivos desse quadrado. A distância entre o ponto A e o
vértice C, oposto a ele, é:
04. (EEAR) Seja ABC um triângulo tal que A(1,1), B(3,–1) e C(5,3). O a) 13 c) 26 e) √26
ponto é o baricentro desse triângulo. b) 2√13 d) √13
a) (2,1) b) (3,3) c) (1,3) d) (3,1)
05. Seja AB um dos catetos de um triângulo retângulo e isósceles
05. (EEAR) Considere os pontos A(2,8) e B(8,0). A distância entre eles é de: ABC, retângulo em A, com A(1; 1) e B(5; 1). Quais as coordenadas
cartesianas do vértice C, sabendo que este vértice pertence ao
a) 14 b) 3 2 c) 3 7 d) 10
primeiro quadrante?
a) (5; 5) c) (4; 4) e) (4; 5)
06. (EEAR) Considere os segmentos de retas AB e CD, onde A(0,10),
b) (1; 5) d) (1; 4)
B(2,12), B(2,12), C(–2,3) e D(4,3). O segmento MN, determinado pelos
pontos médios dos segmentos AB e CD é dado pelos pontos M e N,
06. Seja ABC um triângulo tal que A(1,1), B(3,–1) e C(5,3). O ponto
pertencentes respectivamente a AB e a CD. _____ é o baricentro desse triângulo.
Assinale a alternativa que corresponde corretamente a esses pontos. a) (2,1) b) (3,3) c) (1,3) d) (3,1)
a) M  , 1 e N(–1,3)
1
c) M(1,–2) e N(1,3)
2  07. Considere os segmentos de retas AB e CD A(0,10), B(2,12),
b) M(–2,10) e N(–1,3) d) M(1,11) e N(1,3) C(–2,3) e D(4,3). O segmento MN, determinado pelos pontos médios
dos segmentos AB e CD é dado pelos pontos M e N, pertencentes
respectivamente a AB a CD. Assinale a alternativa que corresponde
07. (EEAR) O triângulo determinado pelos pontos A(–1,–3), B(2,1) e
corretamente a esses pontos.
C(4,3) tem área igual a:
a) M(1/2,1) e N(-1,3) c) M(1,–2) e N(1,3)
a) 1 b) 2 c) 3 d) 6
b) M(–2,10) e N(–1,3) d) M(1,11) e N(1,3)
08. (EFOMM) Calcule a área S do triângulo de vértices A(5,7); B(2,3);
C(9,2). Considerando o plano cartesiano, temos: 08. O triângulo determinado pelos pontos A(–1,–3), B(2,1) e C(4,3)
tem área igual a:
a) 7,8 c) 19 e) 60,5
a) 1 b) 2 c) 3 d) 6
b) 15,5 d) 30

09. Observando a figura, podemos afirmar que a medida da mediana


09. Em um paralelogramo, as coordenadas de três vértices
AM é:
consecutivos são, respectivamente, (1,4), (–2,6) e (0,8). A soma das A(2,6)
coordenadas do quarto vértice é: a) 2 2
a) 8 c) 10 e) 12 b) 3 2
C(6,4)
b) 9 d) 11 c) 2 3
M
d) 3 3
10. No plano cartesiano, M(3,3), N(7,3) e P(4,0) são os pontos médios B(4,2)
e) 3
respectivamente dos lados AB , BC , e AC de um triângulo ABC. A
abscissa do vértice C é:
10. O valor de a para que os pontos A(–1,3–a), B(3,a+1) e C(0,–1)
a) 6 c) 8 e) 0
sejam colineares é um número real:
b) 7 d) 9
a) primo c) positivo e par
b) menor que 1 d) compreendido entre 2 e 5
EXERCÍCIOS DE

TREINAMENTO
11. No plano cartesiano, o triângulo de vértices A(1,–2), B(m,4) e C(0,6)
é retângulo em A. O valor de m é igual a:
a) 47 c) 49 e) 51
b) 48 d) 50
01. Se os pontos A(a,2), B(b,3) e C(–3,0) estão alinhados, o valor de
3a – 2b é: 12. (FUVEST) Se (m + 2n, m – 4) e (2 – m, 2n) representam o mesmo
a) 3 b) 5 c) –3 d) – 5 ponto do plano cartesiano, então mn é igual a:
a) –2 b) 0 c) 1 d) 1/2
02. Dados três pontos colineares A(x,8), B(–3,y) e M(3,5), determine o
valor de x + y, sabendo que M é ponto médio de AB: 13. Sabendo que a mediana de Euler de um quadrilátero é o segmento
a) 3 d) –2,5 que une os pontos médios de suas diagonais, o comprimento da
mediana de Euler do quadrilátero ABCD de vértices A(1,1), B(–2,3),
b) 11 e) 5
C(–2,3), C(–3,–4) e D(3,–1) é:
c) 9

510

PM_BOOK16 - MAT.indb 510 25/11/2022 19:25:33


GEOMETRIA ANALÍTICA: COORDENADAS NO PLANO E DISTÂNCIA

34 e) 17 23. Um ponto está situado a igual distância dos pontos (3,5) e (–2,4), e a
a) c) 2
2 sua distância ao eixo dos y é o dobro da sua distância ao eixo dos x. Sabendo
2 que esse ponto não está no primeiro quadrante, suas coordenadas são:
b) 34
d) 2  14 7   14 7   7 14 
a)  ,  c) − ,  e)  ,− 
 11 11  9 9 9 9 
14. (FUVEST) Sejam A = (1,2) e B = (3,2) dois pontos do plano
d)  , − 
14 7
cartesiano. Nesse plano, o segmento AC é obtido do segmento AB b)  14 , − 7 
por uma rotação de 60°, no sentido anti-horário, em torno do ponto  11 11  9 9
A. As coordenadas do ponto C são: 24. Um hexágono regular possui dois vértices opostos de coordenadas
a) (2, 2 + 3 ) d) (2, 2 – 3 ) (a,–b) e (b,a). A área desse hexágono é:
3( 2 3( 2
5 e) (1 + 3, 2 + 3) a) a + b2 ) 3 3( 2
a + b2 ) e) a + b2 )
b) 1 + 3, 4 c)
4 2
2
3( 2
c) (2, 1 + 3) b) a + b2 ) d)
3 3( 2
a + b2 )
4 2
15. Assinale o valor da área do quadrado de vértices (–2,9), (4,6),
(1,0) e (–5,3). 25. Sejam A (0,3), C(–2,5) os vértices opostos de um quadrado.
Encontre as coordenadas dos dois vértices restantes.
a) 20 c) 45 e) 60
b) 25 d) 45

16. Sejam os pontos A = (0,0) e B = (3,4). EXERCÍCIOS DE


a) Qual é a distância entre A e B?
b) Sabemos que a área do triângulo ABC é igual a 4 e que o vértice
C pertence à reta de equação x + y = 2. Determine o ponto C.
COMBATE
17. Quando representados no sistema de coordenadas xOy, o ponto 01. O jogo da velha tradicional consiste em um tabuleiro quadrado
B é o simétrico do ponto A(–3,2) em relação à origem O; por sua vez, dividido em 9 partes, no qual dois jogadores, alternadamente, vão
o ponto C é o simétrico de B em relação ao eixo x. Com base nessas colocando peças (uma a cada jogada). Ganha o jogo aquele que
informações, é CORRETO afirmar que a medida da área do triângulo alinhar, na horizontal, na vertical ou na diagonal, três de suas peças.
ABC é igual a: Uma versão chamada JOGO DA VELHA DE DESCARTES, em
a) 8 b) 9 c) 10 d) 12 homenagem ao criador da geometria analítica, René Descartes,
consiste na construção de um subconjunto do plano cartesiano, no
18. Se os pontos A = (–1,0), B = (1,0) e C = (x,y) são vértices de um qual cada jogador, alternadamente, anota as coordenadas de um
triângulo equilátero, então a distância entre A e C é: ponto do plano. Ganha o jogo aquele que primeiro alinhar três de seus
pontos. A sequência abaixo é o registro da sequência das jogadas de
a) 1 c) 4 e) 3 uma partida entre dois jogadores iniciantes, em que um anotava suas
b) 2 d) 2 jogadas com a cor preta e o outro, com a cor cinza. Eles desistiram da
partida sem perceber que um deles havia ganhado.
19. Seja d(P, Q) a distância entre os pontos P e Q. Considere A = (-1, 0) ((1, 1), (2, 3), (2, 2), (3, 3), (4, 3), (1, 3), (2, 1), (3, 1), (3, 2), (4, 2))
e B = (1, 0) pontos do plano. O número de pontos X = (x, y) tais que
1 1 Com base nessas informações, é correto afirmar que o jogador que
=
d(X,B) = d(X,A) d(A,B) é igual a: ganhou a partida foi o que anotava sua jogada com a cor
2 2
a) 0 c) 2 e) 4 a) cinza, em sua terceira jogada. c) cinza, em sua quarta jogada.
b) 1 d) 3 b) preta, em sua terceira jogada. d) preta, em sua quarta jogada.

20. O ponto B = (3,b) é equidistante dos pontos A = (6,0) e C = (0,6). 02. Ache as coordenadas do ponto de interseção das medianas do
Logo o ponto B é: retângulo de vértices A(–1,4, 7), B(4,8,–3) e C(–6,0,5).
a) (3,1) c) (3,3) e) (3,0)
03. Determine a natureza do quadrilátero ABCD, sendo A(–2,6), B(0,2),
b) (3,6) d) (3,2)
C(4,0) e D(2,4).
21. Seja (r) a reta que passa pelos pontos P1(-1, 0) e P2(0, 3). Considere
04. Uma partícula se move de tal modo que as suas distâncias aos
M(n,q) um ponto de (r). Se a distância do ponto O(0,0) ao ponto M é
3 pontos (3,4) e (5,–2) são sempre iguais. Determine a equação que
cm, então q – n é igual a: descreve o lugar geométrico deste ponto.
10
a) 4 c) 6 05. A área de um quadrado que tem A = (4,8) e B = (–2,2) como
5 5
vértices opostos é:
b) 1 d) 7 a) 36 c) 18
5
b) 20 d) 16
22. (AFA) Considere os pontos A(4,–2), B(2,0) e todos os pontos
P(x, y), sendo x e y números reais, tais que os segmentos PA e PB são 06. (EEAR 2017) O triângulo ABC formado pelos pontos A(7,3), B(–4,3)
catetos de um mesmo triângulo retângulo. É correto afirmar que, no e C(–4,–2) é:
plano cartesiano, os pontos P(x,y) são tais que:
a) escaleno. c) equiângulo.
a) são equidistantes de C(2,–1) c) o menor valor de y é –3
b) isósceles. d) obtusângulo.
b) o maior valor de x é 3 + 2 d) x pode ser nulo.

511

PM_BOOK16 - MAT.indb 511 25/11/2022 19:25:35


GEOMETRIA ANALÍTICA: COORDENADAS NO PLANO E DISTÂNCIA

07. Quando representados no sistema de coordenadas xOy, o ponto


B é o simétrico do ponto A(–3,2) em relação à origem O; por sua vez, GABARITO
o ponto C é o simétrico de B em relação ao eixo x. Com base nessas
EXERCÍCIOS DE FIXAÇÃO
informações, é CORRETO afirmar que a medida da área do triângulo
ABC é igual a: 01. A 04. D 07. A 10. C
08.( B
C x , 2 − x)
a) 8 c) 10 02. D 05. D
03. A 06. D 09. D
b) 9 d) 12 A B= 4
=
2
EXERCÍCIOS DE TREINAMENTO
08. (ITA 2012) Sejam A = (0,0), B = (0,6) e C = (4,3) vértices de um 6 − 3x − 4x = 8 → x = 2
01. C 14.
A= A = 8→ 2
triângulo. A distância do baricentro deste triângulo ao vértice A, em
02. B 15. D
3 4 6 − 3x − 4x =−8 → x =− 7
unidades de distância, é igual a: x 2− x
03. B 16. a) d = 5
a)
5
3 d)
3
5
04. E b)C (C(2,0)
2,0 ) ou (
ou C − 2 7 ,16 7)
05. B 17. D
b) 97 10
e) 06. D 18. B
3 3
07. D 19. C
109
c) 20. C
3 08. A
09. A 21. C
09. (AFA) Considere no sistema cartesiano ortogonal o triângulo de 22. B
10. A
vértices A(0,3), B(0,–2) e C(3,0). Neste triângulo ABC estão inscritos
11. C 23. D
diversos retângulos com base no eixo das ordenadas. Em relação ao
retângulo de maior área, é incorreto afirmar que o mesmo possui: 12. E 24. C
a) altura e base proporcionais a 3 e 5. 13. A 25. B(–2,3) e C(0,5)
b) perímetro representado por um número inteiro. EXERCÍCIOS DE COMBATE
c) área maior que 4. 01. A 04. DISCURSIVA 07. D 10. C
d) área correspondente a 50% da área do triângulo ABC. 02. DISCURSIVA 05. A 08. B
03. DISCURSIVA 06. A 09. C
10. (IME 2017) Sejam os pontos A(0,0), B(–1,1), C(1,2), D(4,1) e
DESAFIO PRO
E(3,1/2). A reta r passa por A e corta o lado CD, dividindo o pentágono
ABCDE em dois polígonos de mesma área. Determine a soma das 01. C 02. C
coordenadas do ponto de interseção da reta r com a reta que liga C e D.
ANOTAÇÕES
25 53
a) d)
7 14
51 27
b) e)
14 7
c) 26
7

DESAFIO PRO
1 (ITA) Três pontos de coordenadas, respectivamente, (0,0),
(b,2b) e (5b,0), com b > 0, são vértices de um retângulo. As
coordenadas do quarto vértice são dadas por:
a) (–b, –b)
b) (2b, –b)
c) (4b, –2b)
d) (3b, –2b)
e) (2b, –2b)

2 (ITA) A área de um triângulo é de 4 unidades de superfície,


sendo dois de seus vértices os pontos A: (2,1) e B: (3,–2).
Sabendo que o terceiro vértice encontra-se sobre o eixo das
abcissas, pode-se afirmar que suas coordenadas são:
a) (–1/2,0) ou (5,0)
b) (–1/2,0) ou (4,0)
c) (–1/3,0) ou (5,0)
d) (–1/3,0) ou (4,0)
e) (–1/5,0) ou (3,0)

512

PM_BOOK16 - MAT.indb 512 25/11/2022 19:25:36


GEOMETRIA ANALÍTICA:
RETAS

EQUAÇÃO REDUZIDA DA RETA


RETA VERTICAL
A equação de uma reta paralela ao eixo Oy é x = k, onde k é
constante. Neste caso, não existem os coeficientes angular e linear.

∆x
Logo m= tgα= .
∆y
n → Coeficiente linear

O coeficiente linear (n) será


 o responsável por determinar  a
intersecção da reta com e eixo Oy . Toda intersecção com o eixo Oy se
dá num ponto da forma (0, y).
Assim y = m ⋅ 0 + n ⇒ y = n, então o ponto de intersecção será
(0, n).

Vamos ver um exemplo


Exemplo:
Encontrar equação de reta que passa pelos pontos A(2, 1) e B(–3, 4)

Sabemos que para determinar uma reta é necessário, no mínimo, Resolução:


que tenhamos 2 pontos distintos ou também podemos determinar Sendo y = mx + n teremos
uma 
reta conhecendo um ponto e o ângulo formado pela reta e o
eixo Ox . 4 −1 3
m= = −
−3 − 2 5
É conhecido, desde o estudo da função afim, que uma reta é
definida pela equação y = ax + b. Se conhecermos 2 pontos distintos 3
Assim nossa reta terá a equação y =
− x + n.
A ( x A ,y A ), B ( xB ,yB ) podemos fazer 5
Podemos usar o seguinte: ( y − y 0 ) = m ( x − x 0 ) onde x 0 e y 0 são as
y A =a ⋅ x A + b
 − ⇒ y A − yB = a ⋅ x A − a ⋅ xB ⇒ a ( xB − x A ) = ( yB − y A ) ⇒ coordenadas de um ponto que pertença a reta.
 yB =a ⋅ xB + b Ou podemos também simplesmente substituir as coordenadas do
y A =a ⋅ x A + b
 − ⇒ y A − yB = a ⋅ x A − a ⋅ xB ⇒ a ( xB − x A ) = ( yB − y A ) ⇒ 3
 yB =a ⋅ xB + b ponto A ou do ponto B na equação y = − x + n.
= a
( yB − y A=) ⇒ a ∆x 5
( xB − x A ) ∆y Substituindo as coordenadas do ponto A na nossa reta
3 6 11
Se utilizarmos as nomenclaturas utilizadas em geometria analítica 1 =− ⋅ 2 + n ⇒ n =1 + =
5 5 5
a nossa equação será y = mx + n, sendo chamada de equação reduzida.
Nessa equação teremos 2 elementos muito importantes. 3 11
Assim y =
− x+ .
m → Coeficiente angular 5 5
Utilizando ( y − y 0 ) = m ( x − x 0 ) e ponto B teríamos
O coeficiente angular (m) será o responsável pelo ângulo formado
 3 3 3 3 9
pela reta e o eixo Ox. Como vimos o coeficiente angular será m =
∆x
, ( y − 4 ) =− ( x − ( −3) ) ⇒ y =− x − ⋅ 3 + 4 ⇒ y =− x − + 4 ⇒
∆y 5 5 5 5 5
∆x
que ao olharmos para a figura abaixo veremos que é a tangente 3 11
∆y y=
− x+
do ângulo α. 5 5

513

PM_BOOK16 - MAT.indb 513 25/11/2022 19:25:37


GEOMETRIA ANALÍTICA: RETAS

ProBizu ÂNGULO ENTRE DUAS RETAS


Tenha bastante atenção com pegadinhas que são colocadas. A reta
só está na sua forma reduzida quando y está realmente isolado.
Por exemplo 2y = x + 2 não tem seu m = 1 e n = 2 pois o y não está
x+2 x
isolado na equação. Temos de fazer y= = + 1 para só assim
1 2 2
determinarmos m = e n = 2.
2
Atenção!

EQUAÇÃO DE RETA POR


DETERMINANTE
θ=β–α

Aplicando a função tangente em ambos os lados da igualdade


teremos tgθ = tg(β – α) e como mr = tgβ e ms =tgα teremos pela
fórmula de subtração de tangente:
tgβ − tgα mr − ms
tgθ= tg(β − α )= ⇒ mr = tgβ e ms = tgα ⇒ tgθ= ⇒
1 + tgβ ⋅ tgα 1 + mr ⋅ ms
tgβ − tgα tgθ= tg(β − α )= tgβ − tgα ⇒ mm= r −tgmβs e m = tgα ⇒ tgθ= mrm−r m − sm 
tgθ= tg(β − α )= ⇒ mr = tgβ e ms = tgα ⇒ tgθ= r
1 + tgβ ⋅ tgα
⇒s θ =arctg1+ m ⋅ m s⇒
1 + tgβ ⋅ tgα 1 + mr ⋅ ms  1 + rm ⋅sm
r s 

 mr − ms   mr − ms 
Introduzindo o ponto (x, y) na reta formada θ =arctg
pelos 
pontos A e  θ =arctg  
 1+ m r ⋅ ms   1 + mr ⋅ ms 
B teremos 3 pontos alinhados, de onde vimos que a condição para
estarem alinhados é
x y 1 Exemplo:
xA yA 1 = 0 Determine o ângulo formado entre as retas r : y = 3x + 4 e
xB yB 1 s : y = –2x + 8

Exemplo:
Resolução:
A equação de reta que passa pelos pontos A(2, 1) e B(–3, 4) é
Vamos determinar o coeficiente angular de cada uma das retas
x y 1 dadas.
3 11
2 1 1 =0 ⇔ x + 8 − 3y + 3 − 4x − 2y =0 ⇔ −5y − 3x + 11 =0 ⇔ y =− xPara
+ a reta r, temos:
5 5
−3 4 1 3 11
− 3y + 3 − 4x − 2y =0 ⇔ −5y − 3x + 11 =0 ⇔ y =− x + =
y 3x + 4
5 5 m =3 r

Para a reta s, temos:


INTERSECÇÃO DE DUAS RETAS
y=−2x + 8
A teoria da intersecção entre duas retas é a mesma para quaisquer
duas curvas, que é a resolução de um sistema entre as equações que m5 = −2
representam cada uma das curvas.
Aplicando a fórmula do ângulo entre duas retas, obtemos:
=y mx + n
 ms − mr −2 − 3 −5
=y px + q =tgα = =
1 + ms ⋅ mr 1 + ( −2) ⋅ 3 1 − 6
A forma de resolução do sistema é a mesma da álgebra, podendo
se dar pelo método da adição ou da substituição. −5
tgα= = 1= 1
−5

Exemplo: Portanto, α = arctg1 ou α = 45º


O ponto de intersecção entre as retas de equações y = x – 3 e
2y – 3x = 4 é
PARALELISMO E
 y =x − 3 y − x =−3 x (- 3) −3y + 3x =9
 ⇒ ⇒ + ⇒ −y = 13 ⇒ y = −PERPENDICULARISMO
13
 y =x − 3 y − x =−3 = x (-−3)
2y 3x 4−3y + 3x==9− 3x 4
2y =
2y − 3x 4
3y
 + 3x =9 ⇒ ⇒ + ⇒ −y = 13 ⇒ y = −13 A equação reduzida da reta é y = mx + n, onde m é o coeficiente
2y − 3x +4 ⇒ −y = =
= 13 ⇒3xy = 4
2y − −13 =2y − 3x 4 −13 =x − 3 ⇒ x =−10
y − 3x 4 angular e n é coeficiente linear. Onde m = tgα e α é o ângulo formado

⇒ −13 =x − 3 ⇒ x =−10 entre a reta e o eixo OX .
x =−10
P ( −10, −13) y − y A ∆y
m= tgα= B =
xB − x A ∆x

n é o ponto de intersecção da reta com o eixo OY.

514

PM_BOOK16 - MAT.indb 514 25/11/2022 19:25:38


GEOMETRIA ANALÍTICA: RETAS

RETAS PARALELAS EQUAÇÃO PARAMÉTRICA DA RETA


As equações paramétricas são duas equações que representam a
mesma reta utilizando uma incógnita t. Essa incógnita recebe o nome
de parâmetro e faz a ligação entre as duas equações que representam
a mesma reta.
Vamos a um exemplo prático.
x+3
Seja a equação x − 2y + 3 = 0 ⇒ 2y = x + 3 ⇒ y = . Vamos
2
escolher uma parametrização arbitrária para y, por exemplo y = 2t + 1.
x+3
Assim sendo y = 2t + 1 e y = teremos que
x+3 2
= 2t + 1 ⇒ x + 3 = 4t + 2 ⇒ x = 4t – 1
2
 =x 4t − 1
Como as retas r e s possuem o mesmo ângulo αcom o eixo OX Assim temos a equação paramétrica 
então mr = ms .  y 2t + 1
=

Para voltarmos para equação completa ou reduzida basta


RETAS PERPENDICULARES isolarmos t em ambas as equações e igualá-los.
mr − ms x +1
Na equação , do ângulo entre 2 retas para x = 4t − 1 ⇒ t =
1 + mr ⋅ ms 4
mr − ms y −1
termos θ = 90º temos que não pode existir, daí y = 2t + 1 ⇒ t =
1 + mr ⋅ ms 2
1 + mr ⋅ ms =
0 ⇒ mr ⋅ ms = −1.
x +1 y −1
= ⇒ 2x + 2 = 4y − 4 ⇒ 2x − 4y + 6 = 0 ⇒ x − 2y + 3 = 0
4 2
Exemplo:
x
As equações de retas r : y = –2x + 3, s : y = e t : y = –2x + 3. DISTÂNCIA DE PONTO A RETA
2
As restas r e t possuem o mesmo coeficiente angular, mr = mt . Daí
r e t são paralelas.
As retas r e s ou s e t possuem os coeficientes angulares
 1
inversos simétricos entre si, mr ⋅ ms =−2 ⋅   =−1. Daí r // t que são
perpendiculares a s.  2

OUTRAS EQUAÇÕES DE RETA


EQUAÇÃO COMPLETA DA RETA
Da equação reduzida y = mx + n quando isolamos todos os
termos de um mesmo lado da igualdade passamos a ter os termos
ax + by + c = 0 com a, b e c ∈ , que é a equação completa da reta.
Sendo a equação da reta r : ax + by + c = 0 e P ( x 0 ,y 0 ) temos

Exemplo: a ⋅ x0 + b ⋅ y0 + c
dP,r =
x x a2 + b2
y =− + 4 ⇒ y + − 4 =0 ⇒ x + 3y − 12 =0, daí a = 1, b = 3 e
3 3
c = –12. Exemplo:
A distância do ponto P(–1, 1) a reta de equação y = –2x + 3 é
EQUAÇÃO SEGMENTÁRIA DA RETA y =−2x + 3 ⇒ 2x + y − 3 =0 ⇒ a =2, b =1 e c =−3 e
Da equação completa da reta ax + by + c = 0 isolaremos o termo x0 =
−1 e y 0 =
1
c de um lado da igualdade, ax + by = -c e dividiremos todos os termos
a b a.x 0 + b ⋅ y 0 + c 2 ⋅ ( −1) + 1⋅ 1 + ( −3) −4
por –c, assim − x − y = 1. =
dP,r = = = 2 2
c c a2 + b2 ( −1)2 + (1)2 2
Assim teremos por exemplo
2 1 x y
2x – y – 6 = 0 e 2x − y = 6 ⇒ x − y =1 ⇒ − =1 Exemplo:
6 6 3 6
Dado o ponto A(3, –6) e r : 4x + 6y + 2 = 0. Estabeleça a distância
Com a equação segmentária, podemos determinar os pontos de entre A e r
interseção da reta com os eixos ordenados do plano. O termo que
4 ⋅ 3 + 6 ⋅ ( −6 ) + 2 12 − 36 + 2 22 22 ⋅ 52 22 52 11 52
divide x na equação segmentária é abscissa do ponto de intercessão = d = = = = =
da reta com o eixo x, e o termo que divide y é abscissa do ponto de 42 + 62 16 + 36 52 52 ⋅ 52 52 26
interseção da reta com
4 ⋅ 3 +o 6eixo −6y.) Assim:
⋅ ( + 2 12 − 36 + 2 22 22 ⋅ 52 22 52 11 52
= d com o eixo Ox é=
Intersecção = =
(3, 0) 16 + 36 = =
42 +62 52 52 ⋅ 52 52 26
Intersecção com o eixo Oy é (0, –6)

515

PM_BOOK16 - MAT.indb 515 25/11/2022 19:25:39


GEOMETRIA ANALÍTICA: RETAS

DISTÂNCIA ENTRE DUAS RETAS REFLEXÃO EM RELAÇÃO EIXO DAS


PARALELAS ABSCISSAS (Oy)
Se r // s de equações r : ax + by=
+ c1 0 e s : ax + by =
+ c2 0 Quando fazemos a reflexão ou simetria em relação ao eixo das
ordenadas verificamos que a ordenada do ponto em questão se mantém
c1 − c2 enquanto a abscissa do ponto em questão é simétrica. Dessa forma
dr,s = 
a2 + b2 sendo P’ o ponto simétrico de P em relação ao eixo Ox temos que

Nota: duas retas paralelas, quando na forma completa, possuem P ( x, y ) ⇒ P' ( −x, y )
mesmos coeficientes a e b.
Exemplo:
O simétrico do ponto A(1, 3) em relação eixo das abscissas é o
Exemplo: ponto A’(–1, 3).
Calcula a distância entre as retas r : 2x + 3y = 4 e s : 2x + 3y = 1.
4 −1 3 3
dr, s = =
22 + 32 4+9 13

PONTO SIMÉTRICO
A simetria ou reflexão em relação a um objeto, que na maior parte
das vezes é uma reta, deve ser tratada como a reflexão em relação a
um espelho.
Vamos primeiramente verificar a reflexão em relação aos eixos
coordenados.

REFLEXÃO EM RELAÇÃO EIXO DAS


ABSCISSAS (Ox)
Quando fazemos a reflexão ou simetria em relação ao eixo das
abscissas verificamos que a abscissa do ponto em questão se mantém
enquanto a ordenada do ponto em questão é simétrica. Dessa forma

sendo P’ o ponto simétrico de P em relação ao eixo Ox temos que
P ( x, y ) ⇒ P' ( x, −y )

Exemplo: O simétrico do ponto A(–2, –4) em relação eixo das


abscissas é o ponto A’(–2, 4).

REFLEXÃO EM RELAÇÃO A UMA RETA DADA


Seja um ponto P(x, y) e uma reta r : ax + by + c = 0.

Definiremos o ponto P’(m, n) como o ponto simétrico do ponto P


em relação a reta r.

516

PM_BOOK16 - MAT.indb 516 25/11/2022 19:25:40


GEOMETRIA ANALÍTICA: RETAS

Vamos ver o exemplo a seguir.

Exemplo:
Encontre o ponto simétrico do ponto P(–2, 3) em relação a reta
r : y = x + 1.

O ponto P’ define com o ponto P uma reta perpendicular a reta r


e ainda P e P’ estão a mesma distância da reta r.

Vamos chamar o ponto simétrico do ponto P de P’(m, n) e vamos



determinar a equação da reta PP', que iremos chamar de reta s, a
partir do coeficiente angular da reta r.

Coeficiente angular da reta r : y = x + 1


mr = 1

Coeficiente angular da reta s, perpendicular a r.


mr ⋅ ms =−1 ⇒ ms =−1

Equação da reta s
y =ms ⋅ x + n ⇒ y =−x + n

Como a reta s passa pelo ponto P(–2, 3) temos que P ∈ s e assim


3 =− ( −2) + n ⇒ n =3 − 2 =1

Sabendo disso, para determinarmos um ponto simétrico de um E assim nossa reta s é y = –x + 1


ponto dado em relação a uma reta, devemos seguir os seguintes passos. Agora iremos encontrar a interseção entre as retas r e s resolvendo
1. Encontrar a equação da reta perpendicular a reta dada.  y= x + 1
o sistema de equações  .
Faremos isso a partir da relação existente entre os y =−x + 1
coeficientes angulares de duas retas perpendiculares entre si
(m1 ⋅ m2 = −1) .  y= x + 1
+ ⇒ 2y = 2 ⇒ y = 1

2. Iremos encontrar o ponto de interseção entre a reta y =−x + 1
perpendicular que encontramos com reta dada. y = 1⇒ 1 = x + 1⇒ x = 0
Faremos isso colocando as duas equações de reta em sistema
linear de duas equações e duas incógnitas. Logo nosso ponto de interseção será o ponto M(0, 1).
O ponto M(0, 1) será o ponto médio dos pontos P(–2, 3) e
3. A interseção encontrada é o ponto médio do segmento P’(m, n) e assim
determinado pelo ponto dado e seu ponto simétrico.
m + ( −2) m−2
Por fim iremos utilizar o ponto de interseção como o ponto xM = = 0⇒ = 0⇒m−2= 0⇒m= 2
médio para determinar o ponto simétrico (P’). 2 2
n+3
yM = = 1 ⇒ n + 3 =2 ⇒ n =−1
2

Assim o ponto P’, simétrico do ponto P(–2, 3) em relação a reta


r : y = x + 1, é o ponto P’(2, –1).

517

PM_BOOK16 - MAT.indb 517 25/11/2022 19:25:41


GEOMETRIA ANALÍTICA: RETAS

 4 13 
Logo nosso ponto de interseção será o ponto M  , .
5 5 
 4 13 
O ponto M  ,  será o ponto médio dos pontos P(–2, 4) e
5 5 
P’(m, n) e assim
m + ( −2) 4 m−2 4 18
xM =
= ⇒ = ⇒ 5m − 10 = 8 ⇒ 5m = 18 ⇒ m =
2 5 2 5 5
m + ( −2) 4 m−2 4 18
xM = = ⇒ = ⇒ 5m − 10 = 8 ⇒ 5m = 18 ⇒ m =
2 5 2 5 5
n + 4 13 6
yM = = ⇒ 5n + 20 = 26 ⇒ 5n = 6 ⇒ n =
2 5 5
Assim o ponto P’, simétrico do ponto P(–2, 4) em relação a reta
 18 6 
r : y = 2x + 1, é o ponto P'  , .
 5 5
 18 6 
Assim P'  ,  que em decimal é P’(3,6; 1,2) e tem como soma
 5 5
das coordenadas 3,6 + 1,2 = 4,8.

Exercício Resolvido
Vamos a resolução da nossa QUESTÃO DESAFIO
01. Considere as retas r e s definidas por r : kx –(k + 2)y = 2 e
01. Refletindo-se o ponto P(–2, 4) em relação a reta r : y = 2x + 1
s : ky – x = 3k. Determine k de modo que:
obtém-se um ponto cuja soma das coordenadas é
a) r e s sejam concorrentes
a) 3,2
b) r e s sejam paralelas
b) 2,4
c) r e s sejam coincidentes.
c) 3,6
d) 4,8
Resolução:
e) 6,0
Pela definição, retas concorrentes possuem um ponto de interseção,
as paralelas nenhum ponto de interseção e as coincidentes, todos
Resolução: D seus pontos são de interseção. Nos termos de Geometria Analítica,
Vamos chamar o ponto simétrico
 do ponto P de P’(m, n) e vamos analisamos os coeficientes angulares das retas:
determinar a equação da reta PP', que iremos chamar de reta s, a k 2 k
partir do coeficiente angular da reta r. r : kx − (k + 2)y = 2 ⇒ y = x+ ⇒ mr =
k+2 k+2 k+2
Coeficiente angular da reta r : y = 2x + 1 1 1
s : ky − x = 3k ⇒ y = x + 3 ⇒ ms =
k k
mr = 2
a) retas concorrentes possuem os coeficientes angulares diferentes:
Coeficiente angular da reta s, perpendicular a r.
1 k ≠ 2
mr ⋅ ms =1 ⇒ 2 ⋅ ms =−1 ⇒ ms = − k 1 
2 mr ≠ ms ⇒ ≠ ⇒ k 2 − k − 2 ≠ 0 ⇒ (k − 2) ⋅ (k + 1) ≠ 0 ⇒ e
k+2 k k ≠ −1

Equação da reta s
x b) retas paralelas possuem os coeficientes angulares iguais:
=
y ms ⋅ x + n ⇒=
y − +n
2 k = 2
k 1 
Como a reta s passa pelo ponto P(–2, 4) temos que P ∈ s e assim mr = ms ⇒ = ⇒ k 2 − k − 2 = 0 ⇒ (k − 2) ⋅ (k + 1) = 0 ⇒ ou
k+2 k k = −1

4 =−
( −2) + n ⇒ n =4 − 1 =3
2 c) retas coincidentes possuem os coeficientes angulares e os
x lineares iguais:
E assim nossa reta s é y =− + 3
2  k = 2
 k 1 
mr = ms ⇒ k + 2 = k ⇒ k − k − 2 = 0 ⇒ (k − 2) ⋅ (k + 1) = 0 ⇒ ou
2
Agora iremos encontrar a interseção entre as retas r e s resolvendo
 
= y 2x + 1 k = −1
 
o sistema de equações  x . n =n ⇒ 2 4
y =− 2 + 3 =3 ⇒ 3k + 6 =2 ⇒ 3k =−4 ⇒ k =−

r s
k+2 3
= y 2x + 1 Não é possível conciliar os valores de “k”. Logo, não existe um
 x x 5x 4
 x ⇒ 2x + 1 =
− + 3 ⇒ 2x + =3 − 1⇒ =
2 ⇒ 5x =⇒
4 x =valor de “k” que satisfaça a condição pedida.
y =− 2 + 3 2 2 2 5

x 5x 4
=
3 − 1⇒ =
2 ⇒ 5x =⇒
4 x=
2 2 5
4 4 8 13
x = ⇒ y = 2 ⋅ + 1= + 1=
5 5 5 5

518

PM_BOOK16 - MAT.indb 518 25/11/2022 19:25:41


GEOMETRIA ANALÍTICA: RETAS

Exercício Resolvido Exercício Resolvido

02. A reta r determina um ângulo de 120º com a reta s, cujo 04. As retas r, s e t são definidas respectivamente por
1 4x – 7y + 18 = 0, 2x – y – 6 = 0 e 4x + 3y – 2 = 0. A área da
coeficiente angular é − . O coeficiente angular de r vale: região limitada por essas retas é
3
a) 3
y
P
6+5 3 6
b)
3
6+5 3
c) - 4
3 t

6-5 3 Q
d) 2
3 r

Resolução: C 0
-2 0 2 4 6 x
O ângulo agudo formado entre duas retas é dado pela fórmula:
ms − mr
tga = . Como o ângulo indicado é 120º (obtuso), o -2 R
1 + ms ⋅ mr
agudo interno será de 60º. s

1 −1 − 13mr −1 −a)3m30
− − mr − − mr r
ms − mr 3ms − mr 33 −1 − 3mrb) 3⋅ 25 3 −1 − 3mr 3
=
g60º ⇒= 3g60º
= ⇒⇒= = 33 ⇒= 3=
⇒ 3 ⇒= ⇒3 ⋅ ⇒
1 + ms ⋅ mr 1 + 1ms ⋅ mr 3 − m1 r  3 3 − m3r − mr 3 3 − mr
1 −1 − 3mr 11+  −  ⋅ mr − 1 − +  r−  ⋅ mr
13m c) 20
− − mr − − m3r  3 3  3
mr −1 − 33mr ms − mr 3 −1 − 3mr 3 −1 − 3mr d)3 15
⇒= 3 =⇒=
g60º 3 −1 − 3m ⇒= 3⇒= 3 −31 − 3m ⇒⋅ = 3 ⇒ 3 ⇒= 3 ⋅
−1 − 3− m 3 ⇒ −1 − 3 3 3−+1 − 33 3 −1 − 3 3 3 + 3
mr ⇒=  1  ⇒= 1 +−m1 −s ⋅3m
3 r−
m r mrr 3  = 1 −31r−⇒ 3m3r − m 3 −−m 1 −r 3mr
mrr3+= 3 3 3=
3 1+ 3 −  ⋅ mr⇒3 3 = −
⋅ ⇒ 3⇒ 1= + 33 −−m  ⋅ mr − ⇒ 3mrr 3+ = 3 3= −1 − ⇒ 3m−r ⇒3m
3− 3
e)−110 r=
− 3mr ⇒ mr ⋅= ⇒ =
3 − 3 3 +3 −3 3
⋅ ⇒
3 − mr3  3 m
3 3r3 − mr  3  3r − mr 33 − mr 3− 3 3+ 3
mr
3
3mr −1 − 3mr −1−−33m − 3 − 9 3 −−91 − 3m −12 − 3−−103 3− 9 −
r −1 − 3 3 =
361−−+953= 3−12
3 3 +− 103 3 − 6−1 + −5 33 3 Resolução: −1 − 3 3 C3 + 3
⇒ 3= ⇒ −⇒33m
m
= r r=+ 3 3 r= 9
⇒−−1 −333m = ⇒ m=
r ⇒m r r= = ⇒ − 3mr=
6
+− 3 3 =
3
−1⋅ − 3mr ⇒⇒m= r = = ⋅ ⇒
− 3 −− m1 − 3 3 −1 − 33 − 3
m 3 + 3 9 − 3 6 3
− 3 − são 3+ 3
m 3
⇒ r− 3mr + 3 3 = m
−1r − 3mr ⇒ mr = r = ⋅r 3 − ⇒ 3 3− 3 3+ 3 3 3 As retas 3 concorrentes duas a duas. Logo suas interseções
3 − 3 3
−63+−5 33− 9 3 − 9 −12 − 10 3 − 3 3 + 3 6+5 3
formarão um triângulo. Sejam os pontos P, Q e R as interseções
3 − 9 3 − 9 −12 − 10 3
= ⇒ mr ==− = = − respectivamente r ∩ s, r ∩ t e s ∩ t.
0 93− 3 6 + 5 3 6 3 9 − 3 6 3  60
=− 4x= − 7y + 18 0 4x= − 7y + 18 0 = x = 6
3 i)  ⇒ ⇒ 10x = 60 ⇒  10 ⇒ P = ( 6,6
2x − y − 6= 0 → ×( −7) −14x + 7y + 42= 0 = y 2 60 ( 6 ) −6=
6 6
 4x = − 7y + 18 0  4x = − 7y + 18 0  = x =
4x= − 7y + 18 0 4x= − 7y + 18 0 i)  = x − y=
60 ⇒ ⇒ 10x = 60 ⇒  20 10 ⇒ P = ( 6,6
Exercício Resolvido ⇒ ⇒ 10x = 60 ⇒2x − 6= 6 0 → ×⇒ ( −7) P = ( 6,6−14x) + 7y + 42= 0 
=
i) 
2x − y − 6 = 0 → × ( − 7) − 14x + 7y + 42 = 0 
4x= −
10
7y + 18 0  =
4x − 7y + 18 0 y= y = 2 ( 6 )2− =6 6
  10
03. As retas r e (s) de equações 3x – y + 7 = 0 e 4x – y – 5 = 0 ii) = y 2 (6) − = 6 6 ⇒ ⇒ 10y =20 ⇒  60 ⇒ Q =− (
 4x = − 7y + 18 0  4x= − 7y + 18 0  4x 4x=
= x−
 −20
+7y 3y60+−18
= 2=6 00→ ×( −1) 4x  −
= 4x
− 7y− 3y
+ 18+ 2= 0 0 =
= yx x=
20
7=(=2) −26 18
= − 1
respectivamente i)passam pelo ponto P(a, b).
 ⇒ O valor de (a + b) é ⇒ 10x = 60 ⇒ i)  =
y  =10 2
4x 7y + 18 0 ⇒ P = (6,6
⇒  )=
4x − 7y + 18 0 ⇒ 10x = 60 ⇒ 
 10 4 10 ⇒ P = ( 6,6
2x −−7y
=
4x y −+618 = 0→ 0 ×( −7) −4x
=14x + 7y
− 7y + 42=0 0
+ 18 ii) =
2x −10 y − 6= 0 → ×( −7) ⇒ −14x + 7y + 42= 0⇒ 10y =20 ⇒ = 6 6 ⇒ Q =−
y 72( 2( 6) )−−18 (
a) 51 ii)  ⇒ ⇒ 10y =20 ⇒ 4x = y+ 3y 2 (−6 )2− = =60 → 6 ×⇒ ( −Q1) =( −1,2−4x) − 3y + 2= 0  x20
=
 60 = = −1
4x+4x3y = − −7y2=+ 18 0 → 0×( −1) −4x 4x=−−3y7y++218 = 00  2x
x = 7
− = x( 2) −=
y − 6 = 18 60 → × ( 3)  6x − 3y − 18 = 0
⇒ 10x = 20 ⇒  =
= x
  60 = 2
= 62 ⇒ R = ( 2, −2)
b) 52 ⇒ P= 4x= x 10 20 4
i)  ⇒ ⇒ 10x = ii)60 4x ⇒ y= −+7y20+ 10 18 22 000
= −1 ⇒
4x(4x− 7y
6,6 ) +− 18 00 0 ⇒ 10x = 60 ⇒ = y =
c) 53 4x2x
= − y+−18
− 7y 6= 00→ ×( −7) 4x =−14x
− 7y++7y 18+ 42 0= 0 i)  ==
4x
=
4x =4−
3y
− 10
7y + 18 ⇒ = +
= 3y
− 7y 2+ 18 y =2(2)
 20 10
10− 6 =−2 ⇒ P = ( 6,6 )
y− 62= ( 600 )→ 618 6⇒ R = ( 2, −2()
− y= −→=
6××( −(3)
6⇒ Q⇒ −(14x +) 7y + 42 0= 0 ⇒ 10y =20 ⇒ ⇒ Q =−
⇒2x 20−+ 73y−2−6=2 −1,2 − 18 =
ii)  ⇒ ⇒ 10y =20ii) 2x 7) =−
6x 3y = x = = 2−=
= → × − − − + = y 10 27((62))
d) 53 2x4x
 − − = +y 3y 6− 2 =
0 0 →
→ × 3) ( × ( −1) − 4x
6x − 3y − 18 =− 3y + 2
0 = 0 ii)
=x =  
4x =+63y220 ( −) −
62 10 18 0 ( ⇒1) 
=

+
4x

3y 2 ⇒0 10x = 20 ⇒   x = −
= − 1
11= ( 2, −2)
ii)  ⇒ ⇒ 10x = 20 ⇒  10 4xx1= ⇒= R− 4x 3y 2 0 40
1 y =2(2) 20 − 64=−2
e) 55 =4x +4x=3y−−7y 2 + 18 0 0 =4x + 3y =
4x− −2 7y0+ 18 0 A =  = y1 =
− 24 12 = 6(4) − 6( −3) + 1( −2) = 24 + 18 − 2) =  = = 20 = 2
ii)  ⇒ ⇒ 10y = y=
20  ⇒ =2(2)
4x 220− 7y− 610
+ =18 −2 0 2 ⇒ Q  =
4x
=− ( − 7y
1,2 )+ 18 20 y 20
2 10
  6 6 1  Q =( −1,
2x6 − y+−63y
4x 6= −102=→0×→ (3) ×( −1)6x −3y −4x − 3y=
− 18 +0 2= 0 ii)=x4x 2x = −
2
1+x 3y y −
7
2− (
2
6 2= )
= −22= 10 = →
1
−0 →
18 ×
1 (−1(3) ⇒6x  − 3y − 18 = 0
+ 2= = 10⇒
⇒ 10y =20 ⇒=
40 7 ( 2) 2− 18 ⇒⇒
x =
R = ( 2, −2)
Resolução: E ii)  ⇒ ⇒ 10x = 20 ⇒ iii) A = = −+13y −
10 ⇒ =×R6(4)= 1)(⇒ −
2,−−6(2 − 4x+−1(3y
) 3) −2) + 18= −20
2410x 2) ⇒
= x = = 10 20 = −1
1
=4x + 3y − 2 0 1 = 4x + 3y − 2 01 40  4x  2 4 0 =4x + 3y − 2 0
y =2(2) −46 =−2
=  −ponto
A pelo 1 2 P, então 1= 6(4) − 6(  −3) + 1( −2) = 24 + 18 − 2) = y = = 220
2(2) − 6 =−2 2 2 2
Se ambas passam 2 2 ele é um ponto de interseção. 2 2  220 −2 1
Basta resolver o sistema 26 −−y26− 61=1 0 → ×(3)
2x 6x − 3y − 18 = 0 =
 x 6 = 6 2 1  20
ii)  ⇒ ⇒ 10x = 20 ii) ⇒2x1 − 10 y − 6= 0 → ×(⇒ 3) R = (6x 2, −−23y ) − 18 = 0 =x = 2
⇒ R = ( 2, −2)
= 3x − y + 7 0 A = 1= 4x
= +  3y
3x − 2
y + 0
7 1 0  =
4x= + 3y − 2 10x 12
 A 40 =  −1 2 1 = 1 6(4) ⇒−  6( −3) + 1( −2) = 1 ⇒24 10x
+ = 20
18 − ⇒ =  40 10
2) = 20
−1 2 1 = 6(4) − 6( −3) + 1( −2) = 24 + 18 − 2) = = =y =
2 + 2(2)
4x 203y −−26 = 0 −2 = 4x + 3y − 2 0 y 2=2(2) − 6 =−2
 2 ⇒ 2 ⇒ −x + 12= 0 ⇒ 2 2 2 2 
4x − y − 5 = 0 → ×( −1) 2 6−−4x 2 6+1y 1+ 5 = 0 y = 3(12) + 7 = 36 + 7 =243 −2 1
= = x −12
3x y +7 0 = 1 3x − y + 7 0
1 = x 1 12 40 6 6 1
−1a 2= 121 ⇒
7 0
⇒ −x + 12= 0 ⇒ Logo, 4x − P(a,b) = P( −12,A−=29) ⇒⇒ = a +6(4)⇒
b = −12 −12
x6(++ 3)
43=+=1(055
−⇒2) = 24 + 18 − 2)A= = 1 −= 120 2 1 = 1 6(4) − 6( −3) + 1( −2) = 1 24 + 18 − 2) = 40 = 20
+5 = 0 y − 5+=70=→
y = 3(12) 36×+( −71)=243−4x + y + 5 =2 0 y 2= 3(12) + 7 = 36 +227 = 43
2 b−=2 431 2 −2 1
2 2 2
2  a = 12
⇒ a + b = 12 + 43 = 55 Logo, P(a,b) = P( −12, −29) ⇒  ⇒ a + b = 12 + 43 = 55
3 b = 43

519

PM_BOOK16 - MAT.indb 519 25/11/2022 19:25:42


GEOMETRIA ANALÍTICA: RETAS

Exercício Resolvido 07. Sejam as retas r e s de equações y = 2x – 3 e y = –3x + 2. A


tangente do ângulo agudo formado pelas retas r e s é:
05. A equação da reta que é perpendicular à reta 4x + y – 1 = 0 a) 0 b) 1 c) 3 d) 3
e que passa pelo ponto de interseção das retas 2x – 5y + 3 = 0 e 3
x – 3y – 7 = 0 é:
a) x – 4y – 24 = 0 08. As retas y = kx + 2 e y = –x + m interceptam-se no ponto (1,4).
Assim, o valor de k + m é:
b) 4x – y – 24 = 0
a) 8 b) 7 c) 6 d) 5
c) x – y – 24 = 0
d) x + y + 24 = 0 09. O valor de “a” para que as retas r : ax + y – 4 = 0 e s : 3x + 3y – 7 = 0
e) x – 4y + 24 = 0 sejam paralelas é:
a) 1 c) 2 e) –1
Resolução: A b) 1/2 d) 3
= y = −17
2x − 5y + 3 0 =2x − 5y + 3 0 
i) Interseção :  ⇒ ⇒ 5( −17) − 3 ⇒ P =( −44, −17)
 x − 3y − 7 = 0 → × ( − 2)  − 2x + 6y + 14 = 0 x = = −44 r : 5x − 12y = 42
2 
y = −17 1 =0 ⇒ y =−4x + 1 ⇒ mr =−4 10. Dadas as retas  s : 5x + 16y =56 o valor de m para que as três
=2x − 5y + 3 0 ii) r: 4x + y −= − + = − + y = −17 
⇒ ⇒ 
i) Interseção 5( −17) 2x 5y
:  − 3 = −44
3 ⇒ 0P
x
= ( ⇒)
− 44, − 
172x 5y 3 0 
⇒ y = − −17) − 3
5(17
t
⇒ P =( −44,−17): 5x + 20y =m
−2) −2x + 6y + 14= 0 x = − − = = → +
× − − + + = = = −
 = 
 x
2x 1−3y5y +s
7 3: y 0 0 ( n 2) = 
2x2x− 5y6y
+ (
3 −14
44)0 0  x retas 44
sejam concorrentes num mesmo ponto é:
⇒ m:2s= ⇒ 
ys =⊥ −r 17
i)iii)Interseção 4
s
⇒ ⇒ −17 = + n⇒s ⇒
 −175(=−17)
−211−+ 3ns ⇒ ns ⇒ = −P6=( −44, −17)
⇒ mr =
= −4− 5y + 3 0
2x  − 1−=x −4 3y − 7 = = 0 →
(P−44, × (
+ (1−−⇒ − 2) S−
m−r∈17 −2x + 6y + 4
14 = 0  x = a) = −
14 44 c) 36 e) 58
⇒ ⇒ii)r : 4x5(+−y17) 30 ⇒ yP= ⇒−4x = 17)
44, = )4 2
2) −2x + 6y + 14= 0 x = = −44 b) 28 d) 48
( −44) ii) Lorgo, − 1x =
: 4xs +: yy =
2 −+0 ⇒ ys = −− 4x x+ 1 ⇒ m=r 0=−4
s
⇒ −17 = + niii)
s ⇒ s ⊥−17
r ⇒ =m −114=
16nsou
⇒⇒sn:s:xy= = −4y6 − + 24
ns
⇒ −17 =
( −44)
+ ns ⇒ −17 = −11 + ns ⇒ ns = −6
mr =−4 4 s  4
x
17) ∈ S 4
1 Ps := y=( −44,+−n17) s ∈S
4
( −44)
iii) s ⊥ r ⇒ ms = ⇒  4 ⇒ −17 = + ns ⇒ −17 = −11 + ns ⇒ ns = −6 EXERCÍCIOS DE
( −44)

TREINAMENTO
4 P == 4
4 =0⇒ −17 = + ns Lo
⇒go, −17s :=y−= 11x+−n6s ⇒ n −
( −
6 44,
ou ss: x − 4y − 24 =0
− 17) ∈ S
4
)∈S 4
x
Logo, s : y = − 6 ou s : x − 4y − 24 =0
4
=0
01. (EEAR) Para que os pontos A(x,3), B(–2x,0) e C(1,1) sejam
colineares, é necessário que x seja:
EXERCÍCIOS DE a) –2 b) –1 c) 2 d) 3

FIXAÇÃO 02. (EEAR) Considere os pontos A(2,3) e B(4,1) e a reta r : 3x + 4y = 0.


Se dA,r e dB,r são, respectivamente, as distâncias de A e de B até a reta
r, é correto afirmar que:
01. Para que as retas de equações 2x – ky = 3 e 3x + 4y = 1 sejam a) dA,r > dB,r c) dA,r = dB,r
perpendiculares, deve-se ter: b) dA,r < dB,r d) dA,r = 2dB,r
a) k= 3/2 c) k= –1/3 e) k= 2
b) k= 2/3 d) k= –3/2 03. (EEAR) Dada a reta r : 2x – 3y + 5 = 0 e o ponto P(5,6), a distância
de P à reta r é:
02. Dada a reta r : 2x – 3y + 5 = 0 e o ponto P(5,6), a distância de P a) 91 b) 30 13 c) 3 91 d) 3 13
à reta r é: 91 13
a) 91 b) 30 13 c) 3 91 d) 3 13
91 91 04. (EEAR) A equação reduzida da reta que passa pelos pontos A(0,1)
2 e B(6,8) é dada por:
03. A reta s que passa por P(1,6) e é perpendicular a r :=
y x + 3 é: 7 6
3 a) y = 7x + 1 b) y = 6x + 1 c) = y x + 1 d) = y x +1
3 2 20 6 7
a) y = x c) y = − x+
2 5 3
3 15 05. (EEAR) A reta s que passa por P(1,6) e é perpendicular a
b) y = x + 5 d) y = − x+ 2
2 2 r :=
y x + 3 é:
3
04. O coeficiente angular da reta que passa pelos pontos A(–1,3) e 3 2 20
a) y = x c) y = − x+
B(2,–4) é: 2 3 3
a) – 1/2 b) – 7/3 c) 3/2 d) 4/3 3 15
b) y = x + 5 d) y=
− x+
2 2
05. Uma reta paralela à reta r: y = 2x + 3 é a reta de equação:
06. Dadas as retas perpendiculares de equações y = ax + b e y = mx + n,
a) 3y = 2x + 1 c) 2y = 4x – 1 a afirmativa correta é:
b) 2y = 2x – 4 d) y = x + 3 a) a = m c) a = 1/m e) a ⋅ m = 1
b) a = – m d) a ⋅ m = – 1
06. Se as retas r e s são perpendiculares, e a equação de s é 2y + x – 2 = 0,
o coeficiente angular mr da reta r é:
07. A reta 3x – 2y – 5 = 0 é perpendicular à reta:
a) – 1 c) 2
a) 2x – 3y = 5 c) 3x + 2y = 0
b) 1 d) 3
b) 4x + 6y = 1 d) 6 – 4y = 10

520

PM_BOOK16 - MAT.indb 520 25/11/2022 19:25:43


GEOMETRIA ANALÍTICA: RETAS

08. Cada equação a seguir representa uma reta do plano cartesiano. 16. (FGV) Os pontos A(0,1), B(1,1), C(1,0) e D(–k,–k), com k > 0,
Associe cada uma dessas equações ao coeficiente angular da reta que formam o quadrilátero convexo ABCD, com eixo de simetria sobre a
a mesma representa. bissetriz dos quadrantes ímpares.
(1) –x + 2y – 2 = 0 ( ) 1/2
(2) 4x + 8y – 9 = 0 ( ) –2
(3) –10x + 5y – 3 = 0 ( )2
(4) 6x + 3y – 5 = 0 ( ) –1/2
Assinale a alternativa que apresenta a sequência correta.
a) 1 – 3 – 4 – 2 c) 4–2–1–3 e) 2 – 4 – 1 – 3
b) 3 – 4 – 2 – 1 d) 1 – 4 – 3 – 2

3
09. A equação geral da reta de coeficiente angular e de coeficiente
2
linear – 2 é:
a) x + 2 y – 4 = 0 c) 3x – 2 y – 4 = 0
O valor de k para que o quadrilátero ABCD seja dividido em dois
b) 3x – 2 y – 2 = 0 d) 3 2 x – 2 y – 2 = 0
polígonos de mesma área pelo eixo y é igual a:

10. (AFA) O baricentro do triângulo ABO abaixo representado é o ponto: a) 2+ 5 1+ 2 e) 1+ 5


c)
4 2 2
a) (3,2) 3+ 2 1+ 3
b) d)
b) (1,3) 4 2
c) (3,1) 17. (FGV) O ponto da reta x – 3y = 5 que é mais próximo ao ponto (1,3)
d) (2,3) tem coordenadas cuja soma é:
a) 1,6 c) 1,0 e) 0,8
b) 1,2 d) 1,4
11. Os valores de k para que a reta que passa pelos pontos (5,k) e
(1,0) seja paralela à reta definitiva pelos pontos (–2,1) e (k,3):
18. (FGV) Os pontos A(3,–2) e C(–1,4) do plano cartesiano são vértices
a) não são todos racionais c) são todos inteiros de um quadrado ABCD cujas diagonais são AC e BD. A reta suporte da
b) são todos positivos d) são todos negativos diagonal BD intercepta o eixo das ordenadas no ponto de ordenada:
a) 2/3 c) 1/2 e) 0
12. As retas de equações y = 2x – b e y = cx + d se interceptam b) 3/5 d) 1/3
perpendicularmente no ponto (8,2). Qual o valor do produto b.c.d?
a) – 42 c) – 36 e) – 30 19. (FGV) As intersecções de y = x, y = – x e y = 6 são vértices de um
b) – 63 d) – 27 triângulo de área:
a) 36 c) 24 e) 12
13. Dois pontos sobre a reta y = 2 distam 4 unidades da reta b) 30 d) 18
4x – 3y + 2 = 0. A distância, em unidades, entre as abscissas dos pontos é:
a) 10 b) 2 c) 6 d) 4 e) 1 20. (FUVEST) Considere o triângulo ABC no plano cartesiano com
vértices A = (0,0), B(3,4) e C = (8,0). O retângulo MNPQ tem os
14. Seja P(3,1) o ponto médio do segmento AB, onde A é intersecção vértices M e N sobre o eixo das abscissas, o vértice Q sobre o lado AB
da reta (t) com a reta (r) 3x – y = 0 e B, a intersecção de (t) com a reta e o vértice P sobre o lado BC. Dentre todos os retângulos construídos
(s) x + 5y = 0. O coeficiente angular de (t) é: desse modo, o que tem área máxima é aquele em que o ponto P é:
a) negativo.  16   12   8
a)  4,  c)  5,  e)  6, 
b) par positivo.  5   5  5
c) 5, pois (t) é perpendicular à (s).  17   11 
b)  ,3  d)  ,2 
d) nulo, isto é, a reta é do tipo y = k, k = constante.  4  2 
e) um cubo perfeito positivo. 21. (AFA) Seja P(3,1) o ponto médio do segmento AB, onde A é
3x + my = n intersecção da reta (t) com a reta (r) 3x – y = 0 e B, a intersecção de (t)
15. (FGV) Sejam m e n números reais e  um sistema com a reta (s) x + 5y = 0. O coeficiente angular de (t) é:
x + 2y =1
de equações nas incógnitas x e y. A respeito da representação a) negativo.
geométrica desse sistema no plano cartesiano, é correto afirmar que, b) par positivo.
necessariamente, é formada por duas retas:
c) 5, pois (t) é perpendicular à (s).
a) paralelas distintas, se m = 6 e n ≠ 3.
d) nulo, isto é, a reta é do tipo y = k, k = constante.
b) paralelas coincidentes, se m = 6 e n ≠ 3.
c) paralelas distintas, se m = 6. 22. (MACKENZIE) A equação da mediatriz do segmento que une os
d) paralelas coincidentes, se n = 3. pontos P = (1,–2) e Q = (5,4) é:
e) concorrentes, se m ≠ 0. a) 2x + 3y – 9 = 0 d) 3x – 2y – 7 = 0
b) 2x – 3y + 9 = 0 e) 3x + 2y – 11 = 0
c) 2x – 3y – 3 = 0

521

PM_BOOK16 - MAT.indb 521 25/11/2022 19:25:44


GEOMETRIA ANALÍTICA: RETAS

x = sen2 t

23. (AFA) As equações paramétricas  representam:
y = cos t
2

a) um segmento de reta de extremos (0,1) e (1,0)
1
b) uma elipse de eixo maior igual a
2
c) uma hipérbole de eixo real horizontal
d) uma circunferência de centro (0, 0) e raio igual a 1

24. (AFA) As diagonais de um losango estão contidas nas retas


(r) (3m – 1)x + (m –2)y = 0 e (t) x + (m + 1)y + m + 2 = 0. É correto
afirmar que os possíveis valores de m:
a) têm soma igual a 2
b) têm produto igual a 3
c) pertencem ao intervalo ]–3, 3]
(n 3 )
2
n
d) têm sinais opostos a) d)
2 2
b) 3n
25. (ESPCEX) Considere a reta t mediatriz do segmento cujos extremos n (n + 1) 
e)
são os pontos em que a reta s : 2x – 3y + 12 = 0 intercepta os eixos c) 6n 2
coordenados. Então, a distância do ponto M(1,1) à reta t é:
13 3 13 11 3 3 31. (AFA) Considere no plano cartesiano os pontos A(2,0) e B(6,–4)
a) c) e) que são simétricos em relação à reta r. Se essa reta r determina na
11 13 11
circunferência x² + y² – 12x – 4y + 32 = 0 uma corda que mede n
10 13 3 11
b) d) unidades de comprimento, então n pertence ao intervalo:
13 13
a) [4, 5[ b) [3, 4[ c) [2, 3[ d) [1, 2[
26. (AFA) A reta (s), simétrica de (r) x – y + 1 = 0 em relação à reta
(t) 2x + y + 4 = 0, 32. (MACKENZIE) Na figura, as retas r e s são paralelas. Se (x, y) é um
a) passa pela origem. ponto de s, então x – y vale:
b) forma um ângulo de 60º com (r).
1
c) tem – como coeficiente angular.
5
d) é paralela à reta de equação 7y – x + 7 = 0.

27. (AFA) Dadas as retas de equações:


r : y = ax + b
r1 : y = a1x + b1
Determine a relação entre a, a1, b e b1 que está correta.
a) Se a = a1 e b ≠ b1 tem-se r // r1.
b) Se a = a1 e b = b1 tem-se r ≠ r1. a) 2 c) 4 e) 4 2
c) Se a ≠ a1 e tem-se r = r1. b) 2 d) 2 2
d) Se a ≠ a1 e b ≠ b1 tem-se r // r1.
33. (AFA) Considere no plano cartesiano as retas r e s dadas pelas equações:
28. (EFOMM) A forma de uma montanha pode ser descrita pela r : 3x + 3py + p = 0
equação y = –x² + 17x – 66 (6 ≤ x ≤ 11). Considere um atirador munido
s : px + 9y – 3 = 0
de um rifle de alta precisão, localizado no ponto (2,0). A partir de que
ponto, na montanha, um indefeso coelho estará 100% seguro? Onde p ∈ . Baseado nessas informações, marque a alternativa INCORRETA.
a) (8,9) d) (7,5) a) r e s são retas concorrentes se |p| ≠ 3.
b) (8,6) e) (7,4) b) Existe um valor de p para o qual r é equação do eixo das ordenadas
e s é perpendicular a r.
c) (7,9)
c) r e s são paralelas distintas para dois valores reais de p.
29. (EFOMM) A projeção ortogonal de A sobre a reta BC, sabendo-se d) r e s são retas coincidentes para algum valor de p.
que A = (3,7), B = (1,1) e C = (9,6), terá as coordenadas da projeção:
x = 2t
a) x = 468/85; y = 321/89 
34. (AFA) Considere no plano cartesiano as retas r:  1 e
b) x = 478/87; y = 319/87 =
y 3t +
2
k
c) x = 487/84; y = 321/87 s: (k + 1) x – y – =0, onde k ∈ . Sobre as retas r e s é correto
2
d) x = 457/89; y = 319/89
afirmar que NUNCA serão:
e) x = 472/89; y = 295/89
a) concorrentes perpendiculares.

30. (MACKENZIE) Considere os triângulos, nos quais um dos vértices é b) concorrentes oblíquas.
sempre o ponto (0,2) e os outros dois pertencem à reta r, como mostra c) paralelas distintas.
a figura. Para x = 1, 2, 3, ..., n, a soma das áreas dos n triângulos é: d) paralelas coincidentes.

522

PM_BOOK16 - MAT.indb 522 25/11/2022 19:25:45


GEOMETRIA ANALÍTICA: RETAS

35. (AFA) Um quadrado de 9 cm² de área tem vértices consecutivos


sobre a bissetriz dos quadrantes pares do plano cartesiano. Se os EXERCÍCIOS DE
demais vértices estão sobre a reta r, que não possui pontos do 3º
quadrante, é incorreto afirmar que a reta r:
a) pode ser escrita na forma segmentária.
COMBATE
b) possui o ponto P − 2,2 2( ) 01. (EN 2004) O perímetro do triângulo ABC dado na figura abaixo mede:
c) tem coeficiente linear igual a 3 2
d) é perpendicular à reta de equação 2x – 2y = 0

36. (EN) As retas r1 : 2x – y + 1 = 0; r2 : x + y + 3 = 0 e r3 : αx + y – 5 = 0


concorrem em um mesmo ponto P para determinado valor de
α ∈ . Sendo assim, pode-se afirmar que o valor da expressão
 απ  3  ( −3 − α ) π  5 3  −απ 
cos   − 3sen  − tg   é:
 3   8  2  6 
 2 2
a) 3 1 +  d) 3+
 4  4

 2
b) 2−
3 2
4
e) 3 1 −
 4


a) 12 + 2 + 5 c) 12 + 2 + 4 5 e) 12  4  2 5 
b) 6+4 2+ 5 d) 6+ 2+4 5
2
c) 2+
8 02. (ITA 2007) Considere no plano cartesiano xy o triângulo delimitado
pelas retas 2x = y, x = 2y e x = –2y + 10. A área desse triângulo mede
37. (EN) As equações simétricas da reta de interseção dos planos
2x – y – 3 = 0 e 3x + y + 2z – 1 = 0, x, y, z ∈  são: 15 13 11 9 7
a) b) c) d) e)
x y +3 2−z 2 4 6 4 2
a)= =
2 4 5
03. (ESPCEX 2015) O ponto simétrico do ponto (1,5) em relação à reta
x +1 y + 3 z + 3 de equação 2x + 3y – 4 = 0 é o ponto
b) = =
2 4 5 a) (–3,–1) c) (–4,4) e) (3,2)
y +3 2−z b) (–1,–2) d) (3,8)
c)=x =
2 4
3− y z−2 04. (AFA 2013) Sejam a e b dois números reais positivos.
d) x=
−1 =
2 4 As retas r e s se interceptam no ponto (a, b).
x −1 y + 3 z + 2 a   b
e) = = Se  , 0   r e  0,   s, então uma equação para a reta t, que
2 4 5 2   2
passa por (0, 0) e tem a tangente do ângulo agudo formado entre r e
38. (ITA) Considere a reta r : y = 2x. Seja A = (3,3) o vértice de um s como coeficiente angular, é
quadrado ABCD, cuja diagonal BD está contida em r. A área deste a) 3abx + (2a² – b²)y = 0 c) 3ax – a(a² + b²)y = 0
quadrado é:
b) 3bx – b(a² + b²)y = 0 d) 3abx – 2(a² + b²)y = 0
9 21
a) d)
5 5 05. (ITA 2000) A área de um triângulo é de 4 unidades de superfície,
12 24 sendo dois de seus vértices os pontos A : (2,1) e B : (3,–2). Sabendo
b) e)
5 5 que o terceiro vértice encontra-se sobre o eixo das abscissas, pode-se
18 afirmar que suas coordenadas são
c)
5  1   1 
a)   , 0  ou (5,0) d)   , 0  ou (4,0)
39. (ITA) Considere as retas de equações r : y = 2 x + a e  2   3 
s : y = bx + c, e s : y = bx + c, em que a, b, c são reais. Sabendo que r e s  1   1 
são perpendiculares entre si, com r passando por (0,1) e s, por ( 2 ,4),
b)   , 0  ou (4,0) e)   , 0  ou (3,0)
 2   5 
determine a área do triângulo formado pelas retas r, s e o eixo x.
 1 
c)   , 0  ou (5,0)
 3 
40. (ITA) Considere os pontos A = (0,–1), B = (0,5) e a reta
r : 2x – 3y + 6 = 0. Das afirmações a seguir:
06. (UECE 2017) Em um plano, munido do referencial cartesiano usual,
I. d(A,r) = d(B,r). seja A o ponto de interseção das retas 3x + y + 4 = 0 e 2x – 5y + 14 = 0.
II. B é simétrico de A em relação à reta r. Se os pontos B e C são respectivamente as interseções de cada uma
destas retas com o eixo x, então, a área do triângulo ABC, é igual
III. AB é base de um triângulo equilátero ABC, de vértice C = ( −3 3,2)
ou C = (3 3,2). 13 16
a) u.a. c) u.a.
É (são) verdadeira(s) apenas: 3 3
a) I. d) I e III. 14 17
b) u.a. d) u.a.
b) II. e) II e III. 3 3
c) I e II.

523

PM_BOOK16 - MAT.indb 523 25/11/2022 19:25:56


GEOMETRIA ANALÍTICA: RETAS

07. (UERJ 2017) Considere o gráfico a seguir, em que a área S é limitada


pelos eixos coordenados, pela reta r que passa por A(0,4) e B(2,0) e pela
reta perpendicular ao eixo x no ponto P(x0,0), sendo 0 ≤ x0 ≤ 2.
2 (ITA) Se a reta de equação x = a divide o quadrilátero cujos
vértices são (0,1), (2,0), (4,0) e (6,4) em duas regiões da
mesma área, então o valor de a é igual a:

Para que a área S seja a metade da área a) 2 5 − 1 c) 3 5−4 e) 3 7 − 5


do triângulo de vértices C(0,0), A e B, o b) 2 6 − 1 d) 2 7 − 2
valor de x0 deve ser igual a:
a) 2 – 2  25 
b) 3 – 2 3 (ITA) Dados o ponto A =  4,  e a reta r : 3x + 4y – 12 = 0,
 6 
considere o triângulo de vértices ABC, cuja base BC está
c) 4– 2 25
contida em r e a medida dos lados AB e AC é igual a . Então, a
d) 5 – 2 6
área e o perímetro desse triângulo são, respectivamente, iguais a:
22 40
08. (AFA) As diagonais de um losango estão contidas nas retas a) e
(r) (3m – 1)x + (m – 2)y = 0 e (t) x + (m + 1)y + m + 2 = 0. É correto 3 3
afirmar que os possíveis valores de m 23 40
b) e
a) têm soma igual a 2. c) pertencem ao intervalo ]–3,3]. 3 3
b) têm produto igual a 3. d) têm sinais opostos. 25 31
c) e
3 3
09. (AFA) Na figura abaixo, as retas r e s são paralelas. Se P(x,y) d) 25 e 35
pertence à reta s, então x + y é igual a 3 3
25 40
e) e
a) 3 3 3
b) − 3
c) − 6
d) 6 4 (ITA) Sabe-se que a equação 3x² + 5xy – 2y² - 3x + 8y – 6 = 0
representa a reunião de duas retas concorrentes, r e s,
formando um ângulo agudo θ. Determine a tangente de θ.

10. (UFU) Considere o feixe de retas concorrentes no ponto P(8,3).


Seja r a reta desse feixe que determina junto com os eixos cartesianos
5 (ITA) No plano cartesiano são dados o ponto P = (0,3)
e o triângulo de vértices A = (0,0), B = (3,0) e C = (3,2).
Determine um ponto N sobre o eixo dos x de modo que a reta
um triângulo retângulo (ângulo reto na origem) contido no quarto que passa por P e N divida o triângulo ABC em duas regiões de
quadrante e área igual a 6 unidades de área. Na equação geral mesma área.
ax + by + c = 0 da reta r, a soma dos inteiros a + b + c é múltiplo de
a) 7
GABARITO
b) 13
c) 11 EXERCÍCIOS DE FIXAÇÃO
d) 5 01. A 04. B 07. B 10. E
02. D 05. C 08. B
03. D 06. C 09. A
EXERCÍCIOS DE TREINAMENTO

DESAFIO PRO 01. B


02. A
11. C
12. A
21. A
22. A
r= : y
31. A
32. C
2x + 1
03. D 13. A 23. A 33. D
− 2
1 (IME) Sejam os pontos A(0,0), B(–1,1), C(1,2), D(4,1) e
 1
E  3,  . A reta r passa por A e corta o lado CD, dividindo o
04. C
05. D
14. A
15. A
24. D
25. B
=s : y 34. D x + 5

− 2 
35. B
2
⇒ ( 2 )
2
+ =
2 
 5 2 +
 2 ,0 E∈ r 
pentágono ABCDE em dois polígonos de mesma área. Determine
06. D 16. E 26. D  2 36.
 
a soma das coordenadas do ponto de interseção da reta r com 07. B 17. D 27. A 37. A
a reta que liga C e D. 08. D 18. D 28. B (5 2,0 )∈
38. C s
25 09. B 19. A 29. D 22 121 2
a) =S 39. S=
=
7 10. C 20. D 30. B 2 12
40. D
51
b) EXERCÍCIOS DE COMBATE
14
01. E 04. D 07. A 10. A
26
c) 02. A 05. C 08. D
7
03. A 06. D 09. C
53
d)
14 DESAFIO PRO
27 01. C 03. E  1 + 19 
e) 05. N =  , 0  .
7 02. D 04. tg = 7  2 

524

PM_BOOK16 - MAT.indb 524 25/11/2022 19:25:58


GEOMETRIA ANALÍTICA:
CIRCUNFERÊNCIA

CIRCUNFERÊNCIA POTÊNCIA DE UM PONTO EM


A circunferência é o lugar Geométrico dos pontos de um plano RELAÇÃO A UMA CIRCUNFERÊNCIA
equidistante de um ponto fixo do mesmo plano, chamado de centro.
Dados um ponto P(xp, yp) e uma circunferência (x – x0)2 + (y – y0)2 = R2,
chamamos potência do ponto em relação à circunferência ao número
real
k = (xp – x0)2 + (yp – y0)2 – R2.

A soma das duas primeiras parcelas representa o quadrado da


distância do ponto P ao centro C da circunferência, então:
se k > 0 o ponto P é exterior;
se k = 0 o ponto P pertence à circunferência;
se k < 0 o ponto P é interior.

POSIÇÕES RELATIVAS ENTRE RETA E


CIRCUNFERÊNCIA

RETA EXTERIOR

Seja a circunferência de centro O(x0,y0) e raio r e P um ponto


qualquer (x,y).
 
Pela definição temos OP = r mas OP  ( x  x o )2  ( y  y o )2
substituindo r na equação acima e elevando ao quadrado (x – x0)² +
(y – y0)² = r². Que é a equação reduzida da circunferência.
Desenvolvendo a equação reduzida temos:
x² + y² – 2x0x – 2y0y + x0² + y0² – r² = 0

Fazendo –2x0 = D, – 2y0 = E, x0² + y0² – r² = F obtemos x² + y² + Dx


+ Ex + F = 0 (equação geral da circunferência) que é uma circunferência
 D E 1 2 2
de centro (x 0 , y 0 )    ,   e raio igual a D  E  4F.
 2 2 2
| ax 0  by 0  c |
Para que a equação x² + y² + Dx + Ex + F = 0 represente uma d R
a† b†
circunferência devemos ter D² + E² – 4F > 0.

Exercício Resolvido

01. Determine a equação da circunferência, sabendo-se que um


de seus diâmetros é o segmento de extremos A(1,3) e B(5,–3).

Resolução:
O ponto médio de AB é o centro C(3,0) da circunferência e
R2 = (3 – 1)2 + (0 – 3)2 ⇒ R2 = 13
A equação da circunferência é (x – 3)2 + y2 = 13.

525

PM_BOOK16 - MAT.indb 525 25/11/2022 19:26:03


GEOMETRIA ANALÍTICA: CIRCUNFERÊNCIA

RETA TANGENTE As circunferências são INTERIORES se, e somente se, a distância


entre seus centros for maior do que zero e menor do que o módulo da
diferença entre seus raios: 0 < d(O,O’) < |R – r|.

| ax 0  by 0  c |
d R
a† b†

RETA SECANTE As circunferências são TANGENTES INTERIORES se, e somente


se, a distância entre seus centros for igual ao módulo da diferença
entre seus raios: d(O,O’) = |R – r|.

| ax 0  by 0  c |
d R
a† b†

As circunferências são SECANTES se, e somente se, a distância


POSIÇÕES RELATIVAS ENTRE entre seus centros for maior do que o módulo da diferença entre seus
CIRCUNFERÊNCIAS raios e menor do que a soma dos raios: |R – r| < d(O,O’) < R + r.
Sejam duas circunferências de centros O e O’, e raios r e R,
respectivamente.
As circunferências são CONCÊNTRICAS se, e somente se, a
distância entre seus centros for nula: d(O,O’) = 0.

526

PM_BOOK16 - MAT.indb 526 25/11/2022 19:26:04


GEOMETRIA ANALÍTICA: CIRCUNFERÊNCIA

As circunferências são TANGENTES EXTERIORES se, e somente RETA TANGENTE A CIRCUNFERÊNCIA


se, a distância entre seus centros for igual à soma dos raios:
d(O,O’) = R + r. Sempre que desejamos encontrar a intersecção entre 2 curvas
devemos resolver um sistema envolvendo as equações das 2 curvas.
O sistema encontrado entre uma equação de circunferência e
 = y mx + n
uma equação de reta seria  2 , assim por
 x + y 2 + Bx + cy + D =
0
substituição teremos
x 2 + (mx + n) + Bx + c (mx + n) + D =
2
0

x 2 + m2x 2 + 2mnx + n2 + Bx + cmx + cn + D =


0

(m 2
+ 1) x 2 + (B + cm + 2mn) x + (n2 + cn + D) =
0

Logicamente isso não foi exposto para ser decorado mas sim para
mostrar que todo sistema entre uma equação de reta e uma equação
de circunferência resulta numa equação do 2° grau onde daí sabemos
As circunferências são EXTERIORES se, e somente se, a distância que
entre seus centros for maior do que a soma dos raios: d(O,O’) > R + r. Ä
∆ > 0 → 2pontos de intersecção → reta secante
Ä
∆=0 → 1ponto de intersecção → reta tangente
Ä
∆ < 0 → Nenhumponto de intersecção → reta exterior
Daí da equação (m2 + 1) x 2 + (B + cm + 2mn) x + (n2 + cn + D) =
0
resultante do sistema basta fazermos o delta corresponde a situação
de reta que nos for pedida.
Exemplo:
Encontre a equação de reta que passa pelo ponto ( −1, 0 ) e é
tangente a circunferência x + y − 12x + 20 =
2 2
0.
y mx + n que passa por ( −1, 0 ) → 0 = m ( −1) + n → m = n ,
Reta =
assim =
y mx + m .
 =y mx + m
⇒ x 2 + (mx + m ) − 12x + 20 =0 ⇒
2
 2
 x + y 2
− 12x + 20 =
0
POSIÇÃO RELATIVA ENTRE DISTÂNCIA ENTRE
AS CIRCUNFERÊNCIAS SEUS CENTROS x 2 + m2x 2 + 2m2x + m2 − 12x + 20 =
0

CONCÊNTRICAS d(O,O’) = 0 x 2 (m2 + 1) + ( 2m2 − 12) x + (m2 + 20 ) =


0
INTERIORES 0 < d(O,O’) < |R – r|
tangente → ∆ = 0 → ( 2m2 − 12) − 4. (m2 + 1) . (m2 + 20 ) = 0
2

TANGENTES INTERIORES d(O,O’) – |R – r|


4m4 − 48m2 + 144 − 4 (m4 + 21m2 + 20 ) =
0
SECANTES |R – r| < d(O,O’) < R + r
4m4 − 48m2 + 144 − 4m4 − 84m2 − 80 =
0
TANGENTES EXTERIORES d(O,O’) = R + r
64 16 4 33
EXTERIORES d(O,O’) > R + r −132m2 + 64 =⇒
0 m2 = = ⇒ m =±
132 33 33

4 33 4 33
=
Assim nossas retas tangentes são y x+ e
POSIÇÕES RELATIVAS ENTRE CIRCUNFERÊNCIAS 33 33
4 33 4 33
y=
− x−
SEJAM DUAS CIRCUNFERÊNCIAS DE RAIOS R 33 33
E R, E D A DISTÂNCIA ENTRE SEUS CENTROS, TEM-SE:

circunferências exteriores D>R+r

circunferências tangentes
D=R+r
exteriormente

circunferências secantes R – r < d < R + r

circunferência menor interior à maior 0 < d <R – r

circunferências concêntricas d=0

527

PM_BOOK16 - MAT.indb 527 25/11/2022 19:26:05


GEOMETRIA ANALÍTICA: CIRCUNFERÊNCIA

Também é possível encontrar a reta tangente utilizando a fórmula 03. As equações (x + 1)² + (y – 4)² = 64 e (x – 4)² + (y + 8)² = 25
de distância de ponto a reta. Para isso devemos encontrar o centro e representam duas circunferências cuja posição relativa no plano
o raio da circunferência. permite afirmar que são:
x 2 + y 2 − 12x + 20 =0 ⇒ x 2 − 2.6.x + 36 − 36 + ( y − 0 ) + 20 =0 ⇒
2 a) interiores (sem ponto de intersecção).
b) tangentes interiores.
( x − 6) + ( y − 0 ) − 16 =0 ⇒ ( x − 6 ) + ( y − 0 ) =16
2 2 2 2
c) secantes.
d) tangentes exteriores.
Centro ( 6, 0 ) e R = 16 ⇒ R = 4
2

e) exteriores (sem ponto de intersecção).


A reta como vimos é y = mx + m ⇒ mx − y + m= 0
Assim pela fórmula de distância de ponto a reta sendo a reta na 04. Dada a equação da circunferência (x – a)² + (y – b)² = r², sendo
sua forma completa e seus coeficientes a = m,b = −1 e c = m . (a; b) as coordenadas do centro e r a medida do raio, identifique a

( )
m.6 − 1.0 + m 7m 2 equação geral da circunferência de centro (2; 3) e raio igual a 5.
=4 ⇒ =4 ⇒ 49m2 = 4 m2 + 1 ⇒
m + ( −1)
2 2
m +1
2 a) x² + y² – 4x – 6y – 12 = 0
b) x² – 4x = -16
16 4 33 c) x² + y² = 25
49m =
16m + 16 ⇒ 33m =
2
16 ⇒ m = ⇒ m =
2
±2 2

33 33 d) y² – 6y = -9
e) x² + y² – 4xy – 12 = 0
CONDIÇÕES PARA QUE UMA 05. Se a circunferência de equação x² + by² + cx + dy + k = 0 tem
EQUAÇÃO DO 2º GRAU COM DUAS centro C(1, -3) e raio 3 , então “b + c + d + k” é igual a

VARIÁVEIS REPRESENTE UMA a) 12. b) 11. c) 10. d) 9.

CIRCUNFERÊNCIA 06. Se uma circunferência tem centro C(1,0) e raio 1 e outra tem
Procuremos as condições a que deve satisfazer a equação geral equação x² + y² – 2x – 8y + 8 = 0. Então essas circunferências são
do 2º grau com duas variáveis, Ax2 + Bxy + Cy2 + Dx + Ey + F = 0 para a) secantes
representar uma circunferência. b) externas
Concluímos que, dada uma equação do 2º grau com duas c) tangentes internas
variáveis, as condições necessárias para que a mesma represente uma
d) tangentes externas
circunferência, no sistema cartesiano ortogonal, são:
07. Se a distância entre uma reta t e o centro da circunferência
A=C≠0 (λ) x² + (y – 2)² = 16 é 17 , então t e λ são
B=0 a) secantes c) exteriores
b) tangentes d) interiores
Respeitadas estas condições determinamos:
D E  D E  08. Dados os pontos B(1,2) e C(0,1) uma circunferência λ de equação
1. x 0  
2A
e y0  
2A
o centro da circunferência  , . x² + y² – 3x – 4 = 0, é correto afirmar que
 2A 2A 
a) B é interior a λ e C é exterior a λ.
D2  E2  4 AF b) B é exterior a λ e C é interior a λ.
2. R  o raio da circunferência, para isso devemos
2A c) B e C são exteriores a λ.
ter D2 + E2 – 4AF > 0. d) B e C são interiores a λ.

09. A reta y = mx + 2 é tangente à circunferência de equação


EXERCÍCIOS DE (x – 4)² + y² = 4. A soma dos possíveis valores de m é

FIXAÇÃO
a) 0.
b) 4/3.
c) – 4/3.
d) – 3/4.
01. Considere a circunferência (x – 2)² + (y – 4)² = 9 e uma reta t secante
a ela. Uma possível distância entre r e o centro da circunferência é e) 2.

a) 5,67
10. Em um sistema de coordenadas cartesianas no plano, considere
b) 4,63 os pontos O(0,0) e A(8,0). A equação do conjunto dos pontos P(x,y)
c) 3,58 desse plano sabendo que a distância de O a P é o triplo da distância
d) 2,93 de P a A, é uma
a) circunferência de centro (9,0) e raio 3.
b) elipse de focos (6,0) e (12,0), e eixo menor 6.
02. Seja a circunferência de centro (0, –2) e raio 5 . Se (k,0) pertence
à circunferência, sendo k > 0, o valor de k é c) hipérbole de focos (3,0) e (15,0), e eixo real 6.
a) 0 d) parábola de vértice (9,3), que intercepta o eixo das abscissas nos
pontos (6,0) e (12,0).
b) 1
e) reta que passa pelos pontos (6,0) e (9,3).
c) 2
d) 3

528

PM_BOOK16 - MAT.indb 528 25/11/2022 19:26:08


GEOMETRIA ANALÍTICA: CIRCUNFERÊNCIA

EXERCÍCIOS DE 5 10 20 10

TREINAMENTO
a) . c) . e) .
3 3 9
10 5 10
b) . d) .
3 9
01. A equação da circunferência, em que os pontos M(-3,2) e N(5,4)
são extremos de um diâmetro, é 10. (CEFET MG) Em um plano, uma reta que passa pelo ponto P(8,10)
tangencia a circunferência x 2 + y 2 – 4x – 6y – 3 =
0 no ponto A. A
a) x2 + y2 − 5 =0. d) x 2 + y 2 − 2x − 6y − 5 =0. medida do segmento PA, em unidades de comprimento, é
b) x + y − 17 =
2 2
0. e) x 2 + y 2 − 2x − 6y − 12 =
0 a) 12 . c) 45 . e) 85 .
c) x 2 + y 2 − 2x − 6y − 7 =0. b) 34 . d) 69 .

02. (EEAR) As posições dos pontos A(1,7) e B(7,1) em relação à 11. (FGV) No plano cartesiano, a região determinada pelas inequações
circunferência de equação (x − 6)2 + (y − 2)2 =
16 são, respectivamente, simultâneas x 2 + y 2 ≤ 4 e x + y ≤ 0 tem área igual a:
a) interna e interna. c) externa e interna. a) 2π c) 3π e) 4π
b) interna e externa. d) externa e externa. b) 2,5π d) 3,5π

03. O maior valor inteiro de k para que a equação 12. (FGV) No plano cartesiano, a reta de equação 3x + 4y = 17
x 2 + y 2 + 4x − 6y + k =0 represente uma circunferência é tangencia uma circunferência de centro no ponto (1,1).
a) 14 c) 12 e) 8 A equação dessa circunferência é:
b) 13 d) 10 a) x 2 + y 2 − 2x − 2y − 4 =
0 d) x 2 + y 2 − 2x − 2y − 3 =
0

04. Uma corda é determinada pela reta x – y = 0 sobre a circunferência b) x + y − 2x − 2y − 2 =


2 2
0 e) x 2 + y 2 − 2x − 2y − 1 =
0
( x − 2) + ( y + 2) = x 2 + y 2 − 2x − 2y − 5 =
2 2
16 . A área da menor região determinada por essa c) 0
corda e o círculo é:
a) 4π – 8 b) 4π – 16 c) 4π – 2 d) 4π – 4 x y
13. (AFA) A reta − = 1 , a > 0, intercepta os eixos coordenados x e
a a
y nos pontos P e Q, respectivamente. A equação geral da circunferência
05. Dadas a reta de equação y = 3 x e a circunferência de equação tangente ao eixo x no ponto P e tangente ao eixo y no ponto Q é
3
x 2 + y 2 − 4x =
0 . A área do triângulo determinado pelo centro a) x2 + y2 – 2ax + 2ay + a2 = 0
da circunferência e os pontos de intersecção entre a reta e ela, em b) x2 + y2 + 2ax – 2ay + a2 = 0
unidades de área, é igual a c) x2 + y2 + 2ax + 2ay + a2 = 0
a) 3 b) 3 c) 3 3 d) 6 d) x2 + y2 – 2ax – 2ay + a2 = 0

06. (AFA) A equação da circunferência de raio 5, concêntrica à 14. (AFA) A reta s : y = –x + 4 intercepta a circunferência C : x2 + y2 +
circunferência de equação x2 + y2 – 4x – 2y + 3 = 0, é: 2x – 4y – 4 = 0 nos pontos P e Q. Se O é o centro de C, então a área
a) x2 + y2 – 4x – 2y + 20 = 0 do triângulo OPQ, em unidades de área, é
b) x2 + y2 – 4x – 2y – 15 = 0 a) 4 c) 4,5
c) x2 + y2 – 4x – 2y = 0 b) 5 d) 5,5
d) x2 + y2 – 4x – 2y – 20 = 0
15. (FGV) O número de pares ordenados (x,y), com x e y inteiros, que
07. (MACKENZIE) Os valores de a para os quais as circunferências de satisfazem a desigualdade x 2 + y 2 − 8x + 11 ≤ 0 é igual a
1 e (x − a)2 + (y + 2)2 =
equações (x − 3)2 + (y − 2)2 = 16 são tangentes a) 24. c) 19. e) 13.
exteriormente são b) 21. d) 18.
a) -2 e 8 c) -8 e 2 e) -6 e 0
b) 2 e 8 d) 0 e 6 16. (FGV) No plano cartesiano, a equação da reta tangente ao gráfico
de x 2 + y 2 =
25 pelo ponto (3,4) é
08. (CEFET MG) Considere as circunferências λ1 : (x + 2)2 + (y + 1)2 = 5 a) 4x + 3y − 25 =
0. d) 3x + 4y − 25 =
0.
e λ 2 : (x − 4)2 + (y − 3)2 =
9. A área do triângulo cujos os vértices são b) 4x + 3y − 5 =0. e) 3x + 4y − 5 =0.
 5 c) 4x + 5y − 9 =0.
os centros dessas circunferências e o ponto P  0,  , em unidades de
área, é igual a  2
13 9 5 17. (MACKENZIE) A equação da reta que corta o eixo das ordenadas
a) . c) . e) . no ponto P = (0,–6) e que tangencia a circunferência x 2 + y 2 =
4 no
2 4 4
quarto quadrante é
11 7
b) . d) .
2 4 a) y=
−2 2x + 6
b)=y 2 2x − 6
09. (CEFET MG) No plano cartesiano, duas retas r e s se interceptam
num ponto S(x,0) e tangenciam a circunferência x2 + y2 = 10 nos c)=y 2 2x + 6
pontos P(3,p) e Q(3,q), respectivamente. Os pontos P, Q, S e O, sendo d) =
y 4x − 6
O o centro da circunferência, determinam um quadrilátero cuja área,
e) y=−4x + 6
em unidades de área, é

529

PM_BOOK16 - MAT.indb 529 25/11/2022 19:26:09


GEOMETRIA ANALÍTICA: CIRCUNFERÊNCIA

18. (MACKENZIE) Duas pessoas patinam sobre o gelo descrevendo 27. (AFA) Os vértices de um triângulo ABC são os centros das
trajetórias circulares. As circunferências descritas por elas são circunferências:
dadas pelas equações (x + 3)2 + (y + 1)2 = 10 e (x + 3)2 + y 2 =13, (λ1) x2 + y2 + 2x – 4y – 1 = 0
respectivamente. A distância entre os dois pontos de interseção das (λ2) 4x2 + 4y2 + 12x – 8y – 15 = 0
circunferências é
(λ3) (x – 7)2 + (y + 3)2 = 8
a) 3 b) 4 c) 5 d) 6 e) 7
O tetraedro cuja base é o triângulo ABC e cuja altura, em metros, é
igual à média aritmética dos quadrados dos raios das circunferências
19. (AFA) A equação y =3 + 4 − (x − 1)2 representa: acima, também em metros, possui volume, em m3, igual a
a) elipse de eixo maior igual a 2 a) 21/2 b) 21/4 c) 49/2 d) 49/4
1
b) parábola de vértice V (1, 3) e parâmetro p =
2
28. (EFOMM) Sejam as circunferências c1 : x 2 + y 2 − 16 = 0 e
c) hipérbole de eixo real vertical e centro C (1, 3)
c2 : (x − 2)2 + (y + 2)2 =4. Considere A e B os pontos de intersecção
d) semicircunferência de centro C (1, 3) e raio r = 2 dessas circunferências. Determine a distância entre A e B.
a) 2 7 c) 2 14 7
20. (MACKENZIE) A equação da circunferência concêntrica à e)
b) 14 d) 7 2
circunferência (x + 2)2 + (y − 1)2 =
1 e tangente à reta 4x + 3y − 20 =
0 é
a) (x + 2)2 + (y − 1)2 =36 d) (x + 2)2 + (y − 1)2 =
16 29. (EFOMM) Quanto à posição relativa, podemos classificar as
b) (x + 2)2 + (y − 1)2 =25 e) (x + 2)2 + (y − 1)2 =9 circunferências (x − 2)2 + (y − 3)2 =
9 e x 2 + y 2 − 8x + 15 =
0
c) (x + 2)2 + (y − 1)2 =20 a) secantes. d) externas.
b) tangentes internas. e) internas.
21. (MACKENZIE) Há duas circunferências secantes λ1 e λ2, de c) tangentes externas.
equações (x − 1)2 + y 2 =
5 e (x − 3)2 + (y − 2)2 =
1, respectivamente.
A equação da reta que passa pelos pontos de interseção de λ1 e λ2 é 30. (ESPCEX) Uma circunferência tem centro no eixo das abscissas,
a) x+y−4=0 c) x −y −6 =0 e) x−y−8=0 passa pelo ponto (4,4) e não intercepta o eixo das coordenadas. Se a
área do círculo definido por essa circunferência é 17π, a abscissa de
b) x+y+4=0 d) x+y+8=0
seu centro é

22. (MACKENZIE) Considere a região do plano dada pelos pontos (x,y) tais a) 3. b) 4. c) 5. d) 6. e) 7.
que x 2 + y 2 ≤ 2x e x 2 + y 2 ≤ 2y. Fazendo π = 3, a área dessa região é
31. (ESPCEX) Seja C a circunferência de equação
a) 1 c) 2 e) 2,5
x 2 + y 2 + 2x + 4y + 2 =0. Considere em C a corda MN cujo ponto
b) 0,5 d) 1,5 médio é P(-1,-1). O comprimento de MN (em unidade de comprimento)
é igual a
23. (MACKENZIE) Os pontos (x,y) do plano tais que x² + y ≤ 36, com a) 2 c) 2 2 e) 2
x + y ≥ 6 definem uma região de área
b) 3 d) 2 3
a) 6 ( π − 2) c) 9 ( π − 2) e) 18( π − 2)
b) 9−π d) 6−π 32. (ESPCEX) Considere a circunferência que passa pelos pontos (0,0),
(0,6) e (4,0) em um sistema de coordenadas cartesianas ortogonais.
24. (MACKENZIE) Uma circunferência de centro (4,y), com y ∈  é tangente Sabendo que os pontos (0,6) e (4,0) pertencem a uma reta que passa
às retas x + y – 2 = 0 e x – 7y + 2 = 0. O raio dessa circunferência é pelo centro dessa circunferência, uma das retas tangentes a essa
circunferência, que passa pelo ponto (3,-2), tem por equação
a) 4 c) 4 2 e) 6 2
b) 5
a) 3x − 2y − 13 =
0 d) x − 5y − 13 =
0
d) 5 2
b) 2x − 3y − 12 =
0 e) 8x + 3y − 18 =
0
25. (AFA) A circunferência de equação x2 + y2 – 8x + 8y + 16 = 0 e c) 2x − y − 8 =0
centro C é tangente ao eixo das abscissas no ponto A e é tangente ao
eixo das ordenadas no ponto B. A área do triângulo ABC vale. 33. (AFA) Um cursinho tem representado na figura abaixo o seu
a) 4 b) 8 c) 12 d) 16 logotipo que é contornado por um triângulo equilátero ABC, cujo
 3
baricentro é o ponto P  0,  . No interior desse triângulo há o
26. (AFA) Os pontos A (0,0) e B (3,0) são vértices consecutivos de
 3 
um paralelogramo ABCD situado no primeiro quadrante. O lado AD é quadrado DEFG inscrito na circunferência λ1 e, ao mesmo tempo,
perpendicular à reta y = -2x e o ponto D pertence à circunferência de circunscrito à circunferência λ2. Considerando os dados acima,
centro na origem e raio 5 . Então, a diagonal AC mede classifique as alternativas abaixo em (V) verdadeira(s) ou (F) falsa(s).

a) 38
b) 37
c) 34
d) 26

530

PM_BOOK16 - MAT.indb 530 25/11/2022 19:26:11


GEOMETRIA ANALÍTICA: CIRCUNFERÊNCIA

2 3
( ) A equação geral de λ1 é x 2 + y 2 − y= 0 
3  y≥0
( ) A coroa circular sombreada na figura pode ser representada pelo  x≥0
c) a região sombreada é definida por 
 2  3x + 4y ≤ 1
 2  3 1 
 x +  y −  ≥
 (x − 1) + (y − 1) ≥ 1
2 2
  3  3
conjunto de pontos Q (x, y), tais que 
  
2
d) o conjunto de pontos do plano cartesiano equidistantes de A e B
x 2 +  y − 3 1
  ≤
 é representado por 8x – 6y – 7 = 0
  3  6

( ) A reta suporte que contém o segmento BC pode ser representada 36. (AFA) A circunferência (λ) x2 + y2 – 2x – 2y + k = 0 passa pelo
por y = − 3x + 3 ponto A(0,1). Sabendo-se que o ponto P de (λ) mais próximo da
A sequência correta é origem coincide com o baricentro do triângulo MNQ, onde M(0,k),
N(2k, 0) e Q(xQ , yQ ) é correto afirmar que a área do triângulo MNQ é
a) V - V - V b) V - F - V c) F-V-V d) V - V – F um número do intervalo

34. (AFA) No plano cartesiano, a figura abaixo representa duas a)  3 b) 3  c)  5 d) 5 


1, 2   2 ,2 2, 2   2 ,3
circunferências concêntricas λ1 e λ2, cujo centro é o ponto C. Sabe-
se que λ1 é contorno de um círculo representado pela equação
37. (AFA) Considere no plano cartesiano a circunferência λ tangente
(x − 1)2 + (y + 2)2 ≤ 4 e que AB , que mede 8 cm, é corda da
à bissetriz dos quadrantes ímpares no ponto A(1,1). Sabendo que a
circunferência maior λ2. Considerando também que AB é tangente a
reta t: x – y + 4 = 0 tangencia λ no ponto B, marque a opção correta.
λ1, classifique em (V) verdadeira ou (F) falsa, cada proposição a seguir.
a) A soma das coordenadas de B é igual a 3.
b) P(-1,2) é exterior a λ.
c) O ponto de λ mais próximo da origem é Q(0, 2 − 2).
d) A bissetriz dos quadrantes pares é exterior a λ.

38. (AFA) Seja λ : 3x 2 + 3y 2 − 6x − 12y + k =0, uma circunferência que


no plano cartesiano tem intersecção vazia com os eixos coordenados.
Considerando k ∈ , é correto afirmar que
k k 
a) P  ,  é interior a λ.
 3 3
( ) λ1 é tangente ao eixo das abscissas. b) existem apenas dois valores inteiros para k.
( ) A soma das coordenadas de A e B é um número maior que 5. c) a reta t: x = k intersecta λ.
d) se c é o comprimento de λ então c > 2π unidades de comprimento.

 x≥3
( ) A região sombreada é representada por 
( x − 1)2 + ( y + 2)2 ≤ 20 39. (AFA) Considere os pontos A(4,-2), B(2,0) e todos os pontos

x = 1 − t P(x,y), sendo x e y números reais, tais que os segmentos PA e PB são
( ) A reta  é perpendicular à reta que passa pelos pontos A catetos de um mesmo triângulo retângulo.
t
 y=
 2 É correto afirmar que, no plano cartesiano, os pontos p(x,y) são tais
que
e C.
a) são equidistantes de C(2,-1).
A sequência correta é:
a) F – V – V – F c) V–F–F–V b) o maior valor de x é 3 + 2 .

b) V – V – F – F d) V – F – V – V c) o menor valor de y é -3.


d) x pode ser nulo.
35. (AFA) Seja λ uma circunferência inscrita em um triângulo retângulo
AOB cujos catetos estão sobre os eixos cartesianos e medem 3 cm e 4 40. (AFA) Considerando a circunferência de equação
cm, conforme a figura abaixo. λ : x 2 + y 2 + 2x − 4y − 4 =
0, é correto afirmar que
a) λ é concêntrica com α : (x − 1)2 + (y − 2)2 =
1.
b) o ponto O(0,0) é exterior a λ.
c) a reta r: x – y + 3 = 0 é tangente a λ.
d) λ é simétrica da circunferência β : (x − 1)2 + (y + 2)2 =
9, em
relação ao ponto O(0,0).

41. (AFA) Considere no plano cartesiano um triângulo equilátero ABC


em que:
É INCORRETO afirmar que - os vértices B, de abscissa positiva, e C, de abscissa negativa, estão

sobre o eixo OX;
a) o ponto de λ mais próximo da origem tem a soma das coordenadas
 3
igual a 2 − 2 - possui baricentro no ponto G  0, 
 3 
b) a área da região sombreada é menor que 3 cm2

531

PM_BOOK16 - MAT.indb 531 25/11/2022 19:26:12


GEOMETRIA ANALÍTICA: CIRCUNFERÊNCIA

Considere também, nesse mesmo plano cartesiano, a circunferência π 1 π e) π−2


λ1 inscrita e a circunferência λ2 circunscrita ao triângulo ABC. Analise a) − c) −1
4 4 2
as proposições abaixo e escreva (V) para verdadeira e (F) para falsa.
b) π 1 d) π 1
( ) A reta r, suporte do lado AB, passa pelo ponto (–1, b), em que b é − −
o dobro do oposto do coeficiente angular de r. 2 4 4 2

) O círculo delimitado por λ2 contém o ponto  − , 3  .


1
( 47. (EN) Quantas unidades de área possui a região plana limitada pela
 2 
3 curva de equação y =− 3 − x 2 − 2x e a reta y = x – 1?
( ) O ponto da bissetriz dos quadrantes ímpares de abscissa
pertence a λ1. 3 π 1 e) π−2
a) − c) 3π + 2
4 4
A sequência correta é
d) π 1
a) V - F - V b) π 1 −
− 4 2
2 4
b) F - F - V
c) V-F-F 48. (ITA) São dadas as retas (r) x - y + 1 + 2 = 0 e (s) x 3 + y – 2 +
d) F - V - F 3 = 0 e a circunferência (C) x2 + 2x + y2 = 0.
Sobre a posição relativa desses três elementos, podemos afirmar que:
3
42. (AFA) A circunferência λ é tangente à reta r : y =
x também a) r e s são paralelas entre si e ambas são tangentes à C.
4
é tangente ao eixo das abscissas no ponto de abscissa 6. Dentre b) r e s são perpendiculares entre si e nenhuma delas é tangente à C.
as equações abaixo, a que representa uma parábola que contém a c) r e s são concorrentes, r é tangente à C e s não é tangente à C.
origem do plano cartesiano e o centro de λ é
d) r e s são concorrentes, s é tangente á C e r não é tangente à C.
a) 12(y − x) + x 2 =
0 e) r e s são concorrentes e ambas são tangentes à C.
b) 3y 2 − 12y + 2x =
0
49. (ITA) Duas retas r1 e r2 são paralelas à reta 3x – y = 37 e tangentes à
c) 2y 2 − 3x =
0
circunferência x2 + y2 – 2x – y = 0. Se d1 é a distância de r1 até a origem
d) 12y − x 2 =
0 e d2 é a distância de r2 até a origem, então d1 + d2 é igual a
a) 12 .
43. (AFA) No plano cartesiano, a circunferência λ de equação
x 2 + y 2 − 6x + 10y + k =0, com k ∈ , determina no eixo das b) 15 .
ordenadas uma corda de comprimento  = 8. c) 7.
Dessa forma, é correto afirmar que d) 10 .

a) λ é tangente ao eixo Ox . 5.
e)
b) o raio de λ é igual a k .
c) P(k,-1) ∈ λ. 50. (ITA) Uma circunferência passa pelos pontos A = (0, 2), B = (0, 8)
e C = (8, 8).
d) λ é secante à reta x = k.
Então, o centro da circunferência e o valor de seu raio, respectivamente,
44. (EN) Quantas unidades de área possui a região plana limitada são
pela curva de equação x =1 − 1 − y 2 e pelas retas 2y + x − 3 =0, a) (0, 5) e 6. d) (4, 5) e 5.
2y − x + 3 =0 e x = 2? b) (5, 4) e 5. e) (4, 6) e 5.
1 c) (4, 8) e 5,5.
a) π+
2
3
b) π+
2 EXERCÍCIOS DE

c)
π
2
+1
COMBATE
d) π+3
π 3
e) + 01. (EN 2009) A medida da área da região plana limitada pela curva
2 2
de equação y  4 x  x 2 e pela reta de equação y = x mede, em
45. (EN) A equação da circunferência tangente às retas y = x e y = -x unidades de área,
nos pontos (3,3) e (-3,3) é 
a) 2
a) x 2 + y 2 − 12x + 18 =
0 4
b) π–2
b) x 2 + y 2 − 12y + 18 =
0 c) π+4
c) x 2 + y 2 − 6x + 9 =0 d) π+2
d) x 2 + y 2 − 6y + 9 =0 e) π–1
e) x + y − 16x + 20 =
2 2
0
02. (AFA 2003) A circunferência de equação x² + y² – 8x + 8y + 16 = 0
e centro C é imagem ao eixo das abscissas no ponto A e é tangente ao
46. (EN) Quantas unidades de área possui a região plana limitada pela
eixo das ordenadas no ponto B. A área do triangulo ABC vale.
curva de equação y =− − ( x 2 + 6x + 8 ) e pela reta y = x + 2? a) 4 b) 8 c) 12 d) 16

532

PM_BOOK16 - MAT.indb 532 25/11/2022 19:26:15


GEOMETRIA ANALÍTICA: CIRCUNFERÊNCIA

03. (AFA 2001) A circunferência x² + y² = 5 possui duas retas tangentes 10. (ITA 1993) Uma das circunferências que passa pelo ponto P:(0, 0)
t1 e t2 que são paralelas à reta r: y = –2x + 3. As equações gerais das e tangencia as retas (r1): x – y = 0 e (r2): x + y – 2 = 0 tem sua equação
dada por:
retas t1 e t2, respectivamente, são
a) 2x + y – 5 = 0 e 2x + y + 5 = 0 a) (x – 1)2 + (y + 1)2 = 2
b) (x – 1)2 + (y + 1)2 = 2
b) 2x + y – 15 = 0 e 2x + y + 15 = 0
c) (x – 1)2 + (y – 1)2 = 2
c) 2x  y  5 5  0 e 2x  y  5 5  0
d) (x +1)2 + (y – 1)2 = 2
4 5 4 5 e) (x +1)2 + (y + 1)2 = 2
d) 2x  y   0 e 2x  y  0
5 5

04. (EN 2005)

DESAFIO PRO
1 (ITA) Seja y a circunferência de equação x² + y² = 4. Se r e s
são duas retas que se interceptam no ponto P = (1, 3) e são
tangentes a y, então o cosseno do ângulo entre r e s é igual a
a) 1 c) 1 2 6
. . e) .
5 2 5
b) 7 d) 2
. .
7 2
A área da região hachurada na figura acima é igual a:

a)

8
d)

8
2 (ITA) Considere dois círculos no primeiro quadrante:
π
5π - C1 com centro (x1,y1), raio r1 e área .
7π 16
b) e)
6 16 - C2 com centro (x2,y2), raio r2 e área 144π.

c) Sabendo que (x1,y1,r1) e (x2,y2,r2) são duas progressões geométricas
7 7
com somas dos termos iguais a e 21, respectivamente, então
4
05. (ITA 2010) Determine uma equação da circunferência inscrita no
a distância entre os centros de C1 e C2 é igual a
triângulo cujos vértices são A = (1,1), B = (1,7) e C = (5,4) no plano
xOy. a) 123 c) 131 137
. . e) .
2 2 2
06. (ESPCEX) O ponto da circunferência x² + y² + 2x + 6y + 1 = 0 que
129 d) 135
tem ordenada máxima é b) . .
2 2
a) (0, –6) c) (–1, 0) e) (2, –3)
b) (–1, –3) d) (2, 3)

07. (ESPCEX) Considere a circunferência (λ): x² + y² – 4x = 0 e o


3 (ITA) Considere a definição: duas circunferências são
ortogonais quando se interceptam em dois pontos distintos
e nesses pontos suas tangentes são perpendiculares. Com
ponto P(1, √3). Se a reta t é tangente a λ no ponto P, então a abscissa
7, C2 : x + y =
2 2
do ponto de intersecção de t com o eixo horizontal do sistema de relação às circunferências C1 : x 2 + (y + 4)2 = 9 e
coordenadas cartesianas é C3 : (x − 5) + y =
2 2
16, podemos afirmar que
a) – 2 c) 3 e) 3 + 3√3 a) somente C1 e C2 são ortogonais.
b) 2 + √3 d) 3 + √3 b) somente C1 e C2 são ortogonais.
c) C2 é ortogonal a C1 e a C3.
08. (ESPCEX) Sejam dados a circunferência λ: x² + y² + 4x + 10y + 25 = 0 d) C1, C2 e C3 são ortogonais duas a duas.
e o ponto P, que é simétrico de (–1,1) em relação ao eixo das abscissas.
e) não há ortogonalidade entre as circunferências.
Determine a equação da circunferência concêntrica à λ e que passa
pelo ponto P.
a) λ: x² + y² + 4x + 10y + 16 = 0
b) λ: x² + y² + 4x + 10y + 12 = 0
4 (ITA) No plano cartesiano são dadas as circunferências
1 e C2 : (x − 4)2 + y 2 =
C1 : x 2 + y 2 = 4. Determine o centro e
o raio de uma circunferência C tangente simultaneamente a C1
c) λ: x² – y² + 4x – 5y + 16 = 0 e C2, passando pelo ponto A = (3, 3).
d) λ: x² + y² – 4x – 5y + 12 = 0
e) λ: x² – y² – 4x – 10y – 17 = 0
5 (IME) Um triângulo ABC tem o seu vértice A na origem do
sistema cartesiano, seu baricentro é o ponto D(3, 2) e seu
circuncentro é o ponto E(55/18, 5/6). Determine:
09. (ITA 1993) Calculando-se a área da região limitada por
y ≤ 3.(x + 2)/2 e x2 + (y – 3)2 ≤ 13 obtém-se: - a equação da circunferência circunscrita ao triângulo ABC;
a) 2 13 π c) (13π)/2 e) 13 π - as coordenadas dos vértices B e C.
b) 13π d) (3 13 π)/2

533

PM_BOOK16 - MAT.indb 533 25/11/2022 19:26:21


GEOMETRIA ANALÍTICA: CIRCUNFERÊNCIA

GABARITO
EXERCÍCIOS DE FIXAÇÃO
01. D 04. A 07. C 10. A
02. B 05. A 08. D
03. D 06. B 09. C
EXERCÍCIOS DE TREINAMENTO
01. C 14. C 27. D 40. D
02. C 15. B 28. B 41. A
03. C 16. D 29. A 42. B
04. A 17. B 30. C 43. A
05. A 18. D 31. C 44. E
06. D 19. D 32. A 45. B
07. D 20. B 33. B 46. D
08. A 21. A 34. D 47. E
09. B 22. B 35. C 48. E
10. D 23. C 36. B 49. E
11. A 24. D 37. C 50. D
12. B 25. B 38. B
13. A 26. D 39. B
EXERCÍCIOS DE COMBATE
01. B 04. A 07. A 10. B
02. B 05. DISCURSIVA 08. B
03. A 06. C 09. C
DESAFIO PRO
01. A 04. 11 B(3,4) e C(6,2)
02. E 2 05. ou
03. C B(6,2) e C(3,4)

ANOTAÇÕES

534

PM_BOOK16 - MAT.indb 534 25/11/2022 19:26:21


GEOMETRIA ANALÍTICA:
ELIPSE

DEFINIÇÃO GERAL DAS CÔNICAS I. PONTOS PRINCIPAIS:


Cônica é o lugar geométrico dos pontos de um plano cuja razão A 2, A1, B2 e B1 – vértices
entre as distâncias a um ponto fixo F (foco) e a uma reta fixa d F2 e F1 – focos
(diretriz) é igual a uma constante não negativa e (excentricidade da C – centro
cônica). A excentricidade “e” é um número não negativo, pois é a
razão de duas distâncias.
II. SEGMENTO:
Se e = 1, a cônica é uma parábola.
A 2A1 – eixo maior – m(A 2A1) = 2a
Se 0 < e < 1, a cônica é uma elipse.
B2B1 – eixo menor – m(B2B1) = 2b
Se e > 1, a cônica é uma hipérbole.
F2F1 – distância focal – m(F2F1) = 2c

III. RELAÇÕES:
c
e  1 Excentricidade
a
a2  b2  c2 Relação notável tirada do triângulo retângulo B1CF1
b2
p= Parâmetro
a
“Parâmetro de uma cônica é a semicorda focal mínima.”
ELIPSE
b2
Dados dois pontos fixos F1 e F2 de um plano, tais que, p=
F2F1 = 2c ≠ 0, chamamos elipse ao lugar geométrico dos pontos a
deste plano, cuja soma das suas distâncias aos dois pontos F2 e F1 é
constante igual a 2a > 2c. IV. RETAS - DIRETRIZES:
Diretrizes da elipse são duas retas, (D1) e (d2), perpendiculares ao
a
suporte do eixo maior, distando do centro da curva.
e

EQUAÇÃO REDUZIDA
EIXO MAIOR NO EIXO X
 

| F2P |  | FP
1 | 2a

( x  c )a  y 2  ( x  c )2  y 2  2a 
x† 2cx  c† y†  2a  x† 2cx  c† y† 
x† 2cx  c²  y²  4a²  4a x²  2cx  c²  y² 
ELEMENTOS DA ELIPSE
a x²  2cx  c²  y²  a²  cx 
a²x²  2a²cx  a²c²  a²y²  a4  2a²cx  c²x² 
a²x²  c²x²  a²y²  a4  a²c²  (a²  c²)x²  a²y²  a²(a²  c²) 
x² y²
b²x²  a²y²  a²b²   1
a² b²
a
As equações das diretrizes são x  
e

ELIPSE “DE PÉ”


Se F1 (0,C) e F2 (0,–C), o eixo maior está sobre o eixo y.

x2 y2
 1
b2 a2

535

PM_BOOK16 - MAT.indb 535 25/11/2022 19:26:28


GEOMETRIA ANALÍTICA: ELIPSE

EQUAÇÃO DA ELIPSE QUANDO HÁ de r e ε, resolvido substituindo-se uma variável previamente isolada


na equação de r na equação de ε. Com isso, se:
TRANSLAÇÃO DE SISTEMA
I. ∆ > 0: r é secante a ε;
1° caso: elipse com centro O(m,n) e eixo maior horizontal.
II. ∆ = 0: r é tangente a ε;
( x  m)2 ( y  n)2
  1, a  b. III. ∆ < 0: r é exterior a ε.
a2 b2
y y'
EXERCÍCIOS DE

FIXAÇÃO
y P (x, y)
y'
V2 V1 x'
n •
o 01. No plano, com o sistema de coordenadas cartesianas usual, a
x'
equação x 2 + 4y 2 =
4x representa
a) uma circunferência. c) uma parábola.
b) duas retas. d) uma elipse.

o m x x
x2 y2
02. A área delimitada por uma elipse cuja equação é + = 1 é
a2 b2
2° caso: elipse com centro O(m,n) e eixo maior vertical. dada por A = ABπ. Então, a área da região situada entre as elipses de
( x - m)2 ( y - n)2 equações 16x 2 + 25y 2 =
400 e 16x 2 + 9y 2 =
144 é:
 1, a b
b2 a2 a) 12π u.a. 8π u.a. π u.a.
c) e)
y y'
b) 20π u.a. d) 256π u.a.

03. Existem pessoas que nascem com problemas de saúde relacionados


y P (x, y) ao consumo de leite de vaca. A pequena Laura, filha do Sr. Antônio,
nasceu com este problema. Para solucioná-lo, o Sr. Antônio adquiriu
y' uma cabra que pasta em um campo retangular medindo 20 m de
o' x' comprimento e 16 m de largura. Acontece que as cabras comem
n •
tudo o que aparece à sua frente, invadindo hortas, jardins e chácaras
vizinhas. O Sr. Antônio resolveu amarrar a cabra em uma corda presa
x' pelas extremidades nos pontos A e B que estão 12 m afastados um do
outro. A cabra tem uma argola na coleira por onde é passada a corda,
de tal modo que ela possa deslizar livremente por toda a extensão da
corda. Observe a figura e responda a questão a seguir.

o m x x

POSIÇÕES RELATIVAS
ENTRE PONTO E ELIPSE
Sejam ε uma elipse de focos F1 e F2, cujo eixo maior mede 2a, e P
um ponto do ponto do plano de ε.
1° caso: A é ponto da elipse.
A pertence à elipse ε se, e somente se PF1 + PF2 = 2a
2° caso: A é interior à elipse ε. PF1 + PF2< 2a
3° caso: A é exterior à elipse ε. PF1 + PF2> 2a

POSIÇÕES RELATIVAS
Qual deve ser o comprimento da corda para que a cabra possa pastar
ENTRE RETA E ELIPSE na maior área possível, dentro do campo retangular?
Sendo ε uma elipse e r uma reta, contidas em mesmo plano, a) 10 m. c) 20 m. e) 30 m.
temos:
b) 15 m. d) 25 m.
1º caso: a reta r é exterior à elipse se, e somente se, r ∩ ε = ∅
2º caso: a reta r é tangente à elipse se, e somente se, r ∩ ε = {P} 04. Em uma praça dispõe-se de uma região retangular de 20 m
3º caso: a reta r é secante à elipse se, e somente se, r ∩ ε = {P1 , P2} de comprimento por 16 m de largura para construir um jardim. A
exemplo de outros canteiros, este deverá ter a forma elíptica e estar
PROCESSO PRÁTICO: inscrito nessa região retangular. Para aguá-lo, serão colocados dois
aspersores nos pontos que correspondem aos focos da elipse. Qual
A quantidade de pontos de interseção entre a reta r e a elipse ∩ ε
será a distância entre os aspersores?
é dada pelo número de soluções do sistema formado pelas equações

536

PM_BOOK16 - MAT.indb 536 25/11/2022 19:26:30


GEOMETRIA ANALÍTICA: ELIPSE

a) 4 m c) 8m e) 12 m 08. A área sombreada na figura, limitada pela elipse e pela reta


b) 6 m d) 10 m indicadas, é:

05. Suponha que um planeta P descreva uma órbita elíptica em


torno de uma estrela O, de modo que, considerando um sistema de
coordenadas cartesianas ortogonais, sendo a estrela O a origem do
sistema, a órbita possa ser descrita aproximadamente pela equação
 x2   y2 
 + = 1, com x e y em milhões de quilômetros.
 100   25 
A figura representa a estrela O, a órbita descrita pelo planeta e sua
ˆ π
posição no instante em que o ângulo POA mede .
4 a) π. c) 3π. e) 6π.
b) 2π. d) 4π.

09. (EFOMM) A equação (x 2 144) + (y 2 225) =


1 representa uma
a) elipse com focos em (0,9) e (0,–9).
b) circunferência de raio igual 9.
c) parábola.
d) hipérbole.
e) elipse com centro em (12,15).
A distância, em milhões de km, do planeta P à estrela O, no instante
representado na figura, é: 10. (MACKENZIE) A reta de menor coeficiente angular, que passa por
a) 2 5. c) 5 2. e) 5 10. um dos focos da elipse 5x2 + 4y2 = 20 e pelo centro da circunferência
x2 + y2 – 4x – 6y = 3, tem equação:
b) 2 10. d) 10 2.
a) 3x – y – 3 = 0 d) x – 2y – 4 = 0
b) 2x – y – 1 = 0 e) x – y + 1 = 0
06. Um arquiteto projetou, para um salão de dimensões 22 m por 18
m, um teto de gesso em formato de elipse com o eixo maior medindo c) x – 3y – 7 = 0
20 m e o eixo menor, 16 m, conforme ilustra a figura abaixo.

EXERCÍCIOS DE

TREINAMENTO
01. Determine a equação da elipse em que:
a) os focos são F1 (–2,0) e F2 (2,0) e o comprimento do eixo maior
é 6;
b) os vértices são A1 (0,–6), A2 (0,6), B1 (3,0) e B2 (–3,0).
O aplicador do gesso afirmou que saberia desenhar a elipse, desde
que o arquiteto informasse as posições dos focos. 02. A elipse representada na figura tem equação:
Para orientar o aplicador do gesso, o arquiteto informou que, na 2
direção do eixo maior, a distância entre cada foco e a parede mais
próxima é de
a) 3 m. b) 4 m. c) 5 m. d) 6 m. –3 3

07. A figura representa uma elipse.


–2
A partir dos dados disponíveis,
a equação desta elipse é
 x2   y2  2 2
a)   +   = 1. a) x2 + y =1 d) x2 + y =1
5
  7 4 3 36 9
 ( x + 5 )2   ( y − 7 )2  2
x2 + y = 2
b)   +   = 1. b) 1 e) x2 + y =1
 9   16  2 1 9 36
2
c) (x – 5)2 + (y – 7)2 = 1. c) x2 + y =1
9 4
 ( x − 5) 
2
 ( y + 7 )2 
d)   +   = 1.
 9   16  2 y2
03. Determine os focos da elipse x + 1.
=
 ( x + 3) 
2
(y − 4)  2 4 3
e)   +   = 1.
 5   7 

537

PM_BOOK16 - MAT.indb 537 25/11/2022 19:26:32


GEOMETRIA ANALÍTICA: ELIPSE

2 y2
04. A excentricidade da elipse x + =
1 é: 13. A área do triângulo PF1F2, onde P(2,–8) e F1 e F2 são os focos da
7 16 elipse de equação x2/25 + y2/9 = 1, é igual a:
7 3 7 7
a) c) e) a) 8 c) 20 e) 64
3 7 16
b) 16 d) 32
b) 3 4
d)
4 3 14. O gráfico que melhor representa a curva de equação
x2 + 16y2 = 16 é:
05. O eixo maior da elipse 5x2 + 2y2 = 20 mede:
a) d)
a) 2 c) 4 e) 10
b) 2 10 d) 10

06. A equação da circunferência com centro na origem e raio igual ao


semieixo menor da elipse x2 + 4y2 = 4 é:
a) x2 + y2 = 2 d) x2 + y2 = 1 e)
b)
b) x + y = 16
2 2
e) x + y = 2
2 2

c) x2 + y2 = 4

07. Uma elipse está centrada na origem, tem os seus eixos sobre os eixos
coordenados e é tangente simultaneamente a x2 + y2 = 4 e x2 + y2 = 9.
Na determinação desta elipse verifica-se que:
2
c)
x2 + y =1.
a) a solução é
36 16
b) não há solução.
c) a solução é 4x2 + 9y2 = 36.
d) a solução é (x – 3)2 + (y – 2)2 = 1.
e) há mais de uma solução.
15. (ITA) Os focos de uma elipse são F1(0, – 6) e F2(0, 6). Os pontos
A(0, 9) e B(x, 3), x > 0, estão na elipse. A área do triângulo com
08. Determine a equação da elipse em que:
vértices em B, F1 e F2 é igual a
a) os focos são F1 (0,–3) e F1 (0,3) e o comprimento do eixo maior é 8;
b) os focos são F1 (1,0) e F2 (–1,0) e dois vértices são A1 (2,0), A2 a) 22 10 c) 15 10 e) 6 10
(–2,0).
b) 18 10 d) 12 10

09. As coordenadas dos focos da elipse de equação 9x + 25y = 225 2 2


16. Sendo m o maior valor real que x pode assumir na equação
são:
analítica (x – 2)² + 4(y + 5)² = 36, e n o maior valor real que y pode
1   1 
a)  , 0  e  – , 0  assumir nessa mesma equação, então, m + n é igual a
2   2 
a) 8. c) 6. e) 3.
b) (2,0) e (–2,0)
b) 7. d) 4.
c) (0,4) e (0, –4)
d) (4,0) e (–4,0) 17. O número de pontos de interseção das curvas x2 + y2 = 4 e
e) (0,2) e (0,–2) (x2/15) + (y2/2) = 1 é igual a:
a) 0 d) 5
10. A elipse x2 + (y2/2) = 9/4 e a reta y = 2x + 1, do plano cartesiano, b) 3 e) 6
se interceptam nos pontos A e B. Pode-se, pois, afirmar que o ponto
médio do segmento AB é: c) 4

a) (–2/3, –1/3) c) (1/3, –5/3) e) (–1/4,1/2) x2 y2


+
18. (ITA) O coeficiente angular da reta tangente à elipse = 1
b) (2/3, –7/3) d) (–1/3, 1/3) 16 9
no primeiro quadrante e que corta o eixo das abscissas no ponto
11. (ITA) A distância focal e a excentricidade da elipse com centro na P = (8,0) é
origem e que passa pelos pontos (1,0) e (0,–2) são, respectivamente, 3 3
a) − d) −
1 3 4
a) 3 e . d) 3 e 3.
2 2 1
b) − e) 2
1 2 −
b) e 3. 3 4
2 e) 2 3 e .
2 c) 2

3 1 3
c) e .
2 2
19. A equação 9x2 + 4y2 – 18x – 27 = 0 representa, no plano cartesiano,
12. (UECE) A área do quadrilátero cujos vértices são as interseções da uma curva fechada. A área do retângulo circunscrito a essa curva, em
elipse 9x2 + 25y2 = 225 com os eixos coordenados é igual, em unidades unidades apropriadas, vale:
de área, a: a) 36 c) 18 e) 12
a) 30 b) 32 c) 34 d) 36 b) 24 d) 16

538

PM_BOOK16 - MAT.indb 538 25/11/2022 19:26:34


GEOMETRIA ANALÍTICA: ELIPSE

20. (ITA) Tangenciando externamente a elipse α1, tal que α1: 9x2 + 4y2 27. (ESPCEX) Uma elipse tem centro na origem e vértices em (2a,0) e
– 72x – 24y + 144 = 0, considere uma elipse α2, de eixo maior sobre (0,a), com a > 0. A área do quadrado inscrito nessa elipse é
a reta que suporta o eixo menor de α1 e cujos eixos têm a mesma
16a2 d) 8a2
medida que os eixos de α1. Sabendo que α2 está inteiramente contida a) . .
no primeiro quadrante, o centro de α2 é: 5 5
a) (7, 3) b) (8, 2) c) (8, 3) d) (9, 3) e) (9, 2) 4a2 e) 20a2
b) . .
5 5
21. (ITA) Considere a circunferência C de equação x2 + y2 + 2x + 2y + 1 = 0 12a2
e a elipse E de equação x2 + 4y2 – 4x + 8y + 4 = 0. Então: c) .
5
a) C e E interceptam-se em dois pontos distintos.
b) C e E interceptam-se em quatro pontos distintos. 28. (ESPCEX) Os valores reais de n para os quais a reta (t) y = x + n
c) C e E são tangentes exteriormente. seja tangente à elipse de equação 2x 2 + 3y 2 =
6 são iguais a
d) C e E são tangentes interiormente. a) − 5 e 5 d) –2 e 2
e) C e E têm o mesmo centro e não se interceptam.
b) − 3 e 3 e) –5 e 5
22. (UECE) No plano, com o sistema de coordenadas cartesiano c) –3 e 3
usual, a área do quadrilátero convexo cujos vértices são os pontos de
29. (AFA) Sobre a circunferência de menor raio possível que
interseção das elipses representadas pelas equações x 2 + 2y 2 = 2 e
circunscreve a elipse de equação x 2 + 9y 2 − 8x − 54y + 88 =
0 é
2x + y =
2
2 é 2
correto afirmar que
u. a. ≡ unidade de área a) tem raio igual a 1.

9 8 7 5 b) tangencia o eixo das abscissas.


a) u. a. b) u. a. c) u. a. d) u. a.
2 3 3 3 c) é secante ao eixo das ordenadas.
d) intercepta a reta de equação 4x – y = 0.
23. (AFA) No plano cartesiano, os pontos P(x,y) satisfazem a equação
(x − 1)2 (y + 2)2 x2 y2
+ =1 da curva λ. 30. (AFA) No plano cartesiano, os focos F1 e F2 da elipse α : + =
1
25 9 36 32
Se F1 e F2 são os focos de λ, tais que a abscissa de F1 é menor que a são pontos diametralmente opostos da circunferência λ e coincidem
abscissa de F2, é INCORRETO afirmar que com as extremidades do eixo real de uma hipérbole equilátera β.
a) a soma das distâncias de P a F1 e de P a F2 é igual a 10. É INCORRETO afirmar que
b) F1 coincide com o centro da curva x² + y2 + 6x – 4y = 0. a) α ∩β∩λ = ∅ .
c) F2 é exterior a x² + y² = 25. b) λ ∩ β = {F1, F2 } .
d) o ponto de abscissa máxima de λ pertence à reta y = x – 8. c) α ∩ β = {A, B, C, D}, sendo A, B, C, D pontos distintos.
d) α∩λ ≠ ∅.
24. (MACKENZIE) Dadas as cônicas de equações
( I ) x 2 + y 2 − 2x + 8y + 8 =0 e ( II ) 4x 2 + y 2 − 8x + 8y + 16 =
0,
assinale a alternativa INCORRETA.
EXERCÍCIOS DE

COMBATE
a) Os gráficos de (I) e (II) são, respectivamente, uma circunferência
e uma elipse.
b) As duas cônicas têm centro no mesmo ponto.
c) As duas cônicas se interceptam em dois pontos distintos.
d) O gráfico da equação (I) é uma circunferência de raio 3. 01. (AFA 1999) A equação reduzida da cônica, representada no
e) O gráfico da equação (II) é uma elipse com centro C = (1,–4). gráfico abaixo, é
y
9
2 2
x y
25. (EN) Seja P(x, y) um ponto da elipse 2 + 2 = 1, de focos F1 e F2 e
  a b
excentricidade e. Calcule PF1 ⋅ PF2 e assinale a opção correta.
a) ex 2 + a(1 + 2e2 ) d) e2x − a(1 + e2 )
b) e2x − a2 (1 + e) e) e2x 2 + a2 (1 − 2e2 )
c) e x + a (1 − 2e)
2 2 2

26. (MACKENZIE) Com relação às equações das elipses 25x2 + 16y2 + 1


150x + 256y – 351 = 0 e 16x2 + 25y2 – 96x – 200y + 144 = 0, podemos
afirmar que -1 2 x
a) as elipses têm centros coincidentes.
( x  4 )2 ( y  3)2 ( x  1)2 ( y  5)2
b) as elipses têm a mesma distância focal. a)  1 c)  1
9 16 16 9
c) as elipses têm a mesma excentricidade.
( x  5)2 ( y  1)2 ( x  1)2 ( y  5)2
d) as elipses têm focos sobre o eixo das abscissas. b)  1 d)  1
9 16 9 16
e) o eixo maior de uma delas é o dobro do eixo menor da outra.

539

PM_BOOK16 - MAT.indb 539 25/11/2022 19:26:38


GEOMETRIA ANALÍTICA: ELIPSE

02. (AFA 2003) A área do triângulo cujos vértices são os focos da 08. (AFA 2004) Sobre o triângulo PF1F2 onde P(2,2) e F1F2 são da elipse
x2 y2 x² y²
 1 e centro na circunferência x   y  3  1 é:
2 2
elipse  + =
1, é correto afirmar que:
13 4 9 25
a) 9/2 d) 7/2
a) é isósceles.
b) 3 e) 18
b) é obtusângulo.
c) 9
c) tem área igual a 16.
03. (EN 2009) Seja P o ponto de interseção entre as retas r e s de d) tem perímetro igual a 2√2 + 8.
equações 3x – 2y + 4 = 0 e – 4x + 3y – 7 = 0, respectivamente. Seja Q o
centro da circunferência de equação x² + y² + 24 = 6x + 8y. A medida 09. (ESPCEX) Num estádio de futebol em forma de elipse, o gramado
do segmento PQ é igual à quarta parte do comprimento do eixo maior é o retângulo MNPQ, inscrito na cônica, conforme mostra a figura.
da elipse de equação: Escolhendo o sistema de coordenadas cartesianas indicado e tomando
x² y²
a) 2x² + y² – 8x – 2x + 7 = 0 o metro como unidade, a elipse é descrita pela equação + =1.
36² 60²
b) 2x² + y² – 4x – 2y – 1 = 0 Sabe-se também que os focos da elipse estão situados em lados do
c) x² + 4y² – 4x – 24y + 36 = 0 retângulo MNPQ.
d) x² + 2y² – 2x – 8y + 1 = 0
e) x² + 2y² – 4x + 8y + 8 = 0

04. (EN 2012) Um dos focos da elipse 9x² + 4y² = 36 é o ponto:


a) 0, 2 
b)  13, 0
c) 0, 13 
d)  5, 0
e) 0, 5 
05. (AFA 2013) Sobre a circunferência de menor raio possível que Assim, a distância entre as retas MN e PQ é
circunscreve a elipse de equação x² + 9y² – 8x – 54y + 88 = 0 é correto
afirmar que: a) 48 m

a) Tem raio igual a 1. b) 68 m

b) Tangencia o eixo das abscissas. c) 84 m

c) É secante ao eixo das ordenadas. d) 92 m

d) Intercepta a reta de equação 4x – y = 0. e) 96 m

06. (UFT 2010) Considere  o conjunto dos números reais e b ∈ . 10. (ESPCEX) Sobre a curva 9x² + 25y² – 36x + 50y – 164 = 0, assinale
Encontre os valores de b, tais que no plano cartesiano xy, a reta a alternativa correta.
x² a) Seu centro é (–2,1).
y = x + b intercepta a elipse + y² =
1 em um único ponto. A soma
dos valores de b é: 4 b) A medida do seu eixo maior é 25.
a) 0 d) √5 c) A medida do seu eixo menor é 9.
b) 2 e) –2√5 d) A distância focal é 4.
c) 2√5 e) Sua excentricidade é 0,8.

x² y²
07. (AFA) Na figura abaixo, F1 e F2 são focos da elipse + = 1
25 9

DESAFIO PRO
O ponto C, de coordenadas  0, 3  , pertence ao segmento MN.
 2
Os segmentos AC, CB e MN são, respectivamente, paralelos aos
segmentos F1 P, PF2 e FF
1 2.
A área da figura sombreada, em unidades
de área, é
y
P
1 (IME) Seja M um ponto de uma elipse com centro O e focos F e
F’. A reta r é tangente à elipse no ponto M e s é uma reta, que
passa por O, paralela a r. As retas suportes dos raios vetores MF e
a) 3
MF’ interceptam a reta s em H e H’, respectivamente. Sabendo que
b) 6
M C N o segmento FH mede 2 cm, o comprimento F’H’ é:
c) 9 a) 0,5 cm
d) 12 b) 1,0 cm
F1 A B F2 x c) 1,5 cm
d) 2,0 cm
e) 3,0 cm

540

PM_BOOK16 - MAT.indb 540 25/11/2022 19:26:43


GEOMETRIA ANALÍTICA: ELIPSE

2 (IME) Os triângulos ABC e DEF são equiláteros com lados


iguais a m. A área da figura FHCG é igual à metade da área
da figura ABHFG. Determine a equação da elipse de centro na
5 (IME) Seja uma elipse com focos no eixo OX e centrada
na origem. Seus eixos medem 10 e 20/3. Considere uma
hipérbole tal que os focos da elipse são os vértices da hipérbole
origem e eixos formados pelos segmentos FC e GH. e os focos da hipérbole são os vértices da elipse. As parábolas
que passam pelas interseções entre a elipse e a hipérbole e que
são tangentes ao eixo OY, na origem, têm as seguintes equações:
35
a) y 2 = ±2 x
7
5
b) y 2 = ±4 x
7
5
c) y 2 = ±6 x
7
35
d) y 2 = ±6 x
7
35
a) 48x 2 + 36y 2 – 2m2 =
0 e) y 2 = ±8 x
63
2 2 2
b) 8x + 16y – 3m =0

c) 16x 2 + 48y 2 – 3m2 =


0
2 2 2
d) 8x + 24y – m =0 GABARITO
2 2 2
e) 16x – 24y – m = 0 EXERCÍCIOS DE FIXAÇÃO
01. D 04. E 07. B 10. E

3 (IME) Considere uma haste AB de comprimento 10 m. Seja 02. C 05. B 08. C


um ponto P localizado nesta haste a 7 m da extremidade 03. C x² y² 09. A
06. C
A. A posição inicial desta haste é horizontal sobre o semieixo x a) + =1
EXERCÍCIOS DE TREINAMENTO
7 16
positivo, com a extremidade A localizada na origem do plano
cartesiano. A haste se desloca de forma que a extremidade x 2
y 2 x² y² 19. B
01. a) + =
1 b) + = 1
A percorra o eixo y, no sentido positivo, e a extremidade B 9 5 4 3 20. D
percorra o eixo x, no sentido negativo, até que a extremidade B 2 2 09. (–4,0) e (4,0) 21. C
esteja sobre a origem do plano cartesiano. A equação do lugar b) y + x =1
36 9 10. D 22. B
geométrico, no primeiro quadrante, traçado pelo ponto P ao
ocorrer o deslocamento descrito é 02. C 11. E 23. B
a) 2 2
49x + 9y – 280x + 120y – 441 =
0
03. (1,0)/(–1,0) 12. A 24. C
04. B 13. D 25. E
b) 49x 2 – 406x – 49y 2 + 441 =
0
05. B 14. C 26. C
c) 9x 2 + 49y 2 – 441 =
0
06. D 15. D 27. A
d) 9x 2 + 9y 2 + 120y – 441 =
0 07. 36 16. C 28. A
2 2
e) 9x – 49y – 441 = 0 x² y² 17. C 29. B
08. a) + =
1
7 16 18. D 30. D

4 (IME) Uma elipse cujo centro encontra-se na origem e x² y²


b) + = 1
cujos eixos são paralelos ao sistema de eixos cartesianos EXERCÍCIOS
4 3DE COMBATE
possui comprimento da semi-distância focal igual a 3 e 01. D 04. E 07. B 10. E
3 02. C 05. B 08. B
excentricidade igual a . Considere que os pontos A, B, C e D
2 03. D 06. A 09. E
representam as interseções da elipse com as retas de equações DESAFIO PRO
y = x e y = –x. A área do quadrilátero ABCD é
01. D 03. C 05. E
a) 8
02. D 04. D
b) 16
16
c) ANOTAÇÕES
3
16
d)
5
16
e)
7

541

PM_BOOK16 - MAT.indb 541 25/11/2022 19:26:44


GEOMETRIA ANALÍTICA: ELIPSE

ANOTAÇÕES

542

PM_BOOK16 - MAT.indb 542 25/11/2022 19:26:45


GEOMETRIA ANALÍTICA:
HIPÉRBOLE

DEFINIÇÃO Fazendo c² – a² = b² na equação anterior, tem-se finalmente:


A hipérbole é o lugar geométrico dos pontos de um plano b²x² – a²y² = a²b²
cuja diferença das distâncias a dois pontos fixos do mesmo plano é Ou
constante. x2 y2
 1 (2)
Podemos definir da seguinte maneira: sejam 2 pontos fixos F1 e F2 a2 b2
de um plano, com F2F1 = 2c ≠ 0, hipérbole é o lugar geométrico dos
pontos deste plano, cujo módulo da diferença de suas distâncias aos ELEMENTOS DA HIPÉRBOLE
dois pontos F2 e F1 é constante igual a 2a, com 2c > 2a.

EQUAÇÃO
Sejam F e F’ os dois pontos fixos, denominados focos. FF ’ = 2c
é a distância focal.

Tomemos para eixo x a reta que passa por F e F’ e para eixo y a


mediatriz de FF’ do que resulta: F(–c,o) e F’(c,o).
Chamando P(x, y) o ponto genérico da hipérbole, temos, pela
definição:
A1 e A2- vértices
FP  F ’P  2a (1)
F2 e F1 - focos
Por outro lado,
C - centro
FP   x  c 2  y 2
Eixo real: A2A1 = 2a
e
Eixo imaginário: B2B1 = 2b
F ’P   x  c 2  y 2 Distância focal: F2F1 = 2c
 
Substituindo na igualdade (1) vem: Raios vetores: F2P, F1P

 x  c 2  y 2   x  c 2  y 2  2a Relações:
c
Transpondo o segundo radical para o segundo membro, elevando e  1 Excentricidade
a
quadrado e simplificando, obtém-se:
c² = a² + b² Relação notável
a.  x  c 2  y 2  a2  cx Reta: diretrizes são duas retas, (d1) e (d2), perpendiculares ao
a
Elevando novamente ao quadrado e reduzindo os termos suporte do eixo real, distando do centro da hipérbole.
semelhantes vem: e
Assíntotas são duas retas, (a1) e (a2), que passam pelo centro da
c 2
 
 a2 x 2  a2y 2  a2 c2  a2  hipérbole em posições limites das tangentes a ela, quando os pontos
de contato se afastam indefinidamente.
Como c > a, c² – a² é um número positivo.

543

PM_BOOK16 - MAT.indb 543 25/11/2022 19:26:52


GEOMETRIA ANALÍTICA: HIPÉRBOLE

EQUAÇÕES REDUZIDAS Sua equação é:


Seja a hipérbole de eixos real A2A1 e imaginário B2B1 com centro ( x  m)2 ( y  n)2
na origem. Considere P(x,y) um ponto genérico da curva.  1
a2 b2

As equações das diretrizes são


a
x  m
e
e das assíntotas
b
y  m x
a

Quando A2 A1 // Oy

x2 y2
 1
a2 b2

Para y = 0, temos: x = ±a, abscissas dos vértices A1 e A2.


Para x = 0, temos: y = ±bi, o que significa que a curva não é
interceptada pelo eixo dos y.
( x  m)2 ( y  n)2
a  1
As equações das diretrizes (d1) e (d2) são x   . b2 a2
e
a
b As equações das diretrizes assumem a forma y  n  e as das
 y
As equações das assíntotas, y = tg α · x ⇒ x . e
a a
assíntotas y  n  x .
A equação da hipérbole de centro na origem, focos no eixo OY e b
semieixos real e imaginário iguais a a e b é dada por: Equação geral: a equação geral é obtida pelo desenvolvimento
2 2
das formas reduzidas.
x y
 1
b2 a2
HIPÉRBOLE EQUILÁTERA
As diretrizes são, agora, paralelas ao eixo Ox e suas equações são: Uma hipérbole cujos semieixos são iguais (a = b) é chamada de
hipérbole equilátera.
a As suas equações se simplificam com a substituição de b por a.
y
e
a
e as assíntotas y   x . POSIÇÕES RELATIVAS ENTRE
b
PONTO E HIPÉRBOLE
Para y = 0 ⇒ x = ±bi
Uma hipérbole H e um ponto P, coplanares, têm três posições
a curva não intercepta o eixo dos x e para x = 0 ⇒ y = ±a, ordenadas relativas possíveis, onde 2ª é a medida do eixo real da hipérbole, e F1 e
dos vértices A1 e A2 . F2 são os focos da hipérbole.
Hipérbole com centro no ponto C (m,n) e A2A1 // Ox. 1o caso: P é um ponto da hipérbole.
P ∈ H ⇔ |PF1 – PF2| = 2a
2o caso: P é ponto interior à hipérbole.
P é interior à H ⇔ |PF1 – PF2| > 2a
3o caso: P é ponto exterior à hipérbole.
P é exterior à hipérbole H ⇔ |PF1 – PF2| < 2a

POSIÇÕES RELATIVAS ENTRE


RETA E HIPÉRBOLE
Processo prático: seja S o sistema formado pelas equações de r e
H. Se, ao substituirmos uma das variáveis da equação de r na equação
de H, obtivermos:

544

PM_BOOK16 - MAT.indb 544 25/11/2022 19:27:00


GEOMETRIA ANALÍTICA: HIPÉRBOLE

I. uma equação do 2o grau com ∆ > 0. r e H são secantes; d)


II. uma equação do 2o grau com ∆ < 0. r e H são exteriores;
III. uma equação do 2o grau com ∆ = 0. r e H são tangentes.

ProBizu
e)
RECONHECIMENTO DE UMA CÔNICA
Dada uma equação do 2o grau redutível à forma
 x  x0   y  y0 
2 2

 1
k1 k2
04. Reescrevas equações das cônicas abaixo em sua forma reduzida,
k1 > 0, k2 > 0 e k1 > k2 elipse de eixo maior horizontal
identificando-as, e determine suas excentricidades.
k1 > 0, k2 > 0 e k1 < k2 elipse de eixo maior vertical a) x2 + 2y2 – 2x + 4y + 1 = 0
b) 9x2 – 16y2 – 36x – 108 = 0
k1 > 0 e k2 < 0 hipérbole de eixo real horizontal

k1 < 0 e k2 > 0 hipérbole de eixo real vertical 05. Encontre a equação geral e a reduzida das hipérboles abaixo.
a) b)

EXERCÍCIOS DE

FIXAÇÃO
01. Observando o gráfico da hipérbole H de focos e F2, e determine:

06. Esboce o gráfico da hipérbole H em cada um dos seguintes casos


( x − 3) ( y − 1)
2 2
x2
( y + 1)
2
a) − =
1 b) − =
1
16 9 8
07. O eixo transverso de uma hipérbole, de excentricidade 1,5, é
paralelo ao eixo das ordenadas e mede 8. O centro dessa hipérbole é
a) a medida do eixo real o ponto (0,–2).
b) a distância focal a) Obter a medida do eixo conjugado.
c) a medida do eixo imaginário b) Obtenha as equações das assíntotas dessa hipérbole.
d) a excentricidade
08. Determine a equação reduzida da hipérbole que possui eixo real
02. Encontre a equação geral e a reduzida das hipérboles abaixo. 6 e tem focos (–5,0) e (5,0).
a)
09. Determine os focos da hipérbole 4y2 – 9x2 = 36.
2
x2 − y = 1 . Determine a
10. Considere a hipérbole H de equação
7 16
equação da hipérbole cujo eixo real coincide com o eixo imaginário de
b) H e cujo eixo imaginário coincide com o eixo real de H.

EXERCÍCIOS DE

TREINAMENTO
01. Considere o círculo x2 + y2 – r2 = 0 de raio r e a hipérbole x2 – y2 = 1.
03. O produto de duas variáveis reais, x e y, e uma constante. Nesse caso, pode-se afirmar que:
Portanto, dentre os gráficos abaixo, o único que pode representar a) Se r < 1, então as curvas se intersectam em quatro pontos.
essa relação é:
b) Se r = 1, então as curvas tem quatro pontos em comum.
a) b) c) c) Se r = 1, as curvas se intersectam em (0,1) e (0,–1)
d) Se r = 17 , então as curvas se intersectam apenas nos pontos
(3, 2 2 ) e (–3, –2 2 )
e) Se r > 17 , então as curvas se intersectam em quatro pontos.

545

PM_BOOK16 - MAT.indb 545 25/11/2022 19:27:01


GEOMETRIA ANALÍTICA: HIPÉRBOLE

02. O gráfico da curva de equação (x2/4) – (y2/9) = 1 é uma:


a) circunferência. c) hipérbole.
b) elipse. d) parábola.

03. No plano cartesiano, x2 – y2 + 5x – 5y = 0 é uma equação de:


a) um conjunto vazio. d) duas retas paralelas.
b) um conjunto unitário. e) duas retas concorrentes.
c) uma hipérbole.

04. O gráfico da equação x2 – y2 = 4 representa uma hipérbole. Os


focos dessa hipérbole são:
1  e  1  d) (0, 2 ) e (0,– 2 )
a)  ,0   − ,0 
2   2 
 1 e  1
b) (2,0) e (–2,0) e)  0,   0, − 
 2  2
c) (2 2 ,0) e (–2 2 ,0) a) Determine as coordenadas (x, y) do ponto A.
b) Calcule a área da região indicada em amarelo no gráfico.
05. Analise as afirmações dadas a seguir, classifique-as como
verdadeiras (V) ou falsas (F). 10. Baseado em conhecimentos sobre cônicas, assinale o que for
( ) A equação x2 – 2x + y2 + 2y + 1 = 0 representa uma circunferência correto.
que é tangente, tanto ao eixo das abscissas quanto ao eixo das 01) Elipse é o lugar geométrico dos pontos equidistantes de dois
ordenadas. pontos distintos fixos chamados focos.
( ) A elipse de equação 9x2 + 4y2 = 36 intercepta a hipérbole de 02) A equação 4x 2 − 9y 2 − 25 =
0 determina uma hipérbole de focos
equação x2 – 4y2 = 4 em apenas dois pontos, que são os vértices no eixo x.
da hipérbole.
04) Seja r uma reta e P um ponto fora dela, ambos no mesmo plano.
( ) O semieixo maior da elipse 9x2 + 4y2 = 36 é paralelo ao eixo real O lugar geométrico dos pontos equidistantes a r e a P será uma
da hipérbole x2 – 4y2 = 4. parábola.
Assinale a alternativa que contém a sequência correta, de cima para baixo.
08) A elipse de focos (–1,0) e (1,0), com seu eixo maior de extremidades
a) V - V - V c) F-V-F e) V - F - F
x2 y2
b) V - V - F d) F - F - V em (–3,0) e (3,0), tem equação + = 1.
9 8
06. As equações x2 – 9y2 – 6x – 18y – 9 = 0, x2 + y2 – 2x + 4y + 1 = 0 x2 y2
16) O eixo maior da elipse + =
1 tem extremidades (7,0) e
e x2 – 4x – 4y + 8 = 0 representam, respectivamente, uma: 49 36
(–7,0).
a) hipérbole, uma elipse e uma parábola.
b) hipérbole, uma circunferência e uma reta. 11. Em relação às proposições abaixo, é correto afirmar que:
c) hipérbole, uma circunferência e uma parábola. 01) A catedral de Brasília foi projetada pelo arquiteto Oscar Niemeyer.
d) elipse, uma circunferência e uma parábola. Sua estrutura se destaca pela beleza e pela forma, um hiperboloide
de rotação. A figura abaixo destaca os principais elementos da
e) elipse, uma circunferência e uma reta.
hipérbole associada à forma da catedral e é possível perceber que
ela tem como base um círculo de diâmetro d. Supondo que a
07. (EN) A equação 4x − y − 32x + 8y + 52 =
2 2
0, no plano xy, x2 y2
representa: equação dessa hipérbole seja − =1 e que a medida do
225 400
a) duas retas diâmetro tenha 10 metros a mais que a distância focal, então a
b) uma circunferência medida d será igual a 60 metros.
c) uma elipse
d) uma hipérbole
e) uma parábola

08. (ESPCEX) Uma hipérbole tem focos F1( −5, 0) e F2 (5, 0) e passa
pelos pontos P(3, 0) e Q(4, y), com y > 0. O triângulo com vértices
em F1, P e Q tem área igual a
16 7 32 7 8 7
a) . c) . e) .
3 3 5
16 7 8 7
b) . d) .
5 3

09. O gráfico representa uma hipérbole, dada pela função real


3
f(x)= x + . Sabe-se que ABCD é um retângulo, que EC é
2− x
diagonal do retângulo EBCF e que a área da região indicada em rosa
é igual a 4,7 cm².

546

PM_BOOK16 - MAT.indb 546 25/11/2022 19:27:03


GEOMETRIA ANALÍTICA: HIPÉRBOLE

x2 y2 1 15. Para cada número real a, analise as proposições a seguir, referentes


02) A excentricidade da elipse de equação + =
1é .
25 4 3
à representação geométrica da equação x + ay + 2x − 2ay =
2 2
0 em
 k 1
04) O valor de k na matriz A =  −1
 para que se tenha A = A
t
um sistema de coordenadas cartesianas xOy.
 −1 k 
é k = 0. ( ) Se a = 1, a equação representa uma circunferência.
1 3 2  3 4 1  ( ) Se a = 0, a equação representa uma reta.
 , então o det(A⋅ B ) não
t
08) Se A =   e B=
 0 5 −1  0 −2 6  ( ) Se a = 3, a equação representa uma hipérbole.
existe. ( ) Se a = –2, a equação representa uma elipse.
16) Se em uma loja de moda masculina Júlio comprar um par de ( ) Se a = –1, a equação representa a união de duas retas.
sapatos, duas calças e três camisas, ele pagará R$ 520,00. Se
comprar, na mesma loja, um par de sapatos, três calças e cinco Assinalando V para afirmativas verdadeiras e F para afirmativas falsas
camisas, pagará R$ 760,00. Logo, na compra de um par de a sequência correta é
sapatos, de uma calça e de uma camisa, nessa mesma loja, Júlio a) V – F – F – F – F. d) V – F – V – F – V.
pagará R$ 280,00. b) F – F – F – F – V. e) V – F – F – F – V.
1 c) V – V – F – F – V.
12. Considere a hipérbole de equação y = mostrada na figura
abaixo: x
16. Determine as coordenadas do centro e dos vértices da hipérbole
x2 – 4y2 + 6x – 8y + 21 = 0, verificando a direção do eixo real e
determinando as equações das diretrizes e assíntotas.

17. Determine a equação da hipérbole equilátera que passa pelo


ponto P0(13,12) e que tem por eixos de simetria os eixos coordenados,
as coordenadas dos focos e dos vértices.

18. Caracterize a cônica representada pela equação 4x² – 9y² = 36 .

19. Quais são os focos da cônica cuja equação é x² – y² = 1?

20. Determine a equação da reta tangente à hipérbole


x2 – 3y2 – 2x + 36y – 116 = 0 no seu ponto T(7,9).

21. Os eixos, real e imaginário, de uma hipérbole de eixo real


a) Determine os pontos de interseção entre a hipérbole e a reta de horizontal têm, respectivamente, os comprimentos 8 e 6. Determine
equação y − 2 = x + 2. a equação desta hipérbole e da sua conjugada, sendo seu centro o
b) Determine os pontos de interseção entre a hipérbole e a reta de ponto C(1,–3).
equação y − 2 =−x − 2.
c) Para quais valores do parâmetro real m a reta de equação 22. Um ponto P(x,y) se move de tal modo que sua distância ao ponto
P0(3,2) mantém-se sempre igual ao triplo de sua distância à reta (r)
y − 2= m ( x + 2) intersecta a hipérbole em exatamente um
y = –1. Determine a equação do lugar geométrico gerado por P e o
ponto?
caracterize.

13. (ITA) Dada a cônica ë: x2 – y2 = 1, qual das retas abaixo é


23. Determine a equação da hipérbole, nos seguintes casos:
perpendicular à ë no ponto P = (2, 3 )?
a) de focos F(0,±5) e vértices A(0,±3);
a) y = 3x–1 3
d) y = – x −7 b) que passa pelo ponto (–5,3), é equilátera e de eixo real horizontal;
3 5
b) y = x c) que tem como diretrizes as retas 5x ± 32 = 0, como assíntotas as
2 3 retas 3x ± 4y = 0 e eixo real horizontal;
e) y = – x−4
3 2 d) que tem eixo real vertical de comprimento 8 e passa pelo ponto
c) y= x +1 (6,5).
3

14. Um aluno desenhou, em um plano cartesiano, duas cônicas (elipse 24. (ITA) Considere a família de circunferências com centros no
ou hipérbole), uma de excentricidade 0,8 e outra de excentricidade segundo quadrante e tangente ao eixo Oy. Cada uma destas
2,4, tendo ambas como foco o par de pontos (−12,0) e (12,0). circunferências corta o eixo Ox em dois pontos, distantes entre si de
4 cm. Então, o lugar geométrico dos centros destas circunferências é
Assinale o que for correto. parte:
01) A cônica de excentricidade 0,8 é uma hipérbole. a) de uma elipse.
02) A cônica de excentricidade 2,4 passa pelo ponto (5,0). b) de uma parábola.
04) As cônicas descritas possuem quatro pontos em comum. c) de uma hipérbole.
x2 y2 d) de duas retas concorrentes.
08) + =1 é uma equação para a cônica de excentricidade 0,8.
225 81 e) da reta y = –x.
16) A cônica de excentricidade 0,8 passa pelo ponto (0,9).
O somatório das afirmativas corretas é um número do intervalo 25. (IME 2010) Considere as hipérboles que passam pelos pontos
a) [1, 6[. d) [18, 24[ (–4,2) e (–1,–1) e apresentam diretriz na reta y = –4. Determine a
b) [6, 12[. e) [24, 30] equação do lugar geométrico formado pelos focos dessas hipérboles,
associados a esta diretriz, e represente o mesmo no plano cartesiano.
c) [12, 18[.

547

PM_BOOK16 - MAT.indb 547 25/11/2022 19:27:04


GEOMETRIA ANALÍTICA: HIPÉRBOLE

EXERCÍCIOS DE 10. (IME) Uma hipérbole de excentricidade √2 tem centro na origem

COMBATE
e passa pelo ponto (√5,1). A equação de uma reta tangente a esta
hipérbole e paralela a y = 2x é:
a) √3y = 2√3x + 6 c) 3y = 6x + 2√3 e) y = 2x + √3
b) y = – 2x + 3√3 d) √3y = 2√3x + 4
01. Os focos de uma hipérbole são F2(6,2) e F1(6,12) e o comprimento
de seu eixo imaginário é 6. Determine a equação reduzida da
hipérbole.

02. (AFA 99) O valor da excentricidade da cônica


( x  5)2 ( y  2)2
4

9
 1 é: DESAFIO PRO
13 5
a) 2 b) c) d) 3

1
2 2 (ITA) Sabendo que 9y2 – 16x2 – 144y + 224x – 352 = 0 é a
equação de uma hipérbole, calcule sua distância focal.
03. Determine as coordenadas do centro e dos focos da cônica
2x2 – 7y2 – 4x + 14y – 19 = 0.

04. A equação reduzida da hipérbole, cujos focos são os extremos


2 (ITA) Seja o ponto = A (r, 0), r > 0. O lugar geométrico
dos pontos P = (x, y) tais que é de 3r 2 a diferença entre
do eixo menor da elipse de equação 16x² + 25y² = 625, e cuja o quadrado da distância de P a A e o dobro do quadrado da
excentricidade é igual ao inverso da excentricidade da elipse, é: distância de P à reta y = −r, é:
a) 16y² – 9x² = 144 c) 9x² – 16y² = 144 a) uma circunferência centrada em (r, − 2r) com raio r.
b) 9y² – 16x² = 144 d) 16x² – 9y² = 144 b) uma elipse centrada em (r, − 2r) com semieixos valendo r e 2r.
c) uma parábola com vértice em (r, − r).
05. Qual é a cônica representada pela equação d) duas retas paralelas distando r 3 uma da outra.
4 x 2  y 2  32x  8y  52  0 ? e) uma hipérbole centrada em (r,–2r) com semieixos valendo r.

06. Considere o círculo x2 + y2 – r2 = 0 de raio r e a hipérbole x2 – y2 = 1.


Nesse caso, pode-se afirmar que:
a) se r < 1, então as curvas se intersectam em quatro pontos.
3 (ITA) Considere a família de circunferências com centros
no segundo quadrante e tangentes ao eixo Oy. Cada uma
destas circunferências corta o eixo Ox em dois pontos, distantes
b) se r = 1, então as curvas têm quatro pontos em comum. entre si de 4 cm. Então, o lugar geométrico dos centros destas
c) se r = 1, as curvas se intersectam em (0,1) e (0,–1) circunferências é parte:
d) se r = √17, então as curvas se intersectam apenas nos pontos a) de uma elipse.
(3,2√2) e (–3,–2√2) b) de uma parábola.
e) se r > √17, então as curvas se intersectam em quatro pontos. c) de uma hipérbole.
d) de duas retas concorrentes.
07. O gráfico da equação x2 – y2 = 4 representa uma hipérbole. Os
e) da reta y = –x.
focos dessa hipérbole são:

a)  ,0  e  − ,0 
1 1
2   2 
b) (2,0) e (–2,0)
4 (IME) Uma hipérbole de excentricidade
origem e passa pelo ponto ( )
5,1 .
2 tem centro na

c) (2√2,0) e (–2√2,0) A equação de uma reta tangente a esta hipérbole e paralela a


y = 2x é:
d) (0,√2) e (0,–√2)
a) =
3 y 2 3 x+6
 1  1
e)  0,  e  0, −  b) y =
 2  2 −2x + 3 3
08. (AFA) A equação reduzida da hipérbole, cujos focos são os c) = 6x + 2 3
3y
extremos do eixo menor da elipse de equação 16x2 + 25y2 = 625,
d) =
3 y 2 3 x+4
e cuja excentricidade é igual ao inverso da excentricidade da elipse
dada, é: e) =
y 2x + 3
a) 16y2 – 9x2 = 144

5
b) 9y2 – 16x2 = 144 (IME) Uma hipérbole equilátera de eixo igual a 4, com
c) 9x2 – 16y2 = 144 centro na origem, eixos paralelos aos eixos coordenados e
d) 16x2 – 9y2 = 144 focos no eixo das abscissas sofre uma rotação de 45º no sentido
anti-horário em torno da origem. A equação dessa hipérbole
após a rotação é:
09. (ESPCEX) A representação no sistema cartesiano ortogonal da
equação 9x2 – y2 = 36x + 8y – 11 é dada por: a) xy = 2

b) x + xy − y =
2 2
a) duas retas concorrentes. 4
b) uma circunferência. c) x2 − y2 =
2
c) uma elipse.
d) xy = −2
d) uma parábola.
e) x2 − y2 =
−2
e) uma hipérbole.

548

PM_BOOK16 - MAT.indb 548 25/11/2022 19:27:09


GEOMETRIA ANALÍTICA: HIPÉRBOLE

17. x − y =25 ; F( ±5 2,0) ; A( ±5, 0)


2 2
GABARITO
18. Portando a cônica é uma hipérbole com centro (0,0), eixo real
EXERCÍCIOS DE FIXAÇÃO horizontal.
01. a) 2
b) 4 2
( )
19. F1 − 2,0 e F2 ( 2,0 )
20. 2x − 3y + 13 =
0
c) 2 7
(x − 1)2 (y + 3)2 (x − 1)2 (y + 3)2
d) 2 2 21. − 1 e −
= + =
1
16 9 16 9
02. a) (x – 7)2/9 – y2/16 = 1
22. x − 8y − 6x − 22y + 4 =
2 2
0 ; uma hipérbole de eixo real vertical, de
b) (y – 2)2 – (x + 3)2/3 = 1
 11 
03. C centro C  3, − 
 8
04. a) (x – 1)2 + 2(y + 1)2 = 1, elipse, e = 3 /2 23. A
b) (x – 2)2/16 – y2/18 = 1, hipérbole, e = 34 /4 24. C
05. a) x2/9 – y2/16 = 1 25. O lugar geométrico procurado é o conjunto dos pontos da
b) (y + 4)2/4 – (x – 3)2/5 = 1 circunferência C1 que são exteriores à circunferência C2.
06. EXERCÍCIOS DE COMBATE
01. DISCURSIVA 06. E
02. B 07. C
03. DISCURSIVA 08. A
04. A 09. E
05. DISCURSIVA 10. A
DESAFIO PRO
01. 10
07. a) 2 5 02. E
b) y =  2 5 /5x – 2 03. C
x 2
y 2 04. A
08. − =
1
9 16 05. A


09. Fo cos : 
(
 0, − 13 ) ANOTAÇÕES

(
 0, 13 )
x2 y2 y2 x2
10. H : − =
1 ; H' : − =
1
7 16 16 7
EXERCÍCIOS DE TREINAMENTO
01. E
02. C
03. E
04. C
05. B
06. C
07. D
08. A
09. a) (–1, 0)
b) 1,3 cm²
10. 30
11. 21
(
12. a) −2 + 5, 2 + 5 e ) ( −2 − 5, 2 − 5 )
b) não há intersecção entre a hipérbole e a reta.
c) (1 2, 2) .
13. E
14. E
15. E
2
16. C(–3,1); A1(–3, 3) e A2(–3,–1); y = 1 ±
5

549

PM_BOOK16 - MAT.indb 549 25/11/2022 19:27:10


GEOMETRIA ANALÍTICA: HIPÉRBOLE

ANOTAÇÕES

550

PM_BOOK16 - MAT.indb 550 25/11/2022 19:27:10


GEOMETRIA ANALÍTICA:
PARÁBOLA

DEFINIÇÃO
A parábola é o lugar dos geométricos dos pontos de um plano
equidistantes de um ponto fixo e de uma reta fixa do mesmo plano. O
ponto fixo denomina-se foco (F) e a reta fixa, diretriz (d).
Definição geométrica: é a cônica obtida mediante a secção de um
plano secante a um cone quadrático, sendo o plano paralelo a uma e
somente uma geratriz do cone.

ELEMENTOS DA PARÁBOLA

I.
p 
Equação reduzida: o ponto F tem coordenadas  , 0  .
2 
Calculemos u e v:
2
 p p
u  dFP   x    y 2 e v   x
 2 2

Igualando, conforme (1), vem:


2
 p 2 p
x    y   x
 2 2

p2 p2
I. Pontos principais: x 2  px   y2   px  x 2
4 4
F – foco
V – vértice y 2 = 2px
II. Segmentos: Discutamos essa equação:
V’F = p – parâmetro (semicorda focal mínima) i. Interseções


FP – raio vetor Para x = 0, y = 0. A curva representativa de (2) passa pela origem.
III. Relação:
ii. Simetria
p Trata-se de uma curva simétrica apenas em relação ao eixo x.
VF =
2
iii. Extensão
IV. Reta e eixo: a reta fixa (d) é a diretriz e e, eixo que passa pelo foco Resolvendo a equação (2) em relação a y, vem:
e é perpendicular à diretriz, eixo de simetria da parábola.
y   2px

Onde se conclui: se p > 0, não há valores reais de y para x < 0; se


EQUAÇÕES p < 0, não há valores reais de y para x > 0. A valores acrescentes
Equação espontânea. No sistema foco-diretriz a equação de x, em valor absoluto, correspondem valores crescentes de y. Se
espontânea da parábola é: x tende para ∞, y tende para ∞. A curva é aberta, estendendo-se
De acordo com a definição, temos: indefinidamente no plano dos eixos coordenados, à direita do eixo y
para p > 0 e à esquerda desse eixo para p < 0.
FP  MP  u  v (1)

551

PM_BOOK16 - MAT.indb 551 25/11/2022 19:27:17


GEOMETRIA ANALÍTICA: PARÁBOLA

ou
y
y = ax2 + bx + c, com a ≠ 0 (4)
p Comparando (1) e (3):
2 p 1 1
d a p
2 2p 2a
x
O n b
F' b    n  bp  n  
p 2a
y
n2  2mp
p p c  2cp  n2  2mp 
2p
2 2
x c b2 m 4ac  b2 b2  4ac
F' O d    m ou m 
a 4a2 a 4a 4a
 ∆ b  1
então, o vértice é V  − , −  e o parâmetro p = .
 4a 2a  2a
De modo análogo, comparando (2) e (4), concluímos que o vértice
p  b  1
Equação da diretriz: x   é V  – ,   e o parâmetro p = .
2  2a 4a  2a

ProBizu
Imagem geométrica CÔNICAS - RESUMO TEÓRICO
Conclui-se por essa discussão que a cônica de equação (2) é uma
DEFINIÇÕES
parábola de eixo horizontal coincidente com o eixo x e vértice na
origem, tendo a concavidade voltada para a direita se p > 0 e para Dados dos pontos F1 e F2 distantes 2c. Uma elipse
a esquerda se p < 0. de focos em F1 e F2 é o conjunto dos pontos cuja
ELIPSE
soma das distâncias a F1 e F2 é constante 2a, com
II. Se V = 0 e o eixo de simetria coincidir com o eixo dos y. 2a > 2c.
 p
As coordenadas do foco passam a ser  0,  , então a equação Dados dos pontos F1 e F2 distantes 2c. Uma
da parábola toma a forma  2
hipérbole de focos em F1 e F2 é o conjunto dos
HIPÉRBOLE
pontos cujo módulo da diferença das distâncias a
x 2 = 2py
F1 e F2 é constante 2a, com 2a < 2c.
e a da diretriz
Dados um ponto F e uma reta d (F ∈ d) e p a
p
y PARÁBOLA distância entre eles. Parábola é o conjunto dos
2 pontos do plano equidistantes de F e d.
III. Quando a parábola tem V(m, n), portanto, V ≠ 0 e o eixo de
simetria paralelo ao eixo 0x, vem:

( y ’)2 = 2px’ ELEMENTOS PRINCIPAIS

e aplicando a translação de eixos de I resulta: ELIPSE


(y – n) = 2p(x – m) ou (y – n) = –2p(x – m)
2 2
F1 e F2 → focos A1A 2 → eixo maior (2a) 2c → distância focal
IV. Equação geral: a equação geral é obtida, como vimos,
desenvolvendo as reduzidas. Assim: (y – n)2 = 2p(x – m), parábola B1B2 → eixo
O → centro c/a → excentricidade
com eixo horizontal, menor (2b)

1 2 n n2  2mp HIPÉRBOLE
y2 – 2ny + n2 = 2px – 2mp ⇒ x  y  y (1)
2p p 2p F1 e F2 → focos A1A 2 → eixo real (2a) 2c → distância focal
1 1 B1B2 → eixo
Se > 0, concavidade à direita e < 0, concavidade à esquerda. O → centro c/a → excentricidade
2p 2p transverso (2b)
De (x – m)2 = 2p(y – n), parábola com eixo vertical, PARÁBOLA
2
1 2 m m  2np F → foco d → diretriz p → parâmetro V → vértice
x 2 – 2mx + m2 = 2py – 2np ⇒ y  x  x (2)
2p p 2p

1 1
Se > 0, concavidade para cima e < 0, concavidade para RELAÇÕES NOTÁVEIS
baixo. 2p 2p
ELIPSE HIPÉRBOLE PARÁBOLA
Uma equação do 2º grau com duas variáveis representa uma
parábola com eixo horizontal ou vertical se, e somente se, for redutível p
às formas a2 = b2 + c2 c2 = a2 + b2 VF =
2
x = ay2 + by + c, com a ≠ 0 (3)

552

PM_BOOK16 - MAT.indb 552 25/11/2022 19:27:30


GEOMETRIA ANALÍTICA: PARÁBOLA

EQUAÇÕES REDUZIDAS ROTAÇÃO DE EIXOS

ELIPSE As coordenadas de um ponto P(x,y) após a rotação de


eixos de um ângulo θ são dadas por (x',y') tais que
Focos em Ox (–c,0) e (c,0) Focos em Oy (0,–c) e (0,c)
x = x' · cosθ – y' · senθ y = x' · senθ + y' · cosθ
x2 y2 y2 x2
 1  1
a2 b2 a2 b2
INTERPRETAÇÃO DE UMA EQUAÇÃO DO 2º GRAU
HIPÉRBOLE

Focos em Ox (–c,0) e (c,0) Focos em Oy (0,–c) e (0,c) Dada a equação geral do 2o grau Ax 2 + 2Bxy + Cy2 + 2Dx +
2Ey + F = 0 é sempre possível eliminar o seu termo retângulo
(2Bxy) através de um rotação de eixos de um ângulo θ tal que
x2 y2 y2 x2
 1  1
a2 b2 a2 b2 A=C→θ=π/4 A ≠ C ≠ tg 2θ = 2B/ (A – C)
PARÁBOLA

Foco em Ox (p/2,0) Foco em Oy (0,p/2)


EXERCÍCIOS DE
y = 2px x 2 = 2py
FIXAÇÃO
2

EQUAÇÕES REDUZIDAS – CENTRO EM (XO,YO)


01. Dada a equação da parábola x = –3y2 + 6y
ELIPSE
a) Escreva a equação na forma reduzida.
b) Determine o foco e a diretriz dessa parábola.
 x-x 0 2

 y-y 0  2
1
 y-y 0  2

 x-x 0  2
1
2 2 2 2
a b a b 02. A parábola possui foco F(0,–1) e diretriz d: y – 1 = 0. Determine:
a) o vértice da parábola
HIPÉRBOLE
b) a equação geral
 x-x 0 2

 y-y 0  2
1
 y-y 0  2

 x-x 0 2
1 03. A parábola possui vértice V(3,0) e eixo de simetria y = 0, passando
a2 b2 a2 b2 pelo ponto (4,1). Determine:
PARÁBOLA - Equação Reduzida – vértice em (xo,yo) a) a equação reduzida
b) o vértice e o foco
(y – y0)2 = 2p.(x – x0) (x – x0)2 = 2p.(y – y0)
04. Determine as condições dos coeficientes abaixo para que a
equação Ax2 + By2 + Cx + Dy + E = 0, determine:
RECONHECIMENTO DE UMA CÔNICA
a) uma circunferência c) uma hipérbole
Dada uma equação do 2o grau redutível b) uma elipse d) uma parábola

à forma
 x-x 0 2   y-y 0 2 1 05. Qual a inclinação da reta que passa pelo ponto (2,4) e que
k1 k2
intercepta a parábola y = x2 em um único ponto?
k1 > 0, k2 > 0 e k1 > k2 elipse de eixo maior horizontal
06. A região do plano cartesiano, determinada simultaneamente
k1 > 0, k2 > 0 e k1 < k2 elipse de eixo maior vertical pelas três condições, x2 + y2 ≤ 16, y ≥ x2 e x ≥ 0

k1 > 0 e k2 < 0 hipérbole de eixo real horizontal

k1 < 0 e k2 > 0 hipérbole de eixo real vertical

PARÁBOLAS - p = 1/4 | a |

y = ax + bx + c
2
x = ay2 + by + c
diretriz horizontal diretriz vertical

b −∆ −∆ b
xv  e yv = xv = e yv 
2a 4a 4a 2a
é aquela, na figura, indicada com a letra
a > 0 → conc. p/ cima a > 0 → conc. p/ direita a) A d) D
a < 0 → conc. p/ baixo a < 0 → conc. p/ esquerda
b) B e) E
c) C

553

PM_BOOK16 - MAT.indb 553 25/11/2022 19:27:37


GEOMETRIA ANALÍTICA: PARÁBOLA

07. Dada a equação da p arábola y = 4x2 – 16x + 15 11. Seja a parábola de equação y = 3x2 + 4. As equações das retas
a) Escreva a equação na forma reduzida. tangentes ao gráfico da parábola que passam pelo ponto P = (0,1)
são:
b) Determine o foco e a diretriz dessa parábola.
a) y = 5x +1 e y = – 5x + 1
08. A parábola possui vértice V(2,–1) e foco F(5,–1). Determine: b) y = 6x +1 e y = – 6x + 1
a) a equação da diretriz b) a equação geral c) y = (3x/2) +1 e y = – (3x/2) + 1
d) y = (5x/4) +1 e y = – (5x/4) + 1
09. A parábola possui vértice V(–4,3) e diretriz d: y = –1. Determine: e) y = 5x – 1 e y = – 5x –1
a) o foco b) a equação geral
12. (UECE) Se a reta r, tangente à circunferência x2 + y2 = 1 no ponto
10. Para que a parábola y = -2x + mx + 5 = 0 não intercepte a reta
2
2 2
y = 3, devemos ter: , , intercepta a parábola y = x2 + 1 nos pontos (x1,y1) e (x2,y2),
2 2
a) –4 < m < 4 d) m > 5 ou m < –5 então x1 + x2 é igual a
b) m < –3 ou m > 4 e) m = –5 ou m = 5 a) – 2 d) 1– 2
c) m≠0 b) – 1
c) –1– 2
EXERCÍCIOS DE

TREINAMENTO 13. A parábola y = x − tx + 2 tem vértice no ponto (x t , y t ). O lugar


2

geométrico dos vértices da parábola, quando t varia no conjunto dos


números reais, é
a) uma parábola. d) uma reta.
01. Determine a diretriz da parábola de equação y2 = –4x.
b) uma elipse. e) duas retas concorrentes.
c) um ramo de uma hipérbole.
02. O foco da parábola de equação y = 12x é:
2

a) F (0, 3) c) F (6, 0) e) F (–6, 0)


14. Uma reta r é paralela ao eixo x e contém a interseção das parábolas
b) F (–3, 0) d) F (3, 0) y = (x – 1)2 e y = (x – 5)2.
A equação de r é:
03. Determine os pontos de interseção da parábola x2 = –2y com a
a) x = 3 c) y = 3x e) y = x/3
reta y = x.
b) y = 4 d) x = 4y
04. A equação da parábola com vértice na origem e foco no ponto
15. Os gráficos das curvas x2 + y2 = 2 e y = x2 se interceptam nos
1 
F  , 0  é: pontos A e B. Os valores das abscissas de A e B são:
4 
a) –1 e 0 c) –1 e 1 e) –1 e –2
a) x2 = y c) x2 = 4y e) 4y2 = x
b) 0 e 1 d) 1 e 2
b) y2 = x d) y2 = 4x
16. As parábolas dadas pelas equações y=x2 e x=y2
05. A equação da parábola de vértice V(0, 0) e diretriz x = 2 é:
a) nunca se encontram.
a) y2 = –8x c) x2 = 8y e) y2 = –2x
b) se encontra apenas na origem.
b) x2 = –8y d) y2 = 8x
c) se encontram em exatamente dois pontos.
06. A parábola com vértice na origem e foco F  0,  tem equação:
1
d) se encontram em três pontos.
 2
e) se encontram em quatro pontos.
a) y2 = -2x c) x2 = 2y e) y2 = 4x
b) x = -2y
2
d) y = 2x
2
17. A reta s é paralela à reta de equação y = 3x – 4 e intercepta
a parábola de equação y = 2x2 – 3x + 5 no ponto de abscissa 1. A
07. Determine a equação da parábola com vértice na origem, simétrica equação de s é
em relação ao eixo X e que passa pelo ponto P (–2, 1).
a) x + y – 5 = 0 d) x + 3y – 11 = 0

08. Determine as tangentes à parábola y = 2x que passam pelo ponto


2 b) x – y + 3 = 0 e) 3x + y – 7 = 0
P1 (–1, 0). c) 3x – y + 1 = 0

09. Determine a tangente à parábola y2 = 4x no ponto (1, 2). 18. No plano cartesiano, há dois pontos R e S pertencentes à parábola
de equação y = x² e que estão alinhados com os pontos A(0,3) e B(4,0).
10. O vértice, o foco e a reta diretriz da parábola de equação y = x2 A soma das abscissas dos pontos R e S é:
são dados por: a) –0,45 d) –0,75
a) Vértice: (0, 0); Foco: (0, 1/4); Reta diretriz y = –1/4 b) –0,55 e) –0,85
b) Vértice: (0, 0); Foco: (0, 1/2); Reta diretriz y = –1/2 c) –0,65
c) Vértice: (0, 0); Foco: (0, 1); Reta diretriz y = –1
d) Vértice: (0, 0); Foco: (0, –1); Reta diretriz y = 1 19. O conjunto dos pontos P = (x, y), que estão a uma mesma
e) Vértice: (0, 0); Foco: (0, 2); Reta diretriz y = -2 distância do ponto F = (0, 2) e do eixo Ox, no plano cartesiano xy é

554

PM_BOOK16 - MAT.indb 554 25/11/2022 19:27:38


GEOMETRIA ANALÍTICA: PARÁBOLA

x2 c) e)
a) a parábola de equação =
y + 4.
2
x2
b) a parábola de equação =
y + 1.
2
c) a parábola de equação=
y 4x 2 + 1.
d) a parábola de equação=
y 2x 2 + 1.

20. A região do plano cartesiano, destacada na figura a seguir, é


determinada por uma parábola, com vértice na origem, e duas retas.
d)

Esta região pode ser descrita como o conjunto dos pares ordenados
(x, y) ∈  × , satisfazendo 23. Os pontos P e Q estão na parábola dada por y = 4x2 + 7x – 1, e a
x 2
x 3 origem do sistema de coordenadas cartesianas está no ponto médio
a) −2 ≤ x ≤ 2 e ≤y≤− + .
4 4 2 de PQ. Sendo assim, P e Q são pontos que estão na reta
x2 x 3 15x 13x 11x
b) −2 ≤ x ≤ 2 e − ≤y≤ + . a) y= . c) y= . e) y= .
4 4 2 2 2 2
x 3 b) y = 7x. d) y = 6x.
−2 ≤ x ≤ 2 e 4x ≤ y ≤ − + .
2
c)
4 2
x 3 24. Na figura, estão representados, no plano cartesiano xOy, a reta de
d) −2 ≤ x ≤ 2 e −4x 2 ≤ y ≤ − + . 3
4 2 equação y = 2kx, 0 ≤ k ≤ , a parábola de equação y = – x2 + 3x e os
2
x2 x 3 pontos O, P e Q de intersecções da parábola com o eixo Ox e da reta
e) −2 ≤ x ≤ 2 e ≤y≤ + .
4 4 2 com a parábola.

21. Para efeitos práticos, a relação entre as grandezas x e y que,


x2
teoricamente, seria dada por y = 1 + e cujo gráfico cartesiano
4
se vê abaixo, em linha tracejada, foi substituída pela relação linear
representada pela reta que passa por A e B.

Nestas condições, o valor de k para que a área do triângulo OPQ seja


a maior possível é:
1 9 3
a) . c) . e) .
2 8 2
3 11
b) . d) .
4 8

Dessa forma, a diferença dy, que se obtém quando x = 6, vale:


25. A reta r intercepta o eixo das ordenadas em y = 2 e a parábola p
a) 1,5 b) 2,0 c) 2,5 d) 3,0 e) 3,5 em seu vértice. Se a equação de p é y = 3x2 - 6x + 8, então r intercepta
o eixo das abcissas no ponto
3x + y ≤ 2
2 2

22. A solução gráfica do sistema de inequações  2 é a a) (3/4; 0) d) (–1/2; 0)


x + y ≤ 1
2
região sombreada em b) (2/5; 0) e) (–2/3; 0)
a) b) c) (0; 0)

26. Em um plano cartesiano π, Q = (x, y) é um ponto arbitrário e


P = (1, 0) é um ponto fixo. Denotamos por d(A, B) a distância entre
quaisquer dois pontos A e B pertencentes a π. Considere o conjunto
C= {Q ∈ π tal que 2d(G, Q)= d(Q, P)}, em que G = (0, 0) é a
origem de π. Então,

555

PM_BOOK16 - MAT.indb 555 25/11/2022 19:27:40


GEOMETRIA ANALÍTICA: PARÁBOLA

x 32. (EN) Considere a sequência (a,b,2) uma progressão aritmética e a


a) C é a parábola de equação y =−x 2 −   . sequência (b,a,2) uma progressão geométrica não constante, a,b ∈ .
 2
A equação da reta que passa pelo ponto (a,b) e pelo vértice da curva
b) C é a parábola de equação =
y x 2 + 2. y 2 − 2y + x + 3 =0 é
 x   1 a) 6y − x − 4 =0
c) C é a reta de equação= y   −  .
 2  4  b) 2x − 4y − 1 =0
d) C é o círculo de centro em (1,0) e raio 1. c) 2x − 4y + 1 =0
e) C é o círculo de centro em (–1,0) e raio 2. d) x + 2y =
0
e) x − 2y =
0
27. As equações y - 2x = 0, y + x2 = 0 e y2 - x2 + 1 = 0 representam no
plano, respectivamente: 33. (ITA) São dadas as parábolas p1: y = -x2 – 4x – 1 e p2: x2 – 3x + 11/4
a) uma reta, uma hipérbole e uma parábola cujos vértices são denotados, respectivamente, por V1 e V2. Sabendo
b) uma parábola, uma hipérbole e uma reta que r é a reta que contém V1 e V2, então a distância de r até a origem é:
c) uma reta, uma parábola e uma elipse 5 17
a) d)
d) uma elipse, uma parábola e uma hipérbole 26 50
7 11
e) uma reta, uma parábola e uma hipérbole b) e)
26 74
28. Seja A o vértice da parábola de equação y = x 2 − 4x + 6. A 7
c)
reta que passa pela origem O do plano cartesiano e pelo ponto A 50
intercepta a parábola também num ponto B. Pode-se afirmar que:
a) OA = AB c) AB= 2 ⋅ OA e) OA= 3 ⋅ AB 34. (ITA) Considere a hipérbole H e a parábola T, cujas equações são,
respectivamente, 5(x + 3)2 – 4(y – 2)2 = -20 e (y – 3)2 = 4(x – 1).
b) OA= 2 ⋅ AB d) AB= 3 ⋅ OA
Então, o lugar geométrico dos pontos P, cuja soma dos quadrados das
x 2 + y 2 =
 4 distâncias de P a cada um dos focos da hipérbole H é igual ao triplo do
29. Na representação gráfica do sistema de equações  2 quadrado da distância de P ao vértice da parábola T, é:
4x − y =
 2
(x − 3)2 (y + 2)2
no plano cartesiano, uma das soluções é (0, − 2). A distância entre a) A elipse de equação + =1
os pontos que representam as duas outras soluções desse sistema é 4 3
igual a (y + 1)2 (x − 3)2
b) A hipérbole de equação − =
1
5 4
a) 14. 14
d) . c) O par de retas dadas por y = ± (3x – 1)
7 2
b) . d) A parábola de equação y2 = 4x + 4
2 3
e) . e) A circunferência centrada em (9, 5) e raio 120
2
15
c) .
2 35. (ITA) Num sistema de coordenadas cartesianas ortogonais,
30. (ESPCEX) Considere as afirmações: considere a família de circunferências que passam pelo ponto (2,–1/2)
e que são tangenciadas pela reta y = -3/2. Então, a equação do lugar
I. Uma elipse tem como focos os pontos F1( −3, 0), F2 (3, 0) e a geométrico dos centros dessas circunferências é dada por:
x2 y2
medida do eixo maior é 8. Sua equação é + =1. a) x2 – 4x – 2y + 2 = 0
16 7
b) y2 – 2y – 5x – 2 = 0
II. Os focos de uma hipérbole são F1( −10, 0), F2 (10, 0) e sua
5 c) x2 + 2x – 7y + 3 = 0
excentricidade é . Sua equação é 16x − 9y =
2 2
576.
3 d) y2 – 4y – 2x – 3 = 0
III. A parábola 8x = −y 2 + 6y − 9 tem como vértice o ponto V(3, 0). e) x2 + y2 – 2x + y – 2 = 0

Com base nessas afirmações, assinale a alternativa correta.


a) Todas as afirmações são falsas. EXERCÍCIOS DE

COMBATE
b) Apenas as afirmações I e III são falsas.
c) Apenas as afirmações I e II são verdadeiras.
d) Todas as afirmações são verdadeiras.
e) Apenas a afirmação III é verdadeira.
01. (ESPCEX 2015) Uma reta t passa pelo ponto A(–3,0) e é tangente
à parábola de equação x = 3y2 no ponto P. Assinale a alternativa que
31. Uma bola é jogada dentro de uma cesta cuja superfície é obtida
apresenta uma solução correta de acordo com essas informações.
girando a parábola y = x em torno do eixo y O centro da bola ocupa
2

a) t : x – 10y + 3 = 0 e P(27,3)
um ponto de altura y = 3. O raio da bola é:
b) t : 2x – 15y + 6 = 0 e P(12,2)
a) 11 11
d) c) t : 2x –15y + 6 = 0 e P(12,–2)
4
11 d) t : y = 0 e P(0,0)
b)
2 11
e) e) t : x + 6y + 3 = 0 e P(3,–1)
5
11
c)
3

556

PM_BOOK16 - MAT.indb 556 25/11/2022 19:27:43


GEOMETRIA ANALÍTICA: PARÁBOLA

02. (PUC-RJ 2015) A figura abaixo mostra uma reta e uma parábola 07. (AFA) Com relação ao conjunto de pontos P(x, y) equidistantes da
de eixo vertical. reta y = 3 e da origem do sistema cartesiano ortogonal, é INCORRETO
afirmar que é uma curva
y
a) representada por x² – 6y – 9 = 0.
b) cujas coordenadas do vértice tem soma 1,5.
(0,8)
c) que representa uma função par.
d) cujo parâmetro é igual a 3.

08. (AFA) Analise as proposições abaixo e escreva V para a(s)


verdadeira(s) e F para a(s) falsa(s).
(0,2)
( ) A distância entre o vértice e o foco da parábola y2 + 4x – 4 = 0 é
(-2,0) x igual a 1 unidade de comprimento.
(2,0) (4,0) ( ) Numa hipérbole equilátera, as assíntotas são perpendiculares
entre si.
( ) A equação 2x2 + y2 – 4x – 4y + 4 = 0 representa uma elipse que
a) Sabendo que a reta corta os eixos nos pontos (–2,0) e (0,2), tem um dos focos no ponto P(1,4).
encontre a equação da reta.
A sequência correta é
b) Sabendo que a parábola corta os eixos nos pontos (0,8), (2,0) e
(4,0), encontre a equação da parábola. a) F - F - V c) F-V-F
c) Encontre os pontos de interseção entre a reta e a parábola. b) V - F - V d) V - V - F

09. (AFA) Classifique em VERDADEIRO ou FALSO cada item a seguir.


03. (ITA 2013) Sobre a parábola definida pela equação
(2) A parábola cuja equação é x² – 4y = 0 tem diretriz representada
x2 + 2xy + y2 − 2x + 4y + 1 =0 , pode-se afirmar que
pela reta y + 1 = 0 e foco coincidente com o baricentro do triângulo
a) ela não admite reta tangente paralela ao eixo Ox. ABC, onde A é a origem do sistema cartesiano, B (2, 3) e C (–2, 0).
b) ela admite apenas uma reta tangente paralela ao eixo Ox. (3) O conjunto de pontos representados pela equação x² – y² + x +
c) ela admite duas retas tangentes paralelas ao eixo Ox. y = 0 é uma hipérbole equilátera que NÃO tem centro na origem do
sistema cartesiano.
d) a abscissa do vértice da parábola é x = –1.
(8) Na elipse 16x² + 64y² = 1 a medida do eixo vertical é 50% da
e) a abscissa do vértice da parábola é x = –2/3.
medida do eixo horizontal.
04. (EN 2012) Considere a sequência (a,b,2) uma progressão aritmética (16) Existem apenas 4 números inteiros entre os valores de k, para os
e a sequência (b,a,2) uma progressão geométrica não constante, a, b quais o vértice da parábola y² = 4x + 1 é ponto exterior à circunferência
∈ . A equação da reta que passa pelo ponto (a,b) e pelo vértice da x² + y² – 2x + 4y + k = 0.
curva y2 – 2y + x + 3 = 0 é A soma dos itens VERDADEIROS é um número do intervalo
a) 6y – y – 4 = 0 a) [2, 10[ c) [16, 22[
b) 2x – 4y – 1 = 0 b) [10, 16[ d) [22, 30[
c) 2x – 4y + 1 = 0
10. Determine o produto dos valores máximo e mínimo de y que
d) x + 2y = 0
satisfazem às inequações dadas para algum valor de x
e) x – 2y = 0
2
2x − 12x + 10 ≤ 5y ≤ 10 − 2x
05. A distância entre o vértice e o foco da parábola de equação 2x2 – a) –3,2
4x – 4y + 3 = 0 é igual a: b) –1,6
a) 2 c) 0
b) 3/2 d) 1,6
c) 1 e) 3,2
d) 3/4
e) 1/2

 1
06. (ESPCEX) O ponto P  a,  pertence à parábola x =
 3
equação da reta perpendicular à bissetriz dos quadrantes ímpares que
y² + 3
3
.A
DESAFIO PRO
passa por P é:
a) 27x + 27y – 37 = 0
b) 37x + 27y – 27 = 0
1 (ITA) Pelo ponto C : (4,–4) são traçadas duas retas que
tangenciam a parábola y = (x – 4)2 + 2 nos pontos A e B. A
distância do ponto C à reta determinada por A e B é:
c) 27x + 37y – 27 = 0 a) 6√12
d) 27x + 27y – 9 = 0 b) √12
e) 27x + 37y – 9 = 0 c) 12
d) 8
e) 6

557

PM_BOOK16 - MAT.indb 557 25/11/2022 19:27:43


GEOMETRIA ANALÍTICA: PARÁBOLA

2 (ITA) Sobre a parábola definida pela equação


x 2 + 2xy + y 2 − 2x + 4y + 1 =0 pode-se afirmar que
25. E
26. E
29. C
30. C
33. E
34. E
a) ela não admite reta tangente paralela ao eixo Ox. 27. E 31. B 35. A
b) ela admite apenas uma reta tangente paralela ao eixo Ox. 28. B 32. D
c) ela admite duas retas tangentes paralelas ao eixo Ox. EXERCÍCIOS DE COMBATE
d) a abscissa do vértice da parábola é x = –1. 01. E 04. D 07. A 10. A
2 02. DISCURSIVA 05. E 08. D
e) a abscissa do vértice da parábola é x = − .
3 03. B 06. A 09. B

3 (ITA) Considere a parábola de equação y = ax2 + bx + c, que


passa pelos pontos (2,5), (–1,2) e tal que a, b, c formam,
nesta ordem, uma progressão aritmética. Determine a distância
DESAFIO PRO
01. C 04. B
do vértice da parábola à reta tangente à parábola no ponto (2,5). 02. B 05. 2p + 4p + 2p 3 = 2p(3 + 3).
03. 5 u.c.

4
2
(ITA) O lugar geométrico dos pontos (a, b) ∈  tais que a 5
equação, em z ∈ , z2 + z + 2 − (a + ib) =0 possua uma raiz
puramente imaginária é
ANOTAÇÕES
a) uma circunferência.
b) uma parábola.
c) uma hipérbole.
d) uma reta.
e) duas retas paralelas.

5 (IME) É dada uma parábola de parâmetro p. Traça-se


a corda focal MN, que possui uma inclinação de 60° em
relação ao eixo de simetria da parábola. A projeção do ponto
M sobre a diretriz é o ponto Q, e o prolongamento da corda
MN intercepta a diretriz no ponto R. Determine o perímetro do
triângulo MQR em função de p, sabendo que N encontra-se no
interior do segmento MR.

GABARITO
EXERCÍCIOS DE FIXAÇÃO
01. d) A ⋅ B = 0
a) x – 3 = –3(y – 1)2 05. m = 4.
b) 12x – 37 = 0 e (35/12,1) 06. B
02. 07.
a) V(0,0) a) y + 1 = 4(x + 2)2
b) –4y = x 2
b) 16y + 17 = 0 e (–2,–15/16)
03. 08.
a) x – 3 = y2 a) x + 1 = 0
b) (13/4, 0) e 4x + 11 = 0 b) x = (y2 + 2y + 25)/12
ta := y mx + n
⇒ 0 = m.(04.
−1) + n ⇒ n = m → reta : y = mx + m09.
(1, 0 ) ∈ reta :
a) A=B a) (–4,7)
2
= 2x
⇒ (mx + m ) b) x y+=( 2m 2)+x +
2
= 2x
A .⇒ B> m20x 2 + 2m2x + m2 = 2x ⇒ mb) 2 2
(x2 2+−8x m2 = 0
64)/16
mx + m
c) A . B2 < 0 10. A
2x = −2y
angente : ∆ = 0 ⇒ ( 2m2 EXERCÍCIOS
− i)2) − 4.(m2 ).(m ⇒ x2TREINAMENTO
DE
2
)==0−2x
⇒⇒ 4mx42 −+8m
2x 2=⇒+04 − 4m4 = 0⇒
y = x
 2 2 2 15. C
01.m= 2; 1. → 1ª tange =0
x1 nte=:y x+
1 ⇒  1x(x +22) =0 ⇒  08. 2 2 16. C
8m + 4 = 0 ⇒ m =
2 2
⇒ 02.  6; (3,0). x 2 = −2
2 m =→ 2 2 2 17. C
( 0,0 )
1ª tangente :y = − x−
 Pontos 2 : 
2
03.ii) 2 2 18. D
( −2, −2) 09. reta
19. B
04. 1/2; x. 10. A 20. A
05. 4; –8x. 11. B 21. D
06. 1; 2y. 12. B 22. C
07. 1/4; – x/2. 13. A 23. B
14. B 24. B

558

PM_BOOK16 - MAT.indb 558 25/11/2022 19:27:44


EXERCÍCIOS DE REVISÃO I

EXERCÍCIOS DE 10. (UFRJ 2004) n e m são números naturais, n = 100! + 18 e

TREINAMENTO m = 50! + 37. Calcule o resto da divisão de n por 18.

11. (IME 2009) Uma urna contém cinco bolas numeradas de 1 a 5.


Retiram-se, com reposição, 3 bolas desta urna, sendo α o número
01. Uma comissão de k pessoas será escolhida de um grupo de 7 da primeira bola, β o da segunda e λ o da terceira. Dada a equação
mulheres e 4 homens, dentre os quais figuram João e Maria. De quadrática αx2 + βx + λ = 0, a alternativa que expressa a probabilidade
quantos modos isto pode ser feito de modo que: das raízes desta equação serem reais é
a) A comissão tenha 5 pessoas sendo 3 mulheres e 2 homens. 19 24 25
a) c) e)
b) A comissão tenha o mesmo número de homens e mulheres. 125 125 60
23 26
c) A comissão tenha 4 pessoas de modo que pelo menos 2 sejam b) d)
mulheres. 60 60
d) A comissão tenha 4 pessoas sendo João uma dessas pessoas. 12. Num exame há 3 respostas para cada pergunta e apenas uma delas
e) A comissão tenha 4 pessoas, sendo duas de cada sexo e de modo é certa. Portanto, para cada pergunta, um aluno tem probabilidade
que João e Maria não estejam simultaneamente na comissão. 1/3 de escolher a resposta certa se ele está adivinhando e 1 se sabe
a resposta. Um estudante sabe 30% das respostas do exame. Se ele
deu resposta correta para uma das perguntas, qual é a probabilidade
02. De quantos modos podemos formar uma fila de 5 pessoas
de que a adivinhou?
escolhidas em um grupo de 10 pessoas de modo que as pessoas da
fila fiquem da esquerda para direita com os números de suas carteiras
de identidade em ordem crescente? 13. (FUVEST) O polinômio P(x) = x³ – 3x² + 7x – 5 possui uma raiz
complexa ξ cuja parte imaginária é positiva. A parte real de ξ³ é igual a
03. Quantos são os anagramas da palavra “ANAGRAMA” que não a) –11 c) 9 e) 12
possuem duas vogais adjacentes? b) –7 d) 10

04. Há 5 pontos sobre uma reta R e 8 pontos sobre uma reta R’ 14. (FUVEST) Dado o polinômio p(x) = x2(x – 1)(x2 – 4), o gráfico da
paralela a P. Quantos são os triângulos e os quadriláteros convexos função y = p(x – 2) é melhor representado por:
com vértices nesses pontos?

05. (ITA 2007) Dentre 4 moças e 5 rapazes deve-se formar uma


comissão de 5 pessoas com, pelo menos, 1 moça e 1 rapaz. De
quantas formas distintas tal comissão poderá ser formada? a) d)

06. De um baralho comum de 52 cartas, extrai-se sucessivamente e


sem reposição duas cartas. De quantos modos isto pode ser feito se:
a) A primeira carta e uma dama e a segunda carta não é um rei?
b) A primeira carta é uma dama e a segunda carta não é de espadas?
c) A primeira carta é de espadas e a segunda carta não é uma dama?

07. Uma partícula, estando no ponto (x, y, z), pode mover-se para e)
o ponto (x + 1, y, z) ou para o ponto (x, y + 1, z) ou para o ponto b)
(x, y, z + 1). Quantos são os caminhos que a partícula pode tomar
para, partindo do ponto (0,0,0), chegar ao ponto (a, b, c), onde a > 0,
b > 0 e c > 0?

08. (IME 1999) Calcule o valor de (1,02)-10, com dois algarismos


significativos, empregando a expansão do binômio de Newton.

09. (IME 1989) Determine o coeficiente de x–9 no desenvolvimento de


2 5 c)
 2 1  1 .
x + 5  ⋅  x3 + 4 
 x   x 

559

PM_BOOK16 - MAT.indb 559 25/11/2022 19:27:45


EXERCÍCIOS DE REVISÃO I

15. (FUVEST) O polinômio p(x) = x4 + x3 – x2 – 2x – 2 é divisível por a) 3648. c) 3248. e) 2880.


x2 + a, para um certo número real a. Pode-se, pois, afirmar que o b) 3600. d) 3200.
polinômio p.
a) não tem raízes reais. 02. De quantas maneiras é possível colocar 6 anéis diferentes em 4
b) tem uma única raiz real. dedos?
c) tem exatamente duas raízes reais distintas.
03. (AFA) As senhas de acesso a um determinado arquivo de um
d) tem exatamente três raízes reais distintas.
microcomputador de uma empresa deverão ser formadas apenas
e) tem quatro raízes reais distintas. por 6 dígitos pares, não nulos. Sr. José, um dos funcionários dessa
empresa, que utiliza esse microcomputador, deverá criar sua única
16. (UNICAMP) Considere o polinômio cúbico p(x) = x³ + x² – ax – 3, senha. Assim, é INCORRETO afirmar que o Sr. José:
onde a é um número real. Sabendo que r e –r são raízes reais de p(x), a) Poderá escolher sua senha dentre as 212 possibilidades de formá-
podemos afirmar que p(1) é igual a las.
a) 3 c) –2 b) Poderá escolher dentre 120 possibilidades, se decidir optar por
b) 1 d) –4 uma senha com somente 4 dígitos iguais.
c) Terá 4 opções de escolha, se sua senha possuir todos os dígitos
17. (FUVEST) Considere o triângulo ABC no plano cartesiano com iguais.
vértices A = (0,0), B = (3,4) e C = (8,0). O retângulo MNPQ tem os d) Terá 480 opções de escolha, se preferir uma senha com apenas 3
vértices M e N sobre o eixo das abscissas, o vértice Q sobre o lado AB dígitos iguais.
e o vértice P sobre o lado BC. Dentre todos os retângulos construídos
desse modo, o que tem área máxima é aquele em que o ponto P é
04. Um fundo de investimentos disponibiliza números inteiros de
 16  c)  12   8 cotas aos interessados nessa aplicação financeira. No primeiro dia de
a)  4,   5,  e)  6, 
 5   5  5 negociação desse fundo, verifica-se que 5 investidores compraram
 11  cotas, e que foi vendido um total de 9 cotas. Em tais condições, o
 17 
b)  ,3  d)  ,2  número de maneiras diferentes de alocação das 9 cotas entre os 5
 4  2 
investidores é igual a:
18. (FUVEST) No plano cartesiano x0y, a reta de equação x + y = 2 é a) 56. c) 86. e) 126.
tangente à circunferência C no ponto (0,2). Além disso, o ponto (1,0)
b) 70. d) 120.
pertence a C. Então, o raio de C é igual a

a) 3 2 7 2 11 2 05. Quantos inteiros entre 1 e 1.000.000, inclusive, possuem a soma


c) e)
2 2 2 de seus dígitos igual a 13?
5 2 9 2
b) d) 06. (IME 2011) Um trem conduzindo 4 homens e 6 mulheres passa por
2 2
seis estações. Sabe-se que cada um destes passageiros irá desembarcar
19. (ITA) Sejam S1 = {(x,y) ∈ ² : y ≥ ||x| –1|} e S2 = {(x,y) ∈ ² : x² + em qualquer uma das seis estações e que não existe distinção dentre
(y +1)² ≤ 25}. A área da região S1 ∩ S2 é os passageiros de mesmo sexo. O número de possibilidades distintas
de desembarque destes passageiros é:
25 25 75
a) π−2 c) π e) π−2 a) 1287. c) 44200. e) 62822.
4 4 4
25 75 b) 14112. d) 58212.
b) π −1 d) π −1
4 4
07. De quantos modos se podem pintar as faces de uma pirâmide
20. (ITA) Considere os pontos A = (0,–1), B = (0,5) e a reta pentagonal regular usando seis cores diferentes, sendo cada face de
r: 2x – 3y + 6 = 0. Das afirmações a seguir: uma cor?
I. d(A,r) = d(B,r).
08. A soma dos três últimos dígitos de 1992 é:
II. B é simétrico de A em relação à reta r.
a) 6 c) 12 e) 19
III. AB é base de um triângulo equilátero ABC, de vértice C = (–3√3,2)
ou C = (3√3,2). b) 9 d) 15
É (são) verdadeira(s) apenas
09. (ITA) Seja P(x) = a0 + a1x + a2x2 + a3x3 + … + a100x100, onde
a) I. c) I e II. e) II e III. a100 = 1, um polinômio divisível por (x + 9)100. Nestas condições temos:
b) II. d) I e III. a) a2 = 50 · 99 · 998 100!9²
d) a2 =
100! 2!98!
b) a2 = e) nda
EXERCÍCIOS DE 2!98!

COMBATE c) a2 =
99!
2!98!
m

∑ 3 ⋅  k  =
k m
10. (AFA 99) O valor de m que satisfaz a expressão 1024
01. Cinco casais estão dispostos ao redor de uma mesa redonda. Seja
k =0
A o número de maneira que as pessoas podem ser dispostas ao redor
da mesa com a condição de que cada casal deve estar junto; e B o a) 2
número de maneira que elas podem ser dispostas com a condição de b) 3
que homens e mulheres estejam em lugares alternados. O valor de c) 4
A + B é:
d) 5

560

PM_BOOK16 - MAT.indb 560 25/11/2022 19:27:46


EXERCÍCIOS DE REVISÃO I

11. (FGV) Uma escola comprou computadores de 3 fabricantes: 18. (ITA) Considere dois círculos no primeiro quadrante:
A, B e C. 30% foram comprados de A, 30% de B e o restante de π
C. A probabilidade de um computador fabricado por A apresentar - C1 com centro (x1,y1), raio r1 e área .
16
algum tipo de problema, nos próximos 30 meses, é 0,1. As mesmas
probabilidades dos fabricantes B e C são respectivamente 0,15 e 0,2. - C2 com centro (x2,y2), raio r2 e área 144π.
a) Qual a probabilidade de que um computador escolhido ao acaso, Sabendo que (x1,y1,r1) e (x2,y2,r2) são duas progressões geométricas
seja fabricado por A e apresente algum problema nos próximos com somas dos termos iguais a 7/4 e 21, respectivamente, então a
30 meses? distância entre os centros de C1 e C2 é igual a
b) Se um computador apresentar algum problema nos próximos 30 123 131 137
meses, qual a probabilidade de que tenha sido fabricado por A? a) c) e)
2 2 2

12. (FUVEST) O polinômio p(x) = x3 + ax2 + bx, em que a e b são 129 135
b) d)
números reais, tem restos 2 e 4 quando dividido por x – 2 e x – 1, 2 2
respectivamente. Assim, o valor de a é:
a) –6 c) –8 e) –10 19. (FUVEST) P(x) é um polinômio cujas raízes formam uma progressão
geométrica de razão 2 e primeiro termo 2. O coeficiente do termo de
b) –7 d) –9 mais alto grau de P(x) é 1 e o termo independente é igual a 221. O grau
do polinômio é
13. (FUVEST) O polinômio x4 + x2 – 2x + 6 admite 1 + i como raiz, onde
a) 4 c) 6 e) 8
i2 = –1. O número de raízes reais deste polinômio é:
b) 5 d) 7
a) 0 c) 2 e) 4
b) 1 d) 3 π
20. (UNICAMP) Seja x um número real, 0 < x < , tal que a sequência
2
14. (UNICAMP) Considere o polinômio p(x) = xn + xm + 1, em que (tan x, sec x, 2) é uma progressão aritmética (PA). Então, a razão dessa
n > m ≥ 1. Se o resto da divisão de p(x) por x + 1 é igual a 3, então PA é igual a
a) n é par e m é par. a) 1 c) 4/3
b) n é ímpar e m é ímpar. b) 5/4 d) 1/3
c) n é par e m é ímpar.
d) n é ímpar e m é par.
GABARITO
15. (FUVEST) Duas circunferências com raios 1 e 2 têm centros no EXERCÍCIOS DE TREINAMENTO
primeiro quadrante do plano cartesiano e ambas tangenciam os dois
01. a) 210 09. 35
eixos coordenados. Essas circunferências se interceptam em dois
pontos distintos de coordenadas (x1,y1) e (x2,y2). b) 329 10. Resto 0
O valor de (x1 + y1)² + (x2 + y2)² é igual a c) 301 11. C
5 9 13 d) 120 7
a) c) e) 12.
2 2 2 e) 108 16
7 11 02. 252 modos 13. A
b) d)
2 2 03. 120 14. A
04. 280 15. C
16. (FUVEST) No plano cartesiano, os pontos (0, 3) e (–1, 0) pertencem
à circunferência C. Uma outra circunferência, de centro em (–1/2,4) é 05. 125 16. D
tangente a C no ponto (0,3). Então, o raio de C vale 06. a) 188 17. D
5 5 e) 5 b) 153 18. B
a) c)
8 2 c) 612 19. A
5 3 5 (a + b + c)! 20. D
b) d) 07. Paa,b,c
+b+c
=
4 4 a!b!c!
08. 0,82
17. (UNICAMP) A área do triângulo OAB esboçado na figura abaixo é EXERCÍCIOS DE COMBATE
01. A 11. DISCURSIVA
02. DISCURSIVA 12. A
21
a) 03. B 13. A
4
04. B 14. A
23
b) 05. DISCURSIVA 15. C
4
25 06. D 16. E
c)
2 07. DISCURSIVA 17. C
27 08. C 18. E
d)
4 09. A 19. C
10. D 20. D

561

PM_BOOK16 - MAT.indb 561 25/11/2022 19:27:48


EXERCÍCIOS DE REVISÃO I

ANOTAÇÕES

562

PM_BOOK16 - MAT.indb 562 25/11/2022 19:27:48


EXERCÍCIOS DE REVISÃO II

EXERCÍCIOS DE Observação: det(x) denota o determinante da matriz X.

TREINAMENTO a) 0
b) 1
c)
d) 3
2 e) 4

01. (FUVEST) O número real x, com 0 < x < π, satisfaz a equação 06. (EN) Se =
a 3+ 2 e =
b 3 − 2 , seja k o determinante da
log3(1 – cosx) + log2(1 + cosx) = –2. Então, cos2x + senx vale 1 + a 1 1 1 
1 7 10
 1 1− a 1 1 
a) c) e) matriz  , sendo assim, é correto afirmar
3 9 9  1 1 1+ b 1 
2 8  
b) d)  1 1 1 1 − b
3 9 3 3
 1  1
que o coeficiente de x no desenvolvimento  2x + 2  ⋅  x 2 +
k–1
 é
02. (FUVEST) A figura representa um quadrado ABCD de lado 1. O  x   2x 
5 a) 21 c) 23 e) 25
ponto F está em BC, BF mede , o ponto E está em CD e AF é
4 b) 22 d) 24
bissetriz do ângulo BÂE. Nessas condições, o segmento DE mede
07. (ITA) Sobre os elementos da matriz
3 5
a)  x1 x 2 x3 x4 
40 y y y3 y 4 
7 5 =A  1 2 ∈ M4x 4 (  )
b) 0 0 0 1
40  
9 5 1 0 0 0
c)
40 sabe-se que (x1, x2, x3, x4) e (y1, y2, y3, y4) são duas progressões
11 5 geométricas de razão 3 e 4 e de soma 80 e 255, respectivamente.
d) Então, det(A–1) e o elemento (A–1)23 valem, respectivamente,
40
1 1 e) 1 1
e)
13 5 a) e 12. c) − e 12. e .
72 72 72 12
40
1 d) 1 1
b) − e -12. − e .
03. (FUVEST) Sejam α e β números reais com –π/2 < α < π/2 e 72 72 12
0 < β < π. Se o sistema de equações, dado em notação matricial,
08. (FUVEST) As propriedades aritméticas e as relativas à noção de
3 6   tgα   0 
6 8 cos β  =   , for satisfeito, então α + β é igual a ordem desempenham um importante papel no estudo dos números
    −2 3  reais. Nesse contexto, qual das afirmações abaixo é correta?
π c) 0 π a) Quaisquer que sejam os números reais positivos a e b, é verdadeiro
a) − e)
3 π 3 que a+b = a + b.
π d)
b) − 6 b) Quaisquer que sejam os números reais a e b tais que a² – b² = 0,
6
é verdadeiro que a = b.
04. (UNICAMP) Sendo a um número real, considere a matriz c) Qualquer que seja o número real a, é verdadeiro que a2 = a.
1 a  d) Quaisquer que sejam os números reais a e b não nulos tais que
 . Então, A é igual a
2017
 a < b, é verdadeiro que 1/b < 1/a.
 0 −1
e) Qualquer que seja o número real a, com 0 < a < 1, é verdadeiro
 1 0 1 a  1 1  1 a2017  que a2 < a.
a)  . b)  . c)  . d)  .
 0 1  0 −1 1 1  0 −1 
09. (ITA) Sejam r1, r2 e r3 números reais tais que r1 – r2 e r1 + r2 + r3
 1 a −2 são racionais. Das afirmações:
= a − 2 1 1  com a ∈ . Sabe-se que
05. (IME) Seja A  I. se r1 é racional ou r2 é racional, então r3 é racional;
 2 −3 1  II. se r3 é racional, então r1 + r2 é racional;
det(A² – 2A + I) = 16. A soma dos valores de a que satisfazem essa III. se r3 é racional, então r1 e r2 são racionais.
condição é:
É (são) sempre verdadeira(s).

563

PM_BOOK16 - MAT.indb 563 25/11/2022 19:27:50


EXERCÍCIOS DE REVISÃO II

a) apenas I. c) apenas III. e) I, II e III. 13. (ITA) Considere conjuntos A, B ⊂  e C ⊂ A ∪ B). Se A ∪ B,


b) apenas II. d) apenas I e II. A ∩ C e B ∩ C são os domínios das funções reais definidas por ln
x−π
10. (IME) O time de futebol “X” irá participar de um campeonato
( )
x − π , −x² + 6 − 8 e
5−x
, respectivamente, pode-se afirmar que

no qual não são permitidos empates. Em 80% dos jogos, “X” é o a) C = ]√π,5[. c) C = [2, 5[. e) C não é intervalo.
favorito. A probabilidade de “X” ser o vencedor do jogo quando ele b) C = [2, π]. d) C = [π, 4].
é o favorito é 0,9. Quando “X” não é o favorito, a probabilidade de
ele ser o vencedor é 0,02. Em um determinado jogo de “X” contra
“Y”, o time “X” foi o vencedor. Qual a probabilidade de “X” ter sido 14. (EN) Sejam A e B conjuntos de números reais tais que seus
o favorito nesse jogo? elementos constituem, respectivamente, o domínio da função f(x) = n
2(x + x − 2 )
a) 0,80 c) 180/181 e) 170/181 (2 + x + 3 |x| – |x + 1|) e a imagem da função g(x) =−2 + .
Pode-se afirmar que 2
b) 0,98 d) 179/181
a) A = B c) A⊃B e) A – B = _
11. (IME) Considere as alternativas: b) A ∩ B = ∅ d) A ∩ B = +
I. O inverso de um irracional é sempre irracional.
II. Seja a função f: A → B e X e Y dois subconjuntos quaisquer de A, 15. (UNIOESTE) Sejam f e g duas funções, ambas com domínio A
então f(X ∩ Y) = f(X) ∩ f(Y). e imagem B, subconjuntos de  e que admitem inversa. Seja f–1 a
função inversa de f e g–1 a função inversa de g. Suponha ainda que
III. Seja a função f: A → B e X e Y dois subconjuntos quaisquer de A,
f(g–1(x)) = g(f–1(x)) para todo x no domínio das inversas. É correto
então f(X ∪ Y) = f(X) ∪ f(Y).
afirmar que
IV. Dados dois conjuntos A e B não vazios, então A ∩ B = A se, e
a) (f-1 ° g)(x) = (g-1 ° f) (x) para todo x ∈ A.
somente se, B ⊂ A.
b) (f ° g)(x) = (g ° f) (x) para todo x ∈ A.
Observação: f(Z) é a imagem de f no domínio Z.
c) (f ° f)(x) = (g ° g) (x) para todo x ∈ A.
São corretas:
d) (f ° f-1)(x) = (g ° g-1) (x) para todo x ∈ A.
a) I, apenas. c) II e IV, apenas. e) II e III, apenas.
e) f-1(x) = g(x) para todo x ∈ A.
b) I e III, apenas. d) I e IV, apenas.

16. (ITA) Considere funções f, g, f + g:  → . Das afirmações:


12. (UFPR) O tanque de combustível de um posto de gasolina possui o
formato de um cilindro circular reto e está instalado de modo que as I. se f e g são injetoras, f + g é injetora;
bases estão na vertical. Para saber o volume de combustível presente II. se f e g são sobrejetoras, f + g é sobrejetora;
no tanque, o funcionário utiliza uma régua graduada e só necessita
III. se f e g não são injetoras, f + g não é injetora;
observar a altura alcançada pelo combustível dentro do tanque. Essa
régua foi confeccionada com base no estudo da função que relaciona IV. se f e g não são sobrejetoras, f + g não é sobrejetora.
o volume v com a altura h, desde zero até a altura total T. Qual dos É (são) verdadeira(s)
gráficos abaixo mais se aproxima do gráfico dessa função? a) nenhuma. d) apenas III e IV.
a) d)
b) apenas I e II. e) todas.
c) apenas I e III.

17. (EN) Sabendo que log x representa o logaritmo de x na base 10,


 x 
arccos3  log 
 10 
qual é o domínio da função real de variável real f(x) = ?
4x − x 3
a) ]0,2[ c) ]0,1] 1 
e)  , 2
1  d) [1,2[ 2 
b) e) b)  2 , 1
 

18. (FUVEST) A função f:  →  tem como gráfico uma parábola


e satisfaz f(x + 1) – f(x) = 6x – 2, para todo número real x. Então, o
menor valor de f(x) ocorre quando x é igual a
11 5 5
a) c) e) −
6 6 6
7 d) 0
b)
6
c) 19. (ITA) O conjunto de todos os valores de m para os quais a função
x² + (2m + 3)x + (m² + 3)
f(x) = está definida e é não negativa para
x² + (2m + 1)x + (m² + 2)
todo x real é:
1 7  7 1 7
a)  4 , 4   0, 4 
c) e)  4 , 4 
1   1
b)  4 , ∞  d) −∞, 
 4

564

PM_BOOK16 - MAT.indb 564 25/11/2022 19:27:51


EXERCÍCIOS DE REVISÃO II

20. (AFA) Considere a função real f definida por f(x) = ax com a ∈ ]0,1[. 07. (IME) Considere o determinante de uma matriz de ordem n,
Sobre a função real g definida por g(x) = |–b – f(x)| com b ∈ ]–∞,–1[, definido por:
é correto afirmar que 1 1 1 1 ... 1 1
a) possui raiz negativa e igual a loga(–b). Sabendo que ∆1 = 1, o valor de ∆10 é -1 3 0 0 ... 0 0
b) é crescente em todo o seu domínio. a) 59049 0 -1 3 0 ... 0 0
c) possui valor máximo. b) 48725 ∆n =0 0 -1 3 ... 0 0
d) é injetora. c) 29524 ...........................
EXERCÍCIOS DE d) 9841 0 0 0 0 ... 3 0

COMBATE e) 364 0 0 0 0 ... -1 0

08. (UDESC) Sejam A a matriz quadrada de ordem 2 definida


  π 
2s en  2x + 2  sen(x + π) 
01. (FUVEST) Uma quantidade fixa de um gás ideal é mantida a por A =     e f a função definida por
temperatura constante, e seu volume varia com o tempo de acordo  senx 1 
com a seguinte fórmula: V(t) = log2 (5 + 2 sen(πt)), 0 ≤ t ≤ 2 em que t
f(x) = |det(A + AT)|. O gráfico da função f, para x ∈ [–π,π], é:
é medido em horas e V(t) é medido em m³. A pressão máxima do gás
no intervalo de tempo [0,2] ocorre no instante a)
a) t = 0,4 c) t=1 e) t = 2
b) t = 0,5 d) t = 1,5

02. (FUVEST) Sejam x e y números reais positivos tais que x + y = π .


2
Sabendo-se que sen ( y − x ) =1 , o valor de tg²y – tg²x é igual a
3
3 1 1
a) c) e)
2 2 8
5 1 b)
b) d)
4 4

03. (FUVEST) O valor de (tg10° + cotg10°)sen20° é:


a) 1/2 c) 2 e) 4
b) 1 d) 5/2

 a 2a + 1
04. (FUVEST) Considere a matriz A =  em que a é um
 a − 1 a + 1 
número real. Sabendo que A admite inversa A cuja primeira coluna é
-1
c)
2a − 1
 −1  , a soma dos elementos da diagonal principal de A é igual a
-1

 
a) 5 d) 8
b) 6 e) 9
c) 7

a 0
05. (UNICAMP) Considere a matriz A =   , onde a e b são
b 1
números reais. Se A² = A e A é invertível, então
d)
a) a = 1 e b = 1 c) a=0eb=0
b) a = 1 e b = 0 d) a = 0 e b = 1

06. (EFOMM) Calcule o determinante da matriz A de ordem n:


1 1 1 1 1 K 1 
n −1
 
a) det(A) = ∏
n =1
2n  1 3 1
1 1 5 1 1 K
1 1 K 1 
1 
 
n A=1 1 1 7 1 K 1 
=
b) det(A)
n =1
∏ 2n − 1 1 1 1 1 9 K

1 
 e)
n −1 M M M M M O 1 
c) det(A) = ∏ 2n
n =1

 1 1 1 1 1 K 2n − 1

d) det(A) = ∏2
n =1
n −1

e) det(A) = 1

565

PM_BOOK16 - MAT.indb 565 25/11/2022 19:27:53


EXERCÍCIOS DE REVISÃO II

09. (ITA) Sejam X e Y dois conjuntos finitos com X ⊂ Y e X ≠ Y. 12. (UECE) Seja f(x) = 1/x, x ≠ 0. Se f(2 + p) – f(2) = 3/2, então
Considere as seguintes afirmações: f(1 – p) – f(1 + p) é igual a:
I. existe uma bijeção f: X → Y; a) 8/5 b) 2 c) 12/5 d) 20/3
II. existe uma função injetora g: X → Y;
III. o número de funções injetoras f: X → Y é igual ao número de 13. (UFRGS) Considere, na figura a seguir, a região sombreada
funções sobrejetoras g: X → Y. limitada por uma reta e pelo gráfico de uma função quadrática.

É (são) verdadeira(s)
a) nenhuma delas. c) apenas III. e) todas.
b) apenas I. d) apenas I e II.

10. (AFA) Um cursinho de inglês avaliou uma turma completa sendo


que parte dos alunos fez a avaliação A, cujo resultado está indicado
no gráfico abaixo.

Os demais alunos fizeram a avaliação B e todos tiveram 4 acertos.


Assim, o desvio padrão obtido a partir do gráfico acima ficou reduzido As coordenadas dos pontos (x,y) dessa região verificam as
à metade ao ser apurado o resultado da turma inteira. Essa turma do desigualdades
cursinho de inglês tem a) x² – 4x + 1 ≤ y ≤ 1 – x
a) mais de 23 alunos. c) 21 alunos. b) x² – x + 4 ≥ y ≥ 1 – x
b) menos de 20 alunos. d) 22 alunos. c) x² – 2x + 1 ≤ y ≤ 1 – x
d) x² – 4x – 1 ≥ y ≥ 1 – x
11. (FUVEST) Examine o gráfico.
e) x² – 2x + 1 ≥ y ≥ 1 – x

14. Considere a expressão: 5log5 (sen300° + tg1° ⋅ tg2° ⋅ tg3° ⋅ ... ⋅


tg88° ⋅ tg89° ⋅ cos120°)
O seu valor é
a) √3/2
b) √3/4
c) 1/4
d) 3/4

x + 1 1
15. (UESPI) Seja f:  – {–1} →  uma função satisfazendo f  = ,
 x − 1 x
para todo x real e diferente de 1 e de 0. Qual o valor de f(tg²α), para
α real e α ≠ π/2 + kπ, k inteiro?
a) cos (2α) d) –sen(2α)
b) sen (2α) e) tg α
c) –cos (2α)
Com base nos dados do gráfico, pode-se afirmar corretamente que
a idade 16. (ITA) Considere as funções f e g, da variável real x, definidas,
 ax 
a) mediana das mães das crianças nascidas em 2009 foi maior que respectivamente, por f(x) = ex² + ax + b e g ( x ) = ln   , em que a e b
 3b 
27 anos.
são números reais. Se f(–1) = 1 = f(–2), então pode-se afirmar sobre a
b) mediana das mães das crianças nascidas em 2009 foi menor que função composta g ° f que
23 anos.
a) g ° f(1) = In 3.
c) mediana das mães das crianças nascidas em 1999 foi maior que
b) ∃ g ° f(0).
25 anos.
c) g ° f nunca se anula.
d) média das mães das crianças nascidas em 2004 foi maior que 22
anos. d) g ° f está definida apenas em {x ∈ : x > 0}.
e) média das mães das crianças nascidas em 1999 foi menor que e) g ° f admite dois zeros reais distintos.
21 anos.

566

PM_BOOK16 - MAT.indb 566 25/11/2022 19:27:57


EXERCÍCIOS DE REVISÃO II

17. (EN) A concentração de um certo remédio no sangue, t horas após


10t GABARITO
sua administração, é dada pela fórmula = y(t) ,t ≥ 0. Em qual
(t + 1)² EXERCÍCIOS DE TREINAMENTO
dos intervalos abaixo a função y(t) é crescente? 01. E 06. D 11. B 16. A
a) t ≥ 0 02. D 07. C 12. A 17. D
b) t > 10 03. B 08. E 13. C 18. C
c) t>1 04. B 09. E 14. C 19. D
d) 0 ≤ t < 1 05. D 10. C 15. A 20. A
e) 1/2 < t < 10 EXERCÍCIOS DE COMBATE
01. D 06. A 11. D 16. E
100
18. (EN) Considere as funções reais f(x) = e g(x) = 2x/2, x ∈ . 02. A 07. C 12. C 17. D
1 + 2− x
03. C 08. E 13. A 18. B
Qual é o valor da função composta (g ° f-1)(90)?
04. A 09. A 14. B 19. D
a) 1 d) 1/10
05. B 10. A 15. C 20. A
b) 3 e) 1/3
c) 9 ANOTAÇÕES

19. (ITA) Dada a função quadrática f(x) = x2 ln (2/3) + x ln6 – (1/4) ln


(3/2), temos que
a) a equação f(x) = 0 não possui raízes reais.
b) a equação f(x) = 0 possui duas raízes reais distintas e o gráfico f
possui concavidade para cima.
c) a equação f(x) = 0 possui duas raízes reais iguais e o gráfico de f
possui concavidade para baixo.
d) o valor máximo de f é (ln2 ln3)/(ln3 – ln2).
e) o valor máximo de f é 2 (ln2 ln3)/(ln3 – ln2).

20. (UEL) Leia o texto a seguir.


Precisamos de um nome para o novo replicador, um substantivo
que comunique a ideia de unidade de transmissão cultural. “Mimeme”
vem do grego “aquilo que é replicado”, mas eu quero um monossílabo
que se pareça com gene. Eu espero que meus amigos clássicos me
perdoem por abreviar mimeme para meme. Se uma ideia se alastra, é
dita que se propaga sozinha.
Adaptado de: DAWKINS, R. O gene egoísta. Trad. Geraldo H. M. Florsheim.
Belo Horizonte: Itatiaia, 2001. p. 214.

Diversos segmentos têm utilizado serviços de marketing para criação e


difusão de memes de seu interesse. Um partido político com P0 = 20
filiados encomendou um anúncio que se tornou um meme em uma
rede social, sendo que 5% dos K = 2·109 usuários ativos visualizaram o
anúncio no instante t = 1. Sejam e > 1, r > 0 constantes e suponha que
K ⋅ P0 ⋅ er ⋅ t
a função P(t) dada por P(t) = representa a quantidade de
K + P0 (er ⋅ t − 1)
usuários da rede social que visualizaram o meme no instante t.
Assinale a alternativa que apresenta, corretamente, o valor da
constante r para essa rede social.
 108 − 1
a) loge  
 19 
 109 − 1
b) loge  
 19 

 109 − 1
c) loge  
 20 

d) 108 − 1
19

109 − 1
e)
20

567

PM_BOOK16 - MAT.indb 567 25/11/2022 19:27:58


EXERCÍCIOS DE REVISÃO II

ANOTAÇÕES

568

PM_BOOK16 - MAT.indb 568 25/11/2022 19:27:58


EXERCÍCIOS DE REVISÃO III

EXERCÍCIOS DE 06. (EN 2011) Sejam a, b, c as raízes da equação 12x³ – 4x² – 3x + 1 = 0.

TREINAMENTO Qual o valor de a + b + c + 1?


3 3 3

a) 2 21 d) 21
9 9
01. (ITA) Se z é uma solução da equação em , b) 2 7 e) 21
12
  2 −1 2 + 1  3 3
2
z − z + z =−  ( )
2 + i 
3
−i  ,
3   2 7
  c)
pode-se afirmar que 9

a) i(z – z) < 0. d) |z| ∈ [6, 7].


07. (IME 2010) Seja o polinômio p(x) = x³ + (ln a)x + eb, onde a e b
b) i(z – z) > 0. 1 são números reais positivos diferentes de zero. A soma dos cubos das
e) z+ > 8.
c) |z| ∈ [5, 6]. z raízes de p(x) depende
a) apenas de a e é positiva. d) apenas de b e é negativa.
 x + 1 x b) de a e b e é negativa e) de a e b e é positiva.
 1 −1 1
02. (ITA) Sejam A =   B  y − 2 y  matrizes reais tais
e= c) apenas de b e é positiva.
 y −x 1
 z − 3 z 
08. (ITA) Se as dimensões, em centímetros, de um paralelepípedo reto-
que o produto AB é uma matriz antissimétrica. Das afirmações abaixo: retangular são dadas pelas raízes da equação 24x³ – 26x² + 9x –1 = 0,
I. BA é antissimétrica; então o comprimento da sua diagonal é igual a:
II. BA não é inversível; 7 24 cm 73 cm
a) cm c) e)
III. O sistema (BA)X = 0, com Xt = [x1 x2 x3], admite infinitas soluções 12 12 12

É (são) verdadeira(s) 9 61 cm
b) cm d)
24 12
a) Apenas I e II. c) Apenas I. e) Apenas III.
b) Apenas II e III. d) Apenas II.
09. (IME 2007) Seja p(x) = x5 + bx4 + cx3 + dx3 + ex + f um polinômio
com coeficientes inteiros. Sabe-se que as cinco raízes de p(x) são
03. (AFA) O sistema linear nas incógnitas x, y e z abaixo possui uma
números inteiros positivos, sendo quatro deles pares e um ímpar. O
infinidade de soluções.
número de coeficientes pares de p(x) é:
(sen a)x + y − z =0
 a) 0 c) 2 e) 4
x − (sen a)y + z =0
x + y = b) 1 d) 3
 cos a
Sobre o parâmetro a, a ∈ , pode-se afirmar que 10. (ITA 2002) Num sistema de coordenadas cartesianas, duas retas r e
a) a = kπ, k ∈  c) a = π/2 + 2kπ, k ∈  s, com coeficientes angulares 2 e ½, respectivamente, se interceptam
na origem O. Se B ∈ r e C ∈ s são dois pontos no primeiro quadrante
b) a = 2kπ, k ∈  d) a = π/2 + kπ, k ∈  tais que o segmento BC é perpendicular a r e a área do triângulo OBC
é igual a 12 · 10-1, então a distância de B ao eixo das ordenadas vale
04. Encontre A, B e C que tornam verdadeira a identidade
8 2 e) 1
2n + 1 A B C a) c)
=+ + e utilize esse resultado para calcular o 5 5
n (n + 1)(n + 2) n n + 1 n + 2
3 5 7 9 2009 4 1
valor da soma + + + ++ b) d)
1⋅ 2 ⋅ 3 2 ⋅ 3 ⋅ 4 3 ⋅ 4 ⋅ 5 4 ⋅ 5 ⋅ 6 1004 ⋅ 1005 ⋅ 1006 5 5
O valor encontrado será aproximadamente igual a:
a) 1 c) 1,5 e) 2 11. (ITA 1998) As retas y = 0 e 4x + 3y + 7 = 0 são retas suportes das
diagonais de um paralelogramo. Sabendo que estas diagonais medem
b) 1,25 d) 1,75
4 cm e 6 cm, então a área deste paralelogramo, em cm², vale:
05. Considere a equação x3 + px2 + qx + r = 0, de coeficientes reais, 36 44 48
a) c) e)
cujas raízes estão em progressão geométrica. Qual das relações é 5 3 5
verdadeira?
b)
27 48
a) p2 = rq c) 3p2 = r2q e) q3 = rp3 d)
4 3
b) 2p + r = q d) p3 = rq3

569

PM_BOOK16 - MAT.indb 569 25/11/2022 19:28:00


EXERCÍCIOS DE REVISÃO III

 f ( x )  ⋅ g ( x ) 
3 10
12. (ITA 94) Um triângulo equilátero ABC é tal que A: (0, 3), B: (3√3, 0) Assim sendo, o conjunto solução da inequação  ≥0
h ( x ) 
15
e a abcissa do ponto C é maior que 2. A circunferência circunscrita a contém o conjunto
este triângulo tem raio r e centro O: (a, b). Então a2 + b2 + r2 é igual a:
a) [0,8] c) [2,6]
a) 31 c) 33 e) 35
b) [1,7] d) [3,5]
b) 32 d) 34
18. (EFOMM 2002) A soma e o produto das raízes da equação
13. (ITA 96) Sabendo que o ponto (2, 1) é ponto médio de uma corda x x−2
AB da circunferência (x – 1)2 + y2 = 4, então a equação da reta que + −1=0 são iguais a:
1− x x
contém A e B é dada por:
a) –2 c) 0 e) 3
a) y = 2x –3 c) y=–x+3 e) y = –(1/2) · x + 2
b) –1 d) 2
b) y = x – 1 d) y = (3/2) · x – 2
19. (EFOMM 2004) Que valores deve apresentar o coeficiente “a” da
14. (IME 89) Dada a equação: função f(x) = ax² – 2x + 1 para que ela tenha concavidade voltada para
x2 + y2 – 2mx – 4 · (m + 1) · y + 3m + 14 = 0 cima e vértice no primeiro quadrante?
a) a > 0 c) 0<a<1 e) a ≥ ½
a) Determine os valores de m, para que esta equação corresponda
a um círculo. b) 0 < a ≤ 1 d) a > 1
b) Determine o lugar geométrico dos centros destes círculos.
20. (EFOMM 2006) Se M e N são as raízes de x2 – 6x + 10 = 0, o valor
15. (EN 2011) A curva de equação x² – 14 = y² + 2x intercepta a reta 1 1
de + é:
4y + 1 = x nos pontos A e B. Seja C a circunferência com centro no M N
ponto médio do segmento AB e cujo raio é a medida do maior eixo a) 6 d) 3/5
da curva de equação x² + 2y² = 2√3x – 8y – 2. A circunferência C tem b) 2 e) 1/6
por equação
c) 1
35 − x 2 − y 2 x 2 + y 2 − 35
a) x= d) x=
2 2
EXERCÍCIOS DE

b) x=
20 − x 2 − y 2
2
e) x=
25 − x 2 − y 2
2 COMBATE
x + y − 25
2 2
c) x=
2
01. (ITA) Se x é um número natural com 2015 dígitos, então o número
16. (AFA 2010) Considere o esboço dos gráficos das funções reais f, g, de dígitos da parte inteira de 7 x é igual a
h, tais que f é do segundo grau e g e h são do primeiro grau. Sabe-se
a) 285 c) 287 e) 289
que V é o vértice da parábola.
b) 286 d) 288

02. (IFAL) O potencial de hidrogênio (pH) das soluções é dado pela


função: pH = –log[H+], onde [H+] é a concentração do cátion H+ ou
H3O+ na solução. Se, em uma solução, a concentração de H+ é 2 · 10-8,
qual o pH dessa solução? Adote: log2 = 0,3.
a) 2,4 c) 6,7 e) 11
b) 3,8 d) 7,7

03. (ITA) Seja (a1, a2, a3,...) a sequência definida da seguinte forma:
a1 = 1000 e an = log10(1 + an–1) para n ≥ 2. Considere as afirmações
a seguir:
I. A sequência (an) é decrescente.
II. an > 0 para todo n ≥ 1.
III. an < 1 para todo n ≥ 3.
É (são) verdadeira(s)
O conjunto de todos os valores de x para os quais h(x) > g(x) > f(x) é
a) apenas I. d) I, II e III.
a)  – ]1,5[ c)  – [1,3]
b) apenas I e II. e) apenas III.
b)  – [1,5] d)  – ]1,3[
c) apenas II e III.

17. (AFA 2014) Seja f uma função quadrática tal que:


04. (UEM) Sobre a equação |x – 5| + |x + 1| = C, em que C é uma
I. f(x) > 0 para todo x real constante real e x ∈ , assinale o que for correto.
II. tem gráfico interceptando o gráfico da função g, dada por 01) Se C = 0, a equação possui solução.
g(x) = 2, num único ponto cuja abscissa é 2.
02) Se C = 6, a equação possui infinitas soluções.
III. seu gráfico possui o ponto Q, simétrico do ponto R(0, –3) em
04) Se C < 0, a equação possui apenas uma solução.
relação à origem do sistema cartesiano. Seja h uma função afim
cujo gráfico intercepta o gráfico de f no eixo y e no ponto de 08) Se C = 4, a solução será x = 4.
menor ordenada de f. 16) Se C = 10, a equação possui duas soluções.

570

PM_BOOK16 - MAT.indb 570 25/11/2022 19:28:00


EXERCÍCIOS DE REVISÃO III

64 12. (ITA) Considere o sólido de revolução obtido pela rotação de um


05. (IFSUL) A equação 2x +1 − 24 =
− x possui como solução
2 triângulo isósceles ABC em torno de uma reta paralela à base BC que
a) x = 2 e x = 3 c) x=3ex=6 dista 0, 25 cm do vértice A e 0,75 cm da base BC. Se o lado AB mede
b) x = 2 e x = 6 d) x = 4 e x = 8 π² + 1
cm o volume desse sólido, em cm3, é igual a

06. (IME) Seja a equação y log
=3 3y
ylog3 3y − 6, y = 0. O produto das 9 9
a) . d) .
raízes reais desta equação é igual a: 16 24
1 3 e) 3 13 11
a) c) b) . e) .
3 4 96 96
1 d) 2 7
b) c) .
2 24

07. (FUVEST) Um reservatório de água tem o formato de um cone 13. (ITA) Das afirmações:
circular reto. O diâmetro de sua base (que está apoiada sobre o chão I. Duas retas coplanares são concorrentes;
horizontal) é igual a 8m. Sua altura é igual a 12m. A partir de um
II. Duas retas que não têm ponto em comum são reversas;
instante em que o reservatório está completamente vazio, inicia-se
seu enchimento com água a uma vazão constante de 500 litros por III. Dadas duas retas reversas, existem dois, e apenas dois, planos
minuto. O tempo gasto para que o nível de água atinja metade da paralelos, cada um contendo uma das retas;
altura do reservatório é de, aproximadamente, IV. Os pontos médios dos lados de um quadrilátero reverso definem
Dados: um paralelogramo,
- π é aproximadamente 3,14. É (são) verdadeira(s) apenas
- O volume V do cone circular reto de altura h e raio da base r é a) III. d) III e IV.
1 b) I e III. e) I e II e IV.
V= πr²h.
3
c) II e III.
a) 4 horas e 50 minutos. d) 6 horas e 20 minutos.
b) 5 horas e 20 minutos. e) 6 horas e 50 minutos. 14. (IME) Sejam ABC um triângulo equilátero de lado 2 cm e r uma
c) 5 horas e 50 minutos. reta situada no seu plano, distante 3 cm do seu baricentro. Calcule a
área da superfície gerada pela rotação deste triângulo em torno da
08. (FUVEST) Cada aresta do tetraedro regular ABCD mede 10. Por reta r.
um ponto P na aresta AC, passa o plano α paralelo às arestas AB e CD. a) 8π cm²
Dado que AP = 3, o quadrilátero determinado pelas interseções de α b) 9π cm²
com as arestas do tetraedro tem área igual a
c) 12π cm²
a) 21 c) 30 30 3
e) d) 16π cm²
21 2 30 2
b) d) e) 36π cm²
2 2
09. (FUVEST) Os vértices de um tetraedro regular são também vértices 15. Devido aos fortes ventos, uma empresa exploradora de petróleo
de um cubo de aresta 2. A área de uma face desse tetraedro é resolveu reforçar a segurança de suas plataformas marítimas,
a) 2√3 c) 3√2 e) 6 colocando cabos de aço para melhor afixar a torre central. Considere
que os cabos ficarão perfeitamente esticados e terão uma extremidade
b) 4 d) 3√3 no ponto médio das arestas laterais da torre central (pirâmide
quadrangular regular) e a outra no vértice da base da plataforma (que
10. (FUVEST) Uma pirâmide tem como base um quadrado de lado 1, é um quadrado de lados paralelos aos lados da base da torre central
e cada uma de suas faces laterais é um triângulo equilátero. Então, a e centro coincidente com o centro da base da pirâmide), como sugere
área do quadrado, que tem como vértices os baricentros de cada uma a ilustração.
das faces laterais, é igual a
5 1 1
a) c) e)
9 3 9
4 2
b) d)
9 9

11. (UNICAMP) A embalagem de certo produto alimentício, em


formato de cilindro circular, será alterada para acomodar um novo
rótulo com informações nutricionais mais completas. Mantendo
o mesmo volume da embalagem, a sua área lateral precisa ser
aumentada. Porém, por restrições de custo do material utilizado, este
aumento da área lateral não deve ultrapassar 25%. Sejam r e h o
raio e a altura da embalagem original, e R e H o raio e a altura da
embalagem alterada. Nessas condições podemos afirmar que: Se a altura e a aresta da base da torre central medem, respectivamente,
24 m e 6√2m e o lado da base da plataforma mede 19√2m, então a
R 3 H 16 R 4 H 25
a) ≥ e ≤ . c) ≥ e ≤ . medida, em metros, de cada cabo será igual a
r 4 h 9 r 5 h 16
a) √288 d) √400
R 9 H 4 R 16 H 5 b) √313 e) √505
b) ≥ e ≤ . d) ≥ e ≤ .
r 16 h 3 r 25 h 4 c) √328

571

PM_BOOK16 - MAT.indb 571 25/11/2022 19:28:02


EXERCÍCIOS DE REVISÃO III

16. (IME) A base de uma pirâmide é um retângulo de área S. Sabe-se


que duas de suas faces laterais são perpendiculares ao plano da base. GABARITO
As outras duas faces formam ângulos de 30° e 60° com a base. O EXERCÍCIOS DE TREINAMENTO
volume da pirâmide é:
01. E 05. E 09. E 13. C
S S d)
2S S 02. B 06. A 10. B
a)
3 5 03. B 07. D 11. E
S S e)
2S2 04. B 08. D 12. C
b)
6 3 14. a) m < −2 ou m > 1.

c)
2S S b) A reta y0 = 2(x0 + 1), sem o segmento que une os pontos (−2,−2) e (1, 4).
3 15. D 17. D 19. D
16. B 18. A 20. D
17. (IFAL) A equação x³ – 3x² + 7x – 5 = 0, no universo dos números
EXERCÍCIOS DE COMBATE
complexos, tem como solução
01. D 06. A 11. C 16. A
a) {1,1 + 2i, 1 – 2i}
02. D 07. C 12. C 17. A
b) {1,1 + i, 1– i}
03. D 08. A 13. D 18. D
c) {1,2 + 2i, 2 – 2i}
04. DISCURSIVA 09. A 14. E 19. A
d) {–1,1 + 2i, 1 – 2i}
05. A 10. D 15. D 20. C
e) {–1,1 + i, 1– i}
ANOTAÇÕES
2(a + bi)
18. (ITA) Considere a equação (a − bi)501 = .
(a2 + b2 )250 + 1
O número de pares ordenados (a,b) ∈ ² que satisfazem a equação é
a) 500
b) 501
c) 502
d) 503
e) 504

19. (IFSUL) De uma forma criativa, após um exame, o professor


entregou as notas expressas por números complexos aos seus
alunos. Para cada aluno descobrir sua nota, era necessário calcular
o módulo (observe que o módulo de um número complexo
z = a + bi é calculado por = | z | a2 + b2 ) do número complexo descrito
no seu exame. Dessa forma, as notas representadas pelos números
 2π 2π   5π 5π 
complexos N1 = 4 ⋅  cos + i ⋅ sen  , N2 = 3 ⋅  cos + i ⋅ sen 
 3 3   6 6 
e N3 =  + i  ⋅  − i  − i aproximados são, respectivamente,
5 1 3
2  2  4
a) 4,3 e 3,5
b) 3,4 e 3,5
c) 3,4 e 5
d) 4,3 e 5

20. (EFOMM) Analise as afirmações que se seguem.


x+y+z 3
I. Se x, y, z são números reais positivos, então ≥ x⋅y⋅z .
3
II. Se z é um número complexo de módulo unitário que satisfaz a
condição z2n ≠ –1, sendo n um número inteiro positivo, então
zn
é um número real.
1 + z2n
III. Se A4,3 representa a matriz dos coeficientes de um sistema linear
com quatro equações e três incógnitas, esse sistema será possível
e determinado sempre que o posto desta matriz A for menor ou
igual a 3.
Então, pode-se dizer que
a) todas as afirmativas são verdadeiras.
b) todas as afirmativas são falsas.
c) somente as afirmativas I e II são verdadeiras.
d) somente as afirmativas I e III são verdadeiras.
e) somente as afirmativas II e III são verdadeiras.

572

PM_BOOK16 - MAT.indb 572 25/11/2022 19:28:03

Você também pode gostar